Sunteți pe pagina 1din 748

BOOK

JURISDICTION

PEOPLE vs LACSON, 400 SCRA


267.
FACTS:

Respondent and his coaccused were charged with


multiple
murder
for
the
shooting and killing of eleven
male
persons
bandied
as
members
of
the Kuratong
Baleleng Gang.
The Court ruled in the
Resolution
sought
to
be
reconsidered
that
the
provisional
dismissal
of
Criminal Cases Nos. Q-9981679 to Q-99-81689 were with
the express consent of the
respondent
as
he
himself
moved for provisional dismissal
when he filed his motion for
judicial
determination
of
probable
cause
and
for
examination of witnesses.
According to the Court, if
the cases were revived only
after the two-year bar, the State
must be given the opportunity
to justify its failure to comply
with
the
said
time-bar. It
emphasized that the new rule

fixes a time-bar to penalize the


State for its inexcusable delay
in prosecuting cases already
filed in court. However, the
State is not precluded from
presenting compelling reasons
to justify the revival of cases
beyond the two-year bar.
In support of their Motion
for
Reconsideration,
the
petitioners contend that (the
time-bar in said rule should not
be applied retroactively.
The Court agrees with the
respondent
that
procedural
laws
may
be
applied
retroactively. As applied to
criminal law, procedural law
provides or regulates the steps
by
which
one
who
has
committed a crime is to be
punished.

ISSUE:
Being favorable to the
accused , can the timebar
rule
be
applied
retroactively

HELD:
The time-bar under the
new rule was fixed by the Court

to excise the malaise that


plagued the administration of
the criminal justice system for
the benefit of the State and the
accused; not for the accused
only.

QUETO vs CATOLICO, 31 SCRA


52.

The
criminal
regulates the steps
one who committed
to
be

Each one of those affected was


served with a uniformly worded
mimeographed notice from the
Clerk of the Court of First
Instance
of
Misamis
Occidental, presided by herein
respondent Judge Catolico that
they are one among those who
have taken oath of allegiance to
the Republic of the Philippines
where
the
proceedings
in
connection therewith were null
and void, ab initio, in the light
of the recent decisions of the
Supreme
Court
on
naturalization.

procedure
by which
a crime is
punished.

Thus, Judge Agnir, Jr. dismissed


Criminal Cases Nos. Q-9981679 to Q-99-81689 on March
29, 1999. The new rule took
effect on December 1, 2000. If
the Court applied the new timebar retroactively, the State
would have only one year and
three months or until March
31, 2001 within which to revive
these
criminal
cases.
The
period is short of the two-year
period fixed under the new rule.
On the other hand, if the time
limit is applied prospectively,
the State would have two years
from December 1, 2000 or until
December 1, 2002 within which
to revive the cases. This is in
consonance
with
the
intendment of the new rule in
fixing the time-bar and thus
prevent injustice to the State
and
avoid
absurd,
unreasonable,
oppressive,
injurious, and wrongful results
in the administration of justice.

FACTS:

It
effect
nullified
all
the
previous
proceedings

petition,
publication,
trial,
judgment, oath taking and
issuance of the certificate of
naturalization.
To the credit of the lawyers who
appeared
before
him,
in
respectful
language
they
pointed out, that the proper
procedure that should have
been pursued against each of
their
clients
was
through
cancellation
of
the
naturalization
certificate
by
court and therefore respondent
Judge had no jurisdiction to act
motu proprio and declare that
petitioners had not acquired

Filipino citizenship by virtue of


their
naturalization.
ISSUE:
Whether or not respondent
Judge,
motu
proprio,
had
jurisdiction to reopen and
review, to declare null and void
the grant of citizenship to the
petitioners pursuant to final
judgments of competent courts
and
after
the
oaths
of allegiance had been taken
and
the
corresponding
certificates of naturalization
issued.
HELD:
It may be true, as alleged by
respondent
that
the
proceedings for naturalization
were
tainted
with
certain
infirmities, fatal or otherwise,
but that is beside the point in
this case. The jurisdiction of
the court to inquire into and
rule upon such infirmities must
be
properly
invoked
in
accordance with the procedure
laid down by law which is by
cancellation
of
the
naturalization
certificate,
"upon motion made in the
proper proceedings by the
Solicitor
General
or
his
representatives,
or
by
the
proper
provincial
fiscal."
Initiative must come from
these officers, presumably after
previous investigation in each
particular case.
As a general rule a court
proceeding in our judicial set-

up is accusatorial or adversary
and not inquisitorial in nature.
It contemplates two contending
parties before the court, which
hears them impartially and
renders judgment only after
trial. This basic philosophy
would be violated if a judge
were permitted to act as
inquisitor, pursue his own
independent
investigation,
arrive at a conclusion ex-parte,
and then summon the party
affected so as to enable him, if
that were still possible, to show
that
the
conclusion
thus
arrived
at
is
without
justification.

CARIAGA vs PEOPLE, G.R. No.


180010, July 30, 2010.
FACTS:
Municipal
treasurer
of
Cabatuan, Isabela with a Salary
Grade of 24, was charged
before the RTC of Cauayan City
in Isabela with three counts of
malversation of public funds.
The two other Informations in
the second and third criminal
cases,
contain the same
allegations
except
the
malversed amounts.
The RTC found the accused
CENITA M. CARIAGA, GUILTY
beyond reasonable doubt of the
crime of MALVERSATION for

which she is charged in the


three
(3)
separate
informations.
Cariaga appealed from such
ruling to the Court of Appeals
which dismissed petitioner's
appeal for lack of jurisdiction
since the jurisdiction over the
offense is vested with the
Regional
Trial
Court
considering that the position of
Municipal
Treasurer
corresponds to a salary grade
below 27.
Pursuant to Section 4 of
[Presidential Decree No. 1606,
as amended by Republic Act
No.
8249], it
is
the
Sandiganbayan,
to
the
exclusion of all others, which
enjoys appellate jurisdiction
over the offense.
Petitioner, now admitting the
procedural error committed by
her former counsel, implores
the Court to relax the Rules to
afford her an opportunity to
fully ventilate her appeal on the
merits and requests the Court
to endorse and transmit the
records of the cases to the
Sandiganbayan in the interest
of substantial justice

In cases where none of the


accused
are
occupying
positions
corresponding
to
Salary Grade `27' or higher,
Sandiganbayan
shall
exercise exclusive
appellate
jurisdiction over
final
judgments,
resolutions
or
orders of regional trial courts
whether in the exercise of their
own original jurisdiction or of
their appellate jurisdiction.
That appellate jurisdiction in
this case pertains to the
Sandiganbayan is clear. Section
4 of Presidential Decree No.
1606 but since the appeal
involves criminal cases, and the
possibility of a person being
deprived of liberty due to a
procedural
lapse
militates
against
the
Court's
dispensation of justice, the
Court grants petitioner's plea
for a relaxation of the Rules.
For rules of procedure must be
viewed as tools to facilitate the
attainment of justice, such that
any rigid and strict application
thereof
which
results
in
technicalities
tending
to
frustrate
substantial
justice
must always be avoided

ISSUE:
Does the Sandiganbayan has
jurisdiction over the alleged
crime
HELD:

Quotation
from
Albert
vs.
University Publishing House as
per Due Process, G.R. No. L19118, January 30, 1965.

FACTS:
Mariano Albert entered into a
contract
with
University
Publishing Co, Inc. through
Jose Aruego, its president,
whereby University would
p a y p l a i n t i ff f o r t h e e x c l u
s i v e right to publish his
revised
Commentaries
on
the Revised Penal Code. The
contract stipulated that failure
to pay
one
installment
would render the rest of
t h e p a y m e n t s d u e . When
University failed to pay the
second
installment,
Albert
sued
for
collection
and
won. However,
upon
execution, it was found that
University
was
not
registered with the SEC.
Albert
petitioned for
a writ of execution agains
t J o s e M . Aruego as the real
defendant.
University
opposed, on the ground that
Aruego was not a party to the
case
ISSUE:
Whether or not Aruego
can
be
held
personally
l i a b l e t o t h e plaintiff.

assumed such privileges and


obligations
and
became
personally liable
for the
contract entered into or for
other
acts performed as such
agent.
One who has induced
another to act upon his
willful misrepresentation
t h a t a corporation was duly
organized and existing under
the law, cannot thereafter set
up against his victim the
principle
of corporation by
estoppel.
By "due process of law" we
mean "a law which hears before
it condemns; which proceeds
upon
inquiry,
and
renders
judgment only after trial. "Due
process of law" contemplates
notice and opportunity to be
heard
before
judgment
is
rendered,
affecting
one's
person or property."
"Due
process"
clause
of
the
Constitution is designed to
secure justice as a living
reality; not to sacrifice it by
paying
undue
homage
to
formality.

HELD:
Aruego represented a nonexistent entity and induced not
only Albert but the court to
believe in such representation,
he was the real party to the
contract sued upon, and thus

Alonte vs
SCRA 245

Savellano,

Jr.

287

FACTS:
Alonte was accused of raping
JuvieLyn
Punongbayan
with
accomplice
Buenaventura

Concepcion.
The
case
was
brought before RTC Bian. The
counsel and the
prosecutor
moved for a change of venue
due to alleged intimidation.
While the change of venue was
pending, Juvie executed an
affidavit of desistance. The
prosecutor continued on with
the case and the change of
venue
was
done
notwithstanding
opposition
from Alonte. The case was
raffled to the Manila RTC under
Judge
Savellano.
Savellano
found probable cause and had
ordered the arrest of Alonte
and Concepcion. Prosecution
presented
Juvie
and
had
attested the voluntariness of
her desistance the same being
due to media pressure and that
they would rather establish new
life elsewhere. Case was then
submitted for decision and
Savellano
sentenced
both
accused to reclusion perpetua.
Savellano
commented
that
Alonte waived his right to due
process when he did not cross
examine
Juvie
when
clarificatory
questions
were
raised about the details of the
rape and on the voluntariness
of her desistance.
ISSUE:
Whether or not there was
a denial of due process right of
Alonte

HELD:

Silence of the counsel of the


accused during hearing as to
the cross-examination to the
victim
Juvie
does
not
automatically mean that there
was a valid waiver. There was
no clear evidence that Alonte
waived his right to crossexamine Juvie. Existence of the
waiver
must
be
positively
demonstrated. The standard of
waiver requires that it "not only
must be voluntary, but must be
knowing, intelligent, and done
with sufficient awareness of the
relevant
circumstances
and
likely
consequences. Mere
silence of the holder of the
right
should
not
be
as
construed as a waiver of right,
and the courts must indulge
every reasonable presumption
against waiver.
The Court stated that this
case must be remanded for
further proceedings.
Jurisprudence acknowledges
that due process in criminal
proceedings,
in
particular,
require (a) that the court or
tribunal trying the case is
properly clothed with judicial
power to hear and determine
the matter before it; (b) that
jurisdiction is lawfully acquired
by it over the person of the
accused; (c) that the accused is
given an opportunity to be
heard; and (d) that judgment is
rendered only upon lawful
hearing.[12]

Does the RTC Manila


jurisdiction over the case

has

HELD:

LUTGARDA CRUZ vs COURT OF


APPEALS, 388 SCRA 72.
FACTS:
The
City
Prosecutor
of
Manila charged Cruz with
the crime of estafa though
falsifi cation
of
public
documents before the RTC
of Manila. Allegedly, Cruz
executed an Aff idavit of Self
adjudication of a parcel of
land when she knew that
there were other surviving
heirs. The off ended party
did not reserve the right to
fi le a separate civil action.
Hence, it was tried together
with the criminal case. The
RTC acquitted Cruz. On the
civil
aspect,
the
court
ordered the return of the
parcel
of
land
to
the
surviving heirs. Cruz fi led by
registered mail a motion for
reconsideration.
This
was
denied by the trial court.
Petition for certiorari and
mandamus was fi led with the
CA. This was also dismissed
by
the
appellate
court.
Hence,
this
petition
for
review on certiorari.

ISSUE:

There are three important


requisites
which
must
be
present before a court can
acquire criminal jurisdiction.
First, the court must have
jurisdiction over the subject
matter. Second, the court must
have
jurisdiction
over
the
territory where the offense was
committed. Third, the court
must have jurisdiction over the
person of the accused.
In the instant case, the trial
court had jurisdiction over the
subject matter as the law has
conferred on the court the
power to hear and decide cases
involving
estafa
through
falsification
of
a
public
document. The trial court also
had
jurisdiction
over
the
offense charged since the crime
was
committed
within
its
territorial jurisdiction. The trial
court also acquired jurisdiction
over the person of accusedpetitioner
because
she
voluntarily submitted to the
courts
authority.
Where
the
court
has
jurisdiction over the subject
matter and over the person of
the accused, and the crime was
committed within its territorial
jurisdiction,
the
court
necessarily
exercises
jurisdiction over all issues that
the law requires the court to
resolve.

ANTIPORDA
GARCHITORENA,
551.

Jr
321

vs
SCRA

FACTS:

Mayor Licerio Antiporda and


others
were
charged
for
kidnapping and the case was
filed in Sandiganbayan. The
Court ordered the prosecution
to submit amended information
and
the
new
information
contained the place where the
victim was brought.

Accused
then
filed
an
Urgent
Omnibus
Motion
praying that a reinvestigation
of the case be conducted and
the issuance of warrants of
arrest be deferred but it was
denied.
The accused filed a Motion
for
New
Preliminary
Investigation and to Hold in
Abeyance
and/or
Recall
Warrant of Arrest Issued but it
was denied.
Court ruled that 'since none
of the accused have submitted

they to the jurisdiction of the


Court, the accused are not in a
position to be heard on this
matter at this time.
Accused filed a Motion to
Quash
the
Amended
Information
for
lack
of
jurisdiction over the offense
charged but it was denied
saying that at all events there
is an Amended Information
here which makes an adequate
description of the position of
the accused vesting the Court
with
the
office
related
character of the offense of the
accused.
A
motion
for
reconsideration was filed by the
accused wherein it was alleged
that the filing of the Motion to
Quash and the appearance of
their
counsel
during
the
hearing amounted to their
voluntary
appearance
and
invested
the
court
with
jurisdiction over their persons
but it was denied.

ISSUE:
Does the Sandiganbayan
has jurisdiction over the case
HELD:
The Supreme Court do not
have
jurisdiction
over
the
case. The original Information
filed with the Sandiganbayan

did not mention that the


offense
committed
by
the
accused is office-related. It was
only after the same was filed
that the prosecution belatedly
remembered
that
a
jurisdictional fact was omitted.
However, we hold that the
petitioners are estopped from
assailing the jurisdiction of the
Sandiganbayan
for
in
the
supplemental
arguments
to
motion
for
reconsideration
and/or
reinvestigation
filed
with the same court, it was they
who challenged the jurisdiction
of the Regional Trial Court over
the case and clearly stated in
their
Motion
for
Reconsideration that the said
crime is work connected.
It is a well-settled rule that
a party cannot invoke the
jurisdiction of a court to secure
affirmative relief against his
opponent, and after obtaining
or failing to obtain such relief,
repudiate or question that
same jurisdiction.
Sandiganbayan
has
jurisdiction
over
the
case
because of estoppel and it was
thus vested with the authority
to order the amendment of the
Information.

Petitioner Fullero was then


employed as telegraph operator
at the Iriga Citys Bureau of
Telecommunications
Office
until he became as the Acting
Chief Operator. Upon inquiry
made
by
Florenda
B.
Magistrate, a subordinate of
petitioner in the BTO, Iriga
City, with the PRC, it was
verified that petitioner never
passed the board examination
for civil engineering.
Petitioner denied executing and
submitting the subject PDS and
disowned the signature and
thumb mark.
In an Amended Information,
petitioner was charged with
falsification
of
public
document.
He further argued that the RTC
had no jurisdiction to try him
there being no evidence that
the alleged falsification took
place in Legazpi City. After
trial, the Legazpi RTC rendered
a Decision finding petitioner
guilty
of
the
crime
of
falsification.
Petitioner
appealed to the Court of
Appeals. The appellate court
promulgated
its
Decision
affirming in toto the assailed
Legaspi City RTC Decision
ISSUE:

FULLERO vs PEOPLE 533 SCRA


97

Does CA errered in ruling that


RTC Legazpi has jurisdiction

despite the fact that MTC had


no jurisdiction because venue
should have been in TRC Iriga
City where the alleged PDS was
accomplished.

the case
others.

will

exclude

the

HELD:
There are three important
requisites
which
must
be
present before a court can
acquire
jurisdiction
over
criminal cases. First, the court
must have jurisdiction over the
offense or the subject matter.
Second, the court must have
jurisdiction over the territory
where
the
offense
was
committed. And third, the court
must have jurisdiction over the
person of the accused. There is
no dispute that the Legazpi City
RTC has jurisdiction over the
offense and over the person of
petitioner. It is the territorial
jurisdiction of the Legazpi City
RTC
which
the
petitioner
impugns.
The territorial jurisdiction of a
court is determined by the facts
alleged in the complaint or
information as regards the
place
where
the
offense
charged
was
committed. It
should also be emphasized that
where some acts material and
essential to the crime and
requisite to its consummation
occur in one province or city
and some in another, the court
of either province or city has
jurisdiction to try the case, it
being understood that the
court first taking cognizance of

WINONA
M.
BONIFACIO,
JOCELYN
UPANO,
VICENTE
ORTUOSTE and JOVENCIO
PERECHE, JR. v. REGIONAL
TRIAL COURT OF MAKATI and
JESSIE JOHN P. GIMENEZ
G.R. No. 184800, May 5, 2010
FACTS:
13 Informations for Libel were
then filed before the Makati
Regional Trial Court to seek
redress for being unable to
collect under their pre-need
educational plans under PPI,
due to liquidity concerns. The
Informations alleged that the
accused, holding legal title to
the
website,
maliciously
published
the
following
defamatory
article
against
Yuchengco Family and Malayan.
The
Secretary
of
Justice
directed the withdrawal of the
Informations
for
lack
of
probable cause, opining that

the crime of Internet Libel


was non-existent. RTC quashed
the Informations for failure to
allege that the offended parties
were
actually
residing
in
Makati at the time the offense
was committed as in fact they
listed
their
addresses
in
Manila, or to allege that the
article was printed and first
published in Makati.
The prosecution moved for
reconsideration,
contending
that
even
assuming
the
Information was deficient, it
merely
needed
a
formal
amendment.
The RTC-Makati granted the
MR and order the amendment
of the Informations to show
that
the
website
was
accessible in Makati City and
the defamatory article was
first published and accessed by
the private complainant in
Makati City.
Upon
acceptance
of
the
amended
Informations,
the
accused filed a petition for
certiorari and prohibition with
the Supreme Court faulting the
RTC
for
grave
abuse
of
discretion amounting lack or
excess of jurisdiction.
ISSUE:
Whether or not the amended
Informations
for
Internet
Libel were brought before the
proper Court of Jurisdiction,

which in this case is the RTCMakati; Makati being the place


where the article was first
accessed by the complainant.
RULING:
The
amended
Informations were insufficient
to vest jurisdiction in Makati.
Venue
is
jurisdictional
in
criminal actions such that the
place where the crime was
committed determines not only
the venue of the action but
constitutes
an
essential
element of jurisdiction. Venue
of
libel
cases
where
the
complainant
is
a
private
individual is limited only in
either of these two places: (1)
Where the complainant actually
resides at the time of the
commission of the offense; and
(2)
where
the
alleged
defamatory article was printed
and first published. In the
hereby case, the prosecution
chose the second.
Article 360 was amended by
Republic Act No. 4363 which
provides
that
such
action
should be brought where the
article was printed and first
published. The evil sought to be
prevented by the amendment
was
the
indiscriminate
or
arbitrary laying of the venue in
libel cases in distant, isolated,
or far-flung areas, meant to
accomplish nothing more than
to harass or intimidate an
accused, especially when the
offended party is a person of
sufficient means or possesses

influence, and is motivated by


the spite or the need for
revenge.

A Motion for Reconsideration


was denied in a Resolution.
ISSUE:

VICENTE FOZ, JR. and DANNY


G. FAJARDO v. PEOPLE OF THE
PHILIPPINES
G.R. No. 167764
October 9, 2009
FACTS:
Vicente Foz, Jr. and Danny G.
Fajardo, columnists and EditorPublisher,
respectively,
of
Panay
News,
wrote
and
published an article wherein
said Dr. Edgar Portigo, a
physician
and
medical
practitioner
in
Iloilo,
was
portrayed as an incompetent
doctor and opportunist, who
enriched himself at the expense
of the poor, wanting in high
sense of professional integrity,
trust,
and
responsibility
expected of him as a physician,
which
imputation
and
insinuation as both accused
knew were entirely false and
malicious
and
without
foundation
in
fact,
and,
therefore, highly libellous.
An information was filed before
the Regional Trial Court-Iloilo
against Foz and Fajardo for the
crime of libel.
RTC-Iloilo found the petitioners
guilty as charged. The Motion
for Reconsideration later filed
was denied. The case was
brought
to
the
Court
of
Appeals, which affirmed in toto
the decision of the trial court.

Whether
or
nor
the
Information
for
Libel
was
brought before the proper
Court of Jurisdiction, which in
this case is the RTCIloilo; Iloilo
being the place where the
publication has a considerable
circulation.
RULING:
Article 360 of the Revised Penal
Code, as amended by Republic
Act No. 4363, provides the
specific rules as the venue in
cases of written defamation, to
wit:
The
allegation
in
the
Information that Panay News, a
daily
publication
with
a
considerable circulation in the
City of Iloilo and throughout
the region only showed that
Iloilo was the place where
Panay
News
was
in
considerable circulation but did
not establish that the said
publication was printed and
first published in Iloilo City.
Information failed to allege the
venue requirements for a libel
case under Article 360 of the
Revised Penal Code, the Court
finds that the RTC-Iloilo had no
jurisdiction to hear the case.
The
trial
courts
decision
convicting Vicente Foz, Jr. and
Danny G. Fajardo of libel should

be set aside for want of


jurisdiction without prejudice
to its filing with the court of
competent jurisdiction.

WILSON AGBAYANI, CARMELO


BAUTISTA, PABLO PASCUAL,
and RENATO ROMEO DUGAY v.
HONORABLE
SOFRONIO
G.
SAYO,
PEOPLE
OF
THE
PHILIPPINES, and
CONRADO B. MAHINAN
G.R. No. L-47880 April 30, 1979

FACTS:
Conrado B. Mahinan, a lawyer,
was the manager of the GSIS
Cagayan
Valley
Branch
stationed at Cauayan, Isabela,
with Wilson Agbayani, chief of
the investment unit, Carmelo
N.
Bautista,
chief
of
the
production and premium unit,
Pablo R. Pascual, officer-incharge of the legal and claims
unit, and Renato Romeo P.
Dugay, an employee of the
claims unit, were all charged
with libel on an Information
filed by the provincial fiscal of
Nueva Vizcaya, on behalf of
Conrado B. Mahinan, before
the CFI Nueva Vizcaya.

According to the information,


all those documents allegedly
depicated Mahinan "as an
incorrigible managerial misfit,
despoiler
of
public
office,
spendthrift of GSIS funds,
inveterate
gambler,
chronic
falsifier', and an unreformed
ex-convict".
Accused filed a motion to quash
contending
that
CFI-Nueva
Vizcaya has no jurisdiction over
the offense charged because
Mahinan was a public officer
holding office at Cauayan,
Isabela when the alleged libel
was committed and, under
Article 360 of the Revised Penal
Code, the offense charged
comes within the jurisdiction of
the CFI-Isabela.
The motion was opposed by the
fiscal but was denied by the
trial court on the ground that
Mahinan was not a public
officer since the insurance
business of the GSIS is not an
inherently
governmental
function.
ISSUE:
Whether or not the Information
for Libel was brought before
the
proper
Court
of
Jurisdiction, which in this case
is CFI-Nueva Vizcaya.
RULING:
There is no issue as to whether
Mahinan is a public officer. As
GSIS Branch Manager, he is
unquestionably a public officer.

Proper venue of Mahinan's


criminal action for written
defamation
against
the
petitioners is the Court of First
Instance of Isabela, since as a
GSIS branch manager, he was a
public official stationed at
Cauayan,
Isabela
and
the
alleged libel was committed
when he was (as he still) in the
public service. The preliminary
investigation of the complaint
should have been conducted by
the provincial fiscal of Isabela,
or by the municipal judge of
ILAGAN, the provincial capital,
or by the Court of First
Instance of the same province.
The venue was laid in Nueva
Vizcaya. It was alleged in the
information that the libel was
committed in Bambang, a town
located in that province. It was
not alleged that at the time the
libel was committed Bambang
was the actual residence of
complainant Mahinan or that it
was the place where the libel
was printed and first published
or where Mahinan held his
office.
Information in this case is
defective or deficient because it
does not show that the Court of
First
Instance
of
Nueva
Vizcaya, where it was filed, has
jurisdiction to entertain the
criminal action for written
defamation
initiated
by
Mahinan
against
the
petitioners
and
that
the
provincial
fiscal
of
that
province had the authority to

conduct
the
investigation.

preliminary

WILMA T. BARRAMEDA v. THE


COURT
OF
APPEALS
and
LOLITA WATANABE
G.R. No. 96428. September 2,
1999
FACTS:
Lolita Watanabe, a Filipino
working in Japan, sent money
in dollars and yen equivalent to
P50,000.00,
to
Papiniana
Paguinto
(mother)
in
the
Philippines
through
Wilma
Barrrameda, the accused.
Upon notice by her daughter,
Paguinto met Barrameda in
Manila International Airport
Pasay and demanded the money
but the latter said it was
misplaced and will still have to
look for it in her baggage.
Appellant promised to deliver
the
money
the
following
morning but she never did.
Despite
repeated
demands,
appellant never gave the money
to Paguinto.
A charge for Estafa was filed
before the Regional Trial Court
of Pasay City. The Court of
Appeals
affirmed
the
trial
courts decision in toto and
denied the subsequent Motion
for Reconsideration filed by the
accused.
Barrameda
denied

having received the money and


alleged that not a single
element of the crime occurred
within
the
territorial
jurisdiction of the RTC-Pasay.
ISSUE:

A
demand
was
made
by
Papiniana Paguinto at NAIA,
which is within the territorial
jurisdiction of the RTC- Pasay.
The presence of Paguinto at the
airport was
for no other
purpose but to demand the
money entrusted to Barrameda.

Whether or not the Information


for Estafa was brought before
the
proper
Court
of
Jurisdiction, which in this case
is RTC-Pasay; Pasay being the
place where the demand for
remittance was made.
RULING:
The elements of Estafa are as
follows: (1) Money, goods, or
other personal property is
received by the offender in
trust, or on commission, or for
administration, or under any
other obligation involvingthe
duty to make delivery of, or to
return, the same; (2) that there
be
misappropriation
or
conversion of such money or
property by the offender or
denial on his part of receipt;
(3) that such misappropriation
or conversion or denial is to the
prejudice of another; and (4)
that there is a demand made by
the offended party on the
offender.
In all criminal prosecutions,
the action shall be instituted
and tried in the court of the
municipality
or
territory
wherein
the
offense
was
committed or where any of the
essential ingredients thereof
took place.

DAVID B. CAMPANANO, JR. v.


JOSE ANTONIO A. DATUIN
G.R. No. 172142
October 17, 2007
FACTS:
An Information was filed by
Seishin
International
Corporation represented by its
President,
David
B.
Campanano,
against
Jose
Antonio A. Datuin before the
Regional Trial Court of Pasig
City, charging him for violation
of Batas Pambansa Blg. 22 for
allegedly misappropriating the
agency funds allocated for the
purchase of roadrollers.
The
RTC-Pasig
Datuin for Estafa.

convicted

Later, Datuin filed a complaint


for
Incriminating
Against
Innocent Persons, punishable
under Article 363 of the
Revised Penal Code, before the
Office of the City Prosecutor of
Quezon City against petitioner
and a certain Yasunobu Hirota,
claiming that the complain of
Campanano
was
false,
unfounded, and malicious in
light of the cash voucher that
proves the payment in cash for
the two roadrollers.
The
Office
of
the
City
Prosecutor of Quezon City
dismissed the complaint for
lack of jurisdiction.

committed in Quezon City. The


only reference to Quezon City
in the complaint-affidavit is
that it is where the respondent
(or complainant) resides. Thus,
it was properly dismissed by
the City Prosecutor of Quezon
City for lack of jurisdiction.

ISSUE:
Whether or not the complaint
for violation of Article 363 of
the Revised Penal Code was
brought before the proper
Office of Jurisdiction
RULING:
Section 5, Rule 110 of the
Revised Rules on Criminal
Procedure
provides
that
subject to existing laws, the
criminal
action
shall
be
instituted and tried in the
court of the municipality or
territory where the offense was
committed or where any of its
essential
ingredients
occurred.
The
complain-affidavit
for
incriminating innocent person
filed by respondent with the
Office of the City Prosecutor of
Quezon City does not allege
that the crime charged or any
of its essential ingredients was

BAYANI SUBIDO, JR. and RENE


PARINA v. THE HONORABLE
SANDIGANBAYAN
and
THE
PEOPLE OF THE PHILIPPINES
G.R. No. 122641. January 20,
1997
FACTS:
An Information was filed before
the Sandiganbayan charging
Subido
and
Parina
with
Arbitrary Detention defined and
punished by Article 124 of the
Revised Penal Code.

Petitioners filed a Motion to


Quash. They argued that:
(1) Arbitrary Detention
did
not
fall
within
Chapter II, 2, Title VII of
the RPC, but within 1,
Chapter
1,
Title
II
(Crimes
Against
the
Fundamental Laws of the
State),
hence,
not
covered by R.A. No. 7975
and, therefore, the case
should have been filed
with the Regional Trial
Court of Manila.
ISSUE:
Whether
or
nor
the
Information against the public
officers was brought before the
proper Court of Jurisdiction
RULING:
The
Sandiganbayan
jurisdiction over the case.

had

In cases where none of the


principal
accused
are
occupying
positions
corresponding to salary grade
27 or higher, as prescribed in
said R.A. 6758, or PNP officers
occupying
the
rank
of
superintendent or higher, or
their
equivalent,
exclusive
jurisdiction thereof shall be
vested in the proper Regional
Trial Court, Metropolitan Trial
Court, Municipal Trial Court,
and Municipal Circuit Trial
Court, as the case maybe,
pursuant to their respective
jurisdiction as provided in
Batas Blg. 129.

Contrary to the claims of the


petitioners, Republic Act No.
7975 applies since what is
considered is the time of the
commission
of
the
crime,
during which Subido was still
Commissioner
of
BID.
Similarly, although Parina was
holding a position with a
classification lower than Salary
Grady 27, the provisions of the
law still applies to him since he
is
prosecuted
as
a
coconspirator of Subido, the
principal accused. Jurisdiction
is only vested to the other
courts if none of the principal
accused
where
occupying
positions
corresponding
to
Salary Grade 27.

LICERIO A. ANTIPORDA, JR.


ELITERIO RUBIACO, VICTOR
GASCON, and CAESAR TALIA v.
HON.
FRANCIS
E.
GARCHITORENA,
HON.
EDILBERTO SANDOVAL, HON.
CATALINO CASTANEDA, JR., in
their capacity as Presiding
Justice and Associate Justices
of the Sandiganbayan G.R. No.
133289, 23 December 1999
Buena, J.
FACTS:
An Information for the
crime of kidnapping against
Mayor Licerio Antiporda, et al.,
was
filed
before
the
Sandiganbayan. The Court later
ordered the prosecution to
submit
an
amended

information as there it was not


clear whether or not the
subject
matter
of
the
accusation was office-related.
Subsequently, they complied
evenly and the new information
contained the place where the
victim was brought.
The
accused
filed
an
Urgent
Omnibus
Motion
praying that a reinvestigation
be conducted and the issuance
of
warrants
of
arrest
be
deferred but it was denied by
the Ombudsman.
Thereafter, the accused
filed
a
Motion
for
New
Preliminary Investigation and
to hold in abeyance and/or
recall warrant of arrest issued
but the same was also denied.
The accused then filed a
Motion to Quash Amended
Information
for
lack
of
jurisdiction over the offense
charged, which was ignored for
their continuous refusal to
submit themselves to the Court
and
after
their
voluntary
appearance which vested the
Sandiganbayan
jurisdiction
over their persons.
ISSUE:
Whether
or
not
the
Sandiganbayan, which has no
jurisdiction over the offense
charged
in
the
original
Information,
subsequently
acquire jurisdiction by the
simple expedient of amending
the Information to supply
jurisdictional
facts
not

previously
averred
original Information.

in

the

RULING: It is undisputed that


the
Sandiganbayan
had
territorial jurisdiction over the
case, however, the petitioners
were estopped from assailing
the same.
The original Information filed
with the Sandiganbayan did not
mention
that
the
offense
committed by the accused is
office-related. It was only after
the same was filed that the
prosecution
belatedly
remembered
that
a
jurisdictional fact was omitted
therein.
However,
petitioners
are
estopped from assailing the
jurisdiction
of
the
Sandiganbayan
for
in
the
supplemental
arguments
to
motion
for
reconsideration
and/or
reinvestigation
filed
with the same court, it was they
who challenged the jurisdiction
of the Regional Trial Court over
the case and clearly stated in
their
Motion
for
Reconsideration that the said
crime is work connected
It is a well-settled rule that a
party
cannot
invoke
the
jurisdiction of a court to secure
affirmative relief against his
opponent, and after obtaining
or failing to obtain such relief,
repudiate or question that
same jurisdiction.

GOMEZ vs MONTALBAN, G.R.


No. 174414, March 14, 2008
FACTS:
Petitioner
filed
a
Complaint with the RTC for a
sum of money, damages and
payment of attorney's fees
against respondent. Petitioner
filed the Complaint praying for
the payment of P238, 000.00,
representing the principal loan
and interest charges, plus 25%
of the amount to be awarded as
attorney's fees, as well as the
cost of suit.

petitioner and that the RTC had


no jurisdiction as the principal
amount
being
claimed
by
petitioner was only 40,000.00,
an amount falling within the
jurisdiction of the Municipal
Trial Court (MTC).
Respondent filed a Motion for
Reconsideration of
the
dismissal of her Petition for
Relief,
stating
that
her
counsel's failure to appear was
unintentional,
but
due
to
human
shortcomings
or
frailties, constituting honest
mistake
or
excusable
negligence.

Summons
was
served,
but
despite her receipt thereof,
respondent failed to file her
Answer. Consequently, she was
declared in default and upon
motion, petitioner was allowed
to present evidence ex parte.

RTC
granted
respondent's
Petition
for
Relief
from
Judgment and set aside its
previous
Decision
on
the
ground of lack of jurisdiction.

RTC rendered a Decision in


favor of petitioner.

ISSUE:

Respondent filed a Petition for


Relief from Judgment alleging
no proper service of summons
was made since because no
personal service made and that
Mrs. Alicia De la Torre was not
authorized to receive summons
and that the reason of her
failure to file answer was due to
fraud, accident, mistake and
excusable
negligence.
She
claimed that she had good and
valid
defenses
against

Whether or not the Regional


Trial Court has jurisdiction
over the case
HELD:
The Court deemed it irrelevant
that during the course of the
trial,
it
was
proven
that
respondent is only liable to
petitioner
for
the
amount
of P40, 000.00 representing the
principal amount of the loan;
P57, 000.00 as interest thereon

at the rate of 24% per annum


reckoned from 26 August 1998
until the present; and P15,
000.00
as
attorney's
fees.
Contrary
to
respondent's
contention,
jurisdiction
can
neither be made to depend on
the
amount
ultimately
substantiated in the course of
the trial or proceedings nor be
affected by proof showing that
the claimant is entitled to
recover a sum in excess of the
jurisdictional amount fixed by
law. Jurisdiction is determined
by the cause of action as
alleged in the complaint and
not by the amount ultimately
substantiated and awarded.
Basic as a hornbook principle is
that
jurisdiction
over
the
subject matter of a case is
conferred
by
law
and
determined by the allegations
in
the
complaint
which
comprise a concise statement
of
the
ultimate
facts
constituting
the
plaintiff's
cause of action.

DURISOL PHILIPPINES INC vs


COURT OF APPEALS,

G.R. No. 121106. February 20,


2002

FACTS:
Petitioner
Durisol
obtained
loans from DBP (respondent)
which it secured by mortgage
over 2 parcels of land, whose
titles where surrendered to
DBP. Upon default of petitioner,
DBP filed a petition for extrajudicial
foreclosure
of
the
mortgage.
Petitioner,
through
its
president, borrowed the TCTs
in order to obtain new titles in
accordance
with
the
new
subdivision
plan
of
the
properties and DBP gave the
TCTs to them. Subsequently the
properties were foreclosed with
DBP as the highest bidder.
Petitioner filed a complaint for
annulment of the extrajudicial
foreclosure with the trial court
but such was dismissed by the
trial court. The titles were not
given back by petitioner, thus
respondents filed a motion for
writ of execution as to the TCTs
covering
subject
properties
against Durisol but such writ
was not served since petitioner
cannot be found on the address
stated. Alias writ of execution
was issued against Durisols
President Rene Knecht but he
refused to follow the surrender
order as to the titles. Upon
DBPs Motion to the Court,
order was issued directing the
Bulacan Register of Deeds to
cancel seven titles and to issue

new ones covering the same


and it was granted and DBP
had its new certificates and
subsequently sold it.

After more the four years,


Petitioner Durisol filed with CA
a petition to annul the trial
courts ruling and resolution
saying that the trial court had
no jurisdiction over the case
for first time on appeal but it
was denied.

ISSUE:

Even assuming arguendo that


the regional trial court had no
jurisdiction over the surrender
of duplicate title, petitioner can
no longer raise this ground
after
having
actively
participated in the prosecution
of the case. A courts lack of
jurisdiction over the subject
matter and the failure of the
complaint to state a cause of
action are distinct and separate
grounds for dismissal of a case.
Petitioner actively participated
in
the
course
of
the
proceedings both in the trial
court and in the appellate
court.

Does the Trial Court correctly


exercised its jurisdiction

HELD:
At the outset, it should be
stressed that in a petition for
annulment of judgment based
on
lack
of
jurisdiction,
petitioner
must
show
not
merely
an
abuse
of
jurisdictional discretion but an
absolute lack of jurisdiction.
Lack of jurisdiction means
absence of or no jurisdiction,
that is, the court should not
have taken cognizance of the
petition because the law does
not vest it with jurisdiction
over
the
subject
matter.
Jurisdiction over the nature of
the action or subject matter is
conferred
by
law.

MAGNO vs PEOPLE, 647 SCRA


362 (2011)
FACTS:
The Office of the Ombudsman
filed
an
information
for
multiple frustrated murder and
double
attempted
murder
against
several
accused,
including MAGNO, who were
public officers working under
the NBI
Magno, in open court, objected
to the formal appearance and
authority of Atty. Sitoy, who was
there as private prosecutor to
prosecute the case for and on
behalf of the Office of the
Ombudsman

The RTC issued an Order,


ruling that the Ombudsman is
proper, legal and authorized
entity to prosecute this case to
the exclusion of any other.This
prompted the respondents to
file a petition for certiorari
before the CA.
CA original decision: Declared
that the private prosecutor may
appear for the petitioner in the
case, but only insofar as the
prosecution of the civil aspect
of the case is concerned.
CA AMENDED decision: Ruling
that the private prosecutor may
appear for the petitioner in
Criminal Case to intervene in
the prosecution of the offense
charged in collaboration with
any lawyer deputized by the
Ombudsman to prosecute the
case. This amended CA decision
in turn made Magno file for a
review on certiorari under Rule
45 of the Rules of Procedure
before the SC.
PETITIONER allege that CA
did not
have jurisdiction
to entertain the petition for
certiorari the power to hear
and decide that question is
with the Sandiganbayan
Respondents allege that the
Ombudsman did not address
the
contention
that
the
Sandiganbayan, not the CA, has
appellate jurisdiction over the
RTC in this case.

ISSUE:
Does CA has jurisdiction over
the RTCs decision in not allowing
Atty. Sitoy to prosecute the case on
behalf of Ombudman
HELD:
In the present case, the
CA
erred
when
it
took
cognizance of the petition
for certiorari filed by Magno.
While it is true that the
interlocutory order issued by
the
RTC
is
reviewable
by certiorari, the same was
incorrectly
filed
with
the
CA. Magno should have filed
the petition for certiorari with
the Sandiganbayan, which has
exclusive appellate jurisdiction
over the RTC since the accused
are public officials charged of
committing crimes in their
capacity as Investigators of the
National
Bureau
of
Investigation.
The CA should have dismissed
the petition outright. Since it
acted without authority.
The Ombudsman should have
filed the petition for certiorari
with the Sandiganbayan, which
has the exclusive appellate
jurisdiction over the RTC since
the accused are public officials
charged of committing crimes
as NBI Investigators

Jurisdiction is conferred by
law, and the CAs judgment,
issued
without
jurisdiction,
is VOID.

executed, for the second time, a


waiver of rights by way of sale
in favor of Padunan. Claiming
ownership
over
the
land,
Padunan started constructing
thereon
a
house
and
a
warehouse. Objecting to the
construction made by Padunan,
Marcos Rodriguez filed a case
for
injunction
before
the
PARAD of Nueva Ecija. The
PARAD decided in favor of
Marcos Rodriguez, declaring
him
the
lawful
tenantbeneficiary of the subject land,
directing the issuance of the
corresponding EPs in his name
and ordering Padunan to vacate
the premises upon payment of
the mortgage debt.

PADUNAN vs DARAB, G.R. No.


132163. January 28, 2003

Padunan
appealed
to
the
DARAB but the latter affirmed
in toto the decision of PARAD.
Padunan appealed to the Court
of Appeals but it was dismissed.
Padunan now filed a petition
for certiorari under rule 45
contesting the CAs Ruling that
DARAB have jurisdiction to rule
on the validity of emancipation
patents is unlawful.

FACTS:
This case is about the
DARABs
jurisdiction
over
emancipation patents.
Angelina R. Rodriguez was the
original beneficiary under PD
27 of three parcels of land in
Nueva Ecija. Angelina waived
her
rights
over
the
said
landholdings in favor of Marcos
Rodriguez by virtue of the
Sinumpaang
Salaysay
duly
executed and thumb marked by
her. Marcos Rodriguez obtained
a loan from Padunan with the
subject
landholdings
as
collateral.
Emancipation
Patents covering the subject
three parcels of land were
issued to Angelina. Angelina

ISSUE:
Whether or not the Court of
Appeals errered in ruling that
DARAB has jurisdiction to rule
on the emancipation patents
validity.
HELD:

It must be stated at the


outset that it is the law that
confers jurisdiction and not the
rules.
Jurisdiction
over
a
subject matter is conferred by
the Constitution or the law and
rules of procedure yield to
substantive
law.
Otherwise
stated, jurisdiction must exist
as a matter of law.

Secretary of Agrarian Reform


who has the authority to cancel
the same.

With this well-established


principle on jurisdiction, it is
therefore incorrect for the
private
respondent
Marcos
Rodriguez to argue that the
DARAB derives its jurisdiction
from the DARAB Rules of
Procedure. The DARAB derives
its jurisdiction from RA 6657 or
popularly
known
as
the
Comprehensive
Agrarian
Reform Law (CARL) of 1988.

Complaint
was
later
filed
against the petitioners by the
widow of Jerry Macabael with
the Office of the Ombudsman
charging them with conspiracy
to mislead the investigation of
the fatal shootout of Jerry
Macabael by (a) altering his
wound (b) concealing his brain;
(c) falsely stating in police
report that he had several
gunshot wounds when in truth
he had only one; and d) falsely
stating in an autopsy report
that there was no blackening
around his wound when in
truth there was.

DARAB
New
Rules
of
Procedure provide that DARAB
has exclusive jurisdiction over
cases involving the cancellation
of registered EPs.
But, the cancellation of EPs
that are not yet registered with
the Register of Deeds falls
within the authority of the
Agrarian Reform Secretary or
DAR officials duly designated
by him, in the exercise of
his/their
administrative
functions. And since, in the
case at bar, the erroneously
issued EPs in the name of
Angelina
Rodriguez
were unregistered, it is the

SOLLER vs SANDIGANBAYAN,
G.R. No. 144261-62. May 9,
2001.
FACTS:

Petitioners spouses Soller


denied tampering with the
cadaver of Jerry Macabael and
the two 2 police officers who
investigated the crime denied
having planted three 3 shells at
the place where the shooting
took place.
The
Office
of
Ombudsman recommended
filing of an Information
Obstruction of Justice, and

the
the
for
two

(2) Informations were filed with


the Sandiganbayan.

Petitioners filed a Motion to


Quash
saying
that
the
Sandiganbayan
had
no
jurisdiction over the offenses
charged but it was denied
saying that accused Mayor
Prudente D. Soller, Sr. who
occupies a position at Grade
Level 27, is co-accused with his
wife, the Municipal Health
Officer who occupies a position
at Grade Level 24, so that,
necessarily,
the
offense
attributed to the lower ranking
officer elevates the entire case
to this Court primarily because
somebody over whom this Court
has jurisdiction, the Mayor, is
accused together with the
lower ranking officer.

ISSUE:
Whether
or
nor
the
Sandiganbayan has jurisdiction
over the case

HELD:
What
determines
the
jurisdiction
of
the
Sandiganbayan is the specific
factual
allegation
in
the
information that would indicate

close intimacy between the


discharge of the accuseds
official
duties
and
the
commission of the offense
charged in order to qualify the
crime
as
having
been
committed in relation to public
office.
In
this
case,
the
Informations fail to allege that
petitioners had committed the
offenses charged in relation to
their offices. Neither are there
specific allegations of facts to
show
the
intimate
relation/connection
between
the commission of the offense
charged and the discharge of
official
functions
of
the
offenders,
i.e.
that
the
obstruction
of
and
apprehension and prosecution
of
criminal
offenders
was
committed in relation to the
office of petitioner Prudente
Soller, whose office as Mayor is
included in the enumeration in
Section 4 (a) of P.D. 1606 as
amended.
Indeed the offenses defined
in P.D. 1829 may be committed
by any person whether a public
officer or a private citizen, and
accordingly public office is not
an element of the offense.

PABLO DE JESUS vs DE
GARCIA, G.R. No. L-26816,
February 28, 1967.

FACTS:
Ten persons, among whom
are petitioners and respondent,
Maxima de Jesus, are co-owners
of six (6) parcels of land in
Sampaloc,
Manila.
Administratrix thereof and coowners
attorney-in-fact
is
Maxima
de
Jesus.
Her
stipulated compensation is 10%
of the rentals.
Lessee of the property is Shell
Company of the Philippines,
Ltd. This lease was renewed by
instrument executed,
where
under, through the efforts of
Maxima de Jesus, the monthly
rentals were increased from
P850.00 to P3, 500.00 during
the first ten (10) years and to
P4, 000.00 for the subsequent
five (5) years. Shell pays the
rentals by issuing a check in
the name of Maxima de Jesus
who, in turn, distributes the
shares of her co-owners.

Petitioners sought to unjustly


deprive Maxima de Jesus of her
10%
compensation
by
surreptitiously
instructing
Shell not to pay their share in
the
rentals
through
said
Maxima de Jesus but directly to
them.

A complaint was filed by


Maxima on 1966, and the
respondent judge of Manila, on
a
P500.00-bond,
issued exparte a writ of preliminary
injunction. In obedience to the
writ of preliminary injunction,
Shell delivered to Maxima de
Jesus the sum of P3, 500.00,
covering the October, 1966
rental.

ISSUE:
Whether
or
not
the
respondent Judge of Manila has
the jurisdiction over the subject
matter

HELD:
Nothing in the statute
books would confer jurisdiction
on city courts over actions
where specific performance of
contract is primarily sought.
Result: The city court has no

jurisdiction
matter

over

the

subject

The breach of contract


charged against defendants is
total and indivisible. Monthly
rentals will have to run through
a number of years. There is an
unqualified refusal to perform
the contract. Such refusal goes
to the entire contract. It is
treated as a complete breach.
Therefore, but one action
specific performance may be
presented
Jurisdiction
over
the
subject matter is conferred only
by the Constitution or law. It
cannot be fixed by the will of
the parties; it cannot be
acquired through, or waived,
enlarged or diminished by, any
act or omission of the parties.
Neither is it conferred by
acquiescence of the court.
Jurisprudence
teaches
that the averments of the
complaint, taken as a whole,
are what determine the nature
of the action, and therefore, the
court's jurisdiction.

AMADO TOLENTINO vs SOCIAL


SECURITY COMMISSION, GR
L-28870, September 6, 1985.
FACTS:

It was the position of Technical


Assistant (Executive Assistant)
that Tolentino was holding
when
the
SSC
passed
Resolution
affirming
the
decision
of
Administrator
Gilberto
Teodoro
finding
Tolentino guilty of dishonesty,
as charged, and imposing upon
him the penalty of dismissal
from the service, effective on
the first day of his preventive
suspension with prejudice to
reinstatement.
Administrator
filed charges against Tolentino
for
dishonesty
and
electioneering.
Tolentino
answered
in
two
separate
letters. The administrator was
unsatisfied with his denial, and
an investigation ensued, with
Tolentino
in
preventive
suspension. Tolentino received
a letter from the Administrator
informing him of his dismissal
from the service by virtue of
Resolution 1003 of the SSC.
Tolentino filed with the CFI
Rizal a petition for mandamus
with
preliminary
mandatory
injunction
questioning
the
validity of Resolution 1003, the
lower court rendered an order
dismissing Tolentinos petition
for lack of jurisdiction over the
SSC because the latter ranks
with the CFI in the exercise of
the
quasi-judicial
powers
granted to it by the Social
Security Act of 1954. Hence,
the present petitions for review
by
certiorari
involve
two
different
decisions
of
two
different tribunals

ISSUE:
Whether or nor the CSC
COMMISSIONER or the CIR
has the sole jurisdiction to
impose disciplinary actions on
civil service employees

Dela Cruz vs. MOYA, G.R. No. L65192 April 27, 1988

HELD:
Jurisdiction of a court or
tribunal is determined by the
statute in force at the time of
the commencement of the
action
and
once
acquired,
jurisdiction
continues,
regardless
of
subsequent
happenings, until the case is
finally terminated.

Dela Cruz is a Member of the


Armed Forces Intelligence and
Operations
Sectionarmed
with a MISSION ORDER, Dela
Cruz proceeds to Maco, Davao
del Norte to investigate reports
of illegal cockfighting being
conducted.
Delacruz
and
company catches operators of
cockfighting,
including
deceased Eusebio Cabito in
flagrante.
Delacruz
and
company
fails
to
arrest
operators
but
confiscates
evidence
of
the
crime
(eg.Gaffs, fighting cocks, etc).
Delacruz and company were
followed
by
the
cockfight
operators on their way back
to the
PC
headquarters,
fighting
ensued
wherein
Delacruz shot Cabito. Delacruz
is charged with homicide in the
CFI of Davao.
While the case was pending
trial, Presidential Decree Nos.
1822
and
1822-A
were
promulgated by the President
of the Philippines on 1981,
vesting
in
courts-martial
jurisdiction
over
crimes
committed by members of the
Armed
Forces
or
of
the
Philippine
Constabulary
in
performance of their duties.

At the time Resolution


1003 was promulgated and
implemented dismissing Amado
Tolentino,
the
respondentsappellees
Social
Security
Commission, Gilberto Teodoro
and Angel Penano did not have
the power to hear and decide
administrative and disciplinary
charges filed against erring
employees of the Commission.
Still, Resolution 1003 shall not
be dismissed as inutile. The
Social Security Commission, as
an agency of the government,
may
be
considered
a
department
and
Gilberto
Teodoro, its department head.
Resolution 1003 may be treated
as the recommendation of the
department head which may be
submitted to the Civil Service
Commission for decision and or
appropriate action.

FACTS:

Claiming that the crime for


which he was charged was
committed in relation to the
performance of his duties, Dela
Cruz filed with the Court of
First Instance of Davao a
motion to transfer the case to
the military authorities so he
could be tried by court martial.
The motion was denied. Hence,
the present petition.
ISSUE:
Whether the civil courts
have
jurisdiction
over
the
subject matter of Criminal
Case
HELD:
One
of
the
essential
requisites of a valid court
proceeding is that the court
hearing the case must have
jurisdiction over the subject
matter of the case. If the court
is acting without jurisdiction,
then the entire proceedings are
null and void.
Jurisdiction over the subject
matter is determined by the
statute in force at the time of
the commencement of the
action. And once jurisdiction is
vested in the court, it is
retained up to the end of the
litigation.
In
the
instant
case,
the
information was filed on August
2, 1979. On such date, by virtue
of General Order No. 59

military
tribunals
created
under General Order No. 8
exercised exclusive jurisdiction
over "all offenses committed by
military personnel of the Armed
Forces of the Philippines while
in the performance of their
official duty or which arose out
of any act or omission done in
the
performance
of
their
official duty; Provided, that for
the purpose of determining
whether
an
offense
was
committed
while
in
the
performance of official duty or
whether it arose out of an act
or
omission
done
in
the
performance of official duty, a
certificate
issued
by
the
Secretary of National Defense
to
that
effect
shall
be
conclusive unless modified or
revoked by the President.
The proviso merely states that
the certificate issued by the
Secretary of National Defense
is conclusive for the purpose of
determining whether an offense
was committed while in the
performance of official duty, or
arose out of an act or omission
done in the performance of
official duty. It does not in any
way preclude the courts from
making any finding as to
whether an offense is dutyconnected.

FUKUZUME vs PEOPLE, G.R.


No. 143647, November 11,
2005
FACTS:
Yu went to the house of
Fukuzume in Paraaque, that
with the intention of selling the
subject
aluminum
scrap
wires, the latter pretended
that he is a representative
of
Furukawa who is authorized
to
sell
the
said
scrap
wires; that based on the
false pretense of Fukuzume,
Yu agreed to buy the subject
aluminum scrap wires; that Yu
paid
Fukuzume
the
initial
amount
of P50,000.00
of
the total agreed price of
P290,000 that as a result , Yu
suffered damage. Subsequently,
Yu filed a complaint
with the National Bureau of
Investigation (NBI).
Information is filed with the
RTC Makati charged Fukuzume
with Estafa.
The trial court found Fukuzume
guilty as charged which was
also affirmed by the CA.
Hence, this petition.
Fukuzume contended that the
CA erred in ruling that the RTC
of Makati has jurisdiction
over the offense charged since
contract
of
sale
of
the
aluminum scrap wires took
place

at appellants residence in
Paraaque. Contrary to the
allegation in the information
that
Yu went to the house of
Fukuzume in Paraaque; that
with the intention of selling the
subject
aluminum
scrap
wires, the latter pretended
that he is a representative
of
Furukawa who is authorized
to
sell
the
said
scrap
wires; that based on the
false pretense of Fukuzume,
Yu agreed to buy the subject
aluminum scrap wires; that Yu
paid
Fukuzume
the
initial
amount
of P50,000.00
of
the total agreed price of
P290,000.00. that as a result ,
Yu
suffered
damage.
Subsequently,
Yu
filed
a
complaint
with the National Bureau of
Investigation (NBI).
In 1994, an information is filed
with the RTC Makati, charged
Fukuzume with Estafa.
The trial court found Fukuzume
guilty as charged which was
also affirmed by the CA.
Hence, this petition.
ISSUE:
Whether or not Makati
Trial Court has jurisdiction
over the case
HELD:
Jurisdiction
over
the
subject matter, a criminal cases
cannot be conferred upon the

court by the accused, express


waiver or otherwise, since such
jurisdiction is conferred by the
sovereign
authority
which
organized the court, and is
given only by law in the manner
and form prescribed by law.
Settled is the rule that
whenever there is inconsistency
between the affidavit and the
testimony of a witness in court,
the testimony commands greater
weight considering that affidavits
taken ex parte are inferior to
testimony given in court, the
former being almost invariably
incomplete
and
oftentimes
inaccurate.

Nothing in the direct or


cross-examination of Yu to
establish that he gave any
money to Fukuzume or
transacted business with
him with respect to the
subject aluminum scrap
wires inside or within the
premises
of
the
Intercontinental Hotel in
Makati, or anywhere in
Makati for that matter.
Venue in criminal cases is
an
essential
element
of
jurisdiction. However, if the
evidence adduced during the
trial show that the offense was
committed somewhere else, the
court should dismiss the action
for want of jurisdiction.

CUDIA vs COURT OF APPEALS,


G.R. No. 110315. January 16,
1998.
FACTS:
Petitioner Renato Cudia
was arrested in Mabalacat,
Pampanga for the crime of
Illegal Possession of Firearms
and
Ammunition. He
was
brought Angeles City which a
preliminary investigation was
conducted and as a result the
City
Prosecutor
filed
an
information against him and it
was raffled to RTC Angeles City.
Upon arraignment, the court
called the attention of the
parties and contrary to the
information, Renato Cudia had
committed
the
offense
in
Mabalacat and not in Angeles
City. As such the judge ordered
that
the
case
should
be
assigned to a court involving
crimes
committed
outside
Angeles City consequently it
was assigned to Angeles City
RTC. Pampangas Provincial
Prosecutor
also
filed
an
information charging Renato
Cudia with the same crime and
it was likewise assigned to the
Angeles
City
RTC
which
resulted into two Information
filed with the same crime.
It caused the City Prosecutor
to file a Motion to Dismiss/
Withdraw
the
Information
which the trial court granted.
Renato filed a Motion to Quash
the criminal case filed by the
Provincial Prosecutor on the
ground that his continued

prosecution for the offense of


illegal possession of firearms
and ammunition for which he
had been arraigned in the first
criminal case, and which had
been dismissed despite his
opposition would violate his
right not to be put twice in
jeopardy of punishment for the
same offense. The trial court
denied the motion to quash;
hence, petitioner raised the
issue to the Court of Appeals.
The appellate court, stating
that there was no double
jeopardy, dismissed the same
on the ground that the petition
could not have been convicted
under the first information as
the
same
was
defective.
Petitioner's
motion
for
reconsideration was denied,
hence, this appeal

ISSUE:
Whether
or
not
the
Angeles City Prosecutor has the
authority to file the case
HELD:
It is plainly apparent that
the City Prosecutor of Angeles
City had no authority to file the
first information, the offense
having been committed in the
Municipality
of
Mabalacat,
which
is
beyond
his
jurisdiction.
It
is
thus
the
Provincial
Prosecutor of Pampanga, not

the City Prosecutor, who should


prepare
informations
for offenses committed within
Pampanga
but
outside
of
Angeles City. An information,
when required to be filed by a
public
prosecuting
officer,
cannot be filed by another. It
must be exhibited or presented
by the prosecuting attorney or
someone authorized by law. If
not, the court does not acquire
jurisdiction. In fine, there must
have been a valid and sufficient
complaint or information in the
former prosecution. As the
fiscal had no authority to file
the information, the dismissal
of the first information would
not be a bar to petitioners
subsequent prosecution. As the
first information was fatally
defective for lack of authority
of the office.

MOBILIA PRODUCTS INC vs


UMEZAWA, G.R. No. 149357,
March 4, 2005
FACTS:
Umezawa, then the President
and General Manager of MPI,
organized
another
company
with his wife Kimiko, and his
sister, Mitsuyo Yaguchi, to
be known as Astem Philippines
Corporation, without knowledge
of the Board of Directors of MPI.
The said company would be
engaged in the same businesses

They also recruited the former


Production Manager of Mobilia
Products to work in their
company. Umezawa and others
wanted
to
accelerate
the
market potentials of Astem by
participating
in
a
Furniture Fair in Singapore.
The fair required that the
furniture for the exhibits arrive
at a certain date before the
exhibit. Pressed with time,
as Astem had yet no equipment
and machinery, no staff and no
ready
personnel,
Umezawa,
with
grave
abuse
of
the confidence reposed in him
as the president and manager
of Mobilia
Products,
stole
prototype
furniture
from
Mobilia for the exhibit.
The Board of Directors of MPI
filed
a
complaint
against
Umezawa for two counts of
qualified
theft
allegedly
committee. Attached to the
complaint
was
the
Joint
Affidavit of Danilo Lallaban,
George del Rio and Yasushi
Kato. The public prosecutor
filed
an
Information
for
qualified
theft
against
Umezawa with the (RTC) of
Lapu-Lapu City
The trial court asserted that the
controversy involving the criminal
cases was between Umezawa and
the other stockholders of MPI. It
also held that the SEC, not the
trial
court,
had jurisdiction
over
intra-corporate
controversies.CA affirmed the
ruling of the RTC that the dispute

between Umezawa and the other


stockholders and officers over
the implementation of the
MPIs standard procedure is
intra
-corporate in nature; hence, within
the exclusive jurisdiction of the
SEC. The petitioner MPI filed
the instant petition for review
on certiorari.
ISSUE:
Whether the RTC has
jurisdiction over the crimes
charged
in
the
said
Informations
HELD:
In criminal prosecutions,
it is settled that the jurisdiction
of the court is not determined
by what may be meted out to
the offender after trial or even
by the result of the evidence
that would be presented at the
trial, but by the extent of the
penalty
which
the
law
imposes for the misdemeanor,
crime or violation charged in
the complaint. If the facts
recited in the complaint and
the punishment provided for by
law are sufficient to show that
the
court
in
which
the
complaint is presented has
jurisdiction, that court must
assume jurisdiction
Based
on
the
material
allegations of the Informations
in the three cases, the RTC had

exclusive jurisdiction over the


crimes charged.
The bare fact that the
respondent was the president
and general manager of the
petitioner corporation when the
crimes charged were allegedly
committed and was then a
stockholder thereof does not in
itself deprive the RTC its
exclusive jurisdiction over the
crimes charged. The property
of the corporation is not the
property of the stockholders or
members or of its officers who
are stockholders.
The relationship of the
party-litigants with each other
or
the
position
held
by
petitioner as a corporate officer
in respondent MPI during the
time he committed the crime
becomes merely incidental and
holds
no
bearing
on
jurisdiction. What is essential
is that the fraudulent acts are
likewise of a criminal nature
and hence cognizable by the
regular
courts. Thus,
notwithstanding the fact that
respondent Umezawa was the
president and general manager
of
petitioner
MPI
and
a
stockholder thereof, the latter
may still be prosecuted for the
crimes charged. The alleged
fraudulent acts of respondent
Umezawa
in
this
case
constitute estafa.

MACASAET vs PEOPLE, G. R. No.


156747. February 23, 2005
FACTS:
Alfie Lorenzo, Allen Macasaet,
Nicolas Quijano, Jr., and Roger
Parajes,
columnist,
managing
editor, and editor, respectively of
the
newspaper
Abante
were
charged before the RTC Quezon
City, with the crime of libel against
Joey Trinidad, to his damage and
prejudice.
The Public prosecutor argued that
the RTC Q.C. had jurisdiction over
the case and maintained that
during the time material to this
case, private respondent was a
resident of Sikatuna Village, Q.C.
and Marikina, Metro as can be
determined in his reply affidavit.
Petitioners and co-accused alleged
that it was incorrect for the public
prosecutor to refer to the affidavit
purportedly executed by the private
respondent as it is axiomatic that
the resolution of a Motion to Quash
is limited to a consideration of the
information as filed and no other.
Further, as both the complainantaffidavit
executed
by
private
respondent and information filed
before the court state that the
private respondents residence is in
Marikina City, the dismissal of the
case is warranted for the rule is
that jurisdiction is determined
solely by the allegations contained
in the complaint or information.

Private Respondent contended that


the certification issued by the brgy.
Chairman of Brgy. Malaya was
issued after he had already moved
out of the apartment unit was
renting Sikatuna Village Q.C.
Trial court ordered the dismissal of
the case due to lack of jurisdiction
noting
that
although
the
information alleged the venue of
this
case
falls
within
the
jurisdiction of Q.C. RTC.
Private respondent insist that at
the time the alleged libelous article
was published, he was actually
residing in Quezon City, that he
mistakenly stated that he was a
resident of Marikina City at the
time
of
publication
of
the
defamatory article because he
understood the term address to
mean the place where he came
originally.
ISSUE:
Whether
or
not
CA
committed a reversible error in
ruling that the RTC Q.C. has
territorial jurisdiction over the
crime charged
HELD:
In criminal actions, it is a
fundamental rule that venue is
jurisdictional. Thus, the place
where the crime was committed
determines not only the venue
of the action but is an essential
element of jurisdiction.

The law, however, is more


particular in libel cases. The
possible
venues
for
the
institution of the criminal and
the civil aspects of said case
are concisely outlined in Article
360 of the Revised Penal Code,
as amended by Republic Act
No. 4363.
1. Whether the offended party
is a public official or a private
person, the criminal action may
be filed in the Court of First
Instance of the province or city
where the libelous article is
printed and first published.
2. If the offended party is a
private individual, the criminal
action may also be filed in the
Court of First Instance of the
province where he actually
resided at the time of the
commission of the offense.
3. If the offended party is a
public officer whose office is in
Manila at the time of the
commission of the offense, the
action may be filed in the Court
of First Instance of Manila.
4. If the offended party is a
public officer holding office
outside of Manila, the action
may be filed in the Court of
First Instance of the province
or city where he held office at
the time of the commission of
the offense
In the case at bar, private
respondent
was
a
private

citizen at the time of the


publication
of
the
alleged
libelous article, hence, he could
only file his libel suit in the City
of Manila where Abante was
first
published
or
in
the
province or city where he
actually resided at the time the
purported libelous article was
printed.
LACSON
vs
EXECUTIVE
SECRETARY,
G.R.
No.
128096. January 20, 1999
FACTS:
Petitioner
Lacson
is
assailing the constitutionality
of
Sections
4 and
7
of R A N o . 8 2 4 9 a n a c t
w h i c h f u r t h e r d e fi n e s t h
e j u r i s d i c t i o n o f t h e Sand
iganbayan. Lacson was being
held liable for the killing of the
Kuratong Baleleng gang by
elements of the Anti-Bank
Robbery
and
Intelligence
Task Group. It was contended
that it was a rub-out and not a
shoot-out which took place on
the night of May 18, 1995.The
PNP off icers were originally
absolved from any liability
because of the finding of the
Blancaflor Commission that the
incident was a legitimate police
operation. However,
after
a review
of
Ombudsman
Villa,
the
fi ndings
were
modifi ed thus Lacson and 11
others were charged with
murder. All of the accused
questioned the jurisdiction of
the Sandiganbayan saying that

the
RTC
should
take
cognizance of the case at
bar since by virtue of RA
7975, the jurisdiction of the
Sandiganbayan was only to
cases
were
the
principal
accused is a public officer with
salary Grade of 27. It was
contended that the highest
principal
accused
in
the
amended information has the
rank of Chief Inspector only,
and none has the equivalent
of SG 27 While the motions
for
reconsideration
were
pending, RA 8249 was passed
by the Congress, which expands
the
jurisdiction
of
the Sandiganbayan by deleting
the word principal from
the
phrase
principal
accused in Section 2 of
RA7975. The new law now
expands the jurisdiction of
the Sandiganbayan to cover
Lacson.
The amended information
m e r e a l l e g e s t h a t t h e o ff e
n s e c h a r g e d w a s committed
by the accused public officer in
relation to his office.
ISSUE:
Whether
or
not
the
SANDIGANBAYAN
has
jurisdiction over the offense
HELD:
The amendment in RA
no 8249 that in cases where
none of the accused are
occupying
positions
corresponding to Salary grade
27 or higher, as prescribed

in the said RA 6758, or


military and PNP off icers
mentioned
above, exclusive
original jurisdiction thereof
shall be vested in the proper
RTC, MTC, MeTC, MTCT, as
the case may be in pursuant
to BP 129. The previous law
vests jurisdiction in the RTC
where none of the principal
accused
are
occupying
positions
corresponding
to
Salary Grade 27. The term
principal was deleted so that
under the amendment, if an
accomplice belongs to Salary
grade 27, then jurisdiction
is with the Sandiganbayan
even
if
none
of
the
principals belong to a lower
salary
grade. The
amendment
was
applied
retroactively.
The jurisdiction of the court is
defined by the Constitution or
statute. The elements of that
definition must appear in the
complaint or information so as
to ascertain which court has
jurisdiction over a case. Hence,
the
elementary
rule
that
jurisdiction of the court is
determined by the allegations
in
the
complaint
or
information, and not by the
evidence presented the parties
at the trial.

UY vs SANDIGANBAYAN, 312
SCRA 77.

FACTS:
Petitioner George Uy was
the deputy comptroller of the
Philippine Navy designated to
act on behalf of Captain
Fernandez,
the
latters
supervisor, on matters relating
the activities of the Fiscal
Control
Branch. Six
informations
for
Estafa through falsifi cation
of off icial documents and on
e information for violation
of Section 3 of RA 3019
(anti-graft
and
corrupt
practices act) werefiled with
the Sandiganbayan against the
petitioner and 19 other accused
for alleged. The petitioner
was said to have signed a
P.O. stating that the unit
received 1,000 pieces of seal
rings when in fact, only 100
were
ordered.
The
Sandiganbayan recommended
that
the infomations
be
withdrawn against some of
the accused
after
a
comprehensive
investigation.
Petitioner fi led a motion to
quash contending that it is
the Court Martial and not the
Sandiganbayan
which
has
jurisdiction over the offense
charged or the person of the
accused. Petitioner
further
contends
that
RA
1850
which
provides
for
the
jurisdiction of court martial
should govern in this case.
ISSUE:
Whether
Sandiganbayan

or
has

not
no

jurisdiction over the offense


charged or the person of the
accused.

conditions sine qua non before


the Sandiganbayan can validly
take cognizance of the case.

HELD:
The Sandiganbayan has
no jurisdiction over petitioner,
at the time of the filing of the
informations,
and
as
now
prescribed by law.

CUYOS vs GARCIA, 160 SCRA


302
FACTS:

The fundamental rule is


that the jurisdiction of a court
is determined by the statute in
force at the time of the
commencement
of
the
action. Thus,
Sandiganbayan
has no jurisdiction over the
petitioner at the time of the
filing o f t h e i n f o r m a t i o n s
and
as now
prescribed
by law. RA
8249,
the
l a t e s t amendment of PD
1606
creating
the
Sandiganbayan provides that
such will have jurisdiction
over violations of RA 3019 of
members of the Philippines
Army and air force colonels,
naval captains and all officers
of higher rank.
In the instant case, while
petitioner
is
charged
with
violation of Section 3(e) of R.A.
No. 3019, as amended, which is
an offense covered by Section 4
of the Sandiganbayan Law, his
position
as
Lieutenant
Commander (LCMDR.) of the
Philippine Navy is a rank lower
than naval captains and all
officer of higher rank.
Both the nature of the
offense
and
the
position
occupied by the accused are

Petitioner Alfredo Cuyos


was
charged
with homicidew i t h m u l t i p l e
serious physical injuries a
n d d a m a g e t o proeperty
through
reckless
imprudence
before the Municipal Court of San
Fernando, Pampanga. Cuyos
entered a plea of not guilty at
the arraignment and the judge
set the case for trial, but before
it could commence, petitioner
filed a Motion to Remand the
Case to the Court of First
Instance. Cuyos claimed that
there is lack of jurisdiction on
the part of the Municipal Court
and
contended
that
the
damages
suffered
by the
Volkswagen
he hit
amounted to
P18,
000.00.
He
argued
that under
Art. 365, par.
3 of
the Revised Penal Code, the
crime would carry a fi ne in
an amount ranging from the
amount of the damage to three
times the value of the damage
alleged (i.e. 3 x P18, 000.00=P54,
000.00).Under sec. 87 of the
Judiciary
Act
of
1948,MunicipalC o u r t o f Pa m
panga only has jurisdictio

n o v e r o ff e n s e s punishable
by a fine not exceeding P6,
000.00. Cuyos filed an Urgent
Motion
to
Postpone
the
Trial. The
municipal
judge
denied
the
motion
to transfer and set the case
for trial. Cuyos verbal motion
for reconsideration was denied.
Hence, the present petition for
certiorari
ISSUE:
Whether
or
not the
respondent Municipal Court of
San Fernando, Pampanga has
jurisdiction to try the criminal
case against petitioner.

even
a
light
penalty.
Jurisdiction over the whole
complex crime must logically
be lodged with the trial court
having jurisdiction to impose
the maximum and most serious
penalty
imposable
on
an
offense forming part of the
complex crime. A complex
crime must be prosecuted
integrally,
as
it
were,
and not split
into
its
component offenses and the
latter made the subject of
multiple informations possibly
brought in different courts.
This is the thrust of our case
law on the matter.

HELD:
Since
the
maximum
fine
imposable in the present case
is
P54,000.00,
and
the
maximum
imprisonment
imposable (for the homicide
through reckless imprudence)
is six (6) years, clearly, the
criminal charge involved falls
outside the jurisdiction of the
Municipal Trial Court and
consequently
within
the
jurisdiction of the Regional
Trial Court of San Fernando,
Pampanga.
In complex crimes, it is not
uncommon
that
one
constitutive
offense
carries
with it an afflictive penalty
while the other or other
constitutive offenses carry with
them only a correctional or

PALANA vs PEOPLE, 534 SCRA


296.
FACTS:
Isidro Palana was charged for
the violation of BP 22 for
issuing a bouncing check as a
loan security to Alex Carlos, his
business partner, even though
he knew that he did not have
sufficient
funds
with
the
drawee bank. The warrant of
arrest was recalled and set

aside after Palana posted bail.


He was then arraigned and
pleaded not guilty to the
offense charged. Palana raised
the issue of investment. He
alleged that Carlos cajoled him
to issue a check in his favor
allegedly to be shown to a
textile supplier who would
provide the partnership with
the
necessary
raw
materials. Petitioner
alleged
that when the check was
issued, complainant knew that
the same was not funded. The
RTC decided in favor of Carlos.
Palana appealed before the
Court of Appeals but it was
dismissed, affirming the RTCs
decision
in toto. Both thetrial court and
the Court of Appeals found that
the check was issued as a
guaranty for the loan, thereby
rejecting
petitioners
investment theory.
The trial court noted that the
so-called partnership venture,
Palanas
General
Merchandising, was registered
on December 1, 1987 only in
the name ofpetitioner. The
Court of Appeals also held that
the act of lending moneydoes
not necessarily
amount to
an investment of capital
ISSUE:
Whether or not Regional
Trial Court has jurisdiction
over the case despite the fact
that at the time accused was
arraigned the jurisdiction
of

Metropolitian Trial Court was


already in effect.
HELD:
It is hornbook doctrine
that jurisdiction to try a
criminal action is determined
by the law in force at the time
of
the institution of
the
action and
not
during
the
arraignment
of
the
accused. The
Information
charging
petitioner
with
violation of B.P. Blg. 22 was
filed on August 19, 1991.
Violation of B.P. Blg. 22 is
punishable with imprisonment
of not less than 30 days but not
more than one year or by a fine
of not less than but not more
than double the amount of the
check which fine shall in no
case exceed P200,000.00, or
both fine and imprisonment at
the discretion of the court. In
the present case, the fine
imposable
isP200,
000.00
hence, the Regional Trial Court
properly acquired jurisdiction
over the case. The Metropolitan
Trial Court could not acquire
jurisdiction over the criminal
action because its jurisdiction
is only for offenses punishable
with a fine of not more than P4,
000.00.
Where a court has already
obtained and is exercising
jurisdiction over a controversy,
its jurisdiction to proceed to
the final determination of the
cause is not affected by new
legislation placing jurisdiction
over
such
proceedings
in

another tribunal unless the


statute expressly provides, or is
construed to the effect that it is
intended to operate on actions
pending before its enactment.
PEOPLE vs LAGON, 185 SCRA
442.

FACTS:

On July 7 1976 a criminal


action was filed with the City
Court
of
Roxas
charging
Lagon
with
estafa
for
allegedly issuing a P4, 232
check as payment for goods
knowing she had insufficient
funds. However on Dec.2, as
the trial commenced, the City
Court
dismissed
the
information on the ground
that the penalty prescribed
by law for estafa was beyond
the courts authority to impose.
Hence this petition for review

ISSUE:
Whether
the
court
jurisdiction is determined by
the law in force at the time of
the institution of the action or
at the time of the commission
of the crime
HELD:
It
is
firmly
settled
doctrine
that
the
subject
matter jurisdiction of a court in
criminal
law
matters
is

properly measured by the law


in effect at the time of the
commencement of a criminal
action, rather than by the law
in effect at the time of the
commission of the offense
charged
Subject-matter jurisdiction in
criminal cases is determined by
the authority of the court to
impose the penalty imposable
under the applicable statute
given the allegations of a
criminal information.
In the case at bar, the
increased penalty provided for
the offense charged in Criminal
Case by P.D. No. 818 (prison
mayor in its medium period) is
obviously heavier than the
penalty provided for the same
offense originally imposed by
paragraph 2(d) of Article 315 of
the Revised Penal Code (up
to prision correccional in its
minimum period).
Should
the
criminal
information be refiled in the
proper
court,
the
proper
Regional Trial Court, that court
may not impose that more
onerous penalty upon private
respondent
Libertad
Lagon
(assuming the evidence shows
that the offense was committed
before 22 October 1975). But
the Regional Trial Court would
remain vested with subjectmatter jurisdiction to try and
decide the (refiled) case even
though the penalty properly
imposable, given the date of

the commission of the offense


charged, should be the lower
penalty originally provided for
in paragraph 2(d) of Article 315
of the Revised Penal Code
which is otherwise within the
exclusive jurisdiction of the
City Court of Roxas City

People vs. Magallanes, G.R. No.


118013-14, October 11, 1995.
FACTS:
Spouses
Dumancas,
P/Col.Nicolas Torres station
c o m m a n d e r o f t h e P N P, w i
t h Po l i c e I n s p e c t o r A b e t o
s cooperation,
Canuday,
Pahayupan,
Lamis,
civilian
agents:
Fernandez,
Divinagracia,D e l g a d o a n d G a
rgallano
were
charged
w i t h m u r d e r.
Judge
Garvilles voluntarily
inhibited himself and the case
was re-raffled. However, the
prosecution moved for the
transmittal of the records to
the
Sandiganbayan because
t h e o ff e n s e s c h a r g e d w e
r e committed in relation t
o t h e o ff i c e o f t h e a c c u s e
d P N P officers. The trial court
ruled that the Sandiganbayan
does
not have
jurisdiction
because the informations do

not state that the offenses were


committed in relation to the
office
of the
accused
PNP off icers and denied the
Motion
for the Transfer of Records t
o Sandiganbayan.
The cases were then re-raffled
to
Regional
Trial
Court
of Bacolod.
The
prosecution fi led
a petition
for certiorari, prohibition an
d mandamus with a prayer for a
temporary restraining order,
challenging the refusal of the
judge to transfer the cases to
the Sandiganbayan. The private
respondents
were
requiredt o c o m m e n t o n t h
e petition and issued a
t e m p o r a r y restraining order
enjoining the respondent judge
to desist from proceeding with the
trial of the case
ISSUE:
Whether
or
not
the
Sandiganbayan has jurisdiction
over the case
HELD:
It is an elementary rule
that jurisdiction is determined
by the allegations in the
complaint or information, and
not by the result of evidence
after trial. The informations in
Criminal Cases do not indicate
that the accused arrested and
investigated the victims and
then killed the latter in the
course of the investigation. The

informations merely allege that


the accused, for the purpose of
extracting or extorting the sum
of P353, 000.00, abducted,
kidnapped, and detained the
two victims, and failing in their
common purpose, they shot and
killed the said victims. For the
purpose
of
determining
jurisdiction,
it
is
these
allegations that shall control,
and not the evidence presented
by the prosecution at the trial.
The
allegation
of
"taking
advantage of his position" or
"taking advantage of their
respective
positions"
incorporated
in
the
informations is not sufficient to
bring the offenses within the
definition
of
"offenses
committed in relation to public
office."
Jurisdiction once acquired is
not affected by subsequent
legislative enactment placing
jurisdiction in another tribunal.
It remains with the court until
the
case
is
finally
terminated. Sandiganbayan or
the courts, as the case may be,
cannot
be
divested
of
jurisdiction over cases filed
before them by reason of R.A.
No. 7975. They retain their
jurisdiction until the end of the
litigation

PEOPLE vs SANDIGANBAYAN
and AMANTE, G.R. No. 167304,
August 25, 2009.
FACTS;
Victoria Amante was a member
of the
Sangguniang
Panlungsod of Toledo City,
Province of Cebu at the time
pertinent to this case. She was
able to get hold of a cash
advance under a disbursement
voucher in order to defray
seminar
expenses
of
the
Committee
on
Health
and
Environmental
Protection,
which she headed. After almost
two years since she obtained
the said cash advance, no
liquidation was made. Toledo
City Auditor issued a demand
letter to respondent Amante
asking the latter to settle her
unliquidated
cash
advance
within seventy-two hours from
receipt of the same demand
letter.
OMB-Visayas,
issued
a
Resolution recommending the
filing of an Information for
Malversation of Public Funds
against respondent Amante.
The Office of the Special
Prosecutor (OSP), upon review
of the OMB-Visayas' Resolution,
prepared
a
memorandum
finding probable cause to indict
respondent Amante. The OSP
filed an Information with the
Sandiganbayan
accusing
Victoria Amante of violating
Section89 of P.D. No. 1445. The
case
was
raffled
to
the

Sandiganbayan.
Thereafter,
Amante filed with the said
court a MOTION TO DEFER
ARRAIGNMENT AND MOTION
FOR REINVESTIGATION
and
was opposed by The OSP. The
Sandiganbayan,
in
its
Resolution dismissed the case
against Amante.
Hence,
the
present petition
ISSUE:
Whether
or
not
the
Sandiganbayan has Jurisdiction
over the case involving a
Sanguniang Panglungsod

HELD:
The applicable law in this
case is Section 4 of P.D. No.
1606, as amended by Section 2
of R.A. No. 7975 which took
effect on May 16, 1995, which
was again amended onFebruary
5, 1997 by R.A. No. 8249. The
alleged commission of the
offense, as shown in the
Information
was
on
or
about December 19, 1995 and
the filing of the Information
was
on May
21,
2004. The
jurisdiction of a court to try a
criminal
case
is
to
be
determined at the time of the
institution of the action, not at
the time of the commission of
the offense.
In order for the Sandiganbayan
to acquire jurisdiction over the

said offenses, the latter must


be
committed
by,
among
others, officials of the executive
branch occupying positions of
regional director and higher,
otherwise classified as Grade
27
and
higher,
of
the
Compensation
and
Position
Classification
Act
of
1989. However, the law is not
devoid
of
exceptions. Those
that are classified as Grade 26
and below may still fall within
the
jurisdiction
of
the
Sandiganbayan provided that
they hold the positions thus
enumerated by the same law.
By simple analogy, applying the
provisions of the pertinent law,
respondent
Amante,
being
member of the Sangguniang
Panlungsod at the time of the
alleged
commission
of
an
offense in relation to her office,
falls
within
the original
jurisdiction
of
the
Sandiganbayan

PEOPLE vs CAWALING, 293 SCRA


267.
FACTS:
Prior to the institution of
the criminal case against all
the
appellants,
an
administrative case had been
filed before the National Police

Commission,
in
which
Policemen
Ernesto
Tumbagahan, Ricardo De los
Santos,
Hilario
Cajilo
and
Andres
Fontamillas
were
charged by Nelson Ilisaz with
the killing of his brother
Ronie Ilisan. On April 6, 1986,
Adjudication Board rendered
its
Decision
which
found
Tumbagahan, De los Santos,
Cajilo and Fontamillas guilty of
grave misconduct and ordered
their dismissal from the service
with prejudice. The Board
issued a resolution, dismissing
the respondents motion for
reconsideration for lack of
merit.
Subsequently,
Second
Assistant
Provincial
Fiscal
Alexander Mortel filed, before
RTC
of
Odiongan,
Romblon, and an Information
for
murder
against
the
appellants
and
Andres
Fontamillas.
Accused Tumbagahan, De
los
Santos,
Cajilo
and
Fontamillas,
with
the
assistance of their lawyers Atty.
Abelardo V. Calsado and Juanito
Dimaano, pleaded not guilty
when arraigned while Accused
Cawaling, assisted by Counsel
Jovencio Q. Mayor, entered a
plea of not guilty also.
RTC found the all the accused
guilty

In their joint brief, Appellants


Tumbagahan and Cajilo cite
that the lower court committed
error in not dismissing the case
for want of jurisdiction.
ISSUE:
Whether or not the RTC
has jurisdiction over the said
case
HELD:
The jurisdiction of a court
to try a criminal case is
determined by the law in force
at the time of the institution of
the action. Once the court
acquires jurisdiction, it may not
be ousted from the case by any
subsequent events, such as a
new legislation placing such
proceedings
under
the
jurisdiction
of
another
tribunal. The only recognized
exceptions to the rule, which
find no application in the case
at bar, arise when: (1) there is
an express provision in the
statute, or (2) the statute is
clearly intended to apply to
actions pending before its
enactment
The
office
must
be
a
constituent element of the
crime as defined in the statute,
such as, for instance, the
crimes defined and punished in
Chapter Two to Six, Title Seven,
of the Revised Penal Code.
Public office is not the essence
of murder. The taking of human

life
is
either
murder
or
homicide whether done by a
private
citizen
or
public
servant, and the penalty is the
same
except
when
the
perpetrator, being a public
functionary, took advantage of
his office, as alleged in this
case, in which event the penalty
is increased.

(eg.Gaffs, fighting cocks, etc).


Delacruz and company were
followed
by
the
cockfight
operators on their way back
to the
PC
headquarters,
fighting
ensued
wherein
Delacruz shot Cabito. Delacruz
is charged with homicide in the
CFI of Davao.
While the case was pending
trial, Presidential Decree Nos.
1822
and
1822-A
were
promulgated by the President
of the Philippines on January
16, 1981, vesting in courtsmartial jurisdiction over crimes
committed by members of the
Armed
Forces
or
of
the
Philippine
Constabulary
in
performance of their duties.

Dela Cruz vs. MOYA, G.R. No. L65192 April 27, 1988.

Claiming that the crime for


which he was charged was
committed in relation to the
performance of his duties, Dela
Cruz filed with the Court of
First Instance of Davao a
motion to transfer the case to
the military authorities so he
could be tried by court martial.
The motion was denied. Hence,
the present petition.

FACTS:
Dela Cruz is a Member of the
Armed Forces Intelligence and
Operations
Sectionarmed
with a MISSION ORDER, Dela
Cruz proceeds to Maco, Davao
del Norte to investigate reports
of illegal cockfighting being
conducted.
Delacruz
and
company catches operators of
cockfighting,
including
deceased Eusebio Cabito in
flagrante.
Delacruz
and
company
fails
to
arrest
operators
but
confiscates
evidence
of
the
crime

ISSUE:
Whether
or
not
the
DAVAO CFI has jurisdiction
over the case
HELD:
The information
w a s fi l e d o n August 2, 1979.
On such date, General Order
No. 59, dated June 24, 1977

published
in
the
Official
Gazette,
states
that
military tribunals created
under
General Order No.
8 can
exercise
exclusive
jurisdiction over all offenses
committed
by
military
personnel of the Armed Forces
of the Philippines while in the
performance of their duties.
Clearly
PD.
1822and
PD.
1822-A
were
promulgated
after
the
fi lling
of
the
complaint
however,
General
Order 59 was enacted before
the commission of the crime. In
the case at bar, it is not
disputed that at the time of the
commission of the alleged
offense, petitioner Dela Cruz
was a member of the Philippine
Constabulary, and that the
shooting
of
the
deceased
Cabilto was committed while
petitioner was executing the
Mission Order
Jurisdiction over the subject
matter is determined by the
statute in force at the time of
the commencement of the
action. And once jurisdiction is
vested in the court, it is
retained up to the end of the
litigation.

PEOPLE vs SANDIGANBAYAN
and PLAZA, G.R. No. 169004,
September 15, 2010.
FACTS:
The
accused,
Rolando
Plaza was a member of the
Sangguniang Panlungsod of
Toledo City, Cebu, with a salary
grade 25. He was charged in
the Sandiganbayan for violating
Section 89 of P.D. No. 1445 or
The Auditing Code of the
Philippines because of for his
failure to liquidate the cash
advances
he
received
on
December
19,
1995
in
the amount
of
Thirty-Three
Thousand Pesos (P33, 000.00)
which he received by reason of
his office, for which he is duty
bound to liquidate the same
within the period required by
law. Plaza filed a motion to
dismiss
with
the
Sandiganbayan. Eventually, the
Sandiganbayan promulgated its
Resolution dismissing the case
for lack of jurisdiction, without
prejudice to its filing before the
proper
court. Thus,
the
present petition
Petitioner
allege
that
Sandiganbayan has criminal
jurisdiction
over
cases involving public officials
and
employees
enumerated
under Section 4 (a) (1) of P.D.
1606, (as amended by Republic
Act[R.A.] Nos. 7975 and 8249),
whether or not occupying a
position classified under salary
grade 27 and above, who are

charged not only for violation


of R.A. 3019, R.A. 1379 or any
of the felonies included in
Chapter II, Section 2, Title VII,
Book II of the Revised Penal
Code,
but also
for
crimes
committed in relation to office.
ISSUE:
Whether
or
not
the
Sandiganbayan has jurisdiction
over
a
member
of
the Sangguniang
Panlungsod whose salary grade
is below 27 and charged with
violation of The Auditing Code
of the Philippines.
HELD:
The jurisdiction of a court
to try a criminal case is to be
determined at the time of the
institution of the action, not at
the time of the commission of
the offenses.
In order for the Sandiganbayan
to acquire jurisdiction over the
said offenses, the latter must
be
committed
by,
among
others, officials of the executive
branch occupying positions of
regional director and higher,
otherwise classified as Grade
27
and
higher,
of
the
Compensation
and
Position
Classification
Act
of
1989. Those that are classified
as Grade 26 and below may still
fall within the jurisdiction of
the Sandiganbayan provided
that they hold the positions
thus enumerated by the same

law. Particularly and exclusively


enumerated
are
provincial
governors,
vice-governors,
members of the sangguniang
panlalawigan, and provincial
treasurers,
assessors,
engineers, and other provincial
department heads; city mayors,
vice-mayors, members of the
sangguniang
panlungsod.
Section 4 (b) of the same law
provides that other offenses or
felonies committed by public
officials
and
employees
mentioned in subsection (a) in
relation to their office also fall
under the jurisdiction of the
Sandiganbayan.
Respondent
Plaza,
a
member of the Sangguniang
Panlungsod during the alleged
commission of an offense in
relation
to
his
office,
necessarily falls within the
original jurisdiction of the
Sandiganbayan.

PEOPLE vs CHUPECO, L-19568,


October 31, 1967.
FACTS:
Jose
Chupeco
was
charged in the CFI Manila for
executing a Chattel Mortgage
of the SAWMILL MACHINERY
AND EQUIPMENT in favor of
Agricultural
and
Industrial
Bank located in Bataan whose

capital,
assets,
accounts,
contracts
and
choosesinaction were subseq
uently transferred to Rehabi
litation
Finance Corp. herein compla
inant withprincipal off ice in
Manila. W i t h o u t h a v i n g f u
l l y s a t i s fi e d t h e mortgage
and during the term without
the consent of the mortgagee
bank and with intent to defraud
Rehabilitation
Finance Corporation, pled
ge and encumber the said
property to one Mateo Penile.
Accused moved to quash the
information on the ground
that more than one off ense
is charged and that the court had
no jurisdiction.
The Court denied the motion.
After the case was partly tried,
the defense counsel and the
fiscal
entered
into
an
agreement
to
have
the
information amended to the
effect that the charge be only
for
removal
of
properties
mortgaged,
eliminating
the
portion referring to pledging
already pledged property.
The
information,
however,
remained
un-amended.
The
accused then filed a motion to
dismiss
involving
the
agreement,
but
the
court
denied it, and ordered that the
case be tried on the charge "of
having pledged property which
had been previously pledged or
mortgaged". After trial, the
court found the accused guilty
of the said offense

ISSUE:
Whether
or
not
CFI
Manila has jurisdiction over the
case
HELD:
The original terms of the
charge averred the crime of
repledging already encumbered
property without the creditor's
consent,
and
one
of
the
essential ingredients of the
offense (the execution of the
first mortgage) having been
alleged, to have taken place in
Manila, the court of first
instance of that city acquired
jurisdiction over the offense
under the Rules of Court. It is
well-established
that
once
vested, the jurisdiction is not
tolled
by
subsequent
amendment
or
stipulation
which in this case amounted to
no more than an avowal by the
prosecution that it could not
establish the other elements of
the offense.
But the fatal error in the
decision appealed from is its
disregard of the fact that the
evidence fails to show that the
properties mortgaged to the
bank
are
the
same
one
encumbered
afterwards
to
Mateo Pinili.

An assential element common


to the two acts punished under
Article 319 of the Revised Penal
Code is that the property
removed or repledged, as the
case may be should be the same
or identical property that was
mortgaged or pledged before
such removal or repledging.
Even if the Court of First
Instance
of
Manila
had
jurisdiction over the case, the
accused cannot be found guilty
on the evidence on record of
the crime for which he stands
indicted.

Rilloraza v.
Judge Arciga, L23848, October 31, 1967.
FACTS:
Respondent Banayat, head
teacher, filed in the Municipal
Court of San Fernando, La
Union,
against
petitioner,
complaint for direct assault
upon a person in authority,
allegedly
committed
in
Naguilian, La Union. Petitioner
moved to quash, planted on
lack of jurisdiction but it was
denied. Hearing on the merits
in the court's exercise of
original
jurisdiction
started
and
three
prosecution
witnesses were able to testify,

stenographic record of their


testimony
was
taken.
The
petitioner went to CFI La Union
on certiorari and prohibition
and it came up with an order
declaring
the
proceedings
conducted
by
respondent
municipal judge null and void,
and directing him to desist
from
continuing
with
the
hearing of the case, and to
transmit the record thereof to
the
Municipal
Court
of
Naguilian, La Union, for the
necessary
preliminary
investigation.
Judiciary Act of 1948 which
took effect on August 1,
1959, Justices of the peace in
the capitals of provinces and
judges of municipal courts
shall have like jurisdiction as
the Court of First Instance to
try parties charged with an
offense committed within the
province in which the penalty
provided by law does not
exceed prision correccional or
imprisonment for not more
than six years or fine not
exceeding three thousand pesos
or both
Judiciary Act of 1948, amended
by Republic Act 2613 which
took
effect
on
June
22,
1963, Municipal judges in the
capitals of provinces and subprovinces and judges of city
courts
shall
have
like
jurisdiction as the Court of

First Instance to try parties


charged
with
an
offense
committed
within
their
respective
jurisdictions,
in
which the penalty provided by
law does not exceed prision
correccional or
imprisonment
for not more than six years or
fine not exceeding six thousand
pesos or both
ISSUE:
Whether
or
not
municipal
court
jurisdiction over the case

the
has

HELD:
A rule long respected is that
jurisdiction of a court of justice
to try a criminal case is
determined by the law in force
at the time the action is
instituted.3 Since
prosecution
here was started on June 18,
1963 when Republic Act 2613
was in force, this law should be
looked up to in ascertaining
whether or not respondent
judge had jurisdiction to try the
case. By this statute, the justice
of the peace court in provincial
capitals was empowered to hear
and determine cases where the
penalty involved "does not
exceed prision correccional or
imprisonment for not more
than six years or fine not
exceeding three thousand pesos
or both."

The charge is direct assault


upon a person in authority,
encompassed in Article 148 of
the Revised Penal Code. The
penalty
provided
therein
is prision correccional in its
medium and maximum periods
and a fine not exceeding one
thousand pesos. Therefore, the
Justice of the Peace Court of
San Fernando, the capital of La
Union, has jurisdiction to try
this case. Once jurisdiction to
try a criminal case is acquired,
that jurisdiction remains with
the court until the case is
finally determined therein. A
subsequent statute removing
jurisdiction "will not operate to
oust
jurisdiction
already
attached.

AZARCON
vs
SANDIGANBAYAN,
G.R.
No.
116033. February 26, 1997.
FACTS:
Azarcon owned and operated an
earth-moving business, hauling
dirt and ore. His services were
contracted
by
PICOP.
Occasionally, he engaged the
services of sub-contractors like
Jaime Ancla whose trucks were
left at the formers premises.
Warrant
of
Distraint
of
Personal Property was issued
by BIR commanding one of its
Regional Directors to distraint

the goods, chattels or effects


and other personal property of
Jaime Ancla, a sub-contractor
of accused Azarcon
and
a
delinquent taxpayer. A Warrant
of Garnishment was issued to
and
subsequently
signed
by accused Azarcon
ordering
him
to transfer,
surrender,
transmit and/or remit to BIR
the property in his possession
owned by Ancla. Azarcon then
volunteered himself to act as
custodian of the truck owned
by Ancla.
Azarcon wrote a letter to the
Reg. Dir of BIR stating that
while
he
had
made
representations
to
retain
possession of the property
of Ancla,
he
thereby
relinquishes
whatever
responsibility he had over the
said
property
since Ancla surreptitiously
withdrew his equipment from
him which was under his
responsibility as he volunteered
as its custodian.
BIR Dir. Gen. replied that while
Azarcon were in possession of
the dump truck owned by
JAIME
ANCLA
which
he
voluntarily
assumed
the
liabilities of safekeeping and
preserving the unit in behalf of
the
Bureau
of
Internal
Revenue, his failure therefore,
to observe said provisions does
not
relieve
you
of
your
responsibility
Azarcon with his co-accused
was
charged
before
the

Sandiganbayan with the crime


of malversation of public funds
or property and they were
sentenced to suffer the penalty
of imprisonment ranging from
10 yrs and 1 day of prision
mayor in its maximum period to
17 yrs, 4 mos and 1 day of
reclusion temporal. Petitioner
filed a motion for new trial
which was subsequently denied
by Sandiganbayan. Hence, this
petition.

ISSUE:
Whether
or
not
the
Sandiganbayan does not have
jurisdiction
over
crimes
committed solely by private
individuals.
HELD:
It is hornbook doctrine
that in order (to) ascertain
whether
a
court
has
jurisdiction
or
not,
the
provisions of the law should be
inquired into.[25] Furthermore,
the jurisdiction of the court
must appear clearly from the
statute law or it will not be held
to exist. It cannot be presumed
or implied. And for this purpose
in
criminal
cases,
the
jurisdiction of a court is
determined by the law at the
time of commencement of the
action.

In this case, the action was


instituted with the filing of this
information on January 12,
1990; hence, the applicable
statutory provisions are those
of P.D. No. 1606, as amended by
P.D. No. 1861 on March 23,
1983,
but
prior
to
their
amendment by R.A. No. 7975
on May 16, 1995
In case private individuals are
charged
as
co-principals,
accomplices or accessories with
the
public
officers
or
employees,
including
those
employed in government-owned
or controlled corporations, they
shall be tried jointly with said
public officers and employees.
Petitioner did not cease to be a
private individual when he
agreed to act as depositary of
the
garnished
dump
truck. Therefore,
when
the
information charged him and
Jaime
Ancla
before
the
Sandiganbayan
for
malversation of public funds or
property, the prosecution was
in fact charging two private
individuals without any public
officer being similarly charged
as
a
coconspirator. Consequently, the
Sandiganbayan
had
no
jurisdiction
over
the
controversy and therefore all
the proceedings taken below as
well as the Decision rendered
by Respondent Sandiganbayan,
are null and void for lack of
jurisdiction

JALANDONI
SCRA 107.

vs

DRILON,

327

FACTS:
Private respondent Jaime
Ledesma,
filed
an
administrative complaint for
violation of the Revised Penal
Code and the Anti-Graft and
Corrupt Practices Act against
the petitioner with PCGG
There was a publication fullpage
advertisements
simultaneously published in
five
major
dailies
which
contained allegations naming
herein petitioner who was then
a
PCGG
Commissioner
of
having committed illegal and
unauthorized acts, and other
wrongdoings constituting graft
and corruption, relative to
the dacion en pago financing
arrangement entered into by
Piedras Petroleum Co., Inc.
with Rizal Commercial Banking
Corporation
Petitioner Jalandoni filed a
complaint for libel before the

Provincial Prosecutor of Rizal


designated
against
officials/directors of Oriental
Petroleum
&
Minerals
Corporation
(OPMC,
for
brevity), namely, Coyiuto, Jr.,
Ledesma,
Garcia,
Barcelon,
Ozaeta, and Dyhongpo.
A Memorandum was issued by
3rd
Assistant
Prosecutor,
approved by Rizal Provincial
Prosecutor, recommending the
indictment
of
private
respondent
Coyiuto,
Jr. An
Information for libel was filed
thereafter with the Regional
Trial Court of Makati.
Respondents appealed to Sec.
of Justice Drilon which directed
the dismissal of case against
respondents.
ISSUE:
Whether or not the Justice
Secretary has jurisdiction to
dismiss the information for
libel against respondents
HELD:
It is a well-settled rule
that the Secretary of Justice
has
the
power
to
review
resolutions or decisions of
provincial or city prosecutors
or the Chief State Prosecutor
upon petition by a proper party.
Under
the
Revised
Administrative
Code,
the
secretary of justice exercises
the power of direct control and
supervision
over
said
prosecutors.
He
may
thus

affirm,
nullify,
reverse
or
modify their rulings as he may
deem fit
Assuming arguendo that
the
extraordinary
writ
of certiorari must prosper, we
must point out to the petitioner
the oft-cited ruling in the case
of Crespo vs. Mogul, once a
complaint or information is
filed in court, any disposition of
the case such as its dismissal
or its continuation rests on the
sound discretion of the court.
Trial judges are thus required
to make their own assessment
of whether the secretary of
justice committed grave abuse
of discretion in granting or
denying the appeal, separately
and
independently
of
the
prosecution's or the secretary's
evaluation that such evidence
is insufficient or that no
probable cause to hold the
accused for trial exists
DOMONDON
SANDIGANBAYAN,
292.

vs
328 SCRA

FACTS:
Four
separate
informations
were filed at the Sandiganbayan
against certain officials of the
Philippine National Police due
to the discovery of a chain of
irregularities within the PNP
Commands. The petitioner was
included as an accused on
account of his approval
of
the Advice
Allotment in
the

amount of P5M and P15M


respectively which amounts to
a violation of the Anti-Graft
Law.
Sandiganbayan issued 2 orders,
the first was ordering the
prosecution to
demonstrate
probable complicity in the
transaction described in the
information and the second
order was differing action on
the motion for consolidation
considering the uncertainty of
the Court in proceeding the
case at
this
time
and
considering that only one of the
15 accused filed a motion for
consolidation.
A Motion to Admit Amended
Information was filed with the
Sandiganbayan and included
petitioner
as
they
were
recommended
for
further
prosecution
by
the
Ombudsman. Petitioner alleges
that respondents Desierto, Villa
and Tamayo acted with grave
abuse of discretion in denying
his motion for consolidation,
claiming that since all of the
pertinent cases have been
remanded
by
the
Sandiganbayan to the Office of
the Special Prosecutor under
the Office of the Ombudsman
for
reinvestigation,
"jurisdiction has revested" in
the
latter and
"it
is
grave abuse
of discretion
to
refuse to perform the duty of
consolidating these cases

ISSUE:
Whether or not the there
was a grave abuse of discretion
amounting to lack or excess of
jurisdiction when respondents
Villa and Desierto denied the
petitioners
motion
for
reconsideration
HELD:
Courts cannot interfere
with the discretion of the fiscal
or Ombudsman to determine
the specificity and adequacy of
the averments of the offense
charged. He may dismiss the
complaint forthwith if he finds
it to be insufficient in form or
substance or if he otherwise
finds no ground to continue
with the inquiry; or he may
proceed with the investigation
if the complaint is, in his view,
in
due
and
proper
form. However,
while
the
Ombudsman
has
the
full
discretion
to
determine
whether or not a criminal case
should be filed, this Court is
not precluded from reviewing
the Ombudsmans action when
there is an abuse of discretion,
by way of Rule 65 of the Rules
of Court. This is an exercise of
the Ombudsmans powers based
upon constitutional mandate
and the courts should not
interfere in such exercise.
PILAPIL vs GARCHITORENA,
399 SCRA 343.

FACTS:
Accused
was
congressman, who receive an
L300 for ambulance in behalf
of the Municipality of Tigaon,
Camarines Sur from PCSO. He
did not deliver such ambulance.
The mayor of the municipality
requested
from
PCSO
and
found out about the donation.
Sandiganbayan
Presiding
Justice Francis Garchitorena,
requested
an
investigation.
Preliminary investigation was
conducted for Malversation of
Public Property under Art 217
of
the
RPC.
Initially,
Ombudsman
Investigator
recommended
malversation
cannot prosper finding no
probable cause but it was
disapproved and filing was
recommended by the Asst.
ombudsman. Until finally the
crime charged is for violation
of Section 3(e) of Republic Act
No. 3019 recommended by
ombudsman Vasquez.
Warrant of arrest was issued,
accused posted bail. Petitioner
predicated his motion to quash
on the ground of lack of
jurisdiction over his person
because the same was filed
without
probable
cause.
Petitioner cites the fact that
the information for violation of
the Anti-Graft Law was filed
although the complaint upon
which
the
preliminary
investigation was conducted is
for
malversation.
Accused
appealed the decision of the

Sandiganbayan
denying
his
quashal and reconsideration.
ISUUE:
May Sandiganbayan deny
the
motion
to
withdraw
information
filed
by
the
Ombudsman
who
found
no prima
facie case
against petitioner without any
valid reason
HELD:
Settled is the rule that
once
a
complaint
or
information is filed in court any
disposition of the case, as its
dismissal or the conviction or
acquittal of the accused, rests
in the sound discretion of the
court. This rule does not
foreclose a reinvestigation or a
review by a superior authority
of
the
resolution
finding
probable
cause. Nevertheless,
once a motion to dismiss or
withdraw
information
is
thereafter filed, the court may
grant or deny it in faithful
exercise of judicial prerogative,
not out of subservience to the
prosecution arm.
The trial judge must himself
be convinced that there was
indeed
no
sufficient
evidence
against
the
accused, and this conclusion
can be arrived at only after
an
assessment
of
the
evidence in the possession of
the prosecution. What was
imperatively required was
the
trial
judge's
own
assessment
of
such

evidence,
it
not
being
sufficient for the valid and
proper exercise of judicial
discretion merely to accept
the prosecution's word for
its supposed insufficiency.
It has not been satisfactorily
shown that the Second Division
of the Sandiganbayan denied
arbitrarily or despotically the
Motion
to
Withdraw
the
Information
PALANA vs PEOPLE, 534 SCRA
296.
FACTS:
Isidro Palana was charged for
the violation of BP 22 for
issuing a bouncingcheck as a
loan security to Alex Carlos.
The warrant of arrest was
recalled and set aside after
Palana posted bail. He was then
arraigned and pleaded not
guilty to the offense charged.
Palana raised the issue of
investment. He alleged that
Carlos cajoled him to issue a
check in his favor allegedly to
be shown to a textile supplier
who
would
provide
the
partnership with the necessary
raw materials.
The RTC decided in favor of
Carlos. Palana appealed before
the Court of Appeals but it was
dismissed.
Both the trial court and the
Court of Appeals found that the
check was issued as a guaranty
for the loan, thereby rejecting

petitioners
investment
theory. The trial court noted
that the so-called partnership
venture,
Palanas
General
Merchandising only in the
name of petitioner. The Court
of Appeals also held that the
act of lending money does not
necessarily
amount
to
an investment of capital
ISSUE:
Whether or not the court
has jurisdiction over the case
HELD:
It is hornbook doctrine
that jurisdiction to try a
criminal action is determined
by the law in force at the time
of the institution of the action
and not during the arraignment
of
the
accused. The
Information
charging
petitioner with violation of B.P.
Blg. 22 was filed on August 19,
1991.
The
subsequent
amendment of B.P. 129 by R.A.
No. 7691, An Act Expanding the
Jurisdiction of the Municipal
Trial Courts, Municipal Circuit
Trial
Courts
and
the
Metropolitan
Trial
Court on June 15, 1994 cannot
divest the Regional Trial Court
of jurisdiction over petitioners
case.
Where
a
court
has
already
obtained
and
is
exercising jurisdiction over a

controversy, its jurisdiction to


proceed
to
the
final
determination of the cause is
not affected by new legislation
placing jurisdiction over such
proceedings in another tribunal
unless the statute expressly
provides, or is construed to the
effect that it is intended to
operate on actions pending
before its enactment. Indeed,
R.A.
No.
7691
contains
retroactive
provisions. However, these only
apply to civil cases that have
not yet reached the pre-trial
stage. The
jurisdiction
of
the RTC over the case attached
upon the commencement of the
action by the filing of the
Information and could not be
ousted by the passage of R.A.
No. 7691 reapportioning the
jurisdiction of inferior courts,
the application of which to
criminal cases is prospective in
nature
FUKUZUME
SCRA 570.

vs

PEOPLE,

474

FACTS:
Yu went to the house of
Fukuzume in Paraaque that
with the intention of selling the
subject
aluminum
scrap
wires, the latter pretended
that he is a representative
of
Furukawa who is authorized
to
sell
the
said
scrap
wires; that based on the
false pretense of Fukuzume,

Yu agreed to buy the subject


aluminum scrap wires; that Yu
paid
Fukuzume
the
initial
amount
of P50,000.00
of
the total agreed price of
P290,000.00. that as a result ,
Yu
suffered
damage.
Subsequently,
Yu
filed
a
complaint
with the National Bureau of
Investigation (NBI).
Information is filed with the
RTC
Makati,
charged
Fukuzume with Estafa.
The trial court found Fukuzume
guilty as charged which was
also affirmed by the CA.
Hence, this petition.
Fukuzume contended that the
CA erred in ruling that the RTC
of Makati has jurisdiction
over the offense charged since
contract
of
sale
of
the
aluminum scrap wires took
place
at appellants residence in
Paraaque.
ISSUE:
Whether or not the lack of
jurisdiction
raised
by
Fukuzume for the first time in
CA is proper
HELD:
Rule is settled that an
objection based on the ground
that the court lacks jurisdiction
over the offense charged may be
raised
or
considered motu
propio by the court at any stage of
the
proceedings
or
on

appeal. Moreover, jurisdiction over


the subject matter in a criminal
case cannot be conferred upon the
court by the accused, by express
waiver or otherwise, since such
jurisdiction is conferred by the
sovereign
authority
which
organized the court, and is given
only by law in the manner and
form prescribed by law. While an
exception
to
this
rule
was
recognized
by
this
Court
beginning with the landmark case
of Tijam vs. Sibonghanoy, wherein
the defense of lack of jurisdiction
by the court which rendered the
questioned ruling was considered
to be barred by laches, we find
that the factual circumstances
involved in said case, a civil case,
which justified the departure from
the general rule are not present in
the instant criminal case.

Foz Vs People, G.R. No. 167764,


October 9, 2009
Facts:
In an Information filed before
the
RTC
of
Iloilo
City,
petitioners Vicente Foz, Jr., and
Danny G.Fajardo, columnist and
Editor-Publisher, respectively,
of Panay News were charged
with the crime of libel. They
wrote and publish in the
regular issue of the Panay news

a certain article maliciously


injuring and exposing one Dr.
Edgar
Portigo,
a
company
physician of
San
Miguel
Corporation office, SMC, as an
incompetent doctor and an
opportunist
who
enriched
himself at the expense of the
poor. The RTC rendered its
Decision
finding
petitioners
guilty as charged. Dissatisfied,
petitioners filed an appeal with
the CA which rendered its
assailed Decision affirming in
toto the RTC decision. Hence,
this present petition.

ISSUE:
Whether the information
was
sufficient
to
vest
jurisdiction in the RTC of Iloilo
HELD:
Since Dr. Portigo is a private
individual at the time of the
publication
of
the
alleged
libelous article, the venue of
the libel case may be in the
province or city where the
libelous article was printed and
first published, or in the
province where Dr. Portigo
actually resided at the time of
the commission of the offense.
The
allegations
in
the
Information that Panay News,
a daily publication with a
considerable circulation in the
City of Iloilo and throughout
the region only showed that
Iloilo was the place where
Panay
News
was
in
considerable circulation but did

not establish that the said


publication was printed and
first published in Iloilo City.
While the Information alleges
that Dr. Edgar Portigo is a
physician
and
medical
practitioner in Iloilo City, such
allegation did not clearly and
positively indicate that he was
actually residing in Iloilo City
at the time of the commission
of the offense. It is possible
that Dr. Portigowas actually
residing in another place

Settled is the rule that


jurisdiction of a court over a
criminal case is determined by
the allegations of the complaint
or information, and the offense
must have been committed or
any
one
of
its
essential
ingredients took place within
the territorial jurisdiction of
the court. Considering that the
Information failed to allege the
venue requirements for a libel
case under Article 360, the
Court finds that the RTC of
Iloilo City had no jurisdiction to
hear
this
case.
Thus, its
decision convicting petitioners
of the crime of libel should be
set
aside
for
want
of
jurisdiction without prejudice
to its filing with the court of
competent jurisdiction.

GARCIA vs SANDIGANBAYAN,
G.R. No. 165835, June 22, 2005.
FACTS:
The
Office
of
the
Ombudsman filed for a petition
for
the
forfeiture
of
the
properties allegedly amassed by
then Maj. Gen. Carlos Garcia,
his
wife
Clarita
and
two
children.
Another
forfeiture
case was subsequently filed to
recover funds
Prior to the filing of Forfeiture II,
but subsequent to the filing of
Forfeiture I, the OMB charged the
Garcias and three others with
violation of plunder which
placed the value of the property
and funds plundered, raffled to
the Second Division of the
Sandiganbayan.
As per the Sheriffs return, the
corresponding
summons
involving Forfeiture 1 were
issued and all served on Gen.
Garcia at his place of detention
As regards Forfeiture II, the SB
sheriff
served
the
corresponding summons. In his
return, the sheriff stated giving
the copies of the summons to
the OIC/Custodian of the PNP
Detention Center who in turn
handed
them
to
Gen.
Garcia. The general signed his
receipt of the summons, but as
to those pertaining to the other
respondents,
Gen.
Garcia
acknowledged
receiving
the
same, but with the following
qualifying note: Im receiving

the copies of Clarita, Ian Carl,


Juan Paolo & Timothy but these
copieswill not guarantee it
being served to the abovenamed.
ISSUE:
Whether or not Sandiganbayan
did not acquire jurisdiction
over petitioners person and
that of her children due to a
defective substituted service of
summons.
HELD:
It is basic that a court
must acquire jurisdiction over a
party for the latter to be bound
by its decision or orders. Valid
service
of
summons,
by
whatever mode authorized by
and proper under the Rules, is
the means by which a court
acquires jurisdiction over a
person.
1
Impossibility of prompt person
al service
2 Specific details in the return
3
Substituted
serviceeffected on a person of s
uitable age and discretion resid
ing at defendants
house or residence; or on a
competent person in charge of
defendants office or regular
place of business.
It is apparent that no valid
substituted service of summons
was made on petitioner and her

children, as the service made


through Maj. Gen. Garcia did
not comply with the first two
(2) requirements mentioned
above for a valid substituted
service of summons. Moreover,
the third requirement was also
not strictly complied with as
the substituted service was
made not at petitioners house
or
residence
but
in
the
PNP Detention Center where
Maj. Gen. Garcia is detained,
even if the latter is of suitable
age and discretion. Hence, no
valid substituted service of
summons was made.

FUKUZUME
SCRA 570.

vs

PEOPLE,

474

FACTS:
Yu went to the house of
Fukuzume in Paraaque; that
with the intention of selling the
subject
aluminum
scrap
wires, the latter pretended
that he is a representative
of
Furukawa who is authorized
to
sell
the
said
scrap
wires; that based on the
false pretense of Fukuzume,
Yu agreed to buy the subject
aluminum scrap wires; that Yu
paid
Fukuzume
the
initial
amount
of P50,000.00
of
the total agreed price of

P290,000.00. that as a result ,


Yu
suffered
damage.
Subsequently,
Yu
filed
a
complaint
with the National Bureau of
Investigation (NBI).
In 1994, an information is filed
with the RTC Makati, charged
Fukuzume with Estafa.
The trial court found Fukuzume
guilty as charged which was
also affirmed by the CA.
Hence, this petition.
Upon arraignment, Fukuzume
pleaded
not
guilty.
Trial
ensued, finding the accused
guilty as charged. Aggrieved by
the trial courts decision, he
appealed to CA alleging the
RTCs lack of jurisdiction over
the offense charged but the
latter affirmed the trial courts
decision modifying only the
penalty, hence, the petition
before the SC.

proceedings
or
on
appeal. Moreover, jurisdiction
over the subject matter in a
criminal
case
cannot
be
conferred upon the court by the
accused, by express waiver or
otherwise,
since
such
jurisdiction is conferred by the
sovereign
authority
which
organized the court, and is
given only by law in the manner
and
form
prescribed
by
law. While an exception to this
rule was recognized by this
Court
beginning
with
the
landmark case of Tijam vs.
Sibonghanoy, wherein
the
defense of lack of jurisdiction
by the court which rendered
the questioned ruling was
considered to be barred by
laches, we find that the factual
circumstances involved in said
case,
a
civil
case,
which
justified the departure from the
general rule are not present in
the instant criminal case.

ISSUE:
WON Makati Trial Court
has jurisdiction over the case
since the

Foz, Jr. vs People, G.R.


167764, October 9, 2009.

HELD:

FACTS:

It is noted that it was only in


his petition with the CA that
Fukuzume raised the issue of
the trial courts jurisdiction
over
the
offense
charged.
Nonetheless, the rule is settled
that an objection based on the
ground that the court lacks
jurisdiction over the offense
charged may be raised or
considered motu propio by the
court at any stage of the

In an Information filed before


the
RTC
of
Iloilo
City,
petitioners Vicente Foz, Jr., and
Danny G.Fajardo, columnist and
Editor-Publisher, respectively,
of Panay News were charged
with the crime of libel. They
wrote and publish in the
regular issue of the Panay news
a certain article maliciously
injuring and exposing one Dr.

No.

Edgar
Portigo,
a
company
physician of
San
Miguel
Corporation office, SMC, as an
incompetent doctor and an
opportunist
who
enriched
himself at the expense of the
poor. The RTC rendered its
Decision
finding
petitioners
guilty as charged. Dissatisfied,
petitioners filed an appeal with
the CA which rendered its
assailed Decision affirming in
toto the RTC decision. Hence,
this present petition
ISSUE:
Whether the court can
still entertain petitioners lack
of jurisdiction
HELD:
The Court notes that
petitioners raised for the first time
the issue of the RTC's jurisdiction
over the offense charged only in
their Reply filed before this Court
and finds that petitioners are not
precluded from doing so.
The Court cited the case of
Fukuzume v. People which stated
thatIt is noted that it was only in
his
petition
with
the
CA
that Fukuzume raised the issue of
the trial courts jurisdiction over
the offense charged. Nonetheless,
the rule is settled that an
objection based on the ground
that the court lacks jurisdiction
over the offense charged may be

raised
or
considered motu
proprio by the court at any stage
of the proceedings or on appeal.
Moreover, jurisdiction over the
subject matter in a criminal case
cannot be conferred upon the
court by the accused, by express
waiver or otherwise, since such
jurisdiction is conferred by the
sovereign
authority
which
organized the court, and is given
only by law in the manner and
form prescribed by law. While an
exception
to
this
rule
was
recognized
by
this
Court
beginning with the landmark case
of Tijam vs. Sibonghanoy, wherein
the defense of lack of jurisdiction
by the court which rendered the
questioned ruling was considered
to be barred by laches, we find
that the factual circumstances
involved in said case, a civil case,
which justified the departure from
the general rule are not present in
the instant criminal case.

ANTIPORDA
GARCHITORENA,
551

Jr
321

vs
SCRA

FACTS:

Mayor Licerio Antiporda and


others were charged as accused
for kidnapping and the case
was filed in Sandiganbayan.

The accused filed a Motion


for
New
Preliminary
Investigation and to Hold in
Abeyance
and/or
Recall
Warrant of Arrest Issued. The
same was denied "on the
ground that there was nothing
in the Amended Information
that was added to the original
Information so that the accused
could not claim a right to be
heard
separately
in
an
investigation in the Amended
Information. Additionally,
the
Court ruled that 'since none of
the accused have submitted
themselves to the jurisdiction
of the Court, the accused are
not in a position to be heard on
this matter at this time.

Subsequently, the accused filed


a
Motion
to
Quash
the
Amended Information for lack
of jurisdiction over the offense
charged but it was denied
saying that at all events there
is an Amended Information
here which makes an adequate
description of the position of
the accused thus vesting this
Court with the office related
character of the offense of the
accused.

A
motion
for
reconsideration was filed by the
accused wherein it was alleged
that the filing of the Motion to
Quash and the appearance of
their
counsel
during
the
hearing thereof amounted to
their voluntary appearance and
invested
the
court
with
jurisdiction over their persons
but it was denied.

ISSUE:
Whether
or
not
the
Sandiganbayan has jurisdiction
over the case
HELD:
The Supreme Court do not
have
jurisdiction
over
the
case. The original Information
filed with the Sandiganbayan
did not mention that the
offense
committed
by
the

accused is office-related. It was


only after the same was filed
that the prosecution belatedly
remembered
that
a
jurisdictional fact was omitted
therein.
Petitioners
are
estopped
from assailing the jurisdiction
of the Sandiganbayan for in the
supplemental
arguments
to
motion
for
reconsideration
and/or reinvestigation dated
June 10, 1997 filed with the
same court, it was they who
challenged the jurisdiction of
the Regional Trial Court over
the case and clearly stated in
their
Motion
for
Reconsideration that the said
crime is work connected.
It is a well-settled rule that
a party cannot invoke the
jurisdiction of a court to secure
affirmative relief against his
opponent, and after obtaining
or failing to obtain such relief,
repudiate or question that
same jurisdiction.
Sandiganbayan
has
jurisdiction
over
the
case
because of estoppel and it was
thus vested with the authority
to order the amendment of the
Information.

PEOPLE vs MUNAR, 53 SCRA


278.

FACTS:
Appellant
was
originally
charged with grave slander
having
uttered
defamatory
words in calling the offended
party
Erlinda
Munar
an
unmarried woman and a distant
relative
the
paramour
of
somebody" The municipal court
rejected her
defense of alibi,
sentenced her
to pay
a fi n e
with subsidiary
imprisonment
in
case
of
insolvency and to pay the costs.
Accused-appellant
filed
an
appeal directly to the La Union
court of first instance and filed
appeal bond as fixed by the
municipal court. She was duly
re-arraigned and entered her
not-guilty plea. The trial court
rendered its decision.
The accused filed a motion for
reconsideration
praying
for acquittal and for reduction
of the civil liability" The trial
court
denied reconsideration rejec
ting
the belated objection
raised
for the
fi rst
time by accused
and that
since
the
government
prosecutors were present at the
hearings the prosecution of the
case remained under their
control
and
the private prosecutor/s p
resence and participation
which were then not objec
t e d t o w e r e n o t important.
The accused thereafter filed

her notice of appeal directly to


the Supreme Court.

ISSUE:
Whether
or
not
the
municipal
court
have
jurisdiction
considering
the
participation
of
private
prosecutors despite presence of
public prosecutors at hearings

HELD:

Appellant cannot be permitted


to experiment with the court the court of first instance in the
case of herein appellant - by
submitting
herself
to
its
jurisdiction
and
after
the
experiment
has
proved
unsuccessful for her with the
rendition
of
an
adverse
decision to raise for the first
time its lack of jurisdiction.
The ends of justice would not
be served if such belated
jurisdictional questions were to
be
entertained
and
the
proceedings nullified - when
the court's jurisdiction had
been invoked all the time by
the party who would now
belatedly
question
its
jurisdiction because of its
adverse decision.

TIJAM vs SIBONGHANOY, G.R.


No. L-21450, April 15, 1968.

FACTS:

After one month from the


effectivity of the Judiciary Act
of 1948, spouses Tijam filed a
collection case against spouses
Sibonghanoy. The
preliminary
attachment filed by the plaintiff
was dissolve by a counter bond
posted
by
the
defendants
through a surety company.
After being duly served with
summons the defendants filed
their answer in which, after
making some admissions and
denials
of
the
material
averments of the complaint,
they interposed a counterclaim.
This
counterclaim
was
answered by the plaintiffs.
After
trial,
judgment
was
rendered in favor of plaintiffs.
The writ of execution against
defendants
having
been
returned
totally unsatisfied,
plaintiffs moved for issuance of
writ
of
execution
against
Manila Surety & Fidelity Co.,
Inc. to enforce the obligation of
the bond. But the motion was,
upon the surety's opposition,
denied on the ground that
there was "no showing that a
demand had been made, by the
plaintiffs
to
the
bonding
company for payment of the
amount
due
under
the
judgment"
Surety appealed to CA but it
affirmed
the
trial
courts
ruling. Surety now filed a
motion to dismiss on the
ground of lack of jurisdiction of

CFI Cebu due to affectivity of


Judiciary Act of 1948 a month
before the filing of the petition
for recovery and which placed
original exclusive jurisdiction
of inferior courts all civil
actions
for
demands
not
exceeding 2,000 exclusive of
interest.
CA set aside its earlier decision
and referred the case to SC.

ISSUE:

Whether
or
not
jurisdiction of CFI Cebu can be
contested even if the Surety
raised lack of jurisdiction only
during appeal

HELD:
A party may be estopped or
barred from raising a question
in different ways and for
different reasons. Thus we
speak of estoppel in pais, or
estoppel by deed or by record,
and of estoppel by laches.
Laches, in a general sense is
failure or neglect, for an
unreasonable and unexplained
length of time, to do that
which,
by
exercising
due
diligence, could or should have
been
done
earlier;
it
is
negligence
or
omission
to
assert
a
right
within
a
reasonable time, warranting a
presumption that the party

entitled to assert it either has


abandoned it or declined to
assert it.
The doctrine of laches or of
"stale demands" is based upon
grounds of public policy which
requires, for the peace of
society, the discouragement of
stale claims and, unlike the
statute of limitations, is not a
mere question of time but is
principally a question of the
inequity
or
unfairness
of
permitting a right or claim to
be enforced or asserted.
It has been held that a party
cannot invoke the jurisdiction
of a court to sure affirmative
relief against his opponent and,
after obtaining or failing to
obtain such relief, repudiate or
question that same jurisdiction.

PANGILINAN vs COURT
APPEALS, 321 SCRA 51.

OF

FACTS:
Mila G. Pangilinan was
charged with the crime of
Estafa. On 12 March 1991,
appellant was arraigned before
the Regional Trial Court of
Morong,
Rizal,
where
she
entered
a
plea
of
not

guilty. After due trial convicted


the appellant of the crime of
ESTAFA under Article 315 of
the Revised Penal Code.
This unfavorable verdict was
appealed to the Court of
Appeals which on 13 August
1993, affirmed the conviction
but modified the sentence
A
Motion
for
Reconsideration was denied by
the respondent court on 11
November 1993.On 3 December
1993, appellant filed a petition
for New Trial in the Court of
Appeals which was denied by
said Court on 10 January 1994.
The Office of the Solicitor
General contends that the
appellant is barred from raising
the
issue
of
jurisdiction,
estoppel having already set in.
ISSUE:
Whether or not Pangilinan
is barred by laches in raising
lack of jurisdiction despite the
fact that MTC has jurisdiction
on the case not the RTC

HELD:
The Office of the Solicitor
Generals reliance on the said
ruling
is
misplaced. The
doctrine
laid
down
in
the Tijam case is an exception

to
and
not
the
general
rule. Estoppel attached to the
party assailing the jurisdiction
of the court as it was the same
party who sought recourse in
the said forum. In the case at
bar, appellant cannot in anyway
be said to have invoked the
jurisdiction of the trial court.
We apply the general rule
that jurisdiction is vested by
law and cannot be conferred or
waived by the parties. Even on
appeal
and
even
if
the
reviewing parties did not raise
the issue of jurisdiction, the
reviewing
court
is
not
precluded from ruling that the
lower court had no jurisdiction
over the case.
The operation of the principle
of estoppel on the question of
jurisdiction seemingly depends
upon whether the lower court
had jurisdiction or not. If it had
no jurisdiction, but the case
was tried and decided upon the
theory it had jurisdiction, the
parties are not barred, on
appeal, from assailing such
jurisdiction, for the same must
exist as a matter of law, and
may not be conferred by
consent of the parties or by
estoppel.

VALDEPENAS vs PEOPLE, 16
SCRA 871
FACTS:
Ester Ulsano fi led with the
justice of peace a criminal
complaint charging Valdepenas
with forcible abduction with
rape of Ester Ulsano. After the
preliminary investigation, the
second
stage
of
which
w a s w a i v e d b y Va l d e p e n a s ,
t h e j u s t i c e o f p e a c e found
that there was probable cause
and forwarded the complaint to
the CFI. CFI found him guilty
as charged and sentenced him
accordingly. On appeal, CA
modified
the
decision,
convicting him of abduction
with consent. Valdepenas filed
Motion for Reconsideration and
new
trial
contesting
the
fi ndings of CA, to the eff ect
that complainant was below
18 y/o at the time of the
occurrence.
Motion
was
granted. The decision was
s e t a s i d e a n d t h e case was
remanded to the CFI.
CFI
rendered
decision reiterating fi ndings
of CA.
Petitioner
again
appealed
to CA which
aff irmed their
decision.
Motion for reconsideration
was fi led on the ground that
lower
court
had
no jurisdiction over the p
erson and the subject

m a t t e r w i t h t h e o ff e n c e o f
abduction
with
consent
but it was denied
Petitioner's
contends
that
theres
no
complaint
for
abduction with consent has
been filed by either Ester
Ulsano or her mother, Consuelo
Ulsano, and that, accordingly,
the lower court acquired no
jurisdiction over his person or
over the crime of abduction
with
consent
and
had,
therefore,
no
authority
to
convict him of said crime
ISSUE:
Whether the Court of
Appeals erred in not reversing
the decision of the trial court,
for lack of jurisdiction over the
person of the accused and the
subject matter of the action for
the offense of abduction with
consent

HELD:
Jurisdiction
over
the
person
of
an
accused
is
acquired
upon
either
his
apprehension, with or without
warrant, or his submission to
the jurisdiction of the court. In
the case at bar, it is not
claimed that petitioner had not
been apprehended or had not
submitted
himself
to
the
jurisdiction
of
the
court.

Indeed,
although
brought
before the bar of justice as
early as January 25, 1956, first,
before the then justice of the
peace court of Piat, then before
the court of first instance of
Cagayan, later before the Court
of Appeals, thereafter back
before said court of first
instance,
and then,
again,
before the Court of Appeals,
never, within the period of six
(6) years that had transpired
until the Court of Appeals,
rendered
its
last
decision, 17 had he questioned
the judicial authority of any of
these three (3) courts over his
person.
He is deemed, therefore, to
have waived whatever objection
he might have had to the
jurisdiction over his person,
and, hence, to have submitted
himself
to
the
Court's
jurisdiction. What is more, his
behaviour and every single one
of the steps taken by him
before said courts particularly
the motions therein filed by
him implied, not merely a
submission to the jurisdiction
thereof, but, also, that he urged
the courts to exercise the
authority
thereof
over
his
person.
GIMENEZ vs NAZARENO, G.R.
No. L-37933 April 15, 1988

FACTS:
Accused Samson Suan, Alex
Potot,
Rogelio
Mula,
Fernando Cargando, Rogelio
Baguio and
the herein
p r i v a t e r e s p o n d e n t Te o d o r
o d e l a Ve g a J r. , w e r e
charged with the crime of
murder.
Before the scheduled date of
the
fi rst
hearing
the
private respondent escape
d f r o m h i s d e t e n t i o n enter
and on the said date, failed to
appear
in
court. This
prompted the
fi scals
handling
the case
(the
petitioners herein) to fi le a
motion with the lower court
to proceed with the hearing of
the case against all the accused
praying that private respondent
de La Vega, Jr. be tried in
absentia
Lower
court
proceeded
with the trial of the case
but
nevertheless gave the priv
ate respondent the
opportunity to take the witness
stand the moment he shows up
in court.
After
due
trial,
or
on
the lower court rendere
d a decision,
dismissing
the case against the fi ve
accused
while
holding
in abeyance
the proceedings against the
private respondent.
Pe t i t i o n e r s fi l e d a M o t i o n

for Reconsideration
questioning the above quoted
dispositive
portion
on
the ground that it will
render nugatory the
constitutional provision on
"trial
in
absentia"
cited
earlier.
However,
this
was
denied by the lower court.
ISSUE:
Whether or not a court
loses
jurisdiction
over
an
accused
who
after
being
arraigned, escapes from the
custody of the law.
HELD:
In
cases
criminal,
jurisdiction over the person of
the accused is acquired either
by his arrest for voluntary
appearance in court. Such
voluntary
appearance
is
accomplished by appearing for
arraignment as what accusedprivate respondent did in this
case.
Jurisdiction once acquired is
not lost upon the instance of
parties but continues until the
case is terminated.
Where the accused appears at
the arraignment and pleads not
guilty to the crime charged,
jurisdiction is acquired by the
court over his person and this
continues until the termination
of the case, notwithstanding his
escape from the custody of the
law

SAGUPAY
vs
COURT
APPEALS, 183 SCRA 464.

OF

FACTS:
Plaintiff Mobil Philippines, Inc.
filed a complaint for replevin
with
damages
against
defendant Lina Joel Sapugay
before CFI Rizal alleging that
upon the termination of the
Dealership Agreement between
Mobil Oil Philippines, Inc. and
Nemar Marketing Corporation,
defendant
applied
to
the
plaintiff to become a dealer of
the
latter's
products
and
pending consideration of the
dealership application plaintiff
loaned to the defendant the
properties installed in the
premises of Nemar at Sto.
Tomas, Batangas, valued at P1,
500,000. And for three (3)
months from the date of
application, defendant failed to
secure and file the required
surety
bond,
compelling
plaintiff to reject defendant's
application and the return and
redelivery
of
the
loaned
properties
but
defendant
refused
to
return
said
equipments, demanded instead

that defendant be paid first the


sum of P15,000 daily as rental
and guard's fees.
Plaintiff prays for the return of
said properties or its value
including damages, attorney's
fees and costs of suit. The
lower court issued an order for
the issuance of a writ of
replevin. Defendant filed her
answer alleging as affirmative
defences.
Trial court rendered judgment
in
favor
of
the
latter,
dismissing the complaint.
Defendant filed a motion for
application to have plaintiffs
bond posted by the Malayan
Surety Company liable for the
satisfaction of the judgment.
Plaintiff-corporation
filed
a
notice of appeal manifesting
that it was appealing to the
Court of Appeals. The trial
court issued an order denying
the
defendant's
motion
considering that the lower
court no longer had any
jurisdiction to act on the
matter with the perfection of
plaintiffs appeal.

ISSUE:
Whether the trial court
acquired jurisdiction over the
person of Cardenas

HELD:
Cardenas did not file a
motion
to
dismiss
the
counterclaim against him on
the
ground
of
lack
of
jurisdiction.
While it is a settled rule that
the issue of jurisdiction may be
raised even for the first time on
appeal, this does not obtain in
the instant case. Although it
was only Mobil which filed an
opposition to the motion to
declare in default, the fact that
the trial court denied said
motion, both as to Mobil and
Cardenas on the ground that
Mobil's complaint should be
considered as the answer to
petioners'
compulsory
counterclaim, leads us to the
inescapable conclusion that the
trial
court
treated
the
opposition as having been filed
in behalf of both Mobil and
Cardenas and that the latter
had adopted as his answer the
allegations
raised
in
the
complaint of Mobil.
By adopting as his answer the
allegations in the complaint
which seeks affirmative relief,
Cardenas is deemed to have
recognized the jurisdiction of
the trial court over his person
and submitted thereto.

DEFENSOR-SANTIAGO
VASQUEZ, 217 SCRA 633.

vs

FACTS:
Miriam Defensor-Santiago
was charged with violation the
Anti-Graft
and
Corrupt
Practices
Act
before
the
Sandiganbayan. An order of
arrest was issued against her
with bail for her release fixed at
P15,
000.00.
She filed
an
"Urgent Ex-parte Motion for
Acceptance
of
Cash
Bail
Bond". The
Sandiganbayan
issued a resolution authorizing
the Santiago to post cash bond.
Her arraignment was set, but
she asked for the cancellation
of her bail bond and that she be
allowed provisional release on
recognizance.
The
Sandiganbayan deferred the
arraignment.
Meanwhile,
it
issued a hold departure order
against Santiago by reason of
the announcement she made,
which was widely publicized in
both
print
and
broadcast
media, that she would be
leaving for the U.S. to accept a
fellowship
at
Harvard
University. She directly filed a
"Motion
to
Restrain
the
Sandiganbayan from Enforcing
its Hold Departure Order with
Prayer for the Issuance of a
Temporary Restraining Order
and/or Preliminary Injunction"

with the SC. She argued that


the
Sandiganbayan
acted
without
or
in
excess
of
jurisdiction and with grave
abuse of discretion in issuing
the
hold
departure
order
considering that it had not
acquired jurisdiction over her
person as she has neither been
arrested nor has she voluntarily
surrendered.
The
hold
departure order was also issued
sua sponte without notice and
hearing.

ISSUE:
Whether
or
not
SADIGANBAYAN
ACQUIRED
JURISDICTION
OVER
THE
PERSON OF SANTIAGO

HELD:
The Court holds that
petitioner is deemed to have
voluntarily submitted herself to
the jurisdiction of respondent
court upon the filing of her
aforequoted
"Urgent Exparte Motion for Acceptance of
Cash Bail Bond for and in
behalf of Dr. Miriam DefensorSantiago"
wherein
she
expressly sought leave "that
she be considered as having
placed
herself
under
the
jurisdiction
of
the
Sandiganbayan for purposes of

the required trial and other


proceedings," and categorically
prayed "that the bail bond she
is posting in the amount of
P15,000.00 be duly accepted"
and that by said motion "she be
considered as having placed
herself under the custody" of
said court.
The voluntary submission of
the accused to the jurisdiction
of the court may be effected by
filing a motion to quash,
appearing
for
arraignment,
participating in the trial or by
giving bail

EPIMACO VELASCO vs CA, 245


SCRA 677.
FACTS:
Warrant of arrest was
issued by Judge Padolina (RTC)
of Pasig, Metro Manila, against
accused Lawrence Larkins for
violations of B.P. Blg. 22.
Desiree Alinea filed before
(NBI)
a
complaint-affidavit
accusing Larkins of the crime
of rape allegedly committed
against
her.
Petitioners
Special
Investigators Resurreccion and.
Erum, Jr. proceeded to the
office of Larkins and arrested
him.

Larkins posted his in estafa


case. Judge Padolina issued an
order recalling and setting
aside the warrant of arrest
issued,
directing
the
Jail
Warden of the NBI Detention
Cell to release Larkins from
confinement "unless otherwise
detained for some other cause."
The 2 Special Investigators
refused to release Larkins
because he was still detained
for another crime which is rape
The complaint was filed with
the RTC of Antipolo.
Larkins, through his counsel
filed an Urgent Motion for Bail
wherein he alleged that the
evidence of guilt against him
for rape is not strong, that he is
entitled as a matter of right to
bail.
Larkins filed in his rape case,
an Urgent Omnibus Motion for
the Dismissal of the Complaint
and for Immediate Release
based on the alleged illegality
of his warrantless arrest.
Trial court denied the aforesaid
motions, Larkins common-law
wife, Felicitas S. Cuyag, filed
before the Court of Appeals a
petition
for habeas
corpus with certiorari.
Court of Appeals ordered the
respondents to appear and
produce Lawrence A. Larkins
before the court and to show
because why Larkins liberty is
being
restrained.

The 2 Special Investigators


appeared and produced Larkins
at the hearing.
OSG did not appear neither did
Judge Caballes, for he had not
received
a
copy
of
the
resolution.
Cuyag,
appeared
and
manifested that should the
court order the release of
Larkins the alternative prayer
for certiorari would be deemed
abandoned.
After hearing the, the Court of
Appeals rendered the release of
larkin
The petitioners insist that the
respondent
court
erred
in
granting
the
petition
for habeas
corpus
because
Larkins
had
already
been
charged with the crime of rape
and the trial court had denied
his application for bail. They
further
claim
that
the
warrantless arrest in this case
is valid.

same legal import and effect as


the posting of bail in cases
where bail is recommended.
It is settled that the giving or
posting of bail by the accused
is tantamount to submission of
his person to the jurisdiction of
the court.
The assertion that the court
never acquired jurisdiction over
the person of the accused
because the warrant of arrest
issued is null and void because
no probable cause was found by
the court issuing it, cannot be
sustained because he posted a
bail. The giving or posting a
bail
is
tantamount
to
submission of his person to the
jurisdiction of the court. Even
if it is conceded that the
warrant issued was void, the
defendant waived all his rights
to object by appearing and
giving a bond

MIRANDA vs TULIAO, G.R. No.


158763, March 31, 2006.
ISSUE:
FACTS:
Whether or not CA has
jurisdiction
over
the
case
despite the nullity of warrant of
arrest issued
HELD:
The filing of a petition or
motion for bail in cases where
no bail is recommended has the

Two
informations
for
murder were filed against the 5
police officer including SPO2
Maderal in RTC of Santiago
City. The venue was later
transferred to Manila.
RTC Manila convicted all the
accused and sentenced them

two
counts
of
reclusion
perpetua except SPO2 Maderal
who was yet to be arraigned at
that time, being at large. Upon
automatic
review,
the
SC
acquitted the four accused on
the ground
of
reasonable
doubt. In Sept. 1999, Maderal
was arrested. He executed a
sworn confession and identified
the herein petitioner Miranda
and 4 others responsible for
the death of the victims.
Respondent Tuliao then filed a
criminal complaint for murder
against the petitioners. Acting
Presiding
Judge
Tumalian
issued warrant of arrest against
the
petitioners
and
SPO2
Maderal. Petitioners filed an
urgent
motion to
complete
preliminary investigation, to
reinvestigate, and to recall or
quash the warrant of arrest.
In the hearing of the urgent
motion, Judge Tumalian noted
the absence of petitioners and
issued a Joint order denying the
said urgent motion on the
ground that since the court did
not
acquire jurisdiction
over their persons, the motion
cannot be properly
heard by the court. The
petitioners appealed
the
resolution
of
the
Public
prosecutor to the DOJ. The new
Presiding Judge named Judge
Ang had took over the case and
issued a Joint Order reversing
the Joint Order of Judge
Tumalian. He also ordered the
cancellation of the warrant of
arrest. Respondent Tulia filed a

petition
for
certiorari,
mandamus and prohibition with
a prayer for TRO seeking to
enjoin Judge Ang had from
further proceeding of the case
and seeking to nullify the Joint
Orders of the said Judge.
The SC issued a resolution
granting
the
prayer.
Notwithstanding
the
said
resolution,
Judge
Anghad
issued a Joint Order dismissing
the information against the
petition.
Respondent
Tuliao
filed a motion to cite Judge Ang
had in contempt. The SC
referred the said motion to the
CA. The CA rendered the
assailed decision granting the
petition
and
ordering
the
reinstatement of the criminal
cases in the RTC of Santiago
City as well as the issuance
of warrant of arrest. Hence,
this petition.
ISSUE:
Can the accused seek
judicial relief if he refused to
submit jurisdiction over his
person to the court
HELD:
In criminal cases, jurisdiction
over the person of the accused
is deemed waived by the
accused when he files any
pleading seeking an affirmative
relief, except in cases when he
invokes the special jurisdiction
of the court by impugning such
jurisdiction over his person. An
accused
can
invoke
the
processes of the court even

though
there
is
neither
jurisdiction over the person nor
custody of the law. However, if
a person invoking the special
jurisdiction of the court applies
for bail, he must first submit
himself to the custody of the
law.
In cases not involving the socalled special appearance, the
general
rule
applies
the
accused is deemed to have
submitted
himself
to
the
jurisdiction of the court upon
seeking
affirmative
relief.
Notwithstanding this, there is
no requirement for him to be in
the custody of the law

CAMANAG vs GUERRERO, 335


Phil. 945, 970-971 [1997]
FACTS:
(PRC) issued the Table of
Results of those who failed in
CPA Licensure Examinations.
On Page 11, petitioner Olivia B.
Camanag was listed failed.
However
petitioner
in
accomplishing
her
Personal
Data sheet as employee of the
(BIR) she indicated that she
passed. An anonymous letter
was sent to PRC Chairman
claiming
that
certain
BIR
employees allegedly passed the

CPA Licensure Exams under


anomalous circumstances
PRC
Chairman
wrote
Ombudsman Vasquez that BIR
employees Marilyn Lee, Connie
Dimapilis, Eilene Purification,
Elenita Villamor, Lodiminda
Crizaldo,
petitioner
Olivia
Camanag and Maria Rosario de
los Reyes, did not actually pass
the
CPA
licensure
examinations.
Deputy Ombudsman for (AFP)
Casaclang
deputized
respondent City Prosecutor of
Manila Guerrero to file the
corresponding charges against
petitioner and to handle the
prosecution of the cases.
At any rate, the preliminary
investigation conducted by the
City
Prosecutor
yielded
additional
evidence
of
falsification against petitioner.
Respondent City Prosecutor
issued
the
questioned
Resolution, finding sufficient
ground to hold petitioner for
trial' and ordering the filing of
the Information in court.
Informations for falsification of
public documents were filed
against petitioner
Before respondent judge could
act on his motion to reduce bail
bond,
petitioner
filed
the
instant petition alleging that

the case should be dismissed


for
lack
of
preliminary
investigation conducted before
filing to the court despite the
fact
that
City
Prosecutor
conducted
a
preliminary
investigation.
ISSUE:
Whether
or
not
the
information filed before the
sala of the judge without the
preliminary investigation by
the
City
Prosecutor
are
characterized by such fatal
defects as to warrant a Writ of
Prohibition to enjoin Judge
from taking any further action
except to order its dismissal.
HELD:
An injunction will not
generally lie to restrain a
criminal action (Paderanga v.
Drilon, 196 SCRA 86 [1991];
Brocka v. Enriel, 192 SCRA 183
[1990]; Crespo v. Mogul, 151
SCRA 462 [1987]). In the
Brocka case, we laid the
following exceptions to the rule
(1) when the injunction is
necessary to afford adequate
protection to the constitutional
rights of the accused; (2) when
it is necessary for the orderly
administration of justice or to
avoid oppression or multiplicity
of actions; (3) when there is a
prejudicial question which is
subjudice; (4) when the acts of
the officer are without or in
excess of authority; (5) where

the prosecution is under an


invalid
law,
ordinance
or
regulation; (6) when double
jeopardy is clearly apparent; (7)
where
the
Court
has
no
jurisdiction over the offense;
(8) where it is a case of
persecution
rather
than
prosecution; (9) where the
charges are manifestly false
and motivated by the lust for
vengeance; and (10) when
there is clearly no prima facie
case against the accused and a
motion to quash on that ground
has been denied."

Metropolitan Bank and Trust


Company
vs. Rogelio
Reynaldo and Jose C. Adrandea,
G.R. No.164538, August 9, 2010
FACTS:
Petitioner Metropolitan Bank
and Trust Company charged
respondents before the Office
of the City Prosecutor of
Manila with estafa.
In the affidavit of petitioner's
audit officer Aguirre, alleged
that
the
special
audit
conducted on the cash and
lending operations of its Port
Area
branch
uncovered
anomalous/fraudulent
transactions
perpetrated
by
respondents in connivance with
client
Universal
Converter
Philippines, Inc.

Respondents
denied
responsibility and claimed that
they only intended to help the
Port Area branch solicit and
increase its deposit accounts
and
daily
transactions.
Meanwhile,
petitioner
and
Universal entered into a Debt
Settlement Agreement whereby
the latter acknowledged its
indebtedness to the former and
undertook to pay the same in
bi-monthly
amortizations
covered by post-dated checks,
"plus balloon payment of the
remaining principal balance
and interest and other charges.
Assistant City Prosecutor found
no
petitioners
evidence
insufficient on the ground that
execution
of
the
Debt
Settlement
Agreement
puts
complainant bank in estoppel
to argue that the liability is
criminal. Since the agreement
was made even before the filing
of this case, the relations
between the parties [have]
change[d], novation has set in
and prevented the incipience of
any criminal liability on the
part of respondents
Petitioner
appealed
the
resolution
to
DOJ
which
affirmed the finding of the City
Prosecutor
Petitioner appealed to Court of
Appeals on the ground that the
DOJ acted with grave abuse of
discretion
and
therefore
mandamus
is
proper.
CA
affirmed the DOJ ruling

ISSUE:
Whether the execution of the
Debt Settlement Agreement
precluded
petitioner
from
holding respondents liable to
stand trial for estafa
HELD:

Mandamus
is
a
remedial
measure for parties aggrieved.
It
shall
issue
when
"any
tribunal, corporation, board,
officer or person unlawfully
neglects the performance of an
act which the law specifically
enjoins as a duty resulting from
an office, trust or station." The
writ of mandamus is not
available to control discretion
neither may it be issued to
compel
the
exercise
of
discretion. Truly, it is a matter
of discretion on the part of the
prosecutor to determine which
persons appear responsible for
the commission of a crime.
However, the moment he finds
one to be so liable it becomes
his inescapable duty to charge
him therewith and to prosecute
him for the same. In such a
situation, the rule loses its
discretionary
character
and
becomes
mandatory.
Thus,
where, as in this case, despite
the sufficiency of the evidence
before
the
prosecutor,
he
refuses
to
file
the

corresponding
information
against the person responsible,
he abuses his discretion. His
act
is
tantamount
to
a
deliberate refusal to perform a
duty enjoined by law.

SERENA vs SANDIGANBAYAN,
G.R. No. 162059, January 22,
2008.
FACTS:
Petitioner Hannah Eunice D.
Serana was a senior student of
the
UP-Cebu.
She was
appointed by then President
Joseph Estrada on December
21, 1999 as a student regent of
UP, to serve a one-year term
starting January 1, 2000 and
ending on December 31, 2000.
On
September
4,
2000,
petitioner,
with her
siblings
and relatives, registered with
the SEC the Office of the
Student Regent Foundation,
Inc. (OSRFI).
One
of
the
projects of the OSRFI was the
renovation
of the
Vinzons
Hall Annex. President Estrada
gave P15, 000,000.00 to the
OSRFI as financial assistance
for the proposed renovation.
The
source
of the
funds,
according to the information,
was the Office of the President.
The renovation of Vinzons Hall
Annex failed to materialize.

The succeeding student regent,


Kristine Clare Bugayong, and
Christine
Jill
De Guzman,
Secretary
General
of
the
KASAMA sa U.P., a system-wide
alliance of student councils
within
the
state university,
consequently filed a complaint
for Malversation
of
Public
Funds
and
Property
with
the Office of the Ombudsman.
The
Ombudsman
found
probable
cause
to
indict
petitioner and her brother Jade
Ian D. Serena for
estafa and filed the case to the
Sandiganbayan.
Petitioner moved to quash the
information. She claimed that
the Sandiganbayan does not
have any jurisdiction over the
offense
charged
or
over
her person, in her capacity as
UP
student
regent.
The
Sandiganbayan
denied
petitioners motion for lack of
merit. Petitioner filed a motion
for reconsideration
but
was
denied with finality.
ISSUE:
Whether
or
not
the
Sandiganbayan has jurisdiction
over the estafa case
HELD:
Evidently,
the
Sandiganbayan has jurisdiction
over other felonies committed
by public officials in relation to
their office. We see no plausible

or
sensible
reason
to
exclude estafaas one of the
offenses included in Section
4(B)
of
P.D.
No.
1606. Plainly, estafa is one of
those other felonies.
The
jurisdiction
is
simply
subject
to
the
twin
requirements
that
(a)
the
offense is committed by public
officials
and
employees
mentioned in Section 4(A) of
P.D. No. 1606, as amended, and
that
(b)
the
offense
is
committed in relation to their
office
Petitioner
falls
under the
jurisdiction
of
the
Sandiganbayan, even if she
does not have a salary grade
27, as she is placed thereby
express provision of law. As the
Sandiganbayan pointed out, the
BOR performs functions similar
to those of a board of trustees
of a non-stock corporation.
By express mandate of law,
petitioner is, indeed, a public
officer as contemplated by P.D.
No. 1606

ANTIPORDA
vs
GARCHITORENA,
G.R.
No.
133289. December 23, 1999

FACTS:
Accused Mayor Licerio
Antiporda and others were
charged for the crime of
kidnapping, the case was filed
in
the
first
division
of
Sandiganbayan. Subsequently,
the
Court
ordered
the
prosecution to submit amended
information,
which
was
complied evenly and the new
information
contained
the
place where the victim was
brought.
The accused filed an Urgent
Omnibus Motion praying that a
reinvestigation be conducted
and the issuance of warrants of
arrest be deferred but it was
denied by the Ombudsman.
The accused.
Thereafter filed a Motion for
New Preliminary investigation
and to hold in abeyance and/or
recall. Warrant of arrest issued
but the same was also denied.
Subsequently, the accused filed
a Motion to Quash Amended
Information
for
lack
of
jurisdiction over the offense
charged, which was ignored for
their continuous refusal to
submit their selves to the Court
and
after
their
voluntary
appearance which invested the
Sandiganbayan
jurisdiction
over their persons, their motion
for reconsideration was again
denied.

ISSUE:
Whether
or
not
Sandiganbayan has jurisdiction
over the case
HELD:
The original Information
filed with the Sandiganbayan
did not mention that the
committed by the accused is
office-related. It was only after
the same was filed that the
prosecution
belatedly
remembered
that
a
jurisdictional fact was omitted
therein. However, we hold that
the petitioners are estopped
from assailing the jurisdiction
of the Sandiganbayan for in the
supplemental
arguments
to
motion
for
reconsideration
and/or
reinvestigation
filed
with the same court, it was they
who
challenged
the
jurisdiction of the Regional
Trial Court over the case and
clearly stated in their Motion
for Reconsideration that the
said crime is work connected.
It is a well-settled rule that a
party
cannot
invoke
the
jurisdiction of a court to secure
affirmative
Relief against his opponent,
and after obtaining or failing to
obtain such relief, repudiate or
question that same jurisdiction.
Sandiganbayan has jurisdiction
over the case because of
estoppel and it was thus vested
with the authority to order the
amendment of the Information.

SERENA vs SANDIGANBAYAN,
G.R. No. 162059, January 22,
2008
FACTS:
Petitioner Hannah Eunice D.
Serena was a senior student of
the
UP-Cebu.
She was
appointed by then President
Joseph Estrada as a student
regent of UP, to serve a oneyear term. Petitioner, with her
siblings
and
relatives,
registered with the SEC the
Office of the Student Regent
Foundation,
Inc. (OSRFI).One
of the projects of the OSRFI
was
the
renovation
of the
Vinzons Hall Annex. President
Estrada gave P15, 000,000.00
to the OSRFI as financial
assistance
for
the proposed
renovation. The source of the
funds,
according
to
the
information, was the Office of
the President. The renovation
of Vinzons Hall Annex failed to
materialize.
The succeeding student regent,
Kristine Clare Bugayong, and
Christine
Jill
De Guzman,
Secretary
General
of
the
KASAMA sa U.P., a system-wide
alliance of student councils
within
the
state university,
consequently filed a complaint
for Malversation
of
Public
Funds
and
Property
with
the Office of the Ombudsman.

The
Ombudsman
found
probable
cause
to
indict
petitioner and her brother Jade
Ian D. Serana for estafa and
filed
the
case
to the
Sandiganbayan.
Petitioner
moved
to
quash
the
information. She claimed that
the Sandiganbayan does not
have any jurisdiction over the
offense
charged
or
over
her person, in her capacity as
UP
student
regent.
The
Sandiganbayan
denied
petitioners motion for lack of
merit. Petitioner filed a motion
for reconsideration
but
was
denied with finality.
ISSUE:
Whether
or
not
the
Sandiganbayan has jurisdiction
over the estafa case

committed in relation to their


office
Petitioner
falls
under the
jurisdiction
of
the
Sandiganbayan, even if she
does not have a salary grade
27, as she is placed thereby
express provision of law.
P.D. No. 1606 explictly vested
the
Sandiganbayan
with
jurisdiction
over Presidents,
directors
or
trustees,
or
managers of
governmentowned
or
controlled
corporations, state universities
or educational institutions or
foundations. Petitioner falls
under this category. As the
Sandiganbayan pointed out, the
BOR performs functions similar
to those of a board of trustees
of a non-stock corporation.

HELD:
Sandiganbayan
has
jurisdiction over other felonies
committed by public officials in
relation to their office. We see
no plausible or sensible reason
to exclude estafa one of the
offenses included in Section
4(B)
of
P.D.
No.
1606. Plainly, estafa is one of
those other felonies.
The
jurisdiction
is
simply
subject
to
the
twin
requirements
that
(a)
the
offense is committed by public
officials and employees and
that
(b)
the
offense
is

PACTOLIN
SANDIGANBAYAN,
G.R.
161455, May 20, 2008

vs
No.

FACTS:
Rodolfo Pactolin, being a
member of the Sangguniang
Panlalawigan
of
Misamis
Occidental,
committed
falsification
of
public
documents in relation to his
office, and taking advantage of

his
official
position
as
Sangguniang
Panlalawigan
Member and head of the
athletic delegation of Misamis
Occidental, requesting from the
city mayor of Ozamis City
financial
assistance,
by
intercalating
thereon
the
printed name of Mario R.
Ferraren,
and
the
latters
position as OIC Mayor while the
City mayor was in travel locally,
and by imitating the latters
signature thereby making it
appear that OIC Mayor Mario
R.
Ferraren
approved
the
request for financial assistance,
when in truth and in fact,
Mario R.
Ferraren
neither
signed the subject letter nor
approved the said request for
financial assistance.
After arraignment in which
Pactolin appeared on his own
behalf and pleaded not guilty,
and after trial on the merits in
which
Pactolin
repeatedly
failed
to
appear,
the
Sandiganbayan issued an order
against Pactolin as guilty.

ISSUE:
Whether
or
not
the
SANDIGANBAYAN
has
jurisdiction
over
the
falsification case of the OIC
Mayor

HELD:

Falsification
of
public
document under the Revised
Penal Code is within the
jurisdiction
of
the
Sandiganbayan

GEDUSPAN vs PEOPLE,
G.R.
No. 158187. February 11, 2005.

FACTS:
An Information was filed
in the Sandiganbayan alleging
that MARILYN C. GEDUSPAN, a
public
officer,
being
the
Regional Manager/Director, of
the
Philippine
Health
Insurance
Corporation,
Regional office in such capacity
and committing the offense in
relation to office, conniving,
confederating
and
mutually
helping with DR. EVANGELINE
C. FARAHMAND, a private
individual and Chairman of the
Board of Directors of Tiong Bi
Medical Center, Tiong Bi, Inc.,
Mandalangan,
Bacolod City,
release the claims for payments
of patients confined at L.N.
Memorial
Hospital
with

Philippine Health Insurance


Corp., prior to January 1, 2000,
(P91,954.64),
Philippine
Currency, to Tiong Bi Medical
Center, Tiong Bi, Inc. despite
clear provision in the Deed of
Conditional
Sale
executed
involving the sale of West
Negros College, Inc. to Tiong
Bi, Inc. or Tiong Bi Medical
Center, that the possession,
operation and management of
the said hospital will be turned
over by West Negros College,
Inc. to Tiong Bi, Inc. effective
January 1, 2000, thus all
collectibles
or
accounts
receivable accruing prior to
January 1, 2000 shall be due to
West Negros College, Inc.

Both accused filed Motion


to
Quash
contending
Sandiganbayans
Lack
of
jurisdiction principally on the
groud that Geduspan is under
Salary grade 26 only
ISSUE:
Whether
or
not
the
Sandiganbayan has jurisdiction
over the case
HELD:
Petitioner admits that she
holds
the
position
of
Department Manager A of
Philhealth.
She,
however,
contends that the position of
Department
Manager
A
is

classified under salary grade 26


and
therefore
outside
the
jurisdiction
of
respondent
court.
She
is
at
present
assigned at the Philhealth
Regional Office VI as Regional
Director/Manager. Petitioner is
a
public
officer,
being
a
department
manager
of
Philhealth,
a
governmentowned
and
controlled
corporation. The position of
manager is one of those
mentioned in paragraph a,
Section 4 of RA 8249 and the
offense for which she was
charged was committed in
relation
to
her
office
as
department
manager
of
Philhealth.
Accordingly,
the
Sandiganbayan has jurisdiction
over her person as well as the
subject matter of the case.
PEOPLE vs SANDIGANBAYAN
and AMANTE, G.R. No. 167304,
August 25, 2009.
FACTS;
Victoria Amante was a member
of the
Sangguniang
Panlungsod of
Toledo
City,
Cebu. She was able to get hold
of a cash advance in the
amount of P71, 095.00under a
disbursement voucher in order
to defray seminar expenses of
the Committee on Health and
Environmental
Protection,
which she headed. After almost
two years since she obtained
the said cash advance, no
liquidation was made. As such,
Toledo City Auditor Manolo V.

Tulibao issued a demand letter


to respondent Amante asking
the
latter
to
settle
her
unliquidated
cash
advance
within seventy-two hours from
receipt of the same demand
letter. The Commission on
Audit,
submitted
an
investigation report to the
Office
of
the
Deputy Ombudsman for Visayas
(OMB-Visayas),
with
the
recommendation
that
respondent Amante be further
investigated
to
ascertain
whether appropriate charges
could be filed against her
under Presidential Decree(P.D.)
No. 1445, otherwise known as
The Auditing Code of the
Philippines.
Thereafter, the OMB-Visayas,
filed
an
Information
for
Malversation of Public Funds
against respondent Amante.
The Office of the Special
Prosecutor (OSP), upon review
of the OMB-Visayas' Resolution,
prepared
a
memorandum
finding probable cause to indict
respondent Amante. The OSP
filed an Information with the
Sandiganbayan
accusing
Victoria Amante of violating
Section89 of P.D. No. 1445. The
case was raffled to the Third
Division of the Sandiganbayan.
Thereafter, Amante filed with
the said court a MOTION TO
DEFER ARRAIGNMENT AND
MOTION
FOR REINVESTIGATION
and
was opposed by The OSP. The
Sandiganbayan,
in
its

Resolution dismissed the case


against Amante.
Hence,
the
present petition
ISSUE:
Does the Sandiganbayan
has jurisdiction over the case
HELD:
In
order
for
the
Sandiganbayan
to
acquire
jurisdiction
over
the
said
offenses, the latter must be
committed by, among others,
officials of the executive branch
occupying positions of regional
director and higher, otherwise
classified as Grade 27 and
higher, of the Compensation
and Position Classification Act
of 1989. However, the law is not
devoid
of
exceptions. Those
that are classified as Grade 26
and below may still fall within
the
jurisdiction
of
the
Sandiganbayan provided that
they hold the positions thus
enumerated
by
the
same
law. Particularly
and
exclusively; city mayors, vicemayors,
members
of
the sangguniang panlungsod
In
connection
therewith,
Section 4(b) of the same law
provides that other offenses or
felonies committed by public
officials
and
employees
mentioned in subsection (a) in
relation to their office also fall
under the jurisdiction of the
Sandiganbayan.

Court had ruled that as long as


the offense charged in the
information
is
intimately
connected with the office and is
alleged
to
have
been
perpetrated while the accused
was in the performance, though
improper or irregular, of his
official functions, there being
no personal motive to commit
the crime and had the accused
not have committed it had he
not held the aforesaid office,
the accused is held to have
been indicted for an offense
committed in relation to his
office

PEOPLE vs SANDIGANBAYAN
and PLAZA, G.R. No. 169004,
G.R. No. 169004.
FACTS:
The
accused,
Rolando Plaza was a member of
the Sangguniang Panlungsod of
Toledo City, Cebu, with a salary
grade 25. He was charged in
the Sandiganbayan for violating
Section 89 of P.D. No. 1445 or
The Auditing Code of the
Philippines. Allegedly, he failed
to liquidate the cash advances
he received by reason of his
office on December 19, 1995 in
the amount of P30, 000. On
April 7, 2005, Plaza filed a
motion to dismiss with the
Sandiganbayan
which
was
found to be with merit.

The
Sandiganbayan
dismissed the case for lack of
jurisdiction over the case. So,
the petitioner filed this case to
the Supreme Court contending
that the Sandiganbayan has
jurisdiction over criminal cases
involving public officials and
employees enumerated under
Section 4 (a) (1) of P.D. 1606,
whether or not occupying a
position classified under salary
grade 27 and above, who are
charged not only for violation
of R.A. 3019, R.A. 1379 or any
of the felonies included in
Chapter II, Section 2, Title VII,
Book II of the Revised Penal
Code, but also for crimes
committed in relation to their
office.
ISSUE:
Whether
or
not
the
SANDIGANBAYAN
has
Jurisdiction over Plaza since he
is only under Salary grade 25
HELD:
Those
public
officials
enumerated in Sec. 4 (a) of P.D.
No. 1606, as amended, may not
only
be
charged
in
the
Sandiganbayan with violations
of R.A. No. 3019, R.A. No. 1379
or Chapter II, Section 2, Title
VII of the Revised Penal Code,
but also with other offenses or
felonies in relation to their
office. The said other offenses
and felonies are broad in scope
but are limited only to those

that are committed in relation


to
the
public
official
or
employee's office.
This Court had ruled that as
long as the offense charged in
the information is intimately
connected with the office and is
alleged
to
have
been
perpetrated while the accused
was in the performance, though
improper or irregular, of his
official functions, there being
no personal motive to commit
the crime and had the accused
not have committed it had he
not held the aforesaid office,
the accused is held to have
been indicted for an offense
committed in relation to his
office. .
SERENA vs SANDIGANBAYAN,
G.R. No. 162059, January 22,
2008.
FACTS:
Petitioner Hannah Eunice D.
Serena was a senior student of
the
UP-Cebu.
She was
appointed by then President
Joseph Estrada on December
21, 1999 as a student regent of
UP, to serve a one-year term
starting January 1, 2000 and
ending on December 31, 2000.
On
September
4,
2000,
petitioner,
with her
siblings
and relatives, registered with
the SEC the Office of the
Student Regent Foundation,
Inc. (OSRFI).One
of
the
projects of the OSRFI was the
renovation
of the
Vinzons
Hall Annex.. President Estrada

gave P15, 000,000.00 to the


OSRFI as financial assistance
for the proposed renovation.
The
source
of the
funds,
according to the information,
was the Office of the President.
The renovation of Vinzons Hall
Annex failed to materialize.
The succeeding student regent,
Kristine Clare Bugayong, and
Christine
Jill
De Guzman,
Secretary
General
of
the
KASAMA sa U.P., a system-wide
alliance of student councils
within
the
state university,
consequently filed a complaint
for Malversation
of
Public
Funds
and
Property
with
the Office of the Ombudsman.
The
Ombudsman
found
probable
cause
to
indict
petitioner and her brother Jade
Ian D. Serana for
estafa and filed the case to the
Sandiganbayan.
Petitioner
moved
to
quash
the
information. She claimed that
the Sandiganbayan does not
have any jurisdiction over the
offense
charged
or
over
her person, in her capacity as
UP
student
regent.
The
Sandiganbayan
denied
petitioners motion for lack of
merit. Petitioner filed a motion
for reconsideration
but
was
denied with finality.
ISSUE:
Whether
or
not
the
Sandiganbayan has jurisdiction
over the estafa case

HELD:
Sandiganbayan
has
jurisdiction over other felonies
committed by public officials in
relation to their office. We see
no plausible or sensible reason
to exclude estafa one of the
offenses included in Section
4(B)
of
P.D.
No.
1606. Plainly, estafa is one of
those other felonies.
The
jurisdiction
is
simply
subject
to
the
twin
requirements
that
(a)
the
offense is committed by public
officials and employees and
that
(b)
the
offense
is
committed in relation to their
office
Petitioner
falls
under the
jurisdiction
of
the
Sandiganbayan, even if she
does not have a salary grade
27, as she is placed thereby
express provision of law.
P.D. No. 1606 explictly vested
the
Sandiganbayan
with
jurisdiction
over Presidents,
directors
or
trustees,
or
managers of
governmentowned
or
controlled
corporations, state universities
or educational institutions or
foundations. Petitioner falls
under this category. As the
Sandiganbayan pointed out, the
BOR performs functions similar
to those of a board of trustees
of a non-stock corporation.

CUNANAN vs ARCEO, G.R. No.


116615 March 1, 1995

FACTS:
The
information
for murder against Cunanan
contained no avermentthat the
offense charged was in relation
to his public office, hence the
courtp r o c e e d e d t o t r i a l a n
d after both parties prese
n t e d e v i d e n c e , t h e c o u r t de
clared that the case must be
refilled to the Sandiganbayan
ISSUE;
Whether
or
not
SANDIGANBAYAN
Jurisdiction over the case

the
has

HELD:
It is firmly settled that
jurisdiction over the offense
charged is a matter that is
conferred by law. 24 Whenever
the above two (2) requisites are
present, jurisdiction over the
offense
is
vested
in
the
Sandiganbayan. This is true
even though the information
originally filed before the RTC
did not aver that the accused
public officer had committed

the offense charged in relation


to his office. In other words,
the
absence
in
the
old
information filed before the
RTC of an allegation that
petitioner
Cunanan
had
committed the offense charged
in relation to his office, is
immaterial
insofar
as
determination of the locus of
jurisdiction
is
concerned.
Indeed, it may be recalled that
the Asuncion ruling involved a
situation where the information
similarly did not contain an
averment that the accused
public officer had committed
the
offense
charged
while
carrying
out
his
official
duties. Public office is not, of
course, an element of the crime
of murder, since murder may be
committed
by
any
person
whether a public officer or a
private citizen. In the present
case,
however,
the
circumstances quoted above
found
by
the
RTC
bring
petitioner
Cunanan's
case
squarely within the meaning of
an
"offense
committed
in
relation to the [accused's]
public office"

MONTILLA vs HILARIO, G.R.


No. L-4922. September 24,
1951.
FACTS:
The
information
charged
defendants
including
Congressman
Crisologo
of

murder, "taking advantage of


their
respective
public
positions conspiring together . .
., did then and there . . .
assault, attack and shoot with
their firearms" several persons
"with the intent to kill" and did
kill one Claudio Ragasa and
inflict physical injuries on three
others.
From the allegations of the
information it does not appear
that the official positions of the
accused were connected with
the offenses charted.
ISSUE:
Whether
or
not
the
Sandiganbayan has jurisdiction
over the case
HELD:
Public office is not of the
essence of murder. The taking
of human life is either murder
or homicide whether done by a
private
citizen
or
public
servant, and the penalty is the
same
except
when
the
perpetrator, being a public
functionary, took advantage of
his office, as alleged in this
case, in which event the penalty
is
increased.
But the use or abuse of office
does not adhere to the crime as
an element; and even as an
aggravating circumstance, its
materiality arises, not from the
allegations but on the proof,
not from the fact that the
criminals are public officials
but from the manner of the

commission of the crime. The


fact that, as alleged, the
defendants
made
use
of
firearms
which
they
were
authorized to carry or possess
by reason of their positions,
could not supply the required
connection between the office
and
the
crime.
Firearms
however and wherever obtained
are not an ingredient of murder
or homicide.

actions
for
recovery
of
unlawfully acquired property
vs. Pres. Marcos etc.
ISSUE:
Whether
or
not
Sandiganbayan has jurisdiction

HELD:
GARCIA vs SANDUGANBAYAN,
G.R. No. 165835, June 22, 2005
FACTS:
Major General Carlos F.
Garcia was the Deputy Chief of
Staff for Comptrollership of the
AFP. In 2004, Ombudsman,
after due investigation, filed a
COMPLAINT against Garcia for
VIOLATION
OF
Code
of
Conduct of Ethical Standards
for
Public
Officials
and
Employees), Art 183, RPC, Civil
Service Law. His wife and 3
sons
were
impleaded
for
violation of RA 1379 insofar as
they acted as conspirators,
conduits, dummies and fronts
of
petitioner
in
receiving,
accumulating,
using
and
disposing of ill-gotten wealth
Garcia filed MTD based on
LACK OF JURISDICTION over
forfeiture proceedings because
it should be filed with RTC and
that
Sandiganbayans
jurisdiction in Civil Actions
pertains
only
to
separate

The
Sandiganbayan
is
vested with jurisdiction over
violations of R.A. No. 1379,
entitled An
Act
Declaring
Forfeiture in Favor of the State
Any Property Found to Have
Been Unlawfully Acquired by
Any Public Officer or Employee
and
providing
for
the
Proceedings
A reading of R.A. No. 1379
establishes that it does not
enumerate any prohibited acts
the commission of which would
necessitate the imposition of a
penalty. Instead, it provides the
procedure for forfeiture to be
followed in case a public officer
or
employee
has
acquired
during his incumbency an
amount of property manifestly
out of proportion to his salary
as such public officer or
employee and to his lawful
income
and
income
from
legitimately acquired property.
Section 12 of the law provides a
penalty but it is only imposed
upon the public officer or
employee who transfers or
conveys the unlawfully acquired

property; it does not penalize


the officer or employee for
making
the
unlawful
acquisition.
Violations of R.A. No. 1379 are
placed under the jurisdiction of
the
Sandiganbayan,
even
though the proceeding is civil
in nature, since the forfeiture
of
the
illegally
acquired
property amounts to a penalty.
The
soundness
of
this
reasoning becomes even more
obvious when we consider that
the
respondent
in
such
forfeiture proceedings is a
public officer or employee and
the violation of R.A. No. 1379
was committed during the
respondent
officer
or
employees incumbency and in
relation to his office

Heirs of Sarah Marie Palma


Burgos vs. Court of Appeals, 612
SCRA 1(2010)
Topic: Purpose of Criminal
Action
Facts:
A number of assailants attacked
the household of Sarah Marie
Palma Burgos while all were
asleep 2 were killed while 2
survived the attack. Four months
after the incident, the police
arrested 2 assailants who executed
confessions, allegedly admitting
their part in the attack. They
pointed to two others who helped
them, and that respondent Co
allegedly is masterminded of the
whole thing. The Regional Trial
Court (RTC) of Manila tried the
case against 2 assailants while the
three
others
remained
at
large. After
trial,
the
RTC
acquitted them both.
After 10 years or on September 5,
2002 respondent Co surrendered
to
the
National
Bureau
of
Investigation. The
prosecution
charged him with two counts of
murder he pleaded not guilty and
applied for application for bail
which was granted for the reason
that the evidence of guilt is not
strong..
Petitioners then prayed for special
civil
action
of certiorari with
prayer for a temporary restraining
order or preliminary injunction

before the Court of Appeals but


was denied for having been filed
without involving the Office of the
Solicitor General (OSG).
Issue:
Whether or not the CA correctly
dismissed the special civil action
of certiorari, which questioned the
RTCs grant of bail to respondent
Co, for having been filed in the
name of the offended parties and
without the OSGs intervention.
Held:
Yes.
The purpose of a criminal action,
in its purest sense, is to determine
the penal liability of the accused
for having outraged the state with
his crime and, if he be found guilty,
to punish him for it. In this sense,
the parties to the action are the
People of the Philippines and the
accused. The offended party is
regarded merely as a witness for
the state.
Actions essentially involving the
interest of the state, if not initiated
by the Solicitor General, are, as a
rule, summarily dismissed.

People vs. Bautista, 522 SCRA


742
Topic: Prescription
Facts:
Private complainant Felipe Goyena
filed with the City Prosecutor
(OCP) a Complaint for slight
physical injuries against Bautista.
After conducting the preliminary
investigation, Prosecutor Jessica
Junsay-Ong
issued
a
recommendation for the filing of
an Information against Bautista.
Such
recommendation
was
approved by the City Prosecutor,
represented by First Assistant City
Prosecutor Eufrocino A. Sulla, but
the date of such approval cannot
be found in the records. The
Information was, however, filed
with the Metropolitan Trial Court
(MeTC) of Manila, Branch 28 only
on June 20, 2000.
Respondent sought the dismissal
of the case against him on the
ground that by the time the
Information was filed, the 60-day
period of prescription from the
date of the commission of the
crime, that is, on June 12, 1999
had already elapsed
Issue:
Whether or not the prescriptive
period began to run anew after the
investigating
prosecutors
recommendation to file the proper
criminal
information
against
respondent was approved by the
City Prosecutor.

Held:
No.
It is a well-settled rule that the
filing of the complaint with the
fiscals office suspends the running
of the prescriptive period; The
prescriptive period remains tolled
from the time the complaint was
filed with the Office of the
Prosecutor until such time that
respondent is either convicted or
acquitted by the proper court.
The proceedings
against
respondent was not terminated
upon
the
City
Prosecutor's
approval of the investigating
prosecutor's recommendation that
an information be filed with the
court. The
prescriptive
period
remains tolled from the time the
complaint was filed with the Office
of the Prosecutor until such time
that respondent is either convicted
or acquitted by the proper court.

People vs. Uba, et al., 106 Phil.


332
Topic:
Preliminary
Investigation
Suspends
the
Prescriptive Period
Facts:
On August 1, 1952, Demetria
Somod-ong filed a complaint in the
Justice of the Peace Court of
Oroquita, Misamis, Occidental,
charging the accused Juliana and
Calixta, both surnamed Uba with
the
crime
of
serious
oral
defamation said to have been
committed against her on or
before July 25, 1952.
Finding probable cause in the
investigation conducted by it, the
court elevated the case to the
Court of First Instance where the
Provincial
Fiscal
filed
the
corresponding
information.
However, by mistake, Pastora
Somod-ong was designated the
offended
party,
instead
of
Demetria. Because of this, the trial
court dismissed the case. The
Provincial Fiscal was ordered by
us to file a new information,
charging the same defendants with
the
offense
of
serious
oral
defamation
committed
against
Demetria Somod-ong.
The Provincial Fiscal on June 12,
1956, filed another complaint
against Juliana Uba and Calixta
Uba, charging them with a crime
of
serious
oral
defamation
committed
against
Demetria
Somod-on. Counsel for the defense

promptly moved for the dismissal


of the complaint on the ground
that the accused would be placed
twice in jeopardy and on the
ground of prescription. The trial
could dismissed the complaint on
September 5, 1957, holding that
under Article 90 of the Revised
Penal Code, the crime of serious
oral defamation, prescribed in six
months; and since the crime was
said to have been committed on or
about July 25, 1952, involving a
period of more than four years,
then the crime charged had
already prescribed.
Issue:
Whether or not the crime of
serious
oral
defamation
had
already prescribed.
Held:
No.
A crime of serious oral
defamation was committed on July
25, 1952. A complaint was filed
therefore on August 1, 1952 in the
Justice of the Peace Court, which,
after finding probable cause in its
investigation, elevated the case to
the Court of First Instance, where
the Provincial Fiscal filed the
corresponding information. The
trial court dismissed the case
when it found out that a person
other than the offended party was
erroneously designated as such.
The Government appealed the
order of dismissal to the Supreme
Court. The latter court on May 18,
1956, affirmed the order of
dismissal but ordered the Fiscal to

file a new information against the


same defendants, which the latter
did on June 12, 1956. Query: Since
the
crime
of
serious
oral
defamation
prescribes
in
six
months, has the crime prescribed
in this case? Held: The filing of the
complaint on August 1, 1952
suspended the running of the
prescriptive period. Said period
commenced to run again from May
18, 1956, the date of the Supreme
Court decision. From that date
until June 12, 1956, when the
second information was filed by
the Fiscal, less than a month had
elapsed. Adding this period to the
seven days which had already run
from the date of the commission of
the crime on July 25, 1952 until
the filing of the first complaint,
there is a total of only about a
month, which is less than the sixmonth prescriptive period. The
crime charged has, therefore, not
prescribed.

People vs.
588(1939)

Aquino,

68

Phil.

Topic:
Preliminary
Investigation
Suspends
the
Prescriptive Period
Facts:
The case involved a crime of
serious
oral
defamation
committed,
according
to
the
information, on September 8,
1935, in the municipality of
Abucay of the Province of Bataan,
against Marcial Kasilag, who was
then Assistant Director of Public
Works and Commissioner for
Mindanao and Sulu. It was
expressly
alleged
in
said
information which, by the way, is a
repetition of the complaint filed by
the offended party on March 4,
1936, that the act complained of
came to the latter's knowledge
only on the last date abovementioned. By reason of the
involuntary
absences
of
the
complainant who had to go to
Mindanao due to the exigencies of
the office which he was then
holding, the trial of the case had to
be suspended and the provisional
dismissal thereof later ordered,
upon motion of the defendant. The
court, however, resolved that said
dismissal
would
be
"without
prejudice" to the fiscal to whom, in
an order of January 21, 1937, it
expressly reserved the right later
to
reproduce
the
same
action. Twenty-three days after the
dismissal of the case, the offended
party Marcial Kasilag again filed in

the justice of the peace court of


Abucay,
Province
of
Bataan,
another complaint charging the
appellee with the same crime with
which he had previously been
charged. In said court the appellee
asked for the dismissal of the case,
alleging, for the first time, the
defense of prescription.
Issue:
Whether or not the crime of
serious
oral
defamation
had
already prescribed.
Held:
No.
The period of prescription fixed
for crimes of oral defamation is six
months (art. 90, Revised Penal
Code), and this period is computed
from the day on which the crime is
discovered by the offended party
(art. 91, id.). In the case at bar,
according to the facts undisputed
by the parties, the offended party
was
informed
of
the
oral
defamation committed against him
for the first time on March 4, 1936.
This being so, the prescriptive
period of six months was not to
expire until about September 4,
1936. When the first complaint
was dismissed by the court on
January 21, 1937, it may be said
that the period of six months did
not even commence to run
because the filing of the
complaint on March 4, 1936,
had the effect of interrupting,
on that very day, the running of
the period of prescription. This

is provided for in article 91 of the


Revised Penal Code. ' The period
of
prescription
in
question
commenced to run only from the
above-mentioned date, January 21,
1937, and only 23 days elapsed
from said date to February 13th of
the same year. It is clear,
therefore, that the appealed order
is erroneous because it makes the
computation from March 4, 1936,
to February 13, 1937, the date of
the filing of the last information,
when such computation should
have been made by it from January
21, 1937, to February 13th of said
year.

People vs.
756(1960)

Olarte,

108

Phil.

Topic: Prescription
Facts:
Asuncion Olarte is charged with
libel by Visitacion Meris after
sending her several letters with
libelous
and
contemptuous
accusations. The letters started in
February 24, 1954. On January 7,
1956, a libel case was filed with
the
provincial
fiscal
and
on February 26 1956,filed the case
with the Justice of Peace Court. On
July 3, 1956, an information was
filed in the Court of First Instance.
The defendant then moved for
the quashal of the info on the
ground that it prescribed already.
The Solicitor General on the other
hand claims that the filing of the
complaint in the Court of Justice of
Peace interrupted the prescription
period of the case.
Issue:
Whether or not the crime of libel
had already prescribed.
Held:
No.
Barely two months prior to the
passage of Act No. 277, the
Philippine
Commission
had
approved Act No. 194, section 1 of
which vested in "every justice of
the peace in the Philippine
Islands" the "authority to make
preliminary investigation of any
crime alleged to have been

committed within his municipality,


jurisdiction to hear and determine
which is by law * * * vested in the
judges of Courts of First Instance."
Act No. 277 did" not particularize
the class of courts that would hear
and determine criminal actions for
libel. The identity of the court was
to be inferred merely from the
penalty prescribed for said offense
in Act No. 277, considered in
relation to Act No. 136. The result
was that, in view of the nature of
said penalty and the provisions of
section 50 of Act No. 136, criminal
cases for libel wereand still are
within the original jurisdiction of
courts of first instance. It cannot,
however, be said that the framers
of section 2 of Act No. 277 had
evinced the intent, either to
establish an exception to the said
provisions of Act No. 194, or to
divest justices of the peace of such
authority, as regards the crime of
libel, because there is absolutely
nothing in Act No. 277 to indicate
such intent. Consequently, the
filing of a complaint for libel
with the justice of the peace
court interrupts the running of
the statute of limitations.

Francisco vs. Court of Appeals,


122 SCRA 538(1983)
Topic: Prescription
Facts:
On
February
6,
1966,
Dr.
Patrocinio Angeles, who was then
the Director of the Morong
Emergency Hospital, filed a case
for
intriguing
against
honor
allegedly committed on December
26, 1965 by Dr. Emiliano and Atty.
Harry Bernardino. On May 3,
1966, the Provincial Fiscal filed an
information against Francisco and
Bernardino with the CFI of Rizal of
the crime of grave oral defamation.
Later, upon order of the court, the
information was amended by
adding
statements
allegedly
uttered by the two accused
constituting the crime of slander.

On Feb. 1, 1973, the trial court


convicted
Francisco
and
Bernardino of the crime of grave
oral defamation and sentenced
each of them the penalty of arresto
mayor and was made to pay the
complainant P10,000. Upon appeal
in the Court of Appeals, the trial
court's decision was modified
finding the accused guilty of
simple slander. Bernardino passed
away while this petition was
instituted in the Supreme Court.
Francisco, then argues that since
the CA had found that the offense
committed was the lesser offense
of simple slander, which under Art.
90 of the RPC, prescribes in two
months, the CA should have
dismissed the case.
The Solicitor General, however,
contends that "for the purpose of
determining
the
proper
prescriptive period, what should
be considered is the nature of the
offense charged in the information
which is grave oral defamation, not
the crime committed by the
accused, as said crime was found
by the Court to constitute only
simple
slander".
Since
the
prescription
for
grave
oral
defamation is six months, the
crime has not yet prescribed when
it the information was filed.
Moreover, the Solicitor General
argues that the filing of the
complaint in the Fiscal's office
interrupts
the
period
of
prescription. Only 39 days had
passed from the time the offense

was allegedly committed to the day


of the filing of the complaint.
Issue:
A. Whether or not the crime of
simple slander found by the
CA to
be the offense
committed by the petitioners
has prescribed.
B. Whether or not the filing of a
complaint in the Fiscal's
office
interrupts
the
prescription of an offense.
Held:
A. Yes.
Where an accused has been
found to have committed a
lesser offense includible within
the offense charged, he cannot
be convicted of the lesser
offense, if it has already
prescribed. To hold otherwise
would
be
to
sanction
the
circumvention of the law on
prescription
by
the
simple
expedient
of
accusing
the
defendant of the graver offense.
B. Yes.
Doctrine in People vs. Olarte that
filing of complaint in the municipal
court, even if merely for purposes
of preliminary examination or
investigation, interrupts the period
of prescription of felony even if
court
where
complaint
or
information is filed cannot try the
case on its merits, the true and
correct doctrine.

As is a well-known fact, like the


proceedings
in
the
court
conducting
a
preliminary
investigation, a proceeding in the
Fiscals Office may terminate
without conviction or acquittal.
Clearly, therefore, the filing of
the denuncia or complaint for
intriguing against honor by the
offended party, later changed
by the Fiscal to grave oral
defamation, even if it were in
the Fiscals Office, 39 days
after the alleged defamatory
remarks were committed (or
discovered) by the accused
interrupts
the
period
of
prescription.

Zaldivia vs. Reyes,


SCRA 277(1992)

Jr.,

211

against a special law/ordinance


interrupts prescription.

Topic: Prescription

Held:

Facts:

No.

Petitioner Zaldivia is charged with


quarrying for commercial purposes
without a mayor's permit in the
municipality
of
Rodriguez,
Province of Rizal. She moved to
quash the information on the
ground that the crime had
prescribed but it was denied. She
appealed to the RTC and denial
was sustained by the respondent
judge. Petitioner filed for a petition
for review on certiorari arguing
that the case filed against her is
govern by the provisions on the
Rules of Summary Procedure. She
contends that criminal cases like
violations of municipal or city
ordinances
does
not
require
preliminary investigation and shall
be filed directly to the court and
not in the Prosecutors office. She
also invoked Act No. 3226 An
Act to Establish Periods of
Prescription
for
Violations
Penalized by Special Acts and
Municipal Ordinances and to
Provide when Prescription Shall
Begin to Run. Concluding that the
case should have been dismissed
since the case against her was
being filed in court way beyond the
2 month statutory period

The mere filing of complaint to the


fiscals office does not interrupt
the running of prescription on
offenses punishable by a special
law.

Issue:
Whether or not the filing of
information/complaint before the
fiscal office constituting a violation

As a general rule, the filing of the


case in the prosecutors office is
sufficient to interrupt the running
of the prescriptive period except
when the case is covered by the
Rules on Summary Procedure. If it
is any crime, you file it in the
fiscals office; the running of the
prescriptive period is interrupted.
But in the case at bar having only
a penalty of arresto menor it
therefore falls under the provisions
of
the
Rules
on
Summary
Procedure. If it is covered by the
Summary
Rules,
the
period
continues. It must be the filing of
the case in court which will
interrupt the period from running.
The running of the prescriptive
period shall be halted on the date
the case is actually filed in court
and not on any date before that.

Sanrio Company Limited


Lim, 546 SCRA 303(2008)

vs.

Topic: Prescription
Facts:
Petitioner
Sanrio
Company
Limited, a Japanese corporation,
owns the copyright of various
animated characters such as
"Hello Kitty. While it is not
engaged in business in the
Philippines, its products are sold
locally by its exclusive distributor,
Gift Gate Incorporated (GGI).
Due to the deluge of counterfeit
Sanrio products, GGI asked IP
Manila
Associates
(IPMA)
to
conduct
a
market
research.
The research's objective was to
identify
those
factories,
department stores and retail
outlets
manufacturing
and/or
selling fake Sanrio items. After
conducting several test-buys in
various commercial areas, IPMA
confirmed
that
respondent's
Orignamura Trading in Tutuban
Center,
Manila
was
selling
imitations of petitioner's products.
Petitioner
filed
a
complaint
affidavit with the Task-Force on
Anti-Intellectual Property Piracy
(TAPP)
of
the
Department
of Justice (DOJ) against respondent
for violation of Section 217 of the
Intellectual Property
Code
(IPC) but the same was dismissed
by the DOJ due to insufficiency of
evidence.
And
affirmed
by
the Office of the Chief State
Prosecutor of the DOJ. Petitioner

filed a petition for certiorari in the


CA.
but the
appellate
court
dismissed the petition on the
ground of prescription. According
to the CA, if no complaint was filed
in court within two years after the
commission
of
the
alleged
violation, the offense had already
prescribed.
Issue:
Whether or not the violation of
Intellectual Property Code had
already prescribed.
Held:
No.
The filing of a complaint for
purposes
of
preliminary
investigation
interrupts
the
period
of
prescription
of
criminal responsibility.Section
2 of Act 3326 provides that the
prescriptive period for violation of
special laws starts on the day such
offense was committed and is
interrupted by the institution of
proceedings against respondent
(i.e., the accused). Petitioner in
this instance filed its complaintaffidavit on April 4, 2002 or one
year, ten months and four days
after
the
NBI
searched
respondents premises and seized
Sanrio merchandise therefrom.
Although no information was
immediately
filed
in
court,
respondents alleged violation had
not yet prescribed. In the recent
case of Brillantes v. Court of
Appeals, 440 SCRA 541 (2004) we
affirmed that the filing of the

complaint
for
purposes
of
preliminary
investigation
interrupts
the
period
of
prescription
of
criminal
responsibility.
Thus,
the
prescriptive
period
for
the
prosecution of the alleged violation
of
the
IPC was
tolled by
petitioners timely filing of the
complaint-affidavit
before
the
TAPP.

Panaguiton, Jr. vs. Department


of Justice, 571 SCRA 549(2008)
Topic: Prescription
Facts:
In 1992, Cawili borrowed money
from Panaguiton amounting to
P1,979,459. In 1993, Cawili with
his business associate Tongson
issued 3 checks as payment.
Significantly, all three (3) checks
bore the signatures of both Cawili
and Tongson. Upon presentment
for payment on 18 March 1993,
the checks were dishonored, either
for insufficiency of funds or by the
closure of the account. During
preliminary investigation, Tongson
claimed that he was not Cawilis
business associate. On Dec. 6 1995
City Prosecutor III Eliodoro V. Lara
found probable cause only against
Cawili and dismissed the charges
against Tongson. Petitioner then
appeal before DOJ and in 1997
DOJ
found
that
it
was
indeed possible for Tongson to cosigned the checks. It then directed
the City Prosecutor of Quezon City
to conduct a reinvestigation of the
case against Tongson and to refer
the questioned signatures to the
National Bureau of Investigation
(NBI) Assistant City Prosecutor
Ma. Lelibet S. Sampaga (ACP
Sampaga) dismissed the complaint
against Tongson without referring
the matter to the NBI per the
Chief State
Prosecutors
resolution. ACP Sampaga held that
the case had already prescribed
pursuant to Act No. 3326, as

amended, which provides that


violations penalized by B.P. Blg. 22
shall prescribe after four (4) years.
Issue:
A. Whether or not the rule on
prescription as provided for
in Act No. 3326 applies to
offenses under B.P. 22.
B. Whether or not the violation
of BP 22 had already been
prescribed.
Held:
A. Yes.
Act
No.
3326,
appropriately
entitled an Act to Establish
Prescription for Violations of
Special
Acts
and
Municipal
Ordinances and to Provide When
Prescription Shall Begin, is the law
applicable
to
offenses
under
special laws which do not provide
their own prescriptive periods
Act. No. 3326 applies to offenses
under B.P. Blg. 22. An offense
under B.P. Blg. 22 merits the
penalty of imprisonment of not less
than thirty (30) days but not more
than one year or by a fine, hence,
under Act No. 3326, a violation of
B.P. Blg. 22 prescribes in four (4)
years from the commission of the
offense or, if the same be not
known at the time, from the
discovery thereof. Nevertheless,
we cannot uphold the position that
only the filing of a case in court
can toll the running of the
prescriptive period.

Gonzalez
vs.
Hongkong
&
Shanghai Banking Corporation,
537 SCRA 255(2007)
TOPIC: Prosecution of Offense
Facts:
The
case
stemmed
from
a
complaint filed by respondent
HSBC against petitioner Gonzalez
for estafa, more particularly, the
violation of Presidential Decree
No. 115, in relation to Art. 315(1)
(b) of the Revised Penal Code.
Petitioner
Gonzalez
was
the
Chairman and Chief Executive
Officer of Mondragon Leisure and
Resorts
Corporation
(MLRC).
MLRC is the owner, developer and
operator of Mimosa Leisure Estate.
Then petitioner Gonzalez, for and
in behalf of MLRC, acknowledged
receipt
of
various
golfing
equipments and assorted Walt
Disney items, and signed the
corresponding two Trust Receipt
agreements. The trust receipt
coming to maturity and without
any word from Gonzalez and no
turnover of sale or any value of the
goods, Felipe for respondent HSBC
filed for estafa. After preliminary
investigation
the
Provincial
Prosecutor found probable cause
against
Gonzalez.
However,
Gonzalez filed for a review under
DOJ but such was denied. Gonzalez
now claimed that there was error
in finding probable cause against
him.
Issue:

Whether
abuse of
DOJ on
probable

or not there was grave


discretion on the part of
finding that there was
cause.

Held:
No. The executive department of
the government is accountable for
the prosecution of crimes; The
right to prosecute vests the
prosecutor with a wide range of
discretion,
the
discretion
of
whether, what and whom to
charge, the exercise of which
depends on factors which are best
appreciated by prosecutors.
The Court consistently adheres to
the policy of non-interference in
the
conduct
of
preliminary
investigations, and to leave to the
investigating prosecutor sufficient
latitude of discretion in the
determination of what constitutes
sufficient evidence as will establish
probable cause for the filing of an
information against the supposed
offender, courts can only review
whether or not the executive
determination of probable cause
was done without or in excess of
jurisdiction resulting from grave
abuse of discretion. Thus, although
it is entirely possible that the
investigating
prosecutor
may
erroneously exercise the discretion
lodged in him by law, this does not
render his act amenable to
correction and annulment by the
extraordinary
remedy
of certiorari, absent any showing
of grave abuse of discretion

amounting
jurisdiction.

to

excess

of

People vs. Fajardo,


SCRA 360(2007)N

Jr.,

512

TOPIC: Prosecution of Offense


FACTS:
The
case
stemmed
from
a
kidnapping for ransom case. A
group of 10 had kidnapped a teen
ager then later on asked for
ransom
from
the
teenagers
parent. After the kidnapping
incident
investigations
were
conducted by PNP later on an
information was then filed against
the 10 assailants. 4 of the accused
were apprehended while the
remaining were still at large. The
trial court upon motion discharged
one of them to serve as state
witness. The trial court found the 3
guilty, and questioned why one of
them was discharged as a state
witness while he acted as a coconspirator as proved by the
testimony of the offended party.
ISSUE:
Whether or not there was a grave
abuse of discretion on the part of
the prosecutor on discharging and
utilizing one of the accused as
state witness.
HELD:
No.
The
power
to
prosecute
includes the initial discretion
to determine who should be
utilized by the government as a
state witness. The prosecution
has gathered the evidence against

the accused and is in a better


position to decide the testimonial
evidence needed by the State to
press
its
prosecution
to
a
successful conclusion. Under our
Rules, however, it is the courts
that will finally determine whether
the
requirements
have
been
satisfied to justify the discharge of
an accused to become a witness
for the government.

Ritualo vs. People, 591 SCRA


24(2009)
TOPIC: Prosecution of Offense
FACTS:
The case originated from two
Informations, both dated 2 January
2001, which charged Ritualo with
the crimes of Illegal Recruitment
defined and penalized by Republic
Act No. 8042; and Estafa under
Art. 315, par. 2(a) of the Revised
Penal Code. RTC convicted her of
illegal recruitment which was
affirmed by the Court of Appeals
on the grounds that Ritualo's "acts
of
promising
and
assuring
employment overseas to [Biacora]
[fell] squarely within the ambit of
recruitment and placement as
defined by [The Migrant Workers
Act or Republic Act No. 8042].
She argues that there "was no
proof beyond reasonable doubt
that [she] gave Biacora a distinct
impression that she had the power
or ability to send him abroad for
work such that the latter was
convinced to part with his money.
Ritualo insisted that it was merely
to facilitate the latter's application
for an Australian Visa. Particularly,
she
pointed
out
that
the
prosecution failed to present other
witnesses
who
could
have
corroborated the claim of Biacora
that she (Ritualo) promised him
employment abroad.
ISSUE:
Whether or not the RTC and the
Court of Appeals affirmed her

conviction despite failure of the


prosecution to present other vital
witness.
HELD:
No. The prosecution is entitled to
conduct its own case and to decide
what witnesses to call to support
its
charges.[48]
The
defense
posture that the non-presentation
of the wife of Biacora constitutes
suppression of evidence favorable
to petitioner Ritualo is fallacious.
In fact, the same line of reasoning
can be used against petitioner
Ritualo. If the defense felt that the
testimony of Biacora's wife would
support her defense, what she
could and should have done was to
call her (Biacora's wife) to the
stand as her own witness. One of
the constitutional rights of the
accused is "to have compulsory
process to secure the attendance
of witnesses and the production of
evidence in his behalf." And, in the
same vein, since petitioner Ritualo
is setting the cloak of liability on
Seraspe's shoulder, she (petitioner
Ritualo) could and should have had
the former subpoenaed as well.
As held by this Court, the adverse
presumption of suppression of
evidence does not, moreover, apply
where the evidence suppressed is
merely
corroborative
or
cumulative in nature. If presented,
Biacora's
wife
would
merely
corroborate
Biacora's
account
which, by itself, already detailed
what occurred on the day of the

parties' first meeting at the house


of petitioner Ritualo. Hence, the
prosecution committed no fatal
error in dispensing with the
testimony of Biacora's wife.

Tan vs. People,


139(2009)

586

SCRA

TOPIC: Prosecution of Offense


FACTS:
A Panel of Prosecutors of the
Department of Justice (DOJ), on
behalf of the People of the
Philippines (People), filed three
Informations against Dante T. Tan
(petitioner) before the Regional
Trial Court (RTC) of Pasig City.
Criminal
Case
No.
119830
v pertains to allegations that
petitioner employed manipulative
devises in the purchase of Best
World Resources Corporation (BW)
shares.2. Criminal Cases No.
119831 and No. 119832 involve
the alleged failure of petitioner to
file with the Securities and
Exchange Commission (SEC) a
sworn statement of his beneficial
ownership of BW shares. DOJ Chief
Prosecutor filed a motion for
consolidation.
Petitioner
was
arraigned on 16 January 2001, and
pleaded not guilty to the charges
Petitioner
moved
to
dismiss
Criminal Case 119830 due to
failure to prosecute for an
unreasonable length of time. He
was claiming for his right to
speedy trial. The prosecution
opposed the Motion, insisting on
its claim that the parties had an
earlier agreement to defer the trial
of Criminal Case No. 119830 until
after that of Criminal Cases No.
119831-119832,
as
the
presentation of evidence and

prosecution in each of the five


cases involved were to be done
separately. RTC
ordered
the
dismissal of Criminal Case 119830.
Hence Appeal to the CA. CA
reinstated Criminal Case 119830
RTC
to
conduct
further
proceeding. Petitioner moved for a
reconsideration and contends that
the
certificate
of
non-forum
shopping attached to the Peoples
appeal before the Court of Appeals
should have been signed by the
Chairman
of
the
SEC
as
complainant in the cases instead of
Acting DOJ Secretary Merceditas
N. Gutierrez.

ISSUE:
Whether or not the certificate of
non-forum shopping attached to
the Peoples appeal before the
Court of Appeals should have been
signed by the Chairman of the SEC
as complainant in the cases
instead of Acting DOJ.
HELD:
No.
Section 2, Rule 110 of the Rules of
Court leaves no room for doubt
and establishes that criminal cases
are prosecuted in the name of the
People of the Philippines, the
offended party in criminal cases.
Moreover, pursuant to Section 3,
paragraph (2) of the Revised
Administrative Code, the DOJ is
the
executive
arm
of
the
government
mandated
to

investigate the commission of


crimes, prosecute offenders and
administer the probation and
correction system. It is the DOJ,
through its prosecutors, which is
authorized to prosecute criminal
cases on behalf of the People of
the Philippines.

People vs. Dumlao, 580 SCRA


409(2009)
TOPIC: Prescription of Offense
FACTS:
An information was filed before the
Sandiganbayan
charging
respondents Dumlao, Lao and
others with violation of the Section
3(g) of R.A. no. 3019 or Anti-Graft
and Corrupt Practices Act. The
information
alleged
that
the
respondent-members of the Board
of Trustees of GSIS entered into a
contract of lease-purchase with
respondent Lao, a private person
whereby GSIS agreed to sell to
Lao, a GSIS-acquired property
consisting of a land and building
known as the Government Counsel
Centre for P2 Million on an
instalment basis with annual
interest and amortization and
grant Lao the right to sub-lease
the ground floor during the period
of lease, from which he collected
yearly rentals in excess of the
yearly amortization causing gross
disadvantage to the government.
During
arraignment,
Dumlao
pleaded not guilty, and as agreed
by prosecution and respondents, a
Joint Stipulation of Facts and
Admission
of
Exhibits
was
submitted to the court. The Joint
Stipulation admitted additional
facts: (1) 3 members of the Board,
Dumlao being one of them, signed
the Minutes; (2) 7 members of the
Board were present during the
board meeting; and (3) the

documentary evidence of was


authentic and duly executed. It
was further decided for the pretrial to be terminated limiting the
course of the subsequent trial to
matters not disposed of... unless
modified by the court. Dumlao
filed a Motion to Dismiss/Quash on
the ground that the facts charged
do not constitute an offense. He
stated that the prosecutions main
thrust against him was the alleged
approval by the GSIS Board of the
Lease-Purchase Agreement. He
argued that the Resolution was not
in fact approved by the GSIS
Board.
Since the signatures of
fellow respondents did not appear
in the minutes of the meeting,
these people did not participate in
the Lease-Purchase Agreement.
There was no quorum of the board;
thus no resolution approving the
Agreement. Since the resolution
was
not
approved,
he
was
innocent. He added that the
person liable was Atty. Javellana
who
actually
executed
the
contract.
Respondent
Dumlao
submits that his prosecution, to
the exclusion of others, constitutes
unfair discrimination and violates
his constitutional right to equal
protection of the law. He says that
the dismissal of the case against
his co-accused Canlas and Clave
were
not
appealed
by
the
prosecution;
and
the
two
government officials who signed
the Lease-Purchase Agreement,
and the two other members
(Ocampo and Morales) of the GSIS

Board of Trustees who signed the


minutes were not charged.
ISSUE:
Whether or not prosecution, to the
exclusion of others, constitutes
unfair discrimination and violates
his constitutional right to equal
protection of the law.
HELD:

No.
The
manner
in
which
the
prosecution of the case is handled
is within the sound discretion of
the prosecutor, and the noninclusion of other guilty persons is
irrelevant to the case against the
accused;
Mere
speculation,
unsupported
by
convincing
evidence,
cannot
establish
discrimination on the part of the
prosecution and the denial to the
accused of the equal protection of
the laws.

Ong vs. Genio,


188(2009)

609

SCRA

TOPIC: Prosecution of Offense


FACTS:
Petitioner
Elvira
O.
Ong
(petitioner)
filed
a
criminal
complaint against respondent Jose
Casim Genio (respondent) for
Robbery which was dismissed by
the City Prosecutor of Makati City.
However,
pursuant
to
the
Resolutions dated September 15,
2006 and October 30, 2006 of the
Department of Justice, respondent
was charged with the crime of
Robbery in an Information. The
accused allegedly had stolen
kitchen equipments and money of
the petitioner. RTC dismissed the
case for failure to prove the
existence of the elements of
robbery. Despite the dismissal of
the case, respondent filed a Partial
Motion for Reconsideration which
was granted dismissing the whole
case for failure to prove probable
cause. Aggrieved, petitioner filed a
Petition
for
Certiorari
and
Mandamus before the CA on
August 28, 2007. Respondent filed
a Motion to Dismiss the petition,
raising the issue of lack of
personality of petitioner to appeal
the dismissal of the criminal case,
because the authority to do so lies
exclusively with the State as
represented by the Office of the
Solicitor General (OSG).
ISSUE:

Whether or not the petitioner as


the private offended party has
personality to elevate the case to
the Court of Appeals without the
conformity of the Office of the
Solicitor General.
HELD:
No.
It is well-settled that in criminal
cases where the offended party is
the State, the interest of the
private complainant or the private
offended party is limited to the
civil
liability.
Thus,
in
the
prosecution of the offense, the
complainant's role is limited to
that of a witness for the
prosecution. If a criminal case is
dismissed by the trial court or if
there is an acquittal, an appeal
therefrom on the criminal aspect
may be undertaken only by the
State
through
the
Solicitor
General. Only the Solicitor General
may represent the People of the
Philippines on appeal. The private
offended party or complainant may
not take such appeal. However, the
said offended party or complainant
may appeal the civil aspect despite
the acquittal of the accused

Estudillo v. Baloma, 426 SCRA


83
TOPIC: Prosecution of Offense
FACTS:
Jovelyn Estudillo (Jovelyn) assisted
by her mother, Visitacion L.
Estodillo, charges Judge Teofilo D.
Baluma
with
Gross
and
Inexcusable Ignorance of the Law.
Complainant alleges that her
administrative complaint arose
from the dismissal of Criminal
Case for Other Acts of Child
Abuse entitled
People
of
the
Philippines, Plaintiff vs. Fredie
Cirilo Nocos y Urot by respondent
Judge of the Regional Trial Court
of Bohol, Branch 1, a Family Court.
The criminal case was originally
filed for preliminary investigation
with the 2nd Municipal Circuit Trial
Court
of
Tubigon-Clarin,
Bohol. After
the
requisite
preliminary investigation, Judge
James Stewart E. Himalaloan
found that there was sufficient
ground to hold the herein accused
for trial for the offense of Other
Acts of Child Abuse defined in Sec.
10 (1), Article VI of Republic Act
No. 7610. The record of the case
was transmitted to the Office of
the Provincial Prosecutor where,
after a review by Third Assistant
Provincial Prosecutor, Macario I.
Delusa, he failed an Information
dated October 28, 2002. Petitioner
claims there was gross ignorance
of law for failure of the prosecutor

to subscribed and sworn the


information.
The
prosecution
through Prosecutor Delusa filed a
Motion for Reconsideration and
Revival on December 12, 2002
alleging that there was no
necessity for the Information to be
under oath since he merely
concurred with the resolution of
the investigating judge and that he
has
properly
subscribed and
signed the Information with the
approval
of
the
Provincial
Prosecutor.
ISSUE:
Whether or not there is a
requirement that the Information
be sworn by the Prosecutor.
HELD:
No. There is no requirement that
the information be sworn to. An
information requires no oath. Sec.
4 of Rule 110 merely requires that
it be an accusation in writing.
This is because the prosecutor
filing the information is acting
under oath of his office.

Cudia vs. Court of Appeals, 284


SCRA 173
TOPIC: Prosecution of Offense
FACTS:
Renato Cudia was arrested on June
28, 1989 in Mabalacat, Pampanga
for the crime of Illegal Possession
of Firearms and Ammunition. He
was brought to Sto. Domingo,
Angeles City which a preliminary
investigation was conducted and
as a result the City Prosecutor
filed an information against him.
The case against him was raffled
to Branch 60 of the Regional Trial
Court of Angeles City. Upon his
arraignment the court called the
attention of the parties and
contrary
to
the
information,
Renatio Cudia had committed the
offense in Mabalacat and not in
Angeles City. Thus the judge
ordered that the case should be
assigned to a court involving
crimes committed outside Angeles
City consequently it was assigned
to Branch 56 of the Angeles City
RTC. However, the Provincial
Prosecutor of Pampanga filed an
information charging Renato Cudio
with the same crime and it was
likewise assigned to Branch 56 of
the Angeles City RTC which
resulted into two Information filed
with
the
same
crime.
This
prompted the City Prosecutor to
file a Motion to Dismiss/ Withdraw
the Information which the trial
court granted. Renato filed a
Motion to Quash the criminal case
filed by the Provincial Prosecutor

on the ground that his continued


prosecution for the offense of
illegal possession of firearms and
ammunition for which he had been
arraigned in the first criminal
case,
and
which
had
been
dismissed despite his opposition
would violate his right not to be
put
twice
in
jeopardy
of
punishment for the same offense.
ISSUE:
Whether or not the Court of
Appeals erred when it found that
the City Prosecutor of Angeles City
did not have the authority to file
the first information.
HELD:
No. An information, when required
to be filed by a public prosecuting
officer, cannot be filed by another.
It
is
thus
the
Provincial
Prosecutor of Pampanga, not the
City
Prosecutor,
who
should
prepare informations for offenses
committed within Pampanga but
outside of Angeles City. An
information, when required to be
filed by a public prosecuting
officer, cannot be filed by another.
It must be exhibited or presented
by the prosecuting attorney or
someone authorized by law. If not,
the court does not acquire
jurisdiction.
An infirmity in the information,
such as lack of authority of the
officer signing it, cannot be cured
by silence, acquiescence, or even
by express consent.

B. No. The prescriptive period


is
interrupted
by
the
institution of proceedings for
preliminary
investigation
against the accused.
Petitioners filing of his complaintaffidavit before the Office of the
City Prosecutor on 24 August 1995
signified the commencement of the
proceedings for the prosecution of
the accused and thus effectively
interrupted the prescriptive period
for the offenses they had been
charged under B.P. Blg. 22.

Ingco vs. Sandiganbayan, 272


SCRA 563(1997)

Section 3(e), in relation to Section


3(g), of R.A. 3019.

Topic: Prescription

Petitioners
moved
for
the
quashal of the information on the
ground, inter alia, that the facts
alleged in the information did not
constitute an offense under the
invoked law, and that the offense
charged, in any case, had already
prescribed.

Facts:
Domingo Ingco, a former VicePresident
of
the
Philippine
National
Bank
("PNB"),
was
charged, along with top officials of
Cresta
Monte
Shipping
Corporation, namely, its Chairman
of the Board of Directors Ernesto
Magboo
and
its
President
Herminio Alcasid, by PNB before
the Presidential Blue Ribbon
Committee
with
violation
of
Republic Act No. 3019 ("Anti-Graft
and Corrupt Practices Act"). The
matter was at once referred to the
Office of the Ombudsman. PNB
charges Domingo Ingco with
conspiring
with
the
other
respondents in having the loan
applications
approved
even
without a project feasibility study
and notwithstanding the fact that
the credit rating submitted by the
Credit Department showed more
adverse
comments. It
further
alleged that the collaterals offered
by
Cresta
Monte
were
deficient. PNB likewise charged
the officers and directors of Cresta
Monte
with
persuading
and
inducing respondent Ingco to
recommend the approval of the
loans
under
disadvantageous
terms and conditions. Accordingly,
on
21
July
1993,
an
information was filed with the
Sandiganbayan for violation of

Issue:
Whether or not the violations of RA
3019 had already prescribed.
Held:
No.
The complaint filed on 26 May
1987 before the Ombudsman, in
fine, is deemed to have tolled the
running of the prescriptive period,
and thus the filing of the
information on 21 July 1993,
following the approval by the
Ombudsman on 12 July 1993 of the
resolution
recommending
the
prosecution of herein petitioners,
must perforce be held to be well
within the ten-year prescriptive
period.

Securities
and
Exchange
Commission
vs.
Interport
Resources
Corporation,
567
SCRA 354(2008)
Topic: Prescription
Facts:
The Board of Directors of IRC
approved
a
Memorandum
of
Agreement
with
GHB(Ganda
Holdings Berhad). Under said
memorandum of agreement, IRC
acquired 100% of the entire capital
stock of GEHI (Ganda Energy
Holdings Inc.) Which would own
and operate a 102 megawatt gas
turbine power generating barge.
In exchange, IRC will issue to GHB
55% of the expanded capital stock
of IRC. On the side, IRC would
acquire 67% of the entire capital
of PRCI (Philippine Racing Club).
It is alleged herein that a press
release announcing the approval of
the agreement was sent to the
Philippine Stock Exchange through
facsimile and the SEC, but the
facsimile machine of the SEC could
not receive it. However, the SEC
received reports that the IRC
failed to make timely public
disclosures of its negotiations with
GHB and that some of its
directors, heavily traded IRC
shares utilizing this material
insider information.
SEC issued an order finding that
the IRC violated the Rules in
connection with the then Old
Securities Act when it failed to
maketimely disclosures of its

negotiations
with
GHB.
In
addition, the SEC found that the
directors of IRC entered into
transactions involving IRC shares
in
violation
of
the
Revised
Securities Act.
Respondents however have taken
the position that this case is moot
and academic, since any criminal
complaint that may be filed against
them resulting from the SECs
investigation of this case has
already prescribed. They point out
that
the
prescription
period
applicable to offenses punished
under special laws, such as
violations of the Revised Securities
Act, is twelve years under Section
1 of Act No. 3326, as amended by
Act No. 3585 and Act No. 3763,
entitled An Act to Establish
Periods
of
Prescription
for
Violations Penalized by Special
Acts and Municipal Ordinances
and to Provide When Prescription
Shall Begin to Act. Since the
offense was committed in 1994,
they reasoned that prescription set
in as early as 2006 and rendered
this case moot.
Issue:
Whether or not the violation of the
Revised Securities Act and the
Securities Regulations Code had
already prescribed.
Held:
No. The prosecution of offenses
punishable under the Revised
Securities Act and the Securities
Regulations Code is initiated by

the filing of a complaint with the


SEC or by an investigation
conducted by the SEC motu
proprio. Only after a finding of
probable cause is made by the SEC
can the DOJ instigate a preliminary
investigation.
Thus,
the
investigation that was commenced
by the SEC in 1995, soon after it
discovered the questionable acts of
the
respondents,
effectively
interrupted
the
prescription
period. Given the nature and
purpose
of
the
investigation
conducted by the SEC, which is
equivalent to the preliminary
investigation conducted by the DOJ
in
criminal
cases,
such
investigation
would
surely
interrupt the prescription period.

Uy vs. People,
542(2008)

564

SCRA

Topic: WHO MUST PROSECUTE


THE CRIMINAL ACTION
Facts:
Petitioner was charged before the
RTC
of
Makati
City.
The
prosecution has established the
presence of all the elements of the
offense. Petitioner
falsely
represented to private complainant
that he had an on going low-cost
housing
project
in
Agusan,
Cagayan
de
Oro. Relying
on
petitioners
fraudulent
misrepresentations,
private
complainant
invested P3,500,000.00
in
said
project. Said amount was given by
means of a check and handed over
to petitioner simultaneously with
the signing of the Investment
Agreement. As it turned out, per
certification from the HLURB,
petitioner did not have any lowcost housing project in Agusan,
Cagayan
de
Oro. Private
complainant
indeed
suffered
damage. He did not get his return
of investment because the check
he received from petitioner in the
amount
of P4,500,000.00
was
dishonored. Moreover,
petitioner
neither paid private complainant
the 6% compounded interest on
said amount or balance thereon,
nor
did
he
allow
private
complainant to acquire a portion
or portions of the low-cost housing
subdivision in lieu of the payment
of
any
unpaid
amount
or

balance. To date, the amount


private complainant invested in
said low-cost housing has not been
returned. Without
a
doubt,
petitioner is guilty of estafa.
Petitioner however contends he
was denied due process of law
when
he
was
charged and
convicted of estafa instead of
violation of Batas Pambansa Blg.
22.
Issue:
Whether or not the petitioner was
denied due process of law when he
was charged of estafa instead of
violation of Batas Pambansa Blg.
22.
Held:
No.
Under Section 5, Rule 110 of the
Revised
Rules
of
Criminal
Procedure, criminal actions shall
be prosecuted under the direction
and control of the prosecutor. In
the case before us, the prosecutor,
after going over the complaint
found probable cause to charge
him with estafa. This was the
prosecutors
prerogative,
considering that he was the one
who would prosecute the case. The
prosecuting attorney cannot be
compelled to file a particular
criminal information. The fact that
the demand letter may suggest a
violation of Batas Pambansa Blg.
22 cannot control his action as to
what charge he will file, if he sees
evidence showing probable cause

to charge an accused for another


crime. It is the prosecutors
assessment of the evidence before
him which will prevail, and not
what is contained in a demand
letter

Ricarze vs. Court of Appeals,


515 SCRA 302(2007)
Topic: WHO MUST PROSECUTE
THE CRIMINAL ACTION
Facts:
Petitioner was
employed
as a collector-messenger by City
Service
Corporation, adomestic
corporation
engaged
in
messengerial services. He was
assigned to the main officeof
Caltex Philippines, Inc. (Caltex) in
Makati. His primary task was to
collect checks payable o Caltex
and deliver them to the cashier. He
also delivered invoices to Caltexs
customers.Caltex filed a criminal c
omplaint against petitioner for esta
fa through falsification of commerc
ial documents. The manager of its
Banking
and
Insurance
Department, Romano, alleged that
while
his
department
was
conducting a daily electronic
report from PCI Bank, one of its
depositary
banks,
it
was
discovered that unknown to the
department,
a
company
ycheck in the
amount
of P5,790,570.25
payable to Dante R.
Gutierrez,

had been cleared through PCIB.


An investigation also revealed that
two other checks were also
missing and that his signature and
that
of
another
signatory,
Goquinco, were forgeries. A check,
in the amount of P1,790,757.25
likewise payable to Dante R.
Gutierrez,
was
also
cleared
throught he same bank. This check
was likewise not issued by Caltex,
and the signatures appearing
thereon had also been forged.
Upon
verification,
it
was
uncovered that the checks were
deposited at the BDO in the name
of a regular customer of Caltex,
Gutierrez.
Gutierrez,
however,
disowned the savings account as
well as his signatures on the dorsal
portions thereof. He also denied
having withdrawn any amount
from said savings account. Further
investigation
revealed
that
said savings account had actually
been opened by petitioner; the
forged checks were deposited and
endorsed by him under Gutierrezs
name. A bank teller from the BDO
positively identified petitioner as
the person who opened the savings
account using Gutierrezs name.
Petitioner further averred that
unless the Informations were
amended to change the private
complainant to PCIB, his right as
accused would be prejudiced. He
pointed out, however, that the
Informations can no longer be
amended because he had already
been
arraigned under the original Infor

mations. He insisted that the amen


dments of the
Informations
to
substitute PCIB as the offended
party for Caltex would place him in
double jeopardy
Issue:
Whether or not the substitution of
PCIB as the offended party for
Caltex is valid.
Held:
Yes. Under Section 5, Rule 110 of
the Revised Rules of Criminal
Procedure, all criminal actions
covered
by
a
complaint
or
information shall be prosecuted
under the direct supervision and
control of the public prosecutor.
Thus, even if the felonies or
delictual acts of the accused result
in damage or injury to another, the
civil action for the recovery of civil
liability based on the said criminal
acts is impliedly instituted, and the
offended party has not waived the
civil action, reserved the right to
institute it separately or instituted
the civil action prior to the
criminal action, the prosecution of
the action (including the civil)
remains under the control and
supervision
of
the
public
prosecutor. The prosecution of
offenses is a public function.
Under Section 16, Rule 110 of the
Rules of Criminal Procedure, the
offended party may intervene in
the criminal action personally or
by counsel, who will act as private
prosecutor for the protection of his
interests and in the interest of the

speedy
and
inexpensive
administration of justice.
A
separate action for the purpose
would only prove to be costly,
burdensome and time-consuming
for both parties and further delay
the final disposition of the case.
The multiplicity of suits must be
avoided.
With
the
implied
institution of the civil action in the
criminal action, the two actions
are merged into one composite
proceeding, with the criminal
action predominating the civil. The
prime purpose of the criminal
action is to punish the offender in
order to deter him and others from
committing the same or similar
offense, to isolate him from society,
reform and rehabilitate him or, in
general, to maintain social order.
The sole purpose of the civil action
is for the resolution, reparation or
indemnification of the private
offended party for the damage or
injury he sustained by reason of
the delictual or felonious act of the
accused.

Adasa vs. Abalos, 516 SCRA


261(2007)
Topic: WHO MUST PROSECUTE
THE CRIMINAL ACTION
Facts:
Respondent Cecille Abalos alleged
in the complaints and affidavits
that petitioner Bernadette Adasa
was encashed two checks issued in
the name of the respondent
through deceit without knowledge
of respondent Abalos. Adasa failed
to pay to the proceeds of the
checks despite demands of Abalos.
Adasa filed a counter-affidavit
admitting that she received and
encashed the checks and alleged
further in a supplemental affidavit
that Bebie Correa instead received
the 2 checks and that she left the
country. The Office of the City
Prosecutor (OCP) of Iligan City
issued
a
resolution
finding
probable cause against Adasa and
ordered for filing of two separate
informations for Estafa through
falsification
of
commercial
document by a private individual.
Adasa filed a motion upon the trial
court in order for the OCP to
conduct a reinvestigation, in which
the OCP has reaffirmed its finding
of probable cause. Adasa has
entered a not guilty plea during
her arrangement on October 1,
2001 and later filed a petition for
review before the DOJ where it
reversed and set aside the
resolution of the OCP and ordering
it to withdraw the information for
estafa.

The trial court has granted the


petitioners motion to withdraw
information and dismissed the
criminal case, on February 2003.
Issue:
Whether or not the case be
dismissed after a review of DOJ
finding no probable cause even
after
the
court
acquired
jurisdiction over the case.
Held:
No.
It is a cardinal principle that all
criminal actions either commenced
by complaint or by information
shall be prosecuted under the
direction and control of the fiscal;
While it is true that the fiscal has
the quasi-judicial discretion to
determine whether or not a
criminal case should be filed or
not, once the case had already
been
brought
to
court,
whatever disposition the fiscal
may deem proper should be
addressed to the court.

Chua vs.
60(2007)

Padillo,

522

SCRA

Topic: WHO MUST PROSECUTE


THE CRIMINAL ACTION
Facts:
Rodrigo Padillo and Marietta
Padillo, respondents, are the
owners of Padillo Lending Investor
engaged in the money lending
business. Their niece, Marissa
Padillo-Chua, served as the firms
manager. Marissa is married to
Wilson Chua, brother of Renita
Chua,
herein
petitioners.
Sometime in September 1999, a
post-audit was conducted. It was
found that Marissa was engaged in
illegal activities. Some of the
borrowers whose loan applications
she recommended for approval
were fictitious and their signatures
on the checks were spurious, the
cash amounts received were
turned over to Marissa or her
husband Wilson for deposit in their
personal accounts. To facilitate
encashment, Marissa would sign
the check to signify to the bank
that she personally knew the
alternative
payee.
The
total
amount embezzled reached P7
million.
Respondents
filed
complaints against petitioners with
the
National
Bureau
of
Investigation (NBI). Forthwith, the
City
Prosecutor
filed
an
Information
for estafa against
Marissa, Wilson, and Renita Chua.
Believing that a more serious
offense should have been charged
against petitioners, respondents

interposed an appeal to the


Secretary of Justice. The Secretary
of
Justice
found
that
the
participation of Wilson Chua in the
commission of the crime was not
clearly
established
by
the
evidence. As to Renita Chua, the
Secretary of Justice found no proof
of conspiracy between her and
Marissa. Respondents filed a
motion for reconsideration, but it
was denied with finality by the
Secretary of Justice. Respondents
then filed a Petition for Certiorari
with the Court of Appeals. They
alleged that the Secretary of
Justice committed grave abuse of
discretion. They prayed that the
Court of Appeals order the
Prosecutor
to
withdraw
the
Information
and instead,
file
several
Informations
against
petitioners. The Court of Appeals
rendered its Decision dismissing
the petition, holding that there was
no
conspiracy
among
the
petitioners.
Respondents
seasonably filed a motion for
reconsideration and then the Court
of Appeals reverses itself. The
Court of Appeals found that it
overlooked certain
facts and
circumstances
which,
if
considered,
would
establish
probable cause against Wilson and
Renita. The Court of Appeals
identified these facts to be: (1)
Marissas consistent practice of
depositing checks with altered
names of payees to the respective
accounts of Wilson Chua and
Renita Chua; (2) considering that

Wilson and Marissa are husband


and wife, it can be inferred that
one knows the transactions of the
other; and (3) Wilson had full
knowledge
of
the
unlawful
activities of Marissa. . This is
supported by the affidavit of
Ernesto Alcantara.
Issue:
Whether or not Court of Appeals
erred in compelling the Secretary
of Justice to include in the
Information Wilson and Renita.

Held:
No.
Section 5, Rule 110 of the 200
Rules of Criminal Procedure, as
amended, partly provides that "All
criminal actions either commenced
by a complaint or information shall
be prosecuted under the direction
and control of a public prosecutor."
The rationale for this rule is that
since a criminal offense is an
outrage to the sovereignty of the
State, it necessarily follows that a
representative of the State shall
direct and control the prosecution
thereof.. However, that the
public prosecutors exercise of
his discretionary powers is not
absolute. One of the exceptions
is that the Court of Appeals
may review the resolution of
the Secretary of Justice on a
petition for certiorari on the
ground that he committed

grave
abuse
of
discretion
amounting to excess or lack of
jurisdiction.
As found by the Court of Appeals,
the Secretary of Justice either
overlooked or patently ignored the
following
circumstances:
(1)
Marissas practice of depositing
checks, with altered names of
payees, in the respective accounts
of Wilson and Renita Chua; (2) the
fact that Wilson and Marissa are
husband and wife makes it difficult
to believe that one has no idea of
the transactions entered into by
the other; and (3) the affidavit of
Ernesto Alcantara confirming that
Wilson had knowledge of Marissas
illegal activities.

Philippine Rabbit
Inc. vs. People,
456(2004)
Topic:
Actions

Bus
427

Independent

Lines,
SCRA
Civil

Facts:
On
July
27,
1994,
accused
[Napoleon
Roman
y
Macadangdang] was found guilty
and convicted of the crime of
reckless imprudence resulting to
triple homicide, multiple physical
injuries and damage to property .
The court further ruled that
[petitioner], in the event of the
insolvency of accused, shall be
liable for the civil liabilities of the
accused.
The CA ruled that the institution of
a criminal case implied the
institution also of the civil action
arising from the offense. Thus,
once determined in the criminal
case
against
the
accusedemployee,
the
employers
subsidiary civil liability as set forth
in Article 103 of the Revised Penal
Code becomes conclusive and
enforceable. The appellate court
further held that to allow an
employer to dispute independently
the civil liability fixed in the
criminal case against the accusedemployee would be to amend,
nullify or defeat a final judgment.
Since the notice of appeal filed by
the accused had already been
dismissed by the CA, then the
judgment of conviction and the
award of civil liability became final

and executory. Included in the civil


liability of the accused was the
employers subsidiary liability.
Issue:
Whether or not and independent
civil action must be reserve or else
it will be deemed instituted with
the criminal case.
Held:
No.
The 2000 Rules of Criminal
Procedure has clarified what civil
actions are deemed instituted in a
criminal prosecutionit is only the
civil liability of the accused arising
from the crime charged that is
deemed impliedly instituted in a
criminal action.
The 2000 Rules deleted the
requirement
of
reserving
independent civil actions and
allowed
these
to
proceed
separately from criminal actions
what is deemed instituted in every
criminal prosecution is the civil
liability arising from the crime or
delict per se (civil liability ex
delicto), but not those liabilities
arising
from
quasi-delicts,
contracts or quasi-contracts. Even
if a civil action is filed separately,
the ex delicto civil liability in the
criminal prosecution remains, and
the offended party maysubject to
the control of the prosecutorstill
intervene in the criminal action, in
order to protect the remaining civil
interest therein.

Malto vs. People, 533 SCRA


643(2007)
Topic:
Sufficiency
Information

of

Facts:
Sometime during the month of
November 1997 to 1998, Malto
seduced his student, AAA, a minor,
to indulge in sexual intercourse
several times with him. Prior to the
incident, petitioner and AAA had a
mutual
understanding
and
became sweethearts. Pressured
and afraid of the petitioners
threat to end their relationship,
AAA succumbed and both had
sexual intercourse. Petitioner was
then convicted of rape by RTC.
Petitioner questioned the trial
courts decision in the CA. In a
decision dated July 30, 2004, the
appellate
court
affirmed
his
conviction even if it found that his
acts
were
not
covered
by
paragraph (a) but by paragraph (b)
of Section 5, Article III of RA 7610.
Issue:
Whether or not the Information
filed was sufficient.
Held:
Yes.
In all criminal prosecutions, the
accused is entitled to be informed
of the nature and cause of the
accusation against him. Pursuant
thereto,
the
complaint
or

information against him should be


sufficient in form and substance. A
complaint
or
information
is
sufficient if it states the name of
the accused; the designation of the
offense by the statute; the acts or
omissions
complained
of
as
constituting the offense; the name
of
the
offended
party;
the
approximate
date
of
the
commission of the offense and the
place where the offense was
committed.
The complaint or information shall
state the designation of the offense
given by the statute, aver the acts
or omissions constituting the
offense and specify its qualifying
and aggravating circumstances. If
there is no designation of the
offense, reference shall be made to
the section or subsection of the
statute punishing it. The acts or
omissions constituting the offense
and the qualifying and aggravating
circumstances must be stated in
ordinary and concise language and
not necessarily in the language
used in the statute but in terms
sufficient to enable a person of
common understanding to know
what offense is being charged as
well
as
its
qualifying
and
aggravating circumstances and for
the court to pronounce judgment.
The information against petitioner
did not allege anything pertaining
to or connected with child
prostitution. It did not aver that
AAA was abused for profit. What it
charged was that petitioner had
carnal knowledge or committed

sexual intercourse and lascivious


conduct with AAA; AAA was
induced
and/or
seduced
by
petitioner who was her professor
to indulge in sexual intercourse
and lascivious conduct and AAA
was a 17-year old minor. These
allegations support a charge for
violation of paragraph (b), not
paragraph (a), of Section 5, Article
III, RA 7610.
Lazarte, Jr. vs. Sandiganbayan,
581 SCRA 431(2009)
Topic:
Information

Sufficiency

of

Facts:
NHA awarded a contract for
infrastructure works with A.C.
Cruz Construction which was
funded by World Bank in the
amount of P7.67M.
During the
construction, the Project Engineer
discovered that the company
issued a report on an additional
work for the excavation of
unsuitable materials and road
filling works but after further
investigation, learned that there
was no such actual work done
(ghost activity).
The project engineer filed a
recommendation with the project
office to terminate the contract
with the construction company.
After several referrals, the NHA
decided to rescind the said
contract due to the anomalies
discovered. However, despite the
rescission,
the
construction
company continued to work on the

project until the project was


awarded to the new construction
company, Triad Construction.
Further investigations revealed
that AC Cruz Construction was
doing ghost activities and was
undertaking
substandard
and
defective works.
In addition,
officers of the NHA have overpaid
AC Cruz Construction for the
portion that was already started
and finished by the said company
(P232K overpaid) (they were paid
P1M for the partial work done).
Because of this, several officers of
the NHA were charged with
violating Sec.3 of the Anti-Graft
and Corrupt Practices Act.
Petitioner filed a motion to quash
on the ff grounds: 1) the facts
charged in the information do not
constitute an offense; (2) the
information does not conform
substantially to the prescribed
form; (3) the constitutional rights
of the accused to be informed of
the nature and cause of the
accusations against them have
been violated by the inadequacy of
the information; and (4) the
prosecution failed to determine the
individual participation of all the
accused in the information.
Issue:
Whether or not Information filed
constitutes
a
crime
and
is
sufficient.
Held:
Yes.

The test as to sufficiency of


complaint
or
information
is
whether the crime is described in
intelligible
terms
with
such
particularity as to apprise the
accused, with reasonable certainty,
of the offense charged. The raison
detre of the rule is to enable the
accused to suitably prepare his
defensethe use of derivatives or
synonyms or allegations of basic
facts constituting the offense
charged is sufficient.
The test must be whether or not
the
facts
asseverated,
if
hypothetically admitted, would
establish the essential elements of
the crime as defined by law. The
prosecution contends that matter
aliunde should not be considered.
However, in the instant case, the
Court has found the information
itself to be inadequate, as it does
not satisfy the requirements of
particularly alleging the acts or
omissions of the said accusedmovants, which served as the basis
of the allegation of conspiracy
between
the
aforementioned
accused-movants and the other
accused, in the commission of the
offense charged in the information.

People vs.
820(2009)

Cinco,

607

Topic:
Sufficiency
Information

SCRA
of

Facts:
Gualberto Cinco y Soyosa was then
charged of two counts of simple
rape. That on or about the 30th day
of November 1998, in Quezon City,
Philippines, the said accused with
lewd design, did then and there
willfully, unlawfully and feloniously
commit an act of sexual abuse
upon the person of AAA, a minor,
14 years old, by then and there
touching her body and mashing
her breast, against her will and
without her consent which act
debases, degrades, or demeans the
intrinsic worth and human dignity
of said complainant as a human
being,
to
the
damage
and
prejudice of the said offended
party. After due trial he was then
convicted however he contends
that he approximate times and
dates of the commission of the
offense must be stated in the
informations; that the informations
in the instant cases do not state
the approximate times and dates of
the alleged rapes; that although
AAA testified that the first rape
occurred nearly before All Saints
Day of 1998, the information in
Criminal nonetheless, states that
such incident transpired on 1
November
1998;
that
the
informations are fatally defective;
that the times and dates of the
alleged rapes are so indefinite,

thereby depriving appellant of the


opportunity to prepare for his
defense;
that
appellants
constitutional right to be informed
of the nature and cause of the
accusation
against
him
was
violated; and that by reason of the
foregoing, appellant is entitled to
an acquittal.
Issue:
Whether or not the failure to
alleged the exact date and time in
the
information
makes
the
information invalid.
Held:
No.
An information is an accusation in
writing charging a person with an
offense,
subscribed
by
the
prosecutor and filed with the
court. To be considered as valid
and sufficient, an information must
state the name of the accused; the
designation of the offense given by
the statute; the acts or omissions
complained of as constituting the
offense; the name of the offended
party; the approximate date of the
commission of the offense; and the
place where the offense was
committed The purpose of the
requirement for the informations
validity and sufficiency is to enable
the accused to suitably prepare for
his defense, since he is presumed
to have no independent knowledge
of the facts that constitute the
offense.

With respect to the date of the


commission of the offense, Section
11, Rule 110 of the Revised Rules
of Criminal Procedure specifically
provides that it is not necessary to
state in the information the precise
date the offense was committed
except when it is a material
ingredient of the offense, and that
the offense may be alleged to have
been committed on a date as near
as possible to the actual date of its
commission.
In rape cases, failure to specify
the exact dates or times when the
rapes occurred does not ipso
facto make
the
information
defective on its face. The reason is
obvious. The date or time of the
commission of rape is not a
material ingredient of the said
crime because the gravamen of
rape is carnal knowledge of a
woman
through
force
and
intimidation. The
precise
time
when the rape took place has no
substantial
bearing
on
its
commission. As such, the date or
time need not be stated with
absolute accuracy. It is sufficient
that the complaint or information
states that the crime has been
committed at any time as near as
possible to the date of its actual
commission.

[People vs. Teodoro, 607 SCRA


307(2009)]

exact date of the commission of


the offense.

Topic:
Sufficiency
Information

Held:

of

Facts:
Appellant Romar Teodoro was
found guilty beyond reasonable
doubt of two (2) countsof statutory
rape, and sentenced him to suffer
the penalty of reclusion perpetua
foreach count by the Regional Trial
Court of Batangas. However, there
are three differentsets of facts that
were laid down during his trial.
The said set of facts contained
differentdates but of the same
crime. The dates stated were: In
criminal case no 8538 June 18,
1995, in the morning. In criminal
case no 8539. First week of July, in
the morning. In criminal case no
8540. March 30, 1996, about 10:00
in the evening. In his defense, the
appellant invoked denial. He
denied raping the victim on June
18,1995 and on the first week of
July 1995, but admitted having a
consensual sexualintercourse with
AAA on March 30, 1996. We shall
only discuss the incidents of June
18,1995 and of the first week of
July 1995 (subject of Criminal Case
Nos. 8538 and 8539), asthe
appellant
had
already
been
acquitted in Criminal Case No.
8540.
Issue:
Whether or not the information
was defective for not stating the

No. An information, under Section


6, Rule 110 of the 2000 Revised
Rules on CriminalProcedure, is
deemed sufficient if it states the
name
of
the
accused;
the
designation of the offense given by
the statute; the acts or omissions
complained of as constituting
theoffense; the name of the
offended party; the approximate
date of the commission of the
offense; and the place where the
offense was committed. Section 11
of the sameRule also provides that
it is not necessary to state in the
complaint
or
information
theprecise date the offense was
committed, except when the date
of commission is a material
element of the offense. The offense
may thus be alleged to have been
committed on a date as near as
possible to the actual date of its
commission. At the minimum, an
indictment must contain all the
essential elements of the offense
charged to enable the accused to
properly meet the charge and duly
prepare for his defense.
It is not necessary to state the
precise time when the offensewas
committed except when time is a
material ingredient of the offense.
In statutory rape, time is not an
essential element except to prove
that the victim was a minor below
twelve years of age at the time of
the commission of the offense.

Given the victims established date


of birth, she was definitely short of
12 years under the allegations
of the Information and on the basis
of
the
evidence
adduced.
Moreover, objections relating to
the form of the complaint or
information cannot be made for
the first time on appeal. If the
appellant
had
found
the
Information insufficient, he should
have moved before arraignment
either for a bill of particulars, for
him to be properly informed of the
exact date of the alleged rape, or
for the quashal of the Information,
on the ground that it did not
conform with the prescribed form.
Failingto pursue either remedy, he
is
deemed
to
have
waived
objection to any formal defect
inthe Information.

Frias, Sr. vs. People, 534 SCRA


654(2007)
Topic:
Sufficiency
Information

of

Facts:
Petitioner Hermes E. Frias, Sr. was
charged with violation of Article
218 of the Revised Penal Code. He
was accused of misappropriating
public funds, he pleaded not guilty.
However he was convicted by
Sandiganbayan. In hid defense
petitioner asserts that he was
deprived of due process because
the Information against him failed
to identify his acts or omissions
which constituted a violation of
Article 218 of the Revised Penal
Code[ Moreover,
the
Sandiganbayan failed to establish
that he, a municipal mayor, was an
accountable officer and to identify
the particular law or regulation
which required him to render an
account.
Issue:
Whether or not questions about
sufficiency of information may be
raised first time on appeal.
Held:
No.
The
right
to
question
the
sufficiency of an Information is not
absolute. An accused is deemed to
have waived this right if he fails to
object upon his arraignment or
during trial. In either case,
evidence presented during trial

can cure the defect in the


Information. Petitioner waived his
right to assail the sufficiency of the
Information when he voluntarily
entered a plea when arraigned and
participated in the trial. At any
rate, the Information (quoted
above)
adequately
informed
petitioner of the charges against
him. It clearly stated the elements
which constituted the violation of
Article 218 of the Revised Penal
Code.

People vs. Anguac, 588 SCRA


716(2009)
Topic:
Allegations
Information

in

the

Facts:
The
RTC
convicted
accusedappellant Adelado Anguac of rape
and violation of Section 5(a),
Republic Act No. (RA) 7610 or
the Special Protection of Children
Against Child Abuse, Exploitation
and Discrimination Act.
CA
modified the conviction of RTC
holding that
RTC erroneously
convicted accused-appellant based
on the crime designated in the
information for that criminal case.
While the Information pertaining
to that criminal case charged
accused-appellant with violation of
Sec. 5(a) of RA 7610, the facts
alleged in it constitute elements of
a violation of Sec. 5(b) of the same
law

circumstances in the information.


As the Court has previously held,
the character of the crime is
determined neither by the
caption or preamble of the
information
nor
by
the
specification of the provision of
law alleged to have been
violated,
they
being
conclusions of law, but by the
recital of the ultimate facts and
circumstances
in
the
information. Consequently, even
if the designation of the crime in
the information of Criminal Case
No. RTC 2757-I was defective,
what is controlling is the allegation
of the facts in the information that
comprises a crime and adequately
describes the nature and cause of
the accusation against the accused

Issue:
Whether or not the crime is
determined by
the caption or
preamble of the information.
Held:
No.
The character of the crime is
determined neither by the caption
or preamble of the information nor
by
the
specification
of
the
provision of law alleged to have
been
violated,
they
being
conclusions of law, but by the
recital of the ultimate facts and

People vs. Quemeggen,


SCRA 94(2009)

594

Topic:
Allegations
Information

the

in

Facts:
An information for Robbery with
homicide against 2 Appellants

Quemeggen and de Luna for


having killed a victim of the holdup
they have conducted. However the
Court of Appeals held that the
appellants cannot be convicted
with Robbery with Homicide
because of the allegations in the
information of the crime charged
does not constitute the crime
robbery with homicide.
Issue:
Whether or not the designation of
the crime in the information
determine
the
nature
and
character of the crime charged.
Held:
No.
It is axiomatic that the nature and
character of the crime charged are
determined not by the designation
of the specific crime, but by the
facts alleged in the information.
Controlling in an information
should not be the title of the
complaint or the designation of the
offense charged or the particular
law or part thereof allegedly
violated, these being, by and large,
mere conclusions of law made by
the prosecutor, but the description
of the crime charged and the
particular facts therein recited.
There should also be no problem in
convicting an accused of two or
more crimes erroneously charged
in one information or complaint,
but later proven to be independent
crimes, as if they were made the
subject of separate complaints or
informations.

People vs. Amodia, 584 SCRA


518(2009)

People vs. Tumambing,


SCRA 482(2011)]

Topic: Identity of the Accused

Topic: Identity of the Accused

Facts:

Facts:

Pablo has been charged of the


crime murder and an information
was filed against him. However
Pablo
moved
to
quash
the
Information on the ground of
mistaken identity. RTC and CA
convicted him of the crime
charged after due trial. He
contends that the prosecution
failed to appreciate his evidence
and that there was a mistake in
identity.

The city prosecutor charged the


accused
Jenny
Tumambing
(Tumambing)
with
rape.
Tumambing denied committing the
crime. He claimed that on June
26, 2004 he slept at the house of
his employer, Nestor Ledesma. He
went to bed at about 9:00 p.m. and
woke up at 6:00 a.m. Tumambing
swore that he never left his
employers house that night. The
RTC and CA convicted him of the
crime.

Issue:
Whether or not a mistake in the
name of the accused on the
information amounts to mistake in
identity.
Held:
No.
A mistake in the name of the
accused is not equivalent, and
does not necessarily amount to, a
mistake in the identity of the
accused especially when sufficient
evidence is adduced to show that
the accused is pointed to as one of
the perpetrators of the crime. In
this case, the defenses line of
argument is negated by the
undisputed fact that the accuseds
identity was known to both the
eyewitnesses.

644

Issue:
Whether or not the CA and the
trial court erred in finding that
accused Tumambing guilty of the
crime.
Held:
No.
A successful prosecution of a
criminal action largely depends on
proof
of
two
things:
the
identification of the author of the
crime and his actual commission of
the same. An ample proof that a
crime has been committed has no
use if the prosecution is unable to
convincingly prove the offenders
identity.
The
constitutional
presumption of innocence that an
accused enjoys is not demolished
by an identification that is full of
uncertainties.

Here, both the RTC and the CA


gave credence to DKs testimony.
They
maintained
that
DK
categorically
and
positively
identified her rapist.
DKs identification of accused
Tumambing as her rapist is far
from categorical. The Courts
reading of her testimony shows
that she was quite reluctant at the
beginning but eventually pointed
to him when it was suggested that
it might be him after all. Several
witnesses
attested
to
DKs
uncertainties
regarding
the
rapists identity.

Sayson vs. People, 166 SCRA


680(1988)
Topic:
Sufficiency
Information

of

Facts:
An information for the crime of
Estafa through Falsification of a
Commercial Document was filed
against the herein petitioner
,Ramon SAYSON before the CFI of
Manila.
SAYSON was
duly
arraigned were he pleaded not
guilty. Then,trial ensued. The trial
court and the Court found him
guilty of the crime estafa. The
petitioner vigorously maintains
that he cannot be justifiably
convicted under the information
charging him of attempting to
defraud Ernesto Rufino, Sr. and/or
Bank of America because the
totality of the evidence presented
by the prosecution show very
clearly that the accused allegedly
attempted to defraud Mever Films,
Inc., a corporate entity entirely
separate and distinct from Ernesto
Rufino, Sr. He firmly asserts that
his conviction was in gross
violation of his right to be
informed of the nature and cause
of the accusation against him.
Issue:
Whether or not variance between
the allegations on the information
and the evidence presented by the
prosecution entitled the accused of
acquittal.
Held:

No.
Petitioner's claim is unavailing.
The rule in this jurisdiction is that
"variance between the allegations
of the information and the
evidence
offered
by
the
prosecution in support thereof
does not of itself entitle the
accused to an acquittal." The rules
on criminal procedure require the
complaint or information to state
the name and surname of the
person against whom or against
whose property the offense was
committed or any appellation or
nickname by which such person
has been or is known and if there
is no better way of Identifying him,
he must be described under a
fictitious name (Rule 110, Section
11, Revised Rules of Court; now
Rule 110, Section 12 of the 1985
Rules on Criminal Procedure.] In
case of offenses against property,
the designation of the name of the
offended party is not absolutely
indispensable for as long as the
criminal act charged in the
complaint or information can be
properly identified. The Court laid
down the rule that when an offense
shall have been described in the
complaint with sufficient certainty
as to Identify the act, an erroneous
allegation as to the person injured
shall be deemed immaterial as the
same is a mere formal defect
which did not tend to prejudice
any substantial right of the
defendant

Matrido vs. People, 592 SCRA


534(2009)
Topic: Allegations
Information

on

the

of

Facts:
Private respondent, Empire East
Land Holdings Inc. filed a case
against Matrido for estafa in the
Makati Prosecutors Office for
failing to remit payments received
from its clients. By resolution, the
prosecution office dismissed the
complaint
for
estafa
for
insufficiency of evidence but found
probable cause to indict petitioner
for qualified theft.
RTC convicted the Matrido of
qualified theft and was affirmed by
the CA. Petitioner challenges the
conviction by contending that
despite the indictment for qualified
theft, the prosecution was trying to
prove estafa during trial, thus
violating her right to be informed
of the nature and cause of the
accusation against her.
Issue:
Whether or not the petitioner
contention was tenable.
Held:
No.
It is settled that it is the
allegations in the Information that
determine the nature of the
offense, not the technical name
given by the prosecutor in the
preamble of the Information. From
a legal point of view, it is of no

concern to the accused what is the


technical name of the crime of
which he stands charged. It is in
no way aids him in a defense on
the merits. That to which his
attention should be directed,
above all things else, are the facts
alleged. The real question is not
did he commit a crime given in the
law some technical and specific
name, but did he perform the acts
alleged
in
the body
of
the
information in the manner therein
set forth. The recital of facts and
circumstances in the Information
sufficiently constitutes the crime
of qualified theft.
The
allegations
of
facts
constituting the offense charged
are substantial matters and an
accuseds right to question his
conviction based on facts not
alleged in the information cannot
be waived. No matter how
conclusive and convincing the
evidence of guilt may be, an
accused cannot be convicted of
any offense unless it is charged in
the information on which he is
tried or is necessarily included
therein. To convict him of a ground
not
alleged
while
he
is
concentrating his defense against
the ground alleged would plainly
be unfair and underhanded. The
rule is that a variance between the
allegation in the information and
proof adduced during trial shall be
fatal to the criminal case if it is
material and prejudicial to the
accused so much so that it affects
his substantial rights

Guy vs. People,


107(2009)

582

Topic:
Allegations
Information

SCRA

Held:
No.

on

the

Facts:
Petitioner was charged in three
(3) separate Informations with
violation of Section 3 (e) of R. A.
No. 3019, otherwise known as the
Anti-Graft and Corrupt Practices
Act, in connection with the
construction
of
three
(3)
infrastructure projects in Barangay
36, namely: an elevated path
walk, a basketball court and a day
care center. Petitioners, citing the
case of Lacson v. The Executive
Secretary,
assert
that
the
informations do not contain the
specific
factual
allegations showing
the
close
intimacybetween the discharge of
petitioners official duties and the
commission of the offense charged
to qualify the offense as one
committed in relation to public
office. In Lacson, the Court ruled
that
before
the
Sandiganbayan may
acquire
jurisdiction
over
the
offense
charged, the intimate relation
between the offense charged and
the discharge of official duties
must be alleged in the information.

the specific acts of the accused do


not have to be described in detail
in the information, as it is enough
that the offense be described with
sufficient particularity to make
sure the accused fully understand
what he is being charged with. The
particularity must be such that a
person of ordinary intelligence
immediately knows what the
charge is.Moreover, reasonable
certainty in the statement of the
crime suffices.It is often difficult to
say what is a matter of evidence,
as
distinguished
from
facts
necessary to be stated in order to
render the information sufficiently
certain to identify the offense. As a
general rule, matters of evidence,
as
distinguished
from
facts
essential to the description of the
offense, need not be averred. The
particular acts of the accused
which pertain to matters of
evidence, such as how accused city
officials prepared the inspection
reports despite the absence of a
project plan or how the contractor
was able to use substandard
materials, do not have to be
indicated in the information.

Issue::
Whether or not the specific acts of
the accused must be described in
detail in the information to prove
the crimes charged against him.

People
v
Ogarte
GR
182690, May 30, 2011

No.

Topic:
Qualifying
and
Aggravating circumstance must
be averred in the information
Facts:
Two separate Informations were
filed before the RTC, charging
Ogarte with two separate counts of
Rape. Ogarte was convicted of two
counts of rape by using force and
intimidation, qualified by the
concurrent circumstances of AAAs
minority and Ogartes relationship
with AAA. In an effort to escape
the penalty of death, as imposed
by Article 335 of the Revised Penal
Code when the crime of simple
rape is qualified, Ogarte claims
that the courts below erred in
appreciating AAAs minority as a
qualifying circumstance, because
it was never duly proven by the
prosecution.
Issue:
Whether or not the Ogarte may be
convicted of qualified rape even
the qualifying circumstances of
minority and relationship were not
alleged in the information.
Held:
No.
The complaint or information must
specify
the
qualifying
and
aggravating circumstances of the
offense. Every information must
state the qualifying and the
aggravating
circumstances
attending the commission of the
crime for them to be considered in
the imposition of the penalty.

Licyayo vs. People, 547 SCRA


598(2008)

Briones vs. People, 588 SCRA


362(2009)

Topic:
Sufficiency
Information

Topic:
Sufficiency
Information

of

of

Facts:

Facts:

Roberto Licyayo was charged for


the crime of homicide, for the
death of Rufino Guay. Petitioner
points out that the Information
does not specifically mention the
law which he allegedly violated
and for which he was charged.

A criminal information was filed


against Briones for crime of
robbery. Briones allegedly took the
service firearm of S/G Gual while
the latter approached the group
where the former is involved in a
mauling.
S/G
Gual
positively
identified Briones. RTC found
Briones guilty of the crime of
simple theft (Art. 309 Par. 3 of
RPC) after giving weight to
prosecutions positive testimony as
against the defenses of denial and
alibi. On his appeal, he raised the
issue of self-defense. The Court of
Appeals found Briones guilty of
robbery under Article 293 in
relation to par.5 of Art. 294 of RPC,
and not of theft.

Issue:
Whether or not the petitioner may
not be convicted of crime of
Homicide for failure to specifically
designate the crime charged.
Held:
No.
The fact that the information does
not specifically mention Article
249 of the Revised Penal Code as
the
law
which
defines
and
penalizes homicide, does not make
it defective. There is nothing in the
afore-quoted
Rules
which
specifically requires that the
information
must
state
the
particular law under which the
accused is charged in order for it
to be considered sufficient and
valid. What the Rules merely
require, among other things, is
that
the
information
must
designate the offense charged and
aver the acts constituting it, which
in this case, were obviously done.

Issue:
Whether or not the Court of
Appeals was correct on convicting
Briones of Robbery and not of
Theft.
Held:
No.
The
nature
of
the
crime
committed, as proven by the
evidence on record. We agree with
the RTC that only the crime of
theft was committed in the case as
S/G Gual's testimony does not

show that violence or intimidation


attended the taking of the firearm.
The character of the crime is not
determined by the caption or
preamble of the information, or by
the specification of the provision of
law
alleged
to
have
been
violated. The crime committed is
determined by the recital of the
ultimate facts and circumstances
in the complaint or information. In
this case, the allegations in the
Information are sufficient to make
out a charge of theft.

People vs. Montesclaros, 589


SCRA 320(2009)

even if not
information.

Topic: Averments of Qualifying


and Aggravating Circumstance

Held:

Facts:
Present appeal stems from two
criminal cases: (1) Criminal Case
No. 013324-L charging Bartolome
Tampus (Tampus) and Ida as
conspirators in the rape of ABC on
April 1, 1995 at 4:30 p.m.; and (2)
Criminal
Case
No.
013325-L
charging Tampus of raping ABC on
April 4, 1995 at 1:00 a.m.
On September 22, 1995, ABC filed
two Complaints. She accused
Tampus of taking advantage of her
by having carnal knowledge of her,
against her will, while she was
intoxicated and sleeping on April
1, 1995 at 4:30 p.m. She declared
in her Complaint that this was
done in conspiracy with accused
Ida who gave permission to
Tampus to rape her. And again, she
stated that on April 3, 1995, she
was threatened with a wooden
club
by
Tampus, who then
succeeded
in
having
sexual
intercourse with her, against her
will. The qualifying circumstance
of
minority
and
relationship
however was not alleged in the
information.
Issue:
Whether or not the qualifying
circumstance of minority and
relationship may be appreciated as
special qualifying circumstance

alleged

in

the

No.
the case at bar, although the
victim's minority was alleged and
established, her relationship with
the accused as the latter's
daughter was not properly alleged
in the Information, and even
though this was proven during
trial and not refuted by the
accused, it cannot be considered
as
a
special
qualifying
circumstance that would serve to
increase the penalty of the
offender. Under the 2000 Rules of
Criminal Procedure, which should
be
given
retroactive
effect
following the rule that statutes
governing court proceedings will
be construed as applicable to
actions pending and undetermined
at the time of their passage, every
Information must state the
qualifying and the aggravating
circumstances attending the
commission of the crime for
them to be considered in the
imposition of the penalty. Since
in the case at bar, the Information
in Criminal Case No. 013324-L did
not state that Ida is the mother
of ABC, this circumstance
could not be appreciated as a
special
qualifying
circumstance. Ida may only be
convicted as an accomplice in the
crime of simple rape, which is
punishable by reclusion perpetua.
In any event, Republic Act No.

9346,
entitled
an
"An
Act
Prohibiting the Imposition of Death
Penalty in the Philippines, which
was signed into law on June 24,
2006 prohibits the imposition of
the death penalty.

People vs. Mendoza, 383 SCRA


115(2002)
Topic: Averments of Qualifying
and Aggravating Circumstance
Facts:
The accused was charged of
simple rape. After due trial RTC
had convicted him of rape qualified
by the use of a deadly weapon.
However
the
qualified
circumstance of deadly weapon
was not alleged in the information.
Issue:
Whether or not the qualifying
circumstance of deadly weapon
may be appreciated even withour
being alleged in the information.
Held:
No.
It would be a denial of the right of
the accused to be informed of the
charges
against
him,
and
consequently, a denial of due
process, if he is charged with
simple rape, on which he was
arraigned, and be convicted of
qualified rape punishable by death.
aggravating
and
qualifying
circumstances
must
be

categorically
alleged
in
Information;
otherwise,
cannot be appreciated.

the
they

In this case, as contended by both


the defense counsel and the OSG,
appellant cannot be convicted of
rape qualified by the use of a
deadly
weapon,
since
that
circumstance was not alleged in
the Informations. He cannot be
punished for an offense graver
than that for which he was
charged.

Buebos vs. People, 550 SCRA


210(2008)
Topic: Averments of Qualifying
and Aggravating Circumstance
Facts:
On
complaint
of
Adelina,
petitioners Dante and Sarmelito
Buebos, together with Rolando
Buela and Antonio Cornel, Jr., were
indicted
for
arson
in
an
Information . The RTC found all of
the
accused
guilty
beyond
reasonable doubt of arson. There
was no allegation that the house
intentionally burned by petitioners
and their cohorts was inhabited.
Issue:
Whether or not the accused shall
be convicted of simple arson
instead of arson.
Held:
Yes.
Perusing the information, there
was no allegation that the house
intentionally burned by petitioners
and
their
cohorts
was
inhabited. Rather, the information
merely recited that accused,
conspiring,
confederating
and
helping one another, with intent to
cause damage, did then and
there wilfully,
unlawfully,
feloniously and maliciously set on
fire thenipa roof of the house of
ADELINA B. BORBE, to the latters
damage and prejudice.
Under the 2000 Rules of Criminal
Procedure, the information or

complaint
must
state
the
designation of the offense given by
the
statute
and
specify
its
qualifying and generic aggravating
circumstances, otherwise stated,
the accused will not be convicted
of the offense proved during the
trial if it was not properly alleged
in the information; It is elementary
that rules of criminal procedure
are given retroactive application
insofar as they benefit the accused

People vs. Dalisay, 605 SCRA


807(2009)
Topic: Averments of Qualifying
and Aggravating Circumstance
Facts:
The accused, Antonio Dalisay, is
the live-in partner of the victim's
mother. The 16-year old victim was
raped
by the
accused.
Consequently, an Information for
rape in relation to Republic Act
(R.A.) No. 7610 was filed. After
due trial RTC had convicted the
accused of qualified rape. Upon
review the Court of Appeals

modified the ruling of the trial


court convicting the accused of
simple rape instead of qualified
rape.
Issue:
Whether or not the conviction of
Dalisay of simple rape was correct.
Held:
Yes.
While it has been proven that
appellant was the common-law
spouse of the parent of the victim
and the child was a minor at the
time of the incident, the Court
cannot
convict
appellant
of
qualified rape because the special
qualifying
circumstances
of
minority and relationship were not
sufficiently
alleged
in
the
information.
To
recall,
the
information
here
erroneously
alleged that appellant was the
stepfather of the victim. Proven
during the trial, however, was that
appellant was not married to the
victims mother, but was only the
common-law spouse of the latter.
Following settled jurisprudence,
appellant is liable only of simple
rape punishable by reclusion
perpetua.

People vs. Rosas, 570 SCRA


117(2008)
Topic: Averments of Qualifying
and Aggravating Circumstance
Facts:
Accused-appellant was charged
with the crime of Murder in an
Information.
When
arraigned
on January 5, 1999, accusedappellant, assisted by counsel de
oficio, pleaded not guilty to the
crime charged. The trial court
rendered its decision convicting
accused-appellant of the crime of
murder. Accused-appellant argues
that he cannot be convicted of
murder because the Information
failed to state that treachery was a
qualifying circumstance.
Issue:
Whether or not the contention of
the accused tenable.
Held:
No.
Even after the recent amendments
to the Rules of Criminal Procedure,
qualifying circumstances need not
be preceded by descriptive words
such as qualifying or qualified
by to properly qualify an offense
it is not the use of the words
qualifying or qualified by that
raises a crime to a higher category,
but the specific allegation of an
attendant
circumstance
which
adds the essential element raising
the crime to a higher category. The
Information in this case sufficiently

alleged
the
qualifying
circumstance of treachery.

Ojeda v People GR No. 178935


Topic: Conspiracy must
Alleged on the Information

be

Facts:
In
an
Information
dated 12
September
2000,
Ruel
Jayar
Tolentino, Oscar A. Ojeda, Rene M.
Francisco,
Danilo
J.
Lintag,
Antonio Caamic, Michael Umagat,
Amado Gonzales and Police Officer
3 (PO3) Roberto Nadora were
charged before the RTC of Manila
with violation of Section 3601 of
the Tariff and Customs Code of
the Philippines. After due trial, the
petitioner was found guilty as coconspirator to the crime. However
he contends that the information
did not contain any allegation of
conspiracy, either by the use of the
words conspire or its derivatives
and synonyms, or by allegations of
basic facts constituting conspiracy
that will make them liable for the
acts of their co-accused.
Issue:
Whether or not absence of the
word
conspiracy
means
the
absence of such in the information.
Held:
No.
A look at the information readily
shows that the words conspiracy,
conspired or in conspiracy with
does
not
appear
in
the
information. This, however, does
not necessarily mean that the
absence of these words would

signify that conspiracy was not


alleged in the information. After
carefully reading the information,
we find that conspiracy was
properly
alleged
in
the
information.
Conspiracy must be alleged, not
merely
inferred,
in
the
information;
It
does
not
necessarily mean that the absence
of
the
words
conspiracy,
conspired or in conspiracy
with would signify that conspiracy
was not alleged in the information;
Court finds the phrase participate
in and facilitate to be a clear and
definite allegation of conspiracy
sufficient for those being accused
to competently enter a plea and to
make a proper defense.

People vs. Ubia, 527 SCRA


307(2007)
Topic: Averments of Qualifying
and Aggravating Circumstance
Facts:
The case is about the review on
the Decision of the Court of
Appeals (CA) in CA-G.R. CR No.
00012,
which
affirmed
with
modification the August 6, 2003
Decision of the Regional Trial
Court of Tuao, Cagayan, Branch 11
in Criminal Case No. 895-T, finding
appellant Orlando Ubia y Aggalut
guilty beyond reasonable doubt of
the crime of rape. That the
appellate court disregarded the
qualifying
circumstances
of
minority and relationship of the
parties in the imposition of penalty
because it noted that they were
not alleged in the information.
Issue:
Whether or not the court erred its
decision
the
qualifying
circumstance of relationship and
minority in the case.
Held:
No.
The twin circumstances of minority
and relationship under Article 335
of the Revised Penal Code, as
amended by R.A. No. 7659, are in
the
nature
of
qualifying
circumstances because they alter
the nature of the crime of rape and
increase the penalty. As special
qualifying circumstances they

must be specifically pleaded or


alleged with certainty in the
information; If the offender is
merely a relation - not a parent,
ascendant, step-parent, guardian,
or common law spouse of the
mother of the victim the specific
relationship must be alleged in the
information, i.e., that he is "a
relative
by
consanguinity
or
affinity [as the case may be] within
the third civil degree.
The information in the instant case
only mentioned appellant as AAAs
uncle, without specifically stating
that he is a relative within the
third civil degree, either by affinity
or consanguinity. Even granting
that during trial it was proved that
the relationship was within the
third civil degree either of
consanguinity or affinity, still such
proof
cannot
be
appreciated
because appellant would thereby
be denied of his right to be
informed of the nature and cause
of the accusation against him.
Appellant cannot be charged with
committing the crime of rape in its
simple form and then be tried and
convicted of rape in its qualified
form. Thus, the Court of Appeals
correctly
disregarded
the
qualifying
circumstance
of
relationship.

Nombrefia vs.
SCRA 369(2007)

People,

513

Topic: Allegations of Facts on


Information
Facts:
In
an
Information
filed
on
December 11, 1992, petitioner was
charged with violation of Section
261 of B.P. Blg. 881 before the RTC
of Baler, Aurora, Branch 66.4 In a
Decision rendered on March 13,
1998, the trial court found
petitioner Nombrefia guilty beyond
reasonable doubt. Petitioner post
contends that her right to be
informed
of
the
cause
of
accusation
against
her
was
violated because of the failure of
the prosecution to specifically
specify the law she had violated.
Issue:
Whether or not the right to be
informed of the petitioner was
violated by the failure of the
prosecution
to
specify
the
particular violation of the Omnibus
Election Code.
Held:
No.
The right of the petitioner to be
informed of the nature and cause
of the accusation against her was
not violated even if the information
failed to specify the particular
paragraph and/or subparagraphs
in Section 261 of the Omnibus
Election Code.

What determines the real nature


and cause of the accusation
against an accused is the actual
recital of facts stated in the
information or complaint and not
the caption or preamble of the
information or complaint, nor the
specification of the provision of
law alleged to have been violated,
they being conclusions of law. An
incorrect caption is not a fatal
mistake.

Serapio vs. Sandiganbayan, 396


SCRA 443(2003)]
Topic: Allegations of Facts on
the Information
Facts:
Petitioner Edward Serapio was a
member of the Board of Trustees
and the legal counsel of the Erap
Muslim
Youth
Foundation. Sometime 2000,petitio
ner received
on its behalf a donation in theamou
nt of Php 200Mthrough Chavit Sin
gson. Petitioner received the donat
ion worth theFoundations
account. In 2000, Chavit Singson
publicly accused President Estrada
and his family members and
friends of engaging in several
illegal activities which triggered
the filing with the Office of the
Ombudsman
several
criminal
complaints against the petitioner,
Joseph Estrada and his son.
Petitioner asserts that there is no
allegation in paragraph (a) of the
amended
Information
of
a
combination or series of overt or
criminal acts constituting plunder
as described in Section 1(d) of R.A.
7080 as amended. Neither does
the amended Information allege a
pattern of criminal acts. He avers
that his single act of toleration or
protection of illegal gambling
impelled by a single criminal
resolution does not constitute the
requisite combination or series of
acts for plunder. He further claims
that the consideration consisting of

gifts, percentages or kickbacks in


furtherance of said resolution
turned over to and received by
former President Joseph E. Estrada
on several occasions does not cure
the defect in the amended
information. Petitioner insists that
on the face of the amended
Information he is charged only
with bribery or illegal gambling
and not of plunder.
Issue:
Whether or not the petitioners
contention was correct.
Held:
No.
In
this
case,
the
amended
Information specifically alleges
that all the accused, including
petitioner, connived and conspired
with former President Joseph E.
Estrada
to
commit
plunder
through any or a combination or a
series of overt or criminal acts or
similar schemes or means.
The acts or omissions complained
of must be alleged in such form as
is sufficient to enable a person of
common understanding of know
what offense is intended to be
charged and enable the court to
know the proper judgment. The
Information must allege clearly
and accurately the elements of the
crime charged. What facts and
circumstances are necessary to be
included
therein
must
be
determined by reference to the
definition and elements of the

specific crimes. The purpose of the


requirement of alleging all the
elements of the crime in the
Information is to inform an
accused of the nature of the
accusation against him so as to
enable him to suitably prepare for
his defense. Another purpose is to
enable accused, if found guilty, to
plead
his
conviction
in
a
subsequent prosecution for the
same
offense.
The
use
of
derivatives
or
synonyms
or
allegations
of
basic
facts
constituting the offense charged is
sufficient.

People vs.
836(2008)]

Aure,

569

SCRA

Topic: Averments of Qualifying


and Aggravating Circumstance
Facts:
On 20 January 2000, two separate
informations for rape were filed
with the RTC charging appellants
of rape. Subsequently, these cases
were consolidated for joint trial.
When arraigned on 30 May 2000,
appellants,
assisted
by
their
respective
counsels de
parte,
pleaded "Not guilty" to the
charges. Trial on the merits
thereafter followed. After trial, the
RTC rendered a Decision on 5
December
2000
convicting
appellant Aure of rape in Criminal
Case.
AAA was raped by
appellants while she was under the
custody of the CIDG. Further,
appellants were members of the
PNP-CIDG at the time they raped
AAA.
Nonetheless,
these
aggravating/qualifying
circumstances
were
not
specifically
alleged
in
the
informations.
Issue:
Whether or not the aggravating
circumstance may be appreciated
if proved during trial even if not
alleged in the information.
Held:
No.
It
is
settled
that
aggravating/qualifying

the

circumstances be expressly and


specifically
alleged
in
the
information,
otherwise
they
cannot be appreciated, even if
they are subsequently proved
during the trial.In the case
under consideration, AAA was
raped by appellants while she was
under the custody of the CIDG.
Further, appellants were members
of the PNP-CIDG at the time they
raped AAA. Nonetheless, these
aggravating/qualifying
circumstances
were
not
specifically
alleged
in
the
informations. It is settled that the
aggravating/qualifying
circumstances be expressly and
specifically
alleged
in
the
information, otherwise they cannot
be appreciated, even if they are
subsequently proved during the
trial. Thus, the RTC was correct in
imposing the penalty of reclusion
perpetua
on
each
of
the
appellants.

People vs.
356(2009)

Mejia,

595

SCRA

Topic: Averments of Qualifying


and Aggravating Circumstance
Facts:
On 2
March
2003,
private
complainants (AAAs) womanhood
was allegedly violated by a man
cohabiting with her mother (BBB)
as
common-law-spouse.
On 9
October 2003, after appropriate
proceedings, the Office of the
Provincial
Prosecutor
of
Pangasinan filed, with the RTC of
San Carlos City in Pangasinan, two
separate informations for Rape
under Article 266-A of the Revised
Penal Code. Accused-appellant
pleaded NOT GUILTY to both
charges. The trial court found
accused-appellant guilty beyond
reasonable doubt of the crimes of
(a) Rape; and (b) Acts of
Lasciviousness. The trial court
appreciated
the
qualifying
circumstance of minority and
relationship, so that under Article
266-B of Republic Act No. 8353,
the penalty would have been
death. With the suspension of the
death
penalty
due
to
the
enactment of Republic Act No.
9346, the RTC imposed reclusion
perpetua. The Court of Appeals
affirmed accused-appellants guilt
in the two cases, but modified the
decision of the court a quo by
disregarding
the
qualifying
circumstance of minority
Issue:

WHETHER OR NOT THE TRIAL


COURT GRAVELY ERRED IN
CONSIDERING THE QUALIFYING
CIRCUMSTANCE OF MINORITY
OF THE VICTIM ALTHOUGH THE
INFORMATION
DOES
NOT
ALLEGE SUCH CIRCUMSTANCE
AND THAT THE PROSECUTION
INTRODUCED NO PROOF AS TO
THE AGE OF THE VICTIM AT THE
TIME
THE
ALLEGED
RAPE
INCIDENT HAPPENED.
Held:
Yes. The twin circumstances of
minority of the victim and her
relationship to the offender must
concur to qualify the crime of rape.
As
amended,
and
effective 1
December 2000, Secs. 8 and 9,
Rule 110 of the Revised Rules on
Criminal Procedure now provide
that aggravating as well as
qualifying circumstances must be
alleged in the information and
proven during trial; otherwise they
cannot be considered against the
accused. Proof of the age of the
victim cannot consist merely of
testimony. Neither
can
a
stipulation of the parties with
respect to the victims age be
considered sufficient proof of
minority. Thus, the same cannot be
used to impose the higher penalty
of capital punishment on the
accused-appellant

People vs.
156(2008)

Tabio,

544

SCRA

Topic: Duplicity of Offense


Facts:
Appellant
Jimmy Tabio was
charged with three (3) counts of
rape
in
a single
Information
Appellant pleaded not guilty on
arraignment before the Regional
Trial Court (RTC) of Baler,Aurora,.
Trial on the merits ensued. The
RTC handed down a decision
finding
appellant
guilty
and
imposing the penalty of death on
three (3) counts of qualified rape.
The CA affirmed with modification
the decision of thet rial court. The
appellate court found appellant
guilty of all three (3) counts for
simple rape only and not qualified
rape. The Court also observes that
there is duplicity of the offenses
charged in the information, which
is a ground for a motion to quash.
Issue:
Whether or not failure of the
accused to question the duplicity
of offense of the information
constitutes a waiver.
Held:
Yes. Three (3) separate acts of
rape
were
charged
in
one
information only. But the failure
of appellant to interpose an
objection
on
this
ground
constitutes waiver.

The failure of the accused to assert


any ground of a motion to quash
before he pleads to the complaint
or information, either because he
did not file a motion to quash or
failed to allege the same in said
motion, shall be deemed a waiver
of any objections.

People vs. Chingh, 645 SCRA


573(2011)
Topic: Duplicity of Offense
Facts:
On
March
19,
2005,
an
Information for Rape was filed
against Armando for inserting his
fingers and afterwards his penis
into the private part of his minor
victim, VVV. Upon his arraignment,
Armando pleaded not guilty to the
charge. The RTC, after finding the
evidence
of
the
prosecution
overwhelming
against
the
accuseds defense of denial and
alibi,
rendered
a
Decision
convicting Armando of Statutory
Rape.
The
CA
rendered
a
Decision7 finding Armando not only
guilty of Statutory Rape, but also
of Rape Through Sexual Assault.
Issue:
Whether or not the Court of
Appeals was correct on convicting
him of both Statutory Rape and
Rape through sexual assault.
Held:
Yes.
It is clear that Armando was being
charged with two offenses, Rape
under paragraph 1 (d), Article 266A of the Revised Penal Code, and
rape as an act of sexual assault
under paragraph 2, Article 266-A.
Armando was charged with having
carnal knowledge of VVV, who was
under twelve years of age at the
time, under paragraph 1 (d) of

Article 266-A, and he was also


charged with committing an act of
sexual assault by inserting his
finger into the genital of VVV
under the second paragraph of
Article
266-A.
Indeed,
two
instances of rape were proven at
the trial. First, it was established
that Armando inserted his penis
into the private part of his victim,
VVV.
Second,
through
the
testimony of VVV, it was proven
that Armando also inserted his
finger in VVVs private part.
The Information has sufficiently
informed accused-appellant that
he is being charged with two
counts of rape. Although two
offenses were charged, which is a
violation of Section 13, Rule 110 of
the Revised Rules of Criminal
Procedure, which states that "[a]
complaint or information must
charge only one offense, except
when the law prescribes a single
punishment for various offenses."
Nonetheless, Section 3, Rule 120
of the Revised Rules of Criminal
Procedure
also
states
that
"[w]hen two or more offenses
are
charged
in
a
single
complaint or information but
the accused fails to object to it
before trial, the court may
convict the appellant of as
many as are charged and
proved, and impose on him the
penalty
for
each
offense,
setting
out
separately
the
findings of fact and law in each
offense." Consequently, since
Armando failed to file a motion

to quash the Information, he


can be convicted with two
counts of rape.

Agbayani vs. Sayo, 89 SCRA


699(1979)
Topic: Venue of Criminal Action
Facts:
Conrado B. Mahinan, a lawyer, was
the
manager
of the Cagayan Valley Branch of th
e GovernmentService Insurance Sy
stem (GSIS) stationed at
Cauayan, Isabela. Among his subor
dinates wereWilson Agbayani, Car
melo N. Bautista, Pablo R.Pascual,
and Renato Romeo P. Dugay.-On
March 8, 1976, Mahinan filed with
the fiscal's office at Bayombong,
Nueva Vizcaya a complaint for
written defamation against Agbaya
ni, Bautista ,Pascual and Dugay.On
July 23,
1976, the
provincial fiscal
of NuevaVizcaya
filed in the Court of First Instance
of that province an information for
libel charging Agbayani ,Bautista,
Pascual and Dugay with having
maliciously
made
defamatory
imputations against Mahinan on or
about February 17, 1976 in Bamba
ng, NuevaVizcaya.
The four accused filed a motion to
quash contendingthat the Court of
First Instance of Nueva Vizcaya
has
no jurisdiction over
the offense charged becauseMahin
an was a public officer holding offi
ce at
Cauayan, Isabela when the alleged
libel wa scommitted and, under
Article 360 of the Revised Penal
Code, the offense charged comes
within the jurisdiction of the Court

of First
Instance of
Isabela. They argued that the provi
ncial fiscal of Nueva Vizcaya had
no authority to conduct the
preliminary investigation and to
file the information.It was denied
by the trial court in its order of
April25, 1977 on the ground that
Mahinan was not a public officer
within the meaning of article 203
of the Revised Penal Code since
the insurance business of the GSIS
is not an inherently governmental
function.
Issue:
Whether or not CFI of Nueva
Viscaya is the proper venue for the
Criminal Case.
Held:
Yes.
The rules on venue in Article 360
may be restated thus: (1) whether
the offended party is a public
official or a private person, the
criminal action may be filed in the
Court of First Instance of the
province or city where the libelous
article
is
printed
and
first
published. (2) If the offended party
is a private individual, the criminal
action may also be filed in the
Court of First Instance of the
province where he actually resided
at the time of the commission of
the offense. (3) If the offended
party is a public officer whose
office is in Manila at the time of
the commission of the offense, the
action may be filed in the Court of
First Instance of Manila. (4) If the

offended party is a public officer


holding office outside Manila, the
action may be filed in the Court of
First Instance of the province or
city where he held office at the
time of the commission of the
offense.
Foz, Jr. vs. People, 603 SCRA
124(2009)
Topic: Venue/Jurisdiction
FACTS:
In an Information filed before the
RTC of Iloilo City, petitioners
Vicente Foz, Jr., and Danny
G.Fajardo, columnist and EditorPublisher, respectively, of Panay
News were charged with the crime
of libel. They wrote and publish in
the regular issue of the Panay
news a certain article maliciously
injuring and exposing one Dr.
Edgar
Portigo,
a
company
physician of
San
Miguel
Corporation office,
SMC, as
an
incompetent
doctor
and
an
opportunist who enriched himself
at the expense of the poor .The
RTC rendered its Decision finding
petitioners guilty as charged.
Dissatisfied, petitioners filed an
appeal
with
the
CA
which
rendered its assailed Decision
affirming in toto the RTC decision
ISSUE:
Whether or not Iloilo RTC has
jurisdiction over the case.
HELD:
No.

Venue in criminal cases is an es


sential element of jurisdiction.
Article 360 of the Revised Penal Co
de, as amended by Republic
Act No. 4363, provides the specific
rules as to the venue in cases of
written defamation: xxx The
criminal action and civil action for
damages
in cases of written defamations, as
provided for in this chapter shall
be
filed
simultaneously
or
separately with the court of first i
nstance of the province or city whe
re the libelous article is printed an
d firstpublished or where any of
the offended parties actually
resides at the time of the
commission
of
the
offense:
Applying the foregoing law to this
case, since Dr. Portigo is a private i
ndividual at the time of thepublicat
ion of the alleged libelous article,
the venue of the libel case may be
in the province or city where the
libelous article was printed and
first published, or in the province
where Dr. Portigo actually resided
at the time of the commission
of the offense. The allegations in
the Information that Panay News
a
daily
publication
with
a
considerable circulation in the City
of Iloilo and throughout the
region only showed that Iloilo was
the
place
where
Panay News was in considerable c
irculation
but did not establish that the said
publication was printed and first
published in Iloilo City. Settled is

the rule that jurisdiction of a court


over a criminal case is determined
by the allegations of the complaint
or information, and the offense
must have been committed or any
one of its essential ingredients
took place within the territorial
jurisdiction
of
the
court.
Considering that the Information
failed
to
allege
the
venue
requirements for a libel case under
Article 360, the Court finds that
the RTC of Iloilo City had no
jurisdiction to hear this case.
Agustin vs. Pamintuan,
SCRA 601(2005)

467

TOPIC: Venue is Jurisdictional


FACTS:
On June 13, 2000, the Office of the
City Prosecutor of Baguio City,
filed four separate Informations
charging
the
petitioner,
a
Philippine
Daily
Inquirer
columnist, with libel.
He was
arraigned and he pleaded not
guilty to all the charges. On
September 10, 2001, he then filed
a
Motion
to
Quash
the
Informations on the sole ground
that the court had no jurisdiction
over the offenses charged. He
pointed
out
that
the
said
Informations did not contain any
allegation that the offended party
was actually residing in Baguio
City or that the alleged libelous
articles were printed and first
published in a newspaper of
general circulation in Baguio City.
Private complainant opposed the

motion alleging that he was a bona


fide resident and acting general
manager
of
Baguio
Country
Club. The petitioner contented that
in the absence of any allegations in
the Informations that the private
respondent was actually residing
in Baguio City, or that the alleged
libelous articles were printed and
first published in Baguio as
mandated by Article 360 of the
RPC, the trial court had no
jurisdiction over the offenses
charged. He asserted that the
amendments of the Informations
would be improper, considering
that
the
defects
of
the
Informations were not of form but
of substance.
Issue:
Whether or not RTC of Baguio City
has jurisdiction over the offenses
charged in the four Informations.
HELD:
No. RTC of
jurisdiction.

Baguio

has

no

The SC held that venue


in criminal cases is an essential
element
of
jurisdiction.
The
jurisdiction of a court over the
criminal case is determined by the
allegations in the complaint or
Information, and the offense must
have been committed or any of its
essential ingredients took place
within the territorial jurisdiction of
the court. Article 360 of the RPC
provides that the criminal and civil
action

for damages in cases of written


defamations,
shall
be
filed
simultaneously or separately with
the Court of First Instance of the
province or city where the libelous
article
is
printed
and
first
published or where any of the
offended parties actually resides at
the time of the commission of the
offense.
In the case at bar, the
Informations did not allege that
the offended party was actually
residing in Baguio City at the time
of the commission of the offenses,
or that the alleged libelous articles
were printed and first published in
Baguio City. It cannot even be
inferred from the allegation 'the
offended party was the Acting
General Manager of the Baguio
Country Club and of good standing
and reputation in the community
that the private complainant was
actually residing in Baguio City.

Contents
Implied Institution of the Civil Action with the Criminal Action.....................4
Cruz vs Mina, G.R. No. 154207, April 27, 2007...........................................4
Ricarze vs CA, G.R. No. 160451, February 9, 2007.....................................5
Heirs of Sarah Marie Palma Burgos vs CA G.R. No. 169711, February 8,
2010..............................................................................................................7
ABS-CBN Broadcasting Corp. vs Ombudsman G.R. No. 133347, October
15, 2008........................................................................................................8
Hun Hyung Park vs Eun Wong Choi, G.R. No. 165496, February 12, 2007
....................................................................................................................10
When a Civil Action may proceed independently; independent civil actions
and quasi-delicts...........................................................................................12
Philippine Rabbit Bus Lines, Inc vs People, G.R. No. 147703, April 14,
2004............................................................................................................12
No Reservation of the Civil Action in B.P. 22................................................14
Lo Bun Tiong vs Balboa G.R. No. 158177, January 28, 2008.....................14
Hyatt Industrial Manufacturing Corp. vs Asia Dynamic Electrix Corp. , 465
SCRA 454...................................................................................................14
When no Reservation is required; when civil action is not suspended.........17
Casupanan vs Laroya, 388 SCRA 28..........................................................17
Counterclaim, cross-counterclaim, third-party claim in a criminal action. . .19
Maccay vs Nobela, 454 SCRA 504.............................................................19
Effect of Death of the Accused on the Civil Action.......................................21
People vs Bayotas, G.R. No. 102007, September 4, 1994..........................21
People vs Bringas Bunay y Dam-at, G.R. No. 171268, September 14, 2010
....................................................................................................................22
People vs Jaime Ayochok y Tauli, G.R. No. 175784, August 25, 2010........23
Novation: Extiguishment of Criminal Liability..............................................25
Metropolitan Bank and Trust Company vs Rogelio Reynaldo and Jose
Adrandea, G.R. No. 164538, August 9, 2010.............................................25
Effect of Acquittal or the Extinction of the Penal Action on the Civil Action
or Civil Liability.............................................................................................27
Ching vs Nicdao, 522 SCRA 316................................................................27

Bax vs People, 532 SCRA 284....................................................................29


Heirs of Sarah Marie Palma Burgos, G.R. No. 169711, February 28, 2010
....................................................................................................................31
Ramon Garces vs Simplicio Hernandez, et al, G.R. No. 180761, August 18,
2010............................................................................................................32
Romero vs People, G.R. No. 167546, July 17, 2009...................................34
Effect of Payment of Civil Liability................................................................35
Cabico vs Dimaculangan-Querijero, 522 SCRA 300..................................35
Subsidiary Liability of Employer...................................................................36
Rolito Calang vs People, G.R. No. 190696, August 3, 2010.......................36
Concept of Prejudicial Question....................................................................37
Omictin vs CA, 512 SCRA 70......................................................................37
Reyes vs Pearlbank Securities Inc., 560 SCRA 518...................................39
Coca-Cola Bottlers, Inc. vs Social Security Commission, 560 SCRA 719. .41
Reason of Principle of Prejudicial Question..................................................43
Jose vs Suarez, 556 SCRA 773...................................................................43
Sy Thiong Siou vs Sy Chim, G.R. No. 174168, March 30, 2009.................45
Requisites for a Prejudicial Question............................................................47
Magestrado vs People, G.R. No. 148072, July 10, 2009.............................47
Torres vs Garchitorena, G.R. No. 153666, December 27, 2002.................49
Effect of Existence of a Prejudicial Question; suspension of the Criminal
Action............................................................................................................51
IBP vs Atienza, G.R. No. 175241, February 24, 2010................................51
Suspension does not include dismissal.........................................................52
Yap vs Parras, 205 SCRA 625.....................................................................52
Prejudicial Question: Case Illustrations........................................................54
Pimentel vs Pimentel, G.R. No. 172060, September 13, 2010...................54
Yap vs Cabales, G.R. No. 159186, June 5, 2009.........................................56
Sps. Jose vs Sps Suarez, G.R. No. 176795, June 30, 2008.........................58
Land Bank of the Philippines vs Ramon Jacinto, G.R. No. 154622, August
3, 2010........................................................................................................60

Sabandal vs Tongco, G.R. No. 124498, October 5, 2001...........................61


Bobis vs Bobis, 336 SCRA 747...................................................................63
Landicho vs Relova, G.R. No. L-225779, February 23, 1968.....................64
Beltran vs People, G.R. No. 137567, June 20, 2000...................................65
Te vs CA, G.R. No. 126746, November 29, 2000........................................66
Nature of Prejudicial Question; purpose.......................................................67
Presidential Ad Hoc Fact-Finding Committee on Behest Loans vs Desierto,
G.R. No. 135703, April 15, 2009................................................................67
De Chavez vs Ombudsman, G.R. No. 168830-31, February 6, 2007..........69
Manebo vs Acosta, G.R. No. 169554, October 28, 2009............................71
Spouses Balanguan vs CA, G.R. No. 174350, August 13, 2008.................73
Samuel Lee vs KBC Bank, G.R. No. 169554, October 28, 2009.................75
Sales vs Sandiganbayan, G.R. No. 143802, November 16, 2001...............77
Ledesma vs CA, 278 SCRA 656..................................................................79
Marinas vs Siochi, 104 SCRA 423..............................................................81
Nature of the Right to a Preliminary Investigation......................................83
Doromal vs Sandiganbayan, 177 SCRA 354...............................................83
Duterte vs Sandiganbayan, 289 SCRA 721................................................85
Cruz vs People, 233 SCRA 439...................................................................87
Bautista vs CA, G.R. No. 143375, July 6 2001............................................90
Santos vs Go, G.R. No. 156081, October 19, 2005....................................92
Metropolitan Bank and Trust Company vs Tonda, 338 SCRA 254.............94
Right to a Preliminary Investigation; waivable.............................................96
People vs Gomez, 117 SCRA 73.................................................................96
People vs Bulosan, 160 SCRA 492.............................................................97
Go vs CA, 206 SCRA 138............................................................................98
Preliminary Investigation vs. Preliminary Examination..............................100
Peopls vs Inting, 187 SCRA 788...............................................................100
AAA vs Carbonel, 524 SCRA 496.............................................................101
Co vs Republic, 539 SCRA 147................................................................103
Probable Cause in Preliminary Investigation..............................................106

Roberto Kalalo vs Office of the Ombudsman, G.R. No. 158189, April 23,
2010..........................................................................................................106
Ricaforte vs Jurado, G.R. No. 154438, September 5, 2007......................108
Sy Thiong Siou vs Sy Chim, G.R. No. 174168, March 30, 2009...............110
Heirs of Jose Sy Bang vs Sy, G.R. No. 114217, October 13, 2009...........112
Limanch-O Hotel and Leasing Corporation, et al. vs City of Olongapo, G.R.
No. 185121, January 18, 2010.................................................................114
Kinds of Determination of Probable Cause.................................................116
People vs Castillo, G.R. No. 171188, June 19, 2009.................................116
Webb vs De Leon, 247 SCRA 652.............................................................118
Domalanta vs COMELEC, G.R. No. 125586, June 29, 2000.....................118

Implied Institution of the Civil


Action with the Criminal Action
Cruz vs Mina, G.R. No. 154207,
April 27, 2007
Facts:
Ferdinand A. Cruz filed before the
MeTC
a
formal
Entry
of
Appearance, as private prosecutor,
where his father, Mariano Cruz, is
the complaining witness.
The petitioner, describing himself
as a third year law student,
justifies his appearance as private
prosecutor on the bases of Section
34 of Rule 138 of the Rules of
Court and the ruling of the Court
En Banc in Cantimbuhan v. Judge
Cruz, Jr. that a non-lawyer may
appear before the inferior courts
as an agent or friend of a party
litigant.
The
petitioner
furthermore
avers
that
his
appearance was with the prior
conformity
of
the
public
prosecutor and a written authority
of Mariano Cruz appointing him to
be his agent in the prosecution of
the said criminal case.
However, in an Order dated
February 1, 2002, the MeTC
denied permission for petitioner to
appear as private prosecutor on
the ground that Circular No. 19
governing limited law student
practice in conjunction with Rule
138-A of the Rules of Court (Law
Student Practice Rule) should take
precedence over the ruling of the
Court laid down in Cantimbuhan;
and set the case for continuation of
trial.

Issue:
Whether the petitioner, a law
student, may appear before an
inferior court as an agent or friend
of a party litigant
Ruling:
The rule, however, is different if
the law student appears before an
inferior court, where the issues
and procedure are relatively
simple. In inferior courts, based on
Sec. 34, Rule 138, a law student
may appear in his personal
capacity without the supervision of
a lawyer.
Thus, a law student may appear
before an inferior court as an
agent or friend of a party without
the supervision of a member of the
bar.

Every person criminally liable


for a felony is also civilly liable
except in the instances when no
actual damages results from an
offense, such as espionage,
violation of neutrality, flight to
an enemy country, and crime
against popular representation.
(Cruz vs Mina, G.R. No. 154207,
April 27, 2007)

Ricarze vs CA, G.R. No. 160451,


February 9, 2007
FACTS:
Petitioner Eduardo G. Ricarze was
employed as a collector-messenger
by City Service Corporation, a
domestic corporation engaged in
messengerial services. He was
assigned to the main office of
Caltex Philippines, Inc. (Caltex) in
Makati City. His primary task was
to collect checks payable to Caltex
and deliver them to the cashier. He
also delivered invoices to Caltexs
customers.
On November 6, 1997, Caltex,
through its Banking and Insurance
Department
Manager
filed a
criminal
complaint
against
petitioner before the Office of the
City Prosecutor of Makati City for
estafa through falsification of
commercial documents. Romano
alleged that, on October 16, 1997,
while
his
department
was
conducting a daily electronic
report from PCIB, one of its
depositary
banks,
it
was
discovered that unknown to the
department, a company check,
Check No. 74001 dated October
13, 1997 payable to Dante R.
Gutierrez,
had
been
cleared
through PCIB on October 15,
1997.
An
investigation
also
revealed that two other checks
(Check Nos. 73999 and 74000)
were also missing and that in
Check No. 74001, his signature
and that of another signatory,
Victor S. Goquinco, were forgeries.
Another check, Check No. 72922
dated September 15, 1997 likewise
payable to Dante R. Gutierrez, was

also cleared through the same


bank on September 24, 1997; this
check was likewise not issued by
Caltex,
and
the
signatures
appearing thereon had also been
forged. Upon verification, it was
uncovered that Check Nos. 74001
and 72922 were deposited at the
Banco de Oros SM Makati Branch
under Savings Account No. S/A
2004-0047245-7, in the name of a
regular customer of Caltex, Dante
R. Gutierrez.
Gutierrez, however, disowned the
savings account as well as his
signatures on the dorsal portions
thereof. He also denied having
withdrawn any amount from said
savings
account.
Further
investigation revealed that said
savings account had actually been
opened by petitioner; the forged
checks
were
deposited
and
endorsed by him under Gutierrezs
name. A bank teller from the
Banco de Oro, Winnie P. Donable
Dela Cruz, positively identified
petitioner as the person who
opened the savings account using
Gutierrezs name.

ISSUE:
Whether RESPONDENT JUDGE
GRIEVEOUSLY (SIC) ERRED IN
RENDERING ITS ORDER ISSUED
WITH
GRAVE
ABUSE
OF
DISCRETION TANTAMOUNT TO
LACK OF OR IN EXCESS OF
JURISDICTION BY ALLOWING
THE SUBSTITUTION OF PRIVATE
COMPLAINANT,
AFTER
THE
ACUSED
WAS
ALREADY
ARRAIGNED AND PROSECUTION

HAS
ALREADY
TERMINATED
PRESENTING
ITS
EVIDENCE
THEREBY PATENTLY VIOLATING
THE
STRICT
CONDITION
IMPOSED UPON BY RULE 110
SEC. 14 RULES ON CRIMINAL
ROCEDURE.

HELD:
Under Section 5, Rule 110 of the
Revised Rules of Rules, all criminal
actions covered by a complaint or
information shall be prosecuted
under the direct supervision and
control of the public prosecutor.
Thus, even if the felonies or
delictual acts of the accused result
in damage or injury to another, the
civil action for the recovery of civil
liability based on the said criminal
acts is impliedly instituted, and the
offended party has not waived the
civil action, reserved the right to
institute it separately or instituted
the civil action prior to the
criminal action, the prosecution of
the action (including the civil)
remains under the control and
supervision
of
the
public
prosecutor. The prosecution of
offenses is a public function.
Under Section 16, Rule 110 of the
Rules of Criminal Procedure, the
offended party may intervene in
the criminal action personally or
by counsel, who will act as private
prosecutor for the protection of his
interests and in the interest of the
speedy
and
inexpensive
administration
of
justice.
A
separate action for the purpose
would only prove to be costly,
burdensome and time-consuming
for both parties and further delay

the final disposition of the case.


The multiplicity of suits must be
avoided.
With
the
implied
institution of the civil action in the
criminal action, the two actions
are merged into one composite
proceeding, with the criminal
action predominating the civil. The
prime purpose of the criminal
action is to punish the offender in
order to deter him and others from
committing the same or similar
offense, to isolate him from society,
reform and rehabilitate him or, in
general, to maintain social order.
However, any amendment before
plea, which downgrades the nature
of the offense charged in or
excludes any accused from the
complaint or information, can be
made only upon motion by the
prosecutor, with notice to the
offended party and with leave of
court. The court shall state its
reasons in resolving the motion
and copies of its order shall be
furnished all parties, especially the
offended party.
Thus, before the accused enters
his plea, a formal or substantial
amendment of the complaint or
information may be made without
leave of court. After the entry of a
plea, only a formal amendment
may be made but with leave of
court and if it does not prejudice
the rights of the accused. After
arraignment,
a
substantial
amendment is proscribed except if
the same is beneficial to the
accused.

A separate action for the


purpose would only prove to be
costly, burdensome and timeconsuming for both parties and
further
delay
the
final
disposition of the case.
The
multiplicity of suits must be
avoided.
With
the
implied
institution of the civil action in
the criminal action, the two
actions are merged into one
composite proceeding, with the
criminal action predominating
the civil. The prime purpose of
the criminal action is to punish
the offender in order to deter
him
and
others
from
committing the same or similar
offense, to isolate him from
society, reform and rehabilitate
him or, in general, to maintain
social order. (Ricarze vs CA,
G.R. No. 160451, February 9,
2007 )

Heirs of Sarah Marie Palma


Burgos vs CA G.R. No. 169711,
February 8, 2010
FACTS:
Assailants attacked the household
of Sarah Marie Palma Burgos
while all were asleep. Sarah and
her uncle Erasmo Palma were
killed. Two of the assailants were
arrested and they pointed to
respond
Johnny
co
as
the
mastermind.
After
10
years,
respondent surrendered to the NBI
and later filed a petition for
admission to bail, RTC granted bail
on the ground that the evidence of
guilt of respondent Co was not
strong. CA dismissed petitioners'
certiorari (Rule 65) for having
been filed without involving the
office of the Solicitor General
(OSG).
ISSUE:
whether or not the CA correctly
dismissed the special civil action of
certiorari, which questioned the
RTCs grant of bail to respondent
Co, for having been filed in the
name of the offended parties and
without the OSGs intervention.
HELD:
The civil action, in which the
offended party is the plaintiff and
the accused is the defendant, is
deemed
instituted
with
the
criminal action unless the offended
party waives the civil action or
reserves the right to institute it
separately or institutes the civil
action prior to the criminal action.
The law allows the merger of the
criminal and the civil actions to

avoid multiplicity of suits. Thus,


when the state succeeds in
prosecuting
the
offense,
the
offended party benefits from such
result and is able to collect the
damages awarded to him.
But, when the trial court acquits
the accused or dismisses the case
on the ground of lack of evidence
to prove the guilt of the accused
beyond reasonable doubt, the civil
action
is
not
automatically
extinguished since liability under
such an action can be determined
based on mere preponderance of
evidence. The offended party may
peel off from the terminated
criminal action and appeal from
the implied dismissal of his claim
for civil liability.

The civil action, in which the


offended party is the plaintiff
and
the
accused
is
the
defendant, is deemed instituted
with the criminal action unless
the offended party waives the
civil action or reserves the
right to institute it separately
or institutes the civil action
prior to the criminal action.
The law allows the merger of
the criminal and the civil
actions to avoid multiplicity of
suits. Thus, when the state
succeeds in prosecuting the
offense, the offended party
benefits from such result and is
able to collect the damages
awarded to him. (Heirs of Sarah
Marie Palma Burgos vs CA G.R.
No. 169711, February 8, 2010)

ABS-CBN Broadcasting Corp.


vs Ombudsman G.R. No.
133347, October 15, 2008
FACTS:

military, and detained at Fort


Bonifacio for almost five (5) years
until his escape therefrom on
September 30, 1977.

The day after the declaration of


martial law, or on September 22,
1972,
just
before
midnight,
military troops arrived at the ABSCBN Broadcast Center in Bohol
Avenue, Quezon City, and informed
the officers and personnel thereat
of the seizure and closure of the
premises by virtue of Letter of
Instruction (LOI) No. 1 issued by
President Marcos ordering the
closure of all radio and television
stations in the country.

In the years following until the


Marcos government was toppled in
1986, the ABS-CBN stations were
transferred to the National Media
Production Center (NMPC) headed
by Gregorio Cendaa of the
Ministry of Information. Starting in
January 1980, KBS, on a staggered
basis,
transferred
possession,
control and management of ABSCBNs provincial television stations
to NMPC. Some of the radio
stations of ABS-CBN were turned
over to the governments Bureau of
Broadcast,
while
some
were
retained by KBS thru the Banahaw
Broadcasting Corporation (BBC)
and Radio Philippines Network
(RPN).

LOI No. 1 authorized the Secretary


of National Defense to take over or
control, or cause the taking over
and control of all x x x newspapers,
magazines, radio and television
facilities and all other media of
communications throughout the
country. Consequently, a total of
seven (7) television stations owned
and operated by ABS-CBN were
closed down by the government.
When it became apparent that
petitioners would not be granted a
permit to re-open, ABS-CBN on
October 31, 1972, terminated the
services of all its employees, giving
each employee his/her retirement
benefits.
Corollary
thereto,
sometime in November 1972,
Eugenio Lopez, Jr., then president
of ABS-CBN, wrote then Secretary
of National Defense, Juan Ponce
Enrile, of their desire to sell ABSCBN to the government. In that
same month, however, Eugenio
Lopez, Jr. was arrested by the

Parenthetically, during a military


inventory in 1979-1980, and a visit
by ABS-CBN executives at ABSCBNs radio transmitting stations
in Meycauayan, Bulacan, headed
by petitioner Augusto, on August
13, 1984, ABS-CBN properties and
massive equipment were found to
be missing. In addition, the
musical records and radio dramas
accumulated by ABS-CBN in a
span of twenty-five (25) years and
stored in its library were now
gone.
In June 1986, President Corazon
Aquino, acting on the request of
ABS-CBN through Senator Taada,
returned to ABS-CBN these radio
and TV stations on a gradual and
scheduled basis.

ISSUE:
Whether
the
dropping
of
respondents Roberto S. Benedicto
and Salvador (Buddy) Tan as
respondents in this case due to
their death, consistent with our
rulings in People v. Bayotas and
Benedicto v. Court of Appeals

HELD:
First and foremost, there is, as yet,
no
criminal
case
against
respondents,
whether
against
those who are living or those
otherwise dead.
The question posed by petitioners
on this long-settled procedural
issue does not constitute a novel
question of law. Nowhere in People
v. Bayotas does it state that a
criminal complaint may continue
and
be
prosecuted
as
an
independent civil action. In fact,
Bayotas,
once
and
for
all,
harmonized the rules on the
extinguished and on the subsisting
liabilities of an accused who dies.
Second, and more importantly, we
dismissed
the
petition
for
certiorari filed by petitioners
because they failed to show grave
abuse of discretion on the part of
the
Ombudsman
when
he
dismissed
petitioners
criminal
complaint against respondents for
lack of probable cause. We
reiterate that our inquiry was
limited to a determination of
whether
the
Ombudsman
committed
grave
abuse
of
discretion when he found no
probable
cause
to
indict

respondents for various felonies


under the RPC. The invocation of
our certiorari jurisdiction over the
act of a constitutional officer, such
as the Ombudsman, must adhere
to the strict requirements provided
in the Rules of Court and in
jurisprudence. The determination
of whether there was grave abuse
of discretion does not, in any way,
constitute a novel question of law.

The rule on implied institution


of the civil action does not
apply before the filing of the
criminal action or information.
Hence, there is no criminal
case yet when the Ombudsman
is still in the process of finding
probable cause to prosecute
the respondents. The rule that
a
civil
action
is
deemed
instituted
in
the
criminal
action unless the offended
party waives the civil action,
reserves the right to institute it
separately, or institutes the
civil
action
prior
to
the
criminal
action,
is
not
applicable.
(ABS-CBN
Broadcasting
Corp.
vs
Ombudsman G.R. No. 133347,
October 15, 2008)

Hun Hyung Park vs Eun Wong


Choi, G.R. No. 165496,
February 12, 2007
In an Information dated August 31,
2000, respondent, Eung Won Choi,
was charged for violation of Batas
Pambansa Blg. 22, otherwise
known as the Bouncing Checks
Law, for issuing on June 28, 1999
Philippine National Bank Check
No. 0077133 postdated August 28,
1999 in the amount of P1,875,000
which was dishonored for having
been drawn against insufficient
funds.
Upon arraignment, respondent,
with the assistance of counsel,
pleaded "not guilty" to the offense
charged. Following the pre-trial
conference,
the
prosecution
presented its evidence-in-chief.
After the prosecution rested its
case, respondent filed a Motion for
Leave of Court to File Demurrer to
Evidence to which he attached his
Demurrer, asserting that the
prosecution failed to prove that he
received the notice of dishonor,
hence, the presumption of the
element
of
knowledge
of
insufficiency of funds did not arise.
By Decision, RTC held that while
the
evidence
presented
was
insufficient to prove respondents
criminal liability, it did not
altogether extinguish his civil
liability.
Upon respondents motion for
reconsideration, however, the RTC
set aside its decision and ordered
the remand of the case to the
MeTC " for further proceedings, so
that the defendant [-respondent

herein] may adduce evidence on


the civil aspect of the case."
ISSUE:
Whether respondent as a right to
present evidence on civil aspect in
view of his demurrer
HELD:
It bears recalling that the MeTC
acquitted respondent. As a rule, a
judgment
of
acquittal
is
immediately final and executory
and the prosecution cannot appeal
the acquittal because of the
constitutional prohibition against
double jeopardy.
Either the offended party or the
accused may, however, appeal the
civil aspect of the judgment
despite the acquittal of the
accused. The public prosecutor has
generally no interest in appealing
the civil aspect of a decision
acquitting
the
accused.
The
acquittal ends his work. The case
is terminated as far as he is
concerned. The real parties in
interest in the civil aspect of a
decision are the offended party
and the accused.
On the other hand, if the evidence
so far presented is insufficient as
proof beyond reasonable doubt, it
does not follow that the same
evidence is insufficient to establish
a preponderance of evidence. For
if the court grants the demurrer,
proceedings on the civil aspect of
the case generally proceeds. The
only recognized instance when an
acquittal on demurrer carries with
it the dismissal of the civil aspect
is when there is a finding that the

act or omission from which the


civil liability may arise did not
exist. Absent such determination,
trial as to the civil aspect of the
case must perforce continue.

If the filing of a separate civil


action has not been reserved or
priorly
instituted
or
the
enforcement of civil liability is
not waived, the trial court
should, in case of conviction,
state the civil liability or
damages
caused
by
the
wrongful act or omission to be
recovered from the accused by
the offended party, if there is
any.
The real parties in interest in
the civil aspect of a decision
are the offended party and the
accused.
One of the issues in a criminal
case being the civil liability of
the accused arising from the
crime, the governing law is the
Rules of Criminal Procedure,
not the Rules of Civil Procedure
which pertains to a civil action
arising from the initiatory
pleading that gives rise to the
suit. (Hun Hyung Park vs Eun
Wong Choi, G.R. No. 165496,
February 12, 2007)

When a Civil Action may


proceed independently;
independent civil actions and
quasi-delicts

the subsidiary civil liability of the


bus line becomes conclusive and
enforceable.

Philippine Rabbit Bus Lines,


Inc vs People, G.R. No. 147703,
April 14, 2004
Facts:

Whether or not an employer, who


dutifully
participated
in
the
defense of its accused-employee
may appeal the judgment of
conviction independently of the
accused?

This is a petition for review under


rule 45 of the rules of court
assailing
resolutions
of
CA.
Petitioner's
appeal
from
the
judgment of the RTC of San
Fernando, La Union in Criminal
Case No. 2535 was dismissed.
On
July
1994
accused
Macadangdang was found guilty
and convicted of the crime of
reckless imprudence resulting to
triple homicide, multiple physical
injuries and damage to property
and was sentenced to suffer the
penalty. The court ruled that rabbit
bus lines shall be liable for the civil
liabilities of the accused in the
event of the accused insolvency.
Evidently, the judgment against
the accused had become final and
executory.
Then the accused jumped bail,
worth mentioning that rule 8, rule
124 of the rules of court authorizes
the dismissal of appeal when
appellant jumps bail. The counsel
for accused hired by rabbit bus
lines filed a notice of appeal which
was denied by the trial court.
The CA ruled that the institution of
a criminal case implied the
institution also of the civil action
arising from the offense. Making

Issues:

Ruling:
Petition has no merit.
Appeal in Criminal Cases: Section
1 of rule 122 of the 2000 revised
rules of criminal procedures states
"any party may appeal from a
judgment or final order, unless the
accused will be placed in double
jeopardy"
Civil Actions are deemed instituted
in a criminal prosecution. But
rabbit bus line is not a direct party
to the criminal case. While they
may assist their employees, the
employer cannot act independently
on their own behalf, but can only
defend the accused.
No deprivation of due process:
employer became subsidiary liable
only upon proof of the employee's
insolvency and the right to appeal
was lost due to the bail of the
accused employee not the court.

Only the civil liability of the


accused arising from the crime
charged is deemed impliedly
instituted in a criminal action,
that is, unless the offended

party waives the civil action,


reserves the right to institute it
separately, or institutes it prior
to the criminal action.
It is clear that the 2000 Rules
deleted the requirement of
reserving
independent
civil
actions and allowed these to
proceed
separately
from
criminal actions. Thus, the civil
actions referred to in Articles
32, 33, 34 and 2176 of the Civil
Code shall remain separate,
distinct and independent of any
criminal prosecution based on
the
same
act.
(Philippine
Rabbit Bus Lines, Inc vs
People, G.R. No. 147703, April
14, 2004)

No Reservation of the Civil


Action in B.P. 22
Lo Bun Tiong vs Balboa G.R.
No. 158177, January 28, 2008
Facts: Vicente Balboa filed two (2)
cases against Sps. Benito Lo Bun
Tiong and Caroline Siok Ching
Teng:
(1) A CIVIL CASE for sum of
money based on the three (3) postdated checks issued by Caroline in
the total amount of P5,175,250.00.
The Regional Trial Court found the
spouses liable and ordered them to
pay the amount.
(2) A CRIMINAL CASE for
violation of Batas Pambansa Blg.
22 against Caroline covering the
said three checks. The Municipal
Trial Court acquitted Caroline but
held her civilly liable. On appeal,
the RTC modified the MTC
Decision by deleting the award of
civil damages.
The spouses now comes to court
charging Balboa with forumshopping.
Issue:
Whether or not the Balboa's act of
filing civil and criminal cases
constitute forum-shopping.
Held:
Forum shopping is the institution
of two or more actions or
proceedings grounded on the same
cause, on the supposition that one
or the other court would render a
favorable disposition. It is usually
resorted to by a party against

whom an adverse judgment or


order has been issued in one
forum, in an attempt to seek and
possibly to get a favorable opinion
in another forum, other than by an
appeal or a special civil action for
certiorari.
Under Rule 111(b) of the 2000
Revised
Rules
of
Criminal
Procedure, to wit: (b) The criminal
action for violation of Batas
Pambansa Blg. 22 shall be deemed
to include the corresponding civil
action. No reservation to file such
civil action separately shall be
allowed.
Since Balboa instituted the civil
action prior to the criminal action,
then the civil case may proceed
independently of the criminal
cases and there is no forum
shopping to speak of. Even under
the amended rules, a separate
proceeding for the recovery of civil
liability in cases of violations of
B.P. No. 22 is allowed when the
civil case is filed ahead of the
criminal case. Even then, the Rules
encourage the consolidation of the
civil and criminal cases.

A separate proceeding for the


recovery of civil liability in
cases of violation of B.P. 22 is
allowed when the civil case is
filed ahead of the criminal
action. (Lo Bun Tiong vs Balboa
G.R. No. 158177, January 28,
2008)

Hyatt Industrial Manufacturing


Corp. vs Asia Dynamic Electrix
Corp. , 465 SCRA 454
Asia
Dynamic
Electrix
Corporation (ADEC) purchased
from
Hyatt
Industrial
Manufacturing
Corporation
various electrical condults and
fittings amounting P1,622,467.14
ADEC issued several checks in
favor of Hyatt as payment but the
said checks were dishonored by
the drawee bank on the ground of
insufficient funds/account closed.
ADEC still failed to pay despite the
demand.
Hyatt filed before the RTC of
Mandaluyong City of complaint for
recovery of sum of money against
respondent Asia Dynamic Electrix
Corporation.
It
prayed
that
respondent be ordered to pay the
amount of purchase, plus interest
and attorney's fees.
However, prior to the filling
of the case for recovery of sum of
money before he said RTC,
petitioner Hyatt had already filed
separate criminal complaints for
violation of B.P. 22 against the
officers
of
Respondent
Corporation, Gil Santillan and
Juanito Pamatmat. They were both
pending before the Metropolitan
Trial Court of Pasig City. These
cases involve the same checks
which are the subjects of Civil
Case before the Mandaluyong
RTC.
ADEC moved to dismiss the
complaint
on
the
following
grounds:

(1) The civil action was


deemed included in the criminal
actions for violation of Batas
Pambansa Blg. 22 (B.P. 22)
previously filed by petitioner
against the officers of respondent
corporation.
(2) Section 1(b) of Rule 111
of the Revised Rules of Criminal
Procedure prohobits the filling of a
separate civil action in B.P. 22
cases.
RTC denied the motion to
dismiss ruling that since the act
complained of arose from the
alleged
non-payment
of
the
petitioner of its contractual debt,
and not the issuance of checks
with sufficient funds, the civil
action
could
proceed
independently of the criminal
actions pursuant to Article 31 of
the Civil Code. It said that section
1(b) of Rule 111 of the Revised
Rules of Criminal Procedure does
not apply to the obligation in this
case, it being ex-contractu and not
ex-delicto.
In a petition for certiorari,
CA reversed the order of the trial
court saying that the civil actions
deemed instituted with the filing of
criminal cases for violation of B.P.
22 and the Civil Cases instituted
are of the same nature (for sum of
money between the same parties
for the same transaction).
Respondent ADEC filed a
motion for reconsideration but it
was denied by the CA.
Respondent ADEC filed a
petition in the SC contending that
it did not violate Section 1(b) of

Rule 111 of the Revised Rules on


Criminal Procedure when it filed
the complaint in Civil Case No. MC
01-1493.
ISSUE:
Whether a separate civil action for
violations of BP 22 is allowed
RULING:
The petition is unmeritorious:
SC upheld the ruling of the CA that
upon filing of the criminal cases
for violation of B.P 22 the civil
action for the recovery of the
amount of the checks was also
impliedly instituted under Section
1(b) of Rule 111 of the 2000 Rules
in Criminal Procedures. Under the
present revised Rules, the criminal
action for violation of B.P. 22 shall
be
deemed
to
include
the
corresponding civil action. The
reservation to file a separate civil
action is no longer needed.
Section
1(b)
of
Rule
111
specifically states that the criminal
action for violation of B.P. 22 shall
be
deemed
to
include
the
corresponding civil action.

The filing of the criminal cases


for violation of B.P. 22, the civil
action for the recovery of the
amount of the checks was also
impliedly
instituted
under
Section 1(b) of Rule 111 of the
2000
Rules
on
Criminal
Procedure. Under the present
revised Rules, the criminal
action for violation of B.P. 22
shall be deemed to include the
corresponding civil action. The

reservation to file a separate


civil action is no longer needed.
The
only
instance
when
separate
proceedings
are
allowed is when the civil action
is filed ahead of the criminal
case.
(Hyatt
Industrial
Manufacturing Corp. vs Asia
Dynamic Electrix Corp. , 465
SCRA 454)

When no Reservation is
required; when civil action is
not suspended
Casupanan vs Laroya, 388
SCRA 28
Two vehicles, one driven by
respondent Mario Llavore Laroya
and the other owned by petitioner
Roberto Capitulo and driven by
petitioner
Avelino
Casupanan,
figured in an accident. As a result,
two cases were filed with the
MCTC of Capas, Tarlac. Laroya
filed a criminal case against
Casupanan
for
reckless
imprudence resulting in damage to
property. On the other hand,
Casupanan and Capitulo filed a
civil case against Laroya for quasidelict.
When the civil case was filed, the
criminal case was then at its
preliminary investigation stage.
Laroya, defendant in the civil case,
filed a motion to dismiss the civil
case on the ground of forumshopping
considering
the
pendency of the criminal case. The
MCTC granted the motion and
dismissed the civil case.
On Motion for Reconsideration,
Casupanan and Capitulo insisted
that the civil case is a separate
civil action which can proceed
independently of the criminal case.
The MCTC denied the motion for
reconsideration. Casupanan and
Capitulo filed a petition for
certiorari under Rule 65 before the
Regional Trial Court of Capas,
Tarlac assailing the MCTCs Order
of dismissal.

ISSUE:
Whether an accused in a pending
criminal
case
for
reckless
imprudence
can
validly
file,
simultaneously and independently,
a separate civil action for quasidelict
against
the
private
complainant in the criminal case.
HELD:
Under Section 1 of the present
Rule 111, the independent civil
action in Articles 32, 33, 34 and
2176 of the Civil Code is not
deemed
instituted
with
the
criminal action but may be filed
separately by the offended party
even without reservation. The
commencement of the criminal
action does not suspend the
prosecution of the independent
civil action under these articles of
the Civil Code. The suspension in
Section 2 of the present Rule 111
refers only to the civil action
arising from the crime, if such civil
action is reserved or filed before
the commencement of the criminal
action.
Thus, the offended party can file
two separate suits for the same act
or omission. The first a criminal
case where the civil action to
recover civil liability ex-delicto is
deemed instituted, and the other a
civil case for quasi-delict - without
violating the rule on non-forum
shopping. The two cases can
proceed
simultaneously
and
independently of each other. The
commencement or prosecution of
the criminal action will not
suspend the civil action for quasi-

delict. The only limitation is that


the offended party cannot recover
damages twice for the same act or
omission of the defendant. In most
cases, the offended party will have
no reason to file a second civil
action since he cannot recover
damages twice for the same act or
omission of the accused. In some
instances, the accused may be
insolvent, necessitating the filing
of another case against his
employer or guardians.

Under Section 1 of the present


Rule 111, what is deemed
instituted with the criminal
action is only the action to
recover civil liability arising
from the crime or ex-delicto. All
the other civil actions under
Articles 32, 33, 34 and 2176 of
the Civil Code are no longer
deemed instituted, and may be
filed separately and prosecuted
independently even without any
reservation in the criminal
action. The failure to make a
reservation in the criminal
action is not a waiver of the
right to file a separate and
independent civil action based
on these articles of the Civil
Code. (Casupanan vs Laroya,
388 SCRA 28)

Counterclaim, crosscounterclaim, third-party claim


in a criminal action
Maccay vs Nobela, 454 SCRA
504
FACTS:
In the first week of May, 1990,
Adelaida E. Potenciano went to the
public market of Pasig, Metro
Manila, to look for a prospective
buyer or mortgagee of a parcel of
land belonging to Oscar Maccay.
She was introduced by a vendor,
Lydia Reyes, to the spouses
Prudencio and Serlina Nobela who
were engaged in lending money to
market vendors on a daily basis.
Potenciano introduced herself as
Angelita N. Barba, wife of Oscar
Maccay, who desired to sell or
mortgage any of his two parcels of
land, one in Guadalupe and one in
Antipolo. She went to the Nobelas
at
145
Buayang
Bato
at
Mandaluyong, Metro Manila. She
brought with her many titles. She
became friendly with the spouses.
Potenciano went on to brag about
her connections, that she is related
to the late President Ferdinand E.
Marcos; and that the PCGG is after
her so she has to dispose of her
properties.
After two (2) days, she called
Oscar Maccay, who came. They
comported themselves as husband
and wife. Maccay was in uniform.
He is a police colonel who had
jurisdiction over Mandaluyong,
according to Potenciano. The
Nobelas were impressed. They
were pleased when the couple

became very close to them. They


confided their family problems.
They even went to the office of
Maccay in Fort Bonifacio.
In the meantime, Prudencio and
Serlina, who had not been able to
register the sale to them because
of the ailment of Prudencio asked
a real estate agent, Anita de la
Vega, to help them in the
registration of Deed of Sale (Exh.
1, Exh. B.). They knew de la Vega
as she used to frequent a real
estate agent living in their place.
When they were told that for the
P300,000.00 consideration, they
would need around P20,000.00 to
include capital gains taxes, she
gave P21,000.00. The mother of de
la Vega was supposed to know
many people in the Register of
Deeds. The new title (Exh. C) was
delivered on August 10, 1990 to
Serlina. She had to give an
additional 2,000.00 to de la Vega
for other expenses.
ISSUE:
WHETHER THE TRIAL COURT
MAY RULE ON THE CIVIL
LIABILITY OF COMPLAINANT IN
A CRIMINAL CASE WHERE THE
CIVIL
ACTION
WAS
NOT
RESERVED
OR
FILED
SEPARATELY
HELD:
A court trying a criminal case
cannot award damages in favor of
the accused. The task of the trial
court is limited to determining the
guilt of the accused and if proper,
to determine his civil liability. A

criminal case is not the proper


proceedings to determine the
private complainants civil liability,
if any.
The appellate court erred in
affirming the trial courts award of
damages by justifying it as a
counterclaim. No counterclaim,
cross-claim
or
third-party
complaint may be filed by the
accused in the criminal case, but
any cause of action which could
have been the subject thereof may
be litigated in a separate civil
action. Nothing in the records
shows that respondent spouses
filed or attempted to file a
counterclaim. Section 1 of Rule
111
of Rules on Criminal
Procedure prohibit counterclaims
in criminal cases.

No counterclaim, cross-claim
or third-party complaint may be
filed by the accused in the
criminal case, but any cause of
action which could have been
the subject thereof may be
litigated in a separate civil
action. (Maccay vs Nobela, 454
SCRA 504)

Effect of Death of the Accused


on the Civil Action
People vs Bayotas, G.R. No.
102007, September 4, 1994
FACTS
The accused, Rogelio Bayotas, was
charged with rape and eventually.
Pending appeal of his conviction
the accused died. The findings of
the National Bilibid Hospital
declared him dead on February 4,
1992.
The
Supreme
Court
in
its
resolution dated May 20, 1992
dismissed the criminal aspect of
the appeal but then required the
Solicitor General to file its
comment on Bayotas civil liability
arising from the crime. In the
Solicitor Generals comment the
civil liability hasnt yet expired.
The solicitor general based its
judgment on the case of People vs.
Sendaydiego. The counsel of the
accused-appellant had a different
view; where the death occurred
after final judgment the criminal
and
civil
liability
shall
be
extinguished.
ISSUE
Whether or not the death of
Bayotas extinguished his criminal
and civil liability.
RULING
The Supreme Court ruled in favor
of the accused. According to the
Supreme Court, the controlling
statute was Article 89 of the
Revised Penal Code. The provision
states that death extinguishes the

criminal aspect. In the case at bar,


there was no reservation of an
independent civil action against
the accused; the criminal and civil
aspects are therefore considered
as instituted in the criminal action.
Since the civil action was anchored
with the criminal case then it
follows that the death dissolves
both civil and criminal liability.
The
Solicitor
Generals
dependence on the Sendaydiego
case was misplaced. What was
contemplated in the Sendaydiego
case was the civil liability arising
from other sources of obligation
other than delicts. It is therefore
safe to say that what death
extinguishes is criminal liability
and civil liability arising from
delict only.

the death of the accused prior


to final judgment terminates
his criminal liability and only
the
civil
liability
directly
arising from and based solely
on the offense committed, i.e.,
civil liability ex delicto in senso
strictiore. (People vs Bayotas,
G.R. No. 102007, September 4,
1994)

People vs Bringas Bunay y


Dam-at, G.R. No. 171268,
September 14, 2010
FACTS:
The RTC in Luna, Apayao tried and
found the accused guilty of
qualified rape.
On April 20, 2010, the Court
received the letter from Bureau of
Corrections
Assistant
Director
advising that the accused had died
on March 25, 2010 at the New
Bilibid Prison Hospital. The report
of Dr. Marylou V. Arbatin, revealed
that the immediate cause of death
had been cardio-respiratory arrest,
with pneumonia as the antecedent
cause.
On June 22, 2010, the Court
required the Bureau of Corrections
to submit a certified true copy of
the death certificate of the
accused.
By letter dated August 16, 2010,
Armando
T.
Miranda,
Chief
Superintendent of the New Bilibid
Prison,
submitted
the
death
certificate of the accused.
Under
the
foregoing
circumstances, the death of the
accused during the pendency of
his appeal in this Court totally
extinguished his criminal liability.
ISSUE:
Whether or not the death of
accused extinguished his criminal
and civil liability.
HELD:
The death of the accused during
the pendency of his appeal with

the
Supreme
Court
totally
extinguished his criminal liability.
Such extinction is based on Article
89 of the Revised Penal Code. The
death of the accused likewise
extinguished the civil liability that
was based exclusively on the crime
for which the accused was
convicted (i.e., ex delicto), because
no final judgment of conviction
was yet rendered by the time of his
death.
Only
civil
liability
predicated
on
a
source
of
obligation other than the delict
survived the death of the accused,
which the offended party can
recover by means of a separate
civil action.

Only civil liability predicated on


a source of obligation other
than the delict survived the
death of the accused, which the
offended party can recover by
means of a separate civil
action. (People vs Bringas
Bunay y Dam-at, G.R. No.
171268, September 14, 2010)

People vs Jaime Ayochok y


Tauli, G.R. No. 175784, August
25, 2010
FACTS:
That on or about the 15th day
of July, 2001, in the City of Baguio,
Philippines, accused, being then
armed with a gun, with intent to
kill and with evident premeditation
and by means of treachery and
with cruelty by deliberately and
inhumanly outraging at the victim,
did then and there willfully,
unlawfully and feloniously attack,
assault and shoot SPO1 CLAUDIO
CALIGTAN y NGODO in the
following manner, to wit: that
while the victim was relieving
himself with his back turned to the
accused, the latter coming from
the blind side of the victim, shoot
him several times hitting him on
the different parts of his body and
there was no opportunity or means
to defend himself from the
treacherous act of the assailant,
thereby inflicting upon the latter:
hypovolemic shock due to massive
hemorrhage;
multiple
gunshot
wounds on the head, neck, and
upper extremities which directly
caused his death. When arraigned,
Ayochok pleaded not guilty.
After trial on the merits of
Criminal Case, the RTC rendered a
Decision finding the accused guilty
beyond reasonable doubt of the
offense of Murder, under Article
248 of the Revised Penal Code as
amended, qualified by treachery as
charged in the Information and
hereby sentences him to reclusion
perpetua. The accused being a
detention prisoner is entitled to be

credited 4/5 of his preventive


imprisonment in the service of his
sentence in accordance with
Article 29 of the Revised Penal
Code.
However, in a letter dated
February 16, 2010, the Assistant
Director for Prisons and Security
of the Bureau of Corrections,
informed us that Ayochok had died
at the Philippine General Hospital,
Manila. A copy of the death report
signed by a medical officer of the
New Bilibid Prison Hospital was
attached to said letter.
ISSUES:
Whether or not he was guilty
of the crime charged has become
irrelevant since, following Article
89(1) of the Revised Penal Code,
assuming Ayochok had incurred
any criminal liability, it was totally
extinguished
by
his
death.
Moreover,
because
Ayochok's
appeal was still pending and no
final judgment of conviction had
been rendered against him when
he died, his civil liability arising
from the crime, being civil liability
ex
delicto,
was
likewise
extinguished by his death.
RULING:
Clearly, in view of a supervening
event, it is unnecessary for the
Court to rule on Ayochoks appeal.
Whether or not he was guilty of
the crime charged has become
irrelevant since, following Article
89(1) of the Revised Penal Code
and our disquisition in Bayotas,
even
assuming
Ayochok
had
incurred any criminal liability, it

was totally extinguished by his


death.
Moreover,
because
Ayochoks appeal was still pending
and no final judgment of conviction
had been rendered against him
when he died, his civil liability
arising from the crime, being civil
liability ex delicto, was likewise
extinguished by his death.

The death of the accused


pending
appeal
of
his
conviction
extinguishes
his
criminal liability and the civil
liability based solely thereon.
(People vs Jaime Ayochok y
Tauli, G.R. No. 175784, August
25, 2010)

Novation: Extiguishment of
Criminal Liability
Metropolitan Bank and Trust
Company vs Rogelio Reynaldo
and Jose Adrandea, G.R. No.
164538, August 9, 2010
FACTS:
On January 31, 1997, petitioner
Metropolitan Bank and Trust
Company charged respondents
before the Office of the City
Prosecutor of Manila with the
crime of estafa under Article 315,
paragraph 1(b) of the Revised
Penal Code. In the affidavit of
petitioners audit officer, Antonio
Ivan S. Aguirre, it was alleged that
the special audit conducted on the
cash and lending operations of its
Port
Area
branch
uncovered
anomalous/fraudulent transactions
perpetrated by respondents in
connivance with client Universal
Converter
Philippines,
Inc.
(Universal); that respondents were
the only voting members of the
branchs
credit
committee
authorized
to
extend
credit
accommodation to clients up to
P200,000.00; that through the socalled Bills Purchase Transaction,
Universal, which has a paid-up
capital of only P125,000.00 and
actual maintaining balance of
P5,000.00, was able to make
withdrawals
totaling
P81,652,000.00 against uncleared
regional checks deposited in its
account at petitioners Port Area
branch;
that,
consequently,
Universal was able to utilize
petitioners funds even before the
seven-day clearing period for

regional checks expired; that


Universals withdrawals against
uncleared regional check deposits
were without prior approval of
petitioners head office; that the
uncleared
checks
were
later
dishonored by the drawee bank for
the reason Account Closed; and,
that respondents acted with fraud,
deceit, and abuse of confidence.
In their defense, respondents
denied
responsibility
in
the
anomalous
transactions
with
Universal and claimed that they
only intended to help the Port Area
branch solicit and increase its
deposit
accounts
and
daily
transactions.
Meanwhile, on February 26, 1997,
petitioner and Universal entered
into a Debt Settlement Agreement
whereby the latter acknowledged
its indebtedness to the former in
the
total
amount
of
P50,990,976.27[8] as of February
4, 1997 and undertook to pay the
same in bi-monthly amortizations
in the sum of P300,000.00 starting
January 15, 1997, covered by
postdated checks, plus balloon
payment of the remaining principal
balance and interest and other
charges, if any, on December 31,
2001.
ISSUE:
Whether novation and undertaking
to pay the amount embezzled do
not extinguish criminal liability
HELD:
Initially, it is best to emphasize
that novation is not one of the

grounds prescribed by the Revised


Penal Code for the extinguishment
of criminal liability.
Thus, the doctrine is that a
compromise or settlement entered
into after the commission of the
crime
does
not
extinguish
accuseds
liability
for
estafa.
Neither will the same bar the
prosecution
of
said
crime.
Accordingly, in such a situation, as
in this case, the complaint for
estafa against respondents should
not be dismissed just because
petitioner entered into a Debt
Settlement
Agreement
with
Universal.
Novation is not one of the
grounds prescribed by the
Revised Penal Code for the
extinguishment
of
criminal
liability.
It is a hornbook doctrine in our
criminal law that the criminal
liability for estafa is not
affected by a compromise, for it
is a public offense which must
be prosecuted and punished by
the government on its own
motion, even though complete
reparation [has] been made of
the damage suffered by the
private offended party. Since a
criminal offense like estafa is
committed against the State,
the private offended party may
not waive or extinguish the
criminal liability that the law
imposes for the commission of
the crime. (Metropolitan Bank
and Trust Company vs Rogelio
Reynaldo and Jose Adrandea,
G.R. No. 164538, August 9,
2010)

Effect of Acquittal or the


Extinction of the Penal Action
on the Civil Action or Civil
Liability
Ching vs Nicdao, 522 SCRA 316
FACTS:
The respondent Clarita Nicdao was
charged of 11 counts of the
violation of B.P. 22 by petitioner
Samson Ching. 11 Informations
were filed with the MCTC of
Dinalupihan-Hermosa, Bataan. At
about the same time, 14 other
criminal complaints, also for
violation of B.P. 22, were filed
against the respondent by Emma
Nuguid, said to be the common
law spouse of petitioner. The
MCTC
rendered
judgment
convicting the respondent of 11
counts of violation of B.P. 22. On
appeal, the RTC of Dinalupihan,
Bataan affirmed in toto the
decisions of the MCTC convicting
the respondent. The respondent
filed an appeal to the CA. The CA
reversed
the
decision
and
acquitted
the
accused.
The
petitioner appealed the civil aspect
of the case to the SC. The
petitioner vigorously argued that
notwithstanding
respondents
acquittal by the CA, the SC has the
jurisdiction
and
authority
to
resolve and rule on her civil
liability. The petitioner anchored
his contention on Rule 111, Sec
1B: The criminal action for
violation of B.P. 22 shall be deemed
to
necessarily
include
the
corresponding civil action, and no
reservation to file such civil action
separately shall be allowed or

recognized. Moreover, under the


above-quoted
provision,
the
criminal action for violation of B.P.
22
necessarily
includes
the
corresponding civil action, which
is the recovery of the amount of
the dishonored check representing
the civil obligation of the drawer to
the payee. The defendant argued
that Sec 2 of Rule 111 Except in
the cases provided for in Section 3
hereof, after the criminal action
has been commenced, the civil
action which has been reserved
cannot be instituted until final
judgment in the criminal action.
According to her, CAs decision is
equivalent to a finding that the
facts upon which her civil liability
may arise do not exist. The instant
petition, which seeks to enforce
her civil liability based on the 11
checks, is thus allegedly already
barred by the final and executory
decision acquitting her.

ISSUE:
1. Whether or not the SC has
jurisdiction
and
authority
resolve
and
rule
on
respondents civil liability after
acquittal of the respondent in
CA?

the
to
the
the
the

HELD:
Notwithstanding
respondent
Nicdaos
acquittal,
petitioner
Ching is entitled to appeal the civil
aspect of the case within the
reglementary period.

It is axiomatic that "every person


criminally liable for a felony is also
civilly liable." Under the pertinent
provision of the Revised Rules of
Court, the civil action is generally
impliedly
instituted
with
the
criminal action. At the time of
petitioner Chings filing of the
Informations against respondent
Nicdao, Section 1 Rule 111 of the
Revised Rules of Court, quoted
earlier, provided in part:
SEC. 1. Institution of criminal and
civil actions. When a criminal
action is instituted, the civil action
for the recovery of civil liability is
impliedly
instituted
with
the
criminal
action,
unless
the
offended party waives the civil
action, reserves his right to
institute it separately, or institutes
the civil action prior to the
criminal action.
Petitioner Ching correctly argued
that he, as the offended party, may
appeal the civil aspect of the case
notwithstanding
respondent
Nicdaos acquittal by the CA. The
civil action was impliedly instituted
with the criminal action since he
did not reserve his right to
institute it separately nor did he
institute the civil action prior to
the criminal action.
If the accused is acquitted on
reasonable doubt but the court
renders judgment on the civil
aspect of the criminal case, the
prosecution cannot appeal from
the judgment of acquittal as it
would place the accused in double
jeopardy. However, the aggrieved
party, the offended party or the
accused or both may appeal from

the judgment on the civil aspect of


the case within the period therefor.
A painstaking review of the case
leads to the conclusion that
respondent
Nicdaos
acquittal
likewise carried with it the
extinction of the action to enforce
her civil liability. There is simply
no basis to hold respondent Nicdao
civilly liable to petitioner Ching.
CAs
acquittal
of
respondent
Nicdao is not merely based on
reasonable doubt. Rather, it is
based on the finding that she did
not commit the act penalized
under BP 22. In particular, the CA
found that the P20,000,000.00
check was a stolen check which
was never issued nor delivered by
respondent Nicdao to petitioner
Ching.
CA did not adjudge her to be civilly
liable to petitioner Ching. In fact,
the CA explicitly stated that she
had
already
fully
paid
her
obligations. The finding relative to
the P20,000,000.00 check that it
was a stolen check necessarily
absolved respondent Nicdao of any
civil liability thereon as well.
Under the circumstances which
have just been discussed lengthily,
such acquittal carried with it the
extinction of her civil liability as
well.

In
case
of
acquittal,
the
accused may still be adjudged
civilly liable. The extinction of
the penal action does not carry
with it the extinction of the
civil action where; (a) the

acquittal
is
based
on
reasonable
doubt
as
only
preponderance of evidence is
required; (b) the court declares
that the liability of the accused
is only civil; and (c) the civil
liability of the accused does not
arise from or is not based upon
the crime of which the accused
was
acquitted.
(Ching
vs
Nicdao, 522 SCRA 316)

Bax vs People, 532 SCRA 284


FACTS:

14355-14363
and
the
above
Information are similarly worded.

Challenged in the instant Petition


for Review on Certiorari is the
Decision of the Court of Appeals
affirming in toto the Decision
dated December 14, 1998 of the
Regional Trial Court declaring
petitioner guilty of nine counts of
violations of B.P. 22, otherwise
known as the Bouncing Checks
Law.

The MeTC rendered a Decision


finding petitioner guilty as charged
of the crime of Violations of Batas
Pambansa Bilang 22, (10) counts.
RTC modified its decision and
rendered him guilty of violation of
BP 22, (9) counts. The CA affirmed
in toto the RTC decision

That on or about the 13th day of


March 1994 the above-named
accused, did then and there
willfully, unlawfully and feloniously
make or draw and issue to Ilyon
Industrial Corporation amounting
to P47, 250.
Said accused well knowing that at
the time of issue he did not have
sufficient funds in or credit with
the drawee bank for the payment
in full of the face amount of such
check upon its presentment, which
check could have been dishonored
for insufficiency of funds had not
the accused, without any valid
reason, ordered the bank to "Stop
Payment", and despite receipt of
notice of such dishonor, the
accused failed to pay said payee
the face amount of the said check
or made arrangement for full
payment thereof within five (5)
banking days after receiving
notice. CONTRARY TO LAW.
Except as to the numbers and
dates of the other nine checks
issued by petitioner, and the
reason for their dishonor (drawn
against insufficient funds), the
Information in Criminal Cases Nos.

Hence the instant petition.


ISSUE: whether the prosecution
was able to prove the guilt of
petitioner by evidence beyond
reasonable doubt
HELD/RACIO DECIDENDI:
We REVERSE the Decision of the
Court
of
Appeals.
Petitioner
Francisco M. Bax is acquitted in
Criminal Cases Nos. 14355 to
14363 for violations of B.P. 22 for
failure of the prosecution to prove
his guilt beyond reasonable doubt.
We find that the prosecution failed
to prove by evidence beyond
reasonable doubt that petitioner is
guilty of violations of B.P. 22.
the prosecution must prove the
following essential elements of the
offense:
(1) the making, drawing, and
issuance of any check to apply for
account or for value;
(2) the knowledge of the maker,
drawer, or issuer that at the time
of issue there are no sufficient
funds in or credit with the drawee
bank for the payment of such
check in full upon its presentment;
and

(3) the subsequent dishonor of the


check by the drawee bank for
insufficiency of funds or credit or
dishonor for the same reason had
not the drawer, without any valid
cause, ordered the bank to stop
payment.6
We find that the prosecution failed
to prove the second element.
To hold petitioner liable for
violation of B.P. 22, it is not enough
that
the
issued
check
was
subsequently
dishonored
for
insufficiency of funds. It must be
shown beyond reasonable doubt
that he knew of the insufficiency of
funds at the time the check was
issued. Hence, the law provides
that he must be notified of the
dishonor.
While it is true that ILYON,
through its president, Benedict
Tan, asked petitioner to pay the
dishonored checks, however, such
kind of notice is not the one
required by B.P. 22.
Under B.P. 22, the prosecution
must prove not only that the
accused issued a check that was
subsequently dishonored. It must
also establish that the accused was
actually notified in writing that the
check was dishonored, and that he
or she failed, within five banking
days from receipt of the notice, to
pay the holder of the check the
amount due thereon or to make
arrangement for its payment.
Absent proof that the accused
received
such
notice,
a
prosecution for violation of the

Bouncing
prosper.

Checks

Law

cannot

Since petitioner did not receive a


written notice of dishonor of the
checks, obviously, there is no way
of determining when the 5-day
period prescribed in Section 2 of
B.P. 22 would start and end. Thus,
the prima facie evidence of
petitioners knowledge of the
insufficiency of funds or credit at
the time he issued the checks did
not arise.
It is well settled that the civil
liability is not extinguished by
acquittal where such acquittal
is based on lack of proof
beyond reasonable doubt, since
only preponderance of evidence
is required in civil cases. (Bax
vs People, 532 SCRA 284)

Heirs of Sarah Marie Palma


Burgos, G.R. No. 169711,
February 28, 2010
FACTS:
Assailants attacked the household
of Sarah Marie Palma Burgos
while all were asleep. Sarah and
her uncle Erasmo Palma were
killed. Two of the assailants were
arrested and they pointed to
respond
Johnny
co
as
the
mastermind.
After
10
years,
respondent surrendered to the NBI
and later filed a petition for
admission to bail, RTC granted bail
on the ground that the evidence of
guilt of respondent Co was not
strong. CA dismissed petitioners'
certiorari (Rule 65) for having
been filed without involving the
office of the Solicitor General
(OSG).

ISSUE:
whether or not the CA correctly
dismissed the special civil action of
certiorari, which questioned the
RTCs grant of bail to respondent
Co, for having been filed in the
name of the offended parties and
without the OSGs intervention.

HELD:
The civil action, in which the
offended party is the plaintiff and
the accused is the defendant, is
deemed
instituted
with
the
criminal action unless the offended
party waives the civil action or
reserves the right to institute it

separately or institutes the civil


action prior to the criminal action.
The law allows the merger of the
criminal and the civil actions to
avoid multiplicity of suits. Thus,
when the state succeeds in
prosecuting
the
offense,
the
offended party benefits from such
result and is able to collect the
damages awarded to him.
But, when the trial court acquits
the accused or dismisses the case
on the ground of lack of evidence
to prove the guilt of the accused
beyond reasonable doubt, the civil
action
is
not
automatically
extinguished since liability under
such an action can be determined
based on mere preponderance of
evidence. The offended party may
peel off from the terminated
criminal action and appeal from
the implied dismissal of his claim
for civil liability.

The
civil
action
is
not
automatically
extinguished
since liability under such an
action can be determined based
on mere preponderance of
evidence. The offended party
may
peel
off
from
the
terminated criminal action and
appeal
from
the
implied
dismissal of his claim for civil
liability. (Heirs of Sarah Marie
Palma
Burgos,
G.R.
No.
169711, February 28, 2010)

Ramon Garces vs Simplicio


Hernandez, et al, G.R. No.
180761, August 18, 2010
It is stated that the guilt of an
accused rests solely on the
strength
of
the
Prosecutions
evidence and does not depend on
the weakness of the evidence of
the Defense. Moreover, such guilt
must
be
proven
beyond
a
reasonable doubt.
Moreover, it is noted that not one
of the accused went into hiding
even though they have acquired
knowledge about the death of
Rustico. Instead, Simplicio Sr.,
Candido and Simplicio Hernandez
Jr. voluntarily went with the police
investigators on the night of
August 13, 2000. As the oft
repeated dictum states []the guilty
fleeth while the innocent is as
brave as a lion. And, with respect
to accused Rosita Hernandez, she
appears to have been arrested in
Cuta, Batangas City. She must
have been visiting her husband
and children at the Provincial Jail
of Batangas located in Cuta,
Batangas City on March 5, 2000
when
it
happened.
These
actuations
of
the
accused
eloquently
speak
of
their
innocence in the face of unreliable
evidence
presented
by
the
Prosecution.
In the present case, there is no
dispute that the judgment of the
trial court acquitting private
respondents is already final. What
petitioner is assailing is the failure
of public respondent to rule on the
civil
liability
of
private
respondents. However, while an

appeal appears to have been open


and available, petitioner, without
any justifiable reason, did not
resort to this remedy. This is a
fatal procedural lapse. Section 1,
Rule 65 of the Rules of Civil
Procedure
is
plain
and
unambiguous in providing that the
remedy of certiorari may be
availed of only when there is no
appeal, nor any plain, speedy, and
adequate remedy in the ordinary
course of law.
ISSUE:
WHETHER
THE
COURT
OF
APPEALS COMMITTED GRAVE
AND REVERSIBLE ERROR WHEN
IT DENIED THE PETITION FOR
CERTIORARI
CONSIDERING
THAT THERE IS MORE THAN A
PREPONDERANCE OF EVIDENCE
ON RECORD SUPPORTING THE
CLAIMS OF PETITIONER GARCES
AGAINST
THE
PRIVATE
RESPONDENTS
HELD:
In his Petition for Certiorari before
the appellate court, petitioner
admitted that he did not waive the
civil action or reserve the right to
institute it separately nor did he
institute the civil action prior to
the criminal action. Petitioners
remedy then was, as correctly
ruled by the appellate court, to
appeal within the reglementary
period the trial courts decision,
which was silent on the civil aspect
of the case.
A trial court, in case of acquittal of
an accused, is to state whether the
prosecution absolutely failed to

prove his guilt or merely failed to


prove his guilt beyond reasonable
doubt, and in either case, it shall
determine if the act or omission
from which the civil liability might
arise did not exist.

Under Sec. 2 of Rule 120, a


trial
court,
in
case
the
judgment is of acquittal, it
shall
state
whether
the
evidence of the prosecution
absolutely failed to prove the
guilt of the accused or merely
failed to prove his guilt beyond
reasonable doubt. In either
case,
the
judgment
shall
determine
if
the
act
or
omission from which the civil
liability might arise did not
exist.
(Ramon
Garces
vs
Simplicio Hernandez, et al, G.R.
No. 180761, August 18, 2010)

Romero vs People, G.R. No.


167546, July 17, 2009
FACTS:
On April 1, 19992 at around 12:00
noon, the JC Liner driven by
petitioner Sonny Romero and the
Apego Taxi driven by Jimmy Padua
figured in a head-on collision along
Governor
Jose
Fuentebella
Highway at Barangay Hibago,
Ocampo, Camarines Sur. The bus
was bound for Naga City while the
taxi was going in the opposite
direction of Partido Area. The
collision resulted in the death of
Gerardo
Breis,
Sr.,5
Arnaldo
Breis,6 Gerardo Breis, Jr., Rene
Montes, Erwin Breis and Jimmy
Padua. Luckily, Edwin Breis and
his son Edmund Breis survived
although they sustained serious
injuries.
ISSUE:
Whether he should be totally
exonerated from any liability.
HELD:
The rule is that the acquittal of an
accused of the crime charged will
not necessarily extinguish his civil
liability, unless the court declares
in a final judgment that the fact
from which the civil liability might
arise did not exist. Courts can
acquit an accused on reasonable
doubt but still order payment of
civil damages in the same case. It
is not even necessary that a
separate civil action be instituted.
Thus, the MTC (as affirmed by the
RTC and the CA) correctly imposed
civil liability on petitioner despite
his acquittal. Simple logic also

dictates that petitioner would not


have been held civilly liable if his
act from which the civil liability
had arisen did not in fact exist.

The failure of the evidence to


prove negligence with moral
certainty does not negate (and
is in fact compatible with) a
ruling
that
there
was
preponderant evidence of such
negligence.
And
that
is
sufficient to hold him civilly
liable. (Romero vs People, G.R.
No. 167546, July 17, 2009)

Effect of Payment of Civil


Liability
Cabico vs DimaculanganQuerijero, 522 SCRA 300
FACTS:
This is an administrative complaint
filed by spouses Rodolfo and Sylvia
Cabico
(complainants)
against
Judge Evelyn L. DimaculanganQuerijero
(respondent
Judge),
Presiding Judge, Regional Trial
Court of Cabanatuan City, Branch
26 (trial court). Complainants
charged respondent Judge with
ignorance of the law, abuse of
authority, and conduct unbecoming
a trial court judge.
ISSUE:
Whether
Respondent
judge
committed gross ignorance of the
law when she issued the order
dated 12 October 2001, dismissing
the criminal case with prejudice
against both accused after the
latter had paid their individual civil
liability.
HELD:
When a law or a rule is basic, a
judge owes it to his office to simply
apply the law. Anything less is
gross ignorance of the law. As an
advocate of justice and a visible
representation of the law, a judge
is expected to keep abreast with
and
be
proficient
in
the
interpretation of our laws. A judge
should be acquainted with legal
norms and precepts as well as with
statutes and procedural rules.
Having accepted the exalted

position of a judge, respondent


Judge owes the public and the
court she sits in proficiency in the
law. Respondent Judge failed to
live up to these standards.
Also, respondent Judge dismissed
with prejudice the criminal case
against Dela Rosa even when he
had not been arraigned. By her
own admission, respondent Judge
made an oversight that accused
Dela Rosa was already under the
jurisdiction of the court. Thus, in
her order of 21 November 2001,
respondent Judge ordered the
issuance of a warrant of arrest
against Dela Rosa.
Respondent Judges actuations in
the premises only betray her gross
ignorance of procedural rules.
Jurisdiction over the person of the
accused is acquired by arrest.

Payment of civil liability dies


not
extinguish
criminal
liability.
(Cabico
vs
Dimaculangan-Querijero,
522
SCRA 300)

Subsidiary Liability of Employer


Rolito Calang vs People, G.R.
No. 190696, August 3, 2010
Facts:
Calang is a bus driver of Philtranco
who was convicted of multiple
homicide with multiple serious
physical injuries and damage to
property
thru
reckless
imprudence. In the same criminal
case, Philtranco was ordered to
pay jointly and severally with
Calang death indemnity and actual
damages.
Issue:
Whether or not Philtranco should
be solidarily liable in the criminal
case.
Held:
No. Calang was charged criminally
before the RTC. Undisputedly,
Philtranco was not a direct party in
this case. Since the cause of action
against Calang was based on
delict, both the RTC and the CA
erred in holding Philtranco jointly
and severally liable with Calang,
based
on
quasi-delict
under
Articles 2176 and 2180 of the Civil
Code. Articles 2176 and 2180 of
the Civil Code pertain to the
vicarious liability of an employer
for quasi-delicts that an employee
has committed. Such provision of
law does not apply to civil liability
arising from delict.
If at all, Philtranco liability may
only be subsidiary. Article 102 of
the Revised Penal Code states the
subsidiary
civil
liabilities
of

innkeepers, tavern keepers and


proprietors of establishments. The
provisions of the Revised Penal
Code
on
subsidiary
liability
Articles 102 and 103 are deemed
written into the judgments in cases
to which they are applicable. Thus,
in the dispositive portion of its
decision, the trial court need not
expressly
pronounce
the
subsidiary liability of the employer.
Nonetheless, before the employer's
subsidiary liability is enforced,
adequate evidence must exist
establishing that (1) they are
indeed the employers of the
convicted employees; (2) they are
engaged in some kind of industry;
(3) the crime was committed by
the employees in the discharge of
their duties; and (4) the execution
against the latter has not been
satisfied due to insolvency. The
determination of these conditions
may be done in the same criminal
action in which the employee
liability, criminal and civil, has
been pronounced, in a hearing set
for that precise purpose, with due
notice to the employer, as part of
the proceedings for the execution
of the judgment.

Before
the
employers
subsidiary liability is enforced,
adequate evidence must exist
establishing that (1) they are
indeed the employers of the
convicted employees; (2) they
are engaged in some kind of
industry; (3) the crime was
committed by the employees in
the discharge of their duties;

and (4) the execution against


the latter has not been satisfied
due
to
insolvency.
The
determination
of
these
conditions may be done in the
same criminal action in which
the employees liability, criminal
and civil, has been pronounced,
in a hearing set for that precise
purpose, with due notice to the
employer,
as
part
of
the
proceedings for the execution
of the judgment. (Rolito Calang
vs People, G.R. No. 190696,
August 3, 2010)
Concept of Prejudicial Question
Omictin vs CA, 512 SCRA 70
FACTS:
Petitioner Vincent E. Omictin,
Operations Manager Ad Interim of
Saag Phils., Inc., filed a complaint
for two counts of estafa with the
Office of the City Prosecutor of
Makati against private respondent
George I. Lagos. He alleged that
private
respondent,
despite
repeated demands, refused to
return the two company vehicles
entrusted to him when he was still
the president of Saag Phils., Inc..
On February 26, 1999, public
prosecutor Alex G. Bagaoisan
recommended the indictment of
private respondent, and on the
same day, respondent was charged
with the crime of estafa.
Ng and Yeo failed to appear,
however, in the scheduled board
meetings. Instead, on September
30, 1998 they issued a letter
appointing Alex Y. Tan as President
Ad Interim of Saag Phils., Inc. Tan,

in turn, appointed petitioner


Omictin
as
the
companys
Operations Manager Ad Interim.
Citing as a reason the absence of a
board resolution authorizing the
continued operations of Saag
Phils., Inc., private respondent
retained his possession of the
office equipment of the company in
a fiduciary capacity as director of
the
corporation
pending
its
dissolution and/or the resolution of
the intra-corporate dispute. He
likewise changed the locks of the
offices of the company allegedly to
prevent Tan and petitioner from
seizing company property.
Private respondent stressed that
Tans appointment was invalid
because it was in derogation of the
company by-laws requiring that
the president must be chosen from
among the directors, and elected
by the affirmative vote of a
majority of all the members of the
board of directors.5 As Tans
appointment did not have the
acquiescence of the board of
directors, petitioners appointment
by the former is likewise allegedly
invalid. Thus, neither has the
power
or
the
authority
to
represent or act for Saag Phils.,
Inc. in any transaction or action
before the SEC or any court of
justice.
The trial court, in an order dated
September
8,
1999,
denied
respondents motion to suspend
proceedings and motion to recuse.
His motion for reconsideration
having been denied by the trial
court in its order issued on
October 29, 1999, respondent filed

with the CA the petition for


certiorari[6]
assailing
the
aforesaid orders.
ISSUE:
whether or not a prejudicial
question exists to warrant the
suspension
of
the
criminal
proceedings
pending
the
resolution of the intra-corporate
controversy that was originally
filed with the SEC
HELD:
In a case for estafa, a valid
demand made by an offended party
is one of the essential elements. It
appears from the records that the
delay of delivery of the motor
vehicles by petitioner to Saag
Corporation is by reason of
petitioners contention that the
demand made by Omictin and Atty.
Tan to him to return the subject
vehicles is not a valid demand. As
earlier mentioned, petitioner filed
a case with the SEC questioning
therein
private
respondents
appointment.
If the SEC should rule that the
dissolution of Saag Phils. is proper,
or that the appointments of private
respondents
are
invalid,
the
criminal case will eventually be
dismissed due to the absence of
one of the essential elements of
the crime of estafa.
Based on the foregoing, it is clear
that a prejudicial question exists
which calls for the suspension of
the criminal proceedings before
the lower court.

It is clear that a prejudicial


question exists which calls for
the suspension of the criminal
proceedings before the lower
court.
A
prejudicial
question
is
defined as that which arises in
a case, the resolution of which
is a logical antecedent of the
issue involved therein and the
cognizance of which pertains to
another tribunal. (Omictin vs
CA, 512 SCRA 70)

Reyes vs Pearlbank Securities


Inc., 560 SCRA 518
FACTS:
PEARLBANK alleged that in March
2000, it received various letters
from persons who invested in
WINCORP demanding payment of
their matured investments, which
WINCORP
failed
to
pay,
threatening legal action. According
to these investors, WINCORP
informed them that PEARLBANK
was
the
borrower
of
their
investments. WINCORP alleged
that it was unable to repay its
investors because of the failure of
its fund borrowers, one of which
was PEARLBANK, to pay the loans
extended to them by WINCORP. As
proof of their claims, the investors
presented Confirmation Advices,4
Special Powers of Attorney and
Certifications signed and issued to
them by WINCORP.
On 7 April 2000, PEARLBANK filed
two complaints with the Securities
and Exchange Commission (SEC)
against Ong and several John Does
for full and accurate accounting of
the investments of WINCORP and
of PEARLBANKs alleged loan
obligations to WINCORP and/or its
investors.
The
cases
were
docketed as SEC Cases No. 04-006590 and 04-00-6591.
On 6 September 2000, Juanita U.
Tan, Treasurer of PEARLBANK,
filed a complaint on behalf of
PEARLBANK for falsification by
private individuals of commercial
and private documents before the
DOJ. The case was docketed as I.S.
No.
2000-1491.
Named

respondents in the complaint were


the officers and directors of
WINCORP, to wit: petitioner herein
Anthony T. Reyes, Antonio T. Ong,
Gilda C. Lucena,5 Nemesio R.
Briones, Loida C. Tamundong,6
Eric R.G. Espiritu, and John or Jane
Does.
ISSUE:
whether the two cases before the
SEC are prejudicial questions
which have to be resolved before
the criminal cases may proceed.
HELD:
NO. A prejudicial question is
defined as one which arises in a
case the resolution of which is a
logical antecedent of the issue
involved
therein,
and
the
cognizance of which pertains to
another tribunal.
The prejudicial question must be
determinative of the case before
the court, but the jurisdiction to
try and resolve the question must
be lodged in another court or
tribunal. It is a question based on a
fact distinct and separate from the
crime, but so intimately connected
with it that it determines the guilt
or innocence of the accused; and
for it to suspend the criminal
action, it must appear not only that
said case involves facts intimately
related to those upon which the
criminal prosecution would be
based, but also that in the
resolution of the issue or issues
raised in the civil case, the guilt or
innocence of the accused would
necessarily be determined.

It comes into play generally in a


situation in which a civil action
and a criminal action are both
pending and there exists in the
former an issue which must be
preemptively resolved before the
criminal action may proceed,
because howsoever the issue
raised in the civil action is resolved
would be determinative juris et de
jure of the guilt or innocence of
the accused in the criminal case.
If both civil and criminal cases
have similar issues or the issue in
one is intimately related to the
issues raised in the other, then a
prejudicial question would likely
exist, provided the other element
or characteristic is satisfied. It
must appear not only that the civil
case involves the same facts upon
which the criminal prosecution
would be based, but also that the
resolution of the issues raised in
the
civil
action
would
be
necessarily determinative of the
guilt or innocence of the accused.
If the resolution of the issue in the
civil action will not determine the
criminal responsibility of the
accused in the criminal action
based on the same facts, or there
is no necessity "that the civil case
be determined first before taking
up the criminal case," therefore,
the civil case does not involve a
prejudicial question. Neither is
there a prejudicial question if the
civil and the criminal action can,
according
to
law,
proceed
independently of each other.
There is no prejudicial question
here.

The prejudicial question must


be determinative of the case
before the court, but the
jurisdiction to try and resolve
the question must be lodged in
another court or tribunal. It is
a question based on a fact
distinct and separate from the
crime,
but
so
intimately
connected with it that it
determines
the
guilt
or
innocence of the accused; and
for it to suspend the criminal
action, it must appear not only
that said case involves facts
intimately related to those
upon
which
the
criminal
prosecution would be based,
but also that in the resolution
of the issue or issues raised in
the civil case, the guilt or
innocence of the accused would
necessarily
be
determined.
(Reyes vs Pearlbank Securities
Inc., 560 SCRA 518)

Coca-Cola Bottlers, Inc. vs


Social Security Commission,
560 SCRA 719
FACTS
The petitioners are engaged in the
business of buying and selling of
commodities, including day-old
chicks.
Respondent,
as
the
proprietor of Big J Farms and RBL
Farm, was the petitioners supplier
of day-old chicks from September
to December 2001.
Sometime in May 2002, the
respondent sent a demand letter to
the petitioners for the payment of
delivered day-old chicks. The
petitioners, thru petitioner Cadiz,
replied that they have paid
P1,360,000.00, but the respondent
was
able
to
deliver
only
P1,136,150.00 worth of day-old
chicks, leaving a deficiency of
P223,850.00 worth of day-old
chicks. The petitioners demanded
the delivery of the deficiency, or
the return of the overpayment
made. When the parties refused to
comply with each others demands,
both went to court for judicial
relief.
On June 11, 2002, the petitioners
(the buyers of the chicks) filed
before RTC-Tarlac a complaint for
sum of money and damages
against the respondent, docketed
as Civil Case No. 9354. The
petitioners sought the return of
the overpayment made, plus moral
and exemplary damages, and
attorneys fees.
On June 19, 2002, the respondent
(the seller of the chicks) filed
before RTC-Malolos, Bulacan a

complaint for sum of money and


damages against the petitioners,
docketed as Civil Case No. 489-M2002. The respondent alleged that
he delivered P1,368,100.00 worth
of
day-old
chicks,
but
the
petitioners
only
paid
P1,150,000.00.
Thus,
the
respondent
prayed
for
the
payment
of
the
balance
of
P218,100.00.
Shortly upon receipt of the
summons and complaint in Civil
Case No. 9354, or on August 21,
2002, the respondent filed a
motion to dismiss Civil Case No.
9354
before
RTC-Tarlac.
He
argued that Civil Case No. 9354
should be dismissed on the ground
of litis pendentia because it is
merely anticipatory and defensive
of the respondents claim for
collection in Civil Case No. 489-M2002 before RTC-Malolos.
ISSUE
The core issue is whether Civil
Case No. 9354 (the buyers action
for overpayment) filed ahead of
Civil Case No. 489-M-2002 (the
sellers action for collection of
balance) should be dismissed on
the ground of litis pendentia.
RULING
On September 2, 2002, RTC-Tarlac
issued an Order in Civil Case No.
9354 granting the respondents
motion
and
dismissing
the
complaint on the ground of litis
pendentia. It noted that the
petitioners filed Civil Case No.
9354 to preempt the respondents
collection case in Civil Case No.

489-M-2002 before RTC-Malolos. It


found that the petitioners filed
Civil Case No. 9354 only after they
received a demand letter from the
respondent.
The petitioners moved but failed to
secure a reconsideration of the
RTC order and, from thence, came
to us through the present petition
on a pure question of law.

There is prejudicial question


when
(a)
the
previously
instituted civil action involves
an issue similar or intimately
related to the issue raised in
the subsequent criminal action,
and (b) the resolution of such
issue determines whether or
not the criminal action may
proceed. It comes into play
generally in a situation where a
civil action and a criminal
action both pend and there
exists in the former an issue
which must be preemptively
resolved before the criminal
action may proceed. This is so
because howsoever the issue
raised in the civil action is
resolved
would
be
determinative juris et de jure of
the guilt or innocence of the
accused in the criminal case.
(Coca-Cola Bottlers, Inc. vs
Social Security Commission,
560 SCRA 719)

Reason of Principle of
Prejudicial Question
Jose vs Suarez, 556 SCRA 773
FACTS:
Spouses Laureano and Purita
Suarez, had availed of Carolina
Joses (Carolina) offer to lend
money at the daily interest rate of
1% to 2% which was later on
increased
to
5%
per
day.
Respondents were forced to accept
because they allegedly had no
other option left. Purita would then
issue checks in favor of petitioners
in
payment
of
the
amount
borrowed from them with the
agreed 5% daily interest.
In 2004, Sps. Suarez filed a
Complaint against Jose seeking to
nullify the 5% interest per day,
alleging that same is iniquitous,
contrary to morals, done under
vitiated consent and imposed using
undue
influence
by
taking
improper
advantage
of
their
financial distress.
Thereafter, Jose filed several cases
for violation of B.P. Blg. 22 against
respondent Purita before the
MTCC.
Purita, in turn filed motions to
suspend the criminal proceedings
on the ground of prejudicial
question. Respondents claimed
that if the 5% interest per month is
nullified and loans are computed at
1% per month, it would mean that
the checks subject of the B.P. Blg.
22 cases are not only fully paid but
are also in fact overpaid.

The motion to suspend was denied.


Hence, Sps. Suarez filed before the
RTC a Motion for Writ of
Preliminary
Injunction
with
Temporary
Restraining
Order
seeking to restrain the MTCCs
from further proceeding with the
B.P. Blg. 22 cases on the ground of
prejudicial question. The RTC
granted the motion. CA affirmed.
Hence, petitioners appealed.
ISSUE:
Whether or not a prejudicial
question exists such that the
outcome of the validity of the
interest rate is determinative of
the guilt or innocence of the
respondent spouses in the criminal
case.
HELD:
No. There is none.
A prejudicial question has two
essential elements: (i) the civil
action involves an issue similar or
intimately related to the issue
raised in the criminal action; and
(ii) the resolution of such issue
determines whether or not the
criminal action may proceed.
The validity or invalidity of the
interest rate is not determinative
of the guilt of respondents in the
criminal cases. The cause or
reason for the issuance of a check
is inconsequential in determining
criminal culpability under B.P. Blg.
22. What the law punishes is the
issuance of a bouncing check,
which is a malum prohibitum, and
not the purpose for which it was

issued or the terms and conditions


relating to its issuance.
Thus, whether or not the interest
rate imposed by petitioners is
eventually declared void for being
contra bonos mores will not affect
the outcome of the B.P. Blg. 22
cases because what will ultimately
be penalized is the mere issuance
of bouncing checks. The primordial
question is whether the law has
been breached, that is, if a
bouncing check has been issued.
The reason behind the principle of
a prejudicial question is to avoid
two conflicting decisions in the
civil case and in the criminal case.

The reason behind the principle


of a prejudicial question is to
avoid two conflicting decisions
in the civil case and in the
criminal case. (Jose vs Suarez,
556 SCRA 773)

Sy Thiong Siou vs Sy Chim, G.R.


No. 174168, March 30, 2009
On 30 May 2003, four criminal
complaints were filed by Sy Chim
and Felicidad Chan Sy (Spouses
Sy) against Sy Tiong Shiou, Juanita
Tan Sy, Jolie Ross Tan, Romer Tan,
Charlie Tan and Jessie James Tan
(Sy Tiong Shiou, et al.) before the
City Prosecutors Office of Manila.
The cases were later consolidated.
Two of the complaints, I.S. Nos.
03E-15285 and 03E-15286, were
for alleged violation of Section 74
in relation to Section 144 of the
Corporation
Code.
In
these
complaints,
the
Spouses
Sy
averred that they are stockholders
and directors of Sy Siy Ho & Sons,
Inc. (the corporation) who asked
Sy Tiong Shiou, et al., officers of
the corporation, to allow them to
inspect the books and records of
the business on three occasions to
no avail. In a letter dated 21 May
2003, Sy Tiong Shiou, et al. denied
the request, citing civil and intracorporate cases pending in court.
In the two other complaints, I.S.
No. 03E-15287 and 03E-15288, Sy
Tiong Shiou was charged with
falsification under Article 172, in
relation to Article 171 of the
Revised Penal Code (RPC), and
perjury under Article 183 of the
RPC. According to the Spouses Sy,
Sy Tiong Shiou executed under
oath the 2003 General Information
Sheet (GIS) wherein he falsely
stated that the shareholdings of
the Spouses Sy had decreased
despite the fact that they had not
executed any conveyance of their
shares.

The Spouses Sy elevated the DOJs


resolutions to the Court of Appeals
through a petition for certiorari,
imputing grave abuse of discretion
on the part of the DOJ. The
appellate
court
granted
the
petition and directed the City
Prosecutors Office to file the
appropriate informations against
Sy Tiong Shiou, et al. for violation
of Section 74, in relation to
Section 144 of the Corporation
Code and of Articles 172 and 183
of the RPC. The appellate court
ruled that the civil case for
accounting and damages cannot be
deemed
prejudicial
to
the
maintenance or prosecution of a
criminal action for violation of
Section 74 in relation to Section
144 of the Corporation Code since
a finding in the civil case that
respondents
mishandled
or
misappropriated the funds would
not be determinative of their guilt
or innocence in the criminal
complaint. In the same manner,
the
criminal
complaints
for
falsification and/or perjury should
not have been dismissed on the
ground of prejudicial question
because the accounting case is
unrelated and not necessarily
determinative of the success or
failure of the falsification or
perjury charges. Furthermore, the
Court of Appeals held that there
was probable cause that Sy Tiong
Shiou had committed falsification
and that the City of Manila where
the 2003 GIS was executed is the
proper venue for the institution of
the perjury charges. Sy Tiong
Shiou,
et
al.
sought
reconsideration of the Court of

Appeals decision but their motion


was denied.
ISSUE:
Whether he DOJ gravely abused its
discretion when it suspended the
hearing of the charges for violation
of the Corporation Code on the
ground of prejudicial question and
when it dismissed the criminal
complaints
HELD:
As correctly found by the Court of
Appeals, the DOJ gravely abused
its discretion when it suspended
the hearing of the charges for
violation of the Corporation Code
on the ground of prejudicial
question and when it dismissed the
criminal complaints.
A prejudicial question comes into
play generally in a situation where
a civil action and a criminal action
are both pending and there exists
in the former an issue which must
be preemptively resolved before
the criminal action may proceed
since howsoever the issue raised in
the civil action is resolved would
be determinative juris et de jure of
the guilt or innocence of the
accused in the criminal case. The
reason behind the principle of
prejudicial question is to avoid two
conflicting decisions. It has two
essential elements: (a) the civil
action involves an issue similar or
intimately related to the issue
raised in the criminal action; and
(b) the resolution of such issue
determines whether or not the
criminal action may proceed.

The civil action and the criminal


cases
do
not
involve
any
prejudicial question.
The civil action for accounting and
damages, Civil Case No. 03106456 pending before the RTC
Manila, Branch 46, seeks the
issuance of an order compelling
the Spouses Sy to render a full,
complete and true accounting of
all the amounts, proceeds and fund
paid to, received and earned by
the corporation since 1993 and to
restitute it such amounts, proceeds
and funds which the Spouses Sy
have
misappropriated.
The
criminal cases, on the other hand,
charge that the Spouses Sy were
illegally prevented from getting
inside company premises and from
inspecting company records, and
that Sy Tiong Shiou falsified the
entries in the GIS, specifically the
Spouses
Sys
shares
in
the
corporation. Surely, the civil case
presents no prejudicial question to
the criminal cases since a finding
that the Spouses Sy mishandled
the funds will have no effect on the
determination of guilt in the
complaint for violation of Section
74 in relation to Section 144 of the
Corporation Code; the civil case
concerns the validity of Sy Tiong
Shious refusal to allow inspection
of the records, while in the
falsification and perjury cases,
what is material is the veracity of
the entries made by Sy Tiong
Shiou in the sworn GIS.
Anent the issue of probable cause,
the Court also finds that there is
enough probable cause to warrant
the institution of the criminal
cases.

The reason behind the principle


of prejudicial question is to
avoid two conflicting decisions.
It has two essential elements:
(a) the civil action involves an
issue similar or intimately
related to the issue raised in
the criminal action; and (b) the
resolution
of
such
issue
determines whether or not the
criminal action may proceed.
(Sy Thiong Siou vs Sy Chim,
G.R. No. 174168, March 30,
2009)

Requisites for a Prejudicial


Question
Magestrado vs People, G.R. No.
148072, July 10, 2009
That on or about the 27th day of
December, 1997, in Quezon City,
Philippines, the said accused, did
then and there willfully, unlawfully
and feloniously and knowingly
make an untruthful statement
under oath upon a material matter
before
a
competent
officer
authorized
to
receive
and
administer oath and which the law
so require, to wit: the said accused
subscribe and swore to an Affidavit
of Loss before Notary Public
Erlinda B. Espejo, falsely alleging
that he lost Owners Duplicate
Certificate of TCT No. N-173163,
which document was used in
support of a Petition For Issuance
of New Owners Duplicate Copy of
Certificate of Title and filed with
the Regional Trial Court of Quezon
City, docketed as LRC# Q-10052
(98) on January 28, 1998 and
assigned to Branch 99 of the said
court, to which said Francisco M.
Magestrado signed and swore on
its verification, per Doc. 413 Page
84 Book No. CLXXV Series of 1998
of Notary Public Erlinda B. Espejo
of Quezon City; the said accused
knowing fully well that the
allegations in the said affidavit and
petition are false, the truth of the
matter being that the property
subject of Transfer Certificate of
Title No. N-173163 was mortgaged
to complainant Elena M. Librojo as
collateral for a loan in the amount
of P 758,134.42 and as a

consequence of which said title to


the property was surrendered by
him to the said complainant by
virtue of said loan, thus, making
untruthful
and
deliberate
assertions of falsehoods, to the
damage and prejudice of the said
Elena M. Librojo.
On 30 June 1999, petitioner filed a
motion[5]
for
suspension
of
proceedings based on a prejudicial
question. Petitioner alleged that
Civil Case No. Q-98-34349, a case
for recovery of a sum of money
pending before the Regional Trial
Court (RTC) of Quezon City,
Branch 84, and Civil Case No. Q98- 34308, a case for Cancellation
of Mortgage, Delivery of Title and
Damages, pending before the RTC
of Quezon City, Branch 77, must be
resolved first before Criminal Case
No. 90721 may proceed since the
issues in the said civil cases are
similar or intimately related to the
issues raised in the criminal
action.
On 14 July 1999, MeTC-Branch 43
issued
an
Order
denying
petitioners motion for suspension
of proceedings.
ISSUE:
hether or not Judge Estrella T.
Estrada of the Regional Trial
Court, Branch 83, Quezon City, had
committed
grave
abuse
of
discretion amounting to lack or in
excess of her jurisdiction in
denying the Petition for Certiorari
and petitioners subsequent motion
for reconsideration on the ground
of a prejudicial question pursuant

to the Rules on Criminal Procedure


and the prevailing jurisprudence.
HELD:

would necessarily be determined;


and (3) jurisdiction to try said
question must be lodged in
another tribunal.

NO. Prejudial question is defined


as that which arises in a case the
resolution of which is a logical
antecedent of the issue involved
therein, and the cognizance of
which pertains to another tribunal.
The prejudicial question must be
determinative of the case before
the court but the jurisdiction to try
and resolve the question must be
lodged in another court or
tribunal. It is a question based on a
fact distinct and separate from the
crime but so intimately connected
with it that it determines the guilt
or innocence of the accused.

If the resolution of the issue in the


civil action will not determine the
criminal responsibility of the
accused in the criminal action
based on the same facts, or there
is no necessity that the civil case
be determined first before taking
up the criminal case, therefore, the
civil case does not involve a
prejudicial question.[30] Neither is
there a prejudicial question if the
civil and the criminal action can,
according
to
law,
proceed
independently of each other.

For a prejudicial question in a civil


case to suspend criminal action, it
must appear not only that said
case involves facts intimately
related to those upon which the
criminal prosecution would be
based but also that in the
resolution of the issue or issues
raised in the civil case, the guilt or
innocence of the accused would
necessarily be determined.

For a civil action to be


considered prejudicial to a
criminal case as to cause the
suspension of the criminal
proceedings until the final
resolution of the civil case, the
following requisites must be
present: (1) the civil case
involves facts intimately related
to those upon which the
criminal prosecution would be
based; (2) in the resolution of
the issue or issues raised in the
civil
action,
the
guilt
or
innocence of the accused would
necessarily be determined; and
(3) jurisdiction to try said
question must be lodged in
another tribunal.( Magestrado
vs People, G.R. No. 148072, July
10, 2009)

Thus, for a civil action to be


considered
prejudicial
to
a
criminal case as to cause the
suspension
of
the
criminal
proceedings
until
the
final
resolution of the civil case, the
following
requisites
must
be
present: (1) the civil case involves
facts intimately related to those
upon
which
the
criminal
prosecution would be based; (2) in
the resolution of the issue or
issues raised in the civil action, the
guilt or innocence of the accused

Torres vs Garchitorena, G.R.


No. 153666, December 27,
2002
FACTS:
On October 10, 1997, Mayor
Dionisio Torres of Noveleta, Cavite
caused
the
leveling
and
reclamation of the submerged
portion of SRIs property for the
relocation of displaced squatters
from Tirona, Cavite who were
living along river banks and
esteros.
Domingo
Fernandez
protested to the Mayor informing
him that his employer owned the
property
being
levelled
and
reclaimed at the instance of the
Mayor. However, the latter ignored
the protests of Fernandez and
continued with the leveling and
reclamation of the property. On
October 16, 1997, representatives
of SRI conferred with the Mayor
and furnished him with copies of
its titles over the property. The SRI
had the property surveyed to
confirm that the portions of the
land reclaimed by the Mayor were
within the perimeter of its titled
property. On October 27, 1997,
SRI sent a letter to the Mayor
formally protesting the leveling
and reclamation of the submerged
portion of its property and
demanding that the Mayor desist
from
continuing
with
said
reclamation. On October 31, 1997,
the Mayor and representatives of
SRI had a conference during which
the Mayor informed SRI that he
had already spent P1,000.000,00
for the reclamation and offered to
help SRI in connection with its
other projects in Cavite provided

that SRI will no longer file the suit


to enjoin the reclamation. SRI
requested for the deferment of the
reclamation
project
until
November 7, 1997 to enable it to
study the offer of the Mayor.
However, SRI learned that in the
interim, five families of squatters
had already occupied portions of
the reclaimed area; and that more
squatters were due to arrive.
On January 7, 1998, SRI filed a
petition with the Regional Trial
Court for prohibition with a plea
for injunctive relief against the
Mayor, the Municipal Building
Official and Municipal Engineer
Enrico Alvarez to enjoin them from
reclaiming
and
leveling
the
property and for damages and
attorneys fees. The case was
docketed as Special Civil Case No.
N-6639. In their Answer to the
petition,
Torres
and
Alvarez
alleged inter alia that they were
not aware that the subject
property was titled in the name of
SRI and that the records of the
Assessors Office failed to show
that the property had been
declared for taxation purposes
under the name of SRI. The Mayor
insisted that SRI abandoned the
property and justified his acts on
the ground that the reclamation of
the property was for the socialized
housing
program
of
his
constituents.
ISSUE:
WHETHER
THE
PUBLIC
RESPONDENTS
COMMITTED
GRAVE ABUSE OF DISCRETION
AMOUNTING
TO
LACK
OR

EXCESS OF JURISDICTION IN
DENYING PETITIONERS MOTION
TO
SUSPEND
FURTHER
PROCEEDINGS
DESPITE
SUBSTANTIAL
EVIDENCE
SHOWING
THAT
ALL
THE
ELEMENTS FOR A PREJUDICIAL
QUESTION ARE PRESENT IN
THIS CASE.
HELD:
NO.
We agree with the Respondents.
Under Rule 65 of the 1997 Rules of
Civil Procedures, as amended,
petitioners must show that they
had no plain, speedy and adequate
remedy in the ordinary course of
law
against
their
perceived
grievance. Petitioners are not
entitled to a writ of certiorari if
they seek said relief to make up for
the loss, through their oversight or
omission, of their right to file their
petition for certiorari within the
period therefor. The record shows
that petitioners motion to suspend
proceedings had already been
denied by the Sandiganbayan on
January 15, 2001. Petitioners
motion to suspend proceedings
dated June 11, 2002, which was for
all intents and purposes a motion
for
reconsideration
of
the
Resolution of the Sandiganbayan
dated January 15, 2001, was
denied by Sandiganbayan on
August
1,
2001.
However,
petitioners did not file a petition
for certiorari with the Court within
the
period
therefor.
Indeed,
petitioners
filed
an
omnibus
motion with leave of court on
September 13, 2001, praying inter
alia for the suspension of the

proceedings. The omnibus motion


was denied by the respondent
court on April 4, 2002. It was only
on June 11, 2002 that petitioners
filed the petition at bench. By then,
the period for them to file the
same had long lapsed. Why the
petitioners waited for almost a
year from August 1, 2001 to file
their petition for certiorari with
the Court they did not bother to
justify. It bears stressing that the
rule on prejudicial questions was
conceived to afford parties an
expeditious and just disposition of
cases. Indeed, the amendment
under the Rules on Criminal
Procedure was designed to avert
the deleterious practice foisted on
the
judicial
system
by
unscrupulous parties to derail the
placid flow of criminal cases. The
Court has said that it will not
countenance the misuse of the
rules of procedures to frustrate or
delay the delivery of justice.
Under
the
amendment,
a
prejudicial question is understood
in law as that which must precede
the criminal action and which
requires a decision before a final
judgment can be rendered in the
criminal action with which said
question is closely connected. The
civil action must be instituted prior
to the institution of the criminal
action.
In
this
case,
the
Information was filed with the
Sandiganbayan
ahead
of
the
complaint in Civil Case No. 7160
filed by the State with the RTC in
Civil Case No. 7160. Thus, no
prejudicial question exists.

The civil action must be


instituted
prior
to
the
institution of the criminal
action. If the criminal action
was
filed
ahead
of
the
complaint in the civil case, no
prejudicial
question
exists.
(Torres vs Garchitorena, G.R.
No. 153666, December 27,
2002)

Effect of Existence of a
Prejudicial Question;
suspension of the Criminal
Action
IBP vs Atienza, G.R. No.
175241, February 24, 2010
In June 2006, the Integrated Bar of
the Philippines (IBP) filed an
application for a rally permit with
the office of Manila Mayor Jose
Lito Atienza. The IBP sought
their rally to be staged at the
Mendiola Bridge. Atienza granted
the permit but indicated thereon
that IBP is only allowed to stage
their rally at the Plaza Miranda, a
freedom park.
IBP President Jose Anselmo Cadiz
received the rally permit on the
day before the scheduled rally.
Cadiz immediately went to the
Court of Appeals to assail the
permit because what Atienza did
was only a partial grant which was
alleged to be a violation of the
constitutional right to freedom of
expression and a grave abuse of
discretion on the part of Atienza.
Meanwhile, IBP pushed through
with the rally not at Plaza Miranda
but at the Mendiola Bridge.
Subsequently, the Manila Police
District (MPD) filed a criminal case
against
Cadiz
for
allegedly
violating the Public Assembly Act
or specifically, for staging a rally in
a place different from what was
indicated in the rally permit.
The Court of Appeals ruled in favor
of Atienza. The CA ruled that what
Atienza did was within his power;

that freedom of expression is not


absolute.
Cadiz
appealed
before
the
Supreme Court. Cadiz also prayed
for the suspension of the criminal
case against him on the ground
that the certiorari case he filed
against Atienza is a prejudicial
question to the criminal case.
ISSUE
Whether or not the certiorari case
Cadiz filed against Atienza is a
prejudicial question to the criminal
case filed against Cadiz
HELD:
No. It is improper for Cadiz to
raise the issue of prejudicial
question at this stage and in this
certiorari case. Under the Rules of
Court, a prejudicial question is a
ground to suspend the criminal
proceeding. However, Cadiz must
first file a petition to suspend the
criminal proceeding in the said
criminal case. The determination
of the pendency of a prejudicial
question should be made at the
first instance in the criminal
action,
and
not
before
the
Supreme Court in an appeal from
the civil action.
Under the Rules, the existence
of a prejudicial question is a
ground in a petition to suspend
proceedings
in
a
criminal
action. Since suspension of the
proceedings in the criminal
action may be made only upon
petition and not at the instance
of
the
judge
or
the
investigating prosecutor, the

latter cannot take cognizance


of
a
claim
of
prejudicial
question without a petition to
suspend being filed. Since a
petition to suspend can be filed
only in the criminal action, the
determination of the pendency
of a prejudicial question should
be made at the first instance in
the criminal action, and not
before this Court in an appeal
from the civil action. (IBP vs
Atienza,
G.R.
No.
175241,
February 24, 2010)

Suspension does not include


dismissal
Yap vs Parras, 205 SCRA 625
According to Yap, Paras sold IN
1971 to her his share in the
intestate estate for P300.00. The
sale was evidenced by a private
document. Nineteen years later, (in
1990), Paras sold the same
property to Santiago Saya-ang for
P5,000.00. This was evidenced by
a notarized Deed of Absolute Sale.
When Yap learned of the second
sale, she filed a complaint for
estafa against Paras and Saya-ang
with the Office of the Provincial
Prosecutor of General Santos City.
On the same date, she filed a
complaint for the nullification of
the said sale with the Regional
Trial Court of General Santos City.
After investigation, the Provincial
Prosecutor instituted a criminal
complaint for estafa against Paras
with the Municipal Circuit Trial
Court of Glan-Malapatan, South
Cotabato,
presided
by
Judge
Alfredo D. Barcelona, Sr., who
dismissed the criminal case on the
ground that the issue in the civil
case is prejudicial to the criminal
case for estafa.
Issue
Is the Judge correct in motu
proprio dismissing the criminal
case?
Ruling:
The judge is wrong. First, he
should not have dismissed the
criminal case but only suspended

it. Second, it was wrong for him to


dismiss the criminal case outright,
since it requires a motion first
from the proper party.
The
rule
provides:
Sec.
6.
Suspension
by
reason
of
prejudicial question. A petition
for suspension of the criminal
action based upon the pendency of
a prejudicial question in a civil
action may be filed in the office of
the fiscal or the court conducting
the
preliminary
investigation.
When the criminal action has been
filed in court for trial, the petition
to suspend shall be filed in the
same criminal action at any time
before the prosecution rests.
Third,
there
is
actually
no
prejudicial question here.
Anent the issue of prejudicial
question, the rule provides that:
Section 5, Rule 111 of the 1985
Rules on Criminal Procedure as
amended provides:
Sec. 5. Elements of prejudicial
question. The two (2) essential
elements of a prejudicial question
are: (a) the civil action involves an
issue similar or intimately related
to the issue raised in the criminal
action; and (b) the resolution of
such issue determines whether or
not the criminal action may
proceed.
A prejudicial question is defined as
that which arises in a case the
resolution of which is a logical
antecedent of the issue involved
therein, and the cognizance of
which pertains to another tribunal.

The prejudicial question must be


determinative of the case before
the court but the jurisdiction to try
and resolve the question must be
lodged in another court or
tribunal. It is a question based on
a fact distinct and separate from
the crime but so intimately
connected
with
it
that
it
determines the guilt or innocence
of the accused.
It was held that "for a civil case to
be considered prejudicial to a
criminal action as to cause the
suspension of the criminal action
pending the determination of the
civil action, it must appear not only
that the civil case involves the
same facts upon which the
criminal prosecution is based, but
also that the resolution of the
issues raised in said civil action
would
be
necessarily
determinative of the guilt or
innocence of the accused".
Indeed, the civil case at bar does
not involve the same facts upon
which the criminal action is based.
There
was
no
motion
for
suspension in the case at bar; and
no less importantly, the respondent
judge had not been informed of the
defense Paras was raising in the
civil action. Judge Barcelona could
not have ascertained then if the
issue raised in the civil action
would determine the guilt or
innocence of the accused in the
criminal case.

It
plainly
says
that
the
suspension may be made only
upon petition and not at the

instance of the judge alone,


and it also says suspension, and
not dismissal. (Yap vs Parras,
205 SCRA 625)

Prejudicial Question: Case


Illustrations
Pimentel vs Pimentel, G.R. No.
172060, September 13, 2010
Facts:
Respondent,
Maria
Chrysantine Pimentel, filed an
action for frustrated parricide
against petitioner, Joselito R.
Pimentel. The Information for
Frustrated Parricide was dated 30
August 2004 and was raffled to
RTC Quezon City on 25 October
2004. The pre-trial and trial was
set on 14 February 2005. She also
filed on 5 November 2004, a
petition, dated 4 November 2004,
for Declaration of Nullity of
Marriage under Section 36 of the
Family Code on the ground of
psychological incapacity Petitioner
received summons to appear
before the Regional Trial Court of
Antipolo City on 7 February 2005,
for the pre-trial and trial of the
Civil Case. He then filed an urgent
motion to suspend the proceedings
before the RTC Quezon City on the
ground of the existence of a
prejudicial question asserting that
the relationship between the
offender and the victim is a key
element in parricide, the outcome
of Civil Case would have a bearing
in the criminal case filed against
him before the RTC Quezon City.
The RTC Quezon City held that the
pendency of the case before the
RTC Antipolo is not a prejudicial
question
that
warrants
the
suspension of the criminal case
before it. The Court of Appeals
also denied the petition holding
that the issue in the criminal case

for frustrated parricide differs


from the issue in the civil action
for annulment of marriage. It ruled
that even if the marriage between
petitioner and respondent would
be declared void, it would be
immaterial to the criminal case
because prior to the declaration of
nullity,
the
alleged
acts
constituting the crime of frustrated
parricide
had
already
been
committed. At the time of the
commission of the crime, the
marriage is still subsisting.

Issue:
Whether
or
not
the
resolution of the action for
annulment of marriage is a
prejudicial question that warrants
the suspension of the criminal case
for frustrated parricide.

Ruling:
The
elements
of
a
prejudicial question under Section
7, Rule 111 of the 2000 Rules on
Criminal Procedure, which are: (a)
the previously instituted civil
action involves an issue similar or
intimately related to the issue
raised in the subsequent criminal
action and(b) the resolution of
such issue determines whether or
not the criminal action may
proceed, were not met. Civil action
must be instituted first before the
filing of the criminal action. In this
case, the civil case for annulment
was filed afterthe filing of the
criminal
case
for
frustrated
parricide. Further, the resolution
of the civil action is not a
prejudicial question that would

warrant the suspension of the


criminal action. There is a
prejudicial question when a civil
action and a criminal action are
both pending, and there exists in
the civil action an issue which
must be pre-emptively resolved
before the criminal action may
proceed because howsoever the
issue raised in the civil action is
resolved would be determinative of
the guilt or innocence of the
accused in the criminal case. The
relationship between the offender
and the victim is a key element in
the crime of parricide. However,
the issue in the annulment of
marriage
is
not
similar
or
intimately related to the issue in
the criminal case for parricide.
Further, the relationship between
the offender and the victim is not
determinative of the guilt or
innocence of the accused. The
issue in the civil case for
annulment of marriage under
Article 36 of the Family Code is
whether
petitioner
is
psychologically incapacitated to
comply with the essential marital
obligations. The issue in parricide
is whether the accused killed the
victim.
In
this
case,
since
petitioner
was
charged
with
frustrated parricide, the issue is
whether he performed all the acts
of execution which would have
killed
respondent
as
a
consequence
but
which,
nevertheless, did not produce it by
reason of causes independent of
petitioners will. At the time of the
commission of the alleged crime,
petitioner and respondent were
married.
The
subsequent
dissolution of their marriage, in

case the petition in Civil Case is


granted, will have no effect on the
alleged crime that was committed
at the time of the subsistence of
the marriage. In short, even if the
marriage between petitioner and
respondent is annulled, petitioner
could still be held criminally liable
since
at
the
time
of
the
commission of the alleged crime,
he was still married to respondent.

The relationship between the


offender and the victim is a key
element
in
the
crime
of
parricide, which punishes any
person who shall kill his father,
mother,
or
child,
whether
legitimate or illegitimate, or
any of his ascendants or
descendants, or his spouse. The
relationship
between
the
offender
and
the
victim
distinguishes the crime of
parricide
from
murder
or
homicide. However, the issue in
the annulment of marriage is
not
similar
or
intimately
related to the issue in the
criminal case for parricide.
Further,
the
relationship
between the offender and the
victim is not determinative of
the guilt or innocence of the
accused. (Pimentel vs Pimentel,
G.R. No. 172060, September 13,
2010)

Yap vs Cabales, G.R. No.


159186, June 5, 2009
FACTS:
Petitioner Jesse Y. Yap and
his spouse Bessie Yap are engaged
in the real estate business through
their company Primetown Property
Group.
Sometime
in
1996,
petitioner purchased several real
properties from a certain Evelyn
Te (Evelyn). In consideration of
said purchases, petitioner issued
several Bank of the Philippine
Islands (BPI) postdated checks to
Evelyn.
Thereafter,
spouses
Orlando and Mergyl Mirabueno
and spouses Charlie and Jovita
Dimalanta,
rediscounted
the
checks from Evelyn. In the
beginning, the first few checks
were honored by the bank, but in
the early part of 1997, when the
remaining checks were deposited
with the drawee bank, they were
dishonored for the reason that the
"Account is Closed." Demands
were made by Spouses Mirabueno
and Spouses Dimalanta to the
petitioner to make good the
checks. Despite this, however, the
latter failed to pay the amounts
represented by the said checks.
Spouses Mirabueno filed a civil
action for collection of sum of
money, damages and attorney's fee
with prayer for the issuance of a
writ of preliminary attachment
against petitioner. Subsequently,
on various dates, the Office of the
City Prosecutor of General Santos
City filed several informations for
violation of Batas Pambansa Bilang
(B.P. Blg.) 22 against the petitioner
with the Municipal Trial Court in

Cities (MTCC). In the criminal


cases, petitioner filed separate
motions to suspend proceedings on
account of the existence of a
prejudicial question and motion to
exclude the private prosecutor
from
participating
in
the
proceedings. Petitioner prayed
that the proceedings in the
criminal cases be suspended until
the civil cases pending before the
RTC were finally resolved. The
MTCC
denied
the
motion.
Aggrieved,
petitioner
filed
a
Petition for Certiorari with a
Prayer for the Issuance of a Writ of
Preliminary Injunction before the
RTC imputing grave abuse of
discretion on the part of the MTCC
Judge. The RTC denied the
petition.
Thereafter,
petitioner
filed with the CA a Petition for
Certiorari
Prohibition
and
Mandamus with Urgent Prayer for
the Issuance of Status Quo Order
and Writ of Preliminary Injunction.
The CA dismissed the petition for
lack of merit. The CA opined that
the civil cases did not pose a
prejudicial
question
to
the
prosecution of the petitioner for
violation of B.P. Blg. 22.
ISSUES:
Is the collection suit a prejudicial
question to the criminal case
charging him for violation of B.P.
22?
HELD:
A
prejudicial
question
generally exists in a situation
where a civil action and a criminal
action are both pending, and there
exists in the former an issue that

must be preemptively resolved


before the latter may proceed,
because howsoever the issue
raised in the civil action is resolved
would be determinative juris et de
jure of the guilt or innocence of
the accused in the criminal case.
The rationale behind the principle
of prejudicial question is to avoid
two conflicting decisions. It has
two essential elements: (i) the civil
action involves an issue similar or
intimately related to the issue
raised in the criminal action; and
(ii) the resolution of such issue
determines whether or not the
criminal action may proceed.22
The issue in the criminal cases is
whether the petitioner is guilty of
violating B.P. Blg. 22, while in the
civil case, it is whether the private
respondents are entitled to collect
from the petitioner the sum or the
value of the checks that they have
rediscounted from Evelyn. The
resolution of the issue raised in the
civil action is not determinative of
the guilt or innocence of the
accused in the criminal cases
against him, and there is no
necessity that the civil case be
determined first before taking up
the criminal cases. Also, even if
petitioner is declared not liable for
the payment of the value of the
checks and damages, he cannot be
adjudged
free
from
criminal
liability for violation of B.P. Blg. 22.
The mere issuance of worthless
checks with knowledge of the
insufficiency of funds to support
the checks is in itself an offense.
The gravamen of the offense
punished by B.P. Blg. 22 is the act
of making and issuing a worthless

check, it is not the non-payment of


an obligation. Because of its
deleterious effects on the public
interest, the practice is proscribed
by the law.

Even if petitioner is declared


not liable for the payment of
the value of the checks and
damages,
he
cannot
be
adjudged free from criminal
liability for violation of B.P.
Blg. 22. The mere issuance of
worthless
checks
with
knowledge of the insufficiency
of funds to support the checks
is in itself an offense. (Yap vs
Cabales, G.R. No. 159186, June
5, 2009)

Sps. Jose vs Sps Suarez, G.R.


No. 176795, June 30, 2008
FACTS:
Spouses Laureano and Purita
Suarez, had availed of Carolina
Joses (Carolina) offer to lend
money at the daily interest rate of
1% to 2% which was later on
increased
to
5%
per
day.
Respondents were forced to accept
because they allegedly had no
other option left. Purita would then
issue checks in favor of petitioners
in
payment
of
the
amount
borrowed from them with the
agreed 5% daily interest.
In 2004, Sps. Suarez filed a
Complaint against Jose seeking to
nullify the 5% interest per day,
alleging that same is iniquitous,
contrary to morals, done under
vitiated consent and imposed using
undue
influence
by
taking
improper
advantage
of
their
financial distress.
Thereafter, Jose filed several cases
for violation of B.P. Blg. 22 against
respondent Purita before the
MTCC.
Purita, in turn filed motions to
suspend the criminal proceedings
on the ground of prejudicial
question. Respondents claimed
that if the 5% interest per month is
nullified and loans are computed at
1% per month, it would mean that
the checks subject of the B.P. Blg.
22 cases are not only fully paid but
are also in fact overpaid.
The motion to suspend was denied.
Hence, Sps. Suarez filed before the
RTC a Motion for Writ of

Preliminary
Injunction
with
Temporary
Restraining
Order
seeking to restrain the MTCCs
from further proceeding with the
B.P. Blg. 22 cases on the ground of
prejudicial question. The RTC
granted the motion. CA affirmed.
Hence, petitioners appealed.
ISSUE:
Whether or not a prejudicial
question exists such that the
outcome of the validity of the
interest rate is determinative of
the guilt or innocence of the
respondent spouses in the criminal
case.
HELD:
No. There is none.
A prejudicial question has two
essential elements: (i) the civil
action involves an issue similar or
intimately related to the issue
raised in the criminal action; and
(ii) the resolution of such issue
determines whether or not the
criminal action may proceed.
The validity or invalidity of the
interest rate is not determinative
of the guilt of respondents in the
criminal cases. The cause or
reason for the issuance of a check
is inconsequential in determining
criminal culpability under B.P. Blg.
22. What the law punishes is the
issuance of a bouncing check,
which is a malum prohibitum, and
not the purpose for which it was
issued or the terms and conditions
relating to its issuance.

Thus, whether or not the interest


rate imposed by petitioners is
eventually declared void for being
contra bonos mores will not affect
the outcome of the B.P. Blg. 22
cases because what will ultimately
be penalized is the mere issuance
of bouncing checks. The primordial
question is whether the law has
been breached, that is, if a
bouncing check has been issued.

The validity or invalidity of the


interest
rate
is
not
determinative of the guilt of
respondents in the criminal
cases.
The
Court
has
consistently declared that the
cause
or
reason
for
the
issuance
of
a
check
is
inconsequential in determining
criminal culpability under B.P.
Blg. 22. In several instances,
we have held that what the law
punishes is the issuance of a
bouncing check and not the
purpose for which it was issued
or the terms and conditions
relating to its issuance; and
that the mere act of issuing a
worthless
check
is
malum
prohibitum provided the other
elements of the offense are
properly proved. (Sps. Jose vs
Sps Suarez, G.R. No. 176795,
June 30, 2008)

Land Bank of the Philippines vs


Ramon Jacinto, G.R. No.
154622, August 3, 2010
First Womens Credit Corporation
(FWCC) obtained a loan from the
petitioner. As a security of the
loan, respondent as the president
of the corporation, issued 9
postdated checks and drawn
against FWCCs account at PNB.
Before the checks matured, both
petitioner
and
respondent
executed
several
letter
agreements, in terms of payments,
among
others,
having
been
changed or modified. When FWCC
defaulted in the payment of the
loan
obligation,
petitioner
presented for payment
to the
drawee bank as they matured.
However,
all
checks
were
dishonored for the reason Payment
Topped
or
Drawn
Against
Insufficient Funds. Respondent
also failed to make good checks
despite demands.
Hence, petitioner filed a complaint
against respondent for violation of
BP 22. Respondent denied all
charges
and
averred
that
complaint is baseless and utterly
devoid of merit as the said
obligation
has
already
been
extinguished by payment and
novation
by
virtue
of
Reconstructing
Agreement.
Respondent also invoked the
proscription
for
Involuntary
Insolvency which forbade FWCC
from paying any of its debts.
ISSUE:
Whether
exists

prejudicial

question

HELD:
NO. The High Court dismissed
respondents contention that the
novation
of
the
credit
line
agreement
was
a
prejudicial
question in the prosecution for
violation of B.P. 22. According to
the Court, the mere act of issuing
a worthless check, even if merely
as an accommodation, is covered
by B.P. 22. The agreement
surrounding
the
issuance
of
dishonored checks is irrelevant to
the prosecution for violation of B.P.
22, the gravamen of the offense
being the act of making and
issuing a worthless check or a
check that is dishonored upon its
presentment for payment. Thus,
even if it will be subsequently
declared that a novation took place
between
respondents
and
petitioner, respondents are still not
exempt from prosecution for
violation of B.P. 22 for the
dishonored checks.

It is well settled that the mere


act of issuing a worthless
check, even if merely as an
accommodation, is covered by
B.P. 22. The gravamen of the
offense punished by B.P. 22 is
the act of making and issuing a
worthless check or a check that
is
dishonored
upon
its
presentment for payment.
Thus, even if it be subsequently
declared that novation took
place between the FWCC and
petitioner, respondent is not
exempt from prosecution for
violation of B.P. 22 for the

dishonored checks. (Land Bank


of the Philippines vs Ramon
Jacinto,
G.R.
No.
154622,
August 3, 2010)

Sabandal vs Tongco, G.R. No.


124498, October 5, 2001
Facts
On February 18, 1989, Eddie B.
Sabandal
entered
into
a
memorandum of agreement on
dealership
with
respondent
Philippines Today, Inc. for the
distribution of the newspaper
Philippines Today, (now Philippine
Star) in Bacolod City and in
designated
towns
in
Negros
Occidental.
Under the agreement, petitioner
shall pay for an equivalent amount
of one month of deliveries in
advance within the first seven days
of
the
succeeding
month.
Petitioners allowable percentage of
return shall be 10% and be
entitled to a rebate of P0.15 per
copy sold.
After execution of the agreement,
respondent Philippines Today, Inc.
made regular deliveries of the
agreed copies of the newspaper to
petitioner.
In order to make partial payments
for the deliveries, on December 18,
1990 to April 15, 1991, petitioner
issued to respondent several
checks
amounting
to
ninety
thousand (P90,000.00) pesos.
When
respondent
presented
petitioners checks to the drawee
banks for payment, the bank
dishonored
the
checks
for
insufficiency
of
funds
and/or
account
closed.
Consequently,
respondent made oral and written
demands for petitioner to make
good
the
checks.
However,

petitioner failed to pay despite


demands.
In December 1992, on the basis of
a complaint-affidavit filed by
respondent Philippines Today, Inc.,
assistant city prosecutor of Manila
Jacinto A. de los Reyes, Jr. filed
with the Regional Trial Court,
Manila eleven informations for
violation of Batas Pambansa Bilang
22 against petitioner.
Three years later, or on October
11, 1995, petitioner filed with the
Regional Trial Court, Negros
Occidental at Himamaylan, a
complaint
against
Philippines
Today,
Inc.
for
specific
performance,
recovery
of
overpayment and damages.
On October 11, 1995, petitioner
also filed with the Regional Trial
Court, Manila, Branch 42, a motion
to suspend trial in the criminal
cases against him based on a
prejudicial question.
On November 27, 1995, the trial
court denied petitioners motion to
suspend
trial
based
on
a
prejudicial question.
On December 20, 1995, petitioner
filed with the trial court a motion
for reconsideration of the denial.
On January 9, 1996, the trial court
denied
the
motion
for
reconsideration.
Hence, this petition.
Issue
The issue raised is whether a
prejudicial question exists to
warrant the suspension of the trial

of the criminal cases for violation


of Batas Pambansa Bilang 22
against petitioner until after the
resolution of the civil action for
specific performance, recovery of
overpayment, and damages.
HELD:
No. The two (2) essential elements
of a prejudicial question are: (a)
the civil action involves an issue
similar or intimately related to the
issue raised in the criminal action;
and (b) the resolution of such issue
determines whether or not the
criminal action may proceed. In
this case, the issue in the criminal
cases for violation of Batas
Pambansa Bilang 22 is whether the
accused
knowingly
issued
worthless checks. The issue in the
civil
action
for
specific
performance, overpayment, and
damages is whether complainant
Sabandal overpaid his obligations
to Philippines Today, Inc. If, after
trial in the civil case, petitioner is
shown
to
have
overpaid
respondent, it does not follow that
he cannot be held liable for the
bouncing checks he issued, for the
mere issuance of worthless checks
with knowledge of the insufficiency
of funds to support the checks is
itself an offense.

If, after trial in the civil case,


petitioner is shown to have
overpaid respondent, it does
not follow that he cannot be
held liable for the bouncing
checks he issued, for the mere
issuance of worthless checks
with
knowledge
of
the

insufficiency
of
support the checks
offense. (Sabandal
G.R. No. 124498,
2001)

funds
to
is itself an
vs Tongco,
October 5,

Bobis vs Bobis, 336 SCRA 747


FACTS:
On Oct. 21, 1985, respondent
contracted a first marriage with
Maria Dulce Javier. Without said
marriage having been annulled,
respondent contracted a second
marriage with petitioner Imelda
Marbella-Bobis on Jan. 25, 1996
and a third marriage with Julia
Hernandez. An information for
bigamy
was
filed
against
respondent in the RTC of Quezon
City.
Thereafter,
respondent
initiated a civil action for judicial
declaration of absolute nullity of
first marriage and filed a motion to
suspend the criminal case for
bigamy on the ground that the
pending civil case of nullity of the
first marriage is a prejudicial
question to the criminal case. The
trial judge granted said motion.
ISSUE:
WON filing of a civil action for
declaration of nullity of a previous
marriage is a prejudicial question
in a criminal case for bigamy.
HELD:
No,
respondent
should
be
regarded as married man at the
time he contracted his second
marriage with petitioner because
he did not file a petition for the
declaration of nullity of his first
marriage before he contracted his
second marriage. Any decision that
he would acquire now in the civil
action would not erase the fact
that respondent entered into a
second
marriage
during
the
subsistence of the first marriage.

Thus a decision in the civil case is


not essential to the determination
of the criminal charge and is not a
prejudicial question.

respondent was for all legal


intents and purposes regarded
as a married man at the time
he
contracted
his
second
marriage
with
petitioner.
Against this legal backdrop,
any decision in the civil action
for nullity would not erase the
fact that respondent entered
into a second marriage during
the subsistence of a first
marriage. Thus, a decision in
the civil case is not essential to
the
determination
of
the
criminal
charge.
It
is,
therefore, not a prejudicial
question. As stated above,
respondent
cannot
be
permitted to use his own
malfeasance
to
defeat
the
criminal action against him.
(Bobis vs Bobis, 336 SCRA 747)

Landicho vs Relova, G.R. No. L225779, February 23, 1968


FACTS:
Petitioner was charged before the
Court
of
First
Instance
of
Batangas, with the offense, of
bigamy. It was alleged in the
information that petitioner "being
then lawfully married to Elvira
Makatangay, which marriage has
not been legally dissolved, did then
and there wilfully, unlawfully and
feloniously contract a second
marriage with Fe Lourdes Pasia."
An action was filed before the
same court and judge seeking to
declare her marriage to petitioner
as null and void ab initio because
of the alleged use of force, threats
and
intimidation
allegedly
employed
by
petitioner
and
because of its allegedly bigamous
character. On June 15, 1963,
petitioner as defendant in said
case, filed a third-party complaint,
against the third-party defendant
Elvira
Makatangay,
the
first
spouse, praying that his marriage
with the said third-party defendant
be declared null and void, on the
ground that by means of threats,
force
and
intimidation,
she
compelled him to appear and
contract marriage with her before
the Justice of the Peace of Makati,
Rizal.
Petitioner moved to suspend
the hearing of the criminal case
pending the decision on the
question of the validity of the two
marriages involved in the pending
civil suit. Respondent Judge denied
the motion for lack of merit

ISSUES:
Is the existence of a civil suit for
the annulment of marriage at the
instance of the second wife against
petitioner, with the latter in turn
filing a third party complaint
against the first spouse for the
annulment of the first marriage,
constitutes a prejudicial question
in a pending suit for bigamy
against him?
HELD:
No, an action for annulment
of marriage brought by the second
wife is not a prejudicial question.
Parties to a marriage should not be
permitted to judge for themselves
its nullity, only competent courts
having such authority. Prior to
such declaration of nullity, the
validity of the first marriage is
beyond question. A party who
contracts a second marriage then
assumes
the
risk
of
being
prosecuted for bigamy.

A party cannot judge by himself


the nullity of his first marriage
to justify a second marriage
before the dissolution of the
first marriage of the first
marriage and only when the
nullity of the marriage is so
declared by the courts can it be
held as void. (Landicho vs
Relova, G.R. No. L-225779,
February 23, 1968)

Beltran vs People, G.R. No.


137567, June 20, 2000
FACTS
Petitioner
was
married
to
Charmaine Felix on June 16, 1973.
After 24 years of marriage and
having four children, petitioner
filed a petition for nullity of
marriage
on
ground
of
psychological
incapacity.
Charmaine on the other hand filed
a
criminal
complaint
for
concubinage against petitioner and
his paramour. To forestall the
issuance of a warrant of arrest
from the criminal complaint,
petitioner filed for the suspension
of
the
criminal
case
on
concubinage arguing that the civil
case for the nullification of their
marriage is a prejudicial question.
ISSUE
Whether or not the civil case for
nullity
of
marriage
under
psychological incapacity is a
prejudicial question to the criminal
case of concubinage.
RULING
The rationale on the existence of
prejudicial questions is to avoid
two
conflicting
issues.
Its
requisites are 1) that a civil action
involves an issue similar or
intimately related to the issue in
the criminal action and 2) the
resolution of the issue determines
whether or not the criminal action
will proceed. In the present case,
the accused need not present a
final
judgment
declaring
his
marriage void for he can adduce
evidence in the criminal case of

the nullity of his marriage other


than the proof of a final judgment.
More importantly, parties to a
marriage should not be allowed to
judge for themselves its nullity, for
the same must be submitted to the
competent courts. So long as there
is no such final judgment the
presumption is that the marriage
exists for all intents and purposes.
Therefore he who cohabits with a
woman not his wife risks being
prosecuted for concubinage.

An action for a declaration of


nullity of marriage is not a
prejudicial
question
to
a
concubinage case.
The pendency of the case for
declaration
of
nullity
of
petitioner's marriage is not a
prejudicial question to the
concubinage case. For a civil
case
to
be
considered
prejudicial to a criminal action
as to cause the suspension of
the latter pending the final
determination of the civil case,
it must appear not only that the
said civil case involves the
same facts upon which the
criminal prosecution would be
based, but also that in the
resolution of the issue or issues
raised in the aforesaid civil
action, the guilt or innocence
of
the
accused
would
necessarily
be
determined.
(Beltran vs People, G.R. No.
137567, June 20, 2000)

Te vs CA, G.R. No. 126746,


November 29, 2000
FACTS:
In September 14, 1988, Arthur Te
and Lilliana Chu were civilly
married. However, they did not live
together after their marriage but
would meet each other regularly.
After a year, Chuagave birth to a
girl. After that, Te stopped visiting
her. In May 20,1990, Te contracted
to a second marriage while his
marriage with Chua was subsisting
Chua then filled a bigamy case
against Te and subsequently an
administrative case against Te and
his second wife, Juheb Santella.
However, Te petitioned for the
nullity of marriage with Ochua.
Hence, the RIC and Board ordered
their decision while the petition for
annulment of the first marriage
was still pending.
ISSUE:
WON the annullment case should
of resolved first before the
criminal administrative case be
decided.

HELD:
NO, the outcome of the annulment
case has no bearing as to the quill
of Te of bigamy. the ground cited
by Te for the annulment was
voidable marriage. Hence, he was
still validly married when he
committed bigamy. Thus, The
outcome of the civil case for
annulment of petitioners marriage
to private respondent had no
bearing upon the determination of
petitioners innocence or guilt in

the criminal case for bigamy,


because all that is required for the
charge of bigamy to prosper is that
the first marriage be subsisting at
the time the second marriage is
contracted.
The concept of prejudicial question
involves a civil and a criminal case.
We have previously ruled that
there is no prejudicial question
where one case is administrative
and the other is civil.

The outcome of the civil case


for annulment of petitioners
marriage to private respondent
had no bearing upon the
determination of petitioners
innocence or guilt in the
criminal
case
for
bigamy,
because all that is required for
the
charge
of
bigamy
to
prosper
is
that
the
first
marriage be subsisting at the
time the second marriage is
contracted.
The concept of prejudicial
question involves a civil and a
criminal
case.
We
have
previously ruled that there is
no prejudicial question where
one case is administrative and
the other is civil. (Te vs CA,
G.R. No. 126746, November 29,
2000)

Nature of Prejudicial Question;


purpose
Presidential Ad Hoc FactFinding Committee on Behest
Loans vs Desierto, G.R. No.
135703, April 15, 2009
Facts:
In 1992, President Fidel Ramos
created the Presidential Ad Hoc
Fact-Finding Committee on Behest
Loans, which was tasked to
conduct an inventory of all behest
loans,
determine
the
parties
involved and recommend the
appropriate action to be pursued.
The Committee investigated the
accounts of Goldent Country
Farms, Inc. (GCFI), which involved
loans
from
the
National
Investment
Development
Corporation (NIDC) and DBP. After
its investigation, the Committee
concluded
that
GCFIs
loan
transactions with NIDC and DBP
bore badges of behest loan. The
Committee filed a sworn complaint
with the Ombudsman alleging that
GCFIs loan transaction were
behest loans that violated RA
3019. Among others, Spouses
Romualdez listed as possible
individuals who could be held
liable.
In
a
counter-affidavit,
Spouses Romualdez alleged that
the
offenses
charged
had
prescribed
and
not
all
the
elements of a behest loan were
present.
The Ombudsman by a resolution
dismissed the complaint, finding
that
there
was
insufficient
evidence to warrant the indictment

of the persons charged and that


the
alleged
offenses
had
prescribed.
Issue:
a.
Whether or not the offense
had already prescribed;
b.
Whether
or
resolution of the
should be respected.

not
the
ombudsman

Held:
a.
In the matter of prescription,
the computation of the prescriptive
period for offenses involving the
acquisition of behest loans has
been laid to rest in Presidential Ad
Hoc Committee on Behest Loans v.
Hon. Desierto: [I]t was well-nigh
impossible for the State, the
aggrieved party, to have known the
violations of R.A. No. 3019 at the
time the questioned transactions
were made because, as alleged,
the public officials concerned
connived or conspired with the
beneficiaries of the loans. Thus, we
agree with the COMMITTEE that
the prescriptive period for the
offenses
with
which
the
respondents in OMB-0-96-0968
were charged should be computed
from
the
discovery
of
the
commission thereof and not from
the day of such commission.
Applying the foregoing settled
rule,
the
counting
of
the
prescriptive period commenced
from the discovery of the offenses
in 1992 after an exhaustive
investigation by the Committee.
When the complaint was filed in

1997 or after about five years,


prescription had not set in.

b.
Ordinarily, the Court will not
interfere with the Ombudsman
determination as to the existence
or non-existence of probable
cause. The rule, however, does not
apply if there is grave abuse of
discretion.
Considering
the
quantum of evidence needed to
support a finding of probable
cause, the Court holds that the
Ombudsman gravely abused his
discretion when he found such to
be
lacking
here.
Preliminary
investigation is not the occasion
for the full and exhaustive display
of the parties evidence. It is for the
presentation of such evidence only
as may engender a well-founded
belief that an offense has been
committed and that the accused is
probably
guilty
thereof.
The
validity and merits of a partys
accusation or defense, as well as
admissibility of testimonies and
evidence, are better ventilated
during the trial proper.
In the proceedings before the
Ombudsman, the Committee and
spouses
Romualdez
presented
conflicting accounts on whether
GCFI was undercapitalized and the
subject loans undercollateralized.
While the Committee found that
GCFIs capital was way below the
amounts of the loan at the time of
their approval, spouses Romualdez
countered that GCFI had infused
an additional capital of P100
million.
Moreover,
while
the
Committee
averred
that
the
appraised value of GCFIs collateral

fell inadequate as of April 29,


1977,
spouses
Romualdez
contended that GCFI furnished
additional security by causing
NIDC and DBP comptrollers to be
installed at GCFI as signatories to
all disbursements made by the
latter. Clearly, these conflicting
claims of the parties should be
resolved in a full-blown trial.

Preliminary investigation is not


the occasion for the full and
exhaustive
display
of
the
parties evidence. It is for the
presentation of such evidence
only as may engender a wellfounded belief that an offense
has been committed and that
the accused is probably guilty
thereof. The validity and merits
of a partys accusation or
defense, as well as admissibility
of testimonies and evidence,
are better ventilated during the
trial proper. (Presidential Ad
Hoc Fact-Finding Committee on
Behest Loans vs Desierto, G.R.
No. 135703, April 15, 2009)

De Chavez vs Ombudsman, G.R.


No. 168830-31, February 6,
2007
FACTS:
De Chavez, Lontok, Sr. and
Mendoza caused to be collected,
and received the proceeds of,
graduation
fees
from
the
graduating class of SY 2000-2001
without issuing an official receipt
and without remitting the same to
BSU.
The BSU management did not
collect graduation fees for the
commencement exercises of SY
2000-2001 like in the previous
years. It was claimed that the
members of the graduating class,
with
the
guidance
of
their
advisers, were the ones who fixed,
collected
and
disbursed
the
contributions/fees
for
the
commencement exercises.
No public bidding was conducted
for the rental of the caps and
gowns because the BSU did not
enter into contract with any
supplier. The graduating students
have the complete freedom to hire
their caps and gowns from anyone.
The receipts signed by Lontok, Sr.
was merely in acknowledgment of
the receipts of certain amounts
from Magnaye which the latter
requested to be given to Mr.
Fralundio Sulit from whom the
graduating class rented their caps
and gowns.
The graduation fees collected and
the expenses paid out of these fees
during the calendar year 2000
could not be determined due to
failure of the employee concerned

to furnish this Office with certified


statement
of
collections
of
graduation fees and the related
disbursements together with the
supporting papers despite our
request to the College President in
a letter dated January 17, 2001.
Since graduation fee is one of the
items to be recorded under Special
Trust Fund per R.A. No. 8292,
failure to record the same in the
books of accounts of the Institute
understated the cash and trust
liability accounts.
Since there were no records
submitted to this Office pertaining
to graduation fees collected from
graduating
students,
understatement of Cash and Trust
Liability Accounts amounting to
P1,284,950.00 (Annex G) was
based on the number graduating
students and the graduation fee
per student last school year 19992000. The 1999 Annual Audit
Report of the previous COA
Auditor reported a total collections
from
1997
to
1999
of
P2,057,600.00. These amounts
when
added
will
yield
an
aggregate understatement of Cash
and Trust Liability accounts by
about P3,342,550.00.
ISSUE:
Whether RESPONDENT OFFICE
OF
THE
OMBUDSMAN
COMMITTED GRAVE ABUSE OF
DISCRETION
AND
ACTED
WITHOUT
JURISDICTION
IN
FINDING PETITIONERS ALREADY
LIABLE
FOR
CRIMINAL
OFFENSES

HELD:
The petitioners alleged that the
public respondents Supplemental
Resolution dated 12 July 2005
categorically
stated
that
petitioners are liable for the
criminal acts complained of; that
the public respondent did not even
discuss the matter of probable
cause but instead immediately
ruled on their guilt; that the said
resolution did not state or instruct
the filing of the appropriate
criminal informations against them
before the courts of justice. Hence,
the
public
respondents
instantaneous finding of criminal
liability on their part renders any
trial against them an exercise in
futility which inevitably clashes
with Section 14(2) of the 1987
Constitution which grants to the
accused, inter alia, the right to
have a speedy, impartial and public
trial.
Therefore,
the
public
respondent had exceeded its
jurisdiction under Republic Act No.
6770, otherwise known as the
Ombudsman Act of 1989, since
there is nothing in the said statute
which grants to it the power to
determine the guilt or innocence of
the accused.
A preliminary investigation is
merely inquisitorial, and it is often
the only means of discovering the
persons who may be reasonably
charged with a crime, to enable
the prosecutor to prepare his
complaint or information. It is not
a trial of the case on the merits
and has no objective except that of
determining whether a crime has
been committed and whether there
is probable cause to believe that

the respondent is guilty thereof. In


the
conduct
of
preliminary
investigation, the prosecutor does
not decide whether there is
evidence beyond reasonable doubt
of the guilt of respondent. A
prosecutor merely determines the
existence of probable cause, and to
file the corresponding information
if he finds it to be so.

In the conduct of preliminary


investigation, the prosecutor
does not decide whether there
is evidence beyond reasonable
doubt
of
the
guilt
of
respondent.
A
prosecutor
merely
determines
the
existence of probable cause,
and to file the corresponding
information if he finds it to be
so. (De Chavez vs Ombudsman,
G.R. No. 168830-31, February
6, 2007)

Manebo vs Acosta, G.R. No.


169554, October 28, 2009
FACTS:
A complaint for murder was filed
by Nieva Manebo (Manebo), sister
of the victim, against respondents
Acosta and Sapiandante before the
Special Action Unit (SAU) of the
National Bureau of Investigation
(NBI).
The findings of the SAU
recommending the filing of a
murder case against respondents
and a certain John Doe was
referred to the Office of the Chief
State
Prosecutor
(OCSP),
Department of Justice (DOJ), for
preliminary
investigation.
Respondents, in turn, filed directly
with the DOJ a counter-charge of
perjury, offering false witness and
violation of Presidential Decree
(PD) No. 1829 against Manebo,
Bagasan, and Sardia.
Respondents
filed
their
motion for reconsideration, which
was denied. Respondents filed
their
appeal
with
the
DOJ
Secretary that issued a resolution
reversing the appealed resolution.
The DOJ claimed that the evidence
against respondents Acosta and
Sapiandante lack the required
quantum of proof sufficient to
indict them for the offense
charged. The DOJ Secretary found
that the police report prepared
after the killing incident stated
that the person seated beside the
victim,
who
was
watching
television when shot, was Liza
Gragasan. However, the DOJ
Secretary continued that more

than four months after the


incident, a witness appeared in the
person of Flordeliza Bagasan who
claimed to be seated beside, and
witnessed the actual shooting of,
the victim. The DOJ Secretary
found Flordeliza's description of
respondent Acosta different from
the latter's physical attributes. He
then
ruled
that
Flordeliza's
delayed testimony, coupled with
her erroneous description of
respondent Acosta, cast a cloud of
doubt on her credibility.
The
DOJ Secretary also did not give
credence to witness Sardia's
testimony
on
respondent
Sapiandante's participation in the
incident. He found that Sardia was
not among those mentioned in the
police
report,
and
that
his
testimony was likewise belatedly
executed without any reason given
for such delay; that fear could not
have been Sardia's reason, since in
June 1998, he had already filed a
complaint for attempted murder
against respondent Sapiandante,
which was later dismissed; and
that Sardia did not witness the
actual shooting of the victim.
Petitioner filed an appeal
with the Office of the President
(OP) which dismissed the appeal
and affirming in toto the resolution
of the DOJ Secretary. Petitioner's
motion for reconsideration was
denied by the OP. Aggrieved,
petitioner filed a petition for
certiorari under Rule 43 with the
CA. The CA dismissed the petition
for lack of merit.
ISSUES:

1.
Is it appropriate for the
Supreme Court to determine the
existence of probable cause?
2.
Is there probable cause to
indict respondent for murder?
HELD:
1.
Yes,
the
present
case
warrants the application of the
exception.
Ordinarily, the determination of
probable cause is not lodged with
this Court. Its duty in an
appropriate case is confined to the
issue of whether the executive or
judicial determination, as the case
may be, of probable cause was
done without or in excess of
jurisdiction or with abuse of
discretion amounting to want of
jurisdiction. However, this Court
may
ultimately
resolve
the
existence or non-existence of
probable cause by examining the
records
of
the
preliminary
investigation when necessary for
the orderly administration of
justice.
2.
Yes, there was probable
cause to indict the respondents for
murder.
While the initial police report
stated that the name of the person
who was seated beside the victim
when the latter was shot was Liza
Gragasan, such report would not
conclusively establish that Liza
Gragasan could not have been
Flordeliza Bagasan, the witness
who executed an affidavit four
months after the incident. Notably,
Flordeliza's nickname is Liza, and
her surname Bagasan sounds

similar to Gragasan. Under the


rule of idem sonans, two names
are said to be "idem sonantes" if
the attentive ear finds difficulty in
distinguishing
them
when
pronounced. Thus, the presence of
Bagasan at the crime scene was
established,
contrary
to
the
conclusion arrived at by the DOJ
Secretary.
Moreover, a witness' delay in
reporting what she knows about a
crime does not render her
testimony false or incredible, for
the delay may be explained by the
natural reticence of most people to
get involved in a criminal case.
The failure of the police
report to mention Sardia's name as
a witness would not detract from
the fact that he saw respondent
Acosta with an unidentified man
running away from the chapel and
riding the waiting get- away
vehicle driven by Sapiandante.
Entries in a police blotter, though
regularly done in the course of the
performance of official duty, are
not conclusive proof of the truth of
such entries and should not be
given
undue
significance
or
probative value for they are
usually incomplete and inaccurate.
The matter of assigning
value to the declaration of a
witness is best done by the trial
court, which can assess such
testimony in the light of the
demeanor, conduct and attitude of
the witness at the trial stage.]
We need not over-emphasize
that in a preliminary investigation,
the public prosecutor merely
determines
whether
there
is

probable
cause
or
sufficient
ground to engender a well-founded
belief that a crime has been
committed,
and
that
the
respondent is probably guilty
thereof and should be held for
trial. Considering the foregoing,
we find that the CA erred in
affirming the DOJ's finding of the
absence of probable cause to
indict respondents for murder.

Probable
cause
implies
probability
of
guilt
and
requires
more
than
bare
suspicion
but
less
than
evidence to justify a conviction.
(Manebo vs Acosta, G.R. No.
169554, October 28, 2009)

Spouses Balanguan vs CA, G.R.


No. 174350, August 13, 2008
Ketherene Balanguan is a Premier
Customer Service Representative
(PCSR) of HSBC, which handles
premier accounts of the latter. One
Roger York, placed 2.5M dollars in
time deposit on the said bank
which
Ketherene
handled.
However, upon surprise when he
inquired in one the banks
branches, there was no record of
his account. It was then found out
that the transactions were all done
through Ketherenes computer. So
as not to ruin its name and
goodwill
among
its
clients,
respondent HSBC reimbursed York
the 2.5M.
Based on the foregoing factual
circumstances, respondent HSBC,
filed a criminal complaint for
Estafa and/or Qualified Estafa
before the Office of the City
Prosecutor. Following the requisite
preliminary
investigation,
the
Office of the City Prosecutor, in a
resolution, found no probable
cause to hold petitioners liable to
stand trial. On a later date,
respondent HSBC appealed the
resolution to the Secretary of the
DOJ by means of a Petition for
Review, which was denied, as well
as the subsequent motion for
reconsideration.
Undaunted,
respondent HSBC then went to the
CA by mean of a Petition for
Certiorar under Rule 65. The CA
annulled the decisions of the DOJ.
Issue:
Whether or not the CA acted in
grave abuse of discretion when it

annulled the decision/resolution of


the DOJ.
Held:
The general rule is that the
resolutions of the DOJ cannot be
reviewed by the courts, since it is
not a quasi-judical office. However,
it admits of an exception. It can be
annulled when it was done in
excess of authority by way of
Petition for Certiorari under Rule
65
while it is this Courts general
policy not to interfere in the
conduct
of
preliminary
investigations,
leaving
the
investigating officers sufficient
discretion to determine probable
cause, we have nonetheless made
some exceptions to the general
rule, such as when the acts of the
officer are without or in excess of
authority, resulting from a grave
abuse of discretion. Although there
is no general formula or fixed rule
for the determination of probable
cause, since the same must be
decided in the light of the
conditions obtaining in given
situations
and
its
existence
depends to a large degree upon
the finding or opinion of the judge
conducting the examination, such
a finding should not disregard the
facts before the judge (public
prosecutor) or run counter to the
clear dictates of reason Try as we
might, this Court cannot find grave
abuse of discretion on the part of
the Court of Appeals, when it
reversed and set aside the
resolutions of the DOJ. There is no
showing that the appellate court

acted in an arbitrary and despotic


manner, so patent or gross as to
amount to an evasion or unilateral
refusal to perform its legally
mandated duty. On the contrary,
we find the assailed decision and
resolution of the Court of Appeals
to be more in accordance with the
evidence on record and relevant
laws and jurisprudence than the
resolutions of the DOJ.

Probable
cause
has
been
defined as the existence of such
facts and circumstances as
would
excite
belief
in
a
reasonable mind, acting on the
facts within the knowledge of
the prosecutor, that the person
charged was guilty of the crime
for which he was prosecuted. A
finding
of
probable
cause
merely binds over the suspect
to stand trial. It is not a
pronouncement
of
guilt.
(Spouses Balanguan vs CA, G.R.
No. 174350, August 13, 2008)

Samuel Lee vs KBC Bank, G.R.


No. 169554, October 28, 2009
Facts:
Petitioners (Lee and Lim) were
assistant treasurers of Midas
Diversified Export Corporation
(MDEC) who obtained a loan from
KBC, which the former defaulted
in payment. On a later date, Lee
and Lim was charged with estafa
by KBC. In a resolution of the state
prosecutor, he found probable
cause and recommended that two
counts of estafa be filed against
Lim and Lee. Accordingly, two
informations for estafa against Lee
and Lim were filed with the RTC.
After finding probable cause,
Judge Dumayas of RTC issued
warrantsof arrest against Lee and
Lim. Lee and Lim filed a petition
for review with the DOJ and
challenged the resolution of the of
the
State
Prosecutor.
In
a
resolution by the Secretary of
Justice, it directed the withdrawal
of the information against Lee and
Lim on the ground that the
evidence presented by the State
prosecutor constituted hearsay
evidence.
In a one-page order, the RTC
granted the motion to withdraw
the informations against Lee and
Lim in this wise: This Court, after
an in-depth scrutiny of the
arguments
raised
by
the
prosecution
and
private
complainant, finds the contentions
of the prosecution to be sufficient
and meritorious.
Issue:

Whether or not the trial court did


not abdicate its duty to determine
the sufficiency of the prosecutions
reason
for
withdrawing
the
information.
Held:
In Ark Travel Express vs. The
Presiding Judge of Makati it was
held that: It is settled that when
confronted with a motion to
withdraw an Information on the
ground of lack of probable cause
based on a resolution of the
Secretary of the Department of
Justice, the bounden duty of the
trial court is to make an
independent assessment of the
merits of such motion. Having
acquired jurisdiction over the
case, the trial court is not
bound by such resolution but is
required to evaluate it before
proceeding further with the
trial and should embody such
assessment
in
the
order
disposing the motion.
In the present case, Judge
Dumayas, in his 26 March 2003
Order, did not (1) positively state
that the evidence against Lee and
Lim is insufficient, (2) include a
discussion of the merits of the
case, (3) assess whether Secretary
Perezs conclusion is supported by
evidence, (4) look at the basis of
Secretary Perezs recommendation,
(5) embody his assessment in the
order, and (6) state his reasons for
granting the motion to withdraw
the informations . Judge Dumayas
failure to make his own evaluation
of the merits of the case violates
KBC Banks right to due process

and constitutes grave abuse of


discretion. Judge Dumayas 26
March 2003 Order granting the
motion
to
withdraw
the
informations is void.

The presence or absence of the


elements of the crime is
evidentiary in nature and is a
matter of defense that may be
passed upon after a full-blown
trial on the merits. In fine, the
validity and merits of a partys
defense or accusation, as well
as
the
admissibility
of
testimonies and evidence, are
better ventilated during trial
proper than at the preliminary
investigation level. (Samuel Lee
vs KBC Bank, G.R. No. 169554,
October 28, 2009)

Sales vs Sandiganbayan, G.R.


No. 143802, November 16,
2001
Facts:
Herein petitioner is the then
incumbent mayor of Pagudpud,
Ilocos Norte (Year 1999), who
fatally shot the former mayor and
his political rival, Atty. Rafael
Benemerito. After the shooting
incident, petitioner surrendered
and placed himself under the
custody of the municipal police
then asked that he be brought to
the Provincial PNP Headquarters.
The
next
day,
Police
Chief
inspector Crispin Agno and private
respondent Thelma Benemerito,
wife of the victim, filed a criminal
complaint for Murder against
petitioner at MCTC of Bangui,
Ilocos Norte, presided by Judge
Melvin U. Calvan. On a later date,
Judge
Calvan
forwarded
the
records of the case of the Office of
the Provincial Prosecutor of Ilocos
Norte for appropriate action.
Respondent Judge did not conduct
the requisite investigation prior to
issuance of the arrest warrant. The
Rules require an examination in
writing under oath in the form of
searching questions and answers.
The statements of witnesses were
not sworn before him but before
the Provincial Prosecutor. The
purported
transcript
of
stenographic notes does not bear
the signature of the stenographer.
Moreover, he did not complete the
preliminary
investigation.
He
claimed to have examined only the
witnesses of the complainant. He
issued a Resolution and forwarded

the records to the Provincial


Prosecutor without giving the
accused
(petitioner)
an
opportunity to submit counteraffidavits
and
supporting
documents.
Meanwhile, after receipt of the
records of the case from Judge
Calvan as well as petitioneraccuseds counter-affidavits, the
Ilocos Norte Provincial Prosecutor,
instead
of
conducting
a
preliminary investigation of his
own, merely forwarded the said
records to the Ombudsman for the
latter to conduct the same.
Issue:
Whether or not the Sandiganbayan
gravely abused its discretion when
it
relied
on
an
incomplete
preliminary
investigation
conducted by the Ombudsman.
Held:
It was patent error for the
Sandiganbayan to have relied
purely
on
the
Ombudsmans
certification of probable cause
given the prevailing facts of this
case much more so in the face of
the latters flawed report and onesided factual findings. In the order
of procedure for criminal cases,
the task of determining probable
cause for purposes of issuing a
warrant of arrest is a responsibility
which is exclusively reserved by
the Constitution to judges. People
v. Inting clearly delineated the
features of this constitutional
mandate,
viz:
1.]
The
determination of probable cause is
a function of the judge; it is not for

the provincial fiscal or prosecutor


to ascertain. Only the judge and
the judge alone makes this
determination; 2.] The preliminary
inquiry made by a prosecutor does
not bind the judge. It merely
assists
him
in
making
the
determination of probable cause. It
is the report, the affidavits, the
transcripts of stenographic notes,
if any, and all other supporting
documents behind the prosecutors
certification which are material in
assisting
the
judge
in
his
determination of probable cause;
and 3.] Judges and prosecutors
alike
should
distinguish
the
preliminary
inquiry
which
determines probable cause for the
issuance of a warrant of arrest
from the preliminary investigation
proper which ascertains whether
the offender should be held for
trial or be released. Even if the
two inquiries be made in one and
the same proceeding, there should
be no confusion about their
objectives. The determination of
probable cause for purposes of
issuing the warrant of arrest is
made
by
the
judge.
The
preliminary investigation proper
whether or not there is reasonable
ground to believe that the accused
is guilty of the offense charged
and, therefore, whether or not he
should be subjected to the
expense,
rigors
and
embarrassment of trial is the
function of the prosecutor.
Stated differently, while the task of
conducting
a
preliminary
investigation is assigned either to
an inferior court magistrate or to a
prosecutor, only a judge may issue

a warrant of arrest. When the


preliminary
investigation
is
conducted by an investigating
prosecutor, in this case the
Ombudsman, the determination of
probable
cause
by
the
investigating prosecutor cannot
serve as the sole basis for the
issuance by the court of a warrant
of arrest. This is because the court
with whom the information is filed
is tasked to make its own
independent
determination
of
probable cause for the issuance of
the warrant of arrest.

The purpose of a preliminary


investigation or a previous
inquiry of some kind, before an
accused person is placed on
trial, is to secure the innocent
against hasty, malicious and
oppressive prosecution and to
protect him from an open and
public accusation of a crime,
from the trouble, expenses and
anxiety of a public trial. It is
also intended to protect the
state from having to conduct
useless and expensive trials.
(Sales vs Sandiganbayan, G.R.
No. 143802, November 16,
2001)

Ledesma vs CA, 278 SCRA 656


A complaint for libel was filed by
Dr. Juan F. Torres, Jr. against Dr.
Rhodora M. Ledesma, petitioner
herein, before the Quezon City
Prosecutors
Office.
Finding
sufficient legal and factual basis,
the Quezon City Prosecutors Office
filed
on
July
6,
1992
an
Information
for
libel
against
petitioner with the Regional Trial
Court of Quezon City, Branch 104.
Ledesma
was
said
to
have
submitted a letter addressed to
Dr. Esperanza I. Cabral, Director
of Philippine Heart Center for
unfair and inhuman conditions she
went through as a Consultant. She
claimed that there was an unequal
distribution of labor and the
schedule of duties were not strictly
followed. As it was, the schedule of
duties submitted monthly to the
office of the Asst. Director for
Medical Services was simply a
dummy
to
comply
with
administrative
requirements
rather than a guideline for strict
compliance. Both consultants, Dr.
Torres and Dr. Monzon,
have
complete daily time records even if
they did not come regularly. Dr.
Torres came for an hour every
week,
Dr.
Monzon
came
sporadically during the week while
Ledesma was left with everything
from training the residents and
supervising
the
Techs
to
processing and interpreting the
results on a regular basis. A
petition
for
review
of
the
resolution
of
Assistant
City
Prosecutor was filed by petitioner
before the Department of Justice
which was granted and further

directed
the
Quezon
City
prosecutor to move for deferment
of further proceedings and to
elevate the entire records of the
case. Without the consent or
approval of the trial prosecutor,
private complainant filed a Motion
to Lift the Order and to Set the
Case for Arraignment/Trial. The
trial court issued an Order setting
aside its earlier Order deferring
the proceedings and scheduling
petitioners
arraignment
on
January 18, 1993 at two oclock in
the afternoon. In a resolution
dated January 27, 1993, then
Justice Secretary reversed the
Quezon
City
investigating
prosecutor on the ground that they
are
mere
manifestations
of
Ledesmas
earnest
desire
to
pursue proper relief for the alleged
injustice
she
got
from
complainants. If she was motivated
by malice and ill-will in sending
the subject communication to the
Director of the PHCA, she would
not have sent the second letter and
filed the administrative and civil
cases against complainants.

Moreover, it is unbelievable that it


took complainants one year to
realize that the questioned letter
subjected them to public and
malicious imputation of a vice or
omission. It is beyond the ordinary
course of human conduct for
complainants to start feeling the
effects of the alleged libelous
letter - that of experiencing
sleepless nights, wounded feelings,
serious anxiety, moral shock and
besmirched reputation - one year

after they read the communication


in question. It was further ordered
to file a motion to withdraw the
Infromation filed against Ledesma
but it was denied by the court.
Petitioner filed a petition for
certiorari and prohibition with the
Supreme Court. In a Resolution
dated March 31, 1993, this Court
referred the case to the Court of
Appeals for proper determination
and
disposition
pursuant
to
Section 9, paragraph 1 of B.P. 129.

Respondent Court dismissed the


petition for lack of merit, holding
that it had no jurisdiction to
overturn the doctrine laid down in
Crespo vs. Mogul -- once a
complaint or information has been
filed in court, any disposition of
the case, i.e., dismissal, conviction
or acquittal of the accused, rests
on the sound discretion of the trial
court.
Issue:
WON the Judge committed a grave
abuse of discretion in denying the
motion
to
withdraw
the
information
Held:
The determination of probable
cause
during
a
preliminary
investigation
is
judicially
recognized
as
an
executive
function and is made by the
prosecutor. Sound policy supports
this distinction. Otherwise, judges
would be unduly laden with the
preliminary
examination
and
investigation
of
criminal
complaints
instead
of

concentrating on hearing and


deciding cases filed before their
courts. The primary objective of a
preliminary investigation is to free
a
respondent
from
the
inconvenience, expense, ignominy
and
stress
of
defending
himself/herself in the course of a
formal trial, until the reasonable
probability of his or her guilt has
been passed upon in a more or less
summary
proceeding
by
a
competent officer designated by
law for that purpose. Secondarily,
such summary proceeding also
protects the state from the burden
of unnecessary expense and effort
in prosecuting alleged offenses
and in holding trials arising from
false, frivolous or groundless
charges.
The trial judge committed grave
abuse of discretion when it denied
the motion to withdraw the
information, based solely on his
bare and ambiguous reliance on
Crespo. The trial courts order is
inconsistent with the courts
repetitive calls for an independent
and competent assessment of the
issue presented in the motion to
dismiss. The trial judge was tasked
to
evaluate
the
secretarys
recommendation
finding
the
absence of probable cause to hold
petitioner criminally liable for
libel. He failed to do so. He merely
ruled to proceed with the trial
without stating his reasons for
disregarding
the
secretarys
recommendation.

Had he complied with his judicial


obligation,
he
would
have

discovered that there was, in fact,


sufficient ground to grant the
motion
to
withdraw
the
information. The documents before
the trial court judge clearly
showed that there was no probable
cause to warrant a criminal
prosecution for libel

The primary objective of a


preliminary investigation is to
free a respondent from the
inconvenience,
expense,
ignominy
and
stress
of
defending himself/herself in
the course of a formal trial,
until the reasonable probability
of his or her guilt has been
passed upon in a more or less
summary
proceeding
by
a
competent officer designated
by
law
for
that
purpose.
Secondarily,
such
summary
proceeding also protects the
state from the burden of
unnecessary expense and effort
in prosecuting alleged offenses
and in holding trials arising
from
false,
frivolous
or
groundless charges. (Ledesma
vs CA, 278 SCRA 656)

Marinas vs Siochi, 104 SCRA


423
The present controversy arose out
of the issuance by the Municipal
Court of Pasig, Rizal, of a Writ of
Execution in s civil case for
Ejectment.
Petitioner
Antonio
Marinas, Deputy Sheriff of Rizal,
with his co-petitioners Antonio
Montano and Gregorio Rupisan
enforced said Writ of Execution by
levying
upon
the
personal
properties and chattels of private
respondents Victoria Lasin Vda. de
Atienza and Rosario L. Atienza,
and taking out said properties
from their (respondents') rented
house. Respondents were also
ejected from said house. On the
same date, respondent Victoria
Lasin Vda, de Atienza reported to
the police authorities of Pasig that
her jewelry worth P590.00 had
been taken by petitioners without
issuing any receipt therefor, and
in connection therewith, she
executed a written Statement
which was sworn to before Special
Counsel
Lucila
P.
Alcoba.
Respondents re-entered the house
after securing a Court Order.
Respondent Rosario L. Atienza
then discovered that several pieces
of her jewelry and other personal
items, with a total value of
P1,018.00, were missing. She
reported the loss to the authorities
on February 2, 1966, and her
Statement
was
taken.
She
subscribed and swore to the same
before respondent Municipal Judge
Andres S. Siochi. Respondents,
armed with
a
Court
Order
authorizing them to enter the
premises of the said house, did so

again to get their remaining


unlevied properties. They claimed,
however, that on the said date
petitioners and their companions
forcibly compelled them to deliver
the unlevied personal properties
found therein, hauled said articles
into a truck and left. Private
respondents reported the incident
to the police authorities at Pasig.
Victoria
Lasin
executed
a
Statement
alleging
that
the
personal properties forcibly taken
from
them
by
petitioners,
amounting to P2,645.00, were not
included in the levy. Her son,
Tranquilino Atienza, also executed
an Affidavit corroborating her
declaration.
Both Statements
were subscribed and sworn to
before respondent Judge.

On February 7, 1966, two separate


charges for Theft were filed
against petitioners and Carlos
Quintana before the Municipal
Court of Pasig, Rizal, respondent
Judge, presiding. A Complaint for
Grave Coercion was also lodged
against petitioners and three Does
on the same date.
The three
Complaints were filed by Lt. Jose
S. Lontoc, Chief of the Criminal
Investigation Section of the Police
Department of Pasig, Rizal, for and
on behalf of the Chief of Police.
These Complaints contained an
annotation on the lower left hand
corner
reading:
"APPROVED
AFTER
PRELIMINARY
EXAMINATION: (SGD) Lucila P.
Alcoba, Special Counsel." The
Complaints for Theft and Grave
Coercion,
respectively,
were

subscribed and sworn to by Lt.


Jose S. Lontoc before respondent
Judge. One Complaint for Theft
does not show the jurat on its face,
but respondents state that it was
also attested to by Lt. Lontoc
before respondent Judge and that
this appears on the dorsal side of
the Complaint. On February 8,
1966, warrants for the arrest of
petitioners
were
issued
by
respondent Judge in all three cases
after
preliminary
examination
conducted by and by Special
Counsel Lucila P. Alcoba

Issue:
WON there can be due process
without the presence of the
accused during the preliminary
investigation
Held:
What was conducted by the
respondent Judge in these cases is
the preliminary examination before
the issuance of a warrant of arrest
pursuant to section 1, Rule 112.
The 1935 Constitution, in section l
(3), Article III provides that no
warrant shall be issued but upon
probable cause to be determined
by the Judge after examination of
witnesses
under
oath
or
affirmation of the complaint and
the witnesses he may produce.
Conformably thereto, Section 87,
paragraph 3, of the Judiciary Act,
as amended by Republic Act No.
3828, provides that: before a
Municipal Judge may issue a
warrant of arrest, the following
conditions must first be fulfilled:
(1) he must examine the witness or

witnesses personally; (2) the


examination must be under oath;
and (3) the examination must be
reduced to writing in the form of
searching questions and answers.
Respondent Judge was satisfied
that the questions and answers in
a previous investigation by Special
Counsel Alcoba partook of the
nature of his searching questions
and answers and made them his
own.
The
preliminary
examination
referred to is defined, under
Section
l
of
Rule
112
as
examination made before the
arrest of the accused by a Judge,
with whom a Complaint or
Information
has
been
filed
imputing the commission of an
offense cognizable by the Court of
First Instance, for the purpose of
determining whether there is a
reasonable ground to believe that
an offense has been committed and
the accused is probably guilty
thereof, so that a warrant of arrest
may be issued and the accused
held for trial. This section does not
refer
to
the
preliminary
investigation proper provided for
under Section 10, Rule 112, in
which the accused is given access
to the testimony and evidence
presented against him at the
preliminary examination, and to
present evidence if he so desires.
From Section 5 of Rule 112, it is
clear
that,
unlike
in
the
preliminary investigation proper,
an accused is not entitled as a
matter of right to be present,
during
the
preliminary
examination nor to cross-examine
the witnesses presented against

him before his arrest, the purpose


of said examination being merely
to determine whether or not there
is sufficient reason to issue a
warrant of arrest. Section l (3),
Article III of the 1935 Constitution
commanding the determination of
probable cause prior to issuance of
a warrant arrest, requires no
notice
to
an
accused.
A
preliminary
examination
is
generally a proceeding ex-parte in
which the person charged has no
right to participate or be present.
The right to confrontation of
witnesses neither applies to a
preliminary hearing. While section
l (3) Art. III of the 1935
Constitution does require, before
the issuance of a warrant of arrest,
the determination of probable
cause
by
the
Judge
after
examination of witnesses he may
produce, the curtailment of the
presence of an accused during that
preliminary examination entails no
infringement of the constitutional
right to due process of law nor to
equal protection of the laws.
Attention should also be called to
the fact that neither the 1935 nor
the 1973 Constitution requires the
holding
of
a
preliminary
investigation. lt is settled doctrine
that the right hereto is of statutory
character and may be invoked only
when
specifically
created
by
statute. lt is not a fundamental
right and may be waived expressly
or by silence

The fact that neither the 1935


nor the 1973 Constitution
requires the holding of a
preliminary investigation. lt is

settled doctrine that the right


hereto is of statutory character
and may be invoked only when
specifically created by statute.
(Marinas vs Siochi, 104 SCRA
423)

Nature of the Right to a


Preliminary Investigation
Doromal vs Sandiganbayan, 177
SCRA 354
FACTS:
Quintin S. Doromal, a former
Commissioner of the Presidential
Commission on Good Government
(PCGG), for violation of the AntiGraft and Corrupt Practices, in
connection with his shareholdings
and position as president and
director
of
the
Doromal
International Trading Corporation
(DITC) which submitted bids to
supply
electronic,
electrical,
automotive,
mechanical
and
airconditioning equipment to the
Department of Education, Culture
and Sports (or DECS) and the
National Manpower and Youth
Council (or NMYC).
An information was then filed by
the Tanodbayan against Doromal
for the said violation and a
preliminary
investigation
was
conducted.
The petitioner then filed a petition
for certiorari and prohibition
questioning the jurisdiction of the
Tanodbayan
to
file
the
information without the approval
of the Ombudsman.
The Supreme Court held that the
incumbent
Tanodbayan
(called
Special Prosecutor under the 1987
Constitution and who is supposed
to retain powers and duties NOT
GIVEN to the Ombudsman) is
clearly
without
authority
to
conduct preliminary investigations
and to direct the filing of criminal
cases with the Sandiganbayan,

except
upon
orders
of
the
Ombudsman.
Subsequently
annulling the information filed by
the Tanodbayan.

both informations involve the same


subject matter.

A new information, duly approved


by the Ombudsman, was filed in
the Sandiganbayan, alleging that
the Doromal, a public officer, being
then a Commissioner of the
Presidential Commission on Good
Government, did then and there
wilfully and unlawfully, participate
in a business through the Doromal
International Trading Corporation,
a family corporation of which he is
the President, and which company
participated
in
the
biddings
conducted by the Department of
Education, Culture and Sports and
the National Manpower & Youth
Council, which act or participation
is prohibited by law and the
constitution.

(3) Whether or not the information


shall be effected as invalid due to
the
absence
of
preliminary
investigation.

The petitioner filed a motion to


quash the information on the
ground that it was invalid since
there had been no preliminary
investigation
for
the
new
information that was filed against
him.
The motion was denied by
Sandiganbayan
claiming
that
another preliminary investigation
is unnecessary because both old
and new informations involve the
same subject matter.
ISSUES:
(1) Whether or not the act of
Doromal
would
constitute
a
violation of the Constitution.
(2) Whether or not preliminary
investigation is necessary even if

(1) YES. The presence of a signed


document bearing the signature of
Doromal as part of the application
to bid shows that he can rightfully
be
charged
with
having
participated in a business which
act is absolutely prohibited by
Section 13 of Article VII of the
Constitution" because "the DITC
remained a family corporation in
which Doromal has at least an
indirect interest."
Section 13, Article VII of the 1987
Constitution provides that "the
President,
Vice-President,
the
members of the Cabinet and their
deputies or assistants shall not...
during (their) tenure, ...directly or
indirectly... participate in any
business.
(2) YES. The right of the accused
to a preliminary investigation is "a
substantial one." Its denial over his
opposition is a "prejudicial error, in
that it subjects the accused to the
loss of life, liberty, or property
without due process of law"
provided by the Constitution.
Since the first information was
annulled,
the
preliminary
investigation conducted at that
time shall also be considered as
void. Due to that fact, a new
preliminary investigation must be
conducted.

(3) NO. The absence of preliminary


investigation does not affect the
court's jurisdiction over the case.
Nor do they impair the validity of
the information or otherwise
render it defective; but, if there
were no preliminary investigations
and
the
defendants,
before
entering their plea, invite the
attention of the court to their
absence, the court, instead of
dismissing the information should
conduct such investigation, order
the fiscal to conduct it or remand
the case to the inferior court so
that the preliminary investigation
may be conducted.

The preliminary investigation


in criminal cases is not a
creation of the Constitution; its
origin is statutory and it exists
and the right thereto can be
invoked when so established
and granted by law. (Doromal
vs Sandiganbayan, 177 SCRA
354)

Duterte vs Sandiganbayan, 289


SCRA 721
FACTS: Petitioners were charged
before the Sandiganbayan for
violating Sec. 3(g) of R.A. No.
3019, otherwise known as the AntiGraft And Corrupt Practices Act
for allegedly entering into an
anomalous
contract
for
the
purchase of computer hardware
and accessories with the Systems
Plus, Incorporated.
It appears that four years prior to
filing of the information before the
Sandiganbayan, petitioners were
merely directed to submit a pointby-point comment under oath on
the allegations in a civil case filed
against them before the RTC and
on the allegations in an unverified
complaint
filed
before
the
Ombudsman by the Anti-Graft
League. Petitioners had no inkling
that they were being subjected to
a preliminary investigation as in
fact there was no indication in the
order
that
a
preliminary
investigation was being conducted.
Petitioners filed a motion a motion
for reconsideration alleging among
others that they were deprived of
their right to a preliminary
investigation, due process and the
speedy disposition of their case,
which the Sandiganbayan denied.
They filed a motion to quash but
the same was denied by the
Sandiganbayan.
Hence this petition.
ISSUE: W/N the petitioners right
to speedy trial was violated by the
inordinate delay in the conduct of
the preliminary investigation?

HELD:
YES. The preliminary investigation
of the charges against petitioners
has been conducted not in the
manner
laid
down
in
Administrative Order No. 07. The
inordinate delay in the conduct of
the preliminary investigation
infringed
upon
their
constitutionally guaranteed right
to a speedy disposition of their
case. In Tatad vs. Sandiganbayan,
we held that an undue delay of
close to three (3) years in the
termination of the preliminary
investigation in the light of the
circumstances obtaining in that
case warranted the dismissal of
the case.
Petitioners in this case, however,
could not have urged the speedy
resolution of their case because
they were completely unaware that
the investigation against them was
still on-going.
Peculiar to this
case, we reiterate, is the fact that
petitioners were merely asked to
comment, and not file counteraffidavits which is the procedure to
follow
in
a
preliminary
investigation.
After giving their
explanation and after four long
years of being in the dark,
petitioners, naturally, had reason
to assume that the charges against
them had already been dismissed.
Finally, under the facts of the case,
there is no basis in the law or in
fact to charge petitioners for
violation of Sec. 3(g) of R.A. No.
3019. To establish probable cause
against the offender for violation of
Sec. 3(g), the following elements

must be present: (1) the offender


is a public officer; (2) he entered
into a contract or transaction in
behalf of the government; (3) the
contract or transaction is grossly
and manifestly disadvantageous to
the government.

The
right
to
preliminary
investigation is not a mere
formal right, it is a substantive
right. To deny the accused of
such right would be to deprive
him of due process. (Duterte vs
Sandiganbayan, 289 SCRA 721)

Cruz vs People, 233 SCRA 439


FACTS:
The Government Service Insurance
System (GSIS) filed two separate
criminal
complaints
against
petitioner Roman A. Cruz, Jr., a
former public official, for violation
of Section 3(e) of Republic Act No.
3019. The first complaint against
petitioner was filed with the Office
of the Special Prosecutor (OSP)
while the second, which involved
the same set of facts, was filed
with the Presidential Commission
on Good Government (PCGG) but
which was later endorsed to the
Office of the Ombudsman.
A preliminary investigation was
conducted by the PCGG an
Information was filed charging
petitioner with violation of Section
3(e) of Republic Act No. 3019.
During the proceedings before the
OSP, petitioner moved to dismiss
the complaint. The OSP, denied the
motion and filed an Information
charging petitioner with Estafa
through Falsification of Public
Documents.
As a result of the filing of two
informations
with
respondent
Sandiganbayan involving the same
accused (herein petitioner) and the
same set of facts, the cases involed
was
consolidated
which
was
pending before the First Division
of respondent Sandiganbayan.
The Information in the case
involving RA 3019 was ordered
dismissed in compliance with the
ruling in Cojuangco, Jr. vs. PCGG,
et al., which declared null and void

the
preliminary
investigations
conducted by the PCGG in all
criminal cases involving matters
which were the subject matter of
civil cases earlier filed, however
the Information in involving estafa
was
correctly
assailed
by
petitioner as having been filed
without the proper preliminary
investigation.
During
the
preliminary
investigation conducted anew by
the Office of the Ombudsman,
petitioner submitted his counteraffidavit
and
supporting
documents. After the completion of
said investigation, the prosecutor
prepared a Resolution which
recommended the withdrawal of
the Information in the case
involving RA3019.
Respondent Ombudsman, however,
despite the above recommendation
of the investigating prosecutor
ordered
the
prosecution
to
proceed
under
the
existing
Information in the said case.
Petitioner
thus
filed
with
respondent Sandiganbayan (First
Division) an Omnibus Motion to
Quash the Information, wherein he
prayed ". . . for the production of
(the) record of the preliminary
investigation),
and
that
the
information be quashed outright or
the disapproval of the Ombudsman
set aside.
Respondent
Sandiganbayan
promulgated a Resolution denying
the Omnibus Motion of the
petitioner.
ISSUE:

WON Sandiganbayan acted with


grave abuse of discretion in not
dismissing
the
information
considering that the Ombudsmans
disapproval of the order dismissing
the complaint did not state the
factual or legal basis therefor.
RULING:
NO. Petitioner avers that it is the
duty of the Ombudsman to assess
the evidence and defenses of the
respondent in deciding a case, a
failure wherein constitutes a
violation of ones right to due
process of law. He further claims
that "while the duty to deliberate
does not impose the obligation to
decide right, it does imply a
necessity
which
cannot
be
disregarded, namely, that of having
something to support the decision.
The Ombudsman in this case not
only failed to decide right but has
nothing at all to support his
decision."
Respondents, on the other hand,
aver that the Office of the
Ombudsman is not exercising
quasi-judicial or quasi-legislative
powers because "it does not act as
a
court"
when
it
conducts
preliminary investigation of cases
falling under its jurisdiction.
It is settled that the conduct of a
preliminary investigation, which is
defined
as
"an
inquiry
or
proceeding for the purpose of
determining whether there is
sufficient ground to engender a
well-founded belief that a crime
cognizable by the Regional Trial
Court has been committed and
that the respondent is probably
guilty thereof, and should be held

for
trial,"
is,
like
court
proceedings,
subject
to
the
requirements of both substantive
and procedural due process. This
is
because,
a
preliminary
investigation is considered as a
judicial proceeding wherein the
prosecutor or investigating officer,
by the nature of his functions, acts
as a quasi-judicial officer.
As correctly pointed out by
petitioner,
an
indispensable
requisite of due process is that the
person who presides and decides
over a proceeding, including a
preliminary investigation, must
possess the cold neutrality of an
impartial judge.
Although
such
a
preliminary
investigation is not a trial and is
not intended to usurp the function
of the trial court, it is not a casual
affair. The officer conducting the
same investigates or inquires into
the
facts
concerning
the
commission of the crime with the
end in view of determining
whether or not an information may
be prepared against the accused.
Indeed, a preliminary investigation
is in effect a realistic judicial
appraisal of the merits of the case.
Sufficient proof of the guilt of the
accused must be adduced so that
when the case is tried, the trial
court may not be bound as a
matter of law to order an acquittal.
A preliminary investigation has
then been called a judicial inquiry.
It is a judicial proceeding. An act
becomes judicial when there is
opportunity to be heard and for
the production and weighing of
evidence, and a decision is
rendered thereon.

The authority of a prosecutor or


investigating
officer
duly
empowered to preside or to
conduct
a
preliminary
investigation is no less than that of
a municipal judge or even a
regional trial court judge. While
the investigating officer, strictly
speaking is not a "judge," by the
nature of his functions he is and
must be considered to be a quasijudicial officer.
The Ombudsmans findings are
essentially
factual
in nature.
Accordingly, in assailing said
findings on the contention that the
Ombudsman committed a grave
abuse of discretion in holding that
petitioner is liable for estafa
through falsification of public
documents, petitioner is clearly
raising questions of fact here. His
arguments are anchored on the
propriety of or error in the
Ombudsmans
appreciation
of
facts.
Petitioner
cannot
be
unaware that the Supreme Court is
not a trier of facts, more so in the
consideration of the extraordinary
writ of certiorari where neither
questions of fact nor even of law
are entertained, but only questions
of lack or excess of jurisdiction or
grave abuse of discretion. Insofar
as this third issue is concerned,
therefore, we find that no grave
abuse of discretion has been
committed by respondents which
would warrant the granting of the
writ of certiorari.

A preliminary investigation has


then been called a judicial
inquiry.
It
is
a
judicial

proceeding. An act becomes


judicial
when
there
is
opportunity to be heard and for
the production and weighing of
evidence, and a decision is
rendered thereon. (Cruz vs
People, 233 SCRA 439)

Bautista vs CA, G.R. No.


143375, July 6 2001
FACTS:
Petitioner Ruth D. Bautista issued
to private respondent Susan Aloa a
check drawn on Metrobank Cavite
City Branch, private respondent
presented the check for payment,
however, drawee bank dishonored
the check because it was drawn
against insufficient funds (DAIF).
Private
respondent
filed
a
complaint-affidavit with the City
Prosecutor
of
Cavite
City.
Petitioner then submitted her own
counter-affidavit. The investigating
prosecutor issued a resolution
recommending the filing of an
Information against petitioner for
violation of BP 22, which was
approved by the City Prosecutor.
Petitioner filed with the Office of
the Regional State Prosecutor
(ORSP) for a petition for review of
the resolution. The ORSP denied
the petition. The petitioner filed a
motion for reconsideration, which
the ORSP also denied. According
to the ORSP, only resolutions of
prosecutors dismissing a criminal
complaint were cognizable for
review by that office.
Petitioner filed with the Court of
Appeals a petition for review of the
resolution of the ORSP and the
order denying the reconsideration,
which
was
also
dismissed.
Petitioner
submits
that
a
prosecutor
conducting
a
preliminary investigation performs
a quasi-judicial function
ISSUE:

WON the decisions of the Office of


the Prosecutor in approving the
filing of a criminal complaint are
appealable to the Court of Appeals
under Rule 43.
RULING:
NO.
In these cases the Court held that
the power to conduct preliminary
investigation is quasi-judicial in
nature. But this statement holds
true only in the sense that, like
quasi-judicial
bodies,
the
prosecutor is an office in the
executive department exercising
powers akin to those of a court.
Here is where the similarity ends.
A closer scrutiny will show that
preliminary investigation is very
different from other quasi-judicial
proceedings. A quasi-judicial body
has been defined as "an organ of
government other than a court and
other than a legislature which
affects the rights of private parties
through either adjudication or
rule-making."
On the other hand, the prosecutor
in a preliminary investigation does
not
determine
the
guilt
or
innocence of the accused. He does
not exercise adjudication nor rulemaking
functions.
Preliminary
investigation
is
merely
inquisitorial, and is often the only
means of discovering the persons
who may be reasonably charged
with a crime and to enable the
fiscal to prepare his complaint or
information. It is not a trial of the
case on the merits and has no
purpose except that of determining

whether
a
crime
has
been
committed and whether there is
probable cause to believe that the
accused is guilty thereof. While the
fiscal makes that determination, he
cannot be said to be acting as a
quasi-court, for it is the courts,
ultimately, that pass judgment on
the accused, not the fiscal.
Hence, the Office of the Prosecutor
is not a quasi-judicial body;
necessarily, its decisions approving
the filing of a criminal complaint
are not appealable to the Court of
Appeals under Rule 43. The courts
cannot
interfere
with
the
discretion
of
the
fiscal
to
determine the specificity and
adequacy of the offense charged.
He may dismiss the complaint
forthwith if he finds it to be
insufficient in form or substance or
if he finds no ground to continue
with the inquiry; or, he may
otherwise
proceed
with
the
investigation if the complaint is, in
his view, in due and proper form.
From the allegations of the
complaint,
it
is
clear
that
petitioner is being prosecuted for
violation of the first paragraph of
the offense. First, making or
drawing and issuing any check to
apply on account or for value,
knowing at the time of issue that
the drawer does not have sufficient
funds in or credit with the drawee
bank.

The
power
to
conduct
preliminary
investigation
is
quasi-judicial in nature. But
this statement holds true only

in the sense that, like quasijudicial bodies, the prosecutor


is an office in the executive
department exercising powers
akin to those of a court. Here is
where the similarity ends. A
closer scrutiny will show that
preliminary
investigation
is
very different from other quasijudicial proceedings. (Bautista
vs CA, G.R. No. 143375, July 6
2001)

Santos vs Go, G.R. No. 156081,


October 19, 2005
Petitioners are corporate directors
and
officers
of
Fil-Estate
Properties
Inc.
(FEPI).
FEPI
allegedly entered into a project
agreement with Manila Southcoast
Development
Corp.
(MSDC),
whereby
FEPI
undertook
to
develop several parcels of land
allegedly owned by MSDC. Under
the terms, was to convert it into
first class residential, commercial,
resort, leisure and recreational
complex.
The
said
project
agreement clothed FEPI with
authority to market and sell the
subdivision lots to the public.
Respondent offered to buy a lot
with a purchase price agreed upon
by the parties. The Contract to sell
signed by them was standard,
printed form prepared by FEPI.
Under the terms of said contract of
adhesion, Go agreed to pay a
downpayment,
and
a
last
installment. In turn, FEPI would
execute a final Deed of Sale upon
completion of payment.
Go fully complied, however, FEPI
failed to develop the property.
Neither did it release the TCT to
Go. The latter demanded the
fulfillment of the terms and
condition but FEPI balked.
Go was neither satisfied nor
assured by FEPIs statements and
he made several demands upon
FEPI to return the payment but
FEPI failed to comply. Go then filed
a complaint before the HLURB and
likewise filed a separate complaint
before the Office of Prosecutor for

estafa averred that that the


contract to sell categorically stated
that FEPI was the owner of the
property. However, before the
HLURB,
FEPI
denied
the
ownership. Go also alleged that
they committed estafa when they
offered the property for sale since
they knew fully well that the
development and issuance of title
were impossible to accomplish.
Petitioners
challenged
the
jurisdiction of the prosecutor to
conduct preliminary investigation
on the ground that complaint was
not in Pasig and contract was not
executed in Pasig. Besides, the
elements of estafa were not
present.
In his reply, Go stressed that
prosecutor had jurisdiction.
ISSUE:
Whether a petition for review
under Rule 43 is a proper mode of
appeal from a resolution of the
Secretary of Justice directing the
prosecutor to file information in a
criminal case
HELD:
Public
prosecutor
does
not
perform acts of quasi-judicial body.
The Court described a quasijudicial body as an organ of
government other than a court and
other than a legislature which
performs adjudicatory functions.
Said body affects the rights of
private parties either though
adjudication or rule-making. Its
awards,
when
performing
adjudicatory functions, determine
the rights of the parties and their

decisions have the same effect as


judgments of a court. Such is not
the case when a public prosecutor
conducts
a
preliminary
investigation
to
determine
probable
cause
to
file
an
information against a person
charged with a criminal offense, or
when the Secretary of Justice is
reviewing the formers order or
resolutions. Since the DOJ is not a
quasi-judicial body and it is not
one of those agencies whose
decisions, orders or resolutions are
appealable to the Court of Appeals
under Rule 43, the resolution of
the Secretary of Justice finding
probable
cause
to
indict
petitioners for estafa is, therefore,
not appealable to the Court of
Appeals via a petition for review
under Rule 43.

The public prosecutors power


to
conduct
a
preliminary
investigation as quasi-judicial
in nature, this is true only to
the extent that, like quasijudicial bodies, the prosecutor
is an officer of the executive
department exercising powers
akin to those of a court, and
the similarity ends at this
point.
Such is not the case when a
public prosecutor conducts a
preliminary
investigation
to
determine probable cause to
file an information against a
person charged with a criminal
offense, or when the Secretary
of Justice is reviewing the
formers order or resolutions.
Since the DOJ is not a quasi-

judicial body and it is not one


of
those
agencies
whose
decisions, orders or resolutions
are appealable to the Court of
Appeals under Rule 43, the
resolution of the Secretary of
Justice finding probable cause
to indict petitioners for estafa
is, therefore, not appealable to
the Court of Appeals via a
petition for review under Rule
43. (Santos vs Go, G.R. No.
156081, October 19, 2005)

Metropolitan Bank and Trust


Company vs Tonda, 338 SCRA
254
FACTS:
On January 31, 1997, petitioner
Metropolitan Bank and Trust
Company charged respondents
before the Office of the City
Prosecutor of Manila with the
crime of estafa under Article 315,
paragraph 1(b) of the Revised
Penal Code. In the affidavit of
petitioners audit officer, Antonio
Ivan S. Aguirre, it was alleged that
the special audit conducted on the
cash and lending operations of its
Port
Area
branch
uncovered
anomalous/fraudulent transactions
perpetrated by respondents in
connivance with client Universal
Converter
Philippines,
Inc.
(Universal); that respondents were
the only voting members of the
branchs
credit
committee
authorized
to
extend
credit
accommodation to clients up to
P200,000.00; that through the socalled Bills Purchase Transaction,
Universal, which has a paid-up
capital of only P125,000.00 and
actual maintaining balance of
P5,000.00, was able to make
withdrawals
totaling
P81,652,000.00 against uncleared
regional checks deposited in its
account at petitioners Port Area
branch;
that,
consequently,
Universal was able to utilize
petitioners funds even before the
seven-day clearing period for
regional checks expired; that
Universals withdrawals against
uncleared regional check deposits
were without prior approval of

petitioners head office; that the


uncleared
checks
were
later
dishonored by the drawee bank for
the reason Account Closed; and,
that respondents acted with fraud,
deceit, and abuse of confidence.
In their defense, respondents
denied
responsibility
in
the
anomalous
transactions
with
Universal and claimed that they
only intended to help the Port Area
branch solicit and increase its
deposit
accounts
and
daily
transactions.
Meanwhile, on February 26, 1997,
petitioner and Universal entered
into a Debt Settlement Agreement
whereby the latter acknowledged
its indebtedness to the former in
the
total
amount
of
P50,990,976.27[8] as of February
4, 1997 and undertook to pay the
same in bi-monthly amortizations
in the sum of P300,000.00 starting
January 15, 1997, covered by
postdated checks, plus balloon
payment of the remaining principal
balance and interest and other
charges, if any, on December 31,
2001.

ISSUE:
Whether novation and undertaking
to pay the amount embezzled do
not extinguish criminal liability

HELD:

Initially, it is best to emphasize


that novation is not one of the
grounds prescribed by the Revised
Penal Code for the extinguishment
of criminal liability.

Thus, the doctrine is that a


compromise or settlement entered
into after the commission of the
crime
does
not
extinguish
accuseds
liability
for
estafa.
Neither will the same bar the
prosecution
of
said
crime.
Accordingly, in such a situation, as
in this case, the complaint for
estafa against respondents should
not be dismissed just because
petitioner entered into a Debt
Settlement
Agreement
with
Universal.

Preliminary investigation is an
executive,
not
a
judicial
function.[23]
Such
investigation is not part of the
trial,
hence,
a
full
and
exhaustive presentation of the
parties'
evidence
is
not
required, but only such as may
engender
a
well-grounded
belief that an offense has been
committed
and
that
the
accused is probably guilty
thereof. (Metropolitan Bank
and Trust Company vs Tonda,
338 SCRA 254)

Right to a Preliminary
Investigation; waivable
People vs Gomez, 117 SCRA 73
In 1962, four informations were
filed by the prosecuting Fiscal
before the Court of First Instance,
charging appellees with estafa
thru
falsification
of
public
documents.
Three
of
said
informations
contained
certifications of the prosecuting
officers that they had conducted
preliminary investigation and that
they believed that the accused
were guilty of the offense charged.
One information did not contain
such a certification, instead the
district judge himself made a
preliminary investigation and once
satisfied that a prima facie case
existed against the accused, issued
warrants of arrest. All of the
accused filed bonds for provisional
liberty. Thereafter, on June 22,
1966, the accused in the four cases
filed
a
motion
to
declare
informations and warrants of
arrest null and void on the ground
that the prosecution failed to
observe the provisions of the New
Rules
of
Court
regarding
preliminary investigations. Said
motion was denied but upon
motion for reconsideration, the
order was reversed and the cases
were dismissed without prejudice
to the refilling of the same in
accordance with the Rules. The
Prosecution appealed from said
order of dismissal.
ISSUE:
Whether they have waived their
right to preliminary investigation

for failure to invoke said right


before entering plea
HELD:
YES. The SC set aside the order of
dismissal and held that since the
preliminary investigations in said
cases were terminated in 1962, the
New Rules of Court is not
applicable having taken in 1964;
and that even if no preliminary
investigation was held, upon the
accuseds entering a plea and
failing to invoke their right to
preliminary
investigation
upon
arraignment, they were deemed to
have waived said right. Hence,
they waived the right to a
preliminary investigation when
they failed to invoke it prior to, or
at least at, the time of the entry of
their plea in the Court of First
Instance.

They waived the right to a


preliminary investigation when
they failed to invoke it prior to,
or at least at, the time of the
entry of their plea in the Court
of First Instance. (People vs
Gomez, 117 SCRA 73)

People vs Bulosan, 160 SCRA


492
FACTS:
On May 27, 1978, the said accused
armed with deadly sharp pointed
weapon, and by means of force and
intimidation, did then and there,
wilfully, unlawfully and feloniously
lie lie with and succeed in having
carnal knowledge of offended
party, an unmarried 21 year old
woman, against the will and
without the consent of the latter,
as said accused pointed the deadly
weapon on the neck of the
offended party and threatened her
and her family if she refused.
The lower court established the
appellants
guilt
beyond
reasonable doubt.
ISSUE:
Whether the appellant was denied
due process for failure of the judge
to conduct second stage of
preliminary investigation
HELD:
NO. The records show that the
appellant was afforded preliminary
investigation in the MTC. Clearly,
therefore, appellant having been
afforded preliminary investigation
in the MTC, to insist another one
would be to ask for what is fait
accompli. When nothing appears
affirmatively upon the record that
such preliminary investigation has
not in fact conducted, an objection
on the ground of denial thereof
will not be sustained, for it will be
presumed
that
the
court
proceeded in accordance with law.

Moreover, the alleged denial of the


right to the second stage of
preliminary
investigation
was
never raised before the trial court.
Hence,
while
generally,
a
preliminary
investigation
is
mandatory and a certification that
such investigation was held, is
required, still this rule does not
apply if the issue is raised only
after conviction. Thus, it has been
held that after plea of not guilty to
the information, an accused is
deemed to have foregone the right
of preliminary investigation and to
have abandoned the right to
question any irregularity that
surrounds it.

It has been held that after a


plea of not guilty to the
information, an accused is
deemed to have foregone the
right
of
preliminary
investigation
and
to
have
abandoned
the
right
to
question any irregularity that
surrounds
it.
(People
vs
Bulosan, 160 SCRA 492)

Go vs CA, 206 SCRA 138


FACTS
An information was filed charging
herein petitioner Rolito Go for
murder before the Regional Trial
Court of Metro Manila. Eldon
Maguan was driving his car along
Wilson St., San Juan, Metro
Manila,
heading
towards
P.
Guevarra St. Petitioner entered
Wilson St., where it is a one-way
street and started travelling in the
opposite or "wrong" direction. At
the corner of Wilson and J. Abad
Santos
Sts.,
petitioner's
and
Maguan's cars nearly bumped
each other. Petitioner alighted
from his car, walked over and shot
Maguan inside his car. Petitioner
then boarded his car and left the
scene. A security guard at a
nearby restaurant was able to take
down
petitioner's
car
plate
number. The police arrived shortly
thereafter at the scene of the
shooting and there retrieved an
empty shell and one round of live
ammunition for a 9 mm calibre
pistol. Verification at the Land
Transportation Office showed that
the car was registered to one Elsa
Ang Go. Petitioner voluntarily
presented himself together with
his two lawyers to the police upon
obtaining knowledge of being
hunted by the latter. However, he
was immediately detained and
denied his right of a preliminary
investigation unless he executes
and sings a waiver of the
provisions of Article 125 of the
Revised Penal Code. Upon omnibus
motion for immediate release on
recognizance or on bail and proper

preliminary investigation on the


ground that his warrantless arrest
was unlawful and no preliminary
investigation
was
conducted
before the information was filed,
which is violated his rights, the
same was granted but later on
reversed by the lower court and
affirmed by the Court of Appeals.
The appellate court in sustaining
the decision of the lower court
held that petitioner's warrantless
arrest was valid in view of the fact
that the offense was committed,
the
petitioner
was
clearly
identified and there exists valid
information
for
murder
filed
against petitioner.
Issue:
WON petitioner waived his right
preliminary investigation
Held:
No. The rule is that the right to
preliminary investigation is waived
when the accused fails to invoke it
before or at the time of entering a
plea at arraignment. In the instant
case, petitioner Go had vigorously
insisted on his right to preliminary
investigation
before
his
arraignment. At the time of his
arraignment,
petitioner
was
already before the Court of
Appeals on certiorari, prohibition
and mandamus precisely asking
for a preliminary investigation
before being forced to stand trial.
The facts of the case show that
petitioner insisted on his right to
preliminary investigation before
his arraignment and he, through
his counsel denied answering
questions before the court unless

they were afforded the proper


preliminary investigation. For the
above reasons, the petition was
granted and the ruling of the
appellate court was set aside and
nullified. The Supreme Court
however, contrary to petitioner's
allegation, declared that failure to
accord the right to preliminary
investigation did not impair the
validity
of
the
information
charging the latter of the crime of
murder.

His filing of a petition to be


released on bail was a waiver of
any irregularity attending his
arrest and estops him from
questioning its validity. (Go vs
CA, 206 SCRA 138)

Preliminary Investigation vs.


Preliminary Examination
Peopls vs Inting, 187 SCRA 788
Facts:
Mrs. Editha Barba filed a lettercomplaint
against
OIC-Mayor
Dominador Regalado of Tanjay,
Negros
Oriental
with
the
COMELEC
for
allegedly
transferring her, a permanent
Nursing Attendant, Grade I, in the
office of the Municipal Mayor to a
very remote barangay and without
obtaining prior permission or
clearance from COMELEC as
required by law.
After a preliminary investigation of
Barbas complaint, Atty. Lituanas
found a prima facie case. Hence,
on September 26, 1988, he filed
with the respondent trial court a
criminal case for violation of
section 261, Par. (h), Omnibus
Election Code against the OICMayor.
In
an
Order
dated
September
30,
1988,
the
respondent court issued a warrant
of arrest against the accused OIC
Mayor.
However, in an order dated
October 3, 1988 and before the
accused could be arrested, the
trial court set aside its September
30, 1988 order on the ground that
Atty. Lituanas is not authorized to
determine
probable
cause
pursuant to Section 2, Article III of
the 1987 Constitution. The trial
court later on quashed the
information. Hence, this petition.
Issue:

Whiter or not a preliminary


investigation conducted by a
Provincial
Election
Supervisor
involving election offenses have to
be coursed through the Provincial
Prosecutor, before the Regional
Trial Court may take cognizance of
the investigation and determine
whether or not probable cause
exists?
Held:
The 1987 Constitution empowers
the
COMELEC
to
conduct
preliminary investigations in cases
involving election offenses for the
purpose of helping the Judge
determine probable cause and for
filing an information in court. This
power is exclusive with COMELEC.
The
evident
constitutional
intendment in bestowing this
power to the COMELEC is to
insure the free, orderly and honest
conduct of elections, failure of
which
would
result
in
the
frustration of the true will of the
people and make a mere idle
ceremony of the sacred right and
duty of every qualified citizen to
vote. To divest the COMELEC of
the authority to investigate and
prosecute offenses committed by
public officials in relation to their
office would thus seriously impair
its effectiveness in achieving this
clear
constitutional
mandate.
Bearing these principles in mind, it
is apparent that the respondent
trial
court
misconstrued
the
constitutional provision when it
quashed the information filed by
the Provincial Election Supervisor.

A preliminary investigation is
conducted by the prosecutor to
ascertain whether the alleged
offender should be held for
trial, to be subjected to the
expense,
rigors
and
embarrassment of trial or if the
offender is to be released.
(Peopls vs Inting, 187 SCRA
788)

AAA vs Carbonel, 524 SCRA 496


FACTS:
In a rape case, private complainant
failed to appear 4 consecutive
orders to take the witness stand in
order to satisfy the judge for the
existence of probable cause for the
issuance of a warrant of arrest.
Judge
Carbonell
dismissed
Criminal Case No. 6983 for lack of
probable cause on the ground that
the complainant and her witnesses
failed to take the witness stand.
He claims that under Section 2,
Article III of the 1987 Constitution,
no warrant of arrest shall issue
except upon probable cause to be
determined personally by the
judge after examination under
oath
or
affirmation
of
the
complainant and the witnesses he
may produce.
ISSUE:
Is Judge Carbonell correct?
HELD:
No. Judge Carbonell committed
grave abuse of discretion. The
Supreme Court explained that this
constitutional provision does not
mandatorily require the judge to
personally
examine
the
complainant and her witnesses.
Instead, he may opt to personally
evaluate the report and supporting
documents
submitted
by the
prosecutor or he may disregard
the
prosecutors
report
and
require
the
submission
of
supporting affidavits of witnesses.
We reiterated the above ruling in
the case of Webb v. De Leon,

where we held that before issuing


warrants of arrest, judges merely
determine the probability, not the
certainty, of guilt of an accused. In
doing so, judges do not conduct a
de novo hearing to determine the
existence of probable cause. They
just personally review the initial
determination of the prosecutor
finding a probable cause to see if it
is
supported
by
substantial
evidence.
It is well to remember that there is
a
distinction
between
the
preliminary
inquiry
which
determines probable cause for the
issuance of a warrant of arrest and
the
preliminary
investigation
proper which ascertains whether
the offender should be held for
trial
or
be
released.
The
determination of probable cause
for purposes of issuing the warrant
of arrest is made by the judge. The
preliminary investigation proper
whether or not there is reasonable
ground to believe that the accused
is guilty of the offense charged is
the function of the investigating
prosecutor.
True, there are cases where the
circumstances may call for the
judges personal examination of
the complainant and his witnesses.
But it must be emphasized that
such personal examination is not
mandatory and indispensable in
the determination of probable
cause for the issuance of a warrant
of arrest. The necessity arises only
when there is an utter failure of
the evidence to show the existence
of probable cause. Otherwise, the
judge may rely on the report of the

investigating prosecutor, provided


that he likewise evaluates the
documentary evidence in support
thereof.
Indeed, what the law requires as
personal determination on the part
of the judge is that he should not
rely solely on the report of the
investigating prosecutor. In Okabe
v. Gutierrez, we stressed that the
judge should consider not only the
report
of
the
investigating
prosecutor but also the affidavit
and the documentary evidence of
the parties, the counter-affidavit of
the accused and his witnesses, as
well
as
the
transcript
of
stenographic notes taken during
the preliminary investigation, if
any, submitted to the court by the
investigating prosecutor upon the
filing of the Information. If the
report, taken together with the
supporting evidence, is sufficient
to sustain a finding of probable
cause, it is not compulsory that a
personal
examination
of
the
complainant and his witnesses be
conducted.

A preliminary inquiry pr a
preliminary
examination
is
conducted by the judge to
determine probable cause for
the issuance of a warrant of
arrest.
This
is
a
judicial
function. (AAA vs Carbonel, 524
SCRA 496)

Co vs Republic, 539 SCRA 147


While Miguel was at the Caramoan
Municipal Hospital, Sgt. Reyes,
SPO2 Ramil Araas (SPO2 Araas),
and a John Doe allegedly entered
the
Francias
house
against
Jocelyns will. They searched for
papers and other effects, and
caused chaos and disarray in the
house. Accordingly, Jocelyn also
filed a complaint for Violation of
Domicile against Sgt. Reyes, SPO2
Araas, and John Doe.
The accused alleged that Miguel
was drunk and unruly, and
indiscriminately fired his 9 mm
pistol. Sgt. Reyes, who was at the
vicinity, accosted Miguel and fired
a warning shot. However, Miguel
pointed his pistol at Sgt. Reyes.
Sgt. Reyes was forced to shoot
Miguel who was hit at his right
thigh.
Sgt.
Reyes
and
his
companions brought Miguel to the
Caramoan
Municipal
Hospital
where he died due to loss of blood.
The accused filed a motion to
suspend proceedings pending the
motion for reconsideration of the
DOJs 25 June 2002 Resolution.
Jocelyn,
through
a
private
prosecutor, filed a motion for
issuance of warrants of arrest
against
the
accused.
Jocelyn
thereafter filed a motion for
inhibition of Judge Alfredo Cabral
(Judge Cabral) for loss of trust and
confidence due to the delay in the
issuance of the warrants of arrest.
Judge Cabral granted Jocelyns
motion and the cases were
transferred to the sala of Judge
Eufronio K. Maristela of the
Regional Trial Court of San Jose,

Camarines Sur, Branch 30 (trial


court).
In an Order dated 13 September
2002, the trial court denied the
motion to suspend proceedings as
well as the motion for the issuance
of warrants of arrest. Instead, the
trial court set the cases for
preliminary
investigation
of
Jocelyn and her witnesses to
determine probable cause.
During the scheduled preliminary
investigation on 23 September
2002, only Dr. Minerva BalmaceaAguirre of Caramoan Municipal
Hospital appeared. Jocelyn and her
witnesses did not appear for fear
for their lives. The private
prosecutor
moved
for
the
suspension
of
the
judicial
determination of probable cause as
he was filing a petition for change
of venue. In an Order dated 14
October 2002, the trial court held
in abeyance the presentation of
additional evidence for judicial
determination of probable cause.
However, this Court subsequently
denied the private prosecutors
motion for change of venue.
In an Order dated 19 May 2003,
the trial court held that no
probable cause exists against the
accused for the crime of Murder.
The trial court dismissed the
Information for Murder but upheld
the Information for Homicide
against Sgt. Reyes. The trial court
issued another warrant of arrest
against Sgt. Reyes.
The private prosecutor moved for
reconsideration of the 19 May
2003 Order. On 13 August 2003,

the trial court granted the motion


and set the presentation of
additional evidence for judicial
determination of probable cause
on 29 August 2003. Meanwhile, on
12 August 2003, Jocelyn executed
an
Affidavit
of
Retraction/Desistance
absolving
the
accused,
particularly
petitioners,
from
liability
for
Miguels death. On 28 August 2003,
the private prosecutor filed an
Omnibus Motion to Admit Affidavit
of
Desistance
of
Private
Complainant Jocelyn Francia and
to Dismiss the Case. On 29 August
2003, neither the government nor
the private prosecutor appeared at
the scheduled hearing. None of the
witnesses stated in the subpoena
appeared.

despite Jocelyns submission of an


Affidavit withdrawing her Affidavit
of Desistance and despite her lack
of affirmation in open court of the
Affidavit of Desistance. The heirs
of Miguel further alleged that the
trial court committed grave abuse
of discretion in dismissing the
Information for Murder for lack of
probable cause to sustain the
charges against the accused.
ISSUE:
Whether the Court of Appeals
committed a reversible error in
reversing the trial courts 21
October
2003
Order
which
dismissed the Information for
Murder against petitioners, SPO2
Araas, Sgt. Reyes, and John Does
for lack of probable cause.
HELD:

In an Order dated 21 October


2003, the trial court dismissed the
Information for Murder against
SPO2 Araas, Sgt. Reyes, John
Does, and petitioners for lack of
probable cause. The trial court
again sustained the Information
for Homicide against Sgt. Reyes.
The private prosecutor filed a
motion for reconsideration. In its
Order dated 17 December 2003,
the trial court denied the motion.
Jocelyn, joined by Miguels sons
Antonio Francia II and Mark
Anthony Francia (collectively, the
heirs of Miguel), filed a petition for
certiorari before the Court of
Appeals assailing the 21 October
2003 Order of the trial court. The
heirs of Miguel alleged that the 21
October 2003 Order was issued

The petition has no merit.


The Court of Appeals distinguished
between
a
preliminary
investigation for the determination
of sufficient ground for the filing of
information on one hand, and
preliminary examination for the
determination of probable cause
for the issuance of a warrant of
arrest on the other. The Court of
Appeals ruled that in this case, the
incident before the trial court was
the determination of probable
cause for the issuance of warrants
of arrest against the accused. The
trial court was not tasked to
determine whether there was a
probable cause to prosecute the
accused for the crime of Murder.
The determination of probable
cause to prosecute the accused for
Murder falls within the jurisdiction

of the prosecutor. This was already


settled with the issuance of the 25
June 2002 Resolution by the
Secretary of Justice and with the
filing of an Information for Murder
before the trial court. The Court of
Appeals ruled that the trial court
went beyond its jurisdiction when
it assumed the duty and function
of the prosecutor.
In this case, what was brought
before the trial court was the
preliminary examination. The trial
courts jurisdiction is limited to the
determination of whether there is
probable cause for the issuance of
warrants of arrest against the
accused. Instead, the trial court
assumed the function of the
prosecutor
by
determining
whether there was probable cause
for the filing of the information for
Murder.

Preliminary
investigation
should be distinguished as to
whether it is an investigation
for the determination of a
sufficient ground for the filing
of the information or it is an
investigation
for
the
determination of a probable
cause for the issuance of a
warrant of arrest. The first kind
of preliminary investigation is
executive in nature. It is part of
the prosecution's job. The
second kind of preliminary
investigation which is more
properly
called
preliminary
examination
is
judicial
in
nature and is lodged with the
judge. (Co vs Republic, 539
SCRA 147)

Probable Cause in Preliminary


Investigation

Thus, petitioner refused to sign the


said minutes.

Roberto Kalalo vs Office of the


Ombudsman, G.R. No. 158189,
April 23, 2010
In March 2001, petitioner, who
was then the Board Secretary,
claimed that he found in his table,
a final print of the Minutes of the
above-mentioned General Meeting
which
was
forwarded
by
respondent Marcelo Agustin upon
the order of respondent De
Chavez, in order for the petitioner
to certify as to its correctness. The
fact that the said copy of the
Minutes was given to him after a
long period of time and other
inconsistencies found in the same
document, caused suspicion on the
part of the petitioner.

ISSUE:

As claimed by petitioner, the


authentic minutes had eight (8)
pages, while the falsified one had
nine (9) pages. Thus, he concluded
that Resolution Nos. 25 and 26
were mere intercalations on the
minutes of the annual meeting.
Petitioner
also
claimed
that
respondent's deviation from the
usual procedure in signing and
approving the minutes was highly
suspicious. According to petitioner,
the usual procedure was for
respondent De Chavez, in his
capacity as Vice-Chairman, to sign
the minutes only after the same
has been attested by petitioner as
the Board Secretary. However, De
Chavez submitted a copy of the
minutes to petitioner with his
signature already affixed thereon.

Whether public respondent gravely


abused its discretion amounting to
lak and/or excess of jurisdiction in
not finding probable cause
HELD:
As a general rule, courts do not
interfere with the discretion of the
Ombudsman to determine whether
there exists reasonable ground to
believe that a crime has been
committed and that the accused is
probably
guilty
thereof
and,
thereafter,
to
file
the
corresponding information with
the appropriate courts.
This Court has consistently held
that
the
Ombudsman
has
discretion to determine whether a
criminal case, given its facts and
circumstances, should be filed or
not. It is basically his call. He may
dismiss the complaint forthwith
should he find it to be insufficient
in form and substance, or should
he find it otherwise, to continue
with the inquiry; or he may
proceed with the investigation if,
in his view, the complaint is in due
and proper form and substance
In the present case, the Office of
the Ombudsman did not find
probable cause that would warrant
the filing of Information against
respondents.

Probable cause, for purposes of


filing a criminal information,
has been defined as such facts
as are sufficient to engender a
well-founded belief that a crime
has been committed and that
respondents are probably guilty
thereof. The determination of
its existence lies within the
discretion of the prosecuting
officers after conducting a
preliminary investigation upon
complaint of an offended party.
Probable cause is meant such
set of facts and circumstances
which would lead a reasonably
discreet and prudent man to
believe
that
the
offense
charged in the Information, or
any offense included therein,
has been committed by the
person sought to be arrested.
(Roberto Kalalo vs Office of the
Ombudsman, G.R. No. 158189,
April 23, 2010)

Ricaforte vs Jurado, G.R. No.


154438, September 5, 2007
The Facts:
On February 10, 1997, respondent
filed a Complaint3 for estafa and
violation of Batas Pambansa (B.P.)
Blg. 22 against Alicia F. Ricaforte
(petitioner) with the Quezon City
Prosecutors Office. He alleged
that he operates and manages a
rice mill in Bulacan; that sometime
in June 1996, Ruby Aguilar
(Aguilar) procured rice from him
and in payment thereof gave him
two Far East Bank and Trust
Company (FEBTC) checks, to wit:
FEBTC Check No. 08A096028P
dated July 25, 1996 and Check No.
08A096029P dated August 25,
1996,
in
the
amount
of
P431,555.00 each, which were
both issued by petitioner and when
presented
for
payment
were
dishonored.
In her Counter-Affidavit, petitioner
denied the accusation. She alleged
that Aguilar who had lost her
Metrobank checkbook borrowed
her checks to pay off Aguilars
obligations with Leon Jurado
(respondent); that she willingly
lent her checks to Aguilar on
condition that these checks will be
replaced
with
Aguilars
own
checks once her new checkbook is
issued to her by Metrobank; that
Aguilar then used petitioners
checks
to
pay
her
rice
procurement with respondent; that
in
accordance
with
the
arrangement, Aguilar issued two
replacement checks in favor of
respondent in the amount of
P431,555.00 each; that when

Aguilar issued the replacement


checks, petitioner demanded from
respondent the return of her
checks but respondent refused,
thus she was constrained to
request her bank to issue an order
of stop payment. Aguilar executed
an
Affidavit
corroborating
petitioners defense.
Petitioners
Motion
for
Reconsideration was denied in a
Resolution dated July 29, 2002.
The CA ruled that mere issuance of
a bouncing check constitutes a
probable cause for violation of B.P.
Blg. 22; that whether or not the
accused is guilty thereof is
determined in the trial proper; that
preliminary investigation is not a
trial and is not intended to usurp
the function of the trial court; that
Sales, which is invoked by
petitioner, is not applicable to the
instant case
ISSUE:
THE HONORABLE COURT OF
APPEALS COMMITTED A GRAVE
REVERSIBLE ERROR IN NOT
HOLDING
THAT
THE
HONORABLE
SECRETARY
OF
JUSTICE COMMITTED A GRAVE
ABUSE
OF
DISCRETION
IN
ISSUING
HIS
MODIFIED
RESOLUTION
FINDING
PROBABLE
CAUSE
AGAINST
PETITIONER FOR VIOLATION OF
B.P. BLG. 22, DESPITE THE FACT
THAT
THE
HONORABLE
SECRETARY HAS AGREED WITH
THE FINDING OF THE QUEZON
CITY
PROSECUTION
OFFICE
DISMISSING THE CHARGE OF
ESTAFA AGAINST PETITIONER.

HELD:
Herein case is still in the
preliminary investigation stage
which is merely inquisitorial, and it
is often the only means of
discovering the persons who may
be reasonably charged with a
crime, to enable the fiscal to
prepare
his
complaint
or
information.33 It is not a trial of
the case on the merits and has no
purpose except that of determining
whether
a
crime
has
been
committed and whether there is
probable cause to believe that the
accused is guilty.34 It is not the
occasion
for
the
full
and
exhaustive display of the parties
evidence; it is for the presentation
of such evidence only as may
engender a well-grounded belief
that
an
offense
has
been
committed and that the accused is
probably guilty thereof.35 We are
in
accord
with
the
Justice
Secretarys finding that there is
reasonable ground to believe that
a violation of B.P. Blg. 22 has been
committed by petitioner, thus, we
refrain
from
prejudging
the
applicablity or inapplicability of
Magno in this case.

The complainant need not


present at this stage proof
beyond reasonable doubt. A
preliminary investigation does
not
require
a
full
and
exhaustive presentation of the
parties evidence. It is enough
that in the absence of a clear
showing
of
arbitrariness,
credence is given to the finding
and determination of probable

cause by the Secretary of


Justice
in
a
preliminary
investigation.
(Ricaforte
vs
Jurado,
G.R.
No.
154438,
September 5, 2007)

Sy Thiong Siou vs Sy Chim, G.R.


No. 174168, March 30, 2009
On 30 May 2003, four criminal
complaints were filed by Sy Chim
and Felicidad Chan Sy (Spouses
Sy) against Sy Tiong Shiou, Juanita
Tan Sy, Jolie Ross Tan, Romer Tan,
Charlie Tan and Jessie James Tan
(Sy Tiong Shiou, et al.) before the
City Prosecutors Office of Manila.
The cases were later consolidated.
Two of the complaints, I.S. Nos.
03E-15285 and 03E-15286, were
for alleged violation of Section 74
in relation to Section 144 of the
Corporation
Code.
In
these
complaints,
the
Spouses
Sy
averred that they are stockholders
and directors of Sy Siy Ho & Sons,
Inc. (the corporation) who asked
Sy Tiong Shiou, et al., officers of
the corporation, to allow them to
inspect the books and records of
the business on three occasions to
no avail. In a letter dated 21 May
2003, Sy Tiong Shiou, et al. denied
the request, citing civil and intracorporate cases pending in court.
In the two other complaints, I.S.
No. 03E-15287 and 03E-15288, Sy
Tiong Shiou was charged with
falsification under Article 172, in
relation to Article 171 of the
Revised Penal Code (RPC), and
perjury under Article 183 of the
RPC. According to the Spouses Sy,
Sy Tiong Shiou executed under
oath the 2003 General Information
Sheet (GIS) wherein he falsely
stated that the shareholdings of
the Spouses Sy had decreased
despite the fact that they had not
executed any conveyance of their
shares.

The Spouses Sy elevated the DOJs


resolutions to the Court of Appeals
through a petition for certiorari,
imputing grave abuse of discretion
on the part of the DOJ. The
appellate
court
granted
the
petition and directed the City
Prosecutors Office to file the
appropriate informations against
Sy Tiong Shiou, et al. for violation
of Section 74, in relation to
Section 144 of the Corporation
Code and of Articles 172 and 183
of the RPC. The appellate court
ruled that the civil case for
accounting and damages cannot be
deemed
prejudicial
to
the
maintenance or prosecution of a
criminal action for violation of
Section 74 in relation to Section
144 of the Corporation Code since
a finding in the civil case that
respondents
mishandled
or
misappropriated the funds would
not be determinative of their guilt
or innocence in the criminal
complaint. In the same manner,
the
criminal
complaints
for
falsification and/or perjury should
not have been dismissed on the
ground of prejudicial question
because the accounting case is
unrelated and not necessarily
determinative of the success or
failure of the falsification or
perjury charges. Furthermore, the
Court of Appeals held that there
was probable cause that Sy Tiong
Shiou had committed falsification
and that the City of Manila where
the 2003 GIS was executed is the
proper venue for the institution of
the perjury charges. Sy Tiong
Shiou,
et
al.
sought
reconsideration of the Court of

Appeals decision but their motion


was denied.
ISSUE:
Whether he DOJ gravely abused its
discretion when it suspended the
hearing of the charges for violation
of the Corporation Code on the
ground of prejudicial question and
when it dismissed the criminal
complaints
HELD:
As correctly found by the Court of
Appeals, the DOJ gravely abused
its discretion when it suspended
the hearing of the charges for
violation of the Corporation Code
on the ground of prejudicial
question and when it dismissed the
criminal complaints.
A prejudicial question comes into
play generally in a situation where
a civil action and a criminal action
are both pending and there exists
in the former an issue which must
be preemptively resolved before
the criminal action may proceed
since howsoever the issue raised in
the civil action is resolved would
be determinative juris et de jure of
the guilt or innocence of the
accused in the criminal case. The
reason behind the principle of
prejudicial question is to avoid two
conflicting decisions. It has two
essential elements: (a) the civil
action involves an issue similar or
intimately related to the issue
raised in the criminal action; and
(b) the resolution of such issue
determines whether or not the
criminal action may proceed.

The civil action and the criminal


cases
do
not
involve
any
prejudicial question.
The civil action for accounting and
damages, Civil Case No. 03106456 pending before the RTC
Manila, Branch 46, seeks the
issuance of an order compelling
the Spouses Sy to render a full,
complete and true accounting of
all the amounts, proceeds and fund
paid to, received and earned by
the corporation since 1993 and to
restitute it such amounts, proceeds
and funds which the Spouses Sy
have
misappropriated.
The
criminal cases, on the other hand,
charge that the Spouses Sy were
illegally prevented from getting
inside company premises and from
inspecting company records, and
that Sy Tiong Shiou falsified the
entries in the GIS, specifically the
Spouses
Sys
shares
in
the
corporation. Surely, the civil case
presents no prejudicial question to
the criminal cases since a finding
that the Spouses Sy mishandled
the funds will have no effect on the
determination of guilt in the
complaint for violation of Section
74 in relation to Section 144 of the
Corporation Code; the civil case
concerns the validity of Sy Tiong
Shious refusal to allow inspection
of the records, while in the
falsification and perjury cases,
what is material is the veracity of
the entries made by Sy Tiong
Shiou in the sworn GIS.
Anent the issue of probable cause,
the Court also finds that there is
enough probable cause to warrant
the institution of the criminal
cases.

In order that probable cause to


file a criminal case may be
arrived at, or in order to
engender
the
well-founded
belief that a crime has been
committed, the elements of the
crime
charged
should
be
present. This is based on the
principle that every crime is
defined by its elements, without
which there should beat the
mostno criminal offense. (Sy
Thiong Siou vs Sy Chim, G.R.
No. 174168, March 30, 2009)

Heirs of Jose Sy Bang vs Sy,


G.R. No. 114217, October 13,
2009
Respondents are the children of Sy
Bang by his second marriage to
respondent
Rosita
Ferrera-Sy,
while petitioners are the children
of Sy Bang from his first marriage
to Ba Nga, and petitioners Zenaida
Tan and Ma. Emma Sy are the
children of petitioner spouses Jose
Sy Bang and Iluminada Tan.
Sy Bang died intestate in 1971,
leaving behind real and personal
properties,
including
several
businesses
During an out-of-court conference
between
petitioners
and
respondents, it was agreed that
the management, supervision or
administration of the common
properties and/or the entire estate
of the deceased. Moreover, the
judge
rendered
two
partial
decisions, these First Partial
Decision
and
Second
Partial
Decision had long become final,
without an appeal having been
interposed by any of the parties.
After several incidents in the case,
the Court, on May 8, 1989,
referred the petition to the CA for
proper
determination
and
disposition. The CA rendered the
assailed Decision, denying due
course to and dismissing the
petition for lack of merit. It held
that Judge Puno acted correctly in
issuing the assailed Third Partial
Decision. The CA said that the act
of Judge Puno in rendering a
partial decision was in accord with
then Rule 36, Section 4, of the

Rules of Court, which stated that


in an action against several
defendants, the court may, when a
judgment
is
proper,
render
judgment against one or more of
them, leaving the action to
proceed against the others. It
found that the judges decision to
defer resolution on the properties
in the name of respondents would
not affect the resolution on the
properties
in
the
names
petitioners, since the properties
were separable and distinct from
one another such that the claim
that the same formed part of the
Sy Bang estate could be the
subject of separate suits.
The CA also upheld the judges
appointment of a receiver, saying
that the judge did so after both
parties
had
presented
their
evidence and upon verified petition
filed by respondents, and in order
to preserve the properties under
litigation. Further, the CA found
proper the order to cancel the
notice of lis pendens annotated in
the certificates of title in the
names of Rosalino, Rolando and
Bartolome.
ISSUE:
Whether the payment of widows
allowance be implemented at
present because the estate of Sy
Bang the source from which
payment is to be taken has not
been determined with finality.
HELD:
A finding of probable cause does
not conclusively prove the charge

of
falsification
respondents.

against

In a preliminary investigation,
probable cause has been defined
as the existence of such facts and
circumstances as would excite the
belief, in a reasonable mind, acting
on the facts within the knowledge
of the prosecutor, that the person
charged was guilty of the crime for
which he was prosecuted.
It is
well-settled that a finding of
probable cause needs to rest only
on evidence showing that more
likely than not a crime has been
committed and was committed by
the suspects. Probable cause need
not be based on clear and
convincing evidence of guilt,
neither on evidence establishing
guilt beyond reasonable doubt, and
definitely
not
on
evidence
establishing absolute certainty of
guilt.[102]
Hence, until the marriage is finally
declared void by the court, the
same is presumed valid and Rosita
is entitled to receive her widows
allowance to be taken from the
estate of Sy Bang.
We remind petitioners again that
they are duty-bound to comply
with whatever the courts, in
relation to the properties under
litigation, may order.

Probable cause need not be


based on clear and convincing
evidence of guilt, neither on
evidence
establishing
guilt
beyond reasonable doubt, and

definitely
not
on
evidence
establishing absolute certainty
of guilt. (Heirs of Jose Sy Bang
vs Sy, G.R. No. 114217, October
13, 2009)

Limanch-O Hotel and Leasing


Corporation, et al. vs City of
Olongapo, G.R. No. 185121,
January 18, 2010
Sometime in 1993, the respondent
City
of
Olongapo
assessed,
through
its
Public
Utilities
Department
(PUD),
petitioner
Conrado Tiu (the owner, president,
and general manager of petitioner
Limanch-O Hotel and Leasing
Corporation)
his
unregistered
electricity
consumption
from
November 1988 to February 1993
in the amount of P9,364,276.50.
The City threatened to cut off his
electric supply if he did not
immediately settle the amount.
Petitioner Tiu filed an action
against the City before the
Regional Trial Court (RTC) of
Olongapo for injunction with
damages, which he won. The RTC
enjoined the City from collecting
the deficiency amount and from
cutting off Tius power supply.
Pending the RTCs resolution of its
motion for reconsideration, the
City filed criminal complaints
against petitioner Tiu for: (a) theft
of electrical current punished
under Presidential Decree (P.D.)
401; and (b) disengaging and
tampering with his electric meters
potential link, thereby resulting to
a zero-zero power consumption in
violation of City Ordinance 23,
series of 1989, and P.D. 401.
After the preliminary investigation,
the state prosecutor issued a
resolution,
dismissing
the
complaints for insufficiency of
evidence. On appeal, however, the

Acting
Secretary
of
Justice
modified the State Prosecutors
resolution and directed the filing
of the corresponding information
for theft of electricity against
petitioner
Tiu.
Subsequently,
however, the Secretary of Justice
reconsidered and ordered instead
the withdrawal of any information
that might in the meantime have
been filed in court. When the
matter was elevated to the Court
of Appeals (CA) and, ultimately, to
this Court, both courts affirmed
the
dismissal
of
the
Citys
complaints against Tiu.
Claiming
that
petitioner
Tiu
suffered mental anguish, serious
anxiety, besmirched reputation,
wounded feelings, moral shock and
social
humiliation
and
that
petitioner
Limanch-O
Hotel
suffered loss of business goodwill,
financial reverses, and injured
reputation, both filed an action for
damages against the City for
having filed a malicious and
unfounded charge of theft of
electricity against them.
In its answer, the City denied any
ill motive in filing the criminal
complaint. It explained that it filed
the criminal action following an
examination of the electric meter
installed at petitioner Tius building
and registered in his name. The
examination
showed
reverse
polarity markings on the electric
meter, causing it not to register
Tius correct power consumption.
Since this brought tremendous
losses to the PUD and to the City,
the latter argued that it should not
be faulted for doing its job of going

after those who pilfer electricity


and tamper with metering devices.
After petitioners Tiu and LimanchO Hotel rested their case, the City
filed a demurrer to the evidence.
The RTC granted the same and
dismissed
the
complaint.
On
appeal, the CA affirmed the RTC
decision,
having
found
no
reversible error in the same. Tiu
and Limanch-O Hotel, said the
appellate court, were unable to
prove the elements that will
support an action for malicious
prosecution, namely: (a) absence
of probable cause in the filing of
the criminal case and (b) a
showing of legal malice. Their
motion for reconsideration having
been denied, Tiu and Limanch-O
Hotel are now before this Court on
a petition for review.
Issue Presented
whether or not the CA erred in its
finding that petitioners Tiu and
Limanch-O Hotel failed to present
sufficient evidence showing that
respondent City instituted the
criminal complaint for theft of
electricity
against
them
maliciously and without probable
cause.
HELD
It is not enough to say that, since
the Supreme Court sustained the
Secretary of Justices finding that
no probable cause for electricity
theft existed against petitioners
Tiu and Limanch-O Hotel, a case
for malicious prosecution already
exists against the complainant.
When the Supreme Court reviewed
the resolution of the Secretary of

Justice, it merely determined if he


gravely abused his discretion in
the matter. The Courts finding
does not amount to a judicial
determination that the evidence
established probable cause.
The test should be whether
sufficient facts exist which show
that, in bringing the criminal
action, complainant acted without
probable cause, defined as the
existence of such facts and
circumstances as would excite the
belief in a reasonable mind that
the
person
charged
and
prosecuted in a criminal case is
probably guilty of the crime or
wrongdoing. Here, the fact that
the filing of the complaint was
prompted by the result of an
investigation shows that the City
had a reasonable ground to believe
that a crime had probably been
committed. Additionally, the fact
that the Department of Justice at
first found basis for filing the
charge of theft of electricity
indicates that the existence of
probable cause is not clearly
settled,
only
that
its
final
determination had to succumb to
the sound discretion of the
Secretary of Justice under his
power to review, revise, or
overturn the findings of his
subordinates.

The test should be whether


sufficient facts exist which
show that, in bringing the
criminal action, complainant
acted without probable cause,
defined as the existence of such
facts and circumstances as

would excite the belief in a


reasonable
mind
that
the
person charged and prosecuted
in a criminal case is probably
guilty
of
the
crime
or
wrongdoing. (Limanch-O Hotel
and Leasing Corporation, et al.
vs City of Olongapo, G.R. No.
185121, January 18, 2010)

Kinds of Determination of
Probable Cause
People vs Castillo, G.R. No.
171188, June 19, 2009
Sometime in 1993, the respondent
City
of
Olongapo
assessed,
through
its
Public
Utilities
Department
(PUD),
petitioner
Conrado Tiu (the owner, president,
and general manager of petitioner
Limanch-O Hotel and Leasing
Corporation)
his
unregistered
electricity
consumption
from
November 1988 to February 1993
in the amount of P9,364,276.50.
The City threatened to cut off his
electric supply if he did not
immediately settle the amount.
After the preliminary investigation,
the state prosecutor issued a
resolution,
dismissing
the
complaints for insufficiency of
evidence. On appeal, however, the
Acting
Secretary
of
Justice
modified the State Prosecutors
resolution and directed the filing
of the corresponding information
for theft of electricity against
petitioner
Tiu.
Subsequently,
however, the Secretary of Justice
reconsidered and ordered instead
the withdrawal of any information
that might in the meantime have
been filed in court. When the
matter was elevated to the Court
of Appeals (CA) and, ultimately, to
this Court, both courts affirmed
the
dismissal
of
the
Citys
complaints against Tiu.
Claiming
that
petitioner
Tiu
suffered mental anguish, serious
anxiety, besmirched reputation,

wounded feelings, moral shock and


social
humiliation
and
that
petitioner
Limanch-O
Hotel
suffered loss of business goodwill,
financial reverses, and injured
reputation, both filed an action for
damages against the City for
having filed a malicious and
unfounded charge of theft of
electricity against them.
ISSUE
whether or not the CA erred in its
finding that petitioners Tiu and
Limanch-O Hotel failed to present
sufficient evidence showing that
respondent City instituted the
criminal complaint for theft of
electricity
against
them
maliciously and without probable
cause.
HELD
Here, the respondent City did not
concoct out of thin air the criminal
charge for theft of electricity
against
petitioners
Tiu
and
Limanch-O Hotel. It filed the case
based on the result of an
investigation carried out at Tius
premises
which
indicated
a
tampering of the electric meter.
Indeed, petitioners never claimed
that the inspection of Tius
premises was just a farce. The City
did not merely conjure the charge
with the intention of vexing Tiu
and Limanch-O Hotel. It acted
within its right to bring up the
result of that investigation to the
authorities for evaluation and
resolution.

It is not enough to say that, since


the Supreme Court sustained the
Secretary of Justices finding that
no probable cause for electricity
theft existed against petitioners
Tiu and Limanch-O Hotel, a case
for malicious prosecution already
exists against the complainant.
When the Supreme Court reviewed
the resolution of the Secretary of
Justice, it merely determined if he
gravely abused his discretion in
the matter. The Courts finding
does not amount to a judicial
determination that the evidence
established probable cause.
It must be stressed that in our
criminal justice system, the public
prosecutor
exercises
a
wide
latitude
of
discretion
in
determining whether a criminal
case should be filed in court, and
that courts must respect the
exercise of such discretion when
the information filed against the
person charged is valid on its face,
and that no manifest error or
grave abuse of discretion can be
imputed to the public prosecutor.
Thus, absent a finding that an
information is invalid on its face or
that the prosecutor committed
manifest error or grave abuse of
discretion, a judges determination
of probable cause is limited only to
the judicial kind or for the purpose
of deciding whether the arrest
warrants should be issued against
the accused.
Finally, no evidence was shown
that there had been bad blood
between respondent City and
petitioners Tiu and Limanch-O
Hotel prior to the filing of the

criminal
charge,
which
circumstance if present could
justify a malicious motive in filing
the charge. Resort to judicial
processes, by itself, is not an
evidence of ill will which would
automatically
make
the
complainant liable for malicious
prosecution. Otherwise, peaceful
recourse to the courts will be
greatly
discouraged
and
the
exercise of ones right to litigate
would become meaningless and
empty.

It must be stressed that in our


criminal justice system, the
public prosecutor exercises a
wide latitude of discretion in
determining whether a criminal
case should be filed in court,
and that courts must respect
the exercise of such discretion
when the information filed
against the person charged is
valid on its face, and that no
manifest error or grave abuse
of discretion can be imputed to
the public prosecutor.
Thus, absent a finding that an
information is invalid on its
face or that the prosecutor
committed manifest error or
grave abuse of discretion, a
judges
determination
of
probable cause is limited only
to the judicial kind or for the
purpose of deciding whether
the arrest warrants should be
issued against the accused.
(People vs Castillo, G.R. No.
171188, June 19, 2009)

Webb vs De Leon, 247 SCRA


652
Facts: Hubert Webb was one of the
accused in the high-profile case
Vizconde massacre. Preliminary
investigation was provided by NBI
and the case was raffled to Judge
Zosimo Escano who inhibited
himself from the case for being
employed with NBI before. His
pair Judge Escano issued warrant
of arrest to defendants. The case
was re-raffled to Branch 274,
presided
by
Judge
Amelita
Tolentino who issued new warrants
of arrest. Webb and the others
voluntarily surrendered. They files
before the court petition of
certiorari,
prohibition
and
mandamus. They contend that (1)
respondent Judges de Leon and
Tolentino gravely abused their
discretion when they failed to
conduct a preliminary examination
before issuing warrants of arrest
against them: (2) the DOJ Panel
likewise
gravely
abused
its
discretion in holding that there is
probable cause to charge them
with the crime of rape with
homicide; (3) the DOJ Panel denied
them their constitutional right to
due
process
during
their
preliminary investigation; and (4)
the DOJ Panel unlawfully intruded
into judicial prerogative when it
failed to charge Jessica Alfaro in
the Information as an accused.
Issue:
Whether or not the attendant
publicity deprived Webb and the
others of their right to fair trial?
Decision:

Petition dismissed. to warrant a


finding of prejudicial publicity
there must be allegation and proof
that the judges have been unduly
influenced, not simply that they
might be, by the barrage of
publicity. In the case at bar, we
find nothing in the records that
will prove that the tone and
content, of the publicity that
attended the investigation of
petitioners fatally infected the
fairness and impartiality of the
DOJ Panel. Petitioners cannot just
rely on the subliminal effects of
publicity on the sense of fairness
of the DOJ Panel, for these are
basically unbeknown and beyond
knowing. To be sure, the DOJ Panel
is composed of an Assistant Chief
State Prosecutor and Senior State
Prosecutors. Their long experience
in criminal investigation is a factor
to
consider
in
determining
whether they can easily be blinded
by the klieg lights of publicity.
Indeed, their 26-page Resolution
carries no indubitable indicia of
bias for it does not appear that
they considered any extra-record
evidence except evidence properly
adduced by the parties. The length
of time the investigation was
conducted despite its summary
nature and the generosity with
which they accommodated the
discovery motions of petitioners
speak well of their fairness. At no
instance, we note, did petitioners
seek the disqualification of any
member of the DOJ Panel on the
ground of bias resulting from their
bombardment
of
prejudicial
publicity.

Probable cause to warrant an


arrest which is made by the
judge refers to facts and
circumstances
which
would
lead a reasonably discreet and
prudent man to believe that an
offense has been committed by
the
person
sought
to
be
arrested. Other jurisdictions
utilize
the
term
man
of
reasonable caution or the term
ordinarily prudent and cautious
man. (Webb vs De Leon, 247
SCRA 652)

Domalanta vs COMELEC, G.R.


No. 125586, June 29, 2000
From the record, it appears that
on August 4, 1995, then senatorial
candidate Aquilino Pimentel, Jr.
filed a complaint-affidavit charging
Provincial
Election
Supervisor
(PES) Vitaliano Fabros, Provincial
Prosecutor Pacifico Paas and
Division Superintendent of Schools
Dr. Olympia Marquez, Chairman,
Vice-Chairman
and
MemberSecretary, respectively, of the
Provincial Board of Canvassers of
Isabela with alleged violation of
Section 27 (b) of Republic Act No.
6646, otherwise known as the
Guingona Electoral Reform Law of
1987
for
the
irregularity
committed by the Provincial Board
of Canvassers of Isabela in
crediting unauthorized additional
votes. In their defense,Vitaliano
Fabros, Pacifico Paas and Dr.
Olympia Marquez were in unison
in vehemently denying the charges
imputed
against
them
and
declared
that
they
faithfully
performed
their
poll
duties

assigned to them. Respondents Dr.


Teresita Domalanta and Agripina
Francisco, in their joint counteraffidavit, categorically denied the
charges, and declared that they
faithfully
recorded
the
votes
obtained by the candidates as
read. Respondents Dante Limon
and Eduardo Tamang, in their
joint-affidavit, vehemently denied
the charges. They claimed that
their assigned duty is only to
record the names of candidates
and their corresponding number of
votes obtained as announced by
PBC Chairman, Atty. Vitaliano
Fabros because they have no
access to the votes written in the
Municipal Certificate of Canvass.
Respondent George Noriega, in his
counter-affidavit likewise denied
the charges and averred that he
had no direct knowledge in the
preparation of the alleged falsified
Provincial Certificate of Canvass,
and as Tabulator, he only added
what
was
recorded
in
the
Statement of Votes prepared by
other persons and denied any
participation
in
the
alleged
falsification of the Statement of
Votes. Based on the factual
findings,
COMELEC
Law
Department recommended that the
cases
against
Dr.
Teresita
Domalanta, Agripina Francisco,
Dante Limon, Edwardo Tamang
and George Noriega be dismissed
for insufficiency of evidence to
establish a probable cause.
Issue:
WON probable cause exist
Held:

Probable cause is a reasonable


ground of presumption that a
matter is, or may be, well founded,
such a state of facts in the mind of
the prosecutor as would lead a
person of ordinary caution and
prudence to believe or entertain an
honest or strong suspicion, that a
thing is so. The term does not
mean "actual or positive cause"
nor does it import absolute
certainty. It is merely based on
opinion and reasonable belief.
Thus, a finding of probable cause
does not require an inquiry into
whether
there
is
sufficient
evidence to procure a conviction.
It is enough that it is believed that
the act or omission complained of
constitutes the offense charged. It
ought to be emphasized that in
determining probable cause, the
average man weighs facts and
circumstances without resorting to
the calibration of our technical
rules of evidence of which his
knowledge is nil. Rather, he relies
on the calculus of common sense
of which all reasonable men have
abundance.
The peculiar factual circumstances
prevailing in this case hardly
paints a picture of manifest human
error or fatigue in the tabulation of
the
votes
of
the
senatorial
candidates in Isabela. It, in fact,
discloses a pernicious scheme
which would not have been
successfully perpetrated without
the indispensable cooperation of
all members of the PBC and their
support staff which included
herein petitioners. The latters
protestations in the counteraffidavits that they only tabulated

the vote totals of senatorial


candidates Tillah and Tolentino are
at best convenient and self-serving
explanations
to
justify
their
exculpation from any wrong-doing.
Their claims are, moreover, not
substantiated by any of the PBC
members. It can be clearly seen
from the list that the discrepancies
are too substantial and rounded off
to be categorized as a mere
computation error or a result of
fatigue. There is a limit to what
can be construed as an honest
mistake or oversight in the
performance
of
official
duty.
Suffice it to state that the
magnitude of the error as reflected
in the discrepancies itemized
renders unacceptable the defense
of computer error or honest
mistake. A reasonably prudent
man on the other hand would
readily come to the conclusion that
there exists a probable cause to
believe that the petitioners are
culpable together with the other
members of the support staff as
well as the PBC members in the
padding of the vote totals of the
said senatorial candidates. It can
not be denied that the members of
the PBC and their support staff,
including herein petitioners, were
the only ones in control and in
possession of said documents
during its preparation. It need not
be overemphasized, given this fact,
that the padding of the vote totals
could only have been done by all of
them acting in concert with one
another

It ought to be emphasized that


in determining probable cause,

the average man weighs facts


and
circumstances
without
resorting to the calibration of
our technical rules of evidence
of which his knowledge is nil.
Rather,
he
relies
on
the
calculus of common sense of
which all reasonable men have
an abundance. (Domalanta vs
COMELEC, G.R. No. 125586,
June 29, 2000)

Preliminary Investigation
Tabujara III v. People, G.R.
No. 175162, October 29,
2008
Ladlad v. Velasco, 523 SCRA
218
Ruiz v. Beldia, Jr., 451 SCRA
402
People v. Gomez, 117 SCRA
73
Larranaga v. Court of
Appeals, 287 SCRA 581
Vasquez v. Hobilia-Alinio,
271 SCRA 67
Budiongan, Jr. v. De la Cruz,
502 SCRA 626
Crispin Beltran v. People and
Secretary Gonzales, G.R. No.
175013, June 1, 2007
Mago v. Penalosa-Fermo, 582
SCRA 1
Sibulo v. Toledo-Mupas, A.M.
No. MTJ-07-1686, June 12,
2008
Bienvenido Dino and Renato
Comparativo v. Pablo
Olivarez, G.R. No. 170447,
December 4, 2009
Re: Subpeona Duces Tecum
dated January 11, 2010 of
Acting Director Aleu A.

Amante, PIAB-C Office of the


Ombudsman, A.M. No. 10-112-SC, March 2, 2010
Santos-Concio v. Department
of Justice, 543 SCRA 70
De Ocampo v. Secretary of
Justice, 480 SCRA 71
Dupasquier v. Court of
Appeals, G.R. No. 112089,
January 24, 2001
People v. Marquez, 27 SCRA
808
Community Rural Bank of
Guimba (N.E.), Inc. v.
Talavera, 455 SCRA 34
Asetre v. Asetre, G.R. No.
171536, April 7, 2009
Alcaraz v. Gonzales, G.R. No.
164715, September 20, 2006
Sy Thiong Shiou vs Sy Chim,
G.R. No. 174168, March 30,
2009
Angeles v. Gaite, G.R. No.
176596, March 23, 2011
Elvira O. Ong v. Jose Casim
Genio, G.R. No. 182336,
December 23, 2009
Pangan v. Ganay, 445 SCRA
574
Crespo v. Mogul, 151 SCRA
462
Santos v. Orda, Jr. 437 SCRA
504
Dumalo v. Ponferrada, 508
SCRA 426
Filemon Verzano, Jr. v.
Francis Victor D. Paro, G.R.
No. 171643, August 8, 2010
Sierra v. Lopez, A.C. 7549,
August 29, 2008
Bautista v. Sandiganbayan,
G.R. No. 136082, May 12,
2000
Sasot v. People

ARREST
Tabujara v. People, G.R. No.
175162, October 29, 2008
Borlongan v. Pena, G.R. No.
143591, May 5, 2012
People v. Tan, G.R. No.
182310, December 9, 2009
AAA v. Carbonell, G.R. No.
171465, June 8, 2007
Soliven v. Makasiar, G.R.
Nos. L-82585, L-82827,
November 14, 1988

Tabujara III v. People, G.R. No.


175162, October 29, 2008

Facts:
On 17 September 1999,
respondent Daisy Dadivas-Afable
simultaneously filed two criminal
complaints against petitioners for
Grave Coercion and Trespass to
Dwelling. On the 14th day of
September 1999 at around 6:00
oclock in the morning, in Brgy.
Iba, Municipality of Meycauayan,
Province of Bulacan, petitioners
entered the house of DavidasAfable by opening the gate and
forcing her to go with them
against her will. Petitioners denied
the allegations and argued that on
14 September 1999, they went to
the house of respondent to thresh
out matters regarding some
missing pieces of jewelry.
Respondent was a former
employee of Miladay Jewels, Inc., a
company owned by the Dayrits and
who was then being
administratively investigated in
connection with missing jewelries.
Despite several summons to
appear, respondent went on
AWOL. Judge Calixto Adriatico of
Meycauayan MTC conducted the
preliminary investigation and
thereafter dismissed the
complaints for lack of probable
cause on the ground that the
complaint was believed to be
merely leverage against the estafa
case already filed against the
private complainant. Respondent
filed a Motion for Reconsideration
alleging that when she filed the
complaints for grave coercion and
trespass to dwelling on 17

September 1999 against


petitioners, no information for
estafa has yet been filed against
her. In fact, the information was
filed on 5 October 1999. In their
Opposition to the Motion for
Reconsideration, petitioners
argued that even before
respondent filed the criminal
complaints for grave coercion and
trespass to dwelling, she was
already being administratively
investigated for the missing
jewelries; that she was ordered
preventively suspended pending
said investigation; that the theft of
the Miladay jewels was reported to
the Makati Police on 7 September
1999 with respondent Afable being
named as the primary suspect;
that on 17 September 1999, which
corresponded to the date of filing
of the criminal complaints against
petitioners, the employment of
respondent with Miladay, Jewels,
Inc. was terminated. Petitioners
further alleged that respondent
filed the criminal complaints for
grave coercion and trespass to
dwelling as leverage to compel
petitioners to withdraw the estafa
case.
On 2 May 2000, Judge Adriatico
issued an Order reversing his
earlier findings of lack of probable
cause. This time, he found
probable cause to hold petitioners
for trial and to issue warrants of
arrest on the ground that the
sworn statement of witness Mauro
V. De Lara was inadvertently
overlooked. De Lara stated that he
saw the private complainant being
forcibly taken by three persons
from her residence. Petitioners

filed a motion for reconsideration


insisting that the alleged affidavit
of Mauro V. de Lara on which the
court a quo based its findings of
probable cause was hearsay
because it was not sworn before
Judge Adriatico; that De Lara did
not personally appear before the
investigating judge during
preliminary investigation.
Issue:
WON complaints or informations
filed directly with the prosecutor
or MTC also adheres to procedures
determining probable cause.
Held:
Yes. For cases where the penalty
prescribed by law is lower than 4
years, 2 months and 1 day, a
criminal complaint may be filed
directly with the prosecutor or
with the Municipal Trial court. In
either case, the prosecutor or MTC
Judge is still required to adhere to
certain procedures for the
determination of probable cause
and issuance of warrant of arrest.
The procedure described in
Section 6 of Rule 112 is mandatory
because failure to follow the same
would amount to a denial of due
process. With respect to the
issuance by inferior courts of
warrants of arrest, it is necessary
that the judge be satisfied that
probable cause exists: 1) through
an examination under oath and in
writing of the complainant and his
witnesses; which examination
should be 2) in the form of
searching questions and answers.
This rule is not merely a
procedural but a substantive rule

because it gives flesh to two of the


most sacrosanct guarantees found
in the fundamental law: the
guarantee against unreasonable
searches and seizures and the due
process requirement.
Judge Adriatico gravely abused his
discretion in issuing Orders finding
probable cause to hold petitioners
liable for trial and to issue
warrants of arrest because it was
based solely on the statement of
witness Mauro De Lara whom
Judge Adriatico did not personally
examine in writing and under oath;
neither did he propound searching
questions. He merely stated in the
assailed Order that he overlooked
the said statement of De Lara.
Plainly, this falls short of the
requirements imposed by no less
than the Constitution. Any wanton
disregard of the carefully-wrought
out processes established pursuant
to the Constitutions provisions on
search warrants and warrants of
arrest is a serious matter primarily
because its effects on the
individual wrongly-detained are
virtually irremediable.

Ladlad v. Velasco, 523 SCRA 218


Facts:
Ladlad, Maza, et al were called for
a preliminary investigation for the
crime of rebellion in connection
with the CIDG investigation, culled
from the inquest of Beltran. During
the Preliminary Investigation,
CIDG presented a masked man,
later identified as Jaime Fuentes,
who claimed to be an eyewitness
against petitioners. Velasco, who
was the prosecutor, gave copies of
the affidavit of Fuentes to media
members present during the
proceedings. Ladlad moved for the
inhibition of Velasco citing lack of
impartiality and independence,
considering the political milieu
under which petitioners were
investigated, the statements that
the President and the Secretary of
Justice made to the media
regarding petitioners case, and
the manner in which the
prosecution panel conducted the
preliminary investigation.
Furthermore, they contend that
the PI was tainted with
irregularities as not pursuant to
Rule 112 Sec 3
Issue:
WON the Preliminary Investigation
conducted against Ladlad and
Maza were tainted with
irregularity?
Held:
Yes. The prosecutors failed to
comply with Section 3a of Rule 112
which provides that the complaint
must be accompanied by the
affidavits of the complainant and

his witnesses, subscribed and


sworn to before any prosecutor or
government official authorized to
administer oath, or, in their
absence or unavailability, before a
notary public. Here, the
prosecutors treated the
unsubscribed letters of Tanigue
and Mendoza of the CIDG, PNP as
complaints and accepted the
affidavits attached to the letters
even though some of them were
notarized by a notary public
without any showing that a
prosecutor or qualified
government official was
unavailable as required by Section
3(a) of Rule 112.
Section 3(b) of Rule 112 also
mandates that the prosecutor,
after receiving the complaint, must
determine if there are grounds to
continue with the investigation. If
there is none, he shall dismiss the
case, otherwise he shall issue a
subpoena to the respondents.
Here, after receiving the CIDG
letters, respondent prosecutors
peremptorily issued subpoenas to
petitioners requiring them to
appear at the DOJ office on 13
March 2006 to secure copies of
the complaints and its
attachments.
During the investigation,
prosecutors allowed the CIDG to
present a masked Fuentes who
subscribed to an affidavit before
respondent prosecutor Velasco.
Velasco proceeded to distribute
copies of Fuentes affidavit not to
petitioners or their counsels but to
members of the media who
covered the proceedings. Indeed,
by peremptorily issuing the

subpoenas to petitioners,
tolerating the complainants antics
during the investigation, and
distributing copies of a witness
affidavit to members of the media
knowing that petitioners have not
had the opportunity to examine the
charges against them, respondent
prosecutors not only trivialized the
investigation but also lent
credence to petitioners claim that
the entire proceeding was a sham.
Hence, the court concluded that
there was indeed partiality on the
part of the prosecutors who
conducted the PI.

Ruiz v. Beldia, Jr., 451 SCRA 402


Facts:
Shirley C. Ruiz charged
respondent Judge Rolindo D.
Beldia, Jr. of Branch 57, Regional
Trial Court, San Carlos City,
Negros Occidental, with gross
ignorance of the law and grave
abuse of authority in connection
with the grant of bail and issuance
of a release order in favor of one
Lourdes Estrella Santos. Ruiz is
the private complainant in a case
for violation of the Anti-Fencing
Law pending before the
Department of Justice (DOJ).
Santos, who was arrested during
entrapment operations relative to
the carnapping of Ruizs vehicle,
was one of the respondents
therein. After her arrest, Santos
was detained in Camp Crame,
Quezon City, pending the filing of
formal charges in court. Upon
inquest, she executed a waiver of
the provisions of Article 125 of the
Revised Penal Code in relation to
Rule 112, Section 7 of the then
applicable 1985 Rules of Criminal
Procedure. The Inquest Prosecutor
thus set the hearing of the
preliminary investigation on May
31, 2000 at 2:00 PM. However, on
May 30, 2000, Santos obtained an
Order of Release signed by
respondent Judge Beldia who was
then detailed as assisting judge of
Branch 272, Regional Trial Court
of Marikina City (RTC-Marikina
City). Respondent Judge Beldia
apparently granted bail to Santos
and approved the corresponding
bail bond without serving notice to
the prosecutor. Consequently, Ruiz
filed the instant administrative

complaint contending that


respondent Judge Beldia had no
authority to grant bail to Santos
since the Investigating Prosecutor
has yet to conclude the
preliminary investigation. She
claimed that for as long as the
information has not yet been filed
in court, a court has no power to
grant bail to a detained person
since it has not yet acquired
jurisdiction over the person of the
accused.
ISSUE:
WON Judge Beldia is guilty of
gross ignorance of the law
Held:
Yes. Section 17, of Rule 114 states
that any person in custody who is
not yet charged in court may apply
for bail with any court in the
province, city ir municipality
where he is held. Undeniably too,
Santos was entitled to bail as a
matter of right since the offense
with which she was charged does
not carry the penalty of life
imprisonment, reclusion perpetua
or death. Notwithstanding, it was
incumbent upon respondent Judge
Beldia to grant bail to Santos in
accordance with established rules
and procedure. Respondent Judge
Beldia failed in this respect and
must thus be held administratively
liable. The application for bail
should have been filed before the
proper Quezon City court and not
in Marikina City. Furthermore,
record shows that the Executive
Judge and Presiding Judge were
present on May 30, 2000 to act on
the bail application of Santos.
When respondent Judge Beldia

acted on the bail application of


Santos on May 30, 2000, his
designation was merely an
assisting judge in the RTCMarikina City, his permanent
station being in Branch 57, RTCSan Carlos City, Negros
Occidental. As such, his authority
in the Marikina court is limited
and he could only act on an
application for bail filed therewith
in the absence or unavailability of
the regular judge. In addition, it
appears that no formal application
or petition for the grant of bail was
filed before the RTC-Marikina City.
There were no records of the
application or the release order
issued by respondent Judge Beldia.
Neither was there a hearing
conducted thereon nor the
prosecutor notified of the bail
application. Judge Beldia
disregarded basic procedural rules
when he granted bail to Santos
sans hearing and notice and
without the latter having filed a
formal petition for bail.
Accordingly, the prosecution was
deprived of procedural due
process for which respondent
Judge Beldia must be held
accountable.

People v. Gomez, 117 SCRA 73


Facts:
In 1962, four (4) informations were
filed by the prosecuting fiscals
before the Court of First Instance
of Zamboanga City. Respondents
were charged with the crime of
Estafa thru falsification of
public/official documents. For the
three information, the prosecuting
officers certified under oath that
they had conducted a preliminary
investigation of the case in
accordance with law; and that they
believed that the offense charged
had been committed and the
accused were probably guilty
thereof. The corresponding
warrant of arrest for each of the
accused was accordingly issued
and the accused subsequently filed
their bond for provisional liberty.
For the fourth, since the
information did not contain a
certification that a preliminary
investigation of the case had been
made by the prosecutors, the
District Judge himself made the
preliminary investigation and once
satisfied that a prima facie case
against the three accused existed,
issued warrants for their arrest
and the accused subsequently filed
their bond. On June 22, 1966, the
accused in the four (4) cases, thru
their counsel, filed a MOTION TO
DECLARE INFORMATIONS AND
WARRANTS OF ARREST null and
void on the ground that the
prosecution failed to observe the
provisions of Section 13 and 14 of
Rule 112 of the New Rules of
Court regarding preliminary
investigation and prayed the court
to cancel the warrants of arrest

issued. It was denied but


subsequently granted after a
motion for reconsideration. The
prosecution appealed to this Court
alleging that the trial court erred
"in dismissing the cases on the
ground that the preliminary
investigations conducted therein
were not in accordance with
Sections 13 and 14 of Rule 112, in
relation to Rule 144 of the Revised
Rules of Court.
Issue:
WON the required investigations
were complied with
Held:
Yes. The preliminary investigations
in these four (4) cases were
terminated in 1962, or before the
New Rules of Court took effect on
January 1, 1964. Rules 112 and
113 thereof cannot, therefore,
apply to these cases at bar. The
government prosecutors certified
under oath that they had
conducted a preliminary
investigation in said case in
accordance with law, and on the
basis thereof, the Judges involved
issued the corresponding warrant
of arrest against all the accused. It
is clear, therefore, that the
required investigations were
complied with. But then, assuming
that the informations did not
contain the requisite certificates
regarding the Fiscal's having held
a preliminary investigation, the
omissions are not necessarily fatal.
The absence of preliminary
investigations does not affect the
court's jurisdiction over the case.
Nor do they impair the validity of
the information or otherwise

render it defective. If there were


no preliminary investigations and
the defendants, before entering
their plea, invite the attention of
the court to their absence, the
court, "instead of dismissing the
information, should conduct such
investigation, order the fiscal to
conduct it or remand the case to
the inferior court so that the
preliminary investigation may be
conducted." (People vs. Casiano, 1
SCRA 478). The defendants in
these cases did not question the
validity of the informations on the
ground of defective certifications
or the right to preliminary
investigations before they entered
the plea of not guilty. They filed
the motion to declare informations
and warrants of arrest null and
void only after more than one (1)
year thereafter. Consequently,
when they entered a plea of not
guilty, they thereby waived all
objections that are grounds for a
motion to quash, except lack of
jurisdiction or failure of the
information to charge an offense.
Thus, they waived the right to a
preliminary investigation when
they failed to invoke it prior to, or
at least at, the time of the entry of
their plea in the Court of First
Instance.

Larranaga v. Court of Appeals, 287


SCRA 581
Facts:
Petitioner Francisco Juan
Larranaga is charged with two
counts of kidnapping and serious
illegal detention pending before
the Regional Trial Court (RTC),
Branch 7, Cebu City. On October 1,
1997, petitioner, represented by
his mother, Margarita G.
Larranaga, filed with this Court a
petition for certiorari, prohibition
and mandamus with writs of
preliminary prohibitory and
mandatory injunction. Petitioner
alleged that he was denied the
right to preliminary investigation
and sought to annul the
informations as well as the
warrant of arrest issued in
consequence thereof. In the
alternative, petitioner prayed that
a preliminary investigation be
conducted and that he be released
from detention pending the
investigation. The facts show that
on September 15, 1997, some
members of the Philippine
National Police Criminal
Investigation Group (PNP CIG)
went to the Center for Culinary
Arts in Quezon City to arrest
petitioner, albeit without warrant.
Petitioner resisted the arrest and
immediately phoned his sister and
brother-in-law. OSG recommended
that petitioner be accorded his
right to preliminary investigation
and that he be released from
detention during pendency thereof.
The City Prosecutor of Cebu
however insisted that he was only
entitled to an inquest
investigation. Furthermore, Judge

Ocampo called the Courts


attention to the fact that petitioner
has been arraigned and waived his
right to preliminary investigation.
Petitioner also filed an urgent
motion to transfer the venue of the
preliminary investigation from
Cebu City to Manila and to replace
the Office of the City Prosecutor of
Cebu with the Office of the State
Prosecutor, Department of Justice,
as the authority to conduct the
preliminary investigation because
of the extensive coverage of the
proceedings by the Cebu media
which allegedly influenced the
peoples perception of petitioners
character and guilt.
Issue:
1. WON petitioner is entitled to
a regular preliminary
investigation
2. WON petitioner should be
release from detention
pending investigation
Held:
1. Yes. Section 7 of Rule 112
applies only to persons lawfully
arrested without a warrant.
Petitioner in this case was, in the
first place, not arrested either by a
peace officer or a private person.
Petitioners sister sought the aid of
Atty. Raymundo A. Armovit. Atty.
Armovit, over the phone,
dissuaded the police officers from
carrying out the warrantless arrest
and proposed to meet with them at
the CIG headquarters in Camp
Crame, Quezon City. The police
officers yielded and returned to
the CIG headquarters. Petitioner,
together with his sister and

brother-in-law also went to the CIG


headquarters aboard their own
vehicle. After consulting with his
superiors, Legal Officer Zacarias
ordered to stop the arrest and
allowed petitioner to go home.
Atty. Armovit made an undertaking
in writing that he and petitioner
would appear before the Cebu City
Prosecutor on September 17, 1997
for preliminary investigation. The
foregoing facts show no restraint
upon the person of petitioner.
Neither do they show that
petitioner was deprived of his own
will and liberty. Hence, Section 7
of Rule 112 does not apply to
petitioner. It does not appear in
the case at bar that petitioner has
just committed, is actually
committing or is attempting to
commit an offense when the police
officers tried to arrest him on
September 15, 1997. In fact,
petitioner was attending classes at
the Center for Culinary Arts at that
time. The court also rejectd the
prosecutors' argument that
petitioner was actually committing
a crime at the time of the arrest
since kidnapping with serious
illegal detention is a continuing
crime. There is no showing that at
the time of the arrest on
September 15, 1997, Jacqueline
Chiong was being detained by
petitioner who was then residing
in Quezon City. Hence, petitioner
may not be considered as
continually committing the crime
of kidnapping with serious illegal
detention at the time of the arrest.
Mere failure of petitioner and his
counsel to appear before the City
Prosecutor in the afternoon of
September 17, 1997 cannot be

construed as a waiver of his right


to preliminary investigation,
considering that petitioner has
been vigorously invoking his right
to a regular preliminary
investigation since the start of the
proceedings before the City
Prosecutor. A waiver, whether
express or implied, must be made
in clear and unequivocal manner.
The rule is that the right to
preliminary investigation is waived
when the accused fails to invoke it
before or at the time of entering a
plea at arraignment. Petitioner, in
this case, has been actively and
consistently demanding a regular
preliminary investigation even
before he was charged in court.
Also, petitioner refused to enter a
plea during the arraignment
because there was a pending case
in this Court regarding his right to
avail of a regular preliminary
investigation. Clearly, the acts of
petitioner and his counsel are
inconsistent with a waiver.
Preliminary investigation is part of
procedural due process. It cannot
be waived unless the waiver
appears to be clear and informed.
2. No. The records show that on
September 17, 1997, two
informations were filed against
petitioner for kidnapping and
serious illegal detention. Executive
Judge Priscila Agana issued a
warrant of arrest on September
19, 1997. Petitioner was arrested
on September 22, 1997 by virtue
of said warrant. The rule is that if
the accused objects to the
jurisdiction of the court over his
person, he may move to quash the
information, but only on that

ground. If, as in this case, the


accused raises other grounds in
the motion to quash, he is deemed
to have waived that objection and
to have submitted his person to the
jurisdiction of the court. The
absence of a preliminary
investigation will not justify
petitioners release because such
defect did not nullify the
information and the warrant of
arrest against him. The absence of
preliminary investigations does not
affect the courts jurisdiction over
the case. Nor do they impair the
validity of the information or
otherwise render it defective; but,
if there were no preliminary
investigations and the defendants,
before entering their plea, invite
the attention of the court to their
absence, the court, instead of
dismissing the information, should
conduct it or remand the case to
the inferior court so that the
preliminary investigation may be
conducted. The original
warrantless arrest of the petitioner
was doubtless illegal.
Nevertheless, the RTC lawfully
acquired jurisdiction over the
person of the petitioner by virtue
of a warrant arrest it subsequently
issued on the other information.

Vasquez v. Hobilia-Alinio, 271


SCRA 67
Facts:
Corazon Odelmo appeared before
the Office of the Deputy
Ombudsman for the Visayas on 12
February 1993 and filed a
complaint for the murder of her
husband Dionesio Odelmo and
father-in-law Jose Odelmo against
respondents Mayor Luis Mondia Jr.
of Pulupandan, Negros Occidental,
Sgt. Ruben Olvido and others.
After preliminary investigation was
conducted, it was concluded that
there was probable cause and
Office of the Deputy Ombudsman
recommended the filing of an
Information for double murder.
However upon review the Office of
the Special Prosecutor (OSP)
appreciated the evidence
antithetically; it found that two (2)
separate crimes of murder were
committed but the commission
thereof was not in relation to the
performance of the duties of
private respondents. Two
Informations were then filed and
the corresponding warrants of
arrest were issued. Private
respondents claimed that they had
not yet received a copy of the
complaint neither had they been
furnished a copy of the resolution
by the Ombudsman finding two
separate crimes of murder. They
claimed that it resulted in a denial
of their right to seek
reconsideration or reinvestigation
before the Informations were filed.
They filed a motion to recall the
warrants of arrest. Respondent
Judge ordered that the execution
of the warrants of arrest be held in

abeyance and also granted the


motion to quash information. The
Office of the Ombudsman through
the OSP moved for reconsideration
insofar as respondent Judge
granted the motion to quash the
Informations. It was argued that
the failure to furnish a copy of the
resolution to private respondents
was not an authorized ground to
quash the Informations
Issue:
WON the Judge capriciously and
whimsically ordered the quashing
of the Informations
Held:
Yes. Under Sec. 3, Rule 117, of the
Rules of Court, it is clear that
failure of the prosecution to
furnish copy of the resolution to
private respondents is not one of
the grounds to quash an
information. Private respondents'
proposition that the Office of the
Ombudsman had no authority to
file the Informations would have
held solid ground under Sec. 3,
par. (c), of the Rule. But this seems
to have been ignored by
respondent Judge. Be that as it
may, on the assumption that this
ground was invoked and applied, it
would still be inappropriate to
quash the Informations because
the fact that private respondents
had not yet been served with copy
of the resolution rendered the
filing of the Informations
premature, since the avenue for a
reconsideration or reinvestigation
was still accessible. The Court
emphasized in Torralba v.
Sandiganbayan, 230 SCRA 33
(1994), that the incomplete

preliminary investigation in this


case does not warrant the quashal
of the information, nor should it
obliterate the proceedings already
had. Neither is the court's
jurisdiction nor validity of an
information adversely affected by
deficiencies in the preliminary
investigation. Instead, the
Sandiganbayan is to hold in
abeyance any further proceedings
therein and to remand the case to
the Office of the Ombudsman for
the completion of the preliminary
investigation, the outcome of
which shall then be indorsed to
(the) Sandiganbayan for its
appropriate action.

Budiongan, Jr. v. De la Cruz, 502


SCRA 626
Facts:
Municipal Ordinance No. 2, series
of 2001, of the Municipality of
Carmen, Bohol appropriated the
amount of P450,000.00 for the
purchase of a road roller. However,
the Municipal Development
Council recommended that the
said amount be realigned and used
for the asphalt laying of a portion
of the Tan Modesto Bernaldez
Street. The proposed
realignments discussion was
deferred and the Municipal
reasurer later issued a Certificate
of Availability of funds for the
project. Malmis commenced with
the project after entering into a
contract with Mayor Budiongan. It
was discovered later that there
was yet no ordinance approving
the realignment of the funds.
Arlene P. Palgan and Valeriano U.
Nadala filed a complaint against
the petitioners before the Office of
the Deputy Ombudsman for
Visayas alleging illegality in the
conduct of the bidding, award and
notice to commence work since
there was no fund appropriated for
the purpose. Office of the Deputy
Ombudsman for Visayas found
probable cause and recommended
the filing of an information for
violation of Article 2207 of the
Revised Penal Code. Upon review,
the Case Assessment, Review and
Reinvestigation Bureau of the
Office of the Special Prosecutor,
issued the assailed Memorandum
dated April 28, 2004, modifying
the charge from violation of Article
220 of the Revised Penal Code to

(1) violation of Section 3(e) of R.A.


No. 3019 against petitioners for
allegedly giving unwarranted
benefit to Malmis and (2) violation
of Section 3(h) of R.A. No. 3019
against petitioner Budiongan for
allegedly "directly or indirectly
having financial or pecuniary
interest in a contract or
transaction in connection with
which he intervenes or takes part
in his official capacity." Petitioners
filed a Motion to Quash the
information charging them with
violation of Sec. 3(e) of R.A. No.
3019. Sandiganbayan granted the
motion to quash and remanded
Criminal Case No. 28075 to the
Office of the Ombudsman for
amendment of the Information. On
June 27, 2005, an Amended
Information was filed charging
petitioners with violation of Sec.
3(e) of R.A. No. 3019. The
Sandiganbayan admitted the
Amended Information in its
Resolution. Petitioners filed with
the Sandiganbayan a Motion for
Leave of Court to File Motion for
Reinvestigation arguing that the
above Informations were filed
without affording them the
opportunity to file counteraffidavits to answer/rebut the
modified charges. Sandiganbayan
granted the motion but it was for
reconsideration not for
reinvestigation. Petitioners filed
the motion with the Office of the
Special Prosecutor but were
denied.
Issue:
WON the refusal to conduct
reinvestigation has violated
petitioners rights

Held:
No. The absence of a preliminary
investigation does not impair the
validity of the Information or
otherwise render the same
defective. It does not affect the
jurisdiction of the court over the
case or constitute a ground for
quashing the Information. If
absence of a preliminary
investigation does not render the
Information invalid nor affect the
jurisdiction of the court over the
case, then the denial of a motion
for reinvestigation cannot likewise
invalidate the Information or oust
the court of its jurisdiction over
the case.Petitioners were not
deprived of due process because
they were afforded the opportunity
to refute the charges by filing their
counter-affidavits. The
modification of the offense charged
did not come as a surprise to the
petitioners because it was based
on the same set of facts and the
same alleged illegal acts.
Moreover, petitioners failed to aver
newly discovered evidence nor
impute commission of grave errors
or serious irregularities prejudicial
to their interest to warrant a
reconsideration or reinvestigation
of the case as required under
Section 8, Rule III of the Rules of
Procedure of the Office of the
Ombudsman. Thus, the
modification of the offense
charged, even without affording
the petitioners a new preliminary
investigation, did not amount to a
violation of their rights.

Crispin Beltran v. People and


Secretary Gonzales, G.R. No.
175013, June 1, 2007
Facts:
On February 24, 2006, PGMA
signed Presidential Proclamation
No. 1017 declaring a State of
National Emergency. Following
that, police officers arrested
Crispin Beltran on while he was en
route to Marilao, Bulacan, and
detained him in Camp Crame.
Beltran was arrested without a
warrant and the arresting officers
did not inform Beltran of the crime
for which he was arrested. He was
subjected to a first inquest
involving the crime of inciting to
sedition. This was based on a
speech Beltran allegedly gave
during a rally in Quezon City on 24
February 2006. Inquest prosecutor
found probable cause. He was also
subjected to a second inquest
involving the crime of rebellion
conducted by DOJ state
prosecutors. The inquest was
based on two letters from CIDG
executive officer and deputy
director. The letters contained
results of CIDGs investigation
implicating Beltran, et al as
leaders and promoters of an
alleged foiled plot to overthrow the
Arroyo government. DOJ state
prosecutors found probable cause.
Beltran opposes the second
inquest finding probable cause
that he committed rebellion and
that such inquest was void.
Issues:
1. WON the inquest for rebellion
against Beltran was valid?

Held:
The Inquest Proceeding against
Beltran for Rebellion is Void.
Inquest proceedings are proper
only when the accused has been
lawfully arrested without warrant.
The joint affidavit of Beltrans
arresting officers states that the
officers arrested Beltran, without a
warrant, for Inciting to Sedition,
and not for Rebellion. Thus, the
inquest prosecutor could only have
conducted as he did conduct an
inquest for Inciting to Sedition and
no other. Consequently, when
another group of prosecutors
subjected Beltran to a second
inquest proceeding for Rebellion,
they overstepped their authority
rendering the second inquest void.
None of Beltrans arresting officers
saw Beltran commit, in their
presence, the crime of Rebellion.
Nor did they have personal
knowledge of facts and
circumstances that Beltran had
just committed Rebellion,
sufficient to form probable cause
to believe that he had committed
Rebellion. What these arresting
officers alleged in their affidavit is
that they saw and heard Beltran
make an allegedly seditious speech
on 24 February 2006
1.

Mago v. Penalosa-Fermo, 582


SCRA 1
Facts:
Mr. Mago claims that on April 21,
2004 he filed a complaint for
Grave Coercion against DARAB
Sheriff Alex Roberto Angeles.
However, instead of summoning
the accused for a "Preliminary
Investigation", he received a
complaint charging him and his
two (2) sons with Grave Threats.
He stresses the complaint against
him as purely fabricated and that
the complainant in the said case
was not DARAB Sheriff Angeles.
He avers that the affidavits of the
witnesses in the said case could
not be found in the records of the
MTC. Complainant further
declares that on July 20, 2004, he
received a subpoena to attend the
preliminary investigation for the
charge of grave threat. In
compliance, he and his witnesses
attended, and even without the
assistance of counsel, they were
examined through a prepared set
of questions handed to them by the
stenographer. The respondent
judge was not present then. The
complainant also states that right
after the preliminary investigation,
he was immediately arrested and
was imprisoned for three (3) days.
Thereafter, he was released after
he posted bail in the amount of
Php12,000 pesos. The respondent
admitted that the court
stenographer examined
complainant and his witnesses
during the preliminary
investigation of the grave threats
complaint against him with the use
of prepared written set of

questions and explained that this


is convenient when more than one
preliminary examination is
scheduled for the day. This
procedure makes it easier for the
Stenographers and the witnesses,
too, considering the cramped
office space.
Issue:
WON respondent is guilty of gross
ignorance of the law.
Held:
Yes. Respondent admitted that
after the complaint was filed, she
prepared a set of questions based
on the affidavits of the complaining
witnesses and counter affidavits of
the accused. She further added
that during the preliminary
investigation and after briefing the
accused and his witnesses, the
stenographers took charge of the
proceedings. Hence, the
respondent judge violated the
rules on preliminary investigation.
Respondent should not have
allowed her stenographer to
handle the latter part of the
proceedings even if she only
wanted to expedite the
proceedings and it was more
convenient. Respondent judge
should have personally taken
charge of the entire proceedings
since the power to conduct
preliminary investigations vests
only on her and not on the
stenographer. Judges of municipal
trial courts were empowered to
conduct preliminary investigations
in which they exercised discretion
in determining whether there was
probable cause to hale the
respondent into court. Such being

the case, they could not delegate


the discretion to another. An
officer to whom a discretion is
entrusted cannot delegate it to
another, the presumption being
that he was chosen because he
was deemed fit and competent to
exercise that judgment and
discretion, and unless the power to
substitute another in his place has
been given to him, he cannot
delegate his duties to another.
Personal examination of the
complainant in a criminal case and
his witness/es was required.

Sibulo v. Toledo-Mupas, A.M. No.


MTJ-07-1686, June 12, 2008
Facts:
Complainant was the accused in a
case for Grave Threat and Slight
Physical Injuries, which are
pending before respondents court;
that on August 9, 2006,
respondent directed complainant
to submit his counter-affidavit
within ten (10) days from receipt
of the Order and set the case for
conference on October 11, 2006;
that as the parties failed to
amicably settle, the case was
submitted for resolution; and that
on October 25, 2006, respondent
set the case for arraignment after
finding probable cause to indict
complainant of the crimes
charged. Complainant asserted
that respondent, being a judge of a
first level court, no longer had
authority to conduct preliminary
investigation under Rules 112 and
114 of the Rules on Criminal
Procedure, as amended.
Issue:
WON respondent has the authority
to conduct preliminary
investigation
Held:
As correctly pointed out by
complainant, judges of first level
courts are no longer authorized to
conduct preliminary investigation.
This is pursuant to the amendment
made by this Court on August 30,
2005 in A.M. No. 05-8-26-SC Re:
Amendment of Rules 112 and 114
of the Revised Rules on Criminal
Procedure by Removing the

Conduct of Preliminary
Investigation from Judges of the
First Level Courts, which took
effect on October 3, 2005. Even so,
the determination of whether
respondent judge has authority to
conduct preliminary investigation
in the criminal cases filed against
complainant is not decisive in the
resolution of this administrative
case. As the OCA fittingly
observed, the Rules on Summary
Procedure govern the conduct of
the criminal proceedings. Hence,
the order of respondent for
complainant to submit his counteraffidavit is but proper. The
directive should not be taken as a
requirement of preliminary
investigation but one simply
intended to comply with the
provisions of the Rules that state
that the affidavits submitted by the
parties shall constitute the direct
testimonies of the witnesses who
executed the same and that failure
to submit the same would not
allow any witness to testify, except
by way of rebuttal or surrebuttal.

Bienvenido Dino and Renato


Comparativo v. Pablo Olivarez,
G.R. No. 170447, December 4,
2009
Facts:
Petitioners instituted a complaint
for vote buying against respondent
Pablo Olivarez. Based on the
finding of probable cause in the
Joint Resolution issued by
Assistant City Prosecutor
Antonietta Pablo-Medina, with the
approval of the city prosecutor of
Paraaque, two Informations were
filed before the RTC charging
respondent Pablo Olivarez with
Violation of Section 261,
paragraphs a, b and k of Article
XXII of the Omnibus Election
Code. Respondent filed before the
Law Department of the COMELEC
an appeal with Motion to Revoke
Continuing Authority pursuant to
Section 10, Rule 34 of the 1993
COMELEC Rules of Procedure.
Respondent argued that the
pendency of the appeal of the Joint
Resolution before the COMELEC
should prevent the filing of the
Informations before the RTC as
there could be no final finding of
probable cause until the
COMELEC had resolved the
appeal. Moreover, he argued that
the charges made against him
were groundless. The Law
Department of the COMELEC
directed the city prosecutor to
transmit or elevate the entire
records of the case and to suspend
further implementation of the Joint
Resolution dated 20 September
2004 until final resolution of the
said appeal before the COMELEC
en banc. Respondent then filed a

Motion to Quash on the ground


that more than one offense was
charged therein. Assistant
prosecutor filed its opposition and
filed its Motion to Admit Amended
Information with the approval of
the city prosecutor. Respondent
filed an Opposition to the
Admission of the Amended
Informations. He averred that the
city prosecutor was no longer
empowered to amend the
informations, since the COMELEC
had already directed it to transmit
the entire records of the case and
suspend the hearing of the cases
before the RTC until the resolution
of the appeal before the COMELEC
en banc. After setting the date of
arraignment, respondent failed to
appear before the RTC. Thereupon,
Judge Madrona, in open court,
denied the Motion for
Reconsideration of the Order
denying the Motion to Quash and
admitting the Amended
Informations, and ordered the
arrest of respondent and the
confiscation of the cash bond.
Issue:
WON the City Prosecutor abused
its authority when it did not
suspend the implementation of the
joint resolution until final
resolution by the COMELEC
Held:
Yes. The Constitution, particularly
Article IX, Section 20, empowers
the COMELEC to investigate and,
when appropriate, prosecute
election cases. Under Section 265
of the Omnibus Election Code, the
COMELEC, through its duly
authorized legal officers, has the

exclusive power to conduct a


preliminary investigation of all
election offenses punishable under
the Omnibus Election Code, and to
prosecute the same. The
COMELEC may avail itself of the
assistance of other prosecuting
arms of the government.
Furthermore, Section 10 of the
COMELEC Rules of Procedure
gives the COMELEC the power to
motu proprio revise, modify and
reverse the resolution of the Chief
State Prosecutor and/or
provincial/city prosecutors. From
the foregoing, it is clear that the
Chief State Prosecutor, all
Provincial and City Fiscals, and/or
their respective assistants have
been given continuing authority, as
deputies of the Commission, to
conduct a preliminary
investigation of complaints
involving election offenses under
the election laws and to prosecute
the same. Such authority may be
revoked or withdrawn anytime by
the COMELEC, either expressly or
impliedly, when in its judgment
such revocation or withdrawal is
necessary to protect the integrity
of the process to promote the
common good, or where it believes
that successful prosecution of the
case can be done by the
COMELEC. Moreover, being mere
deputies or agents of the
COMELEC, provincial or city
prosecutors deputized by it are
expected to act in accord with and
not contrary to or in derogation of
its resolutions, directives or orders
in relation to election cases that
such prosecutors are deputized to
investigate and prosecute. Being
mere deputies, provincial and city

prosecutors, acting on behalf of


the COMELEC, must proceed
within the lawful scope of their
delegated authority. When the
COMELEC Law Department
directed the City Prosecutor of
Paraaque to transmit the entire
records of the case to the Law
Department, Commission on
Elections, Intramuros, Manila, by
the fastest means available and to
suspend further implementation of
the questioned resolution until
final resolution of said appeal by
the Comelec En Banc, it had the
effect of SUSPENDING THE
AUTHORITY of the City Prosecutor
to prosecute the case. An appeal
has been filed before the
COMELEC and has yet to be
resolved. Since the COMELEC has
already taken cognizance of the
appeal, and the public prosecutor
has been directed to suspend
further implementation of the
questioned resolution until final
resolution of said appeal, it was
but proper for the City Prosecutor
of Paraaque to have held in
abeyance any action until after the
resolution of the appeal by the
COMELEC En Banc. This
suspension of delegated authority
was made permanent and this
delegated authority was revoked.
COMELEC Law Department has
the authority to direct, nay, order
the public prosecutor to suspend
further implementation of the
questioned resolution until final
resolution of said appeal, for it is
speaking on behalf of the
COMELEC.

Re: Subpeona Duces Tecum dated


January 11, 2010 of Acting
Director Aleu A. Amante, PIAB-C
Office of the Ombudsman, A.M.
No. 10-1-12-SC, March 2, 2010
Facts:
A criminal complaint was filed by
the complainants Lozano for the
alleged violation by retired
Supreme Court Chief Justice
Hilario Davide, Jr. and retired
Associate Justice Ma. Alicia
Austria-Martinez of Section 3(e) of
R.A. 3019, as amended (the AntiGraft and Corrupt Practices Act).
The criminal complaint stemmed
from the participation of the
accused in the Resolution the First
Division of this Court issued in
Heirs of Antonio Pael v. Court of
Appeals, docketed as G.R. Nos.
133547 and 133843. The retired
Chief Justice and retired Associate
Justice allegedly committed the
following unlawful acts:1)
Overturning the findings of fact of
the CA; 2) Stating in the
Resolution that the Chin-Mallari
property overlaps the UP property,
when the DENR Survey Report
stated that the UP title/property
overlaps the Chin-Mallari property;
3) Issuing a Resolution, for which
three Justices voted, to set aside a
Decision for which five Justices
voted. By these acts, the retired
Members of this Court are being
held criminally accountable on the
theory that they violated the
Constitution and the law in their
ruling in the cited cases, thereby
causing undue injury to the parties
to these cases. The Office of the
Ombudsman issued a subpoena
duces tecum on the Chief, Office of

the Administrative Services or


AUTHORIZED REPRESENTATIVE,
Supreme Court, Manila, for the
submission to the Office of the
Ombudsman of the latest Personal
Data Sheets and last known
forwarding address of former
Chief Justice Hilario G. Davide, Jr.
and former Associate Justice Ma.
Alicia Austria-Martinez. The
subpoena duces tecum was issued
in relation to a criminal complaint
under above stated, pursuant to
Section 13, Article XI of the
Constitution and Section 15 of
Republic Act No. 6770. Thereafter,
Ombudsman Simeon Marcelo
directed that all complaints
against judges and other members
of the Judiciary be immediately
dismissed and referred to the
Supreme Court for appropriate
action
Issue:
Was the issuance of the subpoena
duces tecum by the Ombudsman
proper
Held:
The issue has become academic
due to the dismissal of the
complaint. However, the Supreme
Court discussed that in the
appropriate case, the Office of the
Ombudsman has full authority to
issue subpoenas, including
subpoena duces tecum, for
compulsory attendance of
witnesses and the production of
documents and information
relating to matters under its
investigation. The grant of this
authority, however, is not
unlimited, as the Ombudsman
must necessarily observe and

abide by the terms of the


Constitution and our laws, the
Rules of Court and the applicable
jurisprudence on the issuance,
service, validity and efficacy of
subpoenas. Under the Rules of
Court, the issuance of subpoenas,
including a subpoena duces tecum,
operates under the requirements
of reasonableness and relevance.
For the production of documents to
be reasonable and for the
documents themselves to be
relevant, the matter under inquiry
should, in the first place, be one
that the Ombudsman can
legitimately entertain, investigate
and rule upon.
Santos-Concio v. Department of
Justice, 543 SCRA 70
Facts:
In the days leading to February 4,
2006, people started to gather in
throngs at the Philsports Arena in
Pasig City, the publicized site of
the first anniversary episode of
Wowowee, a noontime game show
aired by ABS-CBN Broadcasting
Corporation (ABS-CBN). With high
hopes of winning the bonanza,
hundreds queued for days and
nights near the venue to assure
themselves of securing tickets for
the show. Hours before the show
and minutes after the people were
allowed entry through two entry
points at six oclock in the morning,
the obstinate crowd along Capt.
Javier Street jostled even more just
to get close to the lower rate
pedestrian gate. The mad rush of
the unruly mob generated much
force, triggering the horde to
surge forward with such

momentum that led others to


stumble and get trampled upon by
the approaching waves of people
right after the gate opened. This
fatal stampede claimed 71 lives, 69
of whom were women, and left
hundreds wounded which
necessitated emergency medical
support and prompted the
cancellation of the shows episode
DILG through then Secretary
Angelo Reyes, immediately created
an inter-agency fact-finding team
to investigate the circumstances
surrounding the stampede. DOJ
Secretary Raul Gonzalez
constituted a Panel to evaluate the
DILG Report and determine
whether there is sufficient basis to
proceed with the conduct of a
preliminary investigation on the
basis of the documents submitted.
The Evaluating Panel later
submitted to Gonzalez a report
concurring with the DILG Report
but concluding that there was no
sufficient basis to proceed with the
conduct of a preliminary
investigation. NBI on the other
hand recommend the conduct of
preliminary investigation for
Reckless Imprudence resulting in
Multiple Homicide and Multiple
Physical Injuries. Gozales then
made an Investigating Panel to
conduct preliminary investigation,
file the information and prosecute
the same. The petitioners alleged
that the case was already
prejudiced and lost impartiality to
conduct preliminary investigation
and that the complaint affidavits
were not under oath. These
affidavits, petitioners further point
out, nonetheless do not qualify as a
complaint within the scope of Rule

110 of the Rules of Court as the


allegations therein are insufficient
to initiate a preliminary
investigation, there being no
statement of specific and
individual acts or omissions
constituting reckless imprudence.
Issue:
WON the contention of the
petitioner was right
Held:
Complaint for purposes of
conducting a preliminary
investigation differs from a
complaint for purposes of
instituting a criminal prosecution.
Confusion apparently springs
because two complementary
procedures adopt the usage. There
should be no confusion because
preliminary investigation is
conducted precisely to elicit
further facts or evidence. Being
generally inquisitorial, the
preliminary investigation stage is
often the only means of
discovering the persons who may
be reasonably charged with a
crime, to enable the preparation of
a complaint or information. Rule
112 of Revised Rules on Criminal
Procedure recognizes that all
necessary allegations need not be
contained in a single document
unlike a criminal complaint or
information where the averments
must be contained in one
document charging only one
offense, non-compliance with
which renders it vulnerable to a
motion to quash. A complaint for
purposes of conducting
preliminary investigation is not
required to exhibit the attending

structure of a complaint or
information laid down in Rule 110
(Prosecution of Offenses) which
already speaks of the People of the
Philippines as a party, an accused
rather than a respondent, and a
court that shall pronounce
judgment. If a complaint or
information filed in court does not
comply with a set of constitutive
averments, it is vulnerable to a
motion to quash.

De Ocampo v. Secretary of Justice,


480 SCRA 71
Facts:
The case arose from a sworn
statement of respondent Madalena
B. Dacarra stating that her ninyear-old son Ronald complained of
dizziness upon arriving home at
about six in the evening. Ronald
told her that petitioner, who was
Ronalds teacher, banged his head
against that of his classmate
Lorendo Orayan (Lorendo).
Magdalena inspected Ronalds
head and saw a woundless
contusion. Magdalena brought him
to a quack doctor (arbularyo). The
following morning, Magdalena
brought Ronald to the East Avenue
Medical Center where he
underwent an x-ray. The attending
physician informed Magdalena
that Ronalds head had a fracture.
Blood oozed out of Ronalds nose
before he died. Lorendo also
executed a sworn statement
narrating how petitioner banged
his head against Ronalds. An
inquest proceeding took place and
it was ruled that the case is not
proper for inquest and released
the respondent for further
investigation. A preliminary
investigation was then conducted.
Jennilyn Quirong, who witnessed
the head-banging incident, and
Melanie Lugales, who claimed to
be another victim of petitioners
alleged cruel deeds, filed their
sworn statements with the Office
of the Quezon City Prosecutor.
Petitioner invoked the disposition
of the inquest prosecutor finding
insufficient evidence to support
the charges against her. Petitioner

assailed the omission in


Magdalenas sworn statement
about Ronalds head injury due to a
vehicular accident in November
1997. Petitioner pointed out the
absence of damage or injury on
Lorendo as borne out by his
medical certificate. Petitioner
contended that the head-banging
incident was not the proximate
cause of Ronalds death, but the
failed medical attention or medical
negligence. Petitioner also alleged
that Jennilyn Quirong and Melanie
Lugales have immature
perception. Petitioner further
asserted that the causes of death
stated in Ronalds Death Certificate
are hearsay and inadmissible in
the preliminary investigation. The
investigating prosecutor issued a
resolution finding probable cause.
The same was filed for a petition
for review to the DOJ but was also
denied. Petitioner contended that
the investigating prosecutor
showed bias in favor of
complainants Magdalena and
Erlinda (complainants) for not
conducting a clarificatory hearing
and unilaterally procuring the
autopsy report.
Issue:
WON there was bias in
complainants favour when the
investigating prosecutor did not
conduct clarificatory hearing
Held:
The Court rejects petitioners
contention that she was denied
due process when the
investigating prosecutor did not
conduct a clarificatory hearing. A
clarificatory hearing is not

indispensable during preliminary


investigation. Rather than being
mandatory, a clarificatory hearing
is optional on the part of the
investigating officer as evidenced
by the use of the term may in
Section 3(e) of Rule 112. The use
of the word may in a statute
commonly denotes that it is
directory in nature. The term may
is generally permissive only and
operates to confer
discretion.Under Section 3(e) of
Rule 112, it is within the discretion
of the investigation officer whether
to set the case for further hearings
to clarify some matters.
In this case, the investigating
prosecutor no longer conducted
hearings after petitioner submitted
her counter-affidavit. This simply
means that at that point the
investigating prosecutor believed
that there were no more matters
for clarification. It is only in
petitioners mind that some crucial
points still exist and need
clarification. In any event,
petitioner can raise these
important matters during the trial
proper.
Petitioner was not deprived of due
process since both parties were
accorded equal rights in arguing
their case and presenting their
respective evidence during the
preliminary investigation. Due
process is merely an opportunity to
be heard. Petitioner cannot
successfully invoke denial of due
process since she was given the
opportunity of a hearing. She even

submitted her counter-affidavit to


the investigating prosecutor on 18
January 2000.

Dupasquier v. Court of Appeals,


G.R. No. 112089, January 24, 2001
Facts:
In 1987, Carlota P. Valenzuela,
deputy governor of the Central
Bank of the Philippines and
receiver/liquidator of Banco
Filipino Savings and Mortgage
Bank, filed with the Department of
Justice a complaint for estafa
against the following Banco
Filipino officials in connection with
irregular grants of commercial
loans to corporate subsidiaries of
Banco Filipino, namely: Anthony C.
Aguirre, Tomas B. Aguirre,
Teodoro C. Arcenas, Fortunato M.
Dizon, Jr., Alberto C. Aguirre,
Delfin M. Dimagiba, Napoleon L.
Buencamino, Enrique M. Zalamea,
Jr., Eugenio A. Osias, Ramon
Henares, Benjamin E. Elizaga,
Martin L. Calicutan, Eduardo V.
Tacolod, Eduardo F. Quirino,
Cynthia Subijano, Solita M.
Manalaysay, Hautila D. Jose,
Remedios Dupasquier, Nancy L. Ty
and Elena Pallasique. After
conducting preliminary
investigation, Rizal 2nd Assistant
Provincial Prosecutor Herminio T.
Ubana, Sr. issued two (2)
resolutions recommending the
filing of information for estafa
against the above-named officials,
except for Nancy L. Ty and Elena
Pallasique. With the approval of
the Provincial Prosecutor of Rizal,
informations were thus filed
against said officials with the RTC.
Some of the accused-officials
moved for a reconsideration or
reinvestigation alleging serious
irregularities during the
preliminary investigation, to wit:

(a) parties not originally charged


were found indictable in said
resolutions and (b) of many who
were not included in the
resolutions were charged in the
informations filed in court. Rizal
Provincial Prosecutor Mauro M.
Castro issued a resolution granting
a reinvestigation. A panel of
investigators composed of 2nd
Assistant Prosecutor Edwin
Condaya, 3rd Assistant Prosecutor
Domingo Allena and 4th Assistant
Prosecutor Eduardo Bautista
prepared a memorandum for
Provincial Prosecutor Mauro M.
Castro recommending the
dismissal of the charges against
the accused for lack of probable
cause. Provincial Prosecutor
Castro reversed the investigating
panels recommendation and
ordered the prosecution of
petitioners and respondent
Fortunato Dizon, Jr. Petitioners
filed a motion for reconsideration
while Fortunato M. Dizon, Jr. with
others filed with the Office of the
Secretary of Justice a petition for
review. Acting Secretary of Justice
Silvestre H. Bello dismissed
respondent Dizons petition for
review. And later, Secretary of
Justice Eduardo G. Montenegro
denied respondent Dizons motion
for reconsideration. Dizon then
filed with the Court of Appeals a
petition for certiorari, prohibition
and mandamus to annul the
resolution of Acting Secretary of
Justice Eduardo G. Montenegro,
and to order the dismissal of all
the criminal cases against him
before the Makati Regional Trial
Courts. Provincial Prosecutor
Mauro M. Castro denied the

motion for reconsideration of the


petitioners. Petitioners appealed to
the Secretary of Justice but it was
dismissed and denied the motion
for reconsideration they filed.
Petitioners then filed with the
Court of Appeals a petition for
certiorari, prohibition and
mandamus.
Issue:
WON the court may review
findings of the prosecutor on the
existence of probable cause
Held:
The determination of probable
cause during a preliminary
investigation or reinvestigation is
recognized as an executive
function exclusively of the
prosecutor. An investigating
prosecutor is under no obligation
to file a criminal action where he is
not convinced that he has the
quantum of evidence at hand to
support the averments.
Prosecuting officers have equally
the duty not to prosecute when
after investigation or
reinvestigation they are convinced
that the evidence adduced was not
sufficient to establish a prima facie
case. Thus, the determination of
the persons to be prosecuted rests
primarily with the prosecutor who
is vested with discretion in the
discharge of this function.
Consequently, the fact that the
investigating prosecutor
exonerated some of the co-accused
in the preliminary investigation
does not necessarily entitle
petitioners to a similar exoneration
where the investigating prosecutor
found probable cause to prosecute

them for the crime charged.


Courts should give credence, in
the absence of a clear showing of
arbitrariness, to the findings and
determination of probable cause
by prosecutors in a preliminary
investigation. I our criminal justice
system, the public prosecutor has
the quasi-judicial discretion to
determine whether or not a
criminal case should be filed in
court. Courts must respect the
exercise of such discretion when
the information filed against the
accused is valid on its face, and no
manifest error, grave abuse of
discretion or prejudice can be
imputed to the public prosecutor.

People v. Marquez, 27 SCRA 808


Facts:
On November 12, 1962, a
complaint for frustrated homicide
was filed with the then Justice of
the Peace Court of Camalig, Albay,
against appellee Vicente Marquez.
The said complaint was signed by
one Consolacion Musa Solano,
mother of the offended party,
Wenceslao Solano, who was then
confined in the Albay Provincial
Hospital, in consequence of the
crime charged. A warrant of arrest
was issued, but this was not
carried out because the accused
had filed a bail bond. When the
case was later called for purposes
of conducting the second stage of
the preliminary investigation, the
accused, thru counsel, waived his
right thereto. Marquez entered a
plea of not guilty to the charge. On
the day of the trial on September
25, 1964, without asking for leave
to withdraw his previously-entered
plea, appellee filed a motion to
dismiss on the ground that the
information filed by the Provincial
Fiscal of Albay based on the
complaint signed by Consolacion
Musa Solano in behalf of her
victim-son or offended party was
null and void and the court had no
jurisdiction to hear, try and decide
the case. The court dismissed the
case, with costs de oficio, and
ordered the cancellation of the bail
bond of appellee. The Assistant
Provincial Fiscal concerned has
appealed from the dismissal
aforesaid directly to this Court.
Issue:

WON the respondents contention


is correct
Held:
It must be remembered that
appellee did not attack the said
complaint while his case was still
in the justice of the peace court,
where, on the contrary, he waived
the preliminary investigation
proper; he allowed the case to be
remanded to the court of first
instance and folded his arms when
the provincial fiscal filed the
corresponding information; and,
he did not object to his being
arraigned, instead he merely
entered a plea of not guilty at said
arraignment. In this
circumstances, We hold that the
initial complaint has lost his
importance and the case can be
viewed only in the light of the
information subsequently filed by
the provincial fiscal, as suggested
by the Solicitor General.
Therefore, the question to
determine in this case is, what was
the effect of appellee's failure to
object to the information before or
at the time he entered his plea of
not guilty. Assuming that said
information was defective because
it did not contain the requisite
certification regarding the fiscal's
having held a preliminary
investigation where the accused
was given an opportunity to be
present personally or thru counsel,
such an omission is not necessarily
fatal. It should be observed that
section 3 of Rule 110 defines an
information as nothing more than
"an accusation in writing charging
a person with an offense
subscribed by the fiscal and filed

with the court." Thus, it is obvious


that such certification is not an
essential part of the information
itself and its absence cannot
vitiate it as such. True, as already
stated, section 14 of Rule 112
enjoin that "no information ... shall
be filed, without first giving the
accused a chance to be heard in a
preliminary investigation," but, as
can be seen, the injunction refers
to the non-holding of the
preliminary investigation, not the
absence of the certification. In
other words, what is not allowed
is, the filing of the information
without a preliminary investigation
having been previously conducted,
and the injunction that there
should be a certification is only a
consequence of the requirement
that a preliminary investigation
should first be conducted.
Logically, therefore, inasmuch as
the settled doctrine in this
jurisdiction is that the right to the
preliminary investigation petition
itself must be asserted or invoked
before the plea, otherwise, it is
deemed waived, 3 it stands to
reason, that the absence of the
certification in question is also
waived by failure to allege it
before the plea. After all, such
certification is nothing but
evidence of a fact and if the
ommision of the fact itself to be
certified is waived, if not properly
raised before the accused enters
his plea, why should the omission
merely of the certification be given
more importance than the absence
of the fact itself to be certified to?
Is it to be sustained that if in a
given case, there were such a
certification although in fact no

preliminary investigation has been


held, this Court is going to hold
that the requirement of a
preliminary investigation has been
complied with? To ask the question
is to immediately expose the
absurdity of the affirmative answer
to it.

Community Rural Bank of Guimba


(N.E.), Inc. v. Talavera, 455 SCRA
34
Facts:
In September 1997, the Bank
lodged a complaint with the City
Prosecutors Office of Cabanatuan
charging several persons with the
offense of Estafa. After a
preliminary investigation, the
Investigating Fiscal recommended
the filing --of six (6) Informations
for Estafa against the accused.
Respondent was the presiding
judge of Branch 28 to whom two
informations were raffled. The
accused appealed the findings of
the Investigating Fiscal to the
Department of Justice but the
same was denied. Then, the
accused filed a Motion for
Reconsideration, which was denied
again by the DOJ. Hence,
respondent issued a Warrant of
Arrest fixing no bail against the
accused. On 20 November 2000,
the accused filed a Motion for
Reinvestigation and to Lift the
Issuance of Warrant of Arrest
which was also granted without
hearing and notice afforded to the
Bank nor its counsel. It was
conducted by the Assistant
Provincial Prosecutor. Thus, a
reinvestigation proceeding was
conducted and reversed the earlier
findings of the previous
Investigating Fiscal. Thus, on the
same day, a Motion to Dismiss was
filed by Assistant Provincial
Prosecutor Caballero. Neither the
bank nor its counsel was notified
about the said Motion and no
hearing thereon was held to afford
the Bank an opportunity to oppose

the same. The Bank, arguing that


it had been deprived of due
process, filed a Motion for
Reconsideration with
Opposition/Comment to the Motion
to Dismiss and Omnibus Motion for
the Reinstatement of the Criminal
Information and for the Recall of
Order for Release which was
denied. The Bank filed a Petition
for Review under Rule 65 of the
Revised Rules of Court with the
Court of Appeals.
Issue:
Was there a grave ignorance of the
law on the part of the judge
Held:
Dismissing the criminal case
without giving complainant the
opportunity to object to the Motion
for Reinvestigation and Motion to
Dismiss, respondent showed gross
ignorance of the law, for which he
should be sanctioned. In the
present case, the gross ignorance
of respondent judge and his
notorious violation of simple legal
precepts were clearly shown by his
granting the Motion for
Reinvestigation of the accused and
the prosecutors Motion to Dismiss.
First, respondent should not have
entertained the Motion for
Reinvestigation filed by the
accused. The former was fully
aware that the latter had appealed
the unfavorable ruling of the
investigating prosecutor to the
DOJ. Respondent judge must have
in fact taken that appeal into
consideration when he issued a
warrant of arrest against all the
accused only on September 19,
2000, after Justice Secretary

Serafin R. Cuevas had denied their


Petition for Review and affirmed
the presence of prima facie
evidence against them.
Subsequently, on August 15, 2000,
the secretary also denied with
finality the Motion for
Reconsideration filed by the
accused. Inasmuch as the
Resolution of the provincial
prosecutor has been affirmed by
the secretary of justice, the
existence of probable cause to hold
the accused for trial may be
deemed to be the finding of the
secretary himself, not merely of
the prosecutor who had first
investigated the case. Therefore,
what the prosecutor reviewed and
overruled in the reinvestigation
was not the actuation and
resolution of his predecessor, but
of the secretary of justice no less.
The actions of prosecutors are not
unlimited; they are subject to
review by the secretary of justice
who may affirm, nullify, reverse or
modify their actions or opinions.
Consequently the secretary may
direct them to file either a motion
to dismiss the case or an
information against the accused.
In short, the secretary of justice,
who has the power of supervision
and control over prosecuting
officers, is the ultimate authority
who decides which of the
conflicting theories of the
complainants and the respondents
should be believed. The provincial
or city prosecutor has neither the
personality nor the legal authority
to review or overrule the decision
of the secretary. This principle is

elementary. From the abovequoted provision, a motion for


reinvestigation on the ground of
newly discovered evidence must be
filed before the justice secretary
rules on an appeal from a
resolution in a preliminary
investigation.
In the present case, the accused
filed their Motion for
Reinvestigation on November 29,
2000, about three months after the
August 15, 2000 Resolution of the
secretary denying with finality
their Motion for Reconsideration of
the denial of their Petition for
Review. Clearly, therefore, it was
grossly erroneous for respondent
judge to order the reinvestigation
of the case by the prosecutor. This
action enabled the latter to
reprobate and reverse the
secretarys Resolution. In granting
the Motion for Reinvestigation,
respondent effectively demolished
the DOJs power of control and
supervision over prosecutors.

Asetre v. Asetre, G.R. No. 171536,


April 7, 2009
Facts:
On December 27, 2000, Hanz
Dietrich Asetre was found dead in
his residence, which also housed
his printing press business.
Petitioner April Joy GonzagaAsetre, Hanzs wife, alleged that
her husband committed suicide by
hanging himself using bedcovers.
The Office of the City Prosecutor of
Bacolod found probable cause
against April, Hanzs first cousins
Galinzchel and Buenaventura
Gamboa, and printing press
worker Benjie Ebcas. The
investigating prosecutor held that
from the evidence adduced by the
parties, herein petitioners were
physically and actively interacting
with Hanz shortly before he was
found dead. Moreover, from the
actuations of petitioners and the
events that took place, it can be
gleaned that they connived in
killing Hanz and later tried to
cover up the crime. Further, the
prosecutor rejected petitioners
"suicide theory" because it is
inconsistent with the medico-legal
findings that while Hanz might
have wanted to end his life, the
circumstances of his death proved
he could not have done it himself.
The four accused asked the DOJ
for a review of the prosecutors
findings. DOJ Acting Secretary Ma.
Merceditas N. Gutierrez absolved
petitioners and reversed the
investigating prosecutors
resolution, not because she
believed the "suicide theory" of the
petitioners, but rather because she
did not find sufficient evidence to

sustain the theory of the


prosecution of "conspiracy to
commit murder." Secretary
Gutierrez explained that while
there is overwhelming proof that
Hanz might not have committed
suicide, there is no direct or
circumstantial evidence that could
link petitioners as the authors of
the crime. The DOJ denied the
Asetre siblings motion for
reconsideration of the Secretarys
Order. Thereafter, respondent
Asetres filed a petition for
certiorari and mandamus before
the Court of Appeals, arguing that
the DOJ Secretary acted with
grave abuse of discretion in
issuing the December 17, 2002
Resolution despite the
circumstantial evidence against
petitioners. The appellate court
found that the DOJ Secretary
committed grave abuse of
discretion amounting to lack or
excess of jurisdiction in reversing
the investigating prosecutors
finding of probable cause
Issue:
WON the Court of Appeals erred in
reversing the ruling of the DOJ
Secretary and in finding probable
cause to indict petitioners for
murder and parricide
Held:
The full discretionary authority to
determine probable cause in a
preliminary investigation to
ascertain sufficient ground for the
filing of information rests with the
executive branch. Hence, judicial
review of the resolution of the
Secretary of Justice is limited to a
determination whether there has

been a grave abuse of discretion


amounting to lack or excess of
jurisdiction. Courts cannot
substitute the executive branchs
judgment. As department head,
the Secretary of Justice has the
power to alter, modify, nullify or
set aside what a subordinate
officer had done in the
performance of his duties and to
substitute the judgment of the
former for that of the latter. While
it is the duty of the fiscal to
prosecute persons who, according
to evidence received from the
complainant, are shown to be
guilty of a crime, the Secretary of
Justice is likewise bound by his
oath of office to protect innocent
persons from groundless, false or
serious prosecutions. He would be
committing a serious dereliction of
duty if he orders or sanctions the
filing of charge sheets based on
complaints where he is not
convinced that the evidence would
warrant the filing of an action in
court. He has the ultimate power
to decide which as between the
conflicting theories of the parties
should be believed. The Secretary
is empowered to order or perform
the very acts questioned in this
case. It is only where the decision
of the Justice Secretary is tainted
with grave abuse of discretion
amounting to lack or excess of
jurisdiction that the Court of
Appeals may take cognizance of
the case in a petition for certiorari
under Rule 65 of the Revised Rules
of Civil Procedure. The Court of
Appeals decision may then be
appealed to this Court by way of a
petition for review on certiorari.

In this case, however, the


Secretary of Justice committed no
grave abuse of discretion. Based
on the totality of the evidence
presented by both parties, it is
clear that there is a dearth of proof
to hold petitioners for trial. All
circumstances considered, the
Court find that the DOJ Secretary
correctly held that the
circumstantial evidence presented
by private respondents to prove
probable cause against petitioners,
does not support the theory of
conspiracy to commit murder.
Such circumstantial evidence
would not sufficiently warrant a
conclusion that private
respondents are responsible for
the death of Hanz. Petitioners
mere presence at the death scene
does not suffice to establish
probable cause against them. It is
noteworthy that complainants
failed to establish conclusively that
April, Hanzs cousins, and his
workers had an ax to grind against
Hanz. The alleged quarrel of the
couple the night before the
incident is hearsay and could not
establish enough credible motive
on the part of April, contrary to the
opinion of the investigating
prosecutor, because the same
witness who testified about the
alleged fight also stated that the
couple had a good relationship and
that it was not unusual for the
couple to have verbal altercations
occasionally. Equally worth
stressing is the positive proof that
the accused were not the only
persons present inside the couples
house; and that the door of the
gate of the house, including the
door of the room where the victim

was found hanging, were not so


well secured as to exclude the
possibility that the act was
committed by other persons who
were also then present in the
house, or even by intruders. April
was not attempting to reduce the
number of possible witnesses as
stated by the investigating
prosecutor when she sent her
children to Iloilo as it was the
victims decision to send their
children to Iloilo upon his cousins
invitation. Likewise, concerning
the act of burning the bedsheets,
the Court find no grave abuse of
discretion in the ruling of the DOJ
that an ordinary person like April
could have believed that the police
investigation made at the death
scene and the post-mortem
examination conducted on the
body of the victim were already
more than enough to conclude and
close the investigation. Thus, no
grave abuse of discretion on the
part of the Secretary of Justice.

Alcaraz v. Gonzales, G.R. No.


164715, September 20, 2006
Facts:
At around 10:05 a.m. of August 11,
2000, 61-year-old Ramon C.
Gonzalez was driving his Nissan
Cefiro car along the right
outermost lane of the South-Luzon
Expressway. He was on his way to
Makati City and had just passed
the Sucat toll gate.Atty. Arnel C.
Alcaraz, a Customs Collector of the
Bureau of Customs, Batangas Port,
was driving his Nissan Infiniti car.
He was in the middle lane of the
South-Luzon Expressway, between
the Sucat and Bicutan
Interchange, on his way to Manila
from Batangas City. He was armed
with a .38 caliber pistol and had
with him Mission Order No. 6992000, to expire on August 21,
2000. Since Alcaraz intended to
use the Skyway, he signaled, and
proceeded to the right-most lane
which was reserved for vehicles
taking the Skyway. Gonzalez, who
was on the right-most lane, was
forced to swerve his car to the
right to avoid colliding with
Alcarazs vehicle and nearly hit the
concrete island. Nonplussed,
Gonzalez chased after Alcaraz,
opened his windows and shouted
at Alcaraz, demanding to know
why the latter suddenly cut into
his lane. Alcaraz retorted that he
had signaled that he was swerving
to the right. Gonzalez reproved
Alcaraz and drove on. Alcaraz
drove his car to Gonzalezs right.
Upon nearing an island, Alcaraz
raised his pistol towards Gonzalez
and fired twice: the first bullet hit
the right front window of the

vehicle and exited at the left rear


door; the second bullet hit the left
rear window of Gonzalezs car.
Alcaraz hurriedly drove away from
the scene, but was intercepted by
the PNCC guards at the Skyway
toll gate. The guards confiscated
from Alcaraz the .38 pistol with 7
live bullets and 3 empty shells.
Officer-in-Charge of the Paraaque
Police Station, filed a criminal
complaint for attempted homicide
against Alcaraz in the Office of the
City Prosecutor of Paraaque City.
Investigating Prosecutor resolved
to maintain his finding of probable
cause of attempted homicide
against Alcaraz and to retain the
Information. Alcaraz filed a motion
for reconsideration, and when it
was denied, filed a petition for
review with the City Prosecutors
Office, Department of Justice. Then
Secretary of Justice Hernando
Perez issued a Resolution granting
the petition and ordering the City
Prosecutor to withdraw the
Information. According to the
Justice Secretary, Gonzalez failed
to prove beyond reasonable that
Alcaraz had intended to kill him.
Gonzalez filed a motion for
reconsideration, which the
Undersecretary of Justice denied.
He then filed a petition for review
under Rule 43 of the 1997 Rules of
Civil Procedure before the CA,
seeking the reversal of the Justice
Secretarys Resolution. He claimed
that the Secretary acted beyond
his authority in finding no probable
cause to charge Alcaraz with
attempted homicide and for
ordering the City Prosecutor to
withdraw the Information. He
insisted that by invoking self-

defense, Alcaraz thereby admitted


his intention to kill him (Gonzalez).
He claimed that Alcarazs claim of
self-defense should be ventilated
during trial on the merits.
Issue:
WON the petition for review under
Rule 43 of the Rules of Court was
the proper remedy of respondent
Held:
It bears stressing that in the
determination of probable cause
during the preliminary
investigation, the executive branch
of government has full
discretionary authority. Thus, the
decision whether or not to dismiss
the criminal complaint against the
private respondent is necessarily
dependent on the sound discretion
of the Investigating Prosecutor and
ultimately, that of the Secretary of
Justice. Courts are not empowered
to substitute their own judgment
for that of the executive branch.
The resolution of the Investigating
Prosecutor is subject to appeal to
the Justice Secretary who, under
the Revised Administrative Code,
exercises the power of control and
supervision over said Investigating
Prosecutor; and who may affirm,
nullify, reverse, or modify the
ruling of such prosecutor.Thus,
while the CA may review the
resolution of the Justice Secretary,
it may do so only in a petition for
certiorari under Rule 65 of the
Rules of Court, solely on the
ground that the Secretary of
Justice committed grave abuse of
his discretion amounting to excess
or lack of jurisdiction. It bears
stressing that the Resolution of the

Justice Secretary affirming,


modifying or reversing the
resolution of the Investigating
Prosecutor is final. Under the 1993
Revised Rules on Appeals (now the
2000 National Prosecution Service
Rules on Appeals), resolutions in
preliminary investigations or
reinvestigations from the Justice
Secretarys resolution, except the
aggrieved party, has no more
remedy of appeal to file a motion
for reconsideration of the said
resolution of such motion if it is
denied by the said Secretary. The
remedy of the aggrieved party is to
file a petition for certiorari under
Rule 65 of the Rules of Court since
there is no more appeal or other
remedy available in the ordinary
course of law.

Sy Thiong Shiou vs Sy Chim, G.R.


No. 174168, March 30, 2009
Facts:
On 30 May 2003, four criminal
complaints were filed by Sy Chim
and Felicidad Chan Sy (Spouses
Sy) against Sy Tiong Shiou, Juanita
Tan Sy, Jolie Ross Tan, Romer Tan,
Charlie Tan and Jessie James Tan
(Sy Tiong Shiou, et al.) before the
City Prosecutors Office of Manila.
The cases were later consolidated.
The Spouses Sy averred that they
are stockholders and directors of
Sy Siy Ho & Sons, Inc. who asked
Sy Tiong Shiou, et al., officers of
the corporation, to allow them to
inspect the books and records of
the business on three occasions to
no avail. Sy Tiong Shiou, et al.
denied the request, citing civil and
intra-corporate cases pending in
court. In the two other complaints,
Sy Tiong Shiou was charged with
falsification under Article 172, in
relation to Article 171 of the RPC
and perjury under Article 183 of
the RPC. According to the Spouses
Sy, Sy Tiong Shiou executed under
oath the 2003 General Information
Sheet (GIS) wherein he falsely
stated that the shareholdings of
the Spouses Sy had decreased
despite the fact that they had not
executed any conveyance of their
shares. Sy Tiong Shiou, et al.
argued before the prosecutor that
the issues involved in the civil case
for accounting and damages
pending before the RTC of Manila
were intimately related to the two
criminal complaints filed by the
Spouses Sy against them, and thus
constituted a prejudicial question
that should require the suspension

of the criminal complaints. They


also argued that the Spouses Sys
request for inspection was
premature as the latters concern
may be properly addressed once
an answer is filed in the civil case.
Sy Tiong Shiou, on the other hand,
denied the accusations against
him, alleging that before the 2003
GIS was submitted to the
Securities and Exchange
Commission (SEC), the same was
shown to respondents, who at that
time were the President/Chairman
of the Board and Assistant
Treasurer of the corporation, and
that they did not object to the
entries in the GIS. The
investigating prosecutor issued a
resolution recommending the
suspension of the criminal
complaints for violation of the
Corporation Code and the
dismissal of the criminal
complaints for falsification and
perjury against Sy Tiong Shiou.
The reviewing prosecutor
approved the resolution. The
Spouses Sy moved for the
reconsideration of the resolution,
but their motion was denied. The
Spouses Sy thereupon filed a
petition for review with the DOJ
which the latter denied. Their
subsequent motion for
reconsideration was likewise
denied. The Spouses Sy elevated
the DOJs resolutions to the Court
of Appeals through a petition for
certiorari, imputing grave abuse of
discretion on the part of the DOJ.
The appellate court granted the
petition and directed the City
Prosecutors Office to file the
appropriate informations against
Sy Tiong Shiou, et al. for violation

of Section 74, in relation to


Section 144 of the Corporation
Code and of Articles 172 and 183
of the RPC. Sy Tiong Shiou, et al.
argue that findings of the DOJ in
affirming, modifying or reversing
the recommendations of the public
prosecutor cannot be the subject
of certiorari or review of the Court
of Appeals because the DOJ is not
a quasi-judicial body within the
purview of Section 1, Rule 65 of
the Rules of Court
Issue:
WON the petitioners contention is
correct
Held:
Indeed, a preliminary proceeding
is not a quasi-judicial function and
that the DOJ is not a quasi-judicial
agency exercising a quasi-judicial
function when it reviews the
findings of a public prosecutor
regarding the presence of
probable cause.Moreover, it is
settled that the preliminary
investigation proper, i.e., the
determination of whether there is
reasonable ground to believe that
the accused is guilty of the offense
charged and should be subjected
to the expense, rigors and
embarrassment of trial, is the
function of the prosecution. This
Court has adopted a policy of noninterference in the conduct of
preliminary investigations and
leaves to the investigating
prosecutor sufficient latitude of
discretion in the determination of
what constitutes sufficient
evidence as will establish probable
cause for the filing of information
against the supposed offender. As

in every rule, however, there are


settled exceptions. Hence, the
principle of non-interference does
not apply when there is grave
abuse of discretion which would
authorize the aggrieved person to
file a petition for certiorari and
prohibition under Rule 65, 1997
Rules of Civil Procedure. As
correctly found by the Court of
Appeals, the DOJ gravely abused
its discretion when it suspended
the hearing of the charges for
violation of the Corporation Code
on the ground of prejudicial
question and when it dismissed the
criminal complaints.

Angeles v. Gaite, G.R. No. 176596,


March 23, 2011
Facts:
Petitioner was charged of child
abuse by her grandniece Maria
Mercedes Vistan. The preliminary
investigation of the complaint was
assigned to State Prosecutor
Emmanuel Y. Velasco of the DOJ.
Velasco filed a case against
petitioner for 21 counts of Child
Abuse under Republic Act (RA) No.
7610. Petitioner filed a petition for
review with the DOJ Secretary who
ordered the withdrawal of the
Information against petitioner. On
July 7, 2000, petitioner filed with
the DOJ an administrative
complaint for Gross Misconduct,
Gross Ignorance of the Law,
Incompetence and Manifest Bad
Faith against respondent Velasco,
which the DOJ subsequently
dismissed. On reconsideration,
Velasco submitted a comment
which contained statements
pertaining to Judge Angeles
sexuality. Petitioner claimed to be
a direct attack on her character
and reputation as a public servant,
she filed a Complaint for four
counts of libel against respondent
Velasco before the Office of the
City Prosecutor of Manila. It was
dismissed and she filed a petition
for review with the DOJ of the
dismissal. The petition was, again,
dismissed, even upon
reconsideration. Petitioner then
filed a Petition for Review Before
the Office of the President
questioning the DOJ Resolutions
dismissing her petition. The OP
dismissed the Petition for Review,
stating that under Memorandum

Circular (MC) No. 58, no appeal


from or petition for review of the
decision or resolution of the
Secretary of Justice on preliminary
investigation of criminal cases
shall be entertained by the Office
of the President, except those
involving offenses punishable by
reclusion perpetua to death.
Petitioner thereafter filed with the
CA a petition for review under
Rule 43 assailing the OP order. The
CA then ruled that the OP, relying
on MC No. 58, dismissed
petitioner's petition for review and
exercised its prerogative not to
disapprove or overturn the DOJ
Secretary's resolutions, thus,
approving the acts or decision of
the DOJ Secretary, being her alter
ego.
Issue:
WON the CA was erroneous in
applying the doctrine of qualified
political agency
Held:
Memorandum Circular No. 58
provides that no appeal from or
petition for review of
decisions/orders/resolutions of the
Secretary of Justice on preliminary
investigations of criminal cases
shall be entertained by the Office
of the President, except those
involving offenses punishable by
reclusion perpetua to death.
Henceforth, if an appeal or petition
for review does not clearly fall
within the jurisdiction of the Office
of the President, it shall be
dismissed outright The President
himself set the limits of his power
to review
decisions/orders/resolutions of the

Secretary of Justice in order to


expedite the disposition of cases.
Petitioner's argument that the
Memorandum Circular unduly
expands the power of the
Secretary of Justice to the extent
of rendering even the Chief
Executive helpless to rectify
whatever errors or abuses the
former may commit in the exercise
of his discretion is purely
speculative to say the least.
Petitioner cannot second-guess the
President's power and the
President's own judgment to
delegate whatever it is he deems
necessary to delegate in order to
achieve proper and speedy
administration of justice,
especially that such delegation is
upon a cabinet secretary - his own
alter ego.
Elvira O. Ong v. Jose Casim Genio,
G.R. No. 182336, December 23,
2009
Facts:
Sometime in the month of January,
2003, in the City of Makati,
Philippines the respondent did
then and there willfully, unlawfully
and feloniously take, divest and
carry away kitchen and canteen
equipment as well as her personal
things valued at Php 700,000.00,
belonging to complainant, ELVIRA
O. ONG, to the damage and
prejudice of the said owner.
Respondent filed a Motion to
Dismiss the Case for Lack of
Probable Cause Pursuant to Sec.
6(a), Rule 112 of the Rules of
Court and, in View of Compelling
Grounds for the Dismissal of the
Case to Hold in Abeyance the

Issuance of the Warrant of Arrest


(Motion to Dismiss). RTC of Makati
City dismissed the case because
the other elements of the crime of
Robbery, specifically the elements
of intent to gain, and either
violence against or intimidation of
any person or force upon things,
were not specifically alleged in the
Information filed against
respondent. Despite the dismissal
of the case, respondent filed a
Partial Motion for Reconsideration,
reiterating that the Information
should be dismissed in its entirety
for lack of probable cause.
Petitioner filed her Opposition to
this motion. RTC granted
respondents Partial Motion for
Reconsideration and dismissed the
case for lack of probable cause
pursuant to Section 6(a), Rule 112
of the Revised Rules on Criminal
Procedure. Petitioner filed her MR,
claiming that the RTC erred in
relying on Section 6(a), Rule 112
of the Revised Rules on Criminal
Procedure, since the said provision
relates to the issuance of a
warrant of arrest, and it does not
cover the determination of
probable cause for the filing of the
Information against respondent,
which is executive in nature, a
power primarily vested in the
Public Prosecutor. RTC denied
petitioners MR, holding that the
provision authorizes the RTC to
evaluate not only the resolution of
the prosecutor who conducted the
preliminary investigation and
eventually filed the Information in
court, but also the evidence upon
which the resolution was based. In
the event that the evidence on
record clearly fails to establish

probable cause, the RTC may


dismiss the case. Aggrieved,
petitioner filed a Petition for
Certiorari and Mandamus before
the CA. Respondent filed a Motion
to Dismiss the petition, raising the
issue of lack of personality of
petitioner to appeal the dismissal
of the criminal case, because the
authority to do so lies exclusively
with the State as represented by
the OSG. CA observed that the
People of the Philippines was
impleaded as petitioner without
showing, however, the OSG's
participation. Thus, the CA
ordered petitioner to furnish the
OSG with a copy of the Petition,
and the latter to comment thereon.
OSG filed its Comment, taking the
stand of respondent that only the
Solicitor General can bring or
defend actions on behalf of the
People of the Philippines filed
before the CA or the Supreme
Court. The OSG submitted that, for
being fatally defective, the said
Petition should be dismissed
insofar as the criminal aspect was
concerned, without prejudice to
the right of petitioner to pursue
the civil aspect of the case. CA
rendered its Resolution, dismissing
the case without prejudice to the
filing of a petition on the civil
aspect thereof on the basis of the
arguments raised by both
respondent and the OSG.
Petitioner filed an MR which the
CA denied.
Issue:
WON the RTC has the authority to
dismiss the information on the
ground of lack of probable cause
when it has previously concluded

that the same information is


defective
Held:
Section 6(a), Rule 112 of the
Revised Rules on Criminal
Procedure clearly provides that the
RTC judge, upon the filing of an
Information, has the following
options: (1) dismiss the case if the
evidence on record clearly failed to
establish probable cause; (2) if he
or she finds probable cause, issue
a warrant of arrest; and (3) in case
of doubt as to the existence of
probable cause, order the
prosecutor to present additional
evidence within five days from
notice, the issue to be resolved by
the court within thirty days from
the filing of the information.
It bears stressing that the judge is
required to personally evaluate the
resolution of the prosecutor and its
supporting evidence. He may
immediately dismiss the case if the
evidence on record clearly fails to
establish probable cause. This, the
RTC judge clearly complied with in
this case

Pangan v. Ganay, 445 SCRA 574


Facts:
MTC Judge Caroline Pangan made
allegations of gross ignorance of
the law, incompetence, abuse of
authority and dereliction of duty
against Judge Clifton U. Ganay and
Judge Samuel Martires. She
alleged that she handled the
preliminary investigation of the
complaint for murder filed against
Joel Abril and found probable
cause for the filing of the offense
charged and directed the issuance
of a warrant of arrest. The case
was then transmitted to the
Provincial Prosecutor and the
Assistant Provincial Prosecutor
recommended the dismissal of the
case. Due to the conflicting
recommendations of the
complainant and Prosecutor
Lachica, Provincial Prosecutor
Rogelio Hipol created a three-man
panel to reinvestigate the case
which sustained the findings of
probable cause made by the
complainant Judge and resolved to
indict Abril. An Information for
murder was then filed on August
23, 2000 before the sala of
respondent Judge Ganay. l
appeared for arraignment before
the sala of respondent Judge
Martires. However, the latter did
not issue any warrant of arrest
despite the charge of murder
against Abril. Prosecutor Lachica
then filed an Urgent Motion to
Dismiss on the ground of lack of
evidence, but failed to disclose
that a review of the case was being
conducted by the Office of the
Provincial Prosecutor. Abril filed a
petition, which was later amended,

before the sala of respondent


Judge Ganay entitled Re: Petition
to Quash the Warrant of Arrest
and/or for the Release of the
Respondent from Detention which
Judge Ganay granted despite the
fact that the case was still pending
before the Provincial Prosecutor.
The complainant contended that
respondent Judge Ganay should
have informed Abril that his
remedy lies with the Office of the
Provincial Prosecutor, pursuant to
Section 5, Rule 112 of the Revised
Rules of Criminal Procedure, a
petition for certiorari or
prohibition, but not a special
proceeding as the latter was not
among the allowable special
proceedings in the rules. While he
admitted that he did not issue the
warrant of arrest against Abril,
respondent Judge Martires
explained that he was justified in
so doing. After studying the
records of the preliminary
investigation, as transmitted by
the Prosecutors Office, he
immediately noticed that the
accused had already been
arrested. There was thus no longer
a necessity to issue an arrest
warrant or commitment order in
such case, as it would be a mere
superfluity.
Issues:
Did both respondent Judges erred
in not issuing an arrest warrant
Held:
In the case at bar, both respondent
Judges, upon careful perusal of the
records of the case before them,
exercised their discretion and
opted not to issue a warrant of

arrest against Abril. Respondent


Judge Ganays declaration that
there is nothing in the warrant of
arrest or in the order dated
January 10, 2000 issued by
complainant which states that
there was a need to place Joel Toto
Abril under immediate custody so
as not to frustrate the ends of
justice, is correct. Respondent
Judge Martires correctly
manifested that while Prosecutor
Lachica was requesting for the
commitment of accused Abril,
Prosecutor Lachica was also
manifesting that they would be
moving for the dismissal of the
case as they would be filing a case
against another person.
Confronted with this peculiar
situation, respondent Judge
Martires rightly did not issue a
warrant of arrest. There was no
reason to subject accused Abril to
incarceration when the
prosecution believed that accused
Abril was innocent of the crime
charged. It bears stressing that it
is within the discretion of the
Judge to issue a warrant for the
arrest of an accused in a criminal
case. A Judge is required to
personally evaluate the resolution
of the prosecutor and its
supporting evidence. He may
immediately dismiss the case if the
evidence on record clearly fails to
establish probable cause. However,
if he finds probable cause, then he
is mandated by law to issue such
warrant. While before, it was
mandatory for the investigating
Judge to issue a warrant for the
arrest of the accused if he found
probable cause, the rule now is
that the investigating Judges

power to order the arrest of the


accused is limited to instances in
which there is a necessity for
placing him in custody in order not
to frustrate the ends of justice. The
arrest of the accused can be
ordered only in the event that the
prosecutor files the case and the
Judge of the Regional Trial Court
finds probable cause for the
issuance of the warrant of arrest.

Crespo v. Mogul, 151 SCRA 462


Facts:
Petitioner Mario Crespo was
accused for Estafa in the Circuit
Criminal Court of Lucena City.
When the case was set for
arraignment, the accused filed a
motion for defer arraignment on
the ground that there was a
pending petition for review filed
with the Secretary of Justice.
However, Justice Mogul denied the
motion, but the arraignment was
deferred in a much later date to
afford time for the petitioner to
elevate the matter to the appellate
court. The accused filed a petition
for certiorari and prohibition with
prayer for a preliminary writ of
injunction to the CA. The CA
ordered the trial court to refrain
from proceeding with the
arraignment until further orders of
the Court. Undersecretary of
Justice, Hon. Catalino Macaraig Jr.,
resolved the petition for review
reversed the resolution of the
office of the Provincial Fiscal and
directed the Fiscal to move for
immediate dismissal of the
information filed against the
accused. Judge Mogul denied the
motion for dismissal of the case ad
set the arraignment. The accused
then filed a petition for Certiorari,
prohibition and mandamus with
petition for the issuance of
preliminary writ of prohibition
and/or temporary restraining order
in the CA. The CA dismissed the
order and lifted the restraining
order.
Issue:

WON the trial court may refuse to


grant a motion to dismiss ordered
by the Secretary of Justice
HELD:
The rule in this jurisdiction is that
once a complaint or information is
filed in Court any disposition of the
case as its dismissal or the
conviction or acquittal of the
accused rests in the sound
discretion of the Court. Although
the fiscal retains the direction and
control of the prosecution of
criminal cases even while the case
is already in Court he cannot
impose his opinion on the trial
court. The Court is the best and
sole judge on what to do with the
case before it. The determination
of the case is within its exclusive
jurisdiction and competence. A
motion to dismiss the case filed by
the fiscal should be addressed to
the Court who has the option to
grant or deny the same. It does not
matter if this is done before or
after the arraignment of the
accused or that the motion was
filed after a reinvestigation or
upon instructions of the Secretary
of Justice who reviewed the
records of the investigation.

In order therefor to avoid such a


situation whereby the opinion of
the Secretary of Justice who
reviewed the action of the fiscal
may be disregarded by the trial
court, the Secretary of Justice
should, as far as practicable,
refrain from entertaining a petition
for review or appeal from the
action of the fiscal, when the
complaint or information has

already been filed in Court. The


matter should be left entirely for
the determination of the Court.

Santos v. Orda, Jr. 437 SCRA 504


Facts:
On April 2, 2001, Francis Orda, the
son of respondent Domingo Orda
Jr., was shot todeath in Paranaque
City. Gina Azarcon, an eyewitness
in the crime, executed her sworn
statement that she saw three male
persons commit the crime and
later identified two of the
assailants as Rolly Tonion and
Jhunrey Soriano. Thus, an
Information for the crime of
murder was filed against Tonion
and Soriano. Ernesto Regala and
his son, Dennis Regala, also came
out as witnesses. On the day
before Francis was shot to death,
Ernesto saw petitioner Santos
hand a gun to Tonion, saying
Gusto ko malinis ang trabaho at
walang bulilyaso, baka makaligtas
na naman si Orda. For his part,
Dennis alleged Tonion asked him
to return the gun to Santos a few
days after the incident. Santos also
instructed him to monitor the
activities of respondent. Based on
the accounts of Ernesto and
Dennis, an Information was filed
against Santos and Edna Cortez.
Upon further testimony of
Azarcon, the information was
amended to include petitioner
Robert Bunda. Gina, Ernesto and
Dennis later recanted their
testimonies. In June 2002, DOJ
issued a joint resolution directing
the city prosecutor to cause the
withdrawal of the Informations
filed against the accused on the
ground that testimonies of the
witnesses were not credible
because of their recantation. On
motion of the prosecution, the trial

court issued an order allowing the


withdrawal of the Informations
against the accused and
consequently recalling the
warrants for their arrest. This
prompted respondent Orda to
elevate the matter to CA. CA
nullified the trial courtsorders and
declared that the trial court
committed grave abuse of
discretion in allowing the
withdrawal of the Informations
without making an independent
evaluation on the merits of the
case.
Issue:
WON the trial court committed
grave abuse of discretion in
granting the public prosecutors
motion to withdraw the
Informations and in lifting the
warrant of arrest against
thepetitioners on the DOJs finding
that there was no probable cause
for the filing of said Informations
Held:
Yes. The trial court should not rely
solely and merely on the findings
of the public prosecutor or the
Secretary of Justice that no crime
was committed or that the
evidence in the possession of the
public prosecutor is insufficient to
support a judgment of conviction
of the accused. The trial court
must make an independent
evaluation or assessment of the
merits of the case and the
evidence on record of the
prosecution. The trial court may
make an independent assessment
of the merits of the case based on
the affidavits and counteraffidavits, documents, or evidence

appended to the Information; the


records of the public prosecutor
which the court may order the
latter to produce before the court;
or any evidence already adduced
before the court by the accused at
the time the motion is filed by the
public prosecutor. In this case, the
trial court failed to make an
independent assessment of the
merits of the cases and the
evidence on record or in the
possession of the public
prosecutor. In granting the motion
of the public prosecutor to
withdraw the Informations, the
trial court relied solely on the joint
resolution of the Secretary of
Justice, as gleaned from its
assailed order. In granting the
public prosecutors motion, the
trial court abdicated its judicial
power and acted as a mere
surrogate of the Secretary of
Justice. Worse, the trial court knew
that the Joint Resolution of the
Secretary of Justice had not ye
tbecome final and executory
because the respondent, the
private complainant, had filed a
timely motion for the
reconsideration thereof which had
not yet been resolved by the
Secretary of Justice. It behooved
the trial court to wait for the
resolution of the Secretary of
Justice on the motion for
reconsideration of the respondent
before resolving the motion of the
public prosecutor to withdraw the
Informations. In fine, the trial
court acted with inordinate haste

Dumlao v. Ponferrada, 508 SCRA


426
Facts:
On the night of May 8, 1995, just
after the local elections, seven of
petitioners relatives were
murdered in Brgy. Pattao, Buguey,
Cagayan. Two witnesses, Ernesto
Mendoza and Mario Gascon,
executed affidavits stating that
they were riding in a jeepney with
the victims when armed men in
fatigue uniforms flagged them
down. On instruction of ex-Army
Major Romulo Langcay (one of the
passengers), the jeepney did not
stop. The armed men, however,
fired at the jeepney, forcing it to
stop. The witnesses later identified
three of these men to be private
respondents Roy Flores, Godofredo
Flores and Quirino Cabeza. In the
same affidavits, Mendoza and
Gascon stated that when they saw
Roy Flores and his men
approaching the vehicle, they
jumped out and hid behind a guava
tree. From there, they noticed
private respondent Roy Flores
returning to his house about 41
meters away. He talked to private
respondents Atty. Franklin
Tamargo and Atty. Manuel Molina.
Mendoza and Gascon allegedly
overheard Tamargo and Molina
giving orders to Roy Flores to kill
all the passengers of the jeepney.
Thereafter, Roy Flores went back
to the jeepney and ordered all the
passengers to lie flat on the
ground. After a few seconds,
Flores and his companions fired at
them.

Private respondents denied the


accusations. They contended that
they were inside Roy Flores house
when the killings occurred.
Although they heard the gunshots,
they claimed no knowledge of what
precipitated the incident.
According to private respondents,
Mendoza and Gascon were known
loyal supporters of Mayor Licerio
Antiporda Jr. and they merely
fabricated the whole story to pin
down liability on Tamargo who was
then Antipordas closest political
rival. They added that it was
impossible for Mendoza and
Gascon to have heard any order
from Tamargo and Molina 41
meters away from the guava tree
where they were hiding. Apolinario
and Leonardo Time corroborated
private respondents statements,
saying that they were inside Roy
Flores house during the shooting
incident.
After the preliminary investigation,
State Prosecutor Emmanuel
Velasco issued a resolution stating
that there was probable cause to
hold private respondents liable for
multiple murder. Private
respondents filed a petition for
review with the DOJ Then acting
DOJ Secretary Ricardo G.
Nepomuceno, Jr. issued an order
reversing the findings of State
Prosecutor Velasco and directing
the withdrawal of the criminal
charges against private
respondents. Later, the new DOJ
Secretary, Serafin Cuevas, also
ordered the withdrawal of said
Informations. Hence, a motion to
withdraw them were filed. Public
respondent Judge Rodolfo

Ponferrada, presiding judge of the


trial court, granted the motion and
dismissed the cases filed against
private respondents. In their bid to
reverse Judge Ponferradas order,
petitioners essentially insist in this
petition for certiorari that the
Secretary of Justice could no
longer review the resolution of the
public prosecutor after the cases
had been filed in court.
Issue:
WON the Secretary of Justice
could no longer review the
resolution of the prosecutor after
the case has been filed in court
Held:
Under RA 5180, in connection with
Rule 112, Section 4 of the Rules of
Court, and as further implemented
by Department Circular No. 70
(otherwise known as the 2000
National Prosecution Service Rule
on Appeal), the Justice Secretary is
vested with the power to review
resolutions of the provincial, city
prosecutor or chief state
prosecutor. He has the power to
re-evaluate the position taken by
his subordinates in a case.
Corollary to this power, he may
also direct the public prosecutor to
dismiss or cause the dismissal of
the complaint or information.
Contrary to the theory of
petitioners, the filing of a
complaint or information in court
does not prevent the Justice
Secretary from exercising his
review power. Neither can such
complaint or information deter him
from ordering the withdrawal of
the case. As a matter of fact, in
Crespo, we declared that the

public prosecutor may still opt to


withdraw the Information either
upon instruction of the Justice
Secretary or for purposes of
reinvestigation. Furthermore,
Crespo merely laid down the rule
that, while the Secretary of Justice
has the power to alter or modify
the resolution of his subordinate
and thereafter direct the
withdrawal of a case, he cannot,
however, impose his will on the
court. It was held in Crespo that
once a complaint or information is
filed in Court any disposition of the
case as its dismissal or the
conviction or acquittal of the
accused rests the sound discretion
of the Court. Although the fiscal
retains the direction and control of
the prosecution of the criminal
cases even while the case is
already in Court he cannot impose
his opinion on the trial court. The
determination of the case is within
the courts exclusive jurisdiction
and competence. A motion to
dismiss the case filed by the fiscal
should be addressed to the sound
discretion of the Court that has the
option to grant or deny the same.

Filemon Verzano, Jr. v. Francis


Victor D. Paro, G.R. No. 171643,
August 8, 2010
Facts:
On March 2002, Verzano former
district manager of Wyeth
Philippines, Inc. for the islands of
Panay and Negros was dismissed
from service upon administrative
complaint filed against him. The
complaint was founded on
petitioner's alleged violation of
company policy on prohibited sale
of drug samples given for free to
doctors and for the unauthorized
act of transferring of the stocks
within the same area falsely
creating an impression that there
was a sale. After conducting its
own investigation and giving
petitioner an opportunity to
explain his side, wyeth resolved to
dismiss petitioner tendering him a
Notice of Termination. Verzano
filed a complaint for illegal
dismissal with Regional Labor
Arbitration Board, NLRC, Bacolod
City against Wyeth. Attached were
the affidavits of respondents Paro
and Florencio alleging that the
respondents' testimony are false
and incriminatory machination.
The affidavits of the respondents
contained falsehood particularly on
the material date of the alleged
sale of products which are to be
given free to doctors. Subpoenas
were issued by the City Prosecutor
against respondents for the
submission of their respective
counter-affidavits; however, the
return of the subpoenas showed
that respondents could not be
located at their given addresses.In
a resolution, the city prosecutors

resolved to dismiss Verzano's


complaint finding no probable
cause and insufficiency of
evidence. Verzano filed a motion
for reconsideration, which was
denied by the city prosecutor in a
resolution. Verzano appealed the
resolution oof the city prosecutor
to the office of regional state
prosecutor via petition for review,
but regional state prosecutor
finding merit in Verzano's petition
reversed and directed the
prosecutor's office to file
information for perjury against
Paro, Florencio. The respondents
then filed a motion for
reconsideration which was denied
by the Regional State Prosecutor.
Respondents further filed a
petition for certiorari before the
CA under Rule 65 assailing the
resolutions of the regional state
prosecutor which reversed the
earlier resolution of the city
prosecutor and prayed for a TRO
from CA. On October 14, 2004 a
TRO was issued by CA enjoining
the public respondent chief
prosecutor from acting on the
assailed order issued by the
regional state prosecutor for a
period of 60 days from receipt.
Issue:
WON petition filed by respondents
with CA had been rendered moot
and academic by the filing of the
cases in court
Held:
The rule therefore in this
jurisdiction is that once a
complaint or information is filed in
Court any disposition of the case
as its dismissal or the conviction or

acquittal of the accused rests in


the sound discretion of the Court.
Although the fiscal retains the
direction and control of the
prosecution of criminal cases even
while the case is already in Court
he cannot impose his opinion on
the trial court. The Court is the
best and sole judge on what to do
with the case before it. The
determination of the case is within
its exclusive jurisdiction and
competence. A motion to dismiss
the case filed by the fiscal should
be addressed to the Court who has
the option to grant or deny the
same. It does not matter if this is
done before or after the
arraignment of the accused or that
the motion was filed after a
reinvestigation or upon
instructions of the Secretary of
Justice who reviewed the records
of the investigation. The justice
secretary's power of review may
still be availed of despite the filing
of an information in court. The
court may deny or grant a motion
to withdraw an information, not
out of subservience to the
Prosecutor, but in faithful exercise
of judicial discretion and
prerogative.

Sierra v. Lopez, A.C. 7549, August


29, 2008
Facts:
On July 27, 2006 and August 1,
2006, complainant Aurelio M.
Sierra filed several cases before
the Office of the City Prosecutor of
Manila for Misrepresentation
through Deceit and Syndicated
Large Scale Fraud in Land Titling
with Conspiracy, Land Grabbing,
Falsification of Public Document
and Economic Sabotage. These
cases were first assigned to
Assistant City Prosecutor
Alexander T. Yap. The principal
respondents therein, namely:
Alfredo C. Ramos, Presentacion
Ramos, George S.K. Ty, Atty.
Emmanuel Leonardo, and a certain
Mr. Cayaban, did not appear
during the scheduled hearing.
Because of ACP Yaps failure to
require the presence of
respondents in said cases
simultaneously with the
complainant, Mr. Sierra asked for
the prosecutors inhibition. The
cases were then re-raffled to the
respondent ACP Marlo Campanilla,
then to ACP Armando Velasco who
likewise did not require the
presence of the respondents in the
preliminary investigation. City
Prosecutor Jhosep Y. Lopez and 1st
ACP Eufrocino A. Sulla affirmed
the correctness of the manner in
which their investigating
prosecutors handled the cases. On
April 26, 2007, Sierra filed a
complaint with the Supreme Court
for dereliction of duty and gross
ignorance of the law against City
Prosecutor Lopez, 1st ACP Sulla,

ACP Yap, ACP Campanilla, and


ACP Velasco.
Issue:
WON the parties must appear
together before the investigating
prosecutor during preliminary
investigation
Held:
Rule 112, particularly Section 3 of
the Rules of Court, lays down the
basic procedure in preliminary
investigation. The provision of the
Rules does not require a
confrontation between the parties.
Preliminary investigation is
ordinarily conducted through
submission of affidavits and
supporting documents, through
the exchange of pleadings. Since
confrontation between the parties
is not imperative, it follows that it
is not necessary that the counteraffidavit of respondent be sworn to
before the investigating prosecutor
himself. It can be sworn to before
another prosecutor. In fact, this is
specifically provided in paragraph
(c) of Sec. 3, which states that the
counter-affidavit shall be
subscribed and sworn to and
certified as provided in paragraph

Bautista v. Sandiganbayan, G.R.


No. 136082, May 12, 2000
Facts:
An anonymous, unverified and
unsigned letter-complaint dated 20
November 1996 allegedly
prepared by the Contractors
Association of Davao del Sur and
the Good Government Employees
of Davao del Sur was filed with the
Office of the Ombudsman for
Mindanao charging petitioner
Franklin P. Bautista, incumbent
mayor of the Municipality of
Malita, Davao del Sur, for violation
of Sec. 3, par. (e), of the Anti-Graft
and Corrupt Practices Act.The
letter-complaint alleged, among
others, that petitioner caused the
hiring of one hundred and ninetytwo (192) casual employees in the
municipal government for political
considerations and that the
payment of their honoraria and
salaries was charged to the peace
and order fund despite meager
savings of the municipality. Acting
on the letter-complaint, Graft
Investigation Officer II (GIO II)
Corazon A. Arancon issued on 16
January 1997 an Order directing
respondent Franklin P. Bautista to
submit his counter-affidavit. In his
counter-affidavit, petitioner
answered the charges against him,
claimed that the complaint, which
was unsigned, was fictitious and
fabricated as shown by the
affidavits of Enrique Ponce De
Leon, President of the Contractor's
Association of Davao del Sur;
Rogelio E. Llanos, Governor for
Davao del Sur; Eduardo M.
Masiwel Vice Mayor of Malita,
Davao del Sur; Engineer Antonio P.

Cayoca, Department of Public


Works and Highways, 2nd District,
Davao del Sur; Juanito A. Itorralba,
Assistant Provincial Treasurer of
Davao del Sur; Juan L. de Guzman
and Felipe D. Macalinao, both
teachers, therein attached, which
disclaimed any knowledge of the
institution of the complaint nor
cause of its filing. He further
argued that the hiring of the one
hundred ninety-two (192) casuals
and the payment of their honoraria
and wages did not justify the filing
of any charge against him. After
the Informaton was filed against
him, he filed a Motion to Quash
and subsequently a Motion for
Reconsideration after the same
was denied. The Motion for
Reconsideration was, however,
also denied. He asserts that there
was no legal basis for the
Ombudsman to conduct a
preliminary investigation as the
Ombudsman failed to direct the
complainants to reduce their
evidence into affidavits before
requiring him to submit his
counter-affidavit.
Issue:
WON the petitioners contention
was right
Held:
The petitioner was right that only
after complainants reduce their
evidence into affidavits can they
requiring him to submit his
counter-affidavit. However,
petitioner Bautista had already
filed his counter-affidavit before
the Ombudsman and only
questioned the latters failure to
require the complainants to submit

affidavits prior to the submission


of his own counter-affidavit after
the preliminary investigation had
ended and Information already
filed before the Sandiganbayan.
The issue therefore of requiring
the complainants to submit their
affidavits before respondent can be
obliged to submit his counteraffidavit is moot and academic in
light of Bautistas submission of his
counter-affidavit despite absence
of the complainants affidavits. It
may be true that GIO II Arancon in
his Order of 16 January 1997
directed herein petitioner to
submit his counter-affidavit
thereto without requiring the
complainants to submit theirs
which were significantly necessary
because of the unverified,
unsigned and anonymous nature of
their letter. However, despite the
Ombudsman's noncompliance with
the affidavit requirement,
petitioner filed his counteraffidavit on 26 February 1997 and
answered the charges against him.
Hence, having submitted himself
to the jurisdiction of the
Ombudsman and having allowed
the proceedings to go on until the
preliminary investigation was
terminated and the Information
filed at the Sandiganbayan,
petitioner is deemed to have
waived whatever right he may
otherwise have to assail the
manner in which the preliminary
investigation was conducted.
Consequently, petitioner is
likewise estopped from
questioning the validity of the
Information filed before the
Sandiganbayan

Sasot v. People, G.R. No. 143193,


June 29, 2005
Facts:
Some time in May 1997, the
National Bureau of Investigation
(NBI) conducted an investigation
pursuant to a complaint by the
NBA Properties, Inc., against
petitioners for possible violation of
Article 189 of the Revised Penal
Code on unfair competition. NBI
stated that NBA Properties, Inc., is
a foreign corporation organized
under the laws of the United
States of America, and is the
registered owner of NBA
trademarks and names of NBA
basketball teams. These names are
used on hosiery, footwear, t-shirts,
sweatshirts, tank tops, pajamas,
sport shirts, and other garment
products, which are allegedly
registered with the Bureau of
Patents, Trademarks and
Technology Transfer. The Report
further stated that during the
investigation, it was discovered
that petitioners are engaged in the
manufacture, printing, sale, and
distribution of counterfeit NBA
garment products. Hence, it
recommended petitioners
prosecution for unfair competition
under Article 189 of the Revised
Penal Code. In a Special Power of
Attorney dated October 7, 1997,
Rick Welts, as President of NBA
Properties, Inc., constituted the
law firm of Ortega, Del Castillo,
Bacorro, Odulio, Calma &
Carbonell, as the companys
attorney-in-fact, and to act for and
on behalf of the company, in the
filing of criminal, civil and
administrative complaints, among

others.The Special Power of


Attorney was notarized by Nicole
Brown of New York County and
certified by Norman Goodman,
County Clerk and Clerk of the
Supreme Court of the State of New
York. Consul Cecilia B. Rebong of
the Consulate General of the
Philippines, New York,
authenticated the certification.
Welts also executed a ComplaintAffidavit on February 12, 1998,
before Notary Public Nicole J.
Brown of the State of New York.
Petitioners reiterate the argument
that the complaint filed by Rick
Welts of the NBA Properties, Inc.,
is defective and should have been
dismissed by the fiscal because it
should have been personally sworn
to by the complainant before the
investigating prosecutor.
Prosecution Attorney Aileen Marie
S. Gutierrez recommended the
filing of Information against
petitioners for violation of Article
189 of the RPC. Before
arraignment, petitioners filed a
Motion to Quash the Information
on the grounds that the facts
charged do not constitute a crime
and the court had no jurisdiction
over the offense charged or the
person of the accused.
Issue:
WON the complaint was defective
for not having been personally
sworn to by the complainant
before the investigating prosecutor
Held:
Nowhere in Section 3, Rule 112 of
the 1985 Rules of Criminal
Procedure there any mention of
the defect in the complaint filed

before the fiscal and the


complainants capacity to sue as
grounds for a motion to quash. In
fact, a complaint is substantially
sufficient if it states the known
address of the respondent, it is
accompanied by complainants
affidavit and his witnesses and
supporting documents, and the
affidavits are sworn to before any
fiscal, state prosecutor or
government official authorized to
administer oath, or in their
absence or unavailability, a notary
public who must certify that he
personally examined the affiants
and that he is satisfied that they
voluntarily executed and
understood their affidavits. All
these have been duly satisfied in
the complaint filed before
Prosecution Attorney Aileen Marie
S. Gutierrez. It must be noted that
even the absence of an oath in the
complaint does not necessarily
render it invalid. Want of oath is a
mere defect of form, which does
not affect the substantial rights of
the defendant on the merits. In
this case, Weltss ComplaintAffidavit contains an
acknowledgement by Notary
Public Nicole Brown of the State of
New York that the same has been
subscribed and sworn to before
her on February 12, 1998, duly
authenticated by the Philippine
Consulate. While the copy on
record of the complaint-affidavit
appears to be merely a photocopy
thereof, Prosecution Attorney
Gutierrez stated that complainants
representative will present the
authenticated notarized original in
court, and Prosecutor Guray
manifested that the original copy

is already on hand. It is apt to


state at this point that the
prosecutor enjoys the legal
presumption of regularity in the
performance of his duties and
functions, which in turn gives his
report the presumption of
accuracy.
Moreover, records show that there
are other supporting documents
from which the prosecutor based
his recommendation.
Consequently, if the information is
valid on its face, and there is no
showing of manifest error, grave
abuse of discretion and prejudice
on the part of public prosecutor, as
in the present case, the trial court
should respect such determination.

ARREST
Tabujara v. People, G.R. No.
175162, October 29, 2008
Facts:
On 17 September 1999, respondent
Daisy Dadivas-Afable simultaneously
filed two criminal complaints against
petitioners for Grave Coercion and
Trespass to Dwelling. On the 14th day
of September 1999 at around 6:00
oclock in the morning, in Brgy. Iba,
Municipality of Meycauayan, Province
of Bulacan, petitioners entered the
house of Davidas-Afable by opening
the gate and forcing her to go with
them against her will. Petitioners
denied the allegations and argued
that on 14 September 1999, they went
to the house of respondent to thresh
out matters regarding some missing
pieces of jewelry. Respondent was a
former employee of Miladay Jewels,
Inc., a company owned by the Dayrits
and who was then being
administratively investigated in
connection with missing jewelries.
Despite several summons to appear,
respondent went on AWOL. Judge
Calixto Adriatico of Meycauayan MTC
conducted the preliminary
investigation and thereafter dismissed
the complaints for lack of probable
cause on the ground that the
complaint was believed to be merely
leverage against the estafa case
already filed against the private
complainant. Respondent filed a
Motion for Reconsideration alleging
that when she filed the complaints for
grave coercion and trespass to

dwelling on 17 September 1999


against petitioners, no information for
estafa has yet been filed against her.
In fact, the information was filed on 5
October 1999. In their Opposition to
the Motion for Reconsideration,
petitioners argued that even before
respondent filed the criminal
complaints for grave coercion and
trespass to dwelling, she was already
being administratively investigated
for the missing jewelries; that she was
ordered preventively suspended
pending said investigation; that the
theft of the Miladay jewels was
reported to the Makati Police on 7
September 1999 with respondent
Afable being named as the primary
suspect; that on 17 September 1999,
which corresponded to the date of
filing of the criminal complaints
against petitioners, the employment
of respondent with Miladay, Jewels,
Inc. was terminated. Petitioners
further alleged that respondent filed
the criminal complaints for grave
coercion and trespass to dwelling as
leverage to compel petitioners to
withdraw the estafa case.

On 2 May 2000, Judge Adriatico


issued an Order reversing his
earlier findings of lack of probable
cause. This time, he found
probable cause to hold petitioners
for trial and to issue warrants of
arrest on the ground that the
sworn statement of witness Mauro
V. De Lara was inadvertently
overlooked. De Lara stated that he
saw the private complainant being
forcibly taken by three persons

from her residence. Petitioners


filed a motion for reconsideration
insisting that the alleged affidavit
of Mauro V. de Lara on which the
court a quo based its findings of
probable cause was hearsay
because it was not sworn before
Judge Adriatico; that De Lara did
not personally appear before the
investigating judge during
preliminary investigation.
Issue:
WON Judge Adriatico committed
grave abuse of discretion in
issuing Orders finding probable
cause.
Held:
It is constitutionally mandated that
a warrant of arrest shall issue only
upon finding of probable cause
personally determined by the
judge after examination under
oath or affirmation of the
complainant and the witnesses
he/she may produce, and
particularly describing the person
to be seized. Also, the procedure
described in Section 6 of Rule 112
is mandatory because failure to
follow the same would amount to a
denial of due process. With respect
to the issuance by inferior courts
of warrants of arrest, it is
necessary that the judge be
satisfied that probable cause
exists: 1) through an examination
under oath and in writing of the
complainant and his witnesses;
which examination should be 2) in
the form of searching questions
and answers. This rule is not
merely a procedural but a
substantive rule because it gives
flesh to two of the most sacrosanct

guarantees found in the


fundamental law: the guarantee
against unreasonable searches and
seizures and the due process
requirement.
To determine the existence of
probable cause, a preliminary
investigation is conducted. A
preliminary investigation is an
inquiry or proceeding to determine
whether there is sufficient ground
to engender a well-founded belief
that a crime has been committed
and the respondent is probably
guilty thereof, and should be held
for trial. Clearly, Judge Adriatico
gravely abused his discretion in
issuing the assailed 2 May 2000
and 14 July 2000 Orders finding
probable cause to hold petitioners
liable for trial and to issue
warrants of arrest because it was
based solely on the statement of
witness Mauro De Lara whom
Judge Adriatico did not personally
examine in writing and under oath;
neither did he propound searching
questions. He merely stated in the
assailed 2 May 2000 Order that he
overlooked the said statement of
De Lara; nevertheless, without
conducting a personal examination
on said witness or propounding
searching questions, Judge
Adriatico still found De Laras
allegations sufficient to establish
probable cause. Plainly, this falls
short of the requirements imposed
by no less than the Constitution.
When the investigating judge
relied solely on the affidavit of
witness De Lara which was not
sworn to before him and whom he
failed to examine in the form of
searching questions and answers,

he deprived petitioners of the


opportunity to test the veracity of
the allegations contained therein.
Worse, petitioners arguments that
De Laras affidavit was hearsay was
disregarded by the investigating
judge despite the fact that the
allegations therein were
completely rebutted by petitioners
and their witnesses affidavits, all
of whom appeared before and
were personally examined by the
investigating judge

Borlongan v. Pena, G.R. No.


143591, May 5, 2012
Facts:
Respondent Magdaleno Pea
instituted a civil case for recovery
of agents compensation and
expenses, damages, and attorneys
fees, against Urban Bank and the
petitioners, before the RTC of
Negros Occidental, Bago City. He
anchored his claim for
compensation on the contract of
agency, allegedly entered into with
the petitioners wherein the former
undertook to perform such acts
necessary to prevent any intruder
and squatter from unlawfully
occupying Urban Banks property
located along RoxasBoulevard,
Pasay City. Petitioners filed a
Motion to Dismiss arguing that
they never appointed the
respondent as agent or counsel..
Attached to the Motion to Dismiss
were the documentsshowing that
the respondent wasappointed as
agent by ISCI and not by Urban
Bank or by the petitioners. Pea
filed his Complaint-Affidavit with
the Office of the City Prosecutor
.He claimed that said documents
were falsified because the alleged
signatories did not actually affix
their signatures, and the
signatories were neither
stockholders nor officers and
employeesof ISCI. Worse,
petitioners introduced said
documents as evidence before the
RTC knowing that they were
falsified. Informations were filed
with the MTCC. Thereafter, Judge
Primitivo Blanca issued the
warrants for the arrest of the
petitioners. Petitioners filed an

Omnibus Motion to Quash. They


insist that they were denied due
process because of the nonobservance of a proper procedure
on preliminary investigation
prescribed in the Rules of Court;
since no such counter-affidavit and
supporting documents were
submitted by the petitioners, the
trial judge merely relied on the
complaint-affidavit and
attachments of the respondent in
issuing the warrants of arrest, also
in contravention of the Rules.
Moreover they claim that the
respondents affidavit was not
based on the latters personal
knowledge and therefore should
not have been used by the court in
determining probable cause.
MTCC upheld the validity of the
warrant of arrest, saying that it
was issued in accordance with the
Rules. Besides, petitioners could
no longer question the validity of
the warrant since they already
posted bail.
Issue:
Did the Judge correctly issued the
warrant of arrest
Held:
The Judge issued a warrant for the
arrest of the petitioners, including,
Mr. Ben Lim, Jr. despite the filing
of the Omnibus Motion to Quash,
Recall Warrants of Arrest and/or
For Reinvestigation raising among
others the issue that Mr. Ben Lim,
Jr., was not even a member of the
board of directors. With the filing
of the motion, the judge is put on
alert that an innocent person may
have been included in the
complaint.. Enshrined in our

Constitution is the rule that no


warrant of arrest shall issue except
upon probable cause to be
determined personally by the
judge after examination under
oath or affirmation of the
complainant and the witnesses he
may produce, and particularly
describing the persons to be
seized. Interpreting the words
personal determination, we said in
Soliven v. Makasiar (167 SCRA
393[1988]) that it does not thereby
mean that judges are obliged to
conduct the personal examination
of the complainant and his
witnesses themselves. To require
thus would be to unduly laden
them with preliminary
examinations and investigations of
criminal complaints instead of
concentrating on hearing and
deciding cases filed before them.
Rather, what is emphasized merely
is the exclusive and personal
responsibility of the issuing judge
to satisfy himself as to the
existence of probable cause. To
this end, he may: (a) personally
evaluate the report and the
supporting documents submitted
by the prosecutor regarding the
existence of probable cause and,
on the basis thereof, issue a
warrant of arrest; or (b) if on the
basis thereof he finds no probable
cause, disregard the prosecutor's
report and require the submission
of supporting affidavits of
witnesses to aid him in
determining its existence. What he
is never allowed to do is to follow
blindly the prosecutor's bare
certification as to the existence of
probable cause. Much more is
required by the constitutional

provision. Judges have to go over


the report, the affidavits, the
transcript of stenographic notes if
any, and other documents
supporting the prosecutor's
certification.
An arrest without a probable cause
is an unreasonable seizure of a
person, and violates the privacy of
persons which ought not to be
intruded by the State.
Measured against the
constitutional mandate and
established rulings, there was here
a clear abdication of the judicial
function and a clear indication that
the judge blindly followed the
certification of a city prosecutor as
to the existence of probable cause
for the issuance of a warrant of
arrest with respect to all of the
petitioners. The careless inclusion
of Mr. Ben Lim, Jr., in the warrant
of arrest gives flesh to the bone of
contention of petitioners that the
instant case is a matter of
persecution rather than
prosecution.

People v. Tan, G.R. No. 182310,


December 9, 2009
Facts:
Brothers Archie and Jan-jan
suspects in the commission of
parricide and two murders of their
father, their step-mother and step
sister. Archie and Jan-jan stated
that they saw their dead stepsister at the end of the hallway of
their house and when Jan-jan went
to the masters bedroom he saw
their father lying on the bed with
his chest drenched in blood. Archie
later saw their step-mother near
the wall below the air conditioner
in his room also dead. Their alibi is
that they went out at about 12:45
a.m and took a cab to Caldaza Bar
and returned at 3:30 a.m. Based
on Dr. Lebaquins forensic
computation, however, the victims
probably died at about midnight,
more or less. The two were still at
home when the killings happened.
On January 18, 2006 police officer
Eldy Bebit of the CIDG filed a
complaint-affidavit with the City
Prosecutors Office, accusing the
two brothers of parricide and
double murder.[The parties
submitted their affidavits and
pieces of evidence at the
preliminary investigation. On
September 29, 2006 the City
Prosecutors Office filed separate
informations for two murders and
parricide against respondents
Archie and Jan-Jan before the
Regional Trial Court (RTC) of Iloilo
City
Archie and Jan-Jan filed a motion
for judicial determination of
probable cause with a prayer to

suspend the issuance of warrants


of arrest against them in the
meantime. Further, on October 5,
2006 they asked the RTC to defer
further proceedings in order to
give them the opportunity to
question the public prosecutors
resolution in the case before the
Secretary of Justice. The RTC, then
temporarily presided over by Judge
Narciso Aguilar, found no probable
cause against respondents Archie
and Jan-Jan. Judge Aguilar thus
granted their motion to suspend
the issuance of warrants for their
arrest and to defer the
proceedings. The two respondents
then filed a motion to dismiss the
case. Meanwhile, the DOJ issued a
resolution dismissing respondents
Archie and Jan-Jans petition for
review.
After a new presiding judge, Judge
Globert Justalero, took over the
RTC, he reversed the order of the
previous presiding judge. He found
probable cause against
respondents Archie and Jan-Jan
this time and ordered the issuance
of warrants for their arrest.
Without seeking reconsideration of
Judge Justaleros order, Archie and
Jan-Jan filed the present petition
for certiorari with the Court of
Appeals. After hearing, the CA
granted the petition and annulled
the warrants of arrest that Judge
Justalero issued. The CA also
dismissed the criminal cases
against the respondents. The
public prosecutor filed a motion
for reconsideration of the CAs
decision through the Office of the
Solicitor General but the latter

court denied it, hence, this


petition.
Issue:
WON the CA committed error in
ruling that Judge Justalero gravely
abused his discretion when he
made a finding that there is
probable cause to issue a warrant
for the arrest of the two
Held:
The CA held that Judge Justalero
gravely abused his discretion when
he made a finding that there is
probable cause to warrant the
arrest of Archie and Jan-Jan.
Probable cause assumes the
existence of facts that would lead a
reasonably discreet and prudent
man to believe that a crime has
been committed and that it was
likely committed by the person
sought to be arrested. It requires
neither absolute certainty nor
clear and convincing evidence of
guilt. The test for issuing a
warrant of arrest is less stringent
than that used for establishing the
guilt of the accused. As long as the
evidence shows a prima facie case
against the accused, the trial court
has sufficient ground to issue a
warrant for his arrest.
Here, admittedly, the evidence
against respondents Archie and
Jan-Jan is merely circumstantial.
The prosecution evidence shows
that they had motive in that they
had been at odds with their father
and stepmother. They had
opportunity in that they were still
probably home when the crime
took place. Archie took two pairs
of new gloves from his car late that

evening. Cindy was apparently


executed inside Archies room. The
separate rooms of the two accused
had, quite curiously, been wiped
clean even of their own
fingerprints. A trial, unlike
preliminary investigations, could
yield more evidence favorable to
either side after the interrogations
of the witnesses either on direct
examination or on crossexamination. What is important is
that there is some rational basis
for going ahead with judicial
inquiry into the case. This Court
does not subscribe to the CAs
position that the prosecution had
nothing to go on with.

AAA v. Carbonell, G.R. No. 171465,


June 8, 2007
Facts:
Petitioner worked as a secretary at
the Arzadon Automotive and Car
Service Center. On May 27, 2001
at about 6:30 p.m., Arzadon asked
her to deliver a book to an office
located at another building but
when she returned to their office,
the lights had been turned off and
the gate was closed. Nevertheless,
she went inside to get her
handbag. On her way out, she saw
Arzadon standing beside a parked
van holding a pipe. He told her to
go near him and upon reaching his
side, he threatened her with the
pipe and forced her to lie on the
pavement. He removed her pants
and underwear, and inserted his
penis into her vagina. She wept
and cried out for help but to no
avail because there was nobody
else in the premises. Petitioner did
not report the incident because
Arzadon threatened to kill her and
her family. But when she
discovered that she was pregnant
as a consequence of the rape, she
narrated the incident to her
parents. On July 24, 2002,
petitioner filed a complaint for
rape against Arzadon. Assistant
City Prosecutor Imelda Cosalan
found probable cause and
recommending the filing of an
information for rape. Arzadon
moved for reconsideration and
during the clarificatory hearing,
petitioner testified before the
investigating prosecutor. However,
she failed to attend the next
hearing hence, the case was
provisionally dismissed. Petitioner

filed another Affidavit-Complaint


with a comprehensive account of
the alleged rape incident. The case
was assigned to 2nd Assistant
Provincial Prosecutor Georgina
Hidalgo. During the preliminary
investigation, petitioner appeared
for clarificatory questioning. The
investigating prosecutor, finding
that a prima facie case of rape
exists, recommended the filing of
the information. An Information
was filed. Arzadon filed a Motion
to Hold in Abeyance All Court
Proceedings Including the
Issuance of a Warrant of Arrest
and to Determine Probable Cause
for the Purpose of Issuing a
Warrant of Arrest. Judge Antonio
A. Carbonell granted the motion
and directed petitioner and her
witnesses to take the witness
stand for determination of
probable cause. Instead of taking
the witness stand, petitioner filed a
motion for reconsideration
claiming that the documentary
evidence sufficiently established
the existence of probable cause.
Meanwhile, Judge Carbonell issued
the assailed Order dismissing
Criminal Case No. 6983 for lack of
probable cause. Petitioners
motion for reconsideration was
denied hence, this petition.
Petitioner contends that the judge
is not required to personally
examine the complainant and her
witnesses in satisfying himself of
the existence of probable cause for
the issuance of a warrant of arrest.
She argues that respondent Judge
Carbonell should have taken into
consideration the documentary
evidence as well as the transcript
of stenographic notes which

sufficiently established the


existence of probable cause. Judge
Carbonell argues in his Comment
that the finding of probable cause
by the investigating prosecutor is
not binding or obligatory, and that
he was justified in requiring
petitioner and her witnesses to
take the witness stand in order to
determine probable cause. He
claims that under Section 2,
Article III of the 1987 Constitution,
no warrant of arrest shall issue
except upon probable cause to be
determined personally by the
judge after examination under
oath or affirmation of the
complainant and the witnesses he
may produce.
Issue:
WON Judge Carbonell is correct
Held:
Judge Carbonell committed grave
abuse of discretion. The leading
case of Soliven v. Makasiar, the
Court explained that this
constitutional provision does not
mandatorily require the judge to
personally examine the
complainant and her witnesses.
Instead, he may opt to personally
evaluate the report and supporting
documents submitted by the
prosecutor or he may disregard
the prosecutors report and require
the submission of supporting
affidavits of witnesses. Before
issuing warrants of arrest, judges
merely determine the probability,
not the certainty, of guilt of an
accused. In doing so, judges do not
conduct a de novo hearing to
determine the existence of
probable cause. They just

personally review the initial


determination of the prosecutor
finding a probable cause to see if it
is supported by substantial
evidence. There are cases where
the circumstances may call for the
judges personal examination of the
complainant and his witnesses. But
it must be emphasized that such
personal examination is not
mandatory and indispensable in
the determination of probable
cause for the issuance of a warrant
of arrest. The necessity arises only
when there is an utter failure of
the evidence to show the existence
of probable cause. Otherwise, the
judge may rely on the report of the
investigating prosecutor, provided
that he likewise evaluates the
documentary evidence in support
thereof. What the law requires as
personal determination on the part
of the judge is that he should not
rely solely on the report of the
investigating prosecutor.

Soliven v. Makasiar, G.R. Nos. L82585, L-82827, November 14,


1988
Facts:
Luis Beltran is among the
petitioners in this case. He,
together with others, was charged
with libel by the then president
Corzaon Aquino. Cory herself filed
a complaint-affidavit against him
and others. Makasiar averred that
Cory cannot file a complaint
affidavit because this would defeat
her immunity from suit.
Issue:
WON the constitutional rights of
Beltran were violated when
respondent RTC judge issued a
warrant for his arrest without
personally examining the
complainant and the witnesses, if
any, to determine probable cause
Held:
The case calls for an interpretation
of the constitutional provision on
the issuance of warrants of arrest.
The addition of the word
"personally" after the word
"determined" and the deletion of
the grant of authority by the 1973
Constitution to issue warrants to
"other responsible officers as may
be authorized by law," has
apparently convinced petitioner
Beltran that the Constitution now
requires the judge to personally
examine the complainant and his
witnesses in his determination of
probable cause for the issuance of
warrants of arrest. This is not an
accurate interpretation. What the
Constitution underscores is the

exclusive and personal


responsibility of the issuing judge
to satisfy himself of the existence
of probable cause. In satisfying
himself of the existence of
probable cause for the issuance of
a warrant of arrest, the judge is
not required to personally examine
the complainant and his witnesses.
Following established doctrine and
procedure, he shall:
(1) personally evaluate the report
and the supporting documents
submitted by the fiscal regarding
the existence of probable cause
and, on the basis thereof, issue a
warrant of arrest; or (2) if on the
basis thereof he finds no probable
cause, he may disregard the
fiscal's report and require the
submission of supporting affidavits
of witnesses to aid him in arriving
at a conclusion as to the existence
of probable cause. Sound policy
dictates this procedure, otherwise
judges would be unduly laden with
the preliminary examination and
investigation of criminal
complaints instead of
concentrating on hearing and
deciding cases filed before their
courts.
It has not been shown that
respondent judge has deviated
from the prescribed procedure.
Thus, with regard to the issuance
of the warrants of arrest, a finding
of grave abuse of discretion
amounting to lack or excess of
jurisdiction cannot be sustained.

Malacat vs CA
G.R. No. 123595, December 12,
1997
Topic: Arrest (Rule 113)
Facts:
Sammy Malacat y Mandar was
charged with violating Section 3 of
Presidential Decree No. 1866 due
to keeping, possessing and/or
acquiring a hand grenade, without
first securing the necessary license
and/or permit therefor from the
proper authorities.
The petitioner was arrested and
searched without a warrant by
police officers who were
conducting foot patrols in Quiapo,
Manila in response to bomb
threats reported earlier. The Police
alleged petitioner and company
were acting suspiciously with their
eyes moving very fast, and that
they fled when a policeman
approached. Furthermore,
petitioner and Casans confessions
were taken by the police even
without the assistance of counsel.

Issue:
Whether or not the search is valid.
Held:
No. The Court held that for failure
to prove that the warrantless
arrest falls under the exception
provided for by Section 5, Rule
113 of the Rules of Court and all
under instances where warrantless
arrest is allowed, the accused may
be acquitted. In a stop and frisk
search, when the warrantless
arrest is invalid, plainly, the search
conducted on the accused could
not have been one incidental to a
lawful arrest. Also, statements and
admission made by an accused
absent the written waiver of his
rights to remain silent and counsel
shall be inadmissible.
People vs Racho
G.R. No. 186529, August 3,
2010
Topic: Arrest (Rule 113)
Facts:
A buy-bust operation was
conducted by the Philippine
National Police, Philippine Army
intelligence group and the
Philippine Drug Enforcement
Agency in order to apprehend
Racho. A confidential agent of the
police transacted through cellular
phone with appellant for the
purchase of shabu , and gave the
authorities Rachos name and
description.
When appellant alighted from the
bus, the confidential agent pointed
to him as the person he transacted
with earlier. Having alighted from
the bus, appellant stood near the

highway and waited for a tricycle


that would bring him to his final
destination. As appellant was
about to board a tricycle, the team
approached him and invited him to
the police station on suspicion of
carrying shabu. Appellant
immediately denied the
accusation, but as he pulled out his
hands from his pants pocket, a
white envelope slipped therefrom
which, when opened, yielded a
small sachet containing the
suspected drug.
Issue:
Whether or not the arrest is valid.
Held:
No. For a warrantless arrest of a
person caught in flagrante delicto
under paragraph (a) of Section 5
to be valid the person to be
arrested must execute an overt act
indicating that he has just
committed, is actually committing,
or is attempting to commit a crime
and such overt act is done in the
presence or within the view of the
arresting officer.
People vs Laguio
G.R. No. 128587, March 16,
2007
Topic: Arrest (Rule 113)
Facts:
Private respondent Lawrence
Wang was charged with violation
of Dangerous Drugs Act, Illegal
possession of firearms, and
violation of the COMELEC gun
ban. During his arraignment, Wang
refused to enter a plea to all the
Informations and instead

interposed a continuing objection


to the admissibility of the evidence
obtained by the police operatives.
Thus, the trial court ordered that a
plea of "Not Guilty" be entered for
him.
Issue:
Whether or not the arrest and
search are lawful.
Held:
No. The Court held that the
accused-appellant was not, at the
moment of his arrest, committing a
crime nor was it shown that he
was about to do so or that he had
just done so. Further, the Peoples
contention that Wang waived his
right against unreasonable search
and seizure has no factual basis.
For a warrantless arrest of a
person caught in flagrante delicto
under paragraph (a) of Section 5
to be valid the person to be
arrested must execute an overt act
indicating that he has just
committed, is actually committing,
or is attempting to commit a crime
and such overt act is done in the
presence or within the view of the
arresting officer.
Zalameda vs People
G.R. No. 183656, September 4,
2009
Topic: Arrest (Rule 113)
Facts:
Petitioner Gilbert Zalameda was
charged with violating R.A. 9165
or the Comprehensive Dangerous
Drugs Act due to possession of
shabu and drug paraphernalia.
Zalameda was arrested as a result

of an operation of the police in


order to verify a report that they
received regarding the use of
illegal drugs at a barangay in
Makati City. The Police peeped
inside and saw the petitioner and
Villaflor sniffing smoke and
proceeded with the arrest and
search.
Issue:
Whether or not the arrest and
search are valid.
Held:
Yes. The Court held that the
records of the case records
support the conclusion that a
lawful arrest, search and seizure
took place, and that the
prosecution fully discharged its
burden of establishing all the
elements necessary for conviction
for the crimes charged beyond
reasonable doubt. The prosecution
successfully established that the
petitioner was arrested in
flagrante delicto.
People vs Alunday
G.R. No. 181546, September 3,
2008
Topic: Arrest (Rule 113)
Facts:
Ricardo Alunday was found guilty
of violation of Section 9, Republic
Act No. 6425, otherwise known as
the Dangerous Drugs Act of 1972.
He was arrested due to cultivation
and culturing of marijuana fruiting
tops weighing more than 750
grams, with an estimated value of
ten million pesos (P10,000,000)
and possession of an M16 rifle

without a permit in Sadanga,


Mountain Province.
Issue:
Whether or not the arrest is valid.
Held:
The accused-appellant went into
arraignment and entered a plea of
not guilty and thereafter, actively
participated in his trial. He raised
the additional issue of irregularity
of his arrest only during his appeal
to this Court. He is, therefore,
deemed to have waived such
alleged defect by submitting
himself to the jurisdiction of the
court by his counsel-assisted plea
during his arraignment; by his
actively participating in the trial
and by not raising the objection
before his arraignment. Section
5(a) provides that a peace officer
or a private person may, without a
warrant, arrest a person when, in
his presence, the person to be
arrested has committed, is actually
committing, or is attempting to
commit, an offense. Section 5(a)
refers to arrest in flagrante delicto.
In flagrante delicto means caught
in the act of committing a crime.
This rule, which warrants the
arrest of a person without warrant,
requires that the person arrested
has just committed a crime, or is
committing it, or is about to
commit an offense, in the presence
or within view of the arresting
officer.
People vs Cabugatan
G.R. No. 172019, February 12,
2007
Topic: Arrest (Rule 113)

Facts:
Cabugatan was found guilty of
illegal sale and of illegal
possession of methamphetamine
hydrochloride or shabu.
An informant advised the Police
that Cabugatan was engaged in
the illegal sale of shabu in Baguio
City. The Police then immediately
formed a team to conduct a buybust operation to apprehend
Cabugatan. He was then arrested
and searched upon entrapment.
Issue:
Whether or not the arrest was
valid.
Held:
Yes. The Court held that an arrest
made after an entrapment does not
require a warrant in as much as it
is considered a valid warrantless
arrest pursuant to Rule 113,
Section 5(a) of the Rules of Court.
People vs Nuevas
G.R. No. 170233, February 22,
2007
Topic: Arrest (Rule 113)
Facts:
Nuevas was charged with illegal
possession of marijuana in
violation of Section 8, Article II of
Republic Act No. 6425. The Police
conducted a stationary
surveillance and monitoring of
illegal drug trafficking along a
street in Olongapo City due to intel
that a man would make a delivery
of dried marijuana leaves in the
said vicinity. While stationed
thereat, the policemen saw a male
person who fit the description

provided by the informant,


carrying a plastic bag, later
identified as Nuevas, alight from a
motor vehicle. They accosted
Nuevas and informed him that
they are police officers. Nuevas
then tried to cooperate with the
policemen to evade charges.

later taken to the NBI laboratory


for examination. When they were
verified as marijuana leaves, an
information for violation of the
Dangerous Drugs Act was filed
against him. The PC officers were
acting upon a tip that they
received earlier from informants

Issue:
Whether or not the arrest is valid.

Issue:
Whether or not the arrest and
search are valid.

Held:
No. The Court held that reliable
information alone is not sufficient
to justify a warrantless arrest
under Section 5(a), Rule 113;
Accused must perform some overt
act that would indicate that he has
committed, is actually committing
or is attempting to commit an
offense.

Held:
No. The Court ruled that
warrantless arrests allowed under
Rule 113 of the Rules of Court not
justified unless the accused was
caught in flagrante or a crime was
about to be committed or had just
been committed. The evidence of
probable caused should be
determined by a judge and not by
law-enforcement agents.
People vs Molina
G.R. No. 133917, February 19,
2001

People vs Aminnudin
G.R. No. 74869, July 6, 1988
Topic: Arrest (Rule 113)
Facts:
Idel Aminnudin was arrested
shortly after disembarking from
the M/V Wilcon 9 in Iloilo City. The
Philippine Constabulary officers
who were in fact waiting for him
simply accosted him, inspected his
bag and finding what looked like
marijuana leaves took him to their
headquarters for investigation.
The two bundles of suspect articles
were confiscated from him and

Topic: Arrest (Rule 113)


Facts:
Appellants were found guilty of
violation of R.A. 6425 or the
Dangerous Drugs Act of 1972 as
amended. Appellants were
apprehended as a consequence of
the Police acting upon an
information regarding the
presence of an alleged marijuana
pusher in Davao City. The
policemen waited for the
appellants to pass by a certain
road and asked them to stop.
Marijuana was then found in the
possession of the appelants.

Issue:
Whether or not the arrest and
search are valid.
Held:
No. The Court held that it is
settled that reliable information
alone, absent any overt act
indicative of a felonious enterprise
in the presence and within the
view of the arresting officers, are
not sufficient to constitute
probable cause that would justify
an in flagrante delicto arrest.
People vs Mengote
G.R. No. 87059, June, 1992
Facts:
Accused-appellant Rogelio
Mengote was convicted of illegal
possession of firearms on the
strength mainly of the stolen pistol
found on his person at the moment
of his warrantless arrest.
The Western Police District
received a telephone call from an
informer that there were three
suspicious-looking persons in
Tondo. Thereafter, the Police
dispatched a surveillance team
which arrested and searched
Mengote because he tried to run
when approached by the former as
he was seen "looking from side to
side" and holding his abdomen.
Issue:
Whether or not the arres and
search are valid.
Held:
No. The Court held that a
warrantless search in broad
daylight of a person merely looking

from side to side and holding his


stomach is illegal.
People vs Anita Claudio
G.R. No. 72564, April 15, 1988
Topic: Arrest (Rule 113)
Facts:
Claudio transported 1.1 kilos of
Marijuana dried leaves, which are
prohibited drugs for the purpose of
selling the same from Baguio City
to Olongapo City. Daniel Obia, a
policeman assigned to the antinarcotics unit, was on the same
bus as Claudio when the formers
suspicion was aroused by the
letter, prompting him to search the
latters bag. Upon the search, the
policeman inserted one of his
fingers in a plastic bag located at
the bottom of the woven bag and
smelt marijuana. The policeman
did not, however, do anything after
he discovered that there was
marijuana inside the plastic bag of
the accused until they reached
Olongapo City and the accused
alighted from the bus in front of a
Gasoline Station. Right after the
accused alighted from the bus,
policeman Obina intercepted her
and showed her his Id Identifying
himself as a policeman and told
her he will search her bag because
of the suspicion that she was
carrying marijuana inside said bag.
Issue:
Whether or not the search is valid.
Held:
The Court upheld the validity of
the arrest and the search

incidental to it. The warrantless


search being an incident to a
lawful arrest is in itself lawful; No
infirmity in the seizure of the 1.1
kilos of marijuana.
People vs Tangliben
G.R. No. L-63630, April 6, 1990
Topic: Arrest (Rule 113)
Facts:
Policemen, together with a
Barangay Tanod, were conducting
a surveillance mission at the
Victory Liner Terminal compound.
The surveillance was aimed not
only against persons who may
commit misdemeanors at the said
place but also on persons who may
be engaging in the traffic of
dangerous drugs based on
informations supplied by
informers. One evening, said
Patrolmen noticed a person caring
a traveling bag who was acting
suspiciously and they confronted
him. The person was requested by
the policemen to open the red
traveling bag but the person
refused, only to accede later on
when the patrolmen identified
themselves. They found marijuana
leaves wrapped in a plastic
wrapper and weighing one kilo,
more or less inside the bag. The
person was asked of his name and
the reason why he was at the said
place and he gave his name as
Medel Tangliben and explained
that he was waiting for a ride to
Olongapo City to deliver the
marijuana leaves
Issue:

Whether or not the arrest and


search are valid.
Held:
Yes. The Court held that the
accused was caught in flagrante,
since he was carrying marijuana at
the time of his arrest. This case
therefore falls squarely within the
exception. The warrantless search
was incident to a lawful arrest and
is consequently valid.
People vs Maspil
G.R. No. 85177, August 20,
1990
Topic: Arrest (Rule 113)
Facts:
By the virtue of the report coming
from the informants, that two
persons would be transporting a
large quantity of marijuana, Police
officers set up a check point in
Benguet to monitor vehicles bound
for Baguio City. After hours of
waiting, a jeepney was flagged
down in the checkpoint and the
informers identified the two. When
they inspected the jeepney, there
were sacks and cans inside it
containing marijuana leaves. The
two were placed under arrest.
Issue:
Whether or not the Arrest is valid.
Held:
Yes. The court again made a
distinction between the Maspil and
Aminnudin case. In the latter, the
officers were aware of the identity
of the accused, his plans and the
vessel that he would be taking.
Further, the officers have ample of

time to obtain a warrant but failed


to do so. In the former, the officers
had no exact description of the
vehicle of the accused and no idea
as to what the vehicle would
exactly arrive.
The court explained that the
jeepney on the road is not the
same as the passenger boat the
route and the time of arrival of
which are more or less certain and
which ordinarily cannot deviate or
alter its course or select another
destination.
People vs Agojo
G.R. No. 181318, April 16, 2009
Topic: Arrest (Rule 113)
Facts:
Appellant was charged with illegal
sale of shabu and illegal
possession of firearms.
A civilian informant reported the
drug trading activities of appellant
to Police which then formed a
team to conduct a buy-bust
operation. Agojo was apprehended
during the operation after he
received the marked money and
gave a VHS box containing 4
sachets of shabu.
Issue:
Whether or not the arrest is valid.
Held:
Yes. The Court held that there are
two stringent requirements before
a warrantless arrest can be
effected under the second instance
covered by paragraph (b). The
second instance of lawful
warrantless arrest covered by
paragraph (b) cited above

necessitates two stringent


requirements before a warrantless
arrest can be effected: (1) an
offense has just been committed;
and (2) the person making the
arrest has personal knowledge of
facts indicating that the person to
be arrested has committed it. A
review of the records shows that
both requirements were met in
this case.
Abelita vs Doria
G.R. No. 170672, August 14,
2009
Topic: Arrest (Rule 113)
Facts:
The police received information
about a shooting incident involving
the petitioner. The police then
dispatched a team headed by SPO3
Ramirez to investigate the
incident. SPO3 Ramirez later
reported that a certain William Sia
was wounded while petitioner, who
was implicated in the incident, and
his wife just left the place of the
incident. Doria looked for
petitioner and when he found him,
he informed him of the incident
report. Doria requested petitioner
to go with him to the police
headquarters as he was reported
to be involved in the
incident. Petitioner agreed but
suddenly sped up his vehicle and
proceeded to his residence. Doria
and his companions chased
petitioner. Upon reaching
petitioners residence, they caught
up with petitioner as he was about
to run towards his house. The
police officers saw a gun in the
front seat of the vehicle beside the

drivers seat as petitioner opened


the door. They also saw a shotgun
at the back of the drivers seat. The
police officers confiscated the
firearms and arrested
petitioner. P/Supt. Doria alleged
that his men also arrested other
persons who were identified to be
with petitioner during the shooting
incident. Petitioner was charged
with illegal possession of firearms
and frustrated murder.
Issue:
Whether or not the arrest and
search are valid.
Held:
Yes. The Court upheld the validity
of the arrest and seach. Personal
knowledge of facts must be based
on probable cause, which means
an actual belief or reasonable
grounds of suspicion; A reasonable
suspicion, therefore, must be
founded on probable cause,
coupled with good faith on the part
of the peace officers making the
arrest.
People vs Acol
G.R. No. 106288-89, May 17,
1994
Topic: Arrest (Rule 113)
Facts:
In the morning of September 29,
1990, when Percival Tan was
driving his jeepney, two men
boarded the vehicle in Cubao.
When they crossed Pasay Road,
the two wayfarers, together with
two other companions, announced
a hold-up. Percival Tan was

instructed to proceed atop an


interchange where the other
passengers were divested of their
personal belongings, including the
jacket of passenger Rene Araneta.
Thereafter, the robbers alighted,
after which Percival Tan and his
passengers went to Fort Bonifacio
to report the crime. A CAPCOM
team was forthwith formed to
track down the culprits. Victim
Rene Araneta who went with the
responding police officers, upon
seeing four persons, one of whom
was wearing his stolen jacket,
walking casually towards Fort
Bonifacio, told the police
authorities to accost said persons.
After the CAPCOM officers
introduced themselves, the four
men scampered to different
directions but three of them,
namely, Tirso Acol, Pio Boses, and
Albert Blanco, were apprehended.
Tirso Acol and Pio Boses were
each found in possession of an
unlicensed .38 caliber revolver
with bullets. After the arrest, the
three men were brought to Fort
Bonifacio and were identified by
Percival Tan and the passengers
who ganged up on the accused.
Issue:
Whether or not the arrest and
search are valid.
Held:
Yes. The Court held that a
warrantless arrest and seizure was
valid where it was done by a police
team dispatched to look for
persons responsible for the crime
based on information related by
victims who have just been robbed.

People vs Gerente
G.R. No. 95847-88, March 10,
1993
Topic: Arrest (Rule 113)
Facts:
Gerente was convicted for
violation of Dangerous Drugs Act
and Murder.
The Police received s report about
a mauling incident which led to the
death of the victim. Thereafter, the
police proceeded to where the
mauling incident took place. There
they found a piece of wood with
blood stains, a hollow block and
two roaches of marijuana. They
were informed by the prosecution
witness, Edna Edwina Reyes, that
she saw the killing and she pointed
to Gabriel Gerente as one of the
three men who killed Clarito.
Issue:
Whether or not the arrest and
search are lawful.
Held:
Yes. The policemen arrested
Gerente only some three (3) hours
after Gerente and his companions
had killed Blace. They saw Blace
dead in the hospital and when they
inspected the scene of the crime,
they found the instruments of
death: a piece of wood and a
concrete hollow block which the
killers had used to bludgeon him to
death. The eye-witness, Edna
Edwina Reyes, reported the
happening to the policemen and
pinpointed her neighbor, Gerente,
as one of the killers. Under those
circumstances, since the
policemen had personal knowledge

of the violent death of Blace and of


facts indicating that Gerente and
two others had killed him, they
could lawfully arrest Gerente
without a warrant. If they had
postponed his arrest until they
could obtain a warrant, he would
have fled the law as his two
companions did. The search
conducted on Gerente's person
was likewise lawful because it was
made as an incident to a valid
arrest.
People vs Martinez
G.R. No. 191366, December 13,
2010
Topic: Arrest (Rule 113)
Facts:
A concerned citizen entered the
precinct and reported that a pot
session was going on in the house
of one of the accused-appellants in
Dagupan City. Upon receipt of the
report, PO1 Azardon, PO1
Alejandro Dela Cruz and members
of the Special Weapons and
Tactics (SWAT) team located the
house where the said pot session
was held. As the police officers
entered the gate of the house, they
saw accused Orlando
Doria (Doria) coming out of the
side door and immediately
arrested him. Inside the house,
they saw accused Gonzales, Arnold
Martinez, Edgar Dizon, and Rezin
Martinez in a room. The four were
surprised by the presence of the
police. In front of them were open
plastic sachets (containing shabu
residue), pieces of rolled used
aluminum foil and pieces of used
aluminum foil.

Issue:
Whether or not the accused is
estopped from assailing the
legality of his arrest.
Held:
The Court held that the accused is
estopped from assailing the
legality of his arrest if he fails to
raise such issue before
arraignment. However, this waiver
is limited only to the arrest. The
legality of an arrest affects only
the jurisdiction of the court over
the person of the accused. A
waiver of an illegal warrantless
arrest does not carry with it a
waiver of the inadmissibility of
evidence seized during the illegal
warrantless arrest.
People vs Aminola
G.R. No. 178062, September 8,
2010
Topic: Arrest (Rule 113)
Facts:
Nestor Gabuya was closing up
shop at his motorcycle and bicycle
spare parts store located in
Taguig, he then headed home on
his bike. Unbeknownst to him,
accused-appellant Abdul Aminola
and accused Alimudin Laminda
were observing him from a nearby
basketball court. Aminola
proceeded to follow Gabuya. Upon
catching up with Gabuya, Aminola
put his arms around Gabuya and
wrestled for the bag Gabuya was
carrying. Gabuya refused to let go
of his bag, whereupon Aminola
pulled out a gun and shot
him. Gabuya fell to the ground but

still resisted, prompting Aminola to


take another shot. Later that
evening, an informant known told
the police that he witnessed what
had happened to Gabuya and could
tell them where the suspects could
be found. True enough, he led
Major Migano and his men. A
blocking force was organized while
Col. Bernido formed a team to
make the arrests on the suspects.
Issue:
Whether or not the warrantless
arrest is a jurisdictional defect in
this case.
Held:
The Court held that The CA
correctly ruled on the question of
legality of the warrantless arrests
of accused-appellants. A
warrantless arrest is not a
jurisdictional defect and any
objection to it is waived when the
person arrested submits to
arraignment without any objection.
People vs Tan
G.R. No. 191069, November 15,
2010
Topic: Arrest (Rule 113))
Facts:
The Police conducted a manhunt
operation against a suspect in a
robbery case involving Korean
nationals in Makati City. While on
board their civilian vehicle, they
chanced upon a male individual
selling certain items to two
foreigners. They heard him say,
Hey Joe, want to buy Valium 10,
Cialis, Viagra? Curious, they
inquired and the male individual
told them that he was selling

Viagra and Cialis, while, at the


same time, showing them the
contents of his bag which yielded
120 tablets of Valium 10.
The male individual, who later
turned out to be Sonny Boy, was
immediately searched and placed
under arrest, after which they
informed him of the nature of his
apprehension and of his
constitutional rights. Sonny Boy
was then brought to the office of
the Station Anti-Illegal Drugs
Special Operations Task Force,
where the items recovered from
him were marked and inventoried.
Issue:
Whether or not the arrest and
search are valid.
Held:
Yes. The Court held that all
procedural irregularities in the
arrest of accused must be raised at
his arraignment.
Dolera vs Peopla
G.R. No. 180693, September 4,
2009
Topic: Arrest (Rule 113)
Facts:
Dolera was charged with the
violation of the Comprehensive
Dangerous Drugs Act of 2002. The
Police conducted a surveillance
operation in Barangay Payatas,
Quezon City after receiving a
report of drug trafficking in the
said area. While at the target area,
PO2 Labon saw petitioner standing
near an alley, scrutinizing a
transparent plastic sachet
containing white crystalline

substance. PO2 Labon, who was in


civilian clothes, thus alighted from
the vehicle, followed by PO1
Penalosa, and approached
petitioner. After introducing
himself as a policeman, PO2 Labon
asked petitioner what he was
holding, but the latter, who
appeared "natulala," did not reply.
Suspecting that the white
crystalline substance inside the
plastic sachet was shabu, PO2
Labon confiscated the same and
handcuffed petitioner. PO1
Pealosa then frisked petitioner
and recovered a heat-sealed
plastic sachet also containing
white crystalline substance from
the right front pocket of
petitioners pants. After informing
him of his constitutional rights,
petitioner was brought to the
police station for further
investigation.
Issue:
Whether or not the arrest is valid.
Held:
The Court has consistently ruled
that any objection involving a
warrant of arrest or the procedure
for the acquisition by the court of
jurisdiction over the person of the
accused must be made before he
enters his plea; otherwise, the
objection is deemed waived.
Eugenio vs People
G.R. No. 168163, March 26,
2008
Topics: Arrest (Rule 113)
Facts:

The National Bureau of


Investigation (NBI) arranged an
entrapment operation in the house
of a private complainant due to the
crime of Estafa thru Falsification of
Public Documents committed by
the petitioner in transactions
involving parcels of land. Mangali
expected to see petitioner that day
as petitioner had asked for an
additional P33,000 loan. At the
appointed time, petitioner, Ablaza,
and two other unidentified
individuals arrived in Mangalis
house. The NBI agents effected the
arrest while petitioner was
counting the money.
Issue:
Whether irregularities attended
petitioners arrest and
investigation, nullifying her
conviction.
Held:
The Court held that any
irregularity attending the arrest of
an accused, depriving the trial
court of jurisdiction over her
person, should be raised in a
motion to quash at any time before
entering her plea, and failure to
timely raise this objection amounts
to a waiver of such irregularity,
resulting in concomitant
submission to the trial courts
jurisdiction over her person.

Facts:
Walking along the C-5 bridge, AAA
was forcibly brought to a dark
place under the bridge by the
appellant and thereafter, robbed
her and raped her.
While conducting their routine
patrol, members of the
barangay security force chanced
upon appellant, whom they found
sleeping, using several ladies
wallets as pillows, under the C-5
bridge, near the place where AAA
was raped. It appearing that
appellant was drunk and recalling
the rape incident that occurred a
few days earlier, the
barangay security force brought
appellant to the Barangay Hall for
verification. That same day, AAA
positively identified appellant as
her assailant.
Issue:
Whether or not the arrest is valid.
Held:
Yes. The Court held that an
accused is estopped from assailing
the legality of his arrest if he fails
to raise this issue, or to move for
the quashal of the information
against him on this ground, before
arraignment.
Leviste vs Alameda
G.R. No. 182677, August 3,
2010
Topic: Arrest (Rule 113)

People vs Nelson Palma


G.R. No. 189279, March 9, 2010
Topic: Arrest (Rule 113)

Facts:
Petitioner was, by Information,
charged with homicide for the
death of Rafael de las Alas
on January 12, 2007 before the

Regional Trial Court (RTC)


of Makati City. Branch 150 to
which the case was raffled,
presided by Judge Elmo Alameda,
forthwith issued a commitment
order against petitioner who was
placed under police custody while
confined at the Makati Medical
Center.
After petitioner posted a P40,000
cash bond which the trial court
approved, he was released from
detention, and his arraignment
was set.
The private complainants-heirs of
De las Alas filed, with the
conformity of the public
prosecutor, an Urgent Omnibus
Motion praying, inter alia, for the
deferment of the proceedings to
allow the public prosecutor to reexamine the evidence on record or
to conduct a reinvestigation to
determine the proper offense.
Issue:
Whether or not parties are
precluded from questioning the
legality of the arrest or lack of or
irregular preliminary investigation.
Held:
The Court held that before the
filing of complaint or information
in court, the private complainant
may proceed in coordinating with
the arresting officer and the
inquest officer during the latters
conduct of inquest, while the
arrested person has the option to
avail of a 15-day preliminary
investigation, provided he duly
signs a waiver of any objection

against delay in his delivery to the


proper judicial authorities under
Article 125 of the Revised Penal
Code; The accelerated process of
inquest, owing to its summary
nature and the attendant risk of
running against Article 125, ends
with either the prompt filing of an
information in court or the
immediate release of the arrested
personthe rules on inquest do
not provide for a motion
for reconsideration.

Rebellion vs People
G.R. No. 175700, July 5, 2010
Topic: Arrest (Rule 113)
Facts:
Policemen together with members
of the Mayors Action Command
(MAC) were on routine patrol
when they chanced upon two
individuals chanting and in the act
of exchanging something. The
police officers introduced
themselves and then inquired from
petitioner what he was
holding. Petitioner took out from
his possession three strips of
aluminum foil which PO3 Garcia
confiscated. PO3 Sotomayor also
found on petitioner a plastic sachet
which contained white crystalline
substance which looked
like tawas. Suspecting that the
substance was shabu, he
confiscated the plastic

sachet. Petitioner and his


companion, who was later
identified as Clarito Yanson
(Clarito), were brought to the MAC
station at the Criminal
Investigation Division (CID) for
investigation.
Issue:
Whether the facts presented in this
case make out a legitimate
instance of a warrantless arrest.
Held:
Yes. The Court held that an
accused is estopped from assailing
any irregularity of his arrest if he
fails to raise this issue or to move
for the quashal of the information
against him on this ground before
arraignmentany objection
involving a warrant of arrest or the
procedure by which the court
acquired jurisdiction over the
person of the accused must be
made before he enters his plea;
otherwise, the objection is deemed
waived.
People vs Jalosjos
G.R. No. 132875-76, February
3, 2000
Topic: Arrest (Rule 113)
Facts:
The accused-appellant, Romeo G.
Jalosjos, a full-fledged member of
Congress, is confined at the
national penitentiary while his
conviction for statutory rape on
two counts and acts of
lasciviousness on six counts is
pending appeal. The accusedappellant filed a motion asking
that he be allowed to fully
discharge the duties of a

Congressman, including
attendance at legislative sessions
and committee meetings despite
his having been convicted in the
first instance of a non-bailable
offense.
Issue:
Whether or not membership in
Congress exempt an accused from
statutes and rules which apply to
validly incarcerated persons in
general.
Held:
No. The Court held that the
privilege of a senator or a
congressman will not apply when
the offense is punishable by
imprisonment of more than six (6)
years even if Congress is in
session.
Unilab vs Isip
G.R. No. 163858, June 28, 2005
Topic: Searches and Seizures
(Rule 126)
Facts:
The National Bureau of
Investigation (NBI) obtained a
search warrant from RTC Manila
for the search of first and second
floors of the Shalimar Building
where there was allegedly
manufacture, production and/or
distribution of fake drug products
such as Revicon by Shalimar
Philippines. The respondents filed
an Urgent Motion to Quash the
Search Warrant or to Suppress
Evidence. They contended that the
implementing officers of the NBI
conducted their search at the first,
second, third and fourth floors of

the building at No. 1524-A, Lacson


Avenue, Sta. Cruz, Manila, where
items in open display were
allegedly found. They pointed out,
however, that such premises was
different from the address
described in the search warrant,
the first and second floors of the
Shalimar Building located at No.
1571, Aragon Street, Sta. Cruz,
Manila. The respondents, likewise,
asserted that the NBI officers
seized Disudrin and Inoflox
products which were not included
in the list of properties to be
seized in the search warrant.
Issue:
Whether the petitioner is the
proper party to file the petition at
bench.
Held:
Yes, the Court allowed the
petitioner to file such petition and
elucidated that a search warrant
proceeding is, in no sense, a
criminal action or the
commencement of a prosecution
it is a special and peculiar remedy,
drastic in nature, and made
necessary because of public
necessity, resembling in some
respect with what is commonly
known as John Doe proceedings.
A search warrant is a legal process
which has been likened to a writ of
discovery employed by the State to
procure relevant evidence of crime
it is in the nature of a criminal
process, restricted to cases of
public prosecutions and not a
process for adjudicating civil
rights or maintaining mere private
rights.

Skechers, U.S.A., Inc. vs. Inter


Pacific Industrial Trading Corp
G.R. No. 164321, March 23,
2011
Topic: Searches and Seizures
(Rule 126)
Facts:
Petitioner filed with the RTC of
Manila an application for the
issuance of search warrants
against an outlet and warehouse
operated by respondents for
infringement of trademark under
Republic Act No. 8293, otherwise
known as the Intellectual Property
Code of the Philippines. In the
course of its business, petitioner
has registered the trademark
"SKECHERS" and the trademark
"S" (within an oval design) with
the Intellectual Property Office
(IPO).
Two search warrants were issued
by the RTC and were served on the
premises of respondents. As a
result of the raid, more than 6,000
pairs of shoes bearing the "S" logo
were seized.
Later, respondents moved to quash
the search warrants, arguing that
there was no confusing similarity
between petitioners "Skechers"
rubber shoes and its "Strong"
rubber shoes.
The RTC issued an Order quashing
the search warrants and directing
the NBI to return the seized goods.
The RTC agreed with respondents
view that Skechers rubber shoes
and Strong rubber shoes have
glaring differences such that an
ordinary prudent purchaser would
not likely be misled or confused in
purchasing the wrong article.

Issue:
Whether or not the trial court
judge committed a grave abuse of
discretion in granting the motion
to quash upon re-evaluation of
evidence.
Held:
No. The power to issue search
warrants is exclusively vested with
the trial judges in the exercise of
their judicial function; After the
judge had issued a warrant, he is
not precluded to subsequently
quash the same, if he finds upon
re-evaluation of the evidence that
no probable cause exists.
Borlongan, Jr. vs. Pea
G.R. No. 143591, May 5, 2010
Topic: Searches and Seizures
(Rule 126)
Facts:
Petitioners were charged with
violation of par. 2, Article 172 of
the Revised Penal Code or
Introduction of Falsified Document
in a judicial proceeding in relation
to a a civil case for recovery of
agents compensation and
expenses, damages, and attorneys
fees.
Petitioners contend that they were
denied due process as they were
unable to submit their counteraffidavits and were not accorded
the right to a preliminary
investigation.
Issue:
Whether or not the decision of the
Court of Appeals is correct when

it dismissed the petition for


certiorari filed by petitioners and
ruled that the MTCC did not
gravely abuse its discretion in
denying the motion for
reinvestigation and recall of the
warrants of arrest.
Held:
The judges personal examination
depends on the circumstances of
each case, to be sure, he cannot
just rely on the bare certification
alone but must go beyond it. This
is because the warrant of arrest
issues not on the strength of the
certification standing alone but
because of the records which
sustain it.
Marimla vs People
G.R. No. 158467, October 16,
2009
Topic: Searches and Seizures
(Rule 126)
Facts:
The NBI Anti-Organized Crime
Division filed two (2) applications
for search warrant with the RTC of
Manila seeking permission to
search: (1) petitioners house
located in Angeles City and (2) the
premises on Maria Aquino St. in
Porac, Pampanga, both for
Violation of Republic Act (R.A.) No.
6425, as amended. The said
applications uniformly alleged that
Special Investigator Lagascas
request for the issuance of the
search warrants was founded on
his personal knowledge as well as
that of witness Roland D.
Fernandez (Fernandez), obtained
after a series of surveillance

operations and a test buy made at


petitioners house.
Executive Judge Mario Guaria III
(Judge Guaria III) examined in
writing and under oath SI Lagasca
and Fernandez, in the form of
searching questions and answers.
The judge was satisfied that there
was probable cause and issued a
search warrant.
Issue:
Whether or not the court which
issued the questioned search
warrant committed grave abuse of
discretion when it issued the same
because under the law it cannot
issue a search warrant outside its
territorial jurisdiction.
Held:
No. The Court held that A.M. No.
99-10-09-SC authorizes the
Executive Judge and Vice
Executive Judges of the Regional
Trial Courts (RTCs) of Manila and
Quezon City to act on all
applications for search warrants
involving heinous crimes, illegal
gambling, dangerous drugs and
illegal possession of firearms on
application filed by the Philippine
National Police (PNP), National
Bureau of Investigation (NBI),
Presidential Anti-Organized Crime
Task Force (PAOC-TF), and
Reaction Against Crime Task Force
(REACT-TF).
Chemise Lacoste, S.A. vs.
Fernandez
G.R. No. 63796-97, May
21,1984
Topic: Searches and Seizures
(Rule 126)

Facts:
La Chemise Lacoste filed with the
NBI a letter-complaint alleging
acts of unfair competition
committed by Hemandas and
requesting the agencys
assistance. A search warrant was
issued by the trial court. Various
goods and articles were seized
upon the execution of the
warrants. Hemandas filed motion
to quash the warrants, which the
court granted. The search
warrants were recalled, and the
goods ordered to be returned. La
Chemise Lacoste filed a petition
for certiorari.
Issue:
Whether or not an application for
seach warrant is heard ex parte.
Held:
Yes. An application for a search
warrant is heard ex parte. It is
neither a trial nor part of a trial.
Action on these applications must
be expedited for time is of the
essence. Great reliance has to be
accorded by the judge to the
testimonies under oath of the
complainant and the witnesses.
People vs Nuez
G.R. No. 177148, June 30, 2009
Topic: Searches and Seizures
(Rule 126)
Facts:
Operatives of the Sta. Cruz,
Laguna Police Detectives in
coordination with the Los Baos
Police Station (LBPS) and IID
Mobile Force conducted a search

in the house of Raul R. Nuez based


on reports of drug possession.
Before proceeding to appellants
residence, the group
summoned the Barangay Captain
and the Chief Tanod to assist them
in serving the search
warrant. Upon arriving at
appellants house, they called on
appellant to come out. Thereafter,
the Commanding Officer showed
Nuez the warrant. The police
then surveyed appellants room in
his presence while his family, the
others remained in the living
room. There they found thirty-one
(31) packets of shabu, lighters,
improvised burners, tooters, and
aluminum foil with shabu residue
and a ladys wallet
containing P4,610 inside
appellants dresser. The group also
confiscated a component, camera,
electric planer, grinder, drill,
jigsaw, electric tester, and assorted
carpentry tools on suspicion that
they were acquired in exchange
for shabu. Following the search,
SPO1 Ilagan issued a Receipt for
Property Seized and a Certification
of Orderly Search which appellant
signed.
Issue:
Whether or not the search is valid.
Held:
Any objection to the legality of the
search warrant and the
admissibility of the evidence
obtained thereby was deemed
waived when no objection was
raised by appellant during trial.
A search warrant is not a sweeping
authority empowering a raiding
party to undertake a fishing

expedition to confiscate any and all


kinds of evidence or articles
relating to a crime.
Santos vs Pryce Gases, Inc.
G.R. No. 165122, November 23,
2007
Facts:
Pryce Gas sought the assistance of
the Criminal Investigation and
Detection Group (CIDG) to recover
the LPG cylinders allegedly in the
possession of Sun Gas. CIDG
operatives conducted surveillance
on the warehouse of Sun Gas, Inc.
and requested the Bureau of Fire
Protection (BFP) to conduct a
routine fire inspection at Sun Gas,
Inc.s warehouse with some of the
CIDG operatives. The CIDG
operatives entered the warehouse
and were able to take photographs
of the LPG cylinders which were
used as evidence in the application
for a search warrant.
Petitioner filed a Motion
to Quash the search warrant on
the grounds of lack of probable
cause as well as deception and
fraud employed in obtaining
evidence in support of the
application therefor, in violation of
the Rules of Court. Respondent
opposed petitioners Motion to
Quash.
Issue:
Whether or not the search is valid.
Held:
Yes. The Court held that in
quashing the search warrant, it
would appear that the trial court
had raised the standard of
probable cause to whether there

was sufficient cause to hold


petitioner for trial. In so doing, the
trial court committed grave abuse
of discretion.
Requisites for a search warrants
validity, the absence of even one
will cause its downright
nullification: (1) it must be issued
upon probable cause; (2) the
probable cause must be
determined by the judge himself
and not by the applicant or any
other person; (3) in the
determination of probable cause,
the judge must examine, under
oath or affirmation, the
complainant and such witnesses as
the latter may produce; and (4) the
warrant issued must particularly
describe the place to be searched
and persons or things to be seized.
Manly Sportwear
Manufacturing, Inc. vs
Dadodette Enterprises
G.R. No. 165306, September 20,
2005
Topic: Searches and Seizures
(Rule 126)
Facts:
The National Bureau of
Investigation (NBI) applied for a
search warrant before the
Regional Trial Court (RTC) of
Quezon City, based on the
information that Dadodette
Enterprises and/or Hermes Sports
Center were in possession of
goods, the copyright of which
belonged to Manly Sportswear
Mfg., Inc. (MANLY). After finding
reasonable grounds, the judge
issued a search warrant.

Respondents thereafter moved to


quash and annul the search
warrant contending that the same
is invalid since the requisites for
its issuance have not been
complied with. They insisted that
the sporting goods manufactured
by and/or registered in the name
of MANLY are ordinary and
common hence, not among the
classes of work protected under
Section 172 of RA 8293. The
motion to quash was then granted
and the motion for considerqation
of MANLY was denied, hence, it
filed a petition for certiorari.
Issue:
Whether or not the judge
committed a grave abuse of
discretion when she granted the
motion to quash the search
warrant.
Held:
No. The Court held that the trial
court was acting within bounds
when it ruled, in an ancillary
proceeding, that the copyrighted
products of petitioner are not
original creations. This is because
in the determination of the
existence of probable cause for the
issuance or quashal of a warrant, it
is inevitable that the court may
touch on issues properly threshed
out in a regular proceeding. In so
doing, it does not usurp the power
of, much less preclude, the court
from making a final judicial
determination of the issues in a
full-blown trial. Consequently,
MANLYs assertion that the trial
courts order quashing the warrant
pre-empted the finding of the
intellectual property court has no

legal basis. After the judge has


issued a warrant, he is not
precluded to subsequently quash
the same, if he finds upon reevaluation of the evidence that no
probable cause exists.
Kho vs Lanzanas
G.R. No. 150877, May 4, 2006
Topic: Searches and Seizures
(Rule 126)
Facts:
Quintin Cheng registered with the
Bureau of Food and Drugs (BFAD)
as distributor of Chin Chun Su
products. Quintin Cheng
subsequently secured a
supplemental registration for Chin
Chun Su and device. This
supplemental registration was
ordered cancelled by the Bureau of
Patents, Trademarks and
Technology Transfer on the ground
of failure of the registrant to file
the required affidavit of non-use as
required by law.
Notwithstanding this cancellation,
Quintin Cheng executed an
Assignment of a Registered
Trademark and a Supplementary
Deed of Assignment, wherein he
sold all his right, title, interest and
goodwill in the trademark Chin
Chun Su and device to petitioner
Elidad Kho.
Animosity arose between SYCF
and Quintin Cheng resulting in the
termination of their distributorship
agreement.
Consequently, SYCF appointed
respondent Summerville General
Merchandising, represented by
Ang Tiam Chay and Victor Chua,
as its exclusive importer, re-packer

and distributor of Chin Chun Su


products in the Philippines for a
period of five years.
SYCF further executed a Special
Power of Attorney in favor of
Summerville General
Merchandising granting it the
authority to file complaints against
usurpers of Chin Chun Su
trademarks/tradename.
Issue:
Whether or not there is probable
cause to issue a search warrant.
Held:
Yes. The Court held that probable
cause existed for the issuance of
the warrant as shown by the
affidavits of the above affiants who
had personal knowledge of facts
indicating that an offense involving
violation of intellectual property
rights was being committed and
that the objects sought in
connection with the offense are in
the place sought to be
searched. Probable cause means
such reasons, supported by facts
and circumstances as will warrant
a cautious man in the belief that
his action and the means taken in
prosecuting it are legally just and
proper.
Yao, Sr. vs People
G.R. No. 168306, June 19, 2007
Topic: Searches and Seizures
(Rule 126)
Facts:
Petitioners William Yao, Sr. and
several others were incorporators
and officers of Masagana Gas

Corporation. The NBI, acting on


reports that petitioners unlawfully
and in violation of intellectual
property rights of Petron
Corporation and Pilipinas Shell,
produce, sell,distribute LPG
products using LPG cylinders
owned by Petron and Shell and by
virtue of search warrants, raided
the premises of Masagana and
confiscated, among other things,
the motor compressor and refilling
machine owned by Masagana.
Masagana Corporation intervened
in the case and asked for the
return of said pieces of equipment.
It argued that even if the same
was being used by petitioners in
their unlawful activity, the
equipment cannot be confiscated
because having a personality
separate and distinct from that of
its incorporators, directors and
officers, said properties are owned
by the corporation and not by the
petitioners.
Issue:
Whether or not the judge had
sufficient basis in declaring the
existence of probable cause.

G.R. No. 171980, October 6,


2010
Topic: Searches and Seizures
(Rule 126)
Facts:
SPO4 Gotidoc, along with the
members of Intel Operatives of
Tarlac City Police Station and
Philippine Drug Enforcement
Agency (PDEA), implemented a
Search Warrant issued by Judge
Alipio Yumul of the Regional Trial
Court in Capas, Tarlac against the
appellant in her residence at
Barangay Maliwalo, Tarlac City.
Prior to the search, the police
team invited a Barangay
Kagawad of the same barangay to
witness the conduct of the search
and seizure operation in the
appellants house. With a Barangay
Kagawad, the police team
presented the search warrant to
appellant and informed her of the
purpose of the search and her
constitutional rights.

Held:
Yes. The Court held that the
attendant facts and circumstances
were sufficient to establish
probable cause. Although there is
no hard-and-fast rule governing
how a judge should conduct his
investigation, it is axiomatic that
the examination must be probing
and exhaustive not merely
routinary, general, peripheral,
perfunctory or pro forma.

Afterwards, SPO4 Gotidoc, the


designated searcher, started
searching the appellants house, in
the presence of the appellant
and Kagawad Tabamo. During his
search, he found on the top cover
of the refrigerator one (1) plastic
sachet containing white crystalline
substance. The appellant alleged
that the police did not find
anything in the house but instead
planted the shabu there, and
thereafter tried to extort money
from appellant.

People vs. Mamaril

Issue:

Whether or not the judge had


sufficient basis in declaring the
existence of probable cause.
Held:
Yes. The Court held that the
accused-appellant, failed to
present substantial rebuttal
evidence to defeat the
presumption of regularity of duty
of the issuing judge.
A magistrates determination of a
probable cause for the issuance of
a search warrant is paid with great
deference by a reviewing court, as
long as there was substantial basis
for that determination.
Central Bank vs. Morfe
G.R. No. L-20119, June 30,
1967
Topic: Searches and Seizures
(Rule 126)
Facts:
The legal department of the
Central Bank of the Philippines
rendered an opinion to the effect
that the Organization and others of
similar nature are banking
institutions, falling within the
purview of the Central Bank Act.
The Governor of the Bank directed
the coordination of the
investigation and gathering of
evidence on the activities of the
savings and loan associations
which are operating contrary to
law. Soon thereafter, a member of
the intelligence division of the
Bank filed with the Municipal
Court of Manila a verified
application for a search warrant
against the Organization, alleging
that after close observation and

personal investigation, the


premises at No. 2745 Rizal
Avenue, Manila in which the
offices of the Organization were
housed , are being used unlawfully
because said Organization is
illegally engaged in banking
activities by receiving deposits of
money for deposit, disbursement,
safekeeping or otherwise or
transacts the business of a savings
and mortgage bank and/or
building and loan association
without having first complied with
the provisions of R.A. 337.
Issue:
Whether or not there exists a
probable cause for the search to
be valid.
Held:
Yes. The Court held that it cannot
be gainsaid that the constitutional
injunction against unreasonable
searches and seizures seeks to
forestall, not purely abstract or
imaginary evils, but specific and
concrete ones. In the very nature
of things, unreasonableness is a
condition dependent upon the
circumstances surrounding each
case, in much the same way as the
question whether or not "probable
cause" exists is one which must be
decided in the light of the
conditions obtaining in given
situations.
Webb vs De Leon
G.R. No. 121234, August 23,
1995
Topic: Searches and Seizures
(Rule 126)

Facts:
The National Bureau of
Investigation (NBI) filed with the
Department of Justice a lettercomplaint charging petitioners
Hubert Webb, Michael Gatchalian,
Antonio J. Lejano and six (6) other
persons, with the crime of Rape
with Homicide. Forthwith, the
Department of Justice formed a
panel of prosecutors to conduct
the preliminary investigation of
those charged with the rape and
killing Carmela N. Vizconde; her
mother, and her sister in their
home in Paraaque.
Issue:
Whether or not the DOJ committed
grave abuse of discretion in
finding probable cause.
Held:
No. The Court held that in
determining probable cause, facts
and circumstances are weighed
without resorting to technical rules
of evidence, but rather based on
common sense which all
reasonable men have.
A finding of probable cause needs
only to rest on evidence showing
that more likely than not a crime
has been committed and was
committed by the suspects.

seizure conducted by the military


authorities. The articles seized
from him are sought to be used as
evidence in his prosecution for
illegal possession of firearms. He
asks that their admission be
temporarily restrained and
thereafter permanently enjoined.
The challenged search warrant
was issued by the respondent
judge, the petitioner's house was
searched two days later but none
of the articles listed in the warrant
was discovered. However, the
officers conducting the search
found in the premises one Colt
Magnum revolver and eighteen
live bullets which they confiscated.
They are now the bases of the
charge against the petitioner.
Issue:
Whethe or not the search warrant
is valid.

Topic: Searches and Seizures


(Rule 126)

Held:
No. The Court held that to be
valid, a search warrant must be
supported by probable cause to be
determined by the judge or some
other authorized officer after
examining the complainant and the
witnesses he may produce. No less
important, there must be a specific
description of the place to be
searched and the things to be
seized, to prevent arbitrary and
indiscriminate use of the warrant.
An application for search warrant
if based on hearsay cannot,
standing alone, justify issuance of
that writ.

Facts:
The petitioner claims he was the
victim of an illegal search and

Uy vs BIR
G.R. No. 129651, October 20,
2000

Roan vs Gonzales
G.R. No. 71410, November 25,
1986

Topic: Searches and Seizures


(Rule 126)
Facts:
Petitioners assail the validity of the
warrants issued for the search of
the premises of the Unifish
Packing Corporation, and pray for
the return of the items seized by
virtue thereof.
A certain Rodrigo Abos reported to
the Bureau of Internal Revenue
(BIR) that petitioners Unifish
Packing Corporation and Uy Chin
Ho alias Frank Uy were engaged in
activities constituting violations of
the National Internal Revenue
Code. Abos, who claimed to be a
former employee of Unifish,
executed an Affidavit regarding his
personal knowledge of the
activities of Unifish in violation of
the law.
Issue:
Whether or not the search warrant
is valid.
Held:
No. The Court held that the oath
required must refer to the truth of
the facts within the personal
knowledge of the petitioner or his
witnesses, because the purpose
thereof is to convince the
committing magistrate, not the
individual making the affidavit and
seeking the issuance of the
warrant, of the existence of
probable cause.The witnesses, in
turn, must testify under oath to
facts of their own personal
knowledge. The oath required
must refer to the truth of the facts
within the personal knowledge of

the petitioner or his witnesses,


because the purpose thereof is to
convince the committing
magistrate, not the individual
making the affidavit and seeking
the issuance of the warrant, of the
existence of probable cause.
Search warrants are not issued on
loose, vague or doubtful basis of
fact, nor on mere suspicion or
belief.
Quelnan vs People
G.R. No. 166061, July 6, 2007
Topic: Searches and Seizures
(Rule 126)
Facts:
The Police Assistance and Reaction
Against Crime (PARAC) was tasked
to implement a search warrant to a
certain Bernard Lim for probably
possessing MA HCI (Shabu). The
team was escorted to the unit by
the security officer (Punsaran),
upon arrival at the place to be
searched, a male person naked
from the waist up opened the door,
which was later identified as
Quelnan. The team presented the
search warrant and proceeded
with the search. In the presence of
Quelnan and Punsaran, they found
on top of a bedroom table 3 pieces
of transparent plastic sachet
containing white crystalline
substance which was later
examined as Shabu. The next day,
Quelnan was arrested for violation
of RA 6425.
Quelnan in his defense averred
that he is not residing in the said
unit, but he is the registered
owner of the said unit, which he
leased to Sung Kok Lee. That he

was there during the search for he


was collecting the rent. That he
was forced to sign some
documents at gunpoint,
handcuffed and brought to PARAC
Office. Two days later, he was
brought to Makati Prosecutor's
Office for inquest and a case was
filed against him.
Issue:
Whether or not the search warrant
was properly enforced provided
that he was not the subject of the
search warrant.
Held:
Yes. The Court held that Nowhere
in Section 4, Rule 126 or any other
provision of the Revised Rules of
Criminal Procedure is it required
that the search warrant must name
the person who occupies the
described premises.

Whether or not the search warrant


is valid.
Held:
No. The Court held that a warrant
issued against fifty (50) John
Does unconstitutional and void.
Insofar as said warrant is issued
against fifty (50) John Does not
one of whom the witnesses to the
complaint could or would identify,
it is of the nature of a general
warrant, one of a class of writs
long proscribed as unconstitutional
and once anathematized as totally
subversive of the liberty of the
subject. Clearly violative of the
constitutional injunction that
warrants of arrest should
particularly describe the person or
persons to be seized, the warrant
must, as regards its unidentified
subjects, be voided.

Pangandaman vs. Casar


G.R. No. 71782, April 14, 1988

Uy vs Villareal
G.R. No. 16009, September 21,
1920

Topic: Searches and Seizures


(Rule 126)

Topic: Searches and Seizures


(Rule 126)

Facts:
The shooting incident by armed
men in Lanao led to the issuance
of a warrant of arrest. Petitioners
assert that the respondent Judge
issued a warrant of arrest against
fifty (50) John Does
transgressing the Constitutional
provision requiring that such
warrants should particularly
describe the persons or things to
be seized.

Facts:
Ramon Gayanilo, corporal of the
Philippine Constabulary, presented
to the judge of the Court of First
Instance of Iloilo an application for
search warrant, the said Ramon
Gayanilo stating in his application;
"That in the house of Chino Uy
Kheytin, Sto. Nio St., No. 20,
Iloilo, under the writing desk in his
store, there is kept a certain
amount of opium."
The search warrant was issued and
armed with that search warrant,
the respondent, lieutenant of the

Issue:

Philippine Constabulary,
accompanied by some of his
subordinates, on the same day
searched the house of the
petitioner Uy Kheytin and found
therein 60 small cans of opium.
They wanted to search also
the bodega on the ground-floor of
the house, but Uy Kheytin
positively denied that it was his or
that he rented it. Lieutenant
Torralba wanted to be sure, and
for this reason he placed a guard
in the premises to see that nothing
was removed therefrom, and then
went away to find out who the
owner of the bodega was. The next
morning he learned from the
owner of the house, one Segovia,
of the town of Molo, that the
Chinaman Uy Kheytin was the one
who was renting the
bodega. Thereupon Lieutenant
Torralba and his subordinates
resumed the search.
Issue:
Whether or not the confiscated
items should be returned to the
appellants.
Held:
No. The Court held that although
in the issuance of the search
warrant in question the judge did
not comply with the requirements
of section 98 of General Orders
No. 58, the petitioners are not
entitled to the return of the opium
and its paraphernalia which were
found and seized under said
warrant, and much less are they
entitled to be exonerated because
of such omission of the judge. The
law specifically requires that a
search warrant should particularly

describe the place to be searched


and the things to be seized. The
evident purpose and intent of this
requirement is to limit the things
to be seized to those, and only
those, particularly described in the
search
warrantto leave the officers of
the law no discretion regarding
what articles they shall seize, to
the end that "unreasonable
searches and seizures" may not be
made,that abuses may not be
committed. Therefore, no other
property than those described in
the search warrant may be taken
thereunder.
Stonehill vs Diokno
G.R. No. L-19550, June 19,
1967
Facts:
Respondents issued, on different
dates, 42 search warrants against
petitioners personally, and/or
corporations for which they are
officers directing peace officers to
search the persons of petitioners
and premises of their offices,
warehouses and/or residences to
search for personal properties
books of accounts, financial
records, vouchers,
correspondence, receipts, ledgers,
journals, portfolios, credit journals,
typewriters, and other documents
showing all business transactions
including disbursement receipts,
balance sheets and profit and loss
statements and
Bobbins(cigarettes) as the subject
of the offense for violations of
Central Bank Act, Tariff and
Customs Laws, Internal Revenue
Code, and Revised Penal Code.

Upon effecting the search in the


offices of the aforementioned
corporations and on the respective
residences of the petitioners, there
seized documents, papers, money
and other records. Petitioners then
were subjected to deportation
proceedings and were constrained
to question the legality of the
searches and seizures as well as
the admissibility of those seized as
evidence against them.
Issue:
Whether or not general search
warrants are valid.
Held:
No. The Court held that General
search warrants are outlawed
because they place the sanctity of
the domicile and the privacy of
communication and
correspondence at the mercy of
the whims, caprice or passion of
peace officers.
20th Century Fox Film Corp. vs
Court of Appeals
G.R. No. L-76649-51, August
19, 1988
Topic: Searches and Seizures
(Rule 126)
Facts:
Petitioner 20th Century Fox Film
Corporation through counsel
sought the National Bureau of
Investigation's (NBI) assistance in
the conduct of searches and
seizures in connection with the
latter's anti-film piracy campaign.
Specifically, the letter-complaint
alleged that certain videotape
outlets all over Metro Manila are
engaged in the unauthorized sale

and renting out of copyrighted


films in videotape form which
constitute a flagrant violation of
Presidential Decree No. 49
(otherwise known as the Decree on
the Protection of Intellectual
Property).
Acting on the letter-complaint, the
NBI conducted surveillance and
investigation of the outlets
pinpointed by the petitioner and
subsequently filed three (3)
applications for search warrants
against the video outlets owned by
the private respondents. The
applications were consolidated and
heard by the Regional Trial Court
of Makati.
The lower court issued the desired
search warrants.
Armed with the search warrants,
the NBI accompanied by the
petitioner's agents, raided the
video outlets and seized the items
described therein.
Acting on a motion to lift search
warrants and release seized
properties filed by the private
respondents, the lower court
issued an order, lifting the three
(3) search warrants issued earlier
against the private respondents by
the court.
Issue:
Whether or not the judge properly
lifted the search warrants he
issued earlier upon the application
of the National Bureau of
Investigation on the basis of the
complaint filed by the petitioner.
Held:
Yes. The Court held that the
constitutional right against illegal
searches and seizures guaranteed

under Sec. 2, Art. III of the 1987


Constitution protects a citizen
against unreasonable invasion of
his privacy and liberty as to his
person, papers and effects. The
privacy of a person must not be
disturbed except in case of an
overriding social need.
People vs Rubio
G.R. No. L-35500, October 27,
1932
Topic: Searches and Seizures
(Rule 126)
Facts:
Internal Revenue agents received
information about fraudulent
books, invoices and records of
Simplex Trading Corporation & to
subsidiary companies Paramount
Trading Corporation & New York
Trading Corp. which are in the
possession of the occupant of the
house that was reported to them.
They applied for a search warrant,
and it was subsequently issued. On
the same day, internal revenue
agents proceeded to the place
indicated in the warrant, searched
the premises, and took therefrom
books, invoices, and documents
belonging to the Simplex Trading
Corporation of which Jose Rubio
was the manager. Thereafter, as
indicated, a motion was presented
on behalf of Rubio to secure a
pronouncement of nullity of the
search warrant, which motion,
after receiving memoranda in
support and in opposition but
without taking evidence, was
denied.
Issue:

Whether or not the search is valid.


Held:
Yes. While the place to be
searched and the property to be
seized under a search warrant
must be particularly described in
the warrant, yet the description is
required to be specific only in so
far as the conditions will ordinarily
allow.
Kho vs Makalintal
G.R. No. 94902-06, April 21,
1999
Topic: Searches and Seizures
(Rule 126)
Facts:
The NBI applied for the issuance
of search warrants by the
respondent Judge against
petitioner Banjamin V. Kho in his
residence in Paranaque. On the
same day,another NBI agent,
applied with the same court for the
issuance of search warrants
against the said petitioner in his
other house in Paranaque. The
search warrants were applied for
after teams of NBI agents had
conducted a personal surveillance
and investigation in the two houses
referred to on the basis of
confidential information they
received that the said places were
being used as storage centers for
unlicensed firearms and chop-chop
vehicles. Respondent NBI sought
for the issuance of search warrants
in anticipation of criminal cases to
be instituted against petitioner
Kho.
On the same day, the respondent
Judge conducted the necessary

examination of the applicants and


their witnesses, after which he
issued Search Warrants.
On the following day, armed with
Search Warrants, NBI agents
searched subject premise, and
they recovered various highpowered firearms and hundreds of
rounds of ammunition. Meanwhile,
another search was conducted at
the other house by another team of
NBI agents. The said second
search yielded several highpowered firearms with explosives
and more than a thousand rounds
of ammunition. The simultaneous
searches also resulted in the
confiscation of various radio and
telecommunication equipment, two
units of motor vehicles and one
motorcycle. Upon verification with
the Firearms and Explosives Unit
in Camp Crame, the NBI agents
found out that no license has ever
been issued to any person or entity
for the confiscated firearms in
question. Likewise, the radio
agents found out that no license
has ever been issued to any person
or entity for the confiscated
firearms in question. Likewise, the
radio tranceivers recovered and
motor vehicles seized turned out to
be unlicensed and unregistered
per records of the government
agencies concerned.
Issue:
Whether or not the search warrant
is a general search warrant.
Held:
No. The Court held that the use of
the phrase and the like does not
necessarily make a search warrant
a general warrant.

the said warrants comply with


Constitutional and statutory
requirements. The law does not
require that the things to be seized
must be described in precise and
minute detail as to leave no room
for doubt on the part of the
searching authorities. Otherwise,
it would be virtually impossible for
the applicants to obtain a warrant
as they would not know exactly
what kind of things they are
looking for. Since the element of
time is very crucial in criminal
cases, the effort and time spent in
researching on the details to be
embodied in the warrant would
render the purpose of the search
nugatory.
People vs Tee
G.R. No. 140546-47, January
20, 2003
Topic: Searches and Seizures
(Rule 126)
Facts:
Appellant is a Chinese national, a
businessman, and a resident of
Baguio City. A raid conducted by
operatives of the National Bureau
of Investigation (NBI) and
Philippine National Police
Narcotics Command (PNP
NARCOM) at premises allegedly
leased by appellant and at his
residence yielded huge quantities
of marijuana.
Appellant moved to quash the
search warrant on the ground that
it was too general and that the NBI
had not complied with the
requirements for the issuance of a
valid search warrant. The
pendency of said motion, however,

did not stop the filing of the


appropriate charges against
appellant.
Issue:
Whether or not the appellant's
contention that the description on
the serach warrant which says an
undetermined amount of
marijuana, was too general and
hence makes the warrant void for
vagueness.
Held:
No. The Court held that what the
Constitution seeks to avoid are
search warrants of broad or
general characterization or
sweeping descriptions, which will
authorize police officers to
undertake a fishing expedition to
seize and confiscate any and all
kinds of evidence or articles
relating to an offense.
Panuncio vs People
G.R. No. 165678, July 17, 2009
Topic: Searches and Seizures
(Rule 126)
Facts:
The LTO and a special task force
led by PNP Superintendent Panfilo
Lacson and Police Senior Inspector
Cesar Ouano Jr., with search
warrant issued by RTC judge
Pardo raided the residence of
Panuncio a jeepney operator in
Quezon City. They confiscated LTO
documents and 17 pieces of
private vehicle plates, copying
machine, typewriters and other
tools.
Panuncio signed a certification of
orderly search with Brgy.

Chairman Manalo, Panuncio's


employee Velasco and Nidua. They
all signed the receipt of property
seized issued by PO3 Abuda.
Panuncio and one Jaime Lopez
were arrested.
Issue:
Whether or not the search is valid.

Held:
Yes. The Court held that the search
of a house, room, or any other
premise shall be made in the
presence of the lawful occupant
thereof or any member of his
family or in the absence of the
latter, two witnesses of sufficient
age and discretion residing in the
same locality.
Buenaventura vs. People
G.R. No. 171578, August 8,
2007
Topic: Searches and Seizures
(Rule 126)
Facts:
Appellant was charged with the
violation of Dangerous Drugs Act
for possession and selling of
marijuana.
The Police received a report about
the appellant, that the same was
selling marijuana. Thereafter, they
conducted a surveillance operation
to verify the information.A buybust operation was then
conducted, and the appellant was
apprehended in the process as he
sold marijuana to the police in the
said operation.

Issue:
Whether or not the arrest and
search are valid.
Held:
Yes. The Court held that any
objection concerning the issuance
or service of a warrant or a
procedure in the acquisition by the
court of jurisdiction over the
person of the accused must be
made before he enters his plea,
otherwise, the objection is deemed
waived.
People vs Macatingag
G.R. No. 181037, January 19,
2009
Topic: Searches and Seizures
(Rule 126)
Facts:
Members of the Philippine
National Police (PNP) in Calamba
City formed a buy-bust team
because of a report from a
confidential informant about the
drug pushing activities of a certain
"Sai," who later turned out to be
appellant. Upon arriving at the site
of the operation, PO3 Garcia and
the confidential informant waited
for appellant at the entrance gate
of a Subdivision in. The appellant
arrived and was introduced to PO3
Garcia as the prospective buyer.
Appellant, on the other hand,
asked PO3 Garcia about the money
amounting to P52,500.00. PO3
Garcia then pulled out an envelope
containing the two P500.00 bills
with the boodle money from his
pocket, and demanded the drugs.
Appellant thereafter pulled out
from his pocket one plastic sachet

and handed it to PO3 Garcia.


Immediately upon giving appellant
the marked money, PO3 Garcia lost
no time in giving the pre-arranged
signal to PO3 Leona. PO3 Leona
thereupon hurriedly seized from
appellant the marked money, while
PO3 Garcia recovered the plastic
sachet containing suspected shabu
from appellant.
Issue:
Whether or not such buy-bust
operation is lawful.
Held:
Yes. The Court held that the
conduct of a buy-bust operation is
a common and accepted mode of
apprehending those involved in
illegal sale of prohibited or
regulated drugs.

PEREZ, CARLA PATRICIA S.


2D
Criminal Procedure
Justice Amy Lazaro-Javier

BOOK

ARREST,
SEIZURE

SEARCH

AND

1. Dimacuha v. People 516 SCRA


513, Feb. 23, 2007
Topic: Right to meet the witnesses
face to face

Doctrine:
In People v. Chua Ho San,
308 SCRA 432 (1999), the Court
pointed out that the interdiction
against warrantless searches and
seizures is not absolute and that

they have long been deemed


permissible by jurisprudence in
the
following
instances:
(1)
search of moving vehicles; (2)
seizure in plain view; (3) customs
searches; (4) waiver or consented
searches; (5) stop and frisk
situations (Terry search); and (6)
search incidental to a lawful
arrest. The last includes a valid
warrantless search and seizure
pursuant
to
an
equally
warrantless arrest, for, while as a
rule, an arrest is considered
legitimate if effected with a valid
warrant of arrest, the Rules of
Court
recognizes
permissible
warrantless arrest, to wit: (1)
arrest in flagrante delicto; (2)
arrest effected in hot pursuit; and
(3) arrest of escaped prisoners.

they received a tip from informant


Benito Marcelo that a sale of
illegal drugs was to take place at
11 a.m. to 12 nn that day at the
corner to J.M. Basa and Kapwa
Streets, Calumpang, Marikina by a
seller more or less 54 in height
and using a sky blue Toyota
Corrolla. The sale took place and
the seller, Ruby Dimacuha, was
arrested and the drugs (shabu)
seized. At the trial, the prosecution
presented only Valerosa and Ostan
as its witnesses, not Marcelo. RTC
and CA both found Dimacuha
guilty of possession or use of
regulated
drugs
and
sale,
administration,
dispensation,
delivery,
transportation
and
distribution of regulated drugs.

In cases involving violations


of the Dangerous Drugs Act,
credence should be given to the
narration of the incident by the
prosecution witnesses especially
when they are police officers who
are presumed to have performed
their duties in a regular manner
unless there be evidence to the
contrary.

Whether
Dimacuha
was
denied the constitutional right to
meet and confront her accuser as
Marcelo was not presented as
prosecution witness

Facts:
SPO2
Melanio
Valeroso,
SPO2 Vicente Ostan and other
police
officers
were
at
the
Marikina Police Station at around
9:30 a.m. of Aug. 10, 1995 when

Issue:

Held:
No.
The right of the petitioner to
confront the witnesses against her
is not affected by the failure of the
prosecution
to
present
the
informant.
The
matter
of
presentation of witnesses is not for
the accused nor the trial court to
decide. Discretion belongs to the
prosecutor as to how the State

should
present
its
case. The
prosecutor has the right to choose
whom he would present as
witnesses. Informants are usually
not presented in court because of
the need to hide their identity and
preserve their invaluable service
to the police.

2. People v. Nuevas 516 SCRA


463, Feb. 22, 2007
Topic:
Warrantless
search
incidental to a lawful arrest; plain
view doctrine
Doctrine:
A search and seizure must be
carried through or with a judicial
warrant, otherwise such search
and
seizure
becomes
unreasonable and any evidence
obtained
therefrom
is
inadmissible for any purpose in
any proceeding
Where a warrant is not
necessary to effect a valid search
or seizure or when the latter
cannot be performed except
without
a
warrant,
what
constitutes
a
reasonable
or
unreasonable search or seizure is
purely a judicial question.

A
search
substantially
contemporaneous with an arrest
can precede the arrest if the
police have probable cause to

make the arrest at the outset of


the search.

Reliable information alone is


not
sufficient
to
justify
a
warrantless arrest under Section
5(a), Rule 113. The rule requires,
in addition, that the accused
perform some overt act that
would indicate that he has
committed,
is
actually
committing, or is attempting to
commit an offense.

Object in Plain View:


Where the object seized was
inside a closed package, the
object itself is not in plain view
and therefore cannot be seized
without a warrant; If the package
is such that an experienced
observer could infer from its
appearance that it contains the
prohibited article, then the article
is deemed in plain view.

The constitutional immunity


against unreasonable searches
and seizures is a personal right
which may be waived. However, it
must be seen that the consent to
the search was voluntary in order
to validate an otherwise illegal
detention and search, i.e., the
consent
was
unequivocal,
specific, and intelligently given,
uncontaminated by any duress or
coercion. Relevant
to
this
determination are the following

characteristics of the person


giving
consent
and
the
environment in which consent is
given: (1) the age of the
defendant; (2) whether he was in
a public or secluded location; (3)
whether he objected to the search
or passively looked on; (4) the
education and intelligence of the
defendant; (5) the presence of
coercive police procedures; (6)
the defendant's belief that no
incriminating evidence will be
found; (7) the nature of the police
questioning; (8) the environment
in which the questioning took
place; and (9) the possibly
vulnerable subjective state of the
person consenting.

In
case
of
consented
searches or waiver of the
constitutional guarantee against
obtrusive
searches,
it
is
fundamental that to constitute a
waiver, it must first appear that
(1) the right exists; (2) the person
involved had knowledge, either
actual or constructive, of the
existence of such right; and (3)
the said person had an actual
intention to relinquish the right.

The legality of an arrest affects


only the jurisdiction of the court
over the person of the accused; A
waiver of an illegal warrantless
arrest does not also mean a
waiver of the inadmissibility of
evidence seized during an illegal

warrantless arrest. (People v.


Nuevas 516 SCRA 463, Feb. 22,
2007)

Facts:
In the morning of Sept. 27,
1997, PO3 Teofilo Fami and SPO3
Cesar
Cabling
conducted
a
stationary
surveillance
and
monitoring
of
illegal
drug
trafficking in Olongapo City. They
received information that a certain
male person, more or less 54" in
height, 25 to 30 years old, with a
tattoo mark on the upper right
hand, and usually wearing a sando
and maong pants, would make a
delivery of marijuana dried leaves.
Jesus
Nuevas,
who
fit
the
description, alighted from a motor
vehicle.
The
two
officers
questioned Nuevas and shortly, the
latter admitted that he had
marijuana
in
his
possession,
voluntarily handed the same, and
said that his 2 companions, Din
and Inocencio, would be making
another delivery. The 3 were
arrested
and
the
marijuana
confiscated. RTC found the 3
guilty. CA affirmed the decision.

Issues:
Whether the case falls under
the
circumstances
of
a
warrantless search incidental to a
lawful arrest

Whether
the
seized is in plain view

marijuana

3. People v. Tuazon 532 SCRA


152, Sept. 3, 2007
Topic: Warrantless search and
seizure of a moving vehicle

Held:
Both no.
Nuevas, Din and Inocencio
were not committing a crime in the
presence of the police officers.
Police officers Fami and Cabling
did not have personal knowledge
of the facts indicating that the
persons to be arrested had
committed an offense. Reliable
information alone is not sufficient
to justify a warrantless arrest
under Section 5(a), Rule 113. The
rule requires, in addition, that the
accused perform some overt act
that would indicate that he "has
committed, is actually committing,
or is attempting to commit an
offense."
Neither could the searches
be justified under the plain view
doctrine. An object is in plain view
if it is plainly exposed to sight.
Where the object seized was inside
a closed package, the object itself
is not in plain view and therefore
cannot be seized without a
warrant. However, if the package
proclaims its contents, whether by
its distinctive configuration, its
transparency, or if its contents are
obvious to an observer, then the
contents are in plain view and may
be seized.

Doctrine:
A warrantless search of a
moving vehicle is justified on the
ground that it is not practicable
to secure a warrant because the
vehicle can be quickly moved out
of the locality or jurisdiction in
which the warrant must be
sought.

The exception from securing


a search warrant when it comes
to moving vehicles does not give
the police authorities unbridled
discretion
to
conduct
a
warrantless
search
of
an
automobilein recognition of the
possible abuse, jurisprudence
dictates that at all times, it is
required that probable cause
exist in order to justify the
warrantless search of a vehicle.

When a vehicle is flagged


down and subjected to an
extensive
search,
such
a
warrantless search has been held
to be valid as long as the officers
conducting the search have
reasonable or probable cause to
believe prior to the search that
they
would
find
the
instrumentality
or
evidence

pertaining to a crime, in the


vehicle to be searched

Exclusionary Rule: Failure


to
timely
object
to
the
admissibility of the evidence
against the accused on the
ground that the same was
obtained through a warrantless
search amounts to waiver of the
objection on the legality of the
search and the admissibility of
the evidence. (People v. Tuazon
532 SCRA 152, Sept. 3, 2007)

Facts:
In the morning of Mar. 7
1999, the Antipolo City Police
Station
received
through
telephone,
a
confidential
information that a Gemini car
bearing plate number PFC 411
would deliver an unspecified
amount
of
shabu in
Marville
Subdivision, Antipolo City. When
the surveillance team arrived
there, they immediately saw the
car and flagged it down. PO1
Manuel Padlan noticed a gun
tucked in Bernardo Tuazons waist,
for which he failed to show
document to support possession,
and when he was asked to alight,
Padlan noticed 5 plastic sachets of
shabu in the drivers seat. Tuazon
was then brought to the police
station. RTC and CA both found
him guilty.

Issues:
Whether or not the shabu
seized is admissible as evidence
Whether or not Tuazons
failure to timely object to its
admissibility amounts to a waiver
of the objection on the legality of
the search and the admissibility of
the evidence

Held:
Both yes.
In recognition of the possible
abuse, jurisprudence dictates that
at all times, it is required that
probable cause, or the existence of
such facts and circumstances
which could lead a reasonably
discreet and prudent man to
believe that an offense has been
committed and that the items,
articles or objects sought in
connection with said offense or
subject to seizure and destruction
by law is in the place to be
searched, exist in order to justify
the warrantless search of a
vehicle. The circumstances, taken
together, are sufficient to establish
probable cause for the warrantless
search of the Gemini car and the
eventual admission into evidence
of the plastic packets against
appellant.
Tuazon failed to timely
object to the admissibility of the
evidence against him on the

ground that the same was obtained


through a warrantless search. His
failure amounts to a waiver of the
objection on the legality of the
search and the admissibility of the
evidence obtained by the police. It
was only proper for the trial court
to admit said evidence.

4. Valeroso v. CA GR No. 164815,


Sept. 3, 2009
Topic:
Warrantless
search
incidental to a lawful arrest

Doctrine:
When an arrest is made, it is
reasonable for the arresting
officer to search the person
arrested in order to remove any
weapon that the latter might use
in order to resist arrest or effect
his escape, and, in addition, it is
entirely
reasonable
for
the
arresting officer to search for and
seize any evidence on the
arrestees person in order to
prevent
its
concealment
or
destruction. A valid arrest allows
the seizure of evidence or
dangerous weapons either on the
person of the one arrested or
within
the
area
of
his
immediate control, or the area
from within which he might gain
possession of a weapon or
destructible evidence.

A cabinet which is locked


could no longer be considered as
an area within the arrestees
immediate control because there
is no way for him to take any
weapon or to destroy any
evidence that could be used
against him.

The plain view doctrine


may not be used to launch
unbridled
searches
and
indiscriminate seizures or to
extend a general exploratory
search made solely to find
evidence of defendants guilt. The
doctrine is usually applied where
a police officer is not searching
for evidence against the accused,
but nonetheless inadvertently
comes across an incriminating
object. (Valeroso v. CA GR No.
164815, Sept. 3, 2009)

Facts:
There were two conflicting
accounts of the prosecution and
defense as to what transpired. The
prosecution claims that Sr. Insp.
Jerry Valeroso was arrested by
virtue of a warrant near the INP
Central Police Station in Culiat,
Quezon City, while he was about to
board a tricycle. After placing
Valeroso under arrest allegedly for
kidnapping with ransom, the
arresting officers bodily searched
him, and they found the subject
firearm
and
ammunition. The
defense, on the other hand, insists

that he was arrested inside the


boarding
house
of
his
children. After serving the warrant
of arrest (allegedly for kidnapping
with ransom), some of the police
officers searched the boarding
house and forcibly opened a
cabinet where they discovered the
subject firearm. RTC and CA
convicted Valeroso for illegal
possession
of
firearm
and
ammunition.

Issue:
Is the warrantless search
and seizure of the firearm and
ammunition valid?

Held:
No.
SC gave credence to the
version of the defense. A valid
arrest allows the seizure of
evidence or dangerous weapons
either on the person of the one
arrested or within the area of
his immediate control, from
within which he might gain
possession of a weapon or
destructible evidence. Here, the
search was made in the locked
cabinet which cannot be said to
have
been
within
Valerosos
immediate
control. Thus,
the
search exceeded the bounds of
what may be considered as an
incident to a lawful arrest.

5. People v. Molina 352 SCRA


174, Feb. 19, 2001
Topic: Unreasonable search and
seizure

Doctrine:
In cases of in flagrante
delicto arrests, a peace officer
or a private person may, without a
warrant, arrest a person when, in
his presence, the person to be
arrested
has
committed,
is
actually
committing,
or
is
attempting to commit an offense.
The arresting officer must have
personal knowledge of facts or
circumstances
convincingly
indicative or constitutive of
probable cause. Requisities: (1)
the person to be arrested must
execute an overt act indicating
that he has just committed, is
actually
committing,
or
is
attempting to commit a crime;
and (2) such overt act is done in
the presence or within the view of
the arresting officer.

Reliable information alone,


absent any overt act indicative of
a felonious enterprise in the
presence and within the view of
the arresting officers, are not
sufficient to constitute probable
cause that would justify an in
flagrante delicto arrest. (People v.
Molina 352 SCRA 174, Feb. 19,
2001)

Facts:
On Aug. 8, 1996, acting upon
information received that an
alleged pusher will be passing at
NHA, Ma-a, Davao City anytime
that morning, SPO4 Dionisio
Cloribel,
SPO1
Leonardo
Y.
Pamplona, Jr., and SPO1 Marino S.
Paguidopon, Jr. were dispatched to
the location. A trisikad passed by
and the informant pointed to
Gregorio Mula, who was riding it,
as the supposed dealer. The
officers then ordered the trisikad
to stop and asked Mula to open the
black bag that he was holding.
Nasario Molina, who was with
Mula, then told the police that they
could just settle the matter but
Pamplona insisted on opening the
black bag, which revealed dried
marijuana leaves inside. Mula and
Molina
were
arrested
and
handcuffed and the marijuana
seized. RTC found them guilty and
imposed death penalty, which was
still implemented then.
Issue:
Whether the
warrantless
arrest, search and seizure was
valid because the accused were
caught in flagrante de licto

Held:
No.

To constitute a valid in
flagrante
delicto arrest,
two
requisites must concur: (1) the
person to be arrested must
execute an overt act indicating
that he has just committed, is
actually
committing,
or
is
attempting to commit a crime; and
(2) such overt act is done in the
presence or within the view of the
arresting officer. Mula and Molina
manifested no outward indication
that would justify their arrest. In
holding a bag on board a trisikad,
they could not be said to be
committing, attempting to commit
or have committed a crime.

6. Esquillo v. People GR No.


182010, Aug. 25, 2010
Topic: Warrantless search under
the stop-and-frisk concept

Doctrine:
Where the accused only
raised the issue of warrantless
arrestas
well
as
the
inadmissibility
of
evidence
acquired on the occasion thereof
for the first time only on appeal,
she is deemed to have waived any
objections on the legality of her
arrest.

A stop-and-frisk practice
serves a dual purpose: (1) the
general interest of effective crime

prevention and detection, which


underlies the recognition that a
police
officer
may,
under
appropriate circumstances and in
an appropriate manner, approach
a
person
for
purposes
of
investigating possible criminal
behavior even without probable
cause; and (2) the more pressing
interest of safety and selfpreservation which permit the
police officer to take steps to
assure himself that the person
with whom he deals is not armed
with a deadly weapon that could
unexpectedly and fatally be used
against the police officer. It is
essential that a genuine reason
must exist, in light of the police
officers
experience
and
surrounding
conditions, to
warrant the belief that the person
who manifests unusual suspicious
conduct
has
weapons
or
contraband
concealed
about
him. (Esquillo v. People GR No.
182010, Aug. 25, 2010)

Facts:
On Dec. 10, 2002, PO1
Cruzin and PO2 Angel Aguas
conducted
a
surveillance
operation in Pasay City on the
activities of a notorious snatcher
known as Ryan. Cruzin noticed
Suzan Esquillo placing inside a
yellow
cigarette
case
what
appeared to be a small heatsealed
transparent
sachet

containing
white
substance.
When
Cruzin
approached
Esquillo, the latter tried to flee to
her house but was restrained.
Cruzin then asked Esquillo to
take out the sachet, marked it
with her initials SRE, then
brought Esquillo together with
the seized item to the police
station for laboratory exam of the
substance, which revealed that it
was shabu, and drug test on
Esquillo, for which she turned out
positive.

Issue:
Whether the
search was valid

warrantless

Held:
Yes.
When
PO1
Cruzin
saw
petitioner placing a plastic sachet
containing
white
crystalline
substance into her cigarette case,
it was in his plain view. Given his
training as a law enforcement
officer, it was instinctive on his
part to be drawn to curiosity and
to approach her. What is essential
is that a genuine reason must exist
to warrant the belief that the
person who manifests unusual
suspicious conduct has weapons or
contraband
concealed
about
him. Such a stop-and-frisk practice
serves a dual purpose: (1) the
general interest of effective
crime prevention and detection,

which underlies the recognition


that a police officer may, under
appropriate circumstances and
in an appropriate manner,
approach a person for purposes
of
investigating
possible
criminal behavior even without
probable cause; and (2) the more
pressing interest of safety and selfpreservation which permit the
police officer to take steps to
assure himself that the person
with whom he deals is not armed
with a deadly weapon that could
unexpectedly and fatally be used
against the police officer.

7. People v. Bohol 560 SCRA 232,


July 28, 2008
Topic: Buy-bust operation

Doctrine:
An arrest made after an
entrapment operation does not
require
a
warrant.
Such
warrantless arrest is considered
reasonable and valid under Rule
113, Section 5(a) of the Revised
Rules on Criminal Procedure,
which states: Sec. 5. Arrest
without warrant; when lawful.A
peace officer or a private person
may, without a warrant, arrest a
person: (a) When, in his presence,
the person to be arrested has
committed,
is
actually
committing, or is attempting to
commit an offense; x x x x. The
subsequent warrantless search

that will result in seizure of


possible evidence found in the
person of the accused is likewise
valid. (People v. Bohol 560 SCRA
232, July 28, 2008)

Facts:
Acting on a tip that Ricardo
Bohol was engaged in illegal drug
trade in Tondo, P/Sr. Insp. Jessie
Nitullano formed a group of 6
police officers to launch a buy-bust
entrapment operation. They went
to Bohols house and asked for
piso (P100) worth of shabu.
When Bohol received the marked
money and gave a plastic sachet of
shabu in exchange, the police
arrested Bohol and frisked him,
from which they found 3 more
plastic sachets. RTC and CA found
Bohol guilty of violating the
Comprehensive Dangerous Drugs
Act of 2002.

Issue:
Whether the arrest and
search on Bohols person was
legal

Held:
Yes.
An arrest made after an
entrapment operation does not
require a warrant. The arresting
officers were justified in arresting
Bohol as he had just committed a

crime when he sold the shabu to


them. A buy-bust operation is a
form of entrapment which has
repeatedly been accepted to be a
valid means of arresting violators
of the Dangerous Drugs Law. The
subsequent warrantless search
that resulted in the seizure of
the shabu found in his person is
likewise valid as it was a search
for things which may be used as
proof of the commission of the
offense.

8. People v. Leangsiri 252 SCRA


213, Jan. 24, 1996
Topic: Warrantless search
Doctrine:
The
inadmissibility
of
evidence
obtained
in
a
warrantless search incident to a
lawful
arrest
outside
the
suspects
person
and
the
premises under his immediate
control admits of an exception.
The exception obtains when
the Plain View Doctrine applies as
explained in People vs. Musa, in
this wise: x x x Objects in the
plain view of an officer who has
the right to be in the position to
have that view are subject to
seizure and may be presented as
evidence. (People v. Leangsiri
252 SCRA 213, Jan. 24, 1996)

Facts:
On May 31, 1993 at the
NAIA
arrival
area,
Suchinda
Leangsiri was caught in the act of
bringing into the country 8,225.31
grams of heroin hidden under the
false bottom of a black suitcase.
He informed the authorities that
he was to deliver the contraband
to 3 people at the Las Palmas
Hotel in Manila. The Narcotics
Command
(NARCOM)
team
conducted a follow-up operation at
the hotel and likewise arrested
Amidu, Omogbolahan and Bhola.
The
NARCOM
team
then
proceeded to Royal Palm Hotel
where the latter two were billeted
and searched their room, where
the officers yielded two black
suitcases each with false bottoms
and a piece of paper with
Leangsiris name on it. Masking
tape and an empty transparent bag
were also found in the room. RTC
found the 3 guilty of conspiring to
transport heroin, while the case
against Leangsiri was archived
since he was at-large.

Issue:
Whether the
warrantless
search of the room at the Royal
Palm Hotel was valid
Held:
No.
The search of the latter room
was illegal and the evidence

obtained therein is inadmissible,


the place searched being not
within the immediate control of
the person arrested. However, the
Court still affirmed the decision
of the RTC because the heroin
and suitcases were offered as
evidence
not
only
against
Leangsiri but also against Amidu,
Omogbolahan and Bhola.

9. Chimel v. California 395 US


752
Facts: defendant was arrested in
his home for burglary of a coin
shop. Afterwards, the arresting
officers conducted a search of his
entire
three-bedroom
house,
including the attic, the garage, a
small
workshop,
and
drawers. Various items -primarily
coins - were found through the
search, and were admitted in
evidence against him by the trial
court, which convicted him of
burglary.

Held:
not only on the person of
the suspect but also in a
permissible area within his
reach
the
reach
of
a
valid
warrantless search goes beyond
the person of the one arrested and
includes
the premises
or
surroundings
under
his
immediate control (immediate
control test).

United
States
Supreme
Court reversed the conviction as it
struck down the warrantless
search on the ground that the
search of the accuseds home
went far beyond his person and
the area from within which he
might have obtained either a
weapon or something that
could have been used as
evidence against him.

10.
Nolasco v. Pao 139 SCRA
152, Oct. 8, 1985
Topic: Warrantless search
Doctrine:
Articles seized under a
search warrant improperly issued
will not be ordered returned after
a criminal case has been filed, as
a person charged with an offense
may be searched even without a
warrant.

There is some confusion in


the
decisions
as
to
what
constitutes the extent of the place
or premises which may be
searched.
What
must
be
considered is the balancing of the
individual's right to privacy and
the public's interest in the
prevention of crime and the
apprehension
of
criminals.
(Nolasco v. Pao 139 SCRA 152,
Oct. 8, 1985)

Facts:
The accused, who were atlarge for rebellion and subversion,
were arrested by constabulary
officers at the intersection of
Mayon Street and P. Margall
Streets in Quezon City at 11:30
a.m. At 12 nn of the same day,
another team of officers searched
the house of one of the accused
under a warrant procured earlier
in the day.

Issue:
Whether the search of the
house was valid

Held:

guarantee against unreasonable


searches and seizures. In its
decision on the partial motion for
reconsideration, the Court ordered
the return of the seized items.
Justice Cuevas and Justice
Abad Santos dissenting opinions:
The warrantless search must be
limited to subject, time and place
of the arrest. Subject: only with
respect to the person of the
suspect, and things that me be
seized from him are limited to
dangerous weapons or anything
which may be used as proof for the
commission of the offense
Time and place: contemporaneous
with the lawful arrest, must have
been conducted at about the time
of the arrest of immediately
thereafter and only at the place
where the suspect was arrested

No.
The warrantless search of a
person lawfully arrested should be
absolutely limited to his or her
person at the time of and incident
to his or her arrest and to
dangerous weapons or anything
which may be used as proof of the
commission of the offense. Such
warrantless search cannot be
made in a place other than the
place of arrest. The warrantless
search made by the authorities on
the accuseds apartment which
was located a few blocks away
from where she was arrested was
illegal for being an untenable
violation, if not nullification, of the
basic constitutional right and

11.
Espano v. CA 288 SCRA
558, April 1, 1998
Topic: Search incident to a lawful
arrest

Facts:
Police
officers
arrested
Rodolfo Espano in flagrante de
licto selling marijuana at Zamora
and Pandacan Streets, Manila. The
police search his person and
yielded 2 cellophane bags of
marijuana. When asked if he had
more, Espano admitted that he had

marijuana in his house, so the


police proceeded there, made a
search, and found 10 more
cellophane bags of marijuana. RTC
and CA found Espano guilty.

Issues:
1. Whether the search on
Espanos person after his
arrest was valid
2. Whether the search of his
house was valid
Held:
1. Yes.
It was a search made
incident to a lawful arrest. The
search may extend beyond the
person of the arrested to
include
the
premises
or
surroundings
under
his
immediate control.
2. No.
The police officers were not
armed with a search warrant
at the time and Espanos
house was beyond his reach
and control.

Where the arrest of the


accused was lawful, having been
caught in flagrante delicto, there
is no need for a warrant for the
seizure of the fruit of the crime as
well as for the body search upon
him, the same being incidental to
a lawful arrest.

Where a person is lawfully


arrested outside his house, a
warrantless search of his house is
unlawful if the police operatives
are not armed with a search
warrant. (People v. Lua 256 SCRA
539, April 26, 1996)

Facts:
Rodolfo Lua was arrested
outside his house in Caloocan
City in flagrante de licto in a buybust
operation.
The
police
searched
his
person
and
confiscated 2 bags of marijuana
and a paltik revolver. The police
then searched his house and
seized other bags of marijuana.

Issue:
12.
People v. Lua 256 SCRA
539, April 26, 1996
Topic: Search incident to a lawful
arrest

Whether the search of his


person and his house were both
valid

Held:
Doctrine:

No, only the search of his


person.

The subsequent search of


Luas house was invalid and the
marijuana seized therein was
inadmissible as it was not a search
incident to a lawful arrest and was
not within Luas reach and control.

13.
Caballes v. CA 424 Phil.
224, Jan. 15, 2002
Topic: Search of moving vehicle

Doctrine:
Probable
cause:
A
reasonable ground of suspicion
supported
by
circumstances
sufficiently strong in themselves
to warrant a cautious mans belief
that the person accused is guilty
of the offense with which he is
charged; or the existence of such
facts and circumstances which
could lead a reasonably discreet
and prudent man to believe that
an offense has been committed
and that the items, articles or
objects sought in connection with
said offense or subject to seizure
and destruction by law is in the
place to be searched.

It is not practicable to secure


a warrant in searching a moving
vehicle because the vehicle can
be quickly moved out of the
locality or jurisdiction in which
the warrant must be sought.
Searches without warrant of

automobiles is also allowed for


the
purpose
of
preventing
violations
of
smuggling
or
immigration laws, provided such
searches are made at borders or
constructive
borders
like
checkpoints near the boundary
lines of the State

Highly regulated by the


government,
the
vehicles
inherent
mobility
reduces
expectation of privacy especially
when its
transit in public
thoroughfares furnishes a highly
reasonable suspicion amounting
to probable cause that the
occupant committed a criminal
activity. The mere mobility of
vehicles, however, does not give
the police officers unlimited
discretion
to
conduct
indiscriminate searches without
warrants if made within the
interior of the territory and in the
absence of probable cause.

Stop-and-search without
warrant at military or police
checkpoints has been declared to
be not illegal per se, for as long
as it is warranted by the
exigencies of public order and
conducted in a way least intrusive
to motorists, such as (1) where
the officer merely draws aside the
curtain of a vacant vehicle which
is parked on the public fair
grounds; (2) simply looks into a
vehicle; (3) flashes a light therein

without opening the cars doors;


(4) where the occupants are not
subjected to a physical or body
search; (5) where the inspection
of the vehicles is limited to a
visual search or visual inspection;
and (6) where the routine check
is conducted in a fixed area.

A peaceful submission to a
search or seizure is not a consent
or an invitation thereto but is
merely a demonstration of regard
for the supremacy of the law. An
accused is not to be presumed to
have waived the unlawful search
conducted simply because he
failed to object. (Caballes v. CA
424 Phil. 224, Jan. 15, 2002)

Facts:
"On June 28, 1989, Sgt.
Victorino Noceja and Pat. Alex de
Castro, while on a routine patrol
in Pagsanjan, Laguna, spotted a
passenger jeep unusually covered
with
"kakawati"
leaves.
Suspecting that the jeep, driven
by Rudy Caballes, was loaded
with smuggled goods, the two
police officers flagged it down.
The police officers discovered
bundles
of
3.08
mm
aluminum/galvanized conductor
wires
exclusively
owned
by
National
Power
Corporation
(NPC) that weighed 700 kilos
valued at P55, 244.45.

Issue:
Whether the
warrantless
search conducted was valid

Held:
No.
The search which is normally
permissible in this instance is
limited to the following instances:
(1) where the officer merely draws
aside the curtain of a vacant
vehicle which is parked on the
public fair grounds; (2) simply
looks into a vehicle; (3) flashes a
light therein without opening the
car's
doors;
(4)
where
the
occupants are not subjected to a
physical or body search; (5) where
the inspection of the vehicles is
limited to a visual search or visual
inspection; and (6) where the
routine check is conducted in a
fixed area. None of the foregoing
circumstances is obtaining here.
The police officers did not merely
conduct a visual search or visual
inspection of Caballes vehicle.
They had to reach inside the
vehicle, lift the kakawati leaves
and look inside the sacks before
they were able to see the cable
wires. It cannot be considered a
simple routine check.

14.
Valmonte v. De Villa GR
No. 83988, May 24, 1990
Topic: Checkpoints

Yes.
Doctrine:
The routine checkpoint stop
does intrude, to a certain extent,
on motorists right to free
passage without interruption,
but it cannot be denied that, as a
rule, it involves only a brief
detention of travellers during
which the vehicles occupants are
required to answer a brief
question or two. For as long as
the vehicle is neither searched
nor its occupants subjected to a
body search, and the inspection
of the vehicle is limited to a visual
search, said routine checks
cannot be regarded as violative of
an individuals right against
unreasonable search. (Valmonte v.
De Villa GR No. 83988, May 24,
1990)

Facts:
Ricardo Valmonte and the
Union of Lawyers and Advocates
for Peoples Rights sought the
declaration of the checkpoints as
unconstitutional
and
their
dismantling and/or banning

Issue:
Whether or not checkpoints
are valid

Held:

The Court did legalize all


checkpoints, i.e. at all times and
under all circumstances. The Court
declared that checkpoints are not
illegal per se. Under exceptional
circumstances, as where the
survival of organized government
is on the balance, or where the
lives and safety of the people are
in grave peril, checkpoints may be
allowed and installed by the
government.
Implicit
in
this
proposition is that when the
situation clears and such grave
perils are removed, checkpoints
will have absolutely no reason to
remain.

15.
People v. Vinecario 420
SCRA 280, Jan. 20, 2004
Topic: Checkpoints

Doctrine:
Searches
conducted
in
checkpoints are valid for as long
as they are warranted by the
exigencies of public order and are
conducted in a way least intrusive
to motorists. Vehicles may be
stopped and extensively searched
when there is probable cause
which justifies a reasonable belief
of the men at the checkpoints
that either the motorist is a law
offender or the contents of the
vehicle
are
or
have
been

instruments of some offense.


(People v. Vinecario 420 SCRA
280, Jan. 20, 2004)

Facts:
On April 10, 1995 at around
10:45 p.m., as about fifteen police
officers
were
manning
a
checkpoint at Ulas, Davao City
pursuant to the COMELEC gun
ban, a Honda TMX motorcycle with
three men on board sped past
them. One of the police officers
blew his whistle and ordered them
to return to the checkpoint. The
men obliged. When asked to
present identification cards, they
were unable to give any. The police
officers noticed that a big military
backpack was slung over the right
shoulder of Victor Vinecario, who
was
observed to
be acting
suspiciously. The bag later turned
out to be containing 1,700 grams
of marijuana. RTC found the
accused guilty.

Issue:
Whether or not the search
and was valid

Held:
Yes.
Searches
conducted
in
checkpoints are valid for as long as
they
are warranted
by the
exigencies of public order and are

conducted in a way least intrusive


to motorists. For as long as the
vehicle is neither searched nor its
occupants subjected to a body
search, and the inspection of the
vehicle is limited to a visual
search, said routine checks cannot
be regarded as violative of an
individuals
right
against
unreasonable search.

16.
People v. Ramos GR No.
180508, Sept. 4, 2009
Topic: Buy-bust operation

Doctrine:
Buy-bust operation: a form
of entrapment employed by peace
officers to apprehend prohibited
drug law violators in the act of
committing
a
drug-related
offense.

Non-compliance
by
the
apprehending/buy-bust team with
Section 21, Article II of Republic
Act No. 9165 is not fatal as long
as there is justifiable ground
therefor, and as long as the
integrity and the evidentiary
value of the confiscated/seized
items, are properly preserved by
the apprehending officer/team
what is of utmost importance is
the preservation of the integrity
and the evidentiary value of the

seized items, as the same would


be utilized in the determination of
the guilt or innocence of the
accused. (People v. Ramos GR No.
180508, Sept. 4, 2009)

Held:

Facts:
Noel Pulido, an operative of
the Makati Anti Drug Abuse
Council (MADAC), and PO2
Ronnie
Aseboque
(PO2
Aseboque), a member of the
Makati City Police Station AntiIllegal Drugs Special Operations
Task
Force
(SAID-SOTF)
conducted
a
surveillance
operation on the activities of
Antonio Ramos whose name
appeared in the Drug Watch List
of Barangay Pitogo, Makati City
where he was observed to be
selling shabu to tricycle drivers.
The
operation
led
to
the
formation of a buy-bust team that
eventually yielded one small heatsealed transparent plastic sachet
containing a white crystalline
substance for 2 50-peso bills.
Upon searching Ramos person,
the police officers recovered from
his right front pocket the yellow
tin case which yielded two other
plastic sachets also containing
white crystalline substances. RTC
and CA found Ramos guilty.

Issue:
Whether
enforcement

observed the procedure laid down


in Section 21 (1), Article II of R.A.
No. 9165 a requirement essential
to preserving the integrity of the
corpus delicti in these cases

the
officers

law
had

No.
A buy-bust operation is a
form of entrapment employed by
peace
officers
to
apprehend
prohibited drug law violators in
the act of committing a drugrelated offense. The failure of the
police officers to comply with the
procedure in the custody of seized
drugs (inventory) puts to doubt
their origins, and negates any
presumption
of
regularity
accorded to acts undertaken by
police officers in the pursuit of
their official duties. Ramos was
acquitted.

17.
People v. Araneta GR No.
191064, Oct. 20, 2010
Topic: Objective test in buy-bust
operation

Doctrine:
Absent
any
convincing
countervailing
evidence,
the
presumption is that the members
of the buy-bust team performed
their duties in a regular manner.
The Court gives full faith and

credit to the testimonies of the


prosecution witnesses.

A search warrant or warrant


of arrest is not needed in a buybust operation where the accused
is caught in flagrante delicto in
possession
of,
and
selling,
dangerous drugs to the poseurbuyer. (People v. Araneta GR No.
191064, Oct. 20, 2010)

Facts:
Rolando Araneta aka Botong
was arrested in a buy-bust
operation in Pasig City wherein
the police officers found in
Botongs pocket one plastic
sachet of what looked like
marijuana and eight plastic
sachets
containing
white
crystalline substance. RTC and
CA found Botong guilty.

Issue:
1. Whether
or
not
the
prosecution met standard of
objective test
2. Whether or not the seized
items were admissible
Held:
Both yes.
1. The prosecution evidence met
the standard for the objective
test through the testimony of
its
witness,
PO2
Danilo

Damasco,
who
acted
as
poseur-buyer and who related
how the informant introduced
him to the accused; how the
transaction was consummated
through the exchange of
marked money and the sachet
of shabu; and how the accused
was
arrested
by
the
entrapment team. The accused
were arrested in flagrante
delicto
and
that
other
contraband materials were
recovered from them during
the ensuing search. Thus,
corpus
delicti
was
duly
established.
2. A search warrant or warrant of
arrest was not needed because
it was a buy-bust operation
and the accused were caught
in
flagrante
delicto
in
possession of, and selling,
dangerous
drugs
to
the
poseur-buyer.

18.
People v. Doria 301 SCRA
668, 694, Jan. 22, 1999
Topic:
Entrapment
and
instigation

Doctrine:
Probable
cause:
actual
belief or reasonable grounds of
suspicion.
The
grounds
of
suspicion are reasonable when, in
the absence of actual belief of the
arresting officers, the suspicion
that the person to be arrested is
probably guilty of committing the
offense, is based on actual

facts, i.e.,
supported
by
circumstances sufficiently strong
in themselves to create the
probable cause of guilt of the
person
to
be
arrested.
A
reasonable suspicion therefore
must be founded on probable
cause, coupled with good faith on
the part of the peace officers
making the arrest.

Chain of custody rule;


Objective test in buy-bust
operations:
demands
that
the details of the purported
transaction must be clearly and
adequately shown. This must
start from the initial contact
between the poseur-buyer and
the pusher, the offer to purchase,
the promise or payment of the
consideration
until
the
consummation of the sale by the
delivery of the illegal drug
subject of the sale. The manner
by which the initial contact was
made, whether or not through an
informant, the offer to purchase
the drug, the payment of the
buy-bust
money,
and
the
delivery of the illegal drug,
whether to the informant alone or
the police officer, must be the
subject of strict scrutiny by
courts to insure that law-abiding
citizens
are
not
unlawfully
induced to commit an offense.
(People v. Doria 301 SCRA 668,
694, Jan. 22, 1999)

Facts:
The
PNP
Narcotics
Command (NARCOM) acted on
information
received
from
2
informants that one Jun was
engaged in illegal drug activities in
Mandaluyong City. The NARCOM
agents entrapped and arrested Jun
in a buy-bust operation wherein
the agents bought 1 kilo of
marijuana for P1,600. When the
transaction was finished and Jun
was frisked, the agents did not find
the marked money in the person of
Jun as he left with his associate,
Neneng.
When
the
agents
proceeded to Nenengs house, they
saw a box with its flap open, which
contained 10 bricks of what
appeared to be marijuana leaves.
The agents seized the marked
money and the marijuana obtained
from both Jun and Neneng. RTC
convicted both of them.

Issue:
Whether or not the buy-bust
operation was valid

Held:
Yes.
Entrapment
is
the
conception and planning of an
offense by an officer, and his
procurement of its commission by
one
who
would
not
have
perpetrated it except for the

trickery, persuasion or fraud of the


officers. It consists of two (2)
elements: (a) acts of persuasion,
trickery, or fraud carried out by
law enforcement officers or the
agents to induce a defendant to
commit a crime; and (b) the origin
of the criminal design in the minds
of the government officials rather
than
that
of
the
innocent
defendant, such that the crime is
the product of the creative activity
of the law enforcement officer. The
type of entrapment the law forbids
is the inducing of another to
violate the law, the "seduction" of
an otherwise innocent person into
a criminal career. Where the
criminal intent originates criminal
in the mind of the entrapping
person and the accused is lured
into the commission of the offense
charged in order to prosecute him,
there is entrapment and no
conviction may be had. Where,
however,
the
criminal
intent
originates in the mind of the
accused and the criminal offense is
completed, the fact that a person
acting as a decoy for the state, or
public officials furnished the
accused
an
opportunity
for
commission of the offense, or that
the accused is aided in the
commission of the crime in order
to secure the evidence necessary
to prosecute him, there is no
entrapment and the accused must
be convicted. Entrapment is a valid
positive defense. However, for
instance, the fact that an agent of
an owner acts as a supposed

confederate of a thief is no defense


to the latter in a prosecution for
larceny, provided the original
design was formed independently
of such agent.

19.
People v. Cruz GR No.
185381, Dec. 16, 2009
Topic: Prior surveillance before
buy-bust operation

Doctrine:
A prior surveillance of the
suspected offender is not a
prerequisite for the validity of a
buy-bust operation, especially so
if
the
buy-bust
team
is
accompanied by the informant
the delivery of the illicit drug to
the poseur-buyer and the receipt
by the seller of the marked money
successfully consummate the buybust transaction. When time is of
the essence, the police may
dispense with the need for prior
surveillance.
The finding of a dangerous
drug in the house or within the
premises of the house of the
accused is prima facie evidence
of
knowledge
or
animus
possidendi and is enough to
convict in the absence of a
satisfactory explanation. (People
v. Cruz GR No. 185381, Dec. 16,
2009)

Facts:
An informant came to the
Drug Enforcement Unit of the
Taguig City Police and reported
that a certain Danilo Cruz alias
Boy was dealing in illegal drugs
at his residence at 75 MLQ
Street, Tambak, Wawa, Taguig,
Metro
Manila.
The
police
immediately formed a buy-bust
team and prepared 2 100-peso
bills as marked money. The
informant and PO3 Arago then
went to Boys house. Alias Boy
gave PO3 Arago a plastic sachet
containing a white crystalline
substance in exchange for the
P200 marked money. When PO3
Arago ordered appellant to empty
his pockets for any concealed
weapons,
PO2
Aguinaldo
retrieved 2 more plastic sachets
containing
white
crystalline
substance. PO3 Arago inscribed
his signature and the appellants
initials DCC on the sachet given
him by appellant, while PO2
Aguinaldo inscribed those found
in appellants pockets as DCC-1
and DCC-2. RTC and CA found
Cruz guilty.

Issue:
Whether or not the buy-bust
operation was valid

Held:
Yes.

A prior surveillance of the


suspected offender is not a
prerequisite for the validity of a
buy-bust operation, especially so
if
the
buy-bust
team
is
accompanied by the informant.
When time is of the essence, the
police may dispense with the
need for prior surveillance.

20.
People v. Hernandez GR
No. 184804, June 18, 2009
Topic: Absence of record in
police blotter

Doctrine:
Neither
law
nor
jurisprudence requires that the
buy-bust money be entered in the
police blotter. The non-recording
of the buy-bust operation and
buy-bust money in the police
blotter is not essential, since they
are not elements in the illegal
sale of dangerous drugs. (People
v. Hernandez GR No. 184804,
June 18, 2009)

Facts:
An informant went to the
Station Anti-Illegal Drugs (SAID)
Unit of Central Market, Sta. Cruz
Manila Police Station 3 (police
station) and reported the drug
trafficking activities of a certain
Larry and appellants in Callejon

Flores,
Solis
Street,
Tondo,
Manila. A buy-bust team was
formed and proceeded to Larrys
house where the team seized
P200 worth of shabu. RTC and CA
found the accused guilty.

Issue:
Whether
the
buy-bust
operation was valid and the
seized shabu admissible

Held:
Yes.
Neither
law
nor
jurisprudence requires that the
buy-bust money be entered in the
police blotter. the non-recording
of the buy-bust operation and
buy-bust money in the police
blotter is not essential, since they
are not elements in the illegal
sale of dangerous drugs. the only
elements
necessary
to
consummate the crime is proof
that the illicit transaction took
place,
coupled
with
the
presentation in court of the
dangerous
drug
seized
as
evidence. Both were satisfactorily
proved in the present case.

21.
Judge Abelita III v. P/Supt.
Doria and SP03 Ramirez GR
No. 170672, Aug. 14, 2009
Topic: Plain view doctrine

Doctrine:
Under
the
plain
view
doctrine, objects falling in the
plain view of an officer who has a
right to be in the position to have
that view are subject to seizure
and may be presented as
evidence.
The
plain
view
doctrine
applies
when
the
following requisites concur: (1)
the law enforcement officer in
search of the evidence has a prior
justification for an intrusion or is
in a position from which he can
view a particular area; (2) the
discovery of the evidence in plain
view is inadvertent; and (3) it is
immediately apparent to the
officer that the item he observes
may be evidence of a crime,
contraband or otherwise subject
to seizure. (Judge Abelita III v.
P/Supt. Doria and SP03 Ramirez
GR No. 170672, Aug. 14, 2009)

Facts:
The case involves a seizure
of
a
licensed
shotgun
in
connection
with
a
shooting
incident. Upon receiving a call,
P/Supt. German B. Doria and
SPO3
Cesar
Ramirez
were
dispatched
to
investigate
a
shooting incident in Barangay
Nursery. A certain William Sia
was wounded while the judge,
who was implicated in the

incident, and his wife just left the


place of the incident. Doria
looked for the judge and asked
him to come with him to the
police station but the judge sped
up his car, prompting Doria to
chase the former towards his
house. The police officers then
saw a gun in the front seat of the
vehicle beside the drivers seat as
the judge opened the door. They
also saw a shotgun at the back of
the drivers seat. The police
officers confiscated the firearms
and arrested the judge. RTC ruled
against the judge.

Issue:
Whether the
warrantless
arrest and warrantless search
and seizure were illegal

and may be presented as evidence.


The plain view doctrine applies
when the following requisites
concur: (1) the law enforcement
officer in search of the evidence
has a prior justification for an
intrusion or is in a position from
which he can view a particular
area; (2) the discovery of the
evidence
in
plain
view
is
inadvertent;
and
(3)
it
is
immediately
apparent
to
the
officer that the item he observes
may be evidence of a crime,
contraband or otherwise subject to
seizure. Since a shooting incident
just took place and it was reported
that petitioner was involved in the
incident, it was apparent to the
police officers that the firearms
may be evidence of a crime.
Hence, they were justified in
seizing the firearms.

Held:
No.
For the warrantless arrest
under this Rule to be valid, two
requisites must concur: (1) the
offender has just committed an
offense; and (2) the arresting
peace officer or private person has
personal
knowledge
of
facts
indicating that the person to be
arrested has committed it.
Under
the
plain
view
doctrine, objects falling in the
plain view of an officer who has a
right to be in the position to have
that view are subject to seizure

22.
United Laboratories v. Isip
461 SCRA 574, June 28, 2005
Topic: Inadvertence requirement

Doctrine:
A search warrant proceeding
is, in no sense, a criminal action
or the commencement of a
prosecutionit is a special and
peculiar
remedy,
drastic
in
nature, and made necessary
because of public necessity,
resembling in some respect with

what is commonly known as John


Doe proceedings.

A search warrant is a legal


process which has been likened
to a writ of discovery employed
by the State to procure relevant
evidence of crimeit is in the
nature of a criminal process,
restricted to cases of public
prosecutions and not a process
for adjudicating civil rights or
maintaining mere private rights

A search warrant, to be
valid, must particularly describe
the place to be searched and the
things to be seized. The officers
of the law are to seize only those
things particularly described in
the search warrant. A search
warrant is not a sweeping
authority empowering a raiding
party to undertake a fishing
expedition to seize and confiscate
any and all kinds of evidence or
articles relating to a crime. The
search is limited in scope so as
not to be general or exploratory.
Nothing is left to the discretion of
the officer executing the warrant.

The immediate requirement


means that the executing officer
can, at the time of discovery of
the object or the facts therein
available to him, determine
probable cause of the objects
incriminating evidenceto be

immediate, probable cause must


be the direct result of the
officers instantaneous sensory
perception of the object. The
object is apparent if the executing
officer had probable cause to
connect the object to criminal
activity. The incriminating nature
of
the
evidence
becomes
apparent in the course of the
search, without the benefit of any
unlawful search or seizure.

The
requirement
of
inadvertence means that the
officer must not have known in
advance of the location of the
evidence and intend to seize it.
Discovery is not anticipated.

The immediately apparent


test does not require an unduly
high degree of certainty as to the
incriminating
character
of
evidenceincriminating
means
the furnishing of evidence as
proof of circumstances tending to
prove the guilt of a person.
(United Laboratories v. Isip 461
SCRA 574, June 28, 2005)

Facts:
Rolando H. Besarra, NBI Special
Investigator
III
filed
an
application, in the RTC of Manila,
for the issuance of a search
warrant concerning the first and
second floors of the Shalimar

Building, and for the seizure of


fake Revicon multivitamins for
violation of Section 4(a), in relation
to Section 8, of R.A. No. 8203. The
court granted the application for
search warrant and the NBI
proceeded
to
the
building.
However,
no
fake
Revicon
multivitamins were found, only 792
Bottles Disudrin 60 ml. and 30
Boxes (100 pieces each) Inoflox
200 mg.

Issue:
Whether or not the seizure of
the counterfeit drugs is justified
and lawful under the plain view
doctrine and are legally admissible
as evidence

Held:
No.
The
requirement
of
inadvertence means that the
officer must not have known in
advance of the location of the
evidence and intend to seize it.
Discovery is not anticipated. It
requires that the seizure be
presumptively
reasonable
assuming that there is probable
cause to associate the property
with criminal activity; that a
nexus exists between a viewed
object and criminal activity. The
search
warrant
particularly
described that only Revicon
products were to be seized. NBI
did not adduce evidence that the

products they found were in plain


view and thus admissible as
evidence.

23.
Terry v. Ohio 392 US 1
(1968)
Topic: Stop and frisk
Doctrine:
Where
a
police
officer
observes unusual conduct which
leads him reasonably to conclude
in light of his experience that
criminal activity may be afoot and
that the persons with whom he is
dealing may be armed and
presently dangerous, where in
the course of investigating this
behavior he identifies himself as a
policeman and makes reasonable
inquiries, and where nothing in
the initial stages of the encounter
serves to dispel his reasonable
fear for his own or others safety,
he is entitled for the protection of
himself and others in the area to
conduct a carefully limited search
of the outer clothing of such
persons in an attempt to discover
weapons which might be used to
assault him. Such a search is a
reasonable search under the
Fourth amendment.

While probable cause is not


required to conduct a stop-andfrisk, it nevertheless holds that
mere suspicion or a hunch will
not validate a stop-and-frisk. A

genuine reason must exist, in


light of the police officers
experience
and
surrounding
conditions, to warrant the belief
that the person detained has
weapons concealed about him.

Stop-and-frisk serves a
two-fold interest: (1) the general
interest
of
effective
crime
prevention and detection, which
underlies the recognition that a
police
officer
may,
under
appropriate circumstances and in
an appropriate manner, approach
a
person
for
purposes
of
investigating possible criminal
behavior even without probable
cause; and (2) the more pressing
interest of safety and selfpreservation which permit the
police officer to take steps to
assure himself that the person
with whom he deals is not armed
with a deadly weapon that could
unexpectedly, and fatally be used
against the police officer.

Valid stop requires that


the
police
officer
has
a
reasonable and articulable belief
that a criminal activity has
happened or is about to happen

Frisk
- must be done because of a
reasonable belief that the person
stopped is in possession of a

weapon that will pose a danger to


the officer and others
- must be a mere pat down outside
the persons outer garment and
not unreasonably intrusive (Terry
v. Ohio 392 US 1 (1968)

Facts:
On Oct. 31, 1963 while
Martin McFadden, a police officer
for 30 years, was in his usual beat
in downtown Cleveland, he saw
Terry and Chilton, who by their
acts appeared to be engaged in a
store robbery. Suspecting them to
be armed and fearing that they
were to rob the store, McFadden
approached them and introduced
himself a police officer and asked
them to identify themselves.
When Terry and Ohio simply
mumbled and did not give a clear
and audible response, McFadden
patted
down
Terrys
outer
garment and felt a gun in his
pocket and removed it. A gun was
also recovered from Chilton.
Terry was convicted of carrying a
concealed weapon, which the
Ohio Court of Appeals affirmed.

Issue:
Whether or not probable
cause is required in stop and frisk
cases

Held:

No.
The acts of McFadden were
that which a reasonably prudent
man would have done, believing
Terry was armed and presented a
threat to the formers safety. The
more
immediate
interest
of
McFadden in taking steps to
assure himself that Terry was not
armed with a weapon that could
unexpectedly and fatally be used
against the former is more
important. McFaddens actions
were not invasive and overly
intrusive as he did not place his
hands in Terrys and Chiltons
pockets but merely patted down
their outer clothing. He confined
his search to what was minimally
necessary and did not conduct a
general exploratory search. Thus,
it was a proper stop and an
incident frisk.

24.
Malacat v. CA 283 SCRA
159, Dec. 12, 1997
Topic: Stop and frisk vs. search
incident to a lawful arrest

Doctrine:
Stop and frisk and search
incidental to a lawful arrest differ
in terms of the requisite quantum
of proof before they may be
validly effected and in their
allowable scope. In a search
incidental to a lawful arrest, as
the precedent arrest determines

the validity of the incidental


search, the legality of the arrest
is questioned in a large majority
of these cases, e.g., whether an
arrest was merely used as a
pretext for conducting a search.
In this instance, the law requires
that there first be a lawful arrest
before a search can be madethe
process cannot be reversed. At
bottom, assuming a valid arrest,
the arresting officer may search
the person of the arrestee and the
area within which the latter may
reach for a weapon or for
evidence to destroy, and seize any
money or property found which
was used in the commission of
the crime, or the fruit of the
crime, or that which may be used
as evidence, or which might
furnish the arrestee with the
means of escaping or committing
violence.

Reliable information alone,


absent any overt act indicative of
a felonious enterprise in the
presence and within the view of
the arresting officers, is not
sufficient to constitute probable
cause that would justify an in
flagrante delicto arrest. (Malacat
v. CA 283 SCRA 159, Dec. 12,
1997)

Facts:
In response to bomb threats
reported 7 days earlier, Rodolfo

Yu of the Western Police District


was on foot patrol with 3 other
police officers along Quezon
Boulevard, Quiapo, Manila, near
the Mercury Drug store at Plaza
Miranda. They chanced upon two
groups of Muslim-looking men
who were acting suspiciously
with their eyes moving very fast.
When the police approached the
men, they tried to escape but Yu
caught up with Sammy Malacat
and recovered a fragmentation
grenade tucked in his waistline.
The police also recovered from
Abdul Casan from a .38 caliber
revolver. The trial court ruled
that the warrantless search and
seizure of petitioner was akin to a
stop and frisk, where a warrant
and seizure can be effected
without
necessarily
being
preceded by an arrest and whose
object is either to maintain the
status quo momentarily while the
police officer seeks to obtain
more information. RTC found
petitioner guilty of the crime of
illegal possession of explosives.

Issue:
Whether stop and frisk and
search incident to a lawful arrest
are the same

Held:
No.
The trial court confused the
concepts of a "stop-and-frisk" and

of a search incidental to a lawful


arrest. These two types of
warrantless searches differ in
terms of the requisite quantum of
proof before they may be validly
effected and in their allowable
scope. In a search incidental to a
lawful arrest, the precedent
arrest determines the validity of
the incidental search. The law
requires that there first be a
lawful arrest before a search can
be made -- the process cannot be
reversed. The arresting officer
may search the person of the
arrestee and the area within
which the latter may reach for a
weapon or for evidence to
destroy, and seize any money or
property found which was used in
the commission of the crime, or
the fruit of the crime, or that
which may be used as evidence,
or which might furnish the
arrestee with the means of
escaping or committing violence.
The
search
conducted
on
petitioner could not have been
one incidental to a lawful arrest.
While probable cause is not
required to conduct a "stop and
frisk," mere suspicion or a hunch
will not validate it. A genuine
reason must exist, in light of the
police officer's experience and
surrounding conditions, to warrant
the belief that the person detained
has weapons concealed about him.
a "stop-and-frisk" serves a two-fold
interest: (1) the general interest of
effective crime prevention and
detection, which underlies the

recognition that a police officer


may,
under
appropriate
circumstances
and
in
an
appropriate manner, approach a
person
for
purposes
of
investigating
possible
criminal
behavior even without probable
cause; and (2) the more pressing
interest
of
safety
and
selfpreservation which permit the
police officer to take steps to
assure himself that the person
with whom he deals is not armed
with a deadly weapon that could
unexpectedly and fatally be used
against the police officer.

25.
Valdez v. People 538 SCRA
611, Nov. 23, 2007
Topic: Consented searches
Doctrine:
The consent to a warrantless
search must be voluntary, that is,
it must be unequivocal, specific,
and
intelligently
given,
uncontaminated by any duress or
coercion; Consent to a search is
not to be lightly inferred, but
must be shown by clear and
convincing evidence; It is the
State which has the burden of
proving, by clear and positive
testimony, that the necessary
consent was obtained and that it
was freely and voluntarily given.
(Valdez v. People 538 SCRA 611,
Nov. 23, 2007)
Facts:

Barangay
Tanod
Rogelio
Bautista was conducting a routine
patrol
along
the
National
Highway in Barangay San Benito
Norte, Aringay, La Union together
with Aratas and Ordoo when they
noticed Arsenio Valdez, lugging a
bag, alight from a mini-bus. The
tanods observed that Valdez, who
appeared suspicious to them,
seemed
to
be
looking
for
something. They thus approached
him but the latter purportedly
attempted to run away. They
chased
him,
confiscated
marijuana, and put him under
arrest. RTC and CA found Valdez
guilty.
Issue:
Whether
or
not
Valdez
validly consented to the search
Held:
No.
The
consent
must
be
voluntary in order to validate an
otherwise illegal detention and
search, i.e., the consent is
unequivocal,
specific,
and
intelligently
given,
uncontaminated by any duress or
coercion. Consent to a search is
not to be lightly inferred, but
must be shown by clear and
convincing evidence. Relevant to
this
determination
are
the
following characteristics of the
person giving consent and the
environment in which consent is
given: (1) the age of the

defendant; (2) whether he was in


a public or secluded location; (3)
whether he objected to the search
or passively looked on; (4) the
education and intelligence of the
defendant; (5) the presence of
coercive police procedures; (6)
the defendant's belief that no
incriminating evidence will be
found; (7) the nature of the police
questioning; (8) the environment
in which the questioning took
place; and (9) the possibly
vulnerable subjective state of the
person consenting. A waiver of an
illegal warrantless arrest does
not also mean a waiver of the
inadmissibility of evidence seized
during an illegal warrantless
arrest. Valdez was acquitted.
26.
Stonehill v. Diokno
SCRA 383, June 19, 1967

20

Doctrine:
The Moncado ruling, that
illegally
seized
documents,
papers and things are admissible
in evidence, must be abandoned.
The exclusion of such evidence is
the only practical means of
enforcing
the
constitutional
injunction against unreasonable
searches and seizures. The nonexclusionary rule is contrary to
the letter and spirit of the
prohibition against unreasonable
searches and seizures. If there is
competent evidence to establish
probable cause of the commission

of a given crime by the party


against whom the warrant is
intended, then there is no reason
why the applicant should not
comply with the constitutional
requirements. If he has no such
evidence, then it is not possible
for the judge to find that there is
a probable cause, and, hence, no
justification for the issuance of
the warrant. The only possible
explanation for the issuance in
that case is the necessity of
fishing for evidence of the
commission of a crime. Such a
fishing expedition is indicative of
the absence of evidence to
establish
a
probable
cause.
(Stonehill v. Diokno 20 SCRA 383,
June 19, 1967)
Facts:
Upon application of the
officers of the government, Special
Prosecutors Pedro D. Cenzon,
Efren I. Plana and Manuel Villareal
Jr. and Assistant Fiscal Manases G.
Reyes;
Judge
Amado
Roan
(Municipal Court of Manila), Judge
Roman Cansino (Municipal Court
of Manila), Judge Hermogenes
Caluag (Court of First Instance of
Rizal-Quezon City Branch), and
Judge Damian Jimenez (Municipal
Court of Quezon City) issued, on
different dates, a total of 42 search
warrants
against
Harry
S.
Stonehill, Robert P. Brooks, HJohn
J. Brooks, and Karl Beck, and/or
the corporations of which they

were officers, directed to any


peace officer, to search the said
persons and/or the premises of
their offices, warehouses and/or
residences, and to seize and take
possession
of
the
following
personal property to wit: "Books of
accounts,
financial
records,
vouchers,
correspondence,
receipts,
ledgers,
journals,
portfolios,
credit
journals,
typewriters, and other documents
and/or papers showing all business
transactions
including
disbursements receipts, balance
sheets
and
profit
and
loss
statements and Bobbins (cigarette
wrappers)" as "the subject of the
offense; stolen or embezzled and
proceeds or fruits of the offense,"
or "used or intended to be used as
the means of committing the
offense," which is described in the
applications adverted to above as
"violation of Central Bank Laws,
Tariff and Customs Laws, Internal
Revenue (Code) and the Revised
Penal Code." Alleging that the
search warrants are null and void,
as contravening the Constitution
and the Rules of Court, Stonehill,
et. al. filed with the Supreme
Court the original action for
certiorari, prohibition, mandamus
and injunction. On 22 March 1962,
the Supreme Court issued the writ
of preliminary injunction prayed
for in the petition. However, by
resolution dated 29 June 1962, the
writ was
partially
lifted
or
dissolved, insofar as the papers,
documents and things seized from

the offices of the corporations are


concerned; but, the injunction was
maintained as regards the papers,
documents and things found and
seized in the residences of
Stonehill, et. al.

Issue:
Whether Stonehill, et. al. can
assail the legality of the contested
warrants that allowed seizure of
documents, papers and other
effects in the corporate offices,
and other places besides their
residences.

Held:
Stonehill, et. al. maintained
that the search warrants are in the
nature of general warrants and
that, accordingly, the seizures
effected upon the authority thereof
are null and void. No warrant shall
issue but upon probable cause, to
be determined by the judge in the
manner set forth in said provision;
and the warrant shall particularly
describe the things to be seized.
None of these requirements has
been
complied
with
in
the
contested warrants. The grave
violation of the Constitution made
in the application for the contested
search warrants was compounded
by the description therein made of
the effects to be searched for and
seized. The warrants authorized
the search for and seizure of
records pertaining to all business

transactions of Stonehill, et. al.,


regardless
of
whether
the
transactions were legal or illegal.
The warrants sanctioned the
seizure of all records of the
corporate
officers
and
the
corporations,
whatever
their
nature, thus openly contravening
the explicit command of our Bill of
Rights that the things to be
seized be particularly described
as well as tending to defeat its
major objective: the elimination of
general warrants. However, the
documents, papers, and things
seized under the alleged authority
of the warrants in question may be
split into (2) major groups, namely:
(a) those found and seized in the
offices of the corporations and (b)
those
found
seized
in
the
residences of Stonehill, et. al. As
regards the first group, Stonehill,
et. al. have no cause of action to
assail the legality of the contested
warrants and of the seizures made
in pursuance thereof, for the
simple
reason
that
said
corporations have their respective
personalities, separate and distinct
from the personality of Stonehill,
et. al., regardless of the amount of
shares of stock or of the interest of
each of them in said corporations,
and whatever the offices they hold
therein may be. Indeed, it is well
settled that the legality of a
seizure can be contested only by
the party whose rights have been
impaired thereby, and that the
objection to an unlawful search
and seizure is purely personal and

cannot be availed of by third


parties. Consequently, Stonehill,
et. al. may not validly object to the
use in evidence against them of
the documents, papers and things
seized from the offices and
premises of the corporations
adverted to above, since the right
to object to the admission of said
papers
in
evidence
belongs
exclusively to the corporations, to
whom the seized effects belong,
and may not be invoked by the
corporate officers in proceedings
against them in their individual
capacity. With respect to the
documents, papers and things
seized in the residences of
Stonehill, et. al., the 29 June 1962
Resolution of the Supreme Court,
denying the lifting of the writ of
preliminary injunction previously
issued by the Court on the
documents, papers and things
seized in the residences, in effect,
restrained the prosecutors from
using them in evidence against
Stonehill, et. al. Thus, the Court
held that the warrants for the
search of 3 residences are null and
void; that the searches and
seizures therein made are illegal;
that the writ of preliminary
injunction heretofore issued, in
connection with the documents,
papers and other effects thus
seized in said residences is made
permanent, that the writs prayed
for are granted, insofar as the
documents, papers and other
effects so seized in the residences
are concerned; and that the

petition herein is dismissed and


the writs prayed for denied, as
regards the documents, papers
and other effects seized in the 29
places, offices and other premises.

27.
Del Rosario, et al. v.
Doanto, Jr., et al. GR No.
180595, March 5, 2010
Topic: Civil damages; criminal
liability

searched a house at 51 New York


Street,
Villasol
Subdivision,
Angeles City. Their search yielded
no fake Marlboro cigarettes.
Alexander and Arthur del Rosario
filed a complaint for P50 million
in damages against NBI agents
Donato and Gonzaga and two
others before the RTC of Angeles
City. RTC ruled in favor of the Del
Rosarios.
CA
reversed
the
decision.

Issue:
Doctrine:
The proceeding under Rule
126, a limited criminal one, does
not provide for the filing of
counterclaims
for
damages
against those who may have
improperly sought the issuance of
the search warrant. However, the
aggrieved party have the right to
seek
damages,
if
the
circumstances
warranted,
by
separate civil action for the
wrong inflicted on them by an
improperly obtained or enforced
search warrant. (Del Rosario, et
al. v. Doanto, Jr., et al. GR No.
180595, March 5, 2010)

Facts:
The case was about the
proliferation of fake Marlboro
cigarettes in Angeles, Pampanga.
By virtue of a search warrant
procured later, the NBI agents

Whether or not the CA


correctly ruled that the complaint
of the Del Rosarios did not state a
cause of action

Held:
The proceeding under Rule
126, a limited criminal one, does
not provide for the filing of
counterclaims for damages against
those who may have improperly
sought the issuance of the search
warrant. Consequently, the Del
Rosarios had the right to seek
damages, if the circumstances
warranted, by separate civil action
for the wrong inflicted on them by
an
improperly
obtained
or
enforced
search
warrant.
Unfortunately, their complaint, as
worded, failed to state a proper
cause of action.

28.
Marimla v. People GR No.
158467, 2009
Topic: Authority of the Executive
Judge and Vice Executive Judge
re: search warrants in Manila and
Quezon City

was defective as only the deputy


director and not the director
signed it

Doctrine:
Nothing in A.M. No. 99-1009-SC prohibits the heads of the
PNP, NBI, PAOC-TF and REACTTF
from
delegating
their
ministerial duty of endorsing the
application for search warrant to
their assistant heads. Under
Section 31, Chapter 6, Book IV of
the Administrative Code of 1987,
an assistant head or other
subordinate in every bureau may
perform such duties as may be
specified by their superior or
head, as long as it is not
inconsistent with law. (Marimla v.
People GR No. 158467, 2009)

Nothing in A.M. No. 99-1009-SC prohibits the heads of the


PNP, NBI, PAOC-TF and REACT-TF
from delegating their ministerial
duty of endorsing the application
for search warrant to their
assistant heads. Under Section 31,
Chapter 6, Book IV of the
Administrative Code of 1987, an
assistant
head
or
other
subordinate in every bureau may
perform such duties as may be
specified by their superior or head,
as long as it is not inconsistent
with law.

Facts:
NBI Special Investigator Ray
Lagasca filed 2 applications for
search warrant, signed by Deputy
Director Fermin Nasol, with the
RTC of Manila to search the
petitioners houses in connection
with illegal drug operations.
Executive Judge Mario Guaria III
granted the applications and the
NBI agents then proceeded to
search the house and seize drugs.
Issue:
Whether
or
not
the
application for search warrant

Held:
No.

BAIL

29.
People v. Manallo 400
SCRA 129, March 28, 2003
Topic: bail application
Doctrine:
A bail application does not
only involve the right of the
accused to temporary liberty, but
likewise the right of the State to
protect the people and the peace
of the community from dangerous
elements two rights which must
be balanced by a magistrate in
the scale of justice, hence, the

necessity for hearing to guide his


exercise of jurisdiction. (People v.
Manallo 400 SCRA 129, March
28, 2003)
Facts:
The case involved a rape
incident of Rosaldiza by Alex
Manallo, the victims familys
coconut gatherer. Manallo filed a
motion for bail, which was
granted by the executive judge on
the same date it was filed, and
fixed it at P50,000. Manallo
posted a property bond and was
released. After trial, RTC found
Manallo guilty.
Issue:
Whether the executive judge
committed gross ignorance of the
law
Held:
Yes.
A bail application does not
only involve the right of the
accused to temporary liberty, but
likewise the right of the State to
protect the people and the peace
of the community from dangerous
elements. These two rights must
be balanced by a magistrate in the
scale of justice, hence, the
necessity for hearing to guide his
exercise
of
jurisdiction.
The
decision did not state its factual
basis and the bail application was
granted without even affording the
prosecution a chance to be heard

thereon and adduce its evidence in


opposition thereto.

30.
Paderanga v. CA 247 SCRA
741, Aug. 28, 1995
Topic: bail as a Constitutional
right

Doctrine:
As bail is intended to obtain
or
secure
ones
provisional
liberty, the same cannot be
posted before custody over him
has been acquired by the judicial
authorities, either by his lawful
arrest or voluntary surrender.

A person is considered to be
in the custody of the law (a) when
he is arrested either by virtue of a
warrant of arrest issued pursuant
to Section 6, Rule 112, or by
warrantless arrest under Section
5, Rule 113 in relation to Section
7, Rule 112 of the revised Rules
on Criminal Procedure, or (b)
when
he
has
voluntarily
submitted
himself
to
the
jurisdiction of the court by
surrendering
to
the
proper
authorities.

Where bail is a matter of


right, upon proper application for
admission to bail, the court
having custody of the accused
should, as a matter of course,

grant the same after a hearing


conducted
to
specifically
determine the conditions of the
bail.

Where the grant of bail


becomes a matter of judicial
discretion, a hearing, mandatory
in nature and which should be
summary or otherwise in the
discretion of the court, is
required with the participation of
both the defense and a duly
notified representative of the
prosecution, to ascertain whether
or not the evidence of guilt is
strong for the provisional liberty
of the applicant.

Procedure to be followed
when the grant of bail is a matter
of judicial discretion:
Where such a hearing is set upon
proper motion or petition, the
prosecution must be given an
opportunity to present, within a
reasonable time, all the evidence
that it may want to introduce
before the court may resolve the
application, since it is equally
entitled as the accused to due
process. If the prosecution is
denied this opportunity, there
would be a denial of procedural
due process, as a consequence of
which the courts order in respect
of the motion or petition is void.
At the hearing, the petitioner can
rightfully
cross-examine
the

witnesses
presented
by
the
prosecution and introduce his
own evidence in rebuttal. When,
eventually, the court issues an
order either granting or refusing
bail, the same should contain a
summary of the evidence for the
prosecution, followed by its
conclusion as to whether or not
the evidence of guilt is strong.
The court, though, cannot rely on
mere affidavits or recitals of their
contents, if timely objected to, for
these represent only hearsay
evidence,
and
thus
are
insufficient
to
establish
the
quantum of evidence that the law
requires. (Paderanga v. CA 247
SCRA 741, Aug. 28, 1995)

Facts:
Miguel
Paderanga
was
charged as co-conspirator in a
multiple murder case. Paderanga
filed a motion for admission to
bail, which the trial court granted
and fixed at P200,000, despite his
non-appearance
at
the
bail
hearing as he was then confined
in a hospital due to "acute
costochondritis.

Issue:
Whether or not Paderanga
was correctly admitted to bail

Held:

Yes.
The right to bail is personal
in nature and may be waived. It
arises from the time one is placed
in the custody of the law, springs
from the presumption of innocence
accorded every accused upon
whom should not be inflicted
incarceration at the outset since
after trial he would be entitled to
acquittal, unless his guilt be
established beyond reasonable
doubt.

31.
Heirs of Sarah Marie
Palma Burgos v. CA and
Johnny Co Yu GR No. 169711,
Feb. 8, 2010
Topic: bail has no impact on civil
liability

Doctrine:
A criminal case has two
aspects,
the
civil
and
the
criminal. The civil aspect is borne
of the principle that every person
criminally liable is also civilly
liable. The civil action, in which
the offended party is the plaintiff
and the accused is the defendant,
is deemed instituted with the
criminal
action
unless
the
offended party waives the civil
action or reserves the right to
institute it separately or institutes
the civil action prior to the
criminal action.

The law allows the merger of


the criminal and the civil actions
to avoid multiplicity of suits.
Thus, when the state succeeds in
prosecuting the offense, the
offended party benefits from such
result and is able to collect the
damages awarded to him.

When the trial court acquits


the accused or dismisses the case
on the ground of lack of evidence
to prove the guilt of the accused
beyond reasonable doubt, the
civil action is not automatically
extinguished since liability under
such an action can be determined
on
mere
preponderance
of
evidence.

The question of granting bail


to the accused is but an aspect of
the criminal action, preventing
him from eluding punishment in
the event of conviction. The grant
of bail or its denial has no impact
on the civil liability of the accused
that depends on conviction by
final judgment. (Heirs of Sarah
Marie Palma Burgos v. CA and
Johnny Co Yu GR No. 169711,
Feb. 8, 2010)

Facts:
This case is about the legal
standing of the offended parties in

a criminal case to seek, in their


personal capacities and without
the
Solicitor
Generals
intervention, reversal of the trial
courts order granting bail to the
accused on the ground of absence
of strong evidence of guilt.
Cresencio Aman, Romeo Martin,
Artemio Pong Bergonia, Danilo
Say, and Johnny Co killed Sarah
Palma in her sleep and her uncle
Erasmo Palma. Co filed a petition
for admission to bail, which the
trial court granted. RTC denied
Sarahs
heirs
petition
for
reconsideration, prompting them
to file a special civil action for
certiorari with CA. CA dismissed
the petition or having been filed
without involving the Office of the
Solicitor General (OSG).

Issue:
Whether or not the CA
correctly dismissed the special
civil action of certiorari

Held:
Yes.
Actions essentially involving
the interest of the state, if not
initiated by the Solicitor General,
are,
as
a
rule,
summarily
dismissed.
The question of granting bail
to the accused is but an aspect of
the criminal action, preventing him
from eluding punishment in the

event of conviction. The grant of


bail or its denial has no impact on
the civil liability of the accused
that depends on conviction by final
judgment. Co has already been
arraigned. Trial and judgment,
with award for civil liability when
warranted, could proceed even in
his absence.

32.
People v. Cabral GR No.
131909, Feb. 18, 1999
Topic: proof required in granting
or denying bail

Doctrine:
The grant or denial of an
application for bail is dependent
on whether the evidence of guilt
is strong which the lower court
should determine in a hearing
called for the purpose.

Proof
evident
or
Evident
proof
in
this
connection has been held to
mean clear,
strong
evidence
which leads a well-guarded
dispassionate judgment to the
conclusion that the offense has
been committed as charged, that
accused is the guilty agent, and
that he will probably be punished
capitally
if
the
law
is
administered.
Presumption
great exists
when
the
circumstances testified to are

such that the inference of guilt


naturally to be drawn therefrom
is strong, clear, and convincing to
an
unbiased
judgment
and
excludes
all
reasonable
probability
of
any
other
conclusion. Even though there is
a reasonable doubt as to the guilt
of accused, if on an examination
of
the
entire
record
the
presumption is great that accused
is guilty of a capital offense, bail
should be refused.

Duties of a judge in case an


application for bail is filed
(1) Notify the prosecutor of the
hearing of the application for bail
or require him to submit his
recommendation; (2) Conduct a
hearing of the application for bail
regardless of whether or not the
prosecution refuses to present
evidence to show that the guilt of
the accused is strong for the
purpose of enabling the court to
exercise its discretion; (3) Decide
whether the evidence of guilt of
the accused is strong based on
the summary of evidence of the
prosecution; (Italics supplied) (4)
If the guilt of the accused is not
strong, discharge the accused
upon
the
approval
of
the
bailbond.
Otherwise,
petition
should be denied. (People v.
Cabral GR No. 131909, Feb. 18,
1999)

Facts:
Accused-respondent
Roderick Odiamar was charged
with rape upon the complaint of
Cecille Buenafe. In a bid to secure
temporary liberty, filed a motion
praying that he be released on bail
which
petitioner
opposed
by
presenting real, documentary and
testimonial evidence. The lower
court,
however,
found
that
evidence of guilt was not strong
ang granted the motion for bail.
CA denied the Peoples petition
withprayer
for
temporary
restraining order and preliminary
injunction.

Issue:
Whether
or
not
CA
committed
grave
abuse
of
discretion in denying the petition

Held:
Yes.
Accused-respondent
was
being charged with rape qualified
by the use of a deadly weapon
punishable by reclusion perpetua
to death. As such, bail is
discretionary and not a matter of
right. The grant or denial of an
application for bail is, therefore,
dependent
on
whether
the
evidence of guilt is strong which
the lower court should determine
in a hearing called for the purpose.
The determination of whether the

evidence of guilt is strong, in this


regard, is a matter of judicial
discretion. The test is not whether
the evidence establishes guilt
beyond reasonable doubt but
rather whether it shows evident
guilt or a great presumption of
guilt.

33.
Dipatuan v. Mangotara
AM No. RTJ-09-2190, April 23,
2010
Topic: when admission to bail is
discretionary

inference of bad faith, making the


judge liable for gross ignorance
of
the
law.
(Dipatuan
v.
Mangotara AM No. RTJ-09-2190,
April 23, 2010)

Facts:
Hadja
Sohurah
Dipatuan
filed a complaint against Judge
Mamindiara
P.
Mangotara,
Presiding Judge, RTC of Iligan
City,
Branch
1,
for
Gross
Ignorance of the Law and Grave
Abuse of Authority for increasing
accuseds bail bond in a criminal
case for murder from P75,000.00
to P200,000.00.

Doctrine:
In
offenses
punishable
by reclusion perpetua or death,
the accused has no right to bail
when the evidence of guilt is
strong. Thus, as the accused in
Criminal Case No. 3620-01 had
been
sentenced
to reclusion
perpetua, the bail should have
been
cancelled,
instead
of
increasing it as respondent Judge
did. The act of Mangotara in
increasing the bail bond of the
accused instead of cancelling it is
not
a
mere
deficiency
in
prudence,
discretion
and
judgment
on
the
part
of
respondent Judge, but a patent
disregard of well-known rules.
When an error is so gross and
patent, such error produces an

Issue:
Whether or not the judge is
guilty as charged
Held:
Yes.
In offenses punishable by
reclusion perpetua or death, the
accused has no right to bail when
the evidence of guilt is strong.
Thus, as the accused in Criminal
Case No. 3620-01 had been
sentenced to reclusion perpetua,
the bail should have been
cancelled, instead of increasing it
as respondent Judge did. The act
of Mangotara in increasing the
bail bond of the accused instead
of cancelling it is not a mere

deficiency in prudence, discretion


and judgment on the part of
respondent Judge, but a patent
disregard of well-known rules.

the accused is under bond for


appearance at trial in other
cases. (De la Camara v. Enage 41
SCRA 1, Sept. 17, 1971)

Facts:
34.
De la Camara v. Enage 41
SCRA 1, Sept. 17, 1971
Topic: excessive bail

Doctrine:
Lower court judges have the
obligation of fidelity to the
unequivocal command of the
Constitution that excessive bail
shall not be required. Where the
right to bail exists, it should not
be
rendered
nugatory
by
requiring a sum that is excessive.
If there were no such prohibition,
the right to bail becomes
meaningless.

Guidelines in the fixing of


bail
(1) ability of the accused to give
bail; (2) nature of the offense; (3)
penalty for the offense charged;
(4) character and reputation of
the accused; (5) health of the
accused;
(6)
character
and
strength of the evidence; (7)
probability
of
the
accused
appearing in trial; (8) forfeiture of
other bonds; (9) whether the
accused was a fugitive from
justice when arrested; and (10) if

Ricardo
de
la
Camara
assailed
in
a
petition
for
certiorari the order of respondent
Judge Manuel Lopez Enage, fixing
the formers bail in the sum of
P1,195,200.00
as
being
repugnant to the constitutional
mandate prohibiting excessive
bail. De la Camara was involved
in the killing of fourteen and the
wounding
of
twelve
other
laborers of the Tirador Logging
Co. Nonetheless, relief sought
setting aside the above order by
reducing the amount of bail to
P40,000.00 cannot be granted,
De la Camara had escaped from
the provincial jail, thus rendering
this case moot and academic. But
the case was decided for the
guidance of lower courts.

Issue:
Whether or not the judge
violated
the
prohibition
on
excessive bail

Held:
Yes.
Where, however, the right to
bail exists, it should not be

rendered nugatory by requiring a


sum that is excessive. If there
were no such prohibition, the
right
to
bail
becomes
meaningless.
Under
the
circumstances, there being only
two offenses charged, the amount
required as bail could not
possibly exceed P50,000.00 for
the information for murder and
P25,000.00
for
the
other
information
for
frustrated
murder. Nor should it be ignored
in this case that the Department
of Justice did recommend the
total sum of P40,000.00 for the
two offenses.

35.
Comendador v. Villa GR
No. 93177, Aug. 2, 1991
Topic: bail in the military

Doctrine:
The argument that denial
from the military of the right to
bail would violate the equal
protection
clause
is
not
acceptable.
This
guaranty
requires equal treatment only of
persons
or
things
similarly
situated and does not apply
where
the
subject
of
the
treatment
is
substantially
different
from
others.
The
accused officers can complain if
they are denied bail and other
members of the military are not.

But they cannot say they have


been
discriminated
against
because they are not allowed the
same right that is extended to
civilians. (Comendador v. Villa GR
No. 93177, Aug. 2, 1991)

Facts:
The petitioners are officers
of the Armed Forces of the
Philippines facing prosecution for
their alleged participation in the
failed coup d' etat that took place
on December 1 to 9, 1989. They
alleged that the right to bail is is
applicable and covers all military
men
facing
court-martial
proceedings.

Issue:
Whether the petitioners have
a right to bail

Held:
No.
The right to bail invoked has
traditionally not been recognized
and is not available in the military,
as an exception to the general rule
embodied in the Bill of Rights. This
much was suggested in Arula,
where the Court observed that
"the right to a speedy trial is given
more emphasis in the military
where the right to bail does not
exist. The right to equal protection
is not violated as members of the

military and civilians are not


similarly situated and thus are not
allowed the same rights.

36.
Government of the United
States of America v. Purganan
289 SCRA 623, Sept. 24, 2002
Topic:
Bail
in
extradition
proceedings

Doctrine:
As suggested by the use of
the word conviction in Art. III,
Section 13 of the Constitution,
the constitutional provision on
bail, as well as Section 4 of Rule
114 of the Rules of Court, applies
only when a person has been
arrested
and
detained
for
violation of Philippine criminal
lawsit does not apply to
extradition proceedings because
extradition courts do not render
judgments
of
conviction
or
acquittal. The constitutional right
to
bail
flows
from
the
presumption of innocence in favor
of every accused who should not
be subjected to the loss of
freedom as thereafter he would
be entitled to acquittal, unless his
guilt
be
proved
beyond
reasonable doubt. It follows that
the constitutional provision on
bail will not apply to a case like
extradition,
where
the
presumption of innocence is not
at issue.

The rule is that bail is not a


matter of right in extradition
cases. However, to best serve the
ends of justice, after a potential
extraditee has been arrested or
placed under the custody of the
law, bail may be applied for and
granted as an exception, only
upon a clear and convincing
showing (1) that, once granted
bail, the applicant will not be a
flight risk or a danger to the
community; and (2) that there
exist special, humanitarian and
compelling
circumstances
including,
as
a
matter
of
reciprocity, those cited by the
highest court in the requesting
state when it grants provisional
liberty
in
extradition
cases
therein.

Since this exception has no


express or specific statutory
basis, and since it is derived
essentially
from
general
principles of justice and fairness,
the applicant bears the burden of
proving the above two-tiered
requirement
with
clarity,
precision
and
emphatic
forcefulness. (Government of the
United States of America v.
Purganan 289 SCRA 623, Sept.
24, 2002)

Facts:
Pursuant to the existing RPUS Extradition Treaty, the United

States
Government,
through
diplomatic channels, sent to the
Philippine Government Petition
for Extradition of Mark B.
Jimenez, also known as Mario
Batacan Crespo, because he was
the subject of an arrest warrant
issued by the United States
District Court for the Southern
District of Florida on April 15,
1999. RTC granted the motion.
Jimenez sought an alternative
prayer: that in case a warrant
should issue, he be allowed to
post bail in the amount of
P100,000,
which
was
also
granted and Jimenez was afforded
provisional liberty.

Issue:
Are prospective extraditees
entitled to the right to bail and
provisional liberty while the
extradition
proceedings
are
pending?

Held:
No.
After a potential extraditee
has been arrested or placed under
the custody of the law, bail may be
applied for and granted as an
exception, only upon a clear and
convincing showing (1) that, once
granted bail, the applicant will not
be a flight risk or a danger to the
community; and (2) that there exist
special,
humanitarian
and
compelling
circumstances

including,
as
a
matter
of
reciprocity, those cited by the
highest court in the requesting
state when it grants provisional
liberty in extradition cases therein.
it is derived essentially from
general principles of justice and
fairness, the applicant bears the
burden of proving the above twotiered requirement with clarity,
precision
and
emphatic
forcefulness. Jimenez, however,
was not able to successfully
discharge such burden.

37.
Government of Hongkong
Special Administrative Region
v. Olalia, Jr. 521 SCRA 470,
April 19, 2007 (reexamined
Purganan ruling)
Topic:
bail
in
extradition
proceedings

Doctrine:
First, we note that the
exercise of the States power to
deprive an individual of his
liberty is not necessarily limited
to
criminal
proceedings. Respondents
in
administrative
proceedings,
such
as
deportation
and
quarantine,4 have
likewise
been detained.

Second, to limit bail to


criminal proceedings would be to
close
our
eyes
to
our
jurisprudential history. Philippine
jurisprudence has not limited the
exercise of the right to bail to
criminal proceedings only. This
Court has admitted to bail
persons who are not involved in
criminal proceedings. In fact, bail
has been allowed in this
jurisdiction to persons in
detention during the pendency
of administrative proceedings,
taking into cognizance the
obligation of the Philippines
under
international
conventions to uphold human
rights.

If bail can be granted in


deportation cases, we see no
justification why it should not also
be allowed in extradition cases.
Likewise, considering that the
Universal
Declaration
of
Human
Rights
applies
to
deportation cases, there is no
reason why it cannot be
invoked in extradition cases.
After all, both are administrative
proceedings where the innocence
or guilt of the person detained is
not in issue. (Government of
Hongkong Special Administrative
Region v. Olalia, Jr. 521 SCRA
470, April 19, 2007)

Facts:

Private
respondent
Juan
Antonio
Muoz
was
charged
before the Hong Kong Court with
three (3) counts of the offense of
"accepting an advantage as agent,"
in violation of Section 9 (1) (a) of
the
Prevention
of
Bribery
Ordinance, Cap. 201 of Hong
Kong. He also faces seven (7)
counts of the offense of conspiracy
to defraud, penalized by the
common law of Hong Kong. On
Aug. 23, 1997 and October 25,
1999, warrants of arrest were
issued against him. If convicted, he
faces a jail term of seven (7) to
fourteen (14) years for each
charge. On Sept. 13, 1999, the DOJ
received from the Hong Kong
Department of Justice a request for
the provisional arrest of private
respondent. RTC of Manila ordered
Muozs arrest. CA declared the
Order of Arrest void. In the same
petition for extradition, the Hong
Kong government filed a petition
for bail, which Judge Bernardo
initially denied, but granted and
fixed at P750,000 upon motion for
reconsideration.

Issue:
Whether the trial court
committed
grave
abuse
of
discretion amounting to lack or
excess of jurisdiction in admitting
private respondent to bail

Held:

No.
The Court cannot ignore the
following trends in international
law: (1) the growing importance
of the individual person in public
international law who, in the 20th
century, has gradually attained
global recognition; (2) the higher
value now being given to human
rights in the international sphere;
(3) the corresponding duty of
countries
to
observe
these
universal
human
rights
in
fulfilling their treaty obligations;
and (4) the duty of this Court to
balance
the
rights
of
the
individual under our fundamental
law, on one hand, and the law on
extradition, on the other.

The Philippines, along with


the other members of the family
of nations, committed to uphold
the fundamental human rights as
well as value the worth and
dignity of every person. the
Philippine authorities are under
obligation to make available to
every person under detention
such remedies which safeguard
their fundamental right to liberty.
These remedies include the right
to be admitted to bail. While this
Court in Purganan limited the
exercise of the right to bail to
criminal proceedings, however, in
light of the various international
treaties giving recognition and
protection to human rights,
particularly the right to life and
liberty, a reexamination of this
Courts ruling in Purganan is in
order.

38.
Bengzon v. Ocampo, et al.
84 Phil. 611 (1949)
Topic:
bail
in
deportation
proceedings
Doctrine:
A
person
arrested
or
detained cannot be released on
bail, unless that right is granted
expressly by law. Sec. 37(9) of the
Philippine Immigration Act of
1940 provides that: Any alien
under arrest in a deportation
proceeding may be released
under bond or under such other

conditions as may be imposed by


the
Commissioner
of
Immigration.
(Bengzon
v.
Ocampo, et al. 84 Phil. 611 (1949)
Facts:
The Bureau of Immigration
ordered
the
deportation
of
Mariano Chan alias Ong Lian Cho
for maintaining a house of
prostitution. Jose Bengzon filed a
petition for habeas corpus with
RTC Manila, presided by Judge
Buenaventura Ocampo. Ocampo
ordered
Chans
provisional
release, provided he files a bond
of P10,000 and report to the
Bureau
of
Immigration
on
Monday,
Wednesday
and
Saturday of every week.
Issue:
Whether
or
not
the
respondent judge acted in excess
of the court's jurisdiction in
granting the petition of the
respondent Mariano Chan for his
temporary release on bail
Held:
Yes.
The judge in a habeas corpus
proceeding must immediately
proceed to hear and decide the
case, unless for good cause
shown the hearing is adjourned,
in which event the court or judge
shall make such order for the
safekeeping
of
the
person
imprisoned or restrained as the
nature of the case requires. The

phrase "for the safekeeping of the


person imprisoned or restrained"
can not be construed to mean
temporary
release
on
bail,
because a person arrested or
detained can not be released on
bail unless that right is granted
expressly by law.

39.
Go
Tian
Chai
v.
Commissioner of Immigration,
L-20645, Sept. 22, 1966
Topic:
bail
in
deportation
proceedings
Doctrine:
The determination as to the
propriety of allowing an alien,
subject to the deportation under
the Immigration Act, to be
released temporarily on bail, as
well as the conditions thereof,
falls
within
the
exclusive
jurisdiction of the Commissioner,
and not in the courts of justice.
The reason for this is that the
courts
do
not
administer
immigration laws. (Go Tian Chai
v. Commissioner of Immigration,
L-20645, Sept. 22, 1966)
Facts:
Petitioner-appellee
is
a
Chinese
national
who
was
admitted as temporary visitor into
the country on April 23, 1947.
Having failed to leave when his
visa
expired, notwithstanding
several extensions granted to

him, he was arrested by the


immigration authorities on March
1, 1950. He filed a petition for
writ of habeas corpus contending
that since it was unlikely that the
Philippine government would be
able to effect his deportation in
accordance with law in the near
future, he should be allowed
temporary liberty on bail. RTC
denied the petition.
Issue:
Whether a Chinese national
who has been duly found to be
subject to deportation may be
ordered released on bail during
the pendency of the negotiations
between
the
Philippine
Government and the Republic of
Nationalist
China
for
their
readmission to China

Held:
No.
The determination as to the
propriety of allowing an alien,
subject to deportation under the
Immigration Act, to be released
temporarily on bail, as well as the
conditions thereof, falls within
exclusive jurisdiction of the
Commissioner, and not in the
courts of justice. The reason for
this is that the courts do not
administer immigration laws.

40.
Ong
Hee
Sang
v.
Commissioner of Immigration,
L-9700 4 SCRA 442 Feb. 28,
1962
Topic:
bail
in
deportation
proceedings

Doctrine:
The right to bail is not a
matter of right but a matter of
discretion on the part of the
Commissioner of Immigration and
Deportation. Thus, Section 37(e)
of the Philippine Immigration Act
of 1940 provides that any alien
under arrest in a deportation
proceeding may be released
under bond or under such other
conditions as may be imposed by
the
Commissioner
of
Immigration.The use of the word
may in said provision indicates
that the grant of bail is merely
permissive and not mandatory on
the part of the Commissioner. The
exercise of the power is wholly
discretionary.
As
deportation
proceedings do not partake of the
nature of a criminal action, the
constitutional guarantee to bail
may not be invoked by aliens in
said proceedings. (Ong Hee Sang
v. Commissioner of Immigration,
L-9700 4 SCRA 442 Feb. 28,
1962)

Facts:

Petitioners were natives of


Amoy, China who were given
permission
to
stay
in
the
Philippines due to the communist
takeover of mainland China.
Despite the period of time given
them within which to leave the
Philippines, and notwithstanding
the availability of surface and air
transportation
to
Hongkong,
petitioners failed to comply with
the condition imposed for their
temporary stay in the Philippines.
The trial court denied petitioners'
petition for habeas corpus, but
allowed their provisional release
on bail pending their actual
deportation

Issue:
Whether or not the trial
court erred in granting bail to
petitioners-appellees who were at
the time under detention by the
immigration authorities

Held:
Yes.
Aliens
in
deportation
proceedings, as a rule, have no
inherent right to bail. Section 37
(9)
(e)
of
the
Philippine
Immigration Act of 1940 (Comm.
Act No. 613, as amended),
provides that:
Any alien under arrest in a
deportation proceeding may be

released under bond or under such


other conditions as may be
imposed by the Commissioner of
Immigration. The use of the word
"may" in said provision indicates
that the grant of bail is merely
permissive and not mandatory or
obligatory on the part of the
Commissioner. The exercise of the
power is wholly discretionary. As
deportation proceedings do not
partake of the nature of a criminal
action,
the
constitutional
guarantee to bail may not be
invoked
by
aliens
in
said
proceedings.

1.
U.S. vs. A. A. Addison and
Pastor Gomez
G.R. No. L-9635, August 26,
1914
Doctrine: When the obligation of
bail is assumed, the sureties
become in law the jailers of their
principal. They have the obligation
to secure the bailees presence
before the court when required.
Facts:
The
case
involved
the
accused Walter Schultz who was
charged with malversation and
was granted bail by the CFI of
Ilocos
Sur.
However,
his
whereabouts can no longer be
ascertained. The court ordered his
bondsmen,
who
were
the
respondents in this case, to fulfill
their obligation of bringing the
accused
before
the
court.
However, the bondsmen failed to
secure his presence before the
court and thus the bail bond was
forfeited. They contended that they
no longer knew where the bailee
had escaped, making it impossible
to perform their obligation.
Issue:
Whether or not the
respondents can appeal the
forfeiture of their bond for failure
to perform their obligation to
present the accused before the
court
Karl Alen G. Yu
Section 2-D
Topic: Bail

Held:
No. The Supreme Court
declared that when the obligation
of bail is assumed, the sureties
become in law the jailers of their
principal. Their custody of him is

the continuance of the original


imprisonment, and though they
cannot actually confine him, they
are subrogated to all the other
rights and means which the
Government possesses to make
their control of him effective. The
responsibility assumed by the bail,
being purely gratuitous, may be
terminated by them at any time,
and, to effect this end, they may
arrest the principal at pleasure
and surrender him into the hands
of the law. However, the bondsmen
allowed the accused to escape and
did not even attempt to deliver the
accused before the court. Hence,
the judgment is rendered against
the sureties.
1. Dosena-Caspe vs. Judge
Bugtas
A.M. No. RTJ-03-1767,
March 28, 2003
Doctrine: A bail hearing is
mandatory especially when its
grant is discretionary to determine
whether the evidence of guilt is
strong.

Whether or not bail hearings


are mandatory in issuing bail
Held:
Yes. The court fined and
warned Judge Bugtas for gross
ignorance
of
the
law.
Jurisprudence is replete with
decisions
on
the
procedural
necessity of a hearing, whether
summary or otherwise, relative to
the grant of bail, especially in
cases
punishable
by
death,
reclusion
perpetua,
or
life
imprisonment,
in
order
to
determine whether the evidence of
guilt is strong.
2. Maguddatu vs. Court of
Appeals
G.R. No. 139599, February
23, 2000
Doctrine: Bail is available only to
those who are within the custody
of the law or are otherwise
deprived of their liberty.

Facts:
This is an administrative
case against Judge Bugtas for
gross ignorance of the law in
granting bail upon an accused for
murder without conducting a bail
hearing based on the grounds that
the prosecution failed to file an
objection and that the granting of
bail is discretionary. However, the
prosecution contended that bail
hearings are mandatory in order to
grant bail.

Facts:
The petitioners in this case
were convicted by the trial court
for homicide with a prior grant to
bail. On appeal, the petitioners
appealed not only the decision
against them but also an extension
for their grant of bail. However,
the bail application was denied by
the Court of Appeals since the
petitioners remained at large
despite the order for their arrest.
The petitioners contended that
they were willing to surrender and
that the CA decision was tainted
with grave abuse of discretion.

Issue:

Issue:

Whether or not bail may be


granted to those who remain at
large
Held:
No. The Supreme Court
ruled that bail is available only to
those who are within the custody
of the law or are otherwise
deprived of their liberty. The
petitioners in this case continued
to remain at large despite the
order of arrest issued against
them. Their continuous evasion is
a violation of the conditions
stipulated in their previous bail.
Also, the court clarified that the
bail bond that the accused
previously posted can only be used
during the 15-day period to appeal
and not during the entire period of
appeal. This is consistent with
Section 2(a) of Rule 114 which
provides that the bail "shall be
effective
upon
approval
and
remain in force at all stages of the
case, unless sooner cancelled, until
the promulgation of the judgment
of the Regional Trial Court,
irrespective of whether the case
was originally filed in or appealed
to it. Hence, bail is denied.
3. Miranda vs. Tuliao
G.R. No. 158763, March 31,
2006
Doctrine: As a general rule in
criminal cases, an accused seeking
an affirmative relief from the court
is deemed to have submitted to its
jurisdiction.
Facts:
The petitioners in this case
were charged with murder in the

RTC of Manila who filed their


motion to complete preliminary
investigation, to reinvestigate, and
to recall or quash the warrants of
arrest against them despite their
non-submission to the jurisdiction
of the court. The RTC ruled in their
favor
prompting
private
respondent to appeal to the CA on
the ground that Judge Anghad of
the RTC committed grave abuse of
discretion when it granted the
motions of the petitioners despite
the fact that they have not
submitted
themselves
to
the
jurisdiction of the court. The CA
granted the respondents petition
warranting the petitioners to
appeal to the SC.
Issue:
Whether or not petitioners
may seek affirmative relief from
the courts despite their nonsubmission to the jurisdiction of
the courts
Held:
The Supreme Court declared
that: In criminal cases, jurisdiction
over the person of the accused is
deemed waived by the accused
when he files any pleading seeking
an affirmative relief, except in
cases when he invokes the special
jurisdiction of the court by
impugning such jurisdiction over
his person. However, if a person
invoking the special jurisdiction of
the court applies for bail, he must
first submit himself to the custody
of the law.
The
exceptions

following
are
to the general

the
rule

requiring jurisdiction over the


person of the accused:
(1) in civil cases, motions to
dismiss on the ground of lack of
jurisdiction over the person of the
defendant, whether or not other
grounds for dismissal are included;
(2) in criminal cases, motions
to quash a complaint on the
ground of lack of jurisdiction over
the person of the accused; and
(3) motions to quash a
warrant of arrest.
The
first
two
are
consequences of the fact that
failure
to
file
them
would
constitute a waiver of the defense
of lack of jurisdiction over the
person. The third is a consequence
of the fact that it is the very
legality of the court process
forcing the submission of the
person of the accused that is the
very issue in a motion to quash a
warrant of arrest.

in the illegal detention case were


not yet apprehended and were
therefore
not
within
its
jurisdiction. However, the accused
subsequently
surrendered
and
Judge Reyes based from the facts
presented in the initial bail hearing
granted
them
bail.
The
complainants contend that Judge
Reyes
rendered
an
unjust
judgment on the bail petition
without conducting the mandatory
bail hearing. Judge Reyes averred
that a bail hearing was previously
conducted and based from its
merits, it warranted the grant of
bail.
Issue:
Whether or not the prior bail
hearing of a dismissed application
due to lack of jurisdiction is
sufficient to justify a subsequent
grant of bail upon the surrender of
the accused
Held:

4. Guillermo vs. Judge Reyes


A.M.
No.
RTJ-93-1088,
January 18, 1995
Doctrine: A bail hearing is
mandatory to allow the parties to
present evidence to support their
claim.
Facts:
The
case
involved
a
complaint against a judge who
granted a bail petition without
conducting the necessary bail
hearing.
Judge
Reyes,
who
received an application for bail,
initially dismissed the application
during the mandatory bail hearing,
based on the fact that the accused

No. The Supreme Court


declared that a bail hearing is
required in every application of
bail. As a matter of course, what
respondent judge should have
done
upon
the
voluntary
appearance of the accused was to
require another motion for bail
and set the same for hearing, with
the prosecution duly notified
thereof. Respondent judge should
not have relied only on the
evidence presented by the parties
when the application for bail was
heard as his bases for granting
bail. He should have accorded the
parties another opportunity to
introduce
whatever
additional
evidence they may have been

minded to adduced therein as an


aid determining the proper amount
of the bail. This would also have
obviated the impression that he
had abbreviated the normal course
of bail proceedings and resorted to
procedural shortcuts to favor the
accused.
5. Pico vs. Judge Combong, Jr.
A.M. No. RTJ-91-764,
November 6, 1992
Doctrine: In order for the court to
grant bail, the applicant must be
within the custody of the law or is
otherwise deprived of liberty.
Facts:
The complainant in this case
charged respondent judge for
grave misconduct and grave abuse
of discretion for granting bail upon
an accused to a murder case
without
him
being
in
the
jurisdiction of the court.
The
complainant testified that on July
17, 1991, the respondent judge
issued a warrant of arrest against
the accused Eddie Villegas for
killing a minister of the Iglesia
Filipina Independiente. However,
the warrant was not served and it
was returned to the court.
Subsequently, complainant learned
that Villegas was granted bail
despite the fact that he was not
arrested nor did he surrender.
Also, the respondent judge did not
conduct a bail hearing and did not
give the prosecution a chance to
challenge the application for
provisional liberty. Respondent
Judge
contended
that
he
conducted a hearing but admitted
that he committed a mistake in not

allowing the prosecution to object


and that it was not motivated by
any
illegal
or
immoral
consideration.
Issues:
Whether or not the grant of
bail was valid despite the lack of
jurisdiction over the accused
Whether
or
not
the
prosecution must be given the
opportunity to object the bail
application
Held:
The Supreme Court held that
a person applying for admission to
bail must be in the custody of the
law or otherwise deprived of his
liberty. A person who has not
submitted
himself
to
the
jurisdiction of the court has no
right to invoke the processes of
that court. Respondent Judge
should have diligently ascertained
the whereabouts of the applicant
and that he indeed had jurisdiction
over the body of the accused
before considering the application
for bail.
It is well-settled that an
application for bail from a person
charged with a capital offense
must be set for hearing at which
both the prosecution and the
defense
must
be
given
a
reasonable opportunity to prove
that evidence of guilt of the
applicant is strong, or that such
evidence of guilt was not strong.
6. Paderanga vs. Court of
Appeals
G.R. No. 115407, August 28,
1995

Doctrine: It is sufficient for an


accused
who
cannot
submit
himself to the court based on valid
reasons to expressly recognize its
jurisdiction over him in order to be
granted bail.

Whether or not the court


acquired jurisdiction over the
person of the accused
Whether or not the assigned
prosecutor was the only person
allowed to make an objection
Held:

Facts:
The accused in this case,
applied for bail with the RTC of
Cagayan De Oro though he was
not physically within the custody
of neither the court nor the proper
authorities. It was his contention
that he was suffering from an
acute
illness
of
Acute
Costochondritis and was confined
at
Cagayan
Capitol
College
General Hospital. He sent his
counsel to apply for bail along with
the documents that expressly
contained
that
the
accused
recognizes the jurisdiction of the
court, that he is submitting himself
to its jurisdiction and that he is not
able to physically submit since he
was
confined.
The
assistant
prosecutor was present and did
not make any objection to the
application. However, when the
bail was granted, the prosecutor
who was allegedly in-charge of the
case appealed to the CA to annul
the decision due to the fact that
the court did not have jurisdiction
over the person and that it was he
who had authority to object to the
application and not the assistant
prosecutor. The CA reversed the
decision
in
favor
of
the
prosecution.
Issue:

The Supreme Court said that


the general rule in bail application
is that a person must submit
himself to the jurisdiction of the
court to be granted bail. In the
present
case,
the
accused
substantially complied, through
counsel, and was able to recognize
the jurisdiction of the court over
him and that he submitted himself
to its jurisdiction though not
physically
by
presenting
documents to support his claim.
Also, he subsequently surrendered
to the court after he was
discharged from the hospital when
he paid for the bond. Hence, the
RTC had valid jurisdiction over
him.
Likewise,
the
assistant
prosecutors presence and nonobjection is enough to comply with
due process and that it was not
necessary for the chief prosecutor
himself to be present during the
hearing. Therefore, the decision of
the CA was reversed and the
decision
to
grant
bail
was
reinstated.
7. People vs. Gako, Jr.
G.R. No. 135045, December
15, 2000
Doctrine: A bail hearing is
mandatory when it is discretionary
to determine whether the evidence
of guilt is strong.

Facts:
In this case, the decision of
Judge Gako, Jr. regarding an
application for bail by an accused
in a murder case was questioned
by the prosecution since it was
granted with lack of jurisdiction
over the person of the accused,
without the necessary hearing, and
that the decision did not contain a
summary
of
the
evidence
presented
by
the
parties.
Respondent judge contended that
it was only a re-trial and had
authority to grant the bail.
Issue:
Whether or not the bail was
properly ruled upon by the judge
with the proper procedure and
requirements
Held:
No. The Supreme Court only
ruled in favor of the judge
regarding the jurisdictional issue
since, the case involved was only a
re-trial
and
jurisdiction
was
already conferred upon the court
during the previous one and it
continues until the final disposition
of the case. However, a bail
hearing is necessary in cases
where
the
bail
petition
is
discretionary.
It
has
been
consistently held that when bail is
discretionary, a hearing, whether
summary or otherwise in the
discretion of the court, should first
be conducted to determine the
existence of strong evidence or
lack of it, against the accused to
enable the judge to make an
intelligent assessment of the
evidence presented by the parties.
As to the requirement of a

summary of the evidence, the


court said any irregularity is fatal
to the grant of bail. The court laid
down the duties of the judge in
case an application for bail is
required.
(1) Notify the prosecutor of the
hearing for bail or require him to
submit his recommendation;
(2) Conduct a hearing of the
application for bail regardless of
whether or not the prosecution
refuses to present evidence to
show that the guilt of the accused
is strong for the purpose of
enabling the court to exercise its
discretion;
(3) Decide whether the evidence of
guilt of the accused is strong
based on the summary of evidence
of the prosecution;
(4) If the guilt of the accused is not
strong, discharge the accused
upon the approval of the bail bond.
Otherwise, petition should be
denied.
8. Defensor-Santiago
vs.
Vasquez
G.R. No. 99289-90, January
27, 1993
Doctrine: Filing an affirmative
relief from the Court places the
accused within the jurisdiction of
the Court.
Facts:
In this case, petitioner Dr.
Miriam
Defensor-Santiago
questioned the jurisdiction of the
RTC
of
Manila
and
the

Sandiganbayan to try her case for


the alleged violation of the AntiGraft and Corrupt Practices Act.
However, prior to the objection,
petitioner, through counsel filed an
Ex-parte Motion for Acceptance of
Cash Bail Bond based on her
medical condition due to an
accident and that she cannot be
present since she was ill, which
motion was granted. But the
petitioner failed to post the
necessary bail bond and did not
benefit from it. Subsequently
during the trial, the petitioner
questioned the courts jurisdiction
since she was never arrested nor
had she surrendered.
Issue:
Whether or not jurisdiction
over the person of the accused was
acquired by the court despite
failure to utilize the grant for bail
Held:
Yes. The Supreme Court find
and so hold that petitioner is
deemed
to
have
voluntarily
submitted
herself
to
the
jurisdiction of respondent court
upon the filing of her aforequoted
"Urgent
Ex-parte
Motion
for
Acceptance of Cash Bail Bond for
and in behalf of Dr. Miriam
Defensor-Santiago" wherein she
expressly sought leave "that she be
considered
as
having
placed
herself under the jurisdiction of
the Sandiganbayan for purposes of
the required trial and other
proceedings. However, despite the
fact that she did not avail of this
grant and but by said motion "she
be considered as having placed
herself under the custody" of said

court. Petitioner cannot now be


heard to claim otherwise for, by
her own representations, she is
effectively estopped from asserting
the contrary after she had earlier
recognized the jurisdiction of the
court and caused it to exercise that
jurisdiction over the aforestated
pleadings she filed therein.
9. Serapio vs. Sandiganbayan
G.R. No. 148468, January 28,
2003
Doctrine: Arraignment is not
necessary before the court hears
the bail petition
Facts:
This case involved one of the
accused to the high-profile plunder
cases filed against the former
President Joseph Estrada and
many others. Petitioner Atty.
Edward Serapio filed two petitions
for certiorari questioning the
denial to grant him bail, among
others. As to the bail application,
petitioner contends that he was
continuously denied the right to
bail when the prosecution filed
multiple motions for postponement
of bail hearing since they wanted
the
accused
to
initially
be
arraigned before the bail hearing
since according to them this will
guarantee his appearance before
the court and in case the bail is
denied and he decides to escape,
the court may try him in absentia.
However, petitioner contends that
arraignment is not a requisite to
file an application for bail.
Issue:

Whether or not the accused


should be arraigned before the
courts may hear the bail petition
Held:
No. The Supreme Court
clarified that arraignment is not
required for the courts to hear the
bail petition. The Supreme Court
stressed that a person deprived of
his liberty by virtue of his arrest or
voluntary surrender may apply for
bail as soon as he is deprived of his
liberty, even before a complaint or
information is filed against him.
10.
Ruiz vs. Judge Beldia
A.M.
No.
RTJ-02-1731,
February 16, 2005
Doctrine: As a general rule, a bail
application may be filed only with
the court where the case is
pending
except
when
the
information is yet to be filed, the
application may be filed with any
court within the city, municipality
or province where the accused is
held. Also, the prosecutor must be
given notice to a bail hearing.
Facts:
The complaint against Judge
Beldia stemmed from the grant of
bail issued to an accused who
allegedly violated the Anti-Fencing
Law. The accused in that case was
arrested by authorities and was
detained in Camp Crame. Upon
inquest, the accused waived the
provisions of Article 125 of the
Revised Penal Code in relation to
rule 112 of the rules of criminal
procedure. The inquest prosecutor
then set the hearing of the
Preliminary
Investigation.

However, an order of release was


issued by the respondent judge
before
the
preliminary
investigation was conducted. The
order was based on an application
for bail filed by the accused. The
accused contended that, in his
case, bail is a matter of right and
where no case has yet been filed in
any court for the alleged crime, an
application for bail may be filed
with any court, as long as the
accused is already deprived of his
liberty.
Conversely,
the
complainant contended that the
prosecutor was not given notice to
the bail application and the judge
should not have granted the
application since it was not filed
with the proper court with
territorial jurisdiction.
Issue:
Whether or not a bail
application may be filed only
within the court with jurisdiction
over the case
Whether or not, in cases
where bail is a matter of right,
notice to a bail hearing to the
prosecution is no longer necessary
Held:
No.
On the first issue, the court
agreed
with
petitioner.
An
application for bail, where no case
is yet to be filed with any court,
the same may be filed with any
court within the province, city or
municipality where he is held.
As to the second issue, the
Supreme Court said that, while it
is true that bail is a matter of
right, the prosecutor must be

given notice regarding the bail


hearing
or
at
least
his
recommendation is sought in order
to properly fix the amount of bail
to secure the accused presence
before the court. However, Judge
Beldia failed to inform the
prosecutor of the bail hearing.
11.
Barbero vs. Dumlao
A.M. No. MTJ-07-1682, June
19, 2008
Doctrine:
Where
a
formal
complaint of information is already
filed with the court, an application
for bail may only be filed with the
same court.
Facts:
The
administrative
case
against Judge Dumlao stemmed
from
a
complaint
filed
by
petitioner Barbero for allegedly
granting the bail erroneously to a
certain Medina, who petitioner
charged with estafa. The petitioner
contends that he filed a complaint
for estafa against Medina with the
RTC of Santiago City, Isabela.
However,
Medina
filed
an
application for bail with the RTC of
San Mateo, Isabela, where Judge
Dumlao presided. Judge Dumlao
ordered the release of Medina
after
due
hearing.
Barbero
contended that the grant was
improper since, the application
should have been filed with the
RTC of Santiago City. Hence,
Barbero complained Judge Dumlao
for gross ignorance of the law.
Issue:
Whether
or
not
applications may be filed

bail
only

within the court where


complaint or information
already been filed

the
has

Held:
Yes. The Supreme Court
dismissed
Judge
Dumlao
for
persisting to violate the rules of
procedure,
not
only
for
erroneously granting bail but also
for other causes of disobedience.
Regarding the issue on bail, the
Court
clarified
that
bail
applications, where a formal
complaint or information has
already been filed with the proper
court, the application may only be
filed with court where the case is
pending.
12.
Lavides vs. Court of
Appeals
G.R. No. 129670, February 1,
2000
Doctrine:
Arraignment
as
a
condition to the grant of bail is
void.
Facts:
The petitioner in this case
was charged with several counts of
child abuse. He applied for bail
with the RTC of Quezon City which
resolved the application subject to
four conditions conditions which
included the fixed amount of
P80,000.00 for every charge, that
the accused is not entitled to a
waiver of his appearance before
the court, and failure to appear
automatically forfeits his bail
bonds and that the accused must
first be arraigned before the bail is
granted. The petitioner moved to
quash the complaints against him

and appealed to the CA to review


the conditions of his bail. The CA
invalidated
the
conditions
regarding
the
waiver
and
automatic forfeiture of the bail
bonds but declined to decide on
the condition of prior arraignment
since it was already moot and
academic according to the court.
The petitioner also sought to annul
the arraignment conducted since it
was subject to an invalid condition.

arraignment did not emanate from


the invalid condition that "approval
of the bail bonds shall be made
only after the arraignment." Even
without such a condition, the
arraignment of petitioner could not
be omitted.

Issue:

Doctrine: The payment of bail


bonds must be transmitted to the
nearest internal revenue collector
or municipal, city or provincial
treasurer at the nearest possible
time.

Whether or not arraignment


as a pre-condition to the grant of
bail is valid and in case of its
invalidity also invalidates the
conducted arraignment
Held:
No. Arraignment as a
condition to the grant of bail is
void. To condition the grant of bail
to an accused on his arraignment
would be to place him in a position
where he has to choose between
(1) filing a motion to quash and
thus delay his release on bail
because until his motion to quash
can be resolved, his arraignment
cannot be held, and (2) foregoing
the filing of a motion to quash so
that he can be arraigned at once
and thereafter be released on bail.
These
scenarios
certainly
undermine
the
accused's
constitutional right not to be put
on
trial
except
upon
valid
complaint or information sufficient
to charge him with a crime and his
right to bail.
As to the validity of the
already conducted arraignment,
the arraignment stands since the

13.
Agulan
vs.
Judge
Fernandez
A.M. No. MTJ-01-1354, April
4, 2001

Facts:
The
petitioner
filed
a
complaint against Judge Fernandez
for allegedly misappropriating the
bail bonds paid to him by the
petitioner in a case involving
illegal possession of firearms,
which the respondent judge was
handling. Petitioner paid Judge
Fernandez the amount of P72,000
as bail bond which respondent
judge failed to remit to the
municipal,
city
or
provincial
treasurer or the nearest internal
revenue collector as required by
law since according to the judge,
the payment was made at night
time and that the treasurer was on
leave of absence. However, despite
the long amount of time that
lapsed, the judge kept the money
in his deposit box in the court.
After the complaint was filed
against him, the judge reimbursed
the amount of the bail bonds to the

petitioner. But the administrative


case against the judge proceeded.
Issue:
Whether or not the bail
bonds may be paid with the
presiding judge without making
the necessary remittance to the
nearest internal revenue collector
or the municipal, city or provincial
treasurer
Held:
No.
The
judge
is
administratively
liable
for
misappropriating the funds and
not making any steps to pursue the
procedure prescribed by law. The
Supreme Court stressed that the
deposit of bail bonds shall only be
made with the nearest collector of
internal revenue or provincial, city
or municipal treasurer pursuant to
Section 14 of Rule 114 of the
Revised
Rules
of
Criminal
Procedure.
14.
Espiritu
vs.
Judge
Jovellanos
A.M.
No.
MTJ-97-1139,
October 16, 1997
Doctrine: An application for bail is
tantamount to surrender to the
jurisdiction of the court.
Facts:
The petitioner Espiritu filed
a
complaint
against
Judge
Jovellanos for ignorance of the law,
gross partiality, and grave abuse of
authority. It was specified by
petitioner that in the application
for bail filed by the accused in a
frustrated murder case, wherein
petitioner was the offended party,

the respondent judge granted the


bail despite the fact that the
applicant was not within the
custody of the court, that no notice
was served to the prosecution and
that the reduction the amount of
bail was a form of partiality to the
accused since it was granted after
the
request
by
the
father.
Respondent judge contended that
the accused validly submitted to
the jurisdiction of the accused
when he personally filed the
application for bail. The judge
admitted that the reduction was
made after the request of the
father of the accused.
Issue:
Whether or not the court had
jurisdiction over the person of the
accused despite that the accused
was not arrested nor did he
surrender
Whether or not the amount
of bail may be decreased based on
the sole discretion of the judge
Whether or not notice to the
prosecution is necessary
Held:
As to the first issue, the
Supreme Court ruled that the
court had jurisdiction since the
accused
personally
filed
the
application for bail which was
tantamount to surrender to the
jurisdiction of the court. As to the
second issue, the amount for bail
shall not only be based on the sole
discretion of the judge. The law
provides a Bail Bond Guide in
determining the amount of bail
and disregarding it is an injunction
to the Code of Judicial Conduct to
maintain professional competence.

As to the final issue, notice of the


application
for
bail
to
the
prosecution is required and the
failure to observe the above
requirement constitutes ignorance
or incompetence which cannot be
excused by any protestation of
good faith.
15.
Villaseor vs. Judge
Abano
G.R. No. L-23599, September
29, 1967
Doctrine: The amount of bail must
be
fixed
according
to
the
conditions provided by law.
Facts:
The case involves a petition
to reduce the amount for bail
issued by the Judge Abano in a
crime involving the accused for
direct assault upon an agent of a
person in authority with murder.
Previously, the crime charged
against the accused was murder
but
was
amended
by
the
prosecutor to the current crime.
The accused was also previously
granted bail in the amount of
P40,000.00. However, when the
information was amended, the
court increased the bail bond to
P60,000.00 based on the new
circumstances surrounding the
new crime charged. The accused
could not avail of the said amount
and thus prayed for a writ of
preliminary
injunction
and
certiorari on the ground that the
amount of bail is excessive.
Issue:
Whether
or
not
the
determination of the amount of
bail is discretionary upon the court

subject
to
guidelines

limitations

and

Held:
As to the first issue, the
court ruled that along with the
court's power to grant bail in
bailable cases is its discretion to
fix the amount thereof, and, as
stated, to increase or reduce the
same. The question of whether bail
is excessive "lays with the court to
determine." The definition of bail
in Section 1, Rule 114, Rules of
Court, gives this purpose "the
security required and given for the
release of a person who is in the
custody of the law, that he will
appear before any court in which
his appearance may be required as
stipulated in the bail bond or
recognizance." Experience
has
brought forth certain guidelines in
bail
fixing,
which
may
be
summarized as follows: (1) ability
of the accused to give bail; (2)
nature of the offense; (3) Penalty
for the offense charged; (4)
character and reputation of the
accused; (5) health of the accused;
(6) character and strength of the
evidence; (7) probability of the
accused appearing in trial; (8)
forfeiture of other bonds; (9)
whether the accused was a fugitive
from justice when arrested; and
(10) if the accused is under bond
for appearance at trial in other
cases. The judge validly decided
upon the amount of bail based on
the circumstances surrounding the
crime charged.
16.
San Miguel vs. Judge
Maceda

A.M. No. RTJ-03-1749, April


4, 2007
Doctrine: The high risk of flight
does not prohibit the court from
granting bail as a matter of right.
The remedy is to increase the
amount required to post bail as a
guarantee.
Facts:
The administrative complaint
against Judge Maceda stemmed
from an illegal sale of drugs case
involving the complainant San
Miguel who was granted bail by
Judge Alumbres for the amount of
P60,000. But the accused jumped
bail. Subsequently, Judge Alumbres
fixed a new bail bond in the
amount of P120,000. However, the
prosecution filed a motion to order
the cancellation for bail with the
respondent judge because there is
a high probability that the accused
will flee. The schedule of the
hearing was set on September 19,
2001. On September 17, 2001, the
accused filed an opposition to the
motion but on the same day, the
motion to cancel the bail was
granted by the respondent judge in
violation of his right to due
process.
Issue:
Whether or not the bail may
be cancelled based on the high
probability of flight on the part of
the accused
Held:
No. Where bail is a matter of
right and prior absconding and
forfeiture is not excepted from
such right, bail must be allowed

irrespective of such circumstance.


The existence of a high degree of
probability that the defendant will
abscond confers upon the court no
greater discretion than to increase
the bond to such an amount as
would reasonably tend to assure
the presence of the defendant
when it is wanted, such amount to
be subject, of course, to the other
provision that excessive bail shall
not be required.
17.
People vs. Plaza
G.R. No. 176933, October 2,
2009
Doctrine: Bail may be granted
where the offense charged was
previously a capital offense but
later turned out to be bailable.
Facts:
The accused in this case was
charged with murder. However the
RTC judge determined in the
hearing for the demurrer to
evidence, that the evidence of the
prosecution is only sufficient to
convict the accused with homicide
since the qualifying circumstance
of treachery was not supported by
sufficient evidence. Upon this
knowledge, the accused filed a
motion to fix bail based on the
crime of homicide and not of
murder. The prosecution filed an
opposition to the motion to fix bail
since the crime charged was
murder and it is a non-bailable
offense. Also, they contended that
it is the public prosecutor who has
exclusive jurisdiction to determine
what crime the accused shall be
charged with and that the accused
should have filed a motion or

application for bail and not just a


motion to fix the amount. The RTC
judge granted the bail and was
affirmed by the CA.
Issue:
Whether or not the grant of
bail was proper despite the fact
that it was not based on the crime
charged and that it was not
granted
through
the
proper
application
Held:
Yes. The Supreme Court
affirmed the grant of bail since the
crime settled upon during the
resolution of the motion to
demurrer of evidence was that the
evidence could only support a
conviction of homicide which is
bailable and not murder. The
prosecution cannot insist that the
crime is still murder. Also, the
application for bail and a bail
hearing is no longer necessary
since the evidence was already
passed upon in the actual trial
during the hearing for the
demurrer to evidence. What was
only left to determine was the
amount of bail to be fixed.
18.
Valerio vs. Court of
Appeals
G.R.
Nos.
164311-12,
October 10, 2007
Doctrine: Bail is not a matter of
right when the offense charged is a
capital offense.
Facts:
This case stemmed from an
information
for
murder
filed
against certain respondents and

for parricide against Milagros


Valerio, the wife of the deceased.
The petitioner in this case was the
sister of the deceased. The
accused,
Milagros,
filed
an
application for bail claiming that
the evidence against her was not
strong. The prosecution on the
other hand moved to discharge
one of the other accused to
become a state witness. However,
the application for bail was
granted by the RTC but the motion
to convert as state witness was
denied. The decision was appealed
to the CA but the decision was
affirmed.
Issue:
Whether or not the bail
granted to Milagros for the crime
of parricide is proper
Held:
No. The Supreme Court
stressed that bail is not a matter of
right in cases where the person is
charged with a capital offense or
an offense punishable by reclusion
perpetua or life imprisonment. In
this case, the trial court had
disregarded the glaring fact that
the killer himself has confessed to
the crime and has implicated
Milagros as the mastermind. When
taken in conjunction with the other
evidence on record, these facts
show very strongly that Milagros
may have participated as principal
by inducement in the murder of
Jun Valerio. It was thus a grave
error or a grave abuse of
discretion committed by the trial
court to grant her application for
bail. The appellate court clearly
committed a reversible error in

affirming the trial courts decision


granting bail to Milagros Valerio.
19.
People vs. Gomez
G.R.
No.
131946-47,
February 8, 2000
Doctrine: Failure to file the proper
remedy on appeal may prevent the
accused to apply for bail.
Facts:
The accused Rogelio Gomez
was charged and convicted by the
Regional Trial Court of Paraaque
for illegal recruitment in large
scale
resulting
in
economic
sabotage.
The
prosecution
contends that during the bail
hearings although he was given
the chance to cross-examine the
prosecution witnesses, he was
never given the opportunity to
submit
rebuttal
evidence
to
disprove that the evidence of his
guilt was strong. However, the
defense failed to file the proper
remedy of certiorari with the CA.
Issue:
Whether or not the failure to
make the necessary remedy on
appeal removes from the accused
the right to bail
Held:
Yes. The Supreme Court
ruled, in such cases, where the
prosecution was not given the
chance to present evidence to
prove that the guilt of the accused
was strong, we held that the
proper remedy was for him to file a
petition for certiorari under Rule
65. This same principle must apply
to cases where the defense was

not accorded a chance to present


any rebuttal evidence. When the
trial court denied his application
for bail accused-appellant should
have filed a petition for certiorari
before the appellate court. Hence,
it is also too late for him to
question the trial court's decision
of denying his application for bail.
Besides, the conviction of accusedappellant undoubtedly proves that
the evidence of guilt against him
was strong.
20.
Zuo vs. Judge Cabebe
A.M. OCA No. 03-1800-RTJ,
November 26, 2004
Doctrine: A bail hearing
procedural necessity.

is

Facts:
The respondent in this case
was the RTC judge of Batac, Ilocos
Norte who handled an illegal
possession and prohibited sale of
dangerous drugs case wherein the
petitioner
was
the
Chief
Prosecutor. Petitioner alleged that
the
respondent
judge,
motu
proprio and without hearing,
granted bail upon the accused in
the drugs case. However, the judge
contends that he granted bail
because of the continuous delay
caused by the prosecution and
invoking
the
right
of
the
defendants to a speedy trial.
Issue:
Whether or not the Trial
Judge may grant bail motu proprio
based on the right of the
defendants to a speedy trial
without conducting a bail hearing

Held:
No. The Supreme Court said
that jurisprudence is replete with
decisions
on
the
procedural
necessity of a hearing, whether
summary or otherwise, relative to
the grant of bail, especially in
cases
involving
offenses
punishable by death, reclusion
perpetua, or life imprisonment,
where bail is a matter of
discretion. Respondent judge did
not conduct a hearing before he
granted bail to the accused, thus
depriving the prosecution of an
opportunity to interpose objections
to the grant of bail. Irrespective of
his opinion on the strength or
weakness of evidence to prove the
guilt of the accused, he should
have conducted a hearing and
thereafter made a summary of the
evidence of the prosecution. The
importance of a bail hearing and a
summary of evidence cannot be
downplayed, these are considered
aspects of procedural due process
for both the prosecution and the
defense; its absence will invalidate
the grant or denial of bail.
21.
People vs. Nitcha
G.R. No. 113517, January 19,
1995
Doctrine: Where the accused was
convicted by the RTC based on
proof beyond reasonable doubt,
bail must not be granted during
the pendency of the appeal.
Facts:
The respondent in this case
was convicted by the trial court for
murder, but the decision was
appealed. The respondent wanted

to carry over his bail bond during


the pendency of the appeal. He
contends that bail is discretionary
upon the court in case a conviction
was secured in the trial court and
that the previous bail still applies
and his incarceration is not
warranted.
Issue:
Whether or not bail is
discretionary when the accused is
convicted in the trial court based
on proof beyond reasonable doubt
Whether or not the existence
of a previous bail bond does not
warrant the accused incarceration
pending appeal
Held:
No. The clear implication is
that, if an accused who is charged
with a crime punishable by
reclusion perpetua is convicted by
the trial court and sentenced to
suffer such a penalty, bail is
neither a matter of right on the
part of the accused nor of
discretion on the part of the court.
In such a situation, the court
would not have only determined
that the evidence of guilt is strong,
which would have been sufficient
to
deny
bail
even
before
conviction. It would have likewise
ruled that the accused's guilt has
been proven beyond reasonable
doubt. Bail must not then be
granted to the accused during the
pendency of his appeal from the
judgment
of
conviction.
The
subsistence of a bail bond is also
no legal obstacle to accuseds
immediate
incarceration
after
promulgation
of
a
decision

involving a felony punishable by


reclusion perpetua.
22.
Jose Antonio Leviste
vs. Court of Appeals, et al.
G.R. No. 189122, March 17,
2010
Doctrine: Pending appeal, the
appellate court must determine
whether it will exercise sound
discretion or stringent discretion
in granting bail.
Facts:
In this case, the petitioner
was convicted by the Regional
Trial Court for homicide for killing
Rafael de las Alas. Petitioner
appealed his conviction with the
Court of Appeals and applied for
an admission to bail pending
appeal, citing his advanced age,
medical condition and with no risk
or possibility of flight. However,
the CA denied his application for
bail stating that the discretion to
grant bail during appeal should be
exercised with grave caution and
only for strong reasons. The CA
also explained that petitioner
failed to show that he indeed
suffers from an ailment of such
gravity that his confinement will
impair his health. Likewise, the CA
conducted
a
preliminary
investigation of his conviction and
said that there is no sufficient
reason substantial enough to
overturn the evidence of his guilt.
The petitioner contended that the
CA decision was made with grave
abuse of discretion and that absent
any
of
the
circumstances
mentioned in Rule 114, bail should
be granted to him.

Issue:
Whether
or
not
those
convicted in the RTC, absent any
of the conditions stated in Rule
114, should be granted bail
pending appeal
Held:
No. The Supreme Court
declared that pending appeal of a
conviction by the Regional Trial
Court of an offense not punishable
by death, reclusion perpetua, or
life imprisonment, admission to
bail is expressly declared to be
discretionary. Also, any application
for bail pending appeal should be
viewed from the perspective of two
stages: (1) the determination of
discretion
stage,
where
the
appellate court must determine
whether any of the circumstances
in the third paragraph of Section
5, Rule 114 is present; this will
establish whether or not the
appellate court will exercise sound
discretion or stringent discretion
in resolving the application for bail
pending appeal and (2) the
exercise of discretion stage where,
assuming the appellants case falls
within the first scenario allowing
the exercise of sound discretion,
the appellate court may consider
all relevant circumstances, other
than those mentioned in the third
paragraph of Section 5, Rule 114,
including the demands of equity
and justice. On the basis thereof,
the CA may either allow or
disallow bail.
On the other hand, if the
appellants case falls within the
second scenario, the appellate

courts
stringent
discretion
requires that the exercise be
primarily
focused
on
the
determination of the proof of the
presence
of
any
of
the
circumstances that are prejudicial
to the allowance of bail. This is
because the existence of any of
those circumstances is by itself
sufficient to deny or revoke bail.
Nonetheless, a finding that none of
the said circumstances is present
will not automatically result in the
grant of bail. Such finding will
simply authorize the court to use
the less stringent sound discretion
approach.
Hence,
petition
is
dismissed.
23.
Chua vs. Court of
Appeals and Chiok
G.R. No. 140842, April 12,
2007
Doctrine: The proper remedy to
question the grant or denial of bail
is through a motion to review the
order not through a separate
action.
Facts:
The respondent Chiok was
convicted by the Regional Trial
Court for the crime of estafa for
defrauding
petitioner
Chua,
introducing
herself
as
a
stockbroker, and collected 9million
from Chua to supposedly invest in
the stock market. Upon conviction
by the trial court, Chua also
prayed for the cancellation of bail
granted to Chiok. However, Chiok
was nowhere to be found during
the promulgation of the judgment
and through counsel, he appealed
the decision with the Court of

Appeals and also filed a separate


petition for a writ of preliminary
injunction
assailing
the
trial
courts omnibus order cancelling
his bail. The appellate court
granted the writ and ordered the
cancellation of the warrant against
Chiok. Chua appealed the decision
to the Supreme Court saying the
appellate court decided with grave
abuse of discretion to grant bail
upon the accused despite his
disappearance.
Issue:
Whether or not a separate
action is the proper remedy to
challenge the omnibus order of
cancellation of bail arising from
the conviction by the trial court
Whether or not the grant of
bail pending appeal is proper
despite the disappearance of Chiok
Held:
No. The Supreme Court
declared that the proper remedy to
question the denial or grant of bail
by the trial courts is not through a
separate action but through a
motion to review the order with
the appellate courts. Such motion
is an incident in his appeal. The
filing of a separate petition via a
special civil action or special
proceeding
questioning
such
adverse order before the appellate
court
is
proscribed.
Such
independent special civil action
obviously contravenes the rule
against multiplicity of suits and
constitutes forum shopping.
As to the second issue,
respondent had no right to be
freed on bail pending his appeal

from the trial courts judgment.


His conviction carries a penalty of
imprisonment exceeding 6 years
which justifies the cancellation of
his bail pursuant to the third
paragraph of Section 5 (b), (d) and
(e) of Rule 114. Also, His
inexcusable non-appearance not
only violated the condition of his
bail that he "shall appear" before
the court "whenever required" by
the latter or the Rules, but also
showed the probability that he
might flee or commit another
crime while released on bail.
24.
Narciso
vs.
Santa
Romana-Cruz
G.R. No. 134504, March 17,
2000
Doctrine: As a general rule, only
the Solicitor General or public
prosecutor may file an opposition
to the grant of bail but as an
exception, the ends of substantial
justice may allow the intervention
of the offended party.
Facts:
After
the
preliminary
investigation on the death of
Corazon Sta. Romana-Narciso, the
petitioner Joselito Narciso was
charged with the RTC of Quezon
city for parricide. Joselito filed a
motion for re-investigation and to
life the warrant of arrest against
him. The motion was granted but
upon
reinvestigation,
the
prosecutor found no reason to
reverse the findings. Subsequently,
the accused filed an application for
bail which was granted on the
same
day
since
the
public
prosecutor
did
not
file
an

opposition to it. However, private


respondent
Flor
Marie
Sta.
Romana-Cruz,
sister
of
the
deceased, filed an urgent motion
to expunge the order granting bail
for failure to notify the private
prosecutor. On the other hand,
petitioner contended that the
sister of the deceased, through the
private prosecutor, had no legal
standing in filing an opposition to
grant bail.
Issues:
Whether or not the grant of
bail was proper
Whether or not only the
public prosecutor is the only
person that can file an opposition
to the grant of bail
Held:
No. The grant of bail was not
proper since no hearing was
conducted to determine whether
the evidence against the accused
was strong. Jurisprudence is
replete with decisions compelling
judges to conduct the required
hearings in bail applications, in
which the accused stands charged
with a capital offense. The absence
of objection from the public
prosecutor is never a basis for the
grant of bail in such cases, for the
judge has no right to presume that
the prosecutor knows what he is
doing on account of familiarity
with the case. "Said reasoning is
tantamount to ceding to the
prosecutor the duty of exercising
judicial discretion to determine
whether the guilt of the accused is
strong. Judicial discretion is the
domain of the judge before whom
the petition for provisional liberty

will be decided. The mandated


duty to exercise discretion has
never been reposed upon the
prosecutor.
As to the legal standing of
the private respondent, the court
declared that as a general rule
only the Solicitor General and
public prosecutor may challenge
the assailed order. However, as an
exception, the ends of substantial
justice
indeed
require
the
intervention of the offended party.
Clearly, the assailed order of the
RTC judge was issued in grave
abuse of discretion amounting to
lack of jurisdiction. A void order
cannot confer any right or be the
source of any relief. The Supreme
Court is not merely a court of law;
it is likewise a court of justice. To
rule otherwise would leave the
private respondent without any
recourse to rectify the public
injustice brought about by the trial
court's order, leaving her with only
the standing to file administrative
charges for ignorance of the law
against
the
judge
and
the
prosecutor. A party cannot be left
without recourse to address a
substantive issue in law.
25.
Tolentino vs. Judge
Camano, Jr.
A.M.
No.
RTJ-00-1522,
January 20, 2000
Doctrine:
Pending
preliminary
investigation, bail is a matter of
right.
Facts:
The complainant in this case
was the provincial prosecutor of

Camarines Sur, who was in-charge


to prosecute an alleged rape case
of a minor. However, the accused
in the rape case was denied a
preliminary investigation despite
his arrest, and the respondent
judge ordered the prosecutor to
conduct the necessary preliminary
investigation.
Pending
the
preliminary
investigation,
the
accused filed an application for
bail with the same court. The
respondent judge scheduled three
hearings for the bail application
but the public prosecutor failed to
appear despite due notice. A
fourth hearing was schedule but
the petitioner still did not attend.
Instead, he sent the assistant
provincial prosecutor to move for
postponement. The motion was
denied by the judge and granted
the application of bail and fixed
the amount of P100,000. The
prosecutor questioned the grant
for violating due process since the
prosecution was not granted the
opportunity
to
oppose
the
application and that pending
preliminary
investigation,
the
accused should not have been
granted bail. Also, the prosecutor
questioned the amount of bail for
being excessive. The respondent
judge alleged that he did not
violate due process since he gave
them multiple opportunities to
appear in court to question the
application. However, they failed
to appear in court despite notice.
Issues:
Whether or not bail may be
granted during the pendency of
the preliminary investigation

Whether or not there was a


violation of due process when the
judge continued to hear and grant
the application for bail
Whether or not the excessive
amount warrants the punishment
of the judge
Held:
As to the first issue, bail may
be granted despite the pendency of
the preliminary investigation since
during that time, bail is still a
matter of right. Indeed, if the
propriety of charging the accused
was yet to be resolved in the
preliminary investigation, it cannot
be claimed that the evidence of
guilt of the accused was strong so
as to justify the denial of bail to
him.
As to the second issue, the
respondent judge set the hearing
on the bail petition at least four
times
in
order
to
allow
complainant prosecutor present
his evidence. But complainant
failed to present his evidence. It
thus appears that it was the
complainant prosecutor who was
remiss in the performance of his
duties. Respondent judge should
have based his decision from the
evidence. However, the fact that
the case was referred to the Office
of the Provincial Prosecutor for
preliminary
investigation,
the
accused could be considered
entitled to bail as a matter of right.
As to the last issue, the
excessive amount was indeed
improper but the Supreme Court
declared that the remedy to this
was a motion for reconsideration

to reduce the amount. The Court


said that not all errors of the judge
can be the subject of disciplinary
action.
26.
Goodman vs. Judge De
la Victoria
A.M.
No.
RTJ-99-1473,
February 16, 2000
Doctrine: The law does not
prohibit the private complainant
from objecting to the application
for bail. Also, a general inquiry by
the judge does not satisfy the
mandatory
bail
hearing
requirement.
Facts:
The
administrative
case
against Judge De la Victoria
stemmed from a criminal case
involving the murder of an
American national named Jerome
Goodman who was allegedly killed
by Moalboal Mayor Abrenica and
others. The petitioner was the
private complainant in that case
who was denied her day in court
when
the
respondent
judge
granted bail to the accused
without
allowing
the
private
prosecutor to intervene in the
proceedings in the absence of the
public prosecutor. Also, it was
alleged that respondent judge did
not conduct a hearing but a mere
general
inquiry
since
the
preliminary investigation by the
ombudsman was not yet finished
and no evidence was yet adduced.
On the other hand, respondent
judge
alleged
that
private
complainant had no legal standing
to question the grant of bail since
that is vested upon the public

prosecutor and in their case, the


Ombudsman.
Issue:
Whether or not the private
complainant may object to the
application of bail in the absence
of the public prosecutor
Whether or not a general
inquiry is sufficient to justify the
requirement of a bail hearing since
the preliminary investigation is
still pending
Held:
As to the first issue, the
Supreme Court said that the
respondent judge was wrong in
refusing to hear complainant's
counsel at the hearing of the
application for bail. There is no
prohibition
for
counsel
of
complainant to appear before the
court during the hearing for
admission to bail of an accused.
There is no need of any special
authority
from
the
public
prosecutor to do so.
As to the second issue, the
grant of bail to an accused
charged with a capital offense
such as murder, which carries with
it the penalty of reclusion perpetua
to death, is discretionary. Thus, the
judge must conduct a hearing to
determine whether evidence of
guilt is strong. The judge is
mandated to conduct a hearing
even
in
cases
where
the
prosecution chooses to just file a
comment or leave the application
for bail to the discretion of the
court. The brief inquiry conducted
by
respondent
judge
before
granting bail did not constitute the

hearing required by law, for such


proceeding "did not elicit evidence
from the prosecution to guide
respondent
in
the
proper
determination of the petition.
Respondent judge could not have
intelligently assessed the weight of
the evidence against accused
before granting the application for
bail. There were no questions
propounded
by
respondent
verifying the strength of the
prosecution's evidence. In fact, the
prosecution was not represented
during
the
hearing.
Thus,
respondent judge failed to comply
substantially with the requirement
of a hearing in bail applications.
27.
People
vs.
Sandiganbayan
G.R. No. 158754, August 10,
2007
Doctrine: Where the evidence
against the accused is not strong,
bail may be granted by the court.
Facts:
This case stemmed from the
plunder case filed against former
San Juan Mayor and now Senator
Jinggoy Estrada who filed a very
urgent omnibus motion alleging
that no probable cause exists to
put him in trial for plunder but
only for illegal gambling and that
he is entitled to bail as a matter of
right. Initially, the Sandiganbayan
denied his application. On a
second try, Senator Estrada filed a
an application for bail based on his
medical condition and the same
was denied since the court did not
find sufficient medical reasons to
release him on bail. However,

Senator Estrada filed with the


Supreme Court a petition for
certiorari
claiming
that
the
Sandiganbayan committed grave
abuse of discretion in sustaining
the charge against him for alleged
crimes
and
with
alleged
conspirators with whom he is not
even connected and in not fixing
bail for him. Subsequently, the
Supreme Court ordered that bail
hearing be conducted by the
Sandiganbayan based on the
strength of the evidence against
him and not merely on the medical
evidence
adduced.
The
Sandiganbayan later found that
the evidence against him was not
strong and granted the bail
application. The petitioner then
appealed the decision to the
Supreme Court based on the idea
that he is flight risk and that the
evidence
against
him
for
conspiracy
was
strong.
The
petitioner even presented the
evidence before the Supreme
Court to show its strength.
Issue:
Whether or not the grant of
bail by the Sandiganbayan was
proper
Whether or not Senator
Estradas release will cause him to
flee
Held:
Yes.
The
Sandiganbayan
based its decision from the
evidence
presented
by
the
prosecution and even if the
prosecution presents its evidence
before the Supreme Court, the
fundamental principle that the

Supreme Court is not a trier of


facts remains.
As to the issue of being a flight
risk,
The
Supreme
Court
pronounced, it is not open to
serious doubt that the Senator has,
in general, been consistently
respectful of the Court and its
processes. He has not ominously
shown, by word or by deed, that he
is of such a flight risk that would
necessitate
his
continued
incarceration. Bearing in mind his
conduct, social standing and his
other personal circumstances, the
possibility of his escape in this
case seems remote if not nil. The
likelihood of escape on the part
individual respondent is now
almost nil, given his election on
May 10, 2004, as Senator of the
Republic of the Philippines. The
Court takes stock of the fact that
those who usually jump bail are
shadowy characters mindless of
their reputation in the eyes of the
people for as long as they can flee
from the retribution of justice. On
the other hand, those with a
reputation and a respectable name
to protect and preserve are very
unlikely to jump bail.
28.
Bravo vs. De Borja
G.R. No. L-65228, February
18, 1985
Doctrine: A minor who is charged
with a capital offense is entitled to
bail as a matter of right.
Facts:
The petitioner was charged
with murder with the RTC of Naga.
He applied for bail based on two

reasons. First, that the evidence


against him was not strong since
the witness Ferdinand Del Rosario
retracted his previous statement
naming petitioner as the assailant.
And second, that he is a minor of
16 years who is entitled to a
privilege mitigating circumstance
making the murder charge noncapital. However, during the
hearing, the retraction of the
witness was reversed and the
witness declared against petitioner
which prompted the judge to deny
his bail application based on the
fact that the evidence was strong
and his minority was not proven.
Petitioner filed a motion for
reconsideration stating that his
minority was proven by his birth
certificate
attached
to
the
memorandum in support of his
motion for bail and that it was
never challenged by the fiscal.
However, the fiscal opposed the
motion stating that the evidence
against him was strong. The judge
denied
the
motion
for
reconsideration. Subsequently, the
NBI report found that it was the
witness who actually killed the
deceased and not the petitioner.
This prompted petitioner to file a
petition
for
certiorari
and
mandamus with supplementary
petitions seeking the release of
petitioner on bail or under the
custody of the Ministry of Social
Services and Development for
being a minor.
Issue:
Whether or not bail may be
afforded a minor who is charged
with a capital offense

Held:
Yes. The Supreme Court
declared that the Constitution
withholds the guaranty of bail from
one who is accused of a capital
offense where the evidence of guilt
is strong. The obvious reason is
that one who faces a probable
death sentence has a particularly
strong temptation to flee. This
reason does not hold where the
accused has been established
without objection to be a minor
who by law cannot be sentenced to
death. This results that petitioner
is entitled to bail as a matter of
right.
29.
Savella vs. Judge Ines
A.M. No. MTJ-07-1673, April
19, 2007
Doctrine: Granting an application
for bail filed with another court
other than the court where the
case is pending is not proper.
Facts:
An administrative complaint
was filed against Judge Ines,
presiding judge of the MTC of
Sinait, Ilocos Sur, for serious
misconduct. It was alleged that
respondent judge granted bail to
an accused of falsification of public
documents which information was
already filed with the MTCC of
Vigan. However, a warrant of
arrest was issued against the
accused by the MTCC of Vigan.
When the warrant was served, the
daughter of the accused presented
a copy of the order of Judge Ines
granting bail to the accused. But a
copy of the bail order was never
furnished to the MTCC of Vigan.

The respondent judge contended


that due to the heavy workload
and on account of the Holy Week
celebration, she forgot to transmit
the bail order until she was
reminded by the clerk of court.
Issue:
Whether or not Judge Ines is
liable for granting bail when the
complaint for the crime charged
was already filed with another
court
Held:
Yes. Judge Ines is liable for
granting bail knowing that the
information charging the accused
was already filed with another
court. The Rules of Court explicitly
provides that "bail in the amount
fixed may be filed with the court
where the case is pending, or, in
the absence or unavailability of the
judge thereof, with any regional
trial judge, metropolitan trial
judge, municipal trial judge, or
municipal circuit trial judge of the
province or city or municipality."
The falsification case against
accused was filed before the
MTCC-Vigan, presided by Judge
Ante. There was no showing of the
unavailability of Judge Ante at that
time. Following the said rule,
respondent judge clearly erred in
entertaining the bail application
despite knowledge of the pendency
of the falsification case before the
MTCC of Vigan.
30.
Re: Report On The
Judicial Audit Conducted In
The Regional Trial Court,

Branch 4, Dolores, Eastern


Samar
A.M.
No.
06-6-340-RTC,
October 17, 2007
Doctrine: A judge must comply
with all the requirements set by
law in granting bail.
Facts:
The RTC of Dolores, Eastern
Samar was the subject of a judicial
audit since the presiding judge,
Judge Alvarez, was due for
compulsory retirement. However,
the audit team found that the said
judge failed to decide 27 cases for
a considerable length of time and
those cases which he inhibited
from were assigned to Judge
Bugtas of the same court. Some of
those cases were also unresolved.
One of the cases decided by Judge
Bugtas was a bail petition which
he granted but failed to transmit
the bail and release order to Judge
Alvarez who was handling the
criminal case. Judge Bugtas in his
reply letter mentioned that he
accepted the bail bonds since
Judge Alvarez was unavailable and
that he failed to transmit the order
since the accused in that case
failed to cause the annotation of
the lien on the certificate of title of
the property. Subsequently, Judge
Alvarez complied with all the
directives of the audit team but
Judge Bugtas was found to have
erred in deciding the bail petition
and was recommended by the
audit team to be dismissed for
gross inefficiency and ignorance of
the law.
Issue:

Whether or not the bail


petition was properly decided and
handled by Judge Bugtas which
may warrant his sanction

Doctrine: A mere application for


bail does not waive the right of the
accused to question the validity of
his arrest, except when it is
manifestly waived.

Held:
No. The Supreme Court
ruled that in bail applications
there are prerequisites to be
complied
with.
First,
the
application for bail must be filed in
the court where the case is
pending. In the absence or
unavailability of the judge thereof,
the application for bail must be
filed with another branch of the
same court within the province or
city. Second, if the accused is
arrested in a province, city or
municipality other than where the
case is pending, bail may be filed
with any regional trial court of the
place. There was no showing that
Judge Alvarez was absent or
unavailable when Judge Bugtas
approved the bail bond and there
was also no proof that the accused
were arrested in Borongan to
clothe Judge Bugtas with authority
to grant bail. In addition, Judge
Bugtas also failed to transmit the
bail order to the court where the
case is pending. Lastly, the
Supreme Court found that this was
not the first time Judge Bugtas was
found grossly ignorant of the law.
Hence, the Supreme Court ordered
that since Judge Bugtas had
already retired, his retirement
benefits be forfeited.
31.
Borlongan vs. Pea
G.R. No. 143591, May 5,
2010

Facts:
The complainants in this
case questioned the validity of the
issuance of warrants of arrest
against them alleging that they
were denied due process during
the preliminary investigation. They
claimed that they were not
afforded the right to submit their
counter-affidavit and therefore the
trial court judge merely based his
findings on the complaint-affidavit
of respondent. However, the trial
court declared that the petitioners
could no longer question the
validity of the warrant since they
already posted bail. The CA
dismissed the appeal of petitioners
saying it was moot and academic
to question the arrest since the
petitioners posted bail.
Issue:
Whether
or
not
an
application for bail constitutes a
waiver to question the validity of
the warrant of arrest issued
against the accused
Held:
No. The Supreme Court
declared that Section 26, Rule 114
of the Revised Rules on Criminal
Procedure is a new one, intended
to modify previous rulings of this
Court that an application for bail
or the admission to bail by the
accused shall be considered as a
waiver of his right to assail the
warrant issued for his arrest on

the legalities or irregularities


thereon. Even with the application
for bail, it cannot be argued that
petitioners waived their right to
question the finding of probable
cause and to assail the warrant of
arrest issued against them by the
judge. There must be clear and
convincing
proof
that
the
petitioner had an actual intention
to relinquish her right to question
the existence of probable cause.
When the only proof of intention
rests on what a party does, his act
should be so manifestly consistent
with, and indicative of, intent to
voluntarily
and
unequivocally
relinquish the particular right that
no other explanation of his conduct
is possible. Hence, the decision of
the CA is reversed.
Topic: Rights of the Accused
32.
People vs. Sy
G.R. No. 185284, June 22,
2009
Doctrine: It is the right of the
accused to be presumed innocent
until
proven
guilty
beyond
reasonable doubt.
Facts:
The accused Jason Sy was
the subject of a buy-bust operation
involving the illegal sale of shabu
in San Fernando, Pampanga. After
careful planning and execution,
the
police
officers
who
apprehended him validly complied
with all the requirements set by
law in marking, documenting, and
handling the prohibited drugs.
This was corroborated by all the
police officers who were involved

in the operation. On the other


hand,
the
accused
gave
a
completely different story stating
that he was a victim of abduction
and extortion. Because he did not
want to pay the alleged amount, he
was framed to have been selling
prohibited
drugs.
The
RTC
convicted Jason Sy finding more
weight to the evidence of the
prosecution. On appeal, the CA
affirmed the decision.
Issue:
Whether
or
not
the
prosecution was able to disprove
the presumption of innocence
afforded to the accused
Held:
Yes. The Supreme Court said
that
the
constitutional
presumption of innocence of the
accused cannot be overthrown
unless
the
prosecution
has
established by such quantum of
evidence sufficient to overcome
this presumption of innocence and
prove that a crime was committed
and that the accused is guilty
thereof. Jurisprudence has firmly
entrenched
the
following
as
elements in the crime of illegal
sale of prohibited drugs: (1) the
accused sold and delivered a
prohibited drug to another, and (2)
he knew that what he had sold and
delivered was a dangerous drug.
The testimonies of the prosecution
adequately
established
the
elements and there was no reason
to doubt their credibility which
was
even
bolstered
by
the
presence of the physical evidence.
Also, there was compliance with
the chain-of-custody rule. Hence,

the decision
affirmed.

of

conviction

is

33.
People vs. Ong
G.R. No. 137348, June 21,
2004
Doctrine: The presumption of
regularity in the performance of
official duty should not by itself
prevail over the presumption of
innocence.
Facts:
The accused in this case was
a Chinese National who was
apprehended during a buy-bust
operation for the illegal sale of
shabu. During the trial, the
prosecution presented only SPO1
Gonzales as its witness who
testified
that
a
confidential
informant reported to their Chief
regarding a certain William Ong
and his partner were involved in
illicit drug activities. A group of
police officers, including SPO1
Gonzales, planned a buy-bust
operation
along
with
the
informant. The officers asked the
informant to contact Ong to make
an order of drugs and to meet in a
certain place. Upon arriving at the
designated place, the informant
went down of the car and made
contact with Ong showing him the
marked money. After that, Ong
went to his partners car and got a
package of drugs. The informant
and SPO1 Gonzales gave the signal
and arrested the two accused. On
the other hand, the accused denied
the allegations, saying they could
not even speak the local language.
Ongs
alleged
partner
even
tesitified he was only at that place

to pick up his girlfriend and her


mother.
This
story
was
corroborated by the mother of the
accused. The trial court sentenced
the accused with death penalty.
Hence, an automatic appeal was
filed with the Supreme Court.
Issue:
Whether
or
not
the
presumption of innocence of the
accused was overcome by the
prosecution
Held:
No. The Supreme Court
ruled that there was no sufficient
evidenced
adduced
by
the
prosecution to overcome the
accused presumption of innocence.
The alleged entrapment buy-bust
operation of the police officers was
not sufficiently proven since SPO1
Gonzales was the only one who
testified and that he himself was
not the poseur-buyer but was
merely
a
deliveryman.
The
testimony
shows
that
the
confidential informant was the one
who spoke to the alleged seller
and no evidence was adduced as to
the agreement between them as to
the amount of drugs and the
amount to be paid. The invocation
of the presumption of regularity in
the performance of official duty by
law enforcement should not by
itself prevail over the presumption
of
innocence
and
the
constitutionally protected rights of
the individual. Hence, the decision
was reversed and set aside,
acquitting the accused.
34.
Hizon
Appeals

vs.

Court

of

G.R. No. 119619, December


13, 1996
Doctrine: The presumption of guilt
under the Fisheries Decree does
not violate the right of the accused
to be presumed innocent until
proven guilty beyond reasonable
doubt. The statutory presumption
is merely prima facie and may still
be refuted.
Facts:
28 fishermen were arrested
off the coast of San Rafael, Puerto
Princesa City, by the PNP Maritime
Command for allegedly conducting
illegal fishing operations. The
prosecution contended that a valid
warrantless search allows the
authorities
to
search
fishing
vessels for any possible violation of
the Fisheries Decree. The officers
were said to have confiscated 2
sets of fish samples and upon
chemical examination, one set
yielded
that
the
fish
was
contaminated with sodium cyanide
and the other set was clean. The
prosecution also contended that
the evidence acquired regarding
the presence of sodium cyanide in
the fishes was able to overcome
the presumption of innocence of
the accused and the burden of
proof has now shifted to the
accused. The defense contends
that the presumption of guilt in the
fisheries
decree
violates
the
presumption
of
innocence
guaranteed by the constitution.
Issue:
Whether
or
not
the
presumption of guilt under the
fisheries
decree
does
not

contradict the presumption of


innocence guaranteed by the
constitution
Held:
Yes. The Supreme Court
ruled that as early as 1916, the
court has rejected the argument of
the defense. The State having the
right to declare what acts are
criminal, within certain welldefined limitations, has the right to
specify what act or acts shall
constitute a crime, as well as what
proof shall constitute prima facie
evidence of guilt, and then to put
upon the defendant the burden of
showing that such act or acts are
innocent and are not committed
with any criminal intent or
intention, provided there is a
rational connection between the
facts proved and the ultimate fact
presumed. However, the statutory
presumption is merely prima facie.
It cannot, under the guise of
regulating the presentation of
evidence, operate to preclude the
accused
from
presenting
his
defense to rebut the main fact
presumed. At no instance can the
accused be denied the right to
rebut the presumption. As in this
case, the defense properly negated
this presumption when they were
able to show that the authorities
who searched the vessel were on a
fishing
expedition
themselves
since no evidence of poisonous
substance was found on the vessel
and the alleged substance was
merely acquired from one set of
fishes examined which is possible
to have been acquired by the
fishes from another source. Based
from the facts, the fishermen

merely fished by hook and line and


not through the use of obnoxious
or poisonous substance. Hence,
the presumption was properly
rebutted and the fishermen were
acquitted.
35.
People vs. Lagarde
G.R No. 182549, January 20,
2009
Doctrine: It is the constitutional
right of the accused to be informed
of the nature and cause of the
accusation
against
him.
His
conviction of a crime other than
that alleged in the information is a
violation of due process.
Facts:
The accused was charged
with the rape of a minor who was
11 years old. The prosecution
successfully proved the guilt of the
accused based from the testimony
of the witness and that the defense
failed to counter the allegations.
The defense merely relied on an
alibi which was not sufficiently
proven. Hence, the trial court
convicted him of rape and was
sentenced to death due to the
minority of the victim and other
aggravating
circumstance.
On
automatic appeal, the CA reduced
the
punishment
to
reclusion
perpetua on the ground that the
aggravating
circumstance
and
qualifying circumstance were not
alleged
in
the
information,
preventing the accused from
making the proper defense.

qualified rape though the


information only alleged a
simple rape
Held:
No.
It
is
a
basic
constitutional right of the accused
persons to be informed of the
nature and cause of accusation
against them. It would be a denial
of accused-appellants basic right
to due process if he is charged
with simple rape and consequently
convicted with certain qualifying
circumstances which were not
alleged in the information. The
Supreme Court agreed with the
appellate court that the death
penalty is not warranted by the
alleged
aggravating
circumstances,
i.e.,
victims
minority, use of bladed weapon,
and uninhabited place. First, the
death penalty was abolished and
second, the use of a bladed
weapon and uninhibited place
cannot
be
appreciated
here
because these were not specifically
alleged in the information. Hence,
the sentence was reduced to
reclusion perpetua.
36.
People vs. Navarro
G.R. No. 173790, October
11, 2007
Doctrine: The equipoise rule
provides that where the evidence
of the parties in a criminal case is
evenly balanced, the constitutional
presumption of innocence should
tilt the scales in favor of the
accused.

Issue:
Whether or not the accused
may be punished for a

Facts:

The accused in this case was


convicted by the trial court for the
crime of illegally possessing and
selling prohibited drugs. The
police
officers
and
MADAC
members successfully conducted a
buy-bust operation where the
accused was apprehended and was
found with 2 plastic sachets of
shabu, as certified by the Forensic
Chemist. All of the necessary
procedure in drug cases was
complied with by the police
officers. Petitioner contends that
arresting authorities framed him
and gave him the prohibited drugs
during investigation. The accused
believes that the equipoise rule is
applicable and therefore warrants
his acquittal.
Issue:
Whether or not the equipoise
rule is applicable
Held:
No. The Supreme Court
explained the equipoise rule to
apply when, the inculpatory facts
and circumstances are capable of
two or more explanations, one of
which is consistent with the
innocence of the accused and the
other consistent with his guilt,
then the evidence does not fulfill
the test of moral certainty, and
does not suffice to produce a
conviction. In the present case, the
accused failed to present any
motive and evidence on the part of
the authorities to pursue his
alleged
harassment.
The
inculpatory facts are not capable
of any explanation other than
appellants guilt of sale and

possession of methylamphetamine
hydrochloride or shabu.
37.
Malana vs. People
G.R. No. 173612, March 26,
2008
Doctrine: The defense of alibi and
denial are outweighed by positive
identification that is categorical,
consistent and untainted by any ill
motive on the part of the
eyewitness testifying on the matter
and
does
not
warrant
the
application of the equipoise rule.
Facts:
In this case, the accused
Dominador Malana and Rodel
Tiaga were charged with murder
for the death of a certain Betty
Roxas and frustrated murder of
her
husband,
Vicente,
their
daughter
Suzette
and
granddaughter,
Jenny.
The
prosecution presented in evidence
Suzettes testimony which was
corroborated by Vicente. It was
alleged that Suzette personally
identified the two accused along
with another unidentified man
entered their home at night and
started burning the house. The
accused also brought a gallon
container filled with gas and had a
wick which was thrown into the
bedroom and exploded causing the
death of the mother and the
burning of many body parts of the
survivors. The victims alleged that
the accused have been issuing
threats against their family for
allegedly
being
engaged
in
witchcraft. On the other hand, the
accused resorted to alibi stating
that Dominador was currently

working in another province when


the incident happened and that
Rodel was healing from sickness.
Also, the accused invoked the
application of the equipoise rule.
The trail court convicted the
accused giving more weight to the
testimony of Suzette since she
could not have made the story up
due to the fact that she lost her
mother and her family sustained
multiple injuries from the incident.

physically impossible for them to


have been at the scene of the
crime at the
time it was
committed, which they failed to do.

Issue:

PEOPLE OF THE PHILIPPINES


versus JOSELITO NOQUE y
GOMEZ

Whether or not the equipoise


rule is applicable to the case

Right to be informed on the


nature and cause of accusation

Held:
No. The Supreme Court
stated that the equipoise rule is
not applicable to the case. The rule
provides that where the evidence
of the parties in a criminal case is
evenly balanced, the constitutional
presumption of innocence should
tilt the scales in favor of the
accused.
Against
the
direct,
positive and convincing evidence
for the prosecution, appellants
could only offer denials and
uncorroborated
alibi.
It
is
elementary that alibi and denial
are
outweighed
by
positive
identification that is categorical,
consistent and untainted by any ill
motive on the part of the
eyewitness
testifying
on
the
matter. Alibi and denial, if not
substantiated
by
clear
and
convincing evidence, are negative
and
self-serving
evidence
undeserving of weight in law. It is
incumbent upon appellants to
prove that they were at another
place when the felony was
committed, and that it was

FACTS:

Senior Police Officer 4 (SPO4)


Norberto Murillo, went to the
Western Police District (WPD) to tip
off on the drug trafficking activities
of the appellant in Malate, Manila.
SP04 Murillo immediately directed
Police Officers (POs) Christian Balais
(Balais) and Dionisio Borca (Borca) to
conduct surveillance in the area. The
police officers conducted a buy-bust
operation after receiving confirmed
surveillance at No. 630 San Andres
Street, Malate, Manila.
PO1 Balais, the designated
poseur-buyer and the informant
thereafter called out the appellant,
who welcomed the two and brought
them to his bedroom. The informant
asked the appellant if he had
P1,000.00
worth
of

methamphetamine hydrochloride or
shabu then pointed to PO1 Balais as
the actual buyer. When PO1 Balais
handed the marked money to the
appellant, the latter brought out from
under a table a pranela bag from
which he took two plastic sachets
containing white crystalline granules
suspected to be
shabu. The
informant slipped out of the house as
the pre-arranged signal to the buybust team that the sale had been
consummated.

Whether or not conviction of


the appellant for the sale and
possession of shabu, despite the fact
that what was established and
proven was the sale and possession
of
ephedrine,
violated
his
constitutional right to be informed of
the nature and cause of the
accusations against him since the
charges in the Informations are for
selling
and
possessing
methamphetamine hydrochloride.
HELD:

After seeing the informant


leave, the team entered appellants
house. SPO4 Murillo frisked the
appellant and recovered the buy-bust
money. . He also confiscated the
pranela bag that contained a large
quantity of crystalline granules
suspected to be shabu.
The seized articles were taken
to the police station and submitted to
the crime laboratory for examination.
Police
Inspector
(P/Insp.)
and
Forensic Chemical Officer Miladenia
Tapan examined one self-sealing
transparent
plastic
bag
with
markings JNG containing 679.215
grams of white crystalline granules;
and two heat-sealed transparent
plastic sachets each containing white
crystalline substance, pre-marked
JNG-1 weighing 2.779 grams and
JNG-2weighing 2.729 grams. The
qualitative
examinations
yielded
positive results for ephedrine, a
regulated drug.
ISSUE:

The
designations
and
allegations in the informations are
for the crimes of illegal sale and
illegal possession of regulated drugs.
There being no dispute that
ephedrine is a regulated drug,
pursuant to Board Resolution No. 2,
Series of 1988, issued by the
Dangerous Drugs Board on March
17, 1988, the appellant is deemed to
have been sufficiently informed of
the nature of the crime with which
he is accused.
The only difference between
ephedrine and methamphetamine is
the presence of a single atom of
oxygen in the former.
An
offense
charged
is
necessarily included in the offense
proved
when
the
essential
ingredients of the former constitute
or form part of those constituting the
latter. In other words, his right to be
informed of the charges against him
has not been violated because where
an accused is charged with a specific

crime, he is duly informed not only of


such specific crime but also of lesser
crimes or offenses included therein.

PEOPLE OF THE PHILIPPINES


versus ALFREDO PANGILINAN
y TRINIDAD

properly arraigned. Since he was


arraigned only after the case was
submitted
for
decision,
said
irregularity, he argues, is a
procedural
error
which
is
prejudicial to the appellant and is
tantamount to denial of his
constitutional right to be informed
of the accusation against him.

FACTS:

ISSUE:

Accused- appellant Alfredo


Pangilinan was convicted before
the trial court of rape committed
against his daughter, AAA. Records
reveal that Pangilinan repeatedly
raped AAA which started on
September 9, 1995.

Whether or not Trial Court


erred in convicting the accused of
two counts of rape despite the fact
that he was not properly arraigned
and was not informed of the nature
and cause of the accusation
against him.

On March 16, 1997, BBB,


wife of herein appellant, informed
her children that she was leaving
for Singapore again. DDD, AAAs
grandmother, advised BBB not to
leave her children. She told BBB
that appellant had been molesting
AAA. Shocked by the revelation,
BBB confronted AAA. AAA tearfully
confessed
everything
to
her
mother. On March 17, 1997, BBB
brought AAA to the Dinalupihan
District Hospital where she was
examined.
The
examination
revealed that the victim had a nonparous introitus with an old healed
hymenal laceration at the 4 oclock
position. Thus the instant case of
rape was filed.

HELD:

Appellant
assails
his
conviction because he was not

Arraignment is the formal


mode and manner of implementing
the constitutional right of an
accused to be informed of the
nature and cause of the accusation
against him.
Admittedly, appellant
arraigned after the case
submitted for decision.

was
was

Appellants
belated
arraignment did not prejudice him.
This procedural defect was cured
when his counsel participated in
the trial without raising any
objection that his client had yet to
be arraigned. In fact, his counsel
even
cross-examined
the
prosecution
witnesses.
His
counsels active participation in the

hearings is a clear indication that


he was fully aware of the charges
against him; otherwise, his counsel
would have objected and informed
the
court
of
this
blunder.
Moreover, no protest was made
when appellant was subsequently
arraigned.It is only now, after
being convicted and sentenced to
two
death
sentences,
that
appellant
cries
that
his
constitutional right has been
violated. It is already too late to
raise this procedural defect. This
Court will not allow it.

Right to Counsel
JOHN HILARIO y SIBAL versus
PEOPLE OF THE PHILIPPINES
FACTS:
Petitioner, together with one
Gilbert Alijid (Alijid), was charged
with two counts of Murder in the
Regional Trial Court (RTC), Branch
76, Quezon City to which petitioner,
assisted by counsel de parte, pleaded
not guilty.
During trial, Atty. Raul Rivera
of the Public Attorney's Office (PAO),
counsel
of
Alijid,
took
over
representing petitioner in view of the
death of the latter's counsel.

On December 5, 2001, the RTC


rendered
its
Decision
finding
petitioner and his co-accused Alijid
guilty beyond reasonable doubt of the
crime of homicide.
On May 10, 2002, petitioner,
this time unassisted by counsel, filed
with the RTC a Petition for Relief
from the Decision dated December 5,
2001 together with an affidavit of
merit. In his petition, petitioner
contended that at the time of the
promulgation of the judgment, he was
already confined at Quezon City Jail.
He instructed his lawyer to file
the
necessary
motion
for
reconsideration or notice of appeal;
that on May 2, 2002, he was already
incarcerated at the New Bilibid
Prisons, Muntinlupa City and learned
from the grapevine of his impending
transfer to the Iwahig Penal Colony,
Palawan; that believing that the
notice of appeal filed by his counsel
prevented
the
Decision
dated
December 5, 2001 from becoming
final to warrant his transfer, he
instructed his representative to get a
copy of the notice of appeal from the
RTC; that no notice of appeal was
filed by his lawyer in defiance of his
clear instructions; and that the RTC
Decision showed that it was received
by his counsel on February 1, 2002
and yet the counsel did not inform
him of any action taken thereon.
In an Order dated December
13, 2002, the RTC dismissed
petitioner's petition for relief on the
ground that the Court is not

persuaded by petitioner/accused's
allegation that he was prevented from
filing a notice of appeal due to
excusable negligence of his counsel.

the exception is when the negligence


of counsel is so gross, reckless and
inexcusable that the client is deprived
of his day in court.

Petitioner, again by himself,


filed a petition for certiorari with the
CA
which
was
subsequently
dismissed for having been filed
beyond the 15-day reglementary
period, in violation of Section 1, Rule
52 of the Rules of Court failure to
attach to the petition, the relevant
and pertinent documents.

If the incompetence, ignorance


or inexperience of counsel is so great
and the error committed as a result
thereof is so serious that the client,
who otherwise has a good cause, is
prejudiced and denied his day in
court, the litigation may be reopened
to give the client another chance to
present his case.

ISSUES:

If indeed there was such an


instruction to appeal but the lawyer
failed to do so, he could be considered
negligent.

1. Whether or not the RTC


erred in dismissing petitioners
petition finding that no excusable
negligence prevented him from filing
notice of appeal.
2. Whether or not the ground
for dismissing the petitioners petition
for certiorari before the CA is correct.
HELD:
1.
Excusable
negligence
prevented him from filing notice of
appeal.
In this case, petitioner claims
he had instructed the PAO lawyer to
file an appeal. Under the PAO
Memorandum Circular, it was the
duty of the latter to perfect the
appeal.
While as a general rule,
negligence of counsel may not be
condoned and should bind the client,

2. The ground relied upon by


the CA for dismissing the petition is
not correct.
The CA failed to consider the
fact that the petition before it was
filed by petitioner, a detained
prisoner, without the benefit of
counsel. A litigant who is not a lawyer
is not expected to know the rules of
procedure.
In criminal cases, the right of
an accused person to be assisted by a
member of the bar is immutable.
Otherwise, there would be a grave
denial of due process. Thus, even if
the judgment had become final and
executory, it may still be recalled, and
the accused afforded the opportunity
to be heard by himself and counsel.

The filing of the petition for


certiorari by petitioner without
counsel should have alerted the CA
and should have required petitioner
to cause the entry of appearance of
his counsel. Although the petition
filed before the CA was a petition for
certiorari assailing the RTC Order
dismissing the petition for relief, the
ultimate relief being sought by
petitioner was to be given the chance
to file an appeal from his conviction,
thus the need for a counsel is more
pronounced.
To repeat the ruling in Telan,
no arrangement or interpretation of
law could be as absurd as the position
that the right to counsel exists only in
the trial courts and that thereafter,
the right ceases in the pursuit of the
appeal. It is even more important to
note that petitioner was not assisted
by counsel when he filed his petition
for relief from judgment with the
RTC. It cannot be overstressed
therefore, that in criminal cases, as
held in Telan, the right of an accused
person to be assisted by a member of
the bar is immutable; otherwise,
there would be a grave denial of due
process.
No
arrangement
or
interpretation of law could be as
absurd as the position that the right
to counsel exists only in the trial
courts and that thereafter, the right
ceases in the pursuit of the appeal.
The ultimate relief being
sought by petitioner was to be given
the chance to file an appeal from his

conviction, thus the need for a


counsel is more pronounced.

JUANITA A. AQUINO vs.


TERESITA B. PAISTE
FACTS:
At about 9:00 oclock in the
morning of March 14, 1991,
petitioner
Juanita
Aquino,
Elizabeth Garganta, and another
woman
identified
only
as
"Adeling," went to the house of
respondent Teresita Paiste. There,
petitioner started to convince
respondent to buy a gold bar
owned by a certain Arnold. After
respondent was shown a sample of
the gold bar, she agreed to go with
them to a pawnshop in Tondo to
have it tested. She was told that it
was genuine. However, she told
the three that she had no money.
The following day, the two
convinced her to go with them to
meet Arnold and see the gold bar.
They met Arnold who showed them
the gold bar. Arnold informed her
that it was worth PhP 60,000. On
March
16,
1991,
petitioner,
Garganta, and Adeling returned to
the house of respondent. Again,
they failed to convince her to buy
the gold bar.
On the next day, the three
returned, this time they told
respondent that the price was
reduced to PhP 10,000. She

agreed to go with them to Angeles


City to meet Arnold once more.
Arnold pretended to refuse the
PhP 10,000 offer and insisted on
PhP 50,000.
On petitioners insistence, on
March 18, 1991, the two went to
Angeles City and bought the gold
bar for PhP 50,000.
On
March
19,
1991,
respondent had the gold bar tested
and she was informed that it was
fake. Respondent then proceeded
to petitioners house to inform the
latter that the gold bar was fake.
Petitioner replied that they had to
see Garganta, and that she had
nothing to do with the transaction.
On
March
27,
1991,
respondent brought petitioner to
the
National
Bureau
of
Investigation (NBI)-NCR in the
presence of a certain Atty.
Tolentino
where
petitioner
amicably promised respondent
they would locate Garganta, and
the document they both signed
would be disregarded should they
locate Garganta.
The accused/respondent also
executed a waiver of her right to
counsel despite the recital of her
constitutional rights made by NBI
agent
Ely
Tolentino
in
the
presence of a lawyer Gordon S. Uy.
Subsequently,
respondent
filed a criminal complaint from
which an Information against

Garganta, petitioner, and three


others for the crime of estafa.
The prosecution presented
as documentary evidence three (3)
documents, one of which is the
amicable settlement signed in the
NBI
On July 16, 1998, the trial
court
rendered
a
Decision
convicting petitioner of the crime
charged,
This
decision
was
affirmed by the Court of Appeals.
ISSUE:
Whether
or
not
the
respondents constitutional right
was breached and the amicable
settlement with waiver of right to
counsel executed in the NBI is
admissible as evidence.
HELD:
Petitioners contention falls
flat.
Custodial
investigation
involves any questioning initiated
by law enforcement officers after a
person has been taken into
custody or otherwise deprived of
his freedom of action in any
significant way. It is only after the
investigation ceases to be a
general inquiry into an unsolved
crime and begins to focus on a
particular suspect, the suspect is
taken into custody, and the police
carries
out
a
process
of
interrogations that lend itself to

eliciting incriminating statements,


that the rule begins to operate.
Republic Act No. (RA) 7438 has
extended
this
constitutional
guarantee to situations in which an
individual has not been formally
arrested but has merely been
"invited" for questioning.
It is evident that when
petitioner
was
brought
by
respondent before the NBI-NCR on
March 27, 1991 to be investigated,
she was already under custodial
investigation
and
the
constitutional guarantee for her
rights under the Miranda Rule has
set in. Since she did not have a
lawyer then, she was provided
with one in the person of Atty. Uy,
which fact is undisputed.
However, it can be gleaned
from the amicable agreement, as
aptly pointed out by the CA, that
the custodial investigation on the
inquiry or investigation for the
crime was either aborted or did
not push through as the parties,
petitioner, and respondent agreed
to amicably settle. Thus, the
amicable settlement with a waiver
of right to counsel appended was
executed with both parties affixing
their signatures on it in the
presence of Atty. Uy and NBI agent
Atty. Ely Tolentino.

PEOPLE OF THE PHILIPPINES


vs. JERRY RAPEZA y
FRANCISCO

FACTS:
In the afternoon of 21
October 1995, an unidentified
woman went to the Culion
Municipal Station and reported a
killing that had taken place in Sitio
Cawa-Cawa, Barangay Osmea,
Culion, Palawan. The officer-incharge, SPO2 Ciriaco Gapas, sent
to the scene of the crime an
investigating team led by SPO2
Crisanto Cuizon, Jr. and PO2 Isidro
Macatangay. There they saw two
bloodied bodies later identified as
Priscilla Libas and Cesar Ganzon.
Upon information supplied
by a certain Mr. Dela Cruz that
appellant had wanted to confess to
the crimes, SPO2 Gapas set out to
look for appellant. He found
appellant fishing in Asinan Island
and
invited
the
latter
for
questioning. Appellant expressed
his
willingness
to
make
a
confession in the presence of a
lawyer. Appellant was then brought
to the police station after which
SPO2 Gapas requested Kagawad
Arnel
Alcantara
to
provide
appellant with a lawyer. The
following
day,
appellant
was
brought to the house of Atty.
Roberto Reyes, the only available
lawyer in the municipality. The
typewriter at the police station
was out of order at that time and
Atty. Reyes could not go to the
police station as he was suffering
from rheumatism. At the house of
Atty. Reyes, in the presence of
Vice-Mayor Emiliano Marasigan of

Culion, two (2) officials of the


Sangguniang
Barangay,
SPO2
Cuizon and an interpreter, SPO2
Gapas
proceeded
with
the
custodial investigation of appellant
who was assisted by Atty. Reyes.
Appellant was expressly advised
that he was being investigated for
the death of Libas and Ganzon.
Per
the
Sinumpaang
Salaysay that appellant executed,
he
was
informed
of
his
constitutional rights.
An interpreter was provided
appellant as he was not well
versed in Tagalog being a native of
Samar. As he is illiterate, appellant
affixed only his thumbmark on the
statement above his printed name.
Bonifacio Abad, the interpreter,
and Atty. Reyes, as the assisting
counsel, also signed the statement.
Atty. Reyes signed again as the
notary public who notarized the
statement.
ISSUE:
1. Whether or not appellants
extrajudicial
confession
is
admissible in evidence to warrant
the verdict of guilt.
2. Whether or not the
counsel
contemplated
in
the
Constitution
was
adequately
provided to the appellant to make
his
extrajudicial
confession
admissible.
HELD:

1.
There
is
no
direct
evidence
of
appellants
guilt
except for the alleged confession
and the corpus delicti. Upon
careful examination of the alleged
confession and the testimony of
the witnesses, we hold that the
alleged confession is inadmissible
and must perforce be discarded. A
confession
is
admissible
in
evidence if it is satisfactorily
shown to have been obtained
within the limits imposed by the
1987 Constitution. Sec. 12, Art. III
thereof.
If
the
extrajudicial
confession
satisfies
these
constitutional standards, it must
further be tested for voluntariness,
that is, if it was given freely by the
confessant without any form of
coercion or inducement
Republic Act No. 7438,
approved on 15 May 1992, has
reinforced
the
constitutional
mandate protecting the rights of
persons
under
custodial
investigation.
2. Upon careful examination
of the alleged confession and the
testimony of the witnesses, we
hold that the alleged confession is
inadmissible and must perforce be
discarded.
The standards of "competent
counsel" were not met in this.
Although Atty. Reyes signed the
confession as appellants counsel
and he himself notarized the

statement, there is no evidence on


how he assisted appellant. The
confession
itself
and
the
testimonies of SPO2 Gapas and
SPO2 Cuizon bear no indication
that Atty. Reyes had explained to
appellant his constitutional rights.
It appears that his participation in
the proceeding was confined to the
notarization
of
appellants
confession. Such participation is
not the kind of legal assistance
that should be accorded to
appellant in legal contemplation.
Furthermore, Atty. Reyes
was not appellants counsel of
choice but was picked out by the
police officers allegedly through
the barangay officials.
To reiterate, the purpose of
providing counsel to a person
under custodial investigation is to
curb the police-state practice of
extracting a confession that leads
appellant
to
make
selfincriminating statements.
It bears repeating that
appellant was held in the police
station overnight before he was
allegedly taken to the house of
Atty. Reyes. He was not informed
of his rights and there is no
evidence that he was assisted by
counsel. Thus, the possibility of
appellant having been subjected to
trickery and intimidation at the
hands of the police authorities, as
he claims, cannot be entirely
discounted.

In order to comply with the


constitutional
mandates,
there
should likewise be meaningful
communication
to
and
understanding of his rights by the
appellant, as opposed to a routine,
peremptory
and
meaningless
recital
thereof.
Since
comprehension is the objective,
the degree of explanation required
will necessarily depend on the
education, intelligence, and other
relevant personal circumstances of
the
person
undergoing
investigation.

PEOPLE OF THE PHILIPPINES


vs. ERIC GUILLERMO y GARCIA
FACTS:
The victim, Victor Francisco
Keyser, was the owner and
manager
of
Keyser
Plastic
Manufacturing Corp. at Sitio
Halang, Lornaville, San Roque,
Antipolo City. Keyser Plastics
shared its building with Greatmore
Corporation which was separated
by a wall. The part of the wall
made of lawanit had two large
holes, which could allow a person
on one side of the wall to see what
was on the other side.
On
March
22,
1998,
prosecution
witness
Romualdo
Campos, a security guard assigned
to Greatmore was on duty. At
around 8:00 a.m., he saw appellant
Eric G. Guillermo enter the

premises of Keyser Plastics. An


hour later, he saw Victor F. Keyser
arrive.
Later, at around 10:00 a.m.,
he heard some loud noises
(kalabugan) coming from the
Keyser Plastics area. He did not
pay much attention to the sound.
At around noontime, Campos
was surprised when he saw
appellant Guillermo look through
the holes in the dividing wall.
Appellant calmly told him that he
had killed Victor Keyser and
needed
Campos
assistance.
Shocked
by
this
revelation,
Campos immediately dashed off to
telephone the police. The police
told him to immediately secure the
premises and not let the suspect
escape.
A team from the Antipolo
Philippine National Police (PNP)
Station arrived at the crime scene.
Campos
informed
them
that
Guillermo was still inside the
building.
Once
inside,
SPO4
Bautista
and
SPO1
Reyes
immediately accosted Guillermo.
SPO1 Reyes then asked him where
the body of the victim was and
Guillermo
pointed
to
some
cardboard boxes. On opening the
boxes, the police found the
dismembered limbs and chopped
torso of Victor F. Keyser. The
victims head was found stuffed
inside a cement bag.

When the police asked how


he did it, according to the
prosecution witness, Guillermo
said that he bashed the victim on
the head with a piece of wood, and
after Keyser fell, he dismembered
the body with a carpenters saw.
The police then brought
Guillermo to the Antipolo PNP
Station for further investigation.
During
the
interrogation,
no
counsel was present for the
accused but he was apprised of his
constitutional rights by making
him read the written rights posted
on the wall.
ISSUE:
Whether or not the accused
was fully informed of his right
during the police interrogation
that results to understanding of
what is conveyed.
HELD:
Appellants
alleged
confession at the police station
lacks the safeguards required by
the Bill of Rights. The investigating
officer made no serious effort to
make appellant aware of his basic
rights
under
custodial
investigation.
While
the
investigating officer was aware of
the
appellants
right
to
be
represented by counsel, the officer
exerted no effort to provide him
with one on the flimsy excuse that
it was a Sunday. Despite the
absence of counsel, the officer

proceeded with said investigation.


Moreover, the record is bare of any
showing that appellant had waived
his constitutional rights in writing
and in the presence of counsel. As
well said in People v. Dano, even if
the admission or confession of an
accused is gospel truth, if it was
made without the assistance of
counsel, it is inadmissible in
evidence regardless of the absence
of coercion or even if it had been
voluntarily given.
The right of a person under
interrogation to be informed
implies a correlative obligation on
the part of the police investigator
to explain and contemplates an
effective
communication
that
results in an understanding of
what is conveyed. Absent that
understanding, there is a denial of
the right to be informed, as it
cannot be said that the person has
been truly informed of his rights.
Ceremonial
shortcuts
in
the
communication
of
abstract
constitutional principles ought not
be allowed for it diminishes the
liberty of the person facing
custodial investigation.

LOLITA Y. EUGENIO versus


PEOPLE OF THE PHILIPPINES
FACTS:
The prosecution evidence
showed that on 14 November
1995, petitioner went to the house

of private complainant Alfredo


Mangali (Mangali) in Tonsuya,
Malabon,
Metro
Manila
and
introduced Mangali to Epifania
Saquitan
(Saquitan),
Amalia
Ablaza (Ablaza), and another
individual. Petitioner persuaded
Mangali to loan P100,000 to
Saquitan with a parcel of land
covered by Transfer Certificate of
Title (TCT) No. 171602 issued in
Saquitans name, in Sta. Ana,
Metro Manila as security for the
loan. The validity of TCT No.
171602 was later confirmed which
prompted the extension of loan,
subject to Saquitans execution of a
deed of sale. Loan was promised to
be paid 21 December 1995.
Subsequently, petitioner, on
behalf of one Lourdes Ty (Ty),
sought another P100,000 loan
from Mangali, payable in January
1996 with a parcel of land in
Quezon City (Quezon City lot) as
security. Petitioner represented
that the property was covered by
TCT No. 92585 issued in Tys name.
Mangali agreed to extend the loan,
again subject to the condition that
Ty execute a deed of sale over the
Quezon City lot in his favor. After
Ty complied, Mangali released the
partial amount of P75,000.
When the loans lapsed and
remained unpaid, Mangali inquired
from the Register of Deeds of
Manila and Quezon City on the
status of TCT No. 171602 and TCT
No. 92585, respectively. Mangali
discovered that TCT No. 171602

had been cancelled on 5 October


1995 while TCT No. 92585 is not
registered with the Register of
Deeds of Quezon City. Mangali
filed a complaint with the National
Bureau of Investigation (NBI)
which arranged an entrapment
operation on 26 February 1996 in
Mangalis house. Mangali expected
to see petitioner that day as
petitioner had asked for an
additional P33,000 loan. At the
appointed time, petitioner, Ablaza,
and
two
other
unidentified
individuals arrived in Mangalis
house. The NBI agents effected the
arrest
while
petitioner
was
counting the money. The NBI
agents brought petitioner and the
other individuals arrested with her
to the NBI office where petitioner
gave a statement.
Upon investigation by the
NBI, it was discovered that the
Epifania Saquitan who owned the
Sta. Ana lot was a 79-year old
woman who denied mortgaging the
Sta. Ana lot or knowing petitioner
and her co-accused. This Epifania
Saquitan executed an affidavit
attesting to these facts.
ISSUE:
Whether
or
not
the
petitioners contention that her
constitutional right to counsel has
been violated is correct.
HELD:

As for the failure of the NBI


agents to inform petitioner of her
right to counsel during custodial
investigation, this right attains
significance only if the person
under
investigation
makes
a
confession in writing without aid of
counsel which is then sought to be
admitted against the accused
during the trial. In such case, the
tainted confession obtained in
violation of Section 12(1), Article
III
of
the
Constitution
is
inadmissible in evidence against
the accused.
Here,
petitioner
merely
alleges that following her arrest,
she gave a statement to the NBI
agents. The records do not contain
a copy of this statement thus we
have no way of knowing whether
such statement amounts to a
confession under Section 12(3) in
relation to Section 12(1), Article III
of the Constitution. At any rate, no
allegation has been made here that
the prosecution submitted such
statement in evidence during the
trial.

PEOPLE OF THE PHILIPPINES,


vs. WARLITO TOLENTINO y
LAQUIN
FACTS:
Mylene Mendoza testified
that at around six oclock in the
evening of February 6, 1996, her
father, Carlos Mendoza, sent her to

the house of her Lola Asiang to tell


the latter that he could not report
for work the following day as he
needed to repair their kitchen. Her
Lola Asiang was not around when
she arrived at her house. Mylene
then noticed a man, later identified
as Warlito Tolentino standing at
the terrace of the adjacent house
looking at her. Appellant then
approached her, took her to his
houses living room where he
violently struck her on the head,
rendering her unconscious.
Carmelita
testified
that
together with her sister, they
looked for Mylene when it was
already dark and found her lying
unconscious on the street 10
meters away from the appellants
dwelling. They took her to a
hospital. When Mylene regained
consciousness the following day,
Mylene disclosed that she was
brought inside a house where she
was clobbered. She also said she
could easily identify that house.
Barangay Captain Joseph
Colbongan testified that upon
receiving information that an
unconscious woman has been
found, he then gave instructions to
the barangay officials to bring the
victim to the hospital. On the
following day, February 7, 1996,
Colbongan and the barangay
officers of Batal went to the
hospital to elicit information from
the victim. She informed him that
she could not name her assailant
because she did not know his

name, but could recall the face and


appearance of her assailant as well
as the house where she was
brought to.

The barangay officers then


brought Mylene to Sitio Namnama,
Batal, where the incident allegedly
happened. Upon reaching the
vicinity of the Day Care Center in
Namnama, and at a distance of 50
meters,
Mylene
pointed
to
appellants house. Mylene was then
brought to another place in
Namnama, some 20 meters away
from the appellants residence, and
again asked to point to the house
where the incident took place.
Again,
Mylene
pointed
to
appellants house. Colbongan then
requested for police assistance.
Colbongan likewise declared that
after Tolentino was taken into
custody by the police and made to
take part in a police line-up,
Mylene unerringly pointed to him
as her molester.
ISSUE:
Whether
or
not
the
appellants constitutional right to a
counsel was violated when the
police required him to join the lineup
HELD:
Appellant cannot claim that
he was denied the protection of
the safeguards under the Bill of
Rights when he was made to join

the police line-up, since it is


settled that a police line-up is not
part of the custodial investigation

Also without firm basis is


appellants claim that his rights
under Article III, Section 12 of the
Constitution were violated when
he was made to join the police lineup. In Gamboa v. Cruz, we held
that a police line-up was not part
of the custodial inquest, inasmuch
as the accused therein was not yet
being investigated and hence, the
right to counsel had not yet
attached. This ruling was affirmed
in People v. Loveria, and People v.
De Guzman. Both held that where
the accused was not being
investigated by the police, when
the witness was in the process of
identifying him, his right to
counsel was not violated. The
reason is that at this stage, he was
not entitled to the constitutional
guarantee invoked. Under the
circumstances of this case, we see
no reason to depart from these
cited precedents.

THE PEOPLE OF THE


PHILIPPINES vs. FELICIANO
ULIT y TAMPOY
FACTS:
On trial, Lucelle alleged that
sometime in November 1996, she
was sleeping in a room in the
house and was awakened when

she felt her uncle-appellant kissing


her on the cheek while armed with
a bladed weapon (balisong). He
poked the weapon on the left side
of her neck. He warned her that if
she told her parents, he would kill
her. The appellant, thereafter, left
the room after satisfying his carnal
knowledge. Also during the month
of November 1996, the appellant
continued kissing her whenever
her parents were out of the house.
In December 1996, Lucelle
was in the room when the
appellant entered and kissed her
and mashed her private parts.
Sometime in February 1997, the
appellant
again
abused
her
(sinalbahe) while she was in the
same room. On March 2, 1997,
Lucelle urinated in the bathroom
and when she was about to go out,
the appellant entered, pushed her
inside and kissed her on her
cheeks several times.
Lourdes Serrano, victims
mother, testified that her sister
Marina and her brother, the
appellant, also resided in the same
house. The family slept together in
the evenings in the sala of the
house while Marina slept in her
bedroom. At 11:00 p.m. on
February
19,
1997,
Lourdes
noticed that Lucelle was not at her
side. The appellant, who usually
also slept in the sala, was not there
either. Lourdes went to Marinas
bedroom and saw Lucelle in bed
(papag), covered with a blanket.
Beside her was the appellant who

was wearing a pair of short pants


and
undershirt.
When
the
appellant saw Lourdes, he slid
down from the bed, went under
the papag, and furtively left the
room. When Lourdes removed the
blanket, she saw Lucelle lying
sideways with her knees up to her
chin (nakabaluktot). Lucelle was
trembling with fear. When Lourdes
asked Lucelle what happened, she
did not respond.
Lourdes further testified that
at 9:00 p.m. on March 2, 1997, she
and her husband were having
dinner when she noticed that
Lucelle was nowhere to be found.
Momentarily, Lourdes saw the
appellant
emerge
from
the
bathroom. He was in his short
pants and his shirt was on his
shoulder.
He
was
perspiring
profusely.
Lourdes
was
flabbergasted
when
she
saw
Lucelle come out of the bathroom
after the appellant. Lucelle was
crying and looked pale. When
Lourdes asked Lucelle why she
was crying, she told her mother
that she had just urinated. The
appellant later told her sister
Lourdes that he did not do
anything to Lucelle.
Believing that the appellant
had been abusing their daughter,
Celso and Lourdes brought Lucelle
on March 5, 1997, to Barangay
Chairman Romeo Medina. Lucelle
told the barangay chairman that
the appellant sexually abused her.
Thereafter,
Lourdes
filed
a

complaint with the barangay


chairman against the appellant for
sexually molesting Lucelle.
Barangay Tanod Fernando
David testified that on March 6,
1997, the barangay chairman
ordered him and Barangay Tanod
Antonio Echavez to invite and
bring
the
appellant
to
the
barangay hall. The barangay
chairman asked the appellant if he
raped Lucelle and the latter
replied that he did. A Sinumpaang
Salaysay was prepared in the
Office of the Barangay Chairman
in which the appellant admitted
that he raped Lucelle in February
1997, and on March 2, 1997,
despite her resistance, and that he
threatened to kill her and her
family if she divulged the incidents
to her parents. The appellant
signed his statement in the
presence
of
the
barangay
chairman
and
the
barangay
tanods.
ISSUE:
Whether
or
not
the
statement made by the accused
before the barangay chairman
while not assisted by counsel is
admissible as evidence against
him.
HELD:
Although the appellant was
not assisted by counsel at the time
he gave his statement to the
barangay chairman and when he

signed the same, it is still


admissible in evidence against him
because he was not under arrest
nor under custodial investigation
when he gave his statement.
The exclusionary rule is
premised on the presumption that
the defendant is thrust into an
unfamiliar atmosphere and runs
through
menacing
police
interrogation procedures where
the potentiality for compulsion,
physical and psychological, is
forcefully apparent. As intended by
the
1971
Constitutional
Convention,
this
covers
investigation conducted by police
authorities which will include
investigations conducted by the
municipal police, the PC and the
NBI
and such other
police
agencies in our government. The
barangay chairman is not deemed
a law enforcement officer for
purposes of applying Section 12(1)
and (3) of Article III of the
Constitution.
Under
these
circumstances,
it
cannot
be
successfully claimed that the
appellants statement before the
barangay
chairman
is
inadmissible.

PEOPLE OF THE PHILIPPINES


vs. REY SUNGA

FACTS:

That on or about June 29,


1994 in the afternoon Rey Sunga,
Ramil Lansang, Inocencio Pascua,
Jr., and Lito Octac as principals,
and Locil Cui alias Ginalyn Cuyos
as accomplice by means of force,
violence and intimidation, to wit:
by pinning down one JOCELYN
TAN, a minor, fifteen (15) years of
age, succeeded in having carnal
knowledge of her against her will
and without her consent; that on
the occasion of said rape and to
enable them to conceal the
commission of the crime, the
herein accused in furtherance of
the conspiracy together with
LOCIL CUI, a minor, acting with
discernment and who cooperated
in the execution of the offense as
ACCOMPLICE, did then and there
willfully,
unlawfully
and
feloniously, taking advantage of
their
superior
number
and
strength, with intent to kill,
treacherously attack, assault, and
use
personal
violence
upon
JOCELYN TAN by repeatedly
stabbing and smashing a stone on
her head, thereby inflicting upon
her mortal wounds and multiple
fractures on her skull which were
the direct cause of her death
shortly thereafter.

Accused-appellant
Sunga,
who had previously been convicted
for robbery with homicide, denied
having anything to do with the
rape and killing of Jocelyn. He
branded as false the testimony of

Locil whom he claimed is a


prostitute and a pimp and was
always seen loitering at Mendoza
Park. Through a sworn statement,
he averred that: He, Octa and Jun
returned to Irawan, took Jocelyn's
corpse and dumped it at a coffee
plantation in Jacana Road; and that
he did not take part in the rape or
killing of Jocelyn but merely joined
the group due to Lansang's
promise to give him P500.00.

ISSUE:

Whether or not Sunga's


sworn statement is admissible as
evidence.

HELD:
Sunga having had no counsel
when he made his admission
before the NBI and his waiver of
the right to have one being invalid,
his
statementExhibit
I
is
inadmissible.
The testimony of Sunga
during
the
preliminary
investigation before the Municipal
Trial Court whereby he expressly
acknowledged having executed
Exhibit A and affirmed the
contents thereof did not render his
extrajudicial admission into a
judicial one which could be used
against him and his co-appellants.
Neither could his other statements

in such proceeding admitting his


participation in the crime be
utilized to establish his and the
other appellants guilt. For in that
preliminary investigation, Sunga
again was effectively denied of his
essential right to counsel. Atty.
Rocamora was appointed Sungas
counsel de officio but just like the
assistance he extended during the
execution of Exhibit A, Atty.
Rocamora utterly did nothing in
defense of Sungas cause. While
Sunga was being asked by the
judge a barrage of questions
calling for answers which could
and did incriminate him, Atty.
Rocamora did not offer the
slightest objection to shield his
client from the damning nature
thereof.
The right to counsel applies
in certain pretrial proceedings that
can be deemed critical stages in
the criminal process.1[85] The
preliminary investigation can be
no different from the in-custody
interrogations by the police, for a
suspect who takes part in a
preliminary investigation will be
subjected to no less than the
States
processes,
oftentimes
intimidating and relentless, of
pursuing those who might be liable
for criminal prosecution. In the
case at bar, Sunga was thrust into
the preliminary investigation and
while he did have a counsel, for
the latters lack of vigilance and
1

commitment to Sungas rights, he


was virtually denied his right to
counsel.
From the testimony of SPO2
Janoras, it can be gathered that
Atty. Rocamora (Sunga's counsel
during custodial investigation) did
not, if at all, fully apprise Sunga of
his rights and options prior to
giving his (Sunga's) admission.
Evidently, Atty. Rocamora, without
more, merely acted to facilitate the
taking of the admission from
Sunga.

Any information or admission


given by a person while in custody
which may appear harmless or
innocuous at the time without the
competent
assistance
of
an
independent counsel must be
struck down as inadmissible. Even
if the confession contains a grain
of truth or even if it had been
voluntarily given, if it was made
without the assistance of counsel,
it is inadmissible.

The right to counsel involves


more than just the presence of a
lawyer in the courtroom or the
mere propounding of standard
questions and objections; rather it
means an efficient and decisive
legal assistance and not a simple
perfunctory representation.

People vs. Tomaquin G.R. No.


133188 July 23, 2004

FACTS:

The accused-appellant was a


suspect in the death of Jaquelyn
Luchavez Tatoy who was stabbed
with a bladed instrument, hitting
her vital organs that led to her
eventual death. He was charged
with murder. On arraignment,
accused-appellant pleaded not
guilty to the charge, and trial
thereafter ensued. After trial,
accused was found guilty

There were no eyewitnesses


to
the
incident,
and
the
prosecutions evidence, aside from
appellants
extrajudicial
confession,
was
mainly
circumstantial.

Said extrajudicial confession


was given in the presence of a
barangay captain who is also a
lawyer. Appellant questions the
admissibility of the extrajudicial
confession because it was an
uncounselled confession. Accusedappellant
contends
that
the
barangay captain, although a
lawyer, may not be considered an
independent counsel within the
purview of Section 12, Article III of
the 1987 Constitution.

ISSUE:

Whether
or
not
the
extrajudicial confession executed
by appellant, with the assistance of
a barangay captain, is admissible
in evidence against him.

HELD:

No. It is not admissible.

When appellant intimated


that he was willing to confess and
requested the presence of Atty.
Parawan, SPO2 Monilar called up
Atty. Parawan and informed him of
appellants decision. Atty. Parawan
arrived at the Ramos Police
Station only at 2:00 in the
afternoon.2[27] By the time Atty.
Parawan arrived, the investigation
had already started and SPO2
Monilar had already asked and
elicited
information
from
appellant. Worse, Atty. Parawan
merely observed during the entire
investigation and failed to advise
or explain to appellant the
questions being propounded by
SPO2 Monilar. He did not even
bother to ask appellant if the
extrajudicial confession he was
about to execute was being
voluntarily given.
2

Moreover, that Atty. Parawan


is not an effective and vigilant
counsel is bolstered by his own
testimony
that
he
already
suspected appellant as having
committed the crime when the
latter was brought to his house by
the barangay tanods
The Court cannot imagine
how Atty. Parawan could have
effectively safeguarded appellants
rights as an accused during the
investigation when he himself
entertained the suspicion that
appellant is guilty of the crime
charged, and naturally, he would
want appellant to admit having
committed it.

Parawan who presumably


knows the intricacies of the law
and appellants predicament, Atty.
Parawan should have known better
and exercised his sound judgment
before conceding to appellants
choice. But it did not occur to him
to inhibit himself from acting as
appellants counsel and instead, he
even let appellant go through the
investigation and execute the
extrajudicial confession knowing
fully well that he was biased as
regards appellants innocence.

Any
person
under
investigation for the commission of
an offense shall have the right to
be informed of his right to remain
silent and to have competent and

independent counsel preferably of


his own choice. If the person
cannot afford the services of
counsel, he must be provided with
one. These rights cannot be waived
except in writing and in the
presence of counsel.

The words competent and


independent
counsel
in
the
constitutional provision is not an
empty rhetoric. It stresses the
need to accord the accused, under
the uniquely stressful conditions of
a
custodial
investigation,
an
informed judgment on the choices
explained to him by a diligent and
capable lawyer.

ZENON R. PEREZ vs. PEOPLE


OF THE PHILIPPINES and
SANDIGANBAYAN
FACTS:
On December 28, 1988, an
audit team headed by Auditor I
Arlene R. Mandin, Provincial
Auditors Office, Bohol, conducted
a cash examination on the account
of petitioner, who was then the
acting municipal treasurer of
Tubigon, Bohol.
The audit team embodied
their findings in the Report of Cash
Examination, which also contained
an inventory of cash items. Based
on the said audit, petitioner was

supposed to have on hand the total


amount of P94,116.36, instead of
the
P21,331.79,
incurring
a
shortage of P72,784.57. When
asked by the auditing team as to
the location of the missing funds,
petitioner verbally explained that
part of the money was used to pay
for the loan of his late brother,
another portion was spent for the
food of his family, and the rest for
his medicine.
An administrative case was
filed
against
petitioner
on
February 13, 1989. He filed an
Answer dated February 22, 1989
reiterating his
earlier verbal
admission before the audit team.
Later,
Petitioner
was
charged before the Sandiganbayan
with malversation of public funds,
defined and penalized by Article
217 of the Revised Penal Code.
Pre-trial was initially set on
June 4-5, 1990 but petitioners
counsel moved for postponement.
The
Sandiganbayan,
however,
proceeded to hear the case on June
5, 1990, as previously scheduled.
He denied the contents of his first
Answer to the administrative case
filed against him by the audit
team. He claimed it was prepared
without the assistance of counsel
and that at the time of its
preparation and submission, he
was not in peak mental and
physical condition, having been
stricken with diabetes mellitus.

He then revoked his Answer


dated February 22, 1989 and filed
his second Answer dated March 2,
1989. In the latter, he vehemently
denied that he incurred a cash
shortage P72,784.57.
ISSUE:
Whether or not petitioners
first Answer should not be given
probative weight because it was
executed without the assistance of
counsel.
HELD:
Petitioners first Answer may
be taken against him, as he
executed it in the course of the
administrative proceedings. This is
pursuant to Rule 130, Section 26
of the Rules of Court which
provides that the "act, declaration
or omission of a party as to a
relevant fact may be given against
him."
There
is
no
law,
jurisprudence
or
rule
which
mandates that an employee should
be assisted by counsel in an
administrative
case.
On
the
contrary, jurisprudence is in unison
in saying that assistance of counsel
is
not
indispensable
in
administrative proceedings.
The right to counsel, which
cannot be waived unless the
waiver is in writing and in the
presence of counsel, is a right
afforded a suspect or accused

during custodial investigation. It is


not an absolute right and may be
invoked or rejected in a criminal
proceeding and, with more reason,
in an administrative inquiry.
While
investigations
conducted by an administrative
body may at times be akin to a
criminal proceeding, the fact
remains that under existing laws, a
party in an administrative inquiry
may or may not be assisted by
counsel, irrespective of the nature
of the charges and of respondents
capacity to represent himself, and
no duty rests on such body to
furnish
the
person
being
investigated with counsel.
Thus, the right to counsel is
not imperative in administrative
investigations
because
such
inquiries are conducted merely to
determine whether there are facts
that merit disciplinary measures
against erring public officers and
employees, with the purpose of
maintaining
the
dignity
of
government service.

PEOPLE OF THE PHILIPPINES


vs. GODOFREDO B. ADOR and
DIOSDADO B. ADOR III
FACTS:
Accused-appellants
were
charged with the murder of
Absalon "Abe" S. Cuya III and
Rodolfo "Ompong" S. Chavez. Cuya

sustained five (5) gunshot wounds


while Chavez had three (3)
gunshot wounds.
From the evidence of the
prosecution, it appears that on
March 10, 1997, Beria and
several others heard gunshots and
thereafter ran to Chavez and saw
him already lying on the ground,
holding on to his intestines which
were starting to come out. Beria
shook Chavez and asked him what
had happened. Chavez replied
"tinambangan kami na Ador" ("We
were ambushed by the Adors") and
requested that he be brought to
the hospital as he was dying.
About eight (8) meters from where
Chavez was, in a dark spot, lay
"Abe" Cuya, dead.
Upon
learning
of
the
shooting incident, SPO1 Benjamin
Barbosa,
together
with
PO2
Alexander
Diaz,
immediately
proceeded to the crime scene to
conduct an investigation. That
same
evening,
upon
being
informed that the Adors had a
long-standing grudge against the
Cuyas, SPO1 Barbosa sought the
help of then Barangay Captain
Josue Perez to accompany him to
the residence of the Adors. They
spoke
with
their
patriarch,Diosdado Ador Sr. SPO1
Barbosa looked for the other male
members of the Ador family but
was told by Diosdado Sr. that they
were already asleep. Diosdado Sr.
nevertheless promised to present
them the following day.

The
following
morning,
March 11, 1997, Barangay Captain
Perez accompanied the Adors,
namely, Diosdado Sr., Diosdado III,
Godofredo, Rosalino, Allan and
Reynaldo, to SPO1 Barbosa at the
PNP Central Police Headquarters.
The Adors were informed of their
constitutional rights to remain
silent and to choose their own
counsel. They were then brought
to the PNP Crime Laboratory at
the Provincial Headquarters and
subjected to paraffin tests. On the
way to the crime laboratory,
Godofredo told his police escort
that he had been entrusted with a
handgun which he kept in his
residence. The officers retrieved
the gun from under a fallen
coconut trunk and turned it in to
the latter.
ISSUE:
Whether or not trial court
erred in admitting in evidence
those taken against them in
violation of their constitutional
rights to counsel during custodial
investigation.

HELD:
In fine, the admissions made
by Godofredo including the gun in
question cannot be considered in
evidence against him without
violating his constitutional right to
counsel. Godofredo was already
under custodial investigation when
he made his admissions and

surrendered the gun to the police


authorities. The police had already
begun to focus on the Adors and
were carrying out a process of
interrogations that was lending
itself to eliciting incriminating
statements and evidence: the
police went to the Ador residence
that same evening upon being
informed that the Adors had a
long-standing grudge against the
Cuyas; the following day, all the
male members of the Ador family
were told to go to the police
station; the police was also
informed of the dying declaration
of deceased Chavez pointing to the
Adors as the assailants; the Adors
were all subjected to paraffin
examination; and, there were no
other suspects as the police was
not considering any other person
or
group
of
persons.
The
investigation thus was no longer a
general inquiry into an unsolved
crime as the Adors were already
being held as suspects for the
killings of Cuya and Chavez.
Consequently, the rights of a
person
under
custodial
investigation, including the right
to counsel, have already attached
to the Adors, and pursuant to Art.
III, Sec. 12(1) and (3), 1987
Constitution, any waiver of these
rights should be in writing and
undertaken with the assistance of
counsel.
Admissions
under
custodial
investigation
made
without the assistance of counsel
are barred as evidence. The
records are bare of any indication

that the accused have waived their


right to counsel, hence, any of
their admissions are inadmissible
in evidence against them. As we
have held, a suspects confession,
whether verbal or non-verbal,
when taken without the assistance
of counsel without a valid waiver
of such assistance regardless of
the absence of such coercion, or
the fact that it had been
voluntarily given, is inadmissible in
evidence, even if such confession
were gospel truth. Thus, in Aballe
v. People, the death weapon, a
four-inch kitchen knife, which was
found after the accused brought
the police to his house and pointed
to them the pot where he had
concealed it, was barred from
admission as it was discovered as
a consequence of an uncounseled
extrajudicial confession.

PEOPLE vs BAGNATE, 428


SCRA 633

FACTS:

Accused Amado Bagnate was


turned over to SPO2 Ambion for
custodial investigation. Without
asking the name of appellant,
Ambion informed him in the Bicol
dialect of his right to remain silent,
to be assisted by counsel, that
whatever he would say may be
used against or in his favor, and

that he cannot be tortured or


molested. Asked if he was willing
to cooperate, the accused assented
and gave his name. Appellant told
that he was willing to confess and
he was again informed of his
rights. Ambion offered the services
of Atty. Paterno Brotamonte, which
appellant accepted.

Ambion
left
to
fetch
Brotamonte whose office was
located several meters away from
the police station. The lawyer told
Ambion that he will just follow as
he was having his office blessing at
that time. After some time,
Brotamonte arrived at the police
station. Before proceeding with
the investigation, he asked the
policemen
to
leave
the
investigation room and conferred
with appellant. He introduced
himself to appellant and informed
him of his rights. He also asked
and examined appellant to see if
he was physically harmed by the
policemen
and
found
none.
Appellant told the lawyer that he
was willing to give a statement.
The
investigation
was
then
conducted in the Bicol dialect, with
Ambion asking the questions. It
was translated thereafter into
English
with
the
help
of
Brotamonte, for the purpose of
putting it into writing. After typing
the first page of the confession, he
translated and explained the
contents thereof to appellant, then
he and appellant signed thereon.

Appellant
was
brought
before Judge Base the next day,
who subsequently examined the
voluntariness and veracity of the
confession. An information was
thereafter filed charging appellant
with the crimes of murder and
rape with homicide. The trial court
found the accused guilty beyond
reasonable
doubt.
ISSUE:

Whether
or
not
the
extrajudicial confession by the
accused was made voluntarily and
in the presence of a competent and
independent counsel and should
therefore render it admissible.

HELD:

The extrajudicial confession


made by the accused is admissible
as evidence.

Before the onset of the


investigation, Atty. Brotamonte
privately conferred with appellant
to ascertain the voluntariness of
his confession and to make sure
that no force or duress was
employed by the police authorities
on the latter to make him admit
the crimes charged. He informed
appellant of his constitutional

rights and was clear in explaining


to him the questions propounded
by SPO2 Ambion. The testimony of
Atty. Brotamonte during crossexamination leaves no room for
doubt that he adequately assisted
appellant during the investigation.

Appellant
signed
the
confession with the assistance of a
competent
and
independent
counsel, Brotamonte, and it was
also sworn to by him before Judge
Arsenio Base of the Municipal Trial
Court who, before administering
the oath to appellant, conferred
with him and informed him of his
rights and the consequences of his
confession.

To be considered competent
and independent for the purpose of
assisting an accused during a
custodial investigation, it is only
required for a lawyer to be willing
to
fully
safeguard
the
constitutional
rights
of
the
accused, as distinguished from one
who would merely be giving a
routine,
peremptory
and
meaningless
recital
of
the
individual's constitutional rights.

To be admissible in evidence,
an extrajudicial confession must be
express and voluntarily executed
in writing with the assistance of an
independent
and
competent
counsel, and a person under

custodial investigation must be


continuously assisted by counsel
from the very start thereof. The
presence of counsel is intended to
secure the voluntariness of the
extrajudicial confession, and the
assistance
given
must
be
independent and competent, that
is, providing full protection to the
constitutional
rights
of
the
accused.

Right to Speedy Trial; Speedy


Disposition of cases

RENE CABARLES vs. HON.


JUDGE BONIFACIO SANZ
MACEDA

FACTS:

On June 18, 1999, Cabarles


was charged with murder by
stabbing with a deadly weapon
(fan knife) one Antonio Callosa,
which directly caused his death.

Cabarles pleaded not guilty.


The trial court scheduled the case
for hearing on the following dates,
to wit: pre-trial on November 22,
2000;
presentation
of
prosecutions evidence on April 18,

May 4, 11, 18, and 23, 2001; and


presentation of defense evidence
on June 20 and 27, July 4 and 18,
and August 1, 2001.

Through no fault of its own,


the prosecution was unable to
present its evidence on the first
four hearing dates. Instead, trial
on the merits began only on May
23, 2001. The second prosecution
witness was presented in court
during the June 20, 2001 hearing.
In the June 20, 2001 hearing, the
prosecution said it would offer its
evidence and rest its case should
the People fail to present a witness
at the next scheduled hearing.
When the case was called on June
27, 2001, the prosecution failed to
present a witness. Judge Maceda
gave the prosecution a last chance
but warned.

With no witness for the


August 1, 2001 hearing, the
prosecution rested its case and
formally offered its evidence.
Thereafter, Cabarles, with leave of
court, filed a demurrer to evidence
but it was denied by Judge
Maceda.

A day before the scheduled


promulgation of judgment on April
2, 2003, Judge Maceda motu
proprio issued the questioned
order reopening the case. In it, he
observed that the prosecution may

not have been given its day in


court resulting in a miscarriage of
justice.
Judge Maceda denied Cabarless
motion for reconsideration in an
Order dated April 25, 2003 and set
the case for hearing on May 8,
2003 to hear the testimonies of
Pedrosa and Dr. Salen. The
prosecution still failed to present a
witness during the May 8, 2003
hearing.
Nonetheless,
Judge
Maceda,
upon motion,
again
decided
to
extend
to
the
prosecution another chance, giving
the People June 19 and July 3,
2003 as additional hearing dates.
Finally, on June 19, 2003,
one Pedrosa took the witness stand
and
completed
her
direct
examination.
A
few
days
thereafter, Cabarles filed the
present petition questioning Judge
Macedas order, alleging that it
was issued with grave abuse of
discretion.
On July 24, 2003, the
defense counsel agreed on the
facts contained in the death
certificate of the victim, so the
testimony of Dr. Salen was
dispensed with. Thereafter, Judge
Maceda set the date for the
reception of evidence on the civil
aspect of the criminal case on
August 14, 2003, when Carlos, the
deceaseds brother, was recalled to
the witness stand.
Cabarles was then given a
chance to adduce further evidence

on his behalf. On August 9, 2004,


Judge
Maceda
deferred
the
promulgation of judgment and
ordered the case archived pending
this Courts resolution of the case.
ISSUE:
Whether or not petitioners
right to speedy disposition of his
case was violated
HELD:
Petitioners right to speedy
disposition of his case was violated
In this particular case, the
prosecution was given ample
opportunity to present all its
witnesses but it failed to do so. The
failure of the prosecution to take
full advantage of the opportunities
given does not change the fact that
it
was
accorded
such
opportunities. Contrary to the
justification stated in the April 1,
2003 Order, the prosecution was
not deprived of its day in court.
The
presence
of
prosecution
witnesses
in
court
is
the
responsibility
of
the
public
prosecutor and it is incumbent
upon him to take the initiative of
ensuring the attendance of his
witnesses at the trial.
On Cabarless right to a
speedy disposition of his case, we
agree that under the Constitution,
all persons shall have the right to a
speedy disposition of their cases.
Nowhere is this guaranty more

significant and meaningful than in


criminal cases where not only the
fortune, but the life and liberty of
the accused as well, are at stake.
The right to a speedy disposition of
a case, like the right to speedy
trial, is deemed violated only when
the proceeding is attended by
vexatious,
capricious,
and
oppressive
delays;
or
when
unjustified postponements of the
trial are asked for and secured; or
when without cause or justifiable
motive, a long period of time is
allowed to elapse without the party
having his case tried.

GUILLERMO T. DOMONDON
versus HON. FIRST DIVISION,
SANDIGANBAYAN
FACTS:
This is a special civil action
for certiorari under Rule 65 of the
Rules of Court seeking to nullify
the
September
13,
2004
Resolution of the Sandiganbayan
denying petitioners motion to
dismiss and its January 11, 2005
Resolution denying the motion for
reconsideration.
The case arose from the
investigation initiated by a lettercomplaint of then Police Sr.
Superintendent Romeo M. Acop to
the Ombudsman where it appears
that payrolls of 2,000 enlisted men
of
the
Cordillera
Regional
Command (CRECOM), who were

allegedly
recipients
of
the
P20,000,000
appropriated
for
combat, clothing, and individual
equipment (CCIE) allowance, were
falsified.
Subsequent
investigations
determined
that
petitioners
Philippine National Police (PNP)
Director
for
Comptrollership
Guillermo Domondon, and Sr.
Superintendent
Van
Luspo,
together with other PNP officers,
conspired with one another in
approving
without
budgetary
basis, the release of Advice
Allotment SN No. 4363 and Advice
Allotment SN No. 4400 for the
procurement of CCIE for the use of
PNP personnel of the CRECOM,
causing to be issued checks with
an
aggregate
amount
of
P20,000,000 for payment of ghost
purchases of the aforesaid CCIE
items.
On
May
4,
1994,
an
information was filed before the
Sandiganbayan
charging
petitioners Domondon and Luspo,
and the above-named accused,
with violation of Section 3(e) of the
Anti-Graft and Corrupt Practices
Act.
Their arraignment was reset
for
several
times,
hence,
petitioners filed on December 3,
2003 a motion to dismiss claiming
that the failure to arraign them
within the period set under
Republic Act (RA) No. 8493 or the
Speedy Trial Act of 1998 have

resulted in denial of their rights to


speedy trial.
On September 13, 2004, the
Sandiganbayan denied petitioners
motion to dismiss and on January
11, 2005, dismissed petitioners
motion for reconsideration.
ISSUE:
Whether or not the failure to
arraign appellants within the time
set by the Speedy Trial Act of 1998
constitutes a violation of their
right to a speedy trial.
HELD:
No violation of their right to
a speedy trial.
Although
the
scheduled
arraignments
were
postponed
several times, they were however
postponed for valid reasons. One
of the postponements was due to
the request of one of the accused
to reset the arraignment since the
counsel of record is not available
on the scheduled date. To proceed
with the arraignment despite the
noted absence of one of the
counsels would result in inequity
on one of the accused-movants codefendants.
Another
postponement, as pointed out by
the accused-movants, was the time
given by the Court to allow the
prosecutor to file an opposition to
Brizuelas Bill of Particulars. The
comment
made
by
accusedmovants is discriminatory and

unjust. They claim that the delay


caused by the filing of a motion for
bill of particulars by a co-accused
should not be attributable to them
as they did not join the same, and
consequently such is a violation of
their right to speedy trial. They
have
forgotten
that
they
themselves had caused a long
delay in this case by filing a motion
for reinvestigation and the petition
for certiorari and prohibition with
the Honorable Supreme Court,
which is, if such reasoning is to be
followed, to the detriment of the
other accused in this case.

The right to a speedy


disposition of a case, like the right
to speedy trial, is deemed violated
only when the proceeding is
attended by vexatious, capricious,
and oppressive delays; or when
unjustified postponements of the
trial are asked for and secured, or
when without cause or justifiable
motive a long period of time is
allowed to elapse without the party
having his case tried.
Equally applicable is the
balancing test used to determine
whether a defendant has been
denied his right to a speedy trial,
or a speedy disposition of a case
for that matter, in which the
conduct of both the prosecution
and the defendant are weighed,
and such factors as length of the
delay, reason for the delay, the
defendants assertion or nonassertion
of
his
right,
and

prejudice
to
resulting from
considered.

the
the

defendant
delay, are

It must be recalled that in


the
application
of
the
constitutional guaranty of the right
to speedy trial, particular regard
must also be taken of the facts and
circumstances peculiar to each
case.

ASSISTANT SPECIAL
PROSECUTOR III ROHERMIA J.
JAMSANI-RODRIGUEZ versus
JUSTICES GREGORY S. ONG,
JOSE R. HERNANDEZ, and
RODOLFO A. PONFERRADA,
SANDIGANBAYAN
FACTS:
The Revised Internal Rules
of the Sandiganbayan provides
that cases originating from Luzon,
Visayas and Mindanao shall be
heard in the region of origin,
except only when the greater
convenience of the parties and of
the witnesses or other compelling
considerations
require
the
contrary.
Thus, for the period from
April 24 to April 28, 2006, the
Fourth Division scheduled sessions
for the trial of several cases in the
Hall of Justice in Davao City.
Prior
to
the
scheduled
sessions, or on April 17, 2006, the

complainant sent a memorandum


to Special Prosecutor Dennis M.
Villa-Ignacio (Special Prosecutor
Villa-Ignacio) to invite his attention
to the irregular arrangement being
adopted by the Fourth Division in
conducting its provincial hearings.
The memorandum stated among
others that:
-The Fourth Division has
adopted a different procedure.
They do not sit as collegial body,
instead they divide the division
into two. In such a manner, the
Chairman will hear some of the
cases
alone
and
the
other
members will hear other cases,
conducting hearing separately and
simultaneously.

hearing cases
chamber.

in

the

other

ISSUE:
Whether
or
not
the
insistence
of
the
respondent
judges that they adopted the
procedure in order to expedite the
hearing of provincial cases is
sufficient
reason
to
entirely
exonerate them absent malice or
corruption on the adoption of the
said procedure.
HELD:

-We find this procedure to be


advantageous to the Prosecution
and also commendable on the part
of the Justices. While there are no
objections manifested by the
defense
lawyers,
we
are
apprehensive of the consequences,
considering that this constitutes
procedural lapses.

PD 1606, as amended, and


the Revised Internal Rules of the
Sandiganbayan, supra, call for the
actual presence of the three
Justices composing the Division to
constitute a quorum to conduct
business
and
to
hold
trial
proceedings. Respondent Justices
cannot lightly regard the legal
requirement for all of them to sit
together as members of the Fourth
Division
in
the
trial
and
determination of a case or cases
assigned thereto.

-In one of her hearings, the


undersigned has already called the
attention of the Hon. Chairman
and expresses (sic) her concern on
the matter, and even opined that
they
might
be
charged
of
falsification, by issuing orders that
they heard the cases as a collegial
body, when in fact only the
Chairman was present during the
trial and the other members are

Respondent
Justices
adoption
of
the
irregular
procedure cannot be dismissed as
a mere deficiency in prudence or
as a lapse in judgment on their
part, but should be treated as
simple misconduct, which is to be
distinguished from either gross
misconduct or gross ignorance of
the law. The respondent Justices
were
not
liable
for
gross

misconduct
defined
as
the
transgression of some established
or definite rule of action, more
particularly, unlawful behavior or
gross negligence, or the corrupt or
persistent violation of the law or
disregard of well-known legal
rules
considering
that
the
explanations they have offered
herein, which the complainant did
not refute, revealed that they
strove to maintain their collegiality
by holding their separate hearings
within sight and hearing distance
of one another. Neither were they
liable for gross ignorance of the
law, which must be based on
reliable evidence to show that the
act
complained
of
was
illmotivated, corrupt, or inspired by
an intention to violate the law, or
in persistent disregard of wellknown legal rules; on the contrary,
none of these circumstances was
attendant
herein,
for
the
respondent
Justices
have
convincingly shown that they had
not been ill-motivated or inspired
by an intention to violate any law
or legal rule in adopting the
erroneous procedure, but had been
seeking,
instead,
to
thereby
expedite their disposition of cases
in the provinces.
Nonetheless, it remains that
the respondent Justices did not
ensure that their proceedings
accorded with the provisions of the
law
and
procedure.
Their
insistence that they adopted the
procedure in order to expedite the
hearing of provincial cases is not a

sufficient
reason
to
entirely
exonerate them, even if no malice
or corruption motivated their
adoption of the procedure. They
could
have seen
that
their
procedure was flawed, and that
the flaw would prevent, not
promote,
the
expeditious
disposition of the cases by
precluding their valid adjudication
due to the nullifying taint of the
irregularity. They knew as well
that the need to expedite their
cases, albeit recommended, was
not the chief objective of judicial
trials.
Although
a
speedy
determination of an action or
proceeding implies a speedy trial,
it should be borne in mind that
speed is not the chief objective of a
trial.
Careful
and
deliberate
consideration
for
the
administration of justice is more
important than a race to end the
trial. A genuine respect for the
rights of all parties, thoughtful
consideration before ruling on
important questions, and a zealous
regard for the just administration
of law are some of the qualities of
a good trial judge, which are more
important than a reputation for
hasty disposal of cases (State
Prosecutors v. Muro).

EVELYN V. RODRIGUEZ, AND


ANDRES ABONITA, JR. vs.

SANDIGANBAYAN, AND
PEOPLE OF THE PHILIPPINES

FACTS:

Herein
petitioner
Mayor
Rodriguez was alleged to have
been involved in illegal logging
activities, cutting and sawing of
lumbers without proper permit or
license.

On September 24, 1996,


pursuant to an information on an
undergoing illegal logging activity
which was alleged to be under the
order of Rodriguez, lumbers were
confiscated
for
safekeeping.
Subsequently, upon the orders of
Mayor
Rodriguez,
Barangay
Captain Abonita accompanied by
two fully armed policemen who
then and there forcibly took
possession,
hauled,
and
transferred the lumber to the
Municipal Hall of Taytay.

On November 7, 1996, a
complaint was filed for robbery
and violation of Section 1(b), P.D.
No. 1829 (DECREE PENALIZING
OBSTRUCTION
OF
APPREHENSION
AND
PROSECUTION OF CRIMINAL
OFFENDERS) against petitioners
Mayor Rodriguez and Barangay
Captain
Abonita
before
the
Provincial Prosecution Office of
Palawan.
On February 18, 1997, the
Deputized
Ombudsman
Investigator recommended the
filing of an information against
petitioners and the forwarding of
the records of the case to the
Office of the Ombudsman-Luzon
for review.
Following its review of the
case, the Office of the Deputy
Ombudsman-Luzon, by a Joint
Review Action of October 19,
1998, resolved to, as it did file an
information for violation of Section
1(b) P.D. 1829 on December 8,
1998 against petitioners before the
Sandiganbayan.
A warrant of arrest was
accordingly
issued
against
petitioners on December 14, 1998.
On
January
27,
1999,
petitioners filed a Motion to Defer
Arraignment, they having filed on
even date a Motion to Quash. By
Order of January 29, 1999, the
Sandiganbayan
reset
the
arraignment to February 26, 1999.

During
the
scheduled
arraignment on February 26, 1999,
the special prosecutor moved to
defer
the
arraignment
as
recommended changes in the
information were not yet acted
upon by the Ombudsman. Without
objection from petitioners counsel,
the arraignment was reset to April
8, 1999.
In the meantime, the special
prosecutor filed on April 6, 1999
an opposition to petitioners Motion
to Quash.
Subsequently,
the
Sandiganbayan, acting upon a
Motion to Admit Information which
was filed by the special prosecutor,
admitted the amended information
by Order of April 8, 1999.
Petitioners filed on April 26,
1999 a Motion to Quash the
amended information, to which
motion the special prosecutor filed
a comment/opposition on June 9,
1999, explaining that the belated
filing thereof was due to the
transfer of the records of the
Office of the Special Prosecutor to
its
new
office
at
the
Sandiganbayan
Centennial
Building in Quezon City.
Thereafter or on June 28,
1999, the special prosecutor filed
another Ex-parte Motion to Admit
Amended Information which was
set for hearing on November 25,
1999. The scheduled hearing on
November 25, 1999 was, however,

cancelled and reset to December


3, 1999 upon urgent motion by
petitioners counsel upon the
ground that on said date, he
needed to appear before the
Metropolitan
Trial
Court
of
Mandaluyong.
By Order of December 3,
1999, the Sandiganbayan granted
the motion to admit amended
information, denied the motion to
quash the amended information,
and ordered the arraignment of
petitioners on January 17, 2000.
On
January
14,
2000,
petitioners filed a Motion to
Quash/Dismiss
the
second
amended information.
During
the
scheduled
arraignment of petitioners on
January
17,
2000,
the
Sandiganbayan issued in open
court the assailed separate orders
denying petitioners motion to
quash
the
second
amended
information, denying the motion to
defer arraignment, and entering a
plea of not guilty for both accused
herein petitioners, which orders
petitioners
allege
have
been
rendered with grave abuse of
discretion.
ISSUES:
a. Whether or not the pendency
of
the
preliminary
investigation of the case
which dragged for almost
three years is unreasonable

or unjustifiable and violates


their constitutional rights as
accused to due process.
b. Whether or not the repeated
and ex-parte amendment of
the
information
by
the
Ombudsman
resulted
to
inordinate delay in bringing
the case to trial, which is a
ground for dismissal of the
information under Section
13, in relation to Section 7 of
R.A. 8493 (The Speedy Trial
Act of 1998).
HELD:
a. No. It is gathered that the
Provincial
Prosecutor
of
Palawan took only three
months, from November 7,
1996 to February 18, 1997,
to
come
up
with
its
resolution finding probable
cause against petitioners.
The Deputy Ombudsman for
Luzon took eight months to
review the case and come up
with the joint review action
on October 19, 1998. On the
other hand, the Office of the
Ombudsman acted on the
case for around two months.
Considering that the records
were passed upon by three
offices,
the
period
of
preliminary
investigation,
which did not exceed two
years, cannot be deemed to
have
violated
petitioners
constitutionally guaranteed
rights to procedural due

process and to a
disposition of cases.

speedy

b. No. The right to a speedy


disposition of cases, like the
right to a speedy trial, is
deemed violated only when
the
proceedings
[are]
attended
by
vexatious,
capricious, and oppressive
delays;
or
when
the
unjustified postponements of
the trial are asked for and
secured, or when without
cause or unjustifiable motive,
a long period of time is
allowed to elapse without the
party having his case tried.
In the determination of
whether or not the right has
been violated, the factors
that maybe considered and
balanced are: the length of
the delay, the reasons for
such delay, the assertion or
failure to assert such right
by the accused, and the
prejudice caused by the
delay.
Parenthetically,
as
reflected in the following
observation
of
the
Sandiganbayan, petitioners
themselves contributed to
the delay. The delay is
caused not unreasonably but
because of the exercise of
the right of the accused to
determine whether or not
they could be charged under

the Information for which


they have filed Motions to
Quash. Furthermore, the
records show that it was on
account
of
petitioners
continuous filing of motions
that the arraignment was
also deferred.

AURORA E. BALAJEDEONG vs.


JUDGE DEOGRACIAS K. DEL
ROSARIO, MCTC, PATNONGON,
ANTIQUE

FACTS:
This is an administrative
complaint filed by Aurora E.
Balajedeong (Balajedeong), against
Judge Deogracias K. Del Rosario
(Judge Del Rosario), Presiding
Judge of the Municipal Circuit
Trial Court (MCTC), Patnongon,
Antique, for Grave Misconduct;
Conduct Unbecoming a Judge, and
Delay in the Disposition of a Case,
relative to Civil Case No. 367
entitled, "Paterno Colago v. Sps.
Willy and Salvacion Odi," pending
before said court.
Complainant Balajedeong is
the attorney-in-fact of Paterno
Colago, the plaintiff in Civil Case
No. 367, filed against the Spouses
Odi for Forcible Entry with Prayer
for Issuance of a Temporary
Restraining Order before the
MCTC,
Patnongon,
Antique,
presided over by respondent Judge

Del Rosario. She narrated that


after a preliminary conference was
held on 12 May 2003, the parties
were ordered by respondent Judge
Del Rosario to submit their
respective position papers within
10 days. Colago, through his
representative
and
herein
complainant
Balajedeong,
allegedly filed his position paper
on 24 June 2003, while Spouses
Odi failed to do so. On 13 February
2004, Colagos counsel filed a
Motion for Early Decision, but
despite said motion, respondent
Judge
Del
Rosario
never
entertained his plea.
In his Comment dated 26
July 2006, respondent Judge Del
Rosario claims that Spouses Odi
submitted their memorandum on 2
June 2003, while Colago through
his representative and herein
complainant
Balajedeong,
submitted his position paper on 30
June 2003. Respondent Judge Del
Rosario admits that the delay in
the disposition of the subject case
is due mainly to his failing health
as he claims that sometime in July
2003 and September 2003, he had
been hospitalized due to heart
ailment and was advised to
undergo by-pass operation.
Thereafter,
he
was
hospitalized several times more.
Respondent Judge Del Rosario
further states that there was a
time when he was assigned as
Presiding Judge of the 4th MCTC,
Barbaza,
Antique
where
he

reported twice a week to conduct


trial and preliminary examination.
Respondent Judge Del Rosario
further informs this Court that
Civil Case No. 367, subject matter
of this instant administrative
complaint, was already decided on
15 June 2006.
On 24 November 2006, the
Office of the Court Administrator
(OCA)
submitted
its
report,
recommending that the instant
administrative complaint be REDOCKETED
as
a
regular
administrative matter.
On 15 January 2007, we
required the parties herein to
manifest within 10 days from
notice if they were willing to
submit the matter for resolution
based on the pleadings filed.
On
16
February
2007,
complainant
Balajedeong
submitted
her
manifestation
stating that she was submitting the
case for resolution based on the
pleadings filed.
Respondent
Judge
Del
Rosario
failed
to
file
his
manifestation despite notice sent
to and received by him.
Resultantly, the case is
submitted for decision based on
the pleadings filed.
ISSUE:

Whether
or
not
the
respondents failure to observe
Speedy Trial within the period
prescribed by law is warranted on
account of his poor health.
HELD:
No. As a general principle,
rules prescribing the time within
which certain acts must be done,
or certain proceedings taken, are
considered
absolutely
indispensable to the prevention of
needless delays and the orderly
and speedy discharge of judicial
business. By their very nature,
these rules are regarded as
mandatory.
Indeed, we have consistently
impressed upon judges the need to
decide
cases
promptly
and
expeditiously on the principle that
justice delayed is justice denied.
Failure to resolve cases submitted
for decision within the period fixed
by law constitutes a serious
violation of the constitutional right
of the parties to a speedy
disposition of their cases.
Delay in case disposition is a
major culprit in the erosion of
public faith and confidence in the
judiciary and the lowering of its
standards. Failure to decide cases
within the reglementary period,
without strong and justifiable
reason,
constitutes
gross
inefficiency
warranting
the
imposition
of
administrative
sanction on the defaulting judge.

Respondent
Judge
Del
Rosario ascribes the delay in the
resolution of Civil Case No. 367 to
his failing health, as he was
hospitalized several times due to
heart ailment. Even if he was
stricken by an illness which
hampered the due performance of
his duties, still it was incumbent
upon
respondent
Judge
Del
Rosario to inform this Court of his
inability to seasonably decide the
cases assigned to him. His illness
should not be an excuse for his
failure
to
render
the
corresponding
decision
or
resolution within the prescribed
period. While we sympathize with
his woes, the demands of public
service cannot abide by his illness.
In case of poor health, the Judge
concerned needs only to ask this
Court for an extension of time to
decide cases, as soon as it
becomes clear to him that there
would be delay in his disposition of
his cases.

EMIL J. BIGGEL versus JUDGE


FERNANDO VIL. PAMINTUAN
FACTS:
In a verified Complaint dated
5 September 2006, complainant
narrated that after the complaint
for estafa had been filed against
him, the assistant city prosecutor
issued a resolution, subsequently
approved by the city prosecutor,
recommending the filing of an

information in court with a


recommended bail of P60,000.00.
Complainant stated that this was
made without the benefit of a
preliminary investigation and a
subpoena sent to him as the
Assistant City.
Thus, complainants counsel
filed a motion for reinvestigation
before the sala of respondent
judge which was set for hearing on
13 January 2006. On 9 January
2006, respondent judge issued an
order directing Public Prosecutor
Raymond Tabangin to file his
comment on the motion. He also
rescheduled the hearing of the
motion to 25 January 2006. On the
rescheduled date of hearing, in
view
of
Public
Prosecutor
Tabangins
failure
to
file
a
comment, respondent judge reset
the hearing to 1 March 2006 as
well as gave Public Prosecutor
Tagudar, who was new in the case,
time to file her comment.
On
21
February
2006,
complainants
counsel
filed
a
manifestation and motion praying
that his motion for reinvestigation
be
deemed
submitted
for
resolution as Public Prosecutor
Tagudar failed to file the required
comment. On 1 March 2006,
complainants
counsel
again
manifested
that
since
the
prosecution had failed to file its
comment, the motion should be
deemed submitted for resolution.
The parties were then brought
inside the judges chambers and

upon respondent judges prodding,


complainants counsel agreed to
the request of Public Prosecutor
Tabangin, who had reappeared in
the case, that he be given another
period of five (5) days or until 6
March 2006 to file his comment.
Complainant was likewise given
the same period of time to file his
reply upon receipt of the comment.
Immediately thereafter, on 7
March 2006, respondent judge
issued an order denying the
motion for reinvestigation without
awaiting complainants reply to the
comment. In view of this incident,
on 23 March 2006, complainant
filed a motion for inhibition and
motion for reconsideration of the
order denying the motion for
reinvestigation. On 17 April 2006,
respondent judge denied the
motion for his inhibition and
directed
Public
Prosecutor
Tabangin to file his comment to the
motion for reconsideration.
On
11
May
2006,
complainant filed a manifestation
and motion praying that his motion
for reconsideration be deemed
submitted for resolution in view of
the failure of the public prosecutor
to file his comment. On 26 May
2006, complainant filed a motion
for early resolution of his motion
for
reconsideration.
However,
despite several inquiries into the
status of said motion, the motion
remained unresolved, for which
reason complainant filed on 19
June 2006 a motion reminding the

court
that
his
motion
for
reconsideration had not been
acted upon.
On
24
July
2006,
complainant filed a motion to lift
the hold departure order/resolve
the motion for reconsideration. On
26 July 2006, complainant received
a copy of the Order dated 14 July
2006 granting his motion for
reconsideration and directing the
public prosecutor to conduct the
reinvestigation within thirty (30)
days.
On 6 March 2006, Public
Prosecutor Tabangin filed his
comment
which
complainant
received on 10 March 2006. In
said
comment,
the
public
prosecutor contended that there
was no legal infirmity in the
certification issued by Assistant
City Prosecutor Centeno which
stated
that
the
accussed
(complainant in this administrative
matter) not (being) a permanent
resident of the Philippines tends to
indicate that the address so given
was only a temporary one and that
therefore with that basis it could
be assumed that accused cannot
be
subpoenaed.
The
public
prosecutor likewise contended that
the certification issued by the
Assistant City Prosecutor that a
preliminary investigation had been
conducted should be presumed
correct pursuant to the wellentrenched
presumption
of
regularity in the performance of
official duties.

On
4
August
2006,
complainant received the Public
Prosecutors
comment
on
his
motion to lift the hold departure
order, complainants reply to which
was filed on 16 August 2006. On
29 August 2006, complainant filed
a motion to resolve to no avail.

granted the reinvestigation only on


26 July 2006clearly beyond the
mandated
period.
Notably,
respondent judge not only delayed
the submission for resolution of
the motion for reinvestigation but
also delayed the submission of the
motion for reconsideration for
resolution.

ISSUE:
Whether or not the acts of
the judge constitutes undue delay
thereby violating the right to
speedy trial.
HELD:
In
the
instant
case,
complainant filed an urgent motion
for reinvestigation on 5 January
2006. Respondent judge issued an
Order dated 9 January 2006
directing
Public
Prosecutor
Tabangin to file a comment within
ten (10) days from receipt of the
motion. As the public prosecutor
failed
to
file
a
comment,
respondent judge reset the hearing
to 1 March 2006 instead of
submitting
the
motion
for
resolution. On 7 March 2006,
respondent judge denied the
urgent motion for reinvestigation.
Hence, complainant moved for
reconsideration on 23 March 2006.
Respondent judge then directed
the public prosecutor to file a
comment on said motion. Despite
the public prosecutors failure to
file the required comment and
complainants several motions for
resolution,
respondent
judge

Undue delay in the disposition of


cases and motions erodes the faith
and confidence of the people in the
judiciary
and
unnecessarily
blemishes its stature. No less than
the Constitution mandates that
lower courts must dispose of their
cases promptly and decide them
within three months from the filing
of the last pleading, brief or
memorandum required by the
Rules of Court or by the Court
concerned.
The requirement that cases
be
decided
within
the
reglementary period is designed to
prevent delay in the administration
of justice, for obviously, justice
delayed is justice denied. An
unwarranted slow down in the
disposition of cases erodes the
faith and confidence of our people
in
the
judiciary,
lowers
its
standards and brings it into
disrepute.

FEDERICO MIGUEL OLBES vs.


HON. DANILO A. BUEMIO

FACTS:

On complaint of Samir and


Rowena Muhsen, Federico Miguel
Olbes (petitioner) was indicted for
Grave
Coercion
before
the
Metropolitan Trial Court (MeTC) of
Manila by Information dated June
28, 2002.
Denying petitioners motion
to
defer
or
suspend
his
arraignment in light of his pending
petition for review before the
Department of Justice from the
City Fiscals Resolution finding
probable cause to hale him into
court, Judge Hipolito dela Vega
proceeded
with
petitioners
arraignment on February 12, 2003
in which he pleaded not guilty to
the
charge.
Pre-trial
was
thereupon set to May 28, 2003
which was, however, declared a
non-working day due to the
occurrence of typhoon "Chedeng."
The pre-trial was thus reset to
October 23, 2003.
At the scheduled pre-trial on
October 23, 2003, petitioner failed
to appear, prompting the trial
court to issue a warrant for his
arrest,
which
warrant
was,
however,
later
recalled
on
discovery that neither petitioner
nor his counsel was notified of said
schedule. Pre-trial was again reset
to January 21, 2004.
Before the scheduled pretrial on January 21, 2004 or on

November 3, 2003, petitioner filed


a
Motion
to
Dismiss
the
Information on the ground of
violation of his right to a speedy
trial under Republic Act No. 8493
or the Speedy Trial Act of 1998
and Supreme Court Circular (SCC)
No. 38-98. He argued that
"considering that [he] was not without any fault on his part brought to trial within 80 days
from the date he was arraigned,
this case should be dismissed
pursuant to Rule 119, Section 9 in
relation to Rule 119, Section 6 of
the Rules."
The trial court, through
pairing Judge Danilo A. Buemio
(respondent
judge),
denied
petitioners Motion to Dismiss by
Order of December 5, 2003,
holding that petitioner played a big
part in the delay of the case, and
that technical rules of procedure
were meant to secure, not
override, substantial justice.
Petitioners
Motion
for
Reconsideration of the December
5, 2003 Order was denied by
Order of March 3, 2004 after
respondent
judge noted that
during petitioners arraignment on
February 12, 2003, he interposed
no objection to the setting of the
pre-trial to May 28, 2003.
Petitioner
challenged
respondent judges orders via
certiorari and prohibition before
the Regional Trial Court (RTC) of
Manila, alleging that not only was

he (petitioner) not brought to trial


within 80 days from the date of his
arraignment as required under
Section 6, Rule 119, but the
prosecution had failed to establish
the existence of any of the "time
exclusions" provided under Section
3of the same Rule to excuse its
failure to bring him to trial within
the 80-day period.
By Decision of January 31,
2006, the RTC denied the petition,
holding that Section 9 of Rule 119
of the Rules of Court does not call
for the automatic dismissal of a
case just because trial has not
commenced within 80 days from
arraignment.
ISSUE:

Whether
or
not
the
petitioners right to speedy trial
was breached when he was not
held for trial within 80 days from
the date of his arraignment
pursuant to Speedy Trial Act of
1998 and Supreme Court Circular
(SCC) No. 38-98.

HELD:

The right is deemed violated


only when the proceedings are
attended by vexatious, capricious
and oppressive delays, which did
not obtain in the present case,
petitioner himself having been

instrumental in the delay in the


prosecution of the case.
It bears noting, however, that on
his arraignment on February 12,
2003, petitioner interposed no
objection to the setting of the pretrial to May 28, 2003 which was,
as earlier stated, later declared a
non-working day. Inarguably, the
cancellation of the scheduled pretrial on that date was beyond the
control of the trial court.
Besides, respondent judge
held, strict compliance with the
Speedy Trial Act was improbable,
given the volume of cases being
filed with the MeTC. Additionally
respondent judge held that the
term "speedy trial" as applied in
criminal cases is a relative term
such that the trial and disposition
of cases depended on several
factors including the availability of
counsel, witnesses and prosecutor,
and weather conditions.
While justice is administered
with
dispatch,
the
essential
ingredient is orderly, expeditious
and not mere speed. It cannot be
definitely said how long is too long
in a system where justice is
supposed
to
be
swift,
but
deliberate. It is consistent with
delays
and
depends
upon
circumstances.

A balancing test of applying


societal interests and the rights of
the accused necessarily compels

the court to approach speedy trial


cases on an ad hoc basis.

The time limits set by the


Speedy Trial Act of 1998 do not
thus
preclude
justifiable
postponements and delays when so
warranted by the situation. To the
Court,
the
reasons
for
the
postponements
and
delays
attendant to the present case
reflected
above
are
not
unreasonable. While the records
indicate that neither petitioner nor
his counsel was notified of the
resetting of the pre-trial to
October 23, 2003, the same
appears to have been occasioned
by oversight or simple negligence
which, standing alone, does not
prove fatal to the prosecutions
case.

Right against Selfincrimination

PEOPLE OF THE PHILIPPINES


vs. JONATHAN BESONIA
FACTS:
Besonia was charged with
murder
in
two
separate
informations.
Upon
his
arraignment on 22 August 2000,
Besonia entered a plea of not

guilty in each case. Thereafter, a


pre-trial was held.
On 6 March 2001, before the
start of the trial, Besonia, through
his counsel Atty. Calixto Perez,
manifested that he would enter a
plea of guilty to the lesser offense
of homicide after a medical
operation on his gall bladder.
On 29 May 2001, Besonia
manifested his desire to enter a
plea of guilty to murder. Rearraignment was then scheduled
on 5 June 2001. On his rearraignment,
Besonia
pleaded
guilty to the two charges of
murder. The trial court forthwith
conducted a searching inquiry to
determine the voluntariness and
full comprehension of his plea.
PO3 Feliprada testified that
at about 3:30 p.m. of 27 June 2000
he, together with PO3 Gerardo
Jison and PO2 Hagmay Dignadice,
responded to a report of a shooting
incident
in
Guzman
St.,
Mandurriao,
Iloilo
City.
Information from the bystanders
readily identified Besonia as the
assailant who then boarded a
passenger
jeep.
The
officers
immediately chased the jeep.
Besonia attempted to escape, but
to no avail. PO2 Dignadice
recovered from Besonias waist a .
38 caliber revolver. Thereafter,
Besonia was brought to the police
station for investigation, and the
firearm was submitted to the crime

laboratory for gunpowder residue


examination.
After the prosecution had
rested its case, the defense
manifested that it would not
present any evidence. On 26 June
2001, the trial court promulgated
judgment which is now the subject
of this automatic review.
In the Appellants Brief,
Besonia, through his new counsel
de parte Atty. Jose B. Tiangco,
prays for the reversal of the
judgment of conviction and his
acquittal based on this sole
assignment of error: Besonia
argues that the finding of guilt by
the trial court was based mainly on
his
confession,
which
is
inadmissible for having been
obtained in gross violation of his
constitutional right against selfincrimination.

The
right
against
selfincrimination
is
intended
to
prevent the State, with all its
coercive powers, from extracting
from the suspect testimony that
may convict him and to avoid a
person
subjected
to
such
compulsion to perjure himself for
his own protection.
It does not apply where, as
in these cases, the testimony was
freely and voluntarily given by the
accused
himself
without
any
compulsion from the agents of the
State. There is nothing in the
records that would indicate that
Besonia was forced, intimidated,
or compelled by the trial court or
by anybody into admitting the
crimes. At any rate, his plea of
guilty
and
confession
or
admissions during the searching
inquiry cannot be the sole basis for
his conviction.

ISSUE:
Whether or not accusedappellants confession amounts to
violation of his constitutional right
against self-incrimination.
HELD:
We cannot subscribe to
Besonias claim that his confession
and
admissions
during
the
searching inquiry were elicited in
violation of his constitutional right
not to be compelled to testify
against himself.

THE PEOPLE OF THE


PHILIPPINES vs. HON. JUDGE
RUBEN AYSON
FACTS:
Felipe Ramos was a ticket
freight clerk of the PAL assigned at
its Baguio City station. He was
allegedly involved in irregularities
in the sales of plane tickets.
On the day before the
investigation of the irregularities
by the management, Ramos gave

to his superiors handwritten notes


which stated that he is willing to
settle the irregularities allegedly
charged against him. At the
investigation, Ramos was informed
of the findings of the Audit Team.
Thereafter, his answers to every
question by the management were
taken down
in writing.
He
admitted that he did not disclose
the sale of tickets to the
management, that he misused the
proceeds and that he was still
willing to settle his obligation on
an installment basis. However, no
compromise
agreement
was
reached by the parties.
Two months later, Ramos
was charged with the crime of
estafa in connection with the ticket
sales irregularities. Ramos entered
a not guilty plea and trial ensued.
The
private
prosecutors
presented evidence against Ramos
which included his statement
taken during the PAL management
investigation (Exhibit A) as well as
his handwritten admission to
superiors
to
compromise
his
obligations (Exhibit K).
The defendant's attorneys
objected to the evidence presented
on the ground that they appear to
be a confession and was taken
without
the
accused
being
represented by a lawyer.
By Order, the lower court
judge admitted all the exhibits
except Exhibits A and K. He

declared that it does not appear in


Exhibit A and K that the accused
was reminded of his constitutional
rights to remain silent and to
have counsel and that these rights
may be waived provided it was
with the assistance of a counsel
and in writing."
ISSUE:
Whether
or
not
the
contention
of
the
accusedappellant that his constitutional
right against self-incrimination has
been breached can be sustained.
HELD:
Felipe Ramos was not in any
sense
under
custodial
interrogation prior to and during
the administrative inquiry into the
discovered irregularities in ticket
sales in which he appeared to have
had
a
hand.
The constitutional rights of a
person
under
custodial
interrogation did not therefore
come into play. It is also clear that
Ramos had voluntarily answered
questions posed to him on the first
day
of
the
administrative
investigation and agreed that the
proceedings should be recorded.
His offer to compromise his
liability
in
the
alleged
irregularities thru a letter was a
free and even spontaneous act on
his part. They may not be excluded
on the ground that the so-called
"Miranda rights" had not been
accorded to Ramos.

The
right
against
selfincrimination is accorded to every
person
who
gives
evidence,
whether voluntarily or under
compulsion of subpoena, in any
civil, criminal, or administrative
proceeding. The right is not to "be
compelled to be a witness against
himself" However, the right can be
claimed only when the specific
question,
incriminatory
in
character, is actually put to the
witness. It cannot be claimed at
any other time. It does not give a
witness the right to disregard a
subpoena, to decline to appear
before the court at the time
appointed, or to refuse to testify
altogether.
The
right
against
selfincrimination is not self- executing
or automatically operational. It
does not impose on the judge, or
other officer presiding over a trial
to advise a witness of his right
against self-incrimination. If not
claimed by or in behalf of the
witness at the appropriate time, it
follows that the right is waived,
expressly, or impliedly.

ROGER CHAVEZ vs. THE


HONORABLE COURT OF
APPEALS, THE PEOPLE OF
THE PHILIPPINES and THE
WARDEN OF THE CITY JAIL OF
MANILA

FACTS:

Petitioner Roger Chavez is a


defendant in a criminal case
involving qualified theft of a motor
vehicle( one (1) Thunderbird car).
Upon arraignment, all the
accused, except the three Does
who have not been identified nor
apprehended,
pleaded
not
guilty.1wph1.t
On July 23, 1963, trial
commenced before the judge
presiding Branch IX of the Court of
First Instance of Rizal in Quezon
City.
He was called by the
prosecution as the first witness in
that case to testify for the People
during the first day of trial thereof.
Petitioner objected and invoked
the privilege of self-incrimination.
This he broadened by the clear cut
statement that he will not testify.
But petitioner's protestations were
met with the judge's emphatic
statement that it "is the right of
the prosecution to ask anybody to
act as witness on the witness stand
including the accused," and that
defense counsel "could not object
to have the accused called on the
witness stand." The cumulative
impact of all these is that accusedpetitioner had to take the stand.

ISSUE:

Whether or not the act of


Chavez testifying before the court
is equivalent to waiver of his right
against self-incrimation.

HELD:

There is no waiver of the


privilege.

To be effective, a waiver
must be certain and unequivocal,
and intelligently, understandably,
and willingly made; such waiver
following only where liberty of
choice has been fully accorded.
"It has been pointed out that
"courts indulge every reasonable
presumption against waiver" of
fundamental constitutional rights
and that we "do not presume
acquiescence in the loss of
fundamental rights." A waiver is
ordinarily
an
intentional
relinquishment or abandonment of
a known right or privilege."
We have no hesitancy in
saying that petitioner was forced
to testify to incriminate himself, in
full breach of his constitutional
right to remain silent. It cannot be
said now that he has waived his
right. He did not volunteer to take
the stand and in his own defense;
he did not offer himself as a
witness; on the contrary, he

claimed the right upon being


called to testify. If petitioner
nevertheless
answered
the
questions inspite of his fear of
being accused of perjury or being
put
under
contempt,
this
circumstance cannot be counted
against him. His testimony is not of
his own choice. To him it was a
case of compelled submission. He
was a cowed participant in
proceedings before a judge who
possessed the power to put him
under contempt had he chosen to
remain silent. Nor could he escape
testifying.
Here involve is not a mere
question of self-incrimination. It is
a
defendant's
constitutional
immunity from being called to
testify against himself. And the
objection made at the beginning is
a continuing one.

RIGHTS OF THE ACCUSED


Kastigar v. United States
406 U.S. 411, 1972
FACTS: Subpoenaed to appear
before a federal grand jury, the
petitioners refused to answer
questions in asserting the Fifth
Amendment,
despite
the
government having granted them
immunity. The immunity was based

on a part of the Organized Crime


Control Act of 1970 stating that
neither the testimony nor any
other
information
from
the
testimony could be used against
the witnesses. Petitioners argued
that the scope of the immunity
they were given was not as broad
as the scope of the privilege
against
self-incrimination,
and
were unsuccessful in the District
Court. The Ninth Circuit Court of
Appeals affirmed the order, and
the petitioners were granted
certiorari.
ISSUE: Can the Government
compel
testimony
from
subpoenaed persons and not allow
them to invoke their privilege
against self-incrimination if they
are conferred with immunity?
HELD: Yes, The government may
compel
testimony
from
an
unwilling witness who invokes the
Fifth Amendment privilege against
compulsory self-incrimination by
conferring immunity from use of
the compelled testimony and
evidence derived therefrom in
subsequent criminal proceedings.
Tanchanco v. Sandiganbayan
G.R. No. 14167596, November 25,
2005
FACTS: Tanchanco served as NFA
Administrator
during
the
presidency of Ferdinand Marcos.
Tanchanco and PCGG entered into
a cooperation agreement with the
Philippine
Government
in
connection with the latters efforts
in the location and pursuit of
government properties purloined
by Ferdinand and Imelda. In this
cooperation agreement, Tanchanco
bound himself to provide truthful

disclosures, in response to any and


all questions and inquiries that
may be put to him/her in
connection with said investigation.
In exchange thereof, Philippine
Government bound itself to dismiss
all actions that are presently
pending against Tanchanco before
the Sandiganbayan and other
courts and the lifting of the
sequestration orders against the
property of Tanchanco.
In view of the above mentioned
agreement, Tanchanco was called
upon to be a witness in the
prosecution of the case filed
against Imelda for violation of
RICO Act. Nonetheless, a criminal
case
was
still
filed
by
Sandiganbayan against Tanchanco
for malversation of public funds in
the amount of 10M. Tanchanco
moved to dismiss the case invoking
the immunity granted him by the
PCGG. In its reply, Sandiganbayan
argued that the charge for
malversation
of
public
funds
against
Tanchanco
was
not
covered by the immunity as the
latter case has nothing much to do
with recovery of ill-gotten wealth
of Ferdinand and Imelda Marcos
and that PCGG could not have
intended the grant of immunity to
extend to any other crime which
Tanchanco may have committed
while
serving
the
Marcos
administration.
ISSUE: Whether or not the charge
of malversation of public funds is
covered by the immunity granted
by PCGG to Tanchanco.
HELD: Yes. The grant of immunity
to Tanchanco is deliberately broad.
It
is
stipulated
that
the
government shall not bring any

additional civil or criminal charges


against Tanchanco arising from:
(a) service in or for the Marcos
government; and (b) any other
actions revealed by Tanchanco
pursuant
to
his/her
(sic)
cooperation as defined in this
Agreement.
The
reasons
or
motives of the PCGG in agreeing to
so broad an immunity agreement
are not evidently determinable, yet
ultimately excluded from the scope
of judicial inquiry.
Mapa v. Sandiganbayan
100295, April 26, 1994
FACTS: Mapa was charged with
the violation of the Anti-Graft and
Corrupt Practices. PCGG granted
an immunity from suit by the
PCGG related to the previous
charges
against
him
preconditioned with his being
witness against the Marcoses in
criminal proceedings in a separate
case where United States vs
Ferdinand Marcos, during the
RICO, where Ferdinand and Imelda
Marcos were being tried for
charges of corruption. In view of
the immunity granted to Mapa ,
PCGG shouldered the expenses of
Mapa when he flied to Ney York to
testify against the Marcoses.
During the pendency of the
corruption charges filed in New
York, Ferdinand Marcos died. La
Bella, the American Prosecutor
dispensed the testimony of Mapa
and thereby acquitted Imelda
Marcos. Since Mapa was not able
to testify, Sandiganbayan argued
that the immunity from suit of
Mapa in without any force and
effect

ISSUE: Is the immunity given by


the PCGG still in effect and force
despite the failure of Mapa to give
testimony?
HELD: Yes. PCGG has the power
to grant immunity to any person
from being prosecuted provided
that they will meet the conditions
provided by PCGG. In this present
case, Mapa was granted immunity
from the prosecution and PCGG
even shouldered all the expense of
Mapa when he flew to New York to
testify implying that Mapa was
able to meet the conditions .
However, failure of Mapa to testify
against the Marcoses during the
RICO cannot nullify the immunity
given to him by the PCGG since
petitioner was able to testify the
requirement both of the law and
the
parties
implementing
agreements. Though Mapa was not
able
to
testify
against
the
Marcoses in RICO, it can be said
that it is not due to his own fault.
Thus, petitioner Mapa must be
acquitted.
People v. Alcanzado
428 SCRA 681, May 20, 2004
FACTS: Alcanzado pleaded not
guilty during his arraignment and
train
on
the
merit
ensued
thereafter. Upon his motion, RTC
allowed Alcanzado to file a
demurrer to evidence which was
opposed by the prosecution. On
1999, RTC promulgated the herein
assailed
decision
convicting
Alcanzafo. This present petition
was
brought
by
Alcanzado
assailing the said RTC decision. He
argues the RTC committed an
error in promulgating the decision

without first giving Alcanzado the


opportunity to present evidence.
ISSUE: Is the constitutional right
of the accused to be heard on his
defence violated?
HELD: Yes. Where the accused
had filed a motion for leave to file
a demurrer to evidence which was
granted by the trial court, upon
denial thereof if indeed it was
denied, the trial court should give
the accused the opportunity to
present his evidence; Due to the
procedural
unfairness
and
complete miscarriage of justice in
the handling of the proceedings in
the RTC, a remand of the case for
reception of defense is warranted
as the constitutional right of the
accused to be heard has been
violated.
People vs Ortillas
428 SCRA 659, May 20, 20014
FACTS: An Information for murder
was filed against respondent
Ortillas one Jose Labarosa. The
first counsel of the Ortillas, Atty de
Leon,
requested
for
the
postponement
of
the
cross
examination in view of some
professional engagement. The
next hearing was also postponed in
view of the eye problem of the eye
problem of Atty. de Leon. The
hearing was again postponed due
to the withdrawal of appearance of
Atty. de Leon on the ground again
of eye ailment. In view on the
aforementioned
events,
Atty.
Macinas appeared as the new
counsel for Ortillas.
It appears that during the
scheduled cross-examination of the
witness of the prosecution,

Russel, the latter made no


appearance. Nonetheless, Judge
Alumbres
rendered
judgement
convicting Ortillas. In view thereof,
Ortillas
brought
the
present
petition claiming that he was
deprived of his right to crossexamine
Russel
prior
the
rendering of judgement.
ISSUE: Is Ortillas deprived of his
right to cross-examine witness?
HELD: Yes. The cross-examination
of a witness is essential to test his
or
her
credibility,
expose
falsehoods or half truths, uncover
the truth which rehearsed direct
examination
testimonies
may
successfully
suppress,
and
demonstrate inconsistencies in
substantial matters which create
reasonable doubt as to the guilt of
the
accused
and
thus
give
substance to the constitutional
right of the accused to confront
the witnesses against him.
Dico v. CA
G.R. No. 141669, February 28,
2005
FACTS:
Petitioner Dico
was
charged of violations of BP 22 on
three counts. When arraigned,
accused pleaded not guilty to each
charges. The charge was anchored
from the issuance of Dico to
Equitable Banking Corporation a
check in payment for the credit
card obligations to said bank. The
RTC convicted Dico of BP 22 on
three counts. A motion for
reconsideration was filed to the
RTC. This was however denied and
thus Dico went up to the Court of
Appeal to seek reversal of the
judgement of the RTC which
affirmed the decision of the MTCC.

When the action was brought to


CA, Dico argues that Dico failed to
establish all the elements of
violation of BB 22 and thus should
be acquitted of the said charge.
ISSUE: Was the prosecution able
to prove ll the elements of BP 22?
HELD: no. The essential elements
of the offense penalized under
Section 1, B.P. Blg. 22 are as
follows: (1) the making, drawing
and issuance of any check to apply
to account or for value; (2) the
knowledge of the maker, drawer or
issuer that at the time of issue he
does not have sufficient funds or
credit with the drawee bank for
the payment of such check in full
upon its presentment; and (3)
subsequent dishonor of the check
by
the
drawee
bank
for
insufficiency of funds or credit or
dishonor for the same reason had
not the drawer, without any valid
cause, ordered the bank to stop
payment. In the case at bar, the
prosecution failed failed to make a
notice of dishonour to Dico as the
maker of the check. A notice of
dishonour, which may be sent to by
the offended party or the drawee
bank, is indispensable before a
conviction
can
ensue.
Furthermore, the Supreme Court
also remarked that an appeal in a
criminal case opens the entire case
for review and the appellate court
may correct even unassigned
errors.
People vs. Casta
565 SCRA 341, Sept. 16, 2008
FACTS: Appellant Casta was
charged by the prosecution of the
crime of murder after attacking
without warning attack with a

knife Danilo Camba. Casta pleaded


not guilty to the charge upon
arraignment then the prosecution
presented its witnesses. One of the
witnesses is Marlyn Cister who,
while seated on the steps of the
stairs of their house, saw Camba
stab Danilo. Another witness is one
Modesto who narrated that he also
witnesses Danilo appearing from
behind Danilo and stabbed him
using a double bladed knife.
Appellant Casta gave a different
version of the events by arguing
that his stabbing of Danilo Camba
was accidental and that it merely
occurred after the latter boxed
him. He alleged that he has no
previous grudge against Camba.
Nonetheless, RTC convicted Casta
of murder and the same was
affirmed by CA. This present
petition was brought by Casta
contending that RTC erred in
convicting him of the crime of
murder.
ISSUE:
Is the evidence of
prosecution sufficient to convict
Casta of murder?
HELD: Yes. An established rule in
appellate review is that trial courts
factual findings, including its
assessments of the credibility of
the witnesses and the probative
weight of their testimonies, as well
as the conclusions drawn from the
factual findings, are accorded
respect, if not conclusive effect.
These
actual
findings
and
conclusions assume greater weight
if they are affirmed by the CA.
ARRAIGNMENT AND PLEA

Brig. Gen. Jose Ramiscal Jr vs


Sandiganbayan and People of
the Philippines
G.R. No. 172476, September 15,
2010
FACTS: Ramiscal is a retired
officer of AFP and the former
president of AFP Retirement and
Separation Benefits System (AFP
RSBS for brevity). While serving as
such, the Board of Directors of
AFB
RSBS
approved
the
acquisition of 15, 020 sq m of land
development as housing projects.
Later
on,
AFP
RSBS
as
represented by Ramiscal and one
Flavian executed a bilateral Deed
of Sale (first Deed) over the
subject property at the agreed
price of 10, 500 pesos per sam.
After the payment of said amount,
Flavian again signed a Deed of
Sale (second Deed) over the same
property this time with a purchase
price of 3, 000 pesos per sqm.
After the execution of the second
Deed of Sale, it was presented by
Flaviano for registration and he
thereafter secured a Certificate of
Titile
of
the
said
property.
Ramiscal filed his motion for
reconsideration
regarding
the
finding of the Ombudman against
him. Thereafter, a panel of
prosecutors tasked to review the
records concluded that Ramiscal
indeed participated in affixing his
signature on the above mentioned
contracts and found probable
cause. The Ombudman acted
positively on the findings of the
panel
of
prosecutors
and
scheduled the arraignmentt of
Ramiscal. The latter however
refused to enter a plea on the
ground of the pendency of the

resolution of his second motion for


reconsideration.
ISSUE: Is the second Motion for
Reconsideration
valid?
Should
Ramiscal again be held for
arraignment?
HELD: No. Rules of Court
provides that only one motion for
reconsideration or reinvestigation
of an approve resolution shall be
allowed and the filing of a motion
for reconsideration shall not bar
the filing of the corresponding
information in court on the basis of
the finding of probable cause in
the resolution of the subject of the
motion. Arraignment may be
suspended under Section 11 of
Rule 116 of the Rules of Court on
the grounds of unsoundness of
mind, prejudicial question and a
pending petition for review of the
resolution of the prosecutor in the
DOJ in which the suspension shall
not exceed 60 days. None of these
instances a re present in the case
at bar.
People of the Philippines vs.
Pangilinan
518 SCRA 358, March 14, 2007
FACTS:Two Information of Rape
was
filed
against
appellant
Pangilinan. thereafter, appellant
was arrested and detained and
applied for bail. During the haring
for bail, prosecution presented its
witnesses which includes the
private complainant AAA. It was
proven during th bail hearing that
AAA
was
the
daughter
of
Pangilinan. On 30 October 1997,
the prosecution finally offered its
evidence
including
those
resolutions in the bail hearing.

Appellant filed his opposition to


the prosecutions offer of evidence.
Nonetheless, the trial court was
convinced hat appellant indeed
raped AAA who was then 10 years
old. When the case was elevated to
the Supreme Court, appellant
assigned as error that he was not
properly arraigned and was not
informed of the nature and cause
of accusation against him before
the evidence of the prosecution
was
presented.
Admittedly,
appellant was arraigned only after
the case was submitted for
decision.
ISSUE: Did appellants belated
arraignment prejudice him?
HELD:
Appellants
belated
arraignment did not prejudice him.
This procedural defect was cured
when his counsel participated in
the trial without raising any
objection that his client had yet to
be arraigned. In fact, his counsel
even
cross-examined
the
prosecution
witnesses.
His
counsels active participation in
the hear-ings is a clear indication
that he was fully aware of the
charges against him; otherwise,
his counsel would have objected
and informed the court of this
blunder. Moreover, no protest was
made
when
appellant
was
subsequently
arraigned.
The
parties did not question the
procedure undertaken by the trial
court. It is only now, after being
convicted and sentenced to two
death sentences, that appellant
cries that his constitutional right
has been violated. It is already too
late to raise this procedural defect.
Gamas vs Oco

425 SCRA 588, March 17, 2004


FACTS: Complainant herein filed a
complaint for ignorance of the law
against respondent Judge Oco.
Complainants allege that they are
accused in the sala of the
respondent judge. It was alleged
by complainants that one of the
respondents, Adulation enticed
them to plead guilty to the charge,
apply for probation and thus avoid
imprisonment. This was done
through
a
draft
prepared
Adulation. This draft, after being
read by Adulation, was then and
there signed respondent judge
after
the
latter
told
the
complainants O, plead guilty man
kamo. Respondent judge told the
clerk to read the contents of the
decision to complainants and to
instruct them on what to do. The
clerk read the contents of the
document to complainants and
asked
them
to
sign
it.
Complainants signed the document
upon
respondent
Adulacions
assurance that once the police
apprehend the rest of the accused,
the police will revive the case and
respondent Adulacion will present
complainants as star witnesses.
Complainants later found out that
what they signed was an Order (3
October 1996 Order) finding them
guilty of theft and sentencing them
each to imprisonment.
In the present complaint,
petitioner complainants prays that
respondent
judge
be
held
administratively
liable
for
rendering a judgement against
them without the benefit of
arraignment.
ISSUE: Should Judge Oco be held
administratively liable?

HELD: Yes. Arraignment is not an


empty ritual that should be taken
lightly. [S]ection 1(a) of Rule 116
requires that the arraignment
should be made in open court by
the judge himself or by the clerk of
court [1] furnishing the accused a
copy
of
the
complaint
or
information with the list of
witnesses stated therein, then [2]
reading the same in the language
or dialect that is known to him,
and [3] asking him what his plea is
to the charge. The requirement
that the reading be made in a
language or dialect that the
accused understands and knows is
a mandatory requirement, just as
the whole of said Section 1 should
be strictly followed by trial courts.
This the law affords the accused by
way of implementation of the allimportant constitutional mandate
regarding the right of an accused
to be informed of the precise
nature of the accusation leveled at
him and is, therefore, really an
avenue for him to be able to hoist
the necessary defense in rebuttal
thereof. It is an integral aspect of
the due process clause under the
Constitution.
People vs Espinosa
409 SCRA 256, August 15, 2003
FACTS: Several counts of Estafa
was filed before the SBN by the
Office of the Ombudsman against
respondents.
Prior
to
his
arraignment, Espinosa filed a
Motion for Reinvestigation of the
cases. While the cases were being
reevaluated, Espinosa filed with
the SBN a Motion for Leave to
Travel Abroad. As ordered, private
respondent was arraigned, and

thereafter granted his Motion to


Travel. Thereafter, the OMB filed
in
the
same
court
seven
Informations for Malversation of
Public Funds against Espinosa and
several others. These Informations
were docketed as Criminal Case
Nos. 24622 to 24628 and raffled to
the SBN First Division. Thereafter,
the OMB filed in the same court
seven
Informations
for
Malversation of Public Funds
against Espinosa and several
others. These Informations were
docketed as Criminal Case Nos.
24622 to 24628 and raffled to the
SBN First Division.
Petitioner countered that the
arraignment for the two previous
cases was conditional, because it
was made solely for the purpose of
accommodating
private
respondents request to travel
abroad while the matters were
pending reinvestigation.
ISSUE:
Is
there
a
double
jeopardy?
HELD: No legal jeopardy attaches
only: (a) upon a valid indictment;
(b) before a competent court; (c)
after arraignment; (d) [when] a
valid plea [has] been entered; and
(e) the case was dismissed or
otherwise terminated without the
express consent of the accused. As
stressed in the petition, the
arraignment was conditional for if
it was not so, respondent Espinosa
would have been deemed to have
abandoned his recourse for the
reevaluation of his cases before
the Office of the Ombudsman.
Miranda vs. Tuliao
486 SCRA 377, March 31, 2006

FACTS: Two burnt cadavers were


found which were later identified
as bodies of Vicente Bauzon and
Elizer Tuliao, son of private
respondent
Virgilio
Tuliao.
Petitioners are police officers as
responsible
for
the
deaths.
Respondent Tuliao filed a criminal
complaint for murder against
petitioners.
Thereafter,
acting
presiding judge issued warrant of
arrest
against
petitioners.
Petitioners then filed an urgent
motion to complete preliminary
investigation, to reinvestigate and
to recall or quash the warrant of
arrest. Judge Tumaliuan noted the
absence of petitioners and issued a
joint order denying said motion on
the ground that since the court did
not acquire jurisdiction over the
persons of the petitioners. State
Prosecutor
Leo
Reyes
and
respondent Tuliao moved for
reconsideration of the said Joint
Order and prayed for the inhibition
of Judge Anghad. RTC denied the
Motion for Reconsideration. Court
of Appeals, on the other hand,
ordered the reinstatement of the
criminal cases in the RTC as well
as the issuance of warrant of
arrest against petitioners. At this
juncture, petitioners invoke double
jeopardy.
ISSUE: With the reinstatement of
the criminal case, is there a double
jeopardy?
HELD: None. The reinstatement of
a criminal case dismissed before
arraignment does not constitute
double jeopardy. Double jeopardy
cannot be invoked where the
accused has not been arraigned it
was his express motion that the
case was dismissed.

People vs Jalbuena
526 SCRA 500, July 4, 2007
FACTS: AAA, a minor, was raped
by her father in the morning of
August 1996 while her mother was
out. Jalbuena approached AAA
while the latter is on bed and
placed himself on top of her and
inserted his penis in her vagina.
Jalbuena succeeded in having
carnal knowledge with AAA after
being threatened by her father
that something bad will happen to
her if she will not be allowed to
have sexual intercourse with her.
The incident happened on two
other occasions. Dr. Salumbides
then examined AAA with the
finding that the hymen was intact
and that there are no spermatozoa
in the vaginal smear. All these
allegation was denied by By
Jalbuena. The latter contends that
his job as canvasser of plastic ware
requires him to out of the house
most of the time, except on
Saturday. The RTC found AAAs
testimony as clear, consistent,
direct, and without hesitation
when confronted by the presence
of her own abuser. Jalbuenas alibi
was not credited there being no
proof that it was physically
impossible for him to be at the
place, date, and time of the
commission of the offense. Thus,
Jalbuena was convicted and was
found guilty beyond reasonable
doubt. In view thereof, Jalbuena
moved for bill of particulars on the
ground that the exact date of the
commission of the crime was not
properly
alleged
in
the
Information.

ISSUES: Is the Bill of Particulars


proper?
HELD: No. Bill of Particulars
should have been filed before one
Jalbuena entered his plea. His
failure to do so amounted to
waiver of the defect or detail
desired
in
the
information.
Furthermore, it is not necessary to
state
in
the
complaint
or
information the precise time at
which the offense was committed
except when the time is material
ingredient of the offense. Date is
not a material element of rape.
People vs. Mala
411 SCRA 327, September 18,
2003
FACTS: Appellants Mala and Bala
were charged with the offense of
selling and delivering prohibited
drugs as defined under RA 6425.
Upon
arraignment,
both
the
accused pleaded not guilty to the
offense charged. At the trial of the
case, the prosecution presented
the police poseur-buyers in the
buy-bust operations who positively
caught appellants in committing
the offense as proved by their
acceptance of the boodle money.
The counsel de officio, however,
prayed before the RTC that
appellant Bala be not presented
because said counsel had a hard
time communication with him, and
that according to co-accused Mala,
Bala
is
somewhat
deficient
mentally. The trial court did not lift
a finger to have Bala examined by
a physician or to ensure that his
defence was properly undertaken.
RTC
gave
credence
to
the
testimony of the prosecution
witness and accordingly, rendered

judgement convicting appellants.


Unsatisfied, appellants brought the
appeal.
ISSUE: Should Bala be convicted
in view of his unsound mind during
arraignment?
HELD: No. Section 11, Rule 116 of
the Revised Rules of Criminal
Procedure
provides
for
the
suspension of the arraignment of
an accused who appears to be
suffering from an unsound mental
condition. It also imposes a duty
upon the court to order his mental
examination and, if called for, his
confinement for such purpose. The
said Section pertinently reads:
SEC.
11.
Suspension
of
arraignment.Upon motion by the
proper party, the arraignment shall
be suspended in the following
cases:
(a) The accused appears to be
suffering from an unsound mental
condition which effectively renders
him unable to fully understand the
charge against him and to plead
intelligently thereto. In such case,
the court shall order his mental
examination and, if necessary, his
confinement for such purpose. This
provision reinforces Article 12,
paragraph 1, of the Revised Penal
Code, which mandates the trial
court to order the confinement of
an accused who is mentally
unsound at the time of the trial in
one of the hospitals or asylums
established
for
persons
thus
afflicted.
Gandarosa v. Flores
527 SCRA 776, July 17, 2007
FACTS: The Daily Informer, a
newspaper in Iloilo City, published
a banner headline, Gandarosa

Wants Flores Out for Personal


Convenience? According to the
article, petitioner as the ARD of
the BIR, revealed to members of
the
media
that
high-ranking
officials of BIR Officials, among
them RD Sonia Flores, are
involved in anomalous transactions
to favour certain taxpayers in the
assessment of their taxes. On the
newspapers front page, a photo of
Flores was also included. In view
of this publications, Evaristo
Flores, the husband of Sonia
Flores and with the conformity of
the latter filed a complaint against
petitioner.
The
investigating
prosecutor found probable cause
to hold petitioner liable for Libel.
Later, an Information for Libel was
filed before the RTC against
petitioner Gandarosa.
Petitoner filed an urgent
motion for suspension of the
proceedings in the RTC stating
therein that that there is a pending
motion for Reconsideration with
the Office of the City Prosecutor
where petitioner sought to reverse
the finding of probable cause
against him for libel. RTC denied
the motion to suspend proceeding.
Thus,
petitioner
brought
the
present action.
ISSUE: Should the proceeding be
suspended in view of the pending
resolution
of
the
prosecutor
regarding the finding of probable
cause for libel?
HELD: No. Jurisprudence is clear
that with the arraignment of the
accused, the DOJ Secretary can no
longer entertain appeal or petition
for review because petitioner had
already waived or abandoned the
same. Once an information is filed

in court, any disposition of he case


such
as
its
dismissal
or
continuance rests on the sound
discretion of the court.
People vs. Lozada
406 SCRA 494, July 17, 2003
FACTS: The lifeless body of Rosita
Sy was found inside her car along
a remote hacienda in Bacolod City.
The police apprehended appellants
and was thereafter charged with
robbery with homicide before the
RTC. Appellants the entered a plea
on not guilty to the accusation.
Trial on the merit and the trial
court found appellants guilty
beyond reasonable doubt of the
crime with which they were
charged and sentenced each of
them to suffer the penalty of death.
In the present review, appellants
would want to the court to hold
that the RTC erred in not finding
their warrantless arrest unlawful
and that the search conducted on
his person is unlawful.
ISSUE: Considering that the
legality of arrest was raised after
the rendered judgment, should the
question on the legality of arrest
be dismissed?
HELD: Yes. An objection against
an arrest or the procedure in the
acquisition by the court of
jurisdiction over the person of an
accused should be made at or
before arraignment, otherwise the
objection
is
deemed
waived.
Appellants entered their plea of
not guilty to the crime of robbery
with homicide and thereafter
participated in the trial without
questioning the legality of their
arrest.

Borlongan, Jr. v Pena


G.R. No. 143591, November 23,
2007
FACTS: Respondent Atty. Pena
filed the present civil case for
recovery of sums of money
anchored
anchored
from
a
Contatct of Agency allegedly
entered into between petitioners
wherein the former undertook to
perform such acts necessary to
prevent any intruder and squatter
from unlawfully occupying Urban
Banks property located in Pasay.
Petitioners, on the other hand,
filed a motion to dismiss arguing
that Respondent Atty. Pena was
never appointed as agent or
counsel. To support the claim of
petitioners, they presented with
the motion to dismiss several letter
that will attest that Atty. Pena was
indeed never appointed as counsel
or agent.
Respondent Pena filed his
complaint with the Office of the
City Prosecutor where he claimed
that said documents were falsified
for the alleged signatories therein
did
not
actually
affix
their
signatures. The resolution of the
prosecutor concluded that there
exist probable cause of the the
crime of Introducing Falsified
Documents penalized under Art.
172 of the RPC. Later, petitioners
filed a Motion to Quash arguing
that they were denied of due
process for the trial judge merely
relied on the complaint affidavit
and attachments of respondent in
the issuance of warrant of arrest.
On the same day they filed their
Motion to Quash, petitioners also
posted bail. The MTC upheld the
validity of their arrest and did not

allow petitoners to question the


validity of their arrest on the
ground that they already posted
bail.
ISSUE:
Considering
that
petitioners already posted bail, are
they
now
precluded
from
questioning the validity of their
arrest?
HELD: No. The principle that the
accused
is
precluded
from
questioning the legality of his
arrest after arraignment is true
only if he voluntary enters his plea
and participates during the trial,
without previously invoking his
objections thereto. The posting of
a bail does not preclude one from
questioning the validity of his
arrest.
Adasa vs. Abalos
516 SCRA 261, February 19, 2007
FACTS: Respondent Abalos filed a
complaint against Petitioner Adasa
for Estafa through falsification of
commercial document by private
individual. Abalos alleged that
Adasa encashed two checks issued
in the name of respondent through
deceit and without her knowledge.
The Office of the City Prosecutor of
Iligan City issued a Resolution
finding probable cause against
Adasa and ordered the filing of
separate informations for Estafa
through falsification of commercial
document by a private individual. .
Adasa has entered a not guilty plea
during
her
arrangement
on
October 1, 2001 and later filed a
petition for review before the DOJ
where it reversed and set aside the
resolution of the OCP and ordering

it to withdraw the information for


estafa. Respondent Abalos filed a
motion for reconsideration arguing
that
the
DOJ
should
have
dismissed the petition for review
outright contending that Sec 7 of
DOJ Circular no 70 mandates that
If an information has been filed in
court pursuant to the appealed
resolution the petition shall not be
given due course if the accused
had already been arraigned the
aggrieved party cannot file a
petition for review as the secretary
of Justice shall deny it outright.
ISSUE: Should DOJ dismiss the
the petition for review on the
ground that Adasa had already
been arraigned?
HELD: Yes. The settled rule is that
arraignment
of
the
accused
constitutes a waiver of the right of
the
accused
to
preliminary
investigation or reinvestigation.
Such waiver is tantamount to
finding of probabale cause. In this
case,
when
petitioner
unconditionally pleaded to the
charge, she effectively waived
reinvestigation of the case by the
prosecutor as well as the right to
appeal the result thereof to the
DOJ Secretary. Thus, with the
arraignment of the accused, the
DOJ Secretary can no longer
entertain appeal or petition for
review because petitioner had
already waived or abandoned the
same.
Zapatos v. People
411 SCRA 148, September 16,
2003

FACTS: Petitioner, a Community


Environment Natural Resources
Officer of DENR, was charged
before the RTC of the crime of
frustrated murder and murder
committed in relation to his
office in view of attempting to kill
one Socrates Platero and killing
Mayor Cortez, respectively. On
arraignment, petitioner pleaded
not guilty and the trial on the
merits then ensued. The private
prosecutor filed in the Court an
omnibus motion to dismiss on the
ground that RTC lacks jurisdiction
to try the case. RTC granted said
motion. In the current petition,
petitioner prays that the present
conviction
before
the
Sandiganbayan be dismissed on
the ground of double jeopardy.
ISSUE: Is there a double jeopardy
upon the convition of petitioner in
the Sandiganbayan?
HELD: A plea that is made in a
court without jurisdiction does not
give rise to double jeopardy.
Significantly, while petitioner had
already pleaded not guilty before
RTC, jeopardy did not attach as the
latter did not acquire jurisdiction.
Cabangbangan vs. Concepcion
95 Phil 87, May 26, 1954
FACTS: Petitioner was charged
with the crime of illegal possession
of firearms after having to himself
the possession of unlicensed homemade
firearm.
Later,
the
prosecution
amended
the
information by adding that said
unlicensed firearm was carried by
him to coerce one Antonio Pacuan.
When petitioner was arraigned,
the information read to him was
not the amended information. It

appears from the record that,


when the original information was
read to petitioner who pleaded not
guilty, his attorney pointed out to
the court that the arraignment was
not
under
the
amended
information yet the CFI Samar still
sentenced him of illegal possession
of firearm and ammunition. The
present action was brought by
petitioner questioning the validity
of the second information and
contended that his conviction is
erroneous for lack of arraignment
upon the amended information.
ISSUE: Is the conviction of
petitioner erroneous on the ground
of lack of arraignment upon the
amended information?
HELD: Yes, it is erroneous. Where
the accused has been already
arraigned and subsequently, the
information
was
substantially
amended, an arraignment on the
amended information is mandatory
because the accused has the
constitutional right to be informed
of the nature and cause of
accusation against him and more
so because the accused had
repeatedly called the attention of
the court to the absence of
arraignment. If he is not arraigned
and is convicted under the second
information,
the
conviction
constitutes reversible error.

Navarro Hultman. Later, the victim


died thus the private prosecutor
filed an omnibus motion for leave
of court to file the amended
information.
The
amended
information charges Teehankee of
murder. The trial court admitted
the
amended
information.
However during the arraignment,
petitioner refused to enter his plea
on the ground of lack of
preliminary investigation on the
new
charge.
On
behalf
of
petitioner, the judge entered a plea
of not guilty. Thereafter, the
prosecution presented its evidence
. The counsel of petitioner refused
to take part in the proceeding on
the same ground that there was no
preliminary investigation in the
amended information.
ISSUE: Should the amended
information be admitted despite
the
absence
of
preliminary
investigation thereon?
HELD: Yes. Where the amended
Information is only as to form,
there is no need for another
preliminary investigation and the
retaking of plea of the accused; in
substitution
of
information,
another preliminary investigation
is entailed and the accused is
required to plead anew to the new
information. It is evident that
frustrated murder is but a stage in
the execution of the crime of
murder; hence the former is
necessarily included in the latter.

Teehankee, Jr. v. Madayag


207 SCRA 134, March 6, 1992
FACTS: An information for the
crime of frustrated murder was
filed against Claudio Teehankee Jr.
allegedly committed to Maureen

People vs. Baetiong,


2 Phil 126, April 14, 1903
FACTS: Complaint charges the
defendant with the offense of
aiding and abetting a band of
brigands by supplying them with

food. Defendant alleges that the


facts charged in the information
were truebut that he supplied the
brigands with food through fear
that if he did not do so they would
kill him. Without hearing any
evidence, the court thereupon
found the defendant guilty and
sentenced him to ten years of
imprisonment.
ISSUE: What plea should be
entered for the defendant, guilty
or not guilty?
HELD: when the accused admits
the facts in the information but
alleges that he performed the acts
as charged because he feared for
his life, it is proper to enter a plea
of not guilty. What the defendant
stated when arraigned amounted
to
a
denial
of
criminal
responsibility, based on facts
which, he conceived, constituted a
lawful excuse for his conduct.
People vs Pua
415 SCRA 540, November 11,
2003
FACTS: It was laleged in the
Information that appellants carried
away feloniously one Jocelyn
Lariosa and deprived her of liberty
for the purpose of extorting
ransom of 1.5 Million Pesos as a
condition for the latters release.
To this charge, appellants entered
a plea of not guilty. After the trial,
the trial court rendered judgment
finding
appellants
guilty
as
charged. The present petition was
brought by appellants questioning
the validity of their arrest arguing
that they were arrested despite
absence of probable cause.

ISSUE: Considering that the


validity
of
the
arrest
was
questioned
only
after
RTC
rendered judgment, should the
petitioner
of
appellants
be
dismissed?
HELD: Yes. The accused by
entering a plea of not guilty,
submits himself to the jurisdiction
of the court, thereby curing any
defect of his arrest. In the present
case, the appellant did not
question the validity of his arrest
before
he
was
arraigned.
Moreover,
the
appellant
participated in the proceedings
before the trial court and even
adduced evidence on his behalf.
The appellant is thus precluded
from questioning his arrest and
the procedure therefor.
People vs. Madraga
344 SCRA 628, November 15,
2000
FACTS: Madraga was charged
with two counts of rape committed
against his own 16 year-old
daughter. During the arraignment,
Atty. Banico, counsel for the
accused, manifested that the
accused was willing to enter plea
of guilty to the first crime of rape
provided that the other rape case
be tried for another date. Thus, the
accused pleaded guilty to the first
charge of rape Trial on the merits
ensued in the first rape charged
and the RTC rendered judgement
convicting the accused of the
crime as charged.
Free Legal Assistance Group
(FLAG), et al. filed Appellants
Brief arguing that the accuseds
plea of guilty is null and void for
violating Section 3, Rule 116 which

requires the judge to conduct


search
inquiry
as
to
the
voluntariness
and
full
comprehension of the accuseds
plea of guilty to the capital
offence.
ISSUE: Is the plea of guilty valid?
HELD: No. If the accused pleaded
guilty but it can be inferred from
the arguments made by his
counsel and in his appeal brief that
the accused is asking that a
certain penalty be imposed upon
him in view of some mitigating
circumstances, this is a conditional
plea and is equivalent to a plea of
not guilty.

People vs. Magat


332 SCRA 517, May 31, 2000
FACTS: Two Informations of rape
were
filed
against
appellant
Magat.
Upon
arraignment,
appellant
pleaded
guilty
but
bargained to a lesser penalty for
each case. Complainants mother,
Ofelia Limpoco Magat, and the
public prosecutor, Rio Espiritu
agreed with the plea bargain.
ISSUE: Is the conditional plea/plea
bargaining valid?
HELD: No. It is the essence of a
plea of guilty that the accused
admits
absolutely
and
unconditionally
his
guilt
and
responsibility for the offense
imputed to him. Hence, the
accused may not foist a conditional
plea of guilty on the court by
admitting his guilt provided that a
certain penalty will be meted unto
him. Accused-appellants plea of
guilty is undoubtedly a conditional
plea. Hence, the trial court should

have vacated such a plea and


entered a plea of not guilty for a
conditional plea of guilty, or one
subject to a proviso that a certain
penalty be imposed upon him, is
equivalent to a plea of not guilty
and would, therefore, require a full
blown trial before judgement may
be rendered.
People vs Albert
251 SCRA 136, December 11, 1995
FACTS: When first arraigned for
the crime of murder for stabbing
Alfonso
Quiemen,
appellant
entered a plea of not guilty and
then the case went to trial. Later,
however, appellant admitted the
commission of the crime as
charged. He admitted that he
indeed stabbed Quiemen after he
lost his mind. In view of said
admission, the trial court ordered
that the plea be withdrawn and a
plea of guilty be recorded in lieu
thereof.
ISSUE: Is the plea of guilty
entered into valid?
HELD: No. A formal plea of not
guilty should properly be entered
if an accused admits the truth of
some or all the allegations of the
information,
but
interposes
excuses or additional facts which,
if duly established exempt or
relieve him in whole or in part of
criminal responsibility. The trial
court should have been sufficiently
forewarned that the sapient course
to take , since a plausible insanity
defines was being set up, was to
restore the former plea of not
guilty.
People vs. Balisacan

17 SCRA 1119, August 31, 1996


FACTS: Appellant Balisacan was
charged with homicide. During
arraignment, he entered a plea of
guilty and was assisted by counsel
in doing so. He then presented his
evidence and admitted that he
indeed stabbed the deceased but
in self defence. The trial court,
after trial on the merits ensued,
rendered judgment acquitting the
accused. Thus the prosecution
brought the present appeal.
ISSUE: Did the present appeal
placed the accused in double
jeopardy?
HELD: No. The existence of a valid
pleas an essential requisite of
double jeopardy. The accused had
first entered a plea of guilty but
however testified that he acted in
complete
self-defense.
Said
testimony had the effect of
vacating his plea of guilty and the
court a quo should have required
him to plead a new charge, or at
least direct that a new plea of not
guilty be entered for him. This was
not done. Therefore, there has
been no standing of plea during
the judgment of acquittal, so there
can be no double jeopardy with
respect to the appeal herein.

People vs Comendador
G.R. L-38000, September 19, 1980

FACTS: Appellant Comendador


was charged of robbery with
homicide
qualified
by
craft,
uninhabited place and abuse of
confidence
or
obvious
ungratefulness.
During
arraignment, he pleaded guilty
thereto and was sentenced to the
penalty of death. After being
convicted with the offense as
charged, appellant brought the
present appeal contending that he
should not be sentenced with the
capital penalty of death as the
aforementioned
qualifying
circumstances were not proven
during the trial. Thus he, argued
that
the
RTC
should
have
convicted him only with the lesser
penalty of reclusion perpetua.
ISSUE: Is the RTC correct in
imposing the penalty of death in
view
of
the
qualifying
circumstances?
HELD: No. As a rule, a plea of
guilty is a judicial confession of
guilt- an admission of all the
material facts alleged in the
information , including all the
aggravating
circumstance
mentioned therein. However, when
an accused who lacks instruction
pleads
guilty
to
parricide
described in the information as
having been committed with
aggravating
circumstances
of
treachery
and
evident
premeditation and his testimony
under oath before the trial court
fails to show the existence of such
aggravating circumstances, the
plea of guilty shall be understood
as being admission of having
committed the crime of parricide,
not having done so with treachery
and evident premeditation.

Daan v. Sandiganbayan,
G.R. Nos. 163972-77, March 28,
2008
FACTS:Appellant was charged for
three counts of malversation of public
funds by falsifying the time book and
payrolls for given period making it
appear that some laborers worked on
the construction of the new municipal
hall building of Bato, Leyte and
collected their respective salaries
thereon when, in truth and in fact,
they did not. In the falsification cases,
the accused offered to withdraw their
plea of not guilty and substitute the
same with a plea of guilty, provided,
the mitigating circumstances of
confession or plea of guilt and
voluntary
surrender
will
be
appreciated in their favor. In the
alternative, if such proposal is not
acceptable, said accused proposed
instead to substitute their plea of not
guilty to the crime of falsification of
public document by a public officer or
employee with a plea of guilty, but to
the lesser crime of falsification of a
public document by a private
individual. On the other hand, in the
malversation cases, the accused
offered to substitute their plea of not
guilty thereto with a plea of guilty, but
to the lesser crime of failure of an
accountable
officer
to
render
accounts.
The Sandiganbayan denied
petitioners Motion to Plea Bargain,
despite favorable recommendation by
the prosecution, on the main ground
that no cogent reason was presented
to justify its approval..
ISSUE:
Is
the
denial
of
Sandiganbayan
of
the
plea
bargaining agreement proper?

HELD: No. Plea bargaining in


criminal cases is a process where
the accused and the prosecution
work
a mutually satisfactory
disposition of the case subject to
the court approval. It usually
involves the defendants pleading
guilty to a lesser offense or to only
one or some of the counts of multi
count indictment in return for a
lighter sentence than that for the
graver charge.
People vs. Magat
G.R. No. 130026, May 31, 2000
FACTS: Two Informations for
incestuous rape were filed against
the
appellant
heren.
When
arraigned, appellant pleaded guilty
but bargained for a lesser offense
for each case. The mother of the
complainant together with the
prosecutor agreed and an order
was issued on the same day
imposing
ten
years
of
imprisonment for each case. Three
months later, the case was revived
at the instance of the complainant
on the ground that the penalty was
too light. Appellant was then rearraigned and he entered a plea of
not guilty. After two mnths again,
he entered anew a plea of guilty. In
view thereof, the court imposed
the penalty of death. He now
appeals on the ground that there
was double jeopardy upon rearraignment
in
the
same
information
ISSUE:
Is
there
a
double
jeopardy?
HELD: No. The first order issued
by the trial court is void on the
ground that accuseds plea is not
the plea bargaining contemplated

by law and the rules of procedure.


The only instance where a plea
bargaining is allowed under the
Rules is when the accused pleads
guilty to a lesser offense. Here the
reduction of the penalty is only a
consequence of the plea of guilt to
a lesser penalty. The appellant did
not plead to a lesser offense but
pleaded guilty to the rape charges
and only bargained for a lesser
penalty. When there is a plea of
guilty to a lesser offense and the
same was allowed by the court,
there is no need to amend the
information
or
complaint.
A
conviction under this plea shall be
equivalent to a conviction of the
offense charged for purposes of
double jeopardy.

People v. Ibaez
407 SCRA 406, July 30 2003
FACTS: An information for the
crime of murder and frustrated
murder was charged against
appellant. Upon being arraigned,
appellant, assisted by his counsel
de oficio, entered a plea of "guilty".
The prosecution was ordered to
adduce evidence as required by
the Rules of Court. Court held that
the
appellant
guilty
beyond
reasonable doubt of the crimes of
murder and frustrated murder and
sentenced
him to suffer the
penalty of death and reclusion
temporal, respectively.
Appellant now assail the
judgment of conviction arguing
that the RTC failed to consider the
mitigating
circumstances
of
voluntary
surrender,
voluntary
confession of guilt ad intoxication.

ISSUE:
Should
the
above
mentioned
mitigating
circumstances be appreciated?
HELD:A plea of guilty made after
arraignment and after trial had
begun does not entitle the accused
to have such plea considered as
mitigating.

People vs. Espidol


442 SCRA 360, November 12,
2004
FACTS:
An
Information
was
charged against appellant for
Robbery in Band with Homicide.
Apellant was arraigned and with
the assistance of his counsel, Atty.
Hermilo Barrios, he pleaded not
guilty
On 13 June 2001, shortly
after
the
prosecutions
third
witness was sworn in, appellant
changed his tune. Atty. Fatima
Vitamog, appellants new counsel,
manifested in open court about
appellants desire to change his
plea to guilty. Appellant was rearraigned, with the information
read to him in the local dialect. To
the charge of Robbery with
Homicide in a Band, he pleaded
guilty. hereat, the trial court
inquired from appellant if his new
counsel clearly explained to him
the legal effects and consequences
of his plea of guilty. He answered
in the affirmative. Without much
ado, the trial court deemed the
case submitted for decision and
rendered the assailed decision.
In light of the imposition of
the death penalty upon appellant,

the records of the case were


elevated
to
this
Court
for
automatic
review.
Appellant
argued in this review that the RTC
erred in failing to consider
appellants plea of guilty as
improvident.
ISSUE: Whether or not the plea of
guilty by appellant was validly
made.
HELD:The court must still require
the introduction of evidence for
the purpose of establishing the
guilt and degree of culpability of
the defendant because a plea of
guilty in only a supporting
evidence or secondary basis for
finding
of
culpability.
The
mandatory nature of the requisites
for a valid plea of guilty to a
capital offense is easily deducible
from the letter of the law.
Lamentably, the court a quo failed
to play the rules of the game.
In the case at bar, a cursory
look at the transcripts easily reveal
the fact that appellants plea of
guilty was far from spontaneous
and persistent as envisioned by
Section 3, Rule 116 of the Revised
Rules of Criminal Procedure. The
trial court not only failed to probe
into the spontaneity of appellants
plea, it made no effort to apprise
him of the impact of his change of
plea as the accused herein was
merely told that he could face the
penalty of reclusion perpetua to
death.

People vs. Francisco

G.R. No. 192818, November 17,


2010
FACTS: An Information for Murder
was filed against appellant after
killing one Ramil Tablate. During
arraignment, appellant pleaded
not guilty to the crime charged.
Consequently,
on
the
same
hearing, he was re-arraigned and
he pleaded this time as guilty to
the crime charged. It was shown
that RTS conducted searching
questions
to
ascertain
the
voluntariness of the plea and the
understanding of its consequences.
The RTC furthered ordered the
setting of the case for the
prosecution to adduce evidence to
prove the guilt of the accused
beyond reasonable doubt.
After admitting the death of
Ramil resulting frol appelants
assault, the defsne however did
not present any witness, but
simply argued that the offense
appellant is only homicide and not
murder.
Nonetheless,
RTC
convicted appellant of the crime of
murder.
Hence,
this
present
appeal.
ISSUE: Whether or not RTC erred
in convicting appellant of murder
instead of homicide.
HELD: Notwithstanding the plea
of guilty of the accused, evidence
must still adduced to to determine
the precise participation of the
accused in the perpetuation of the
capital
offense-whether
as
principal,
accomplice,
or
accessory- as well as the presence
of modifying circumstances. And
the accused may also present
evidence in his behalf either to
rebut the prosecutions evidence
or to show the presence of

mitigating circumstances. In the


present case however, the defense
chose not to present any evidence
which amounts to a waiver to
present evidence. This was not
objected by appellant.

People vs. Oscar Documento


G.R. No. 188706, March 17, 2010
FACTS: Appellant Documento was
charged before the RCT for two
counts of rape. He was accused of
raping his daughter AAA, the 16
years old. Upon arraignment,
Documento pleaded not guilty.
Later however, appellant changed
his plea to guilty. Thus, in
accordance with the Rules, RTC
ordered the re-arraignment of the
appellant and entered the plea of
guilt. Thereafter, the prosecution
presented its evidence consisting
of the testimonies of private
complainant herself, her mother
and Dr. Hugo. On the other hand,
appellant was the sole witness of
the defense. He argued that he did
not rape AAA and that to the
contrary they had consensual
sexual relationship. He further
alleged that he pleaded guilty to
the crime of rape only because
Prosecutor Salise convinced him to
do so.
ISSUE: Did the RTC err in failing
to conduct searching inquiry as to
the
voluntariness
and
full
comprehension of the accused?
HELD: Yes. . Nothing in the
records of the case at bench shows
that the trial court complied with
the guidelines set forth by the
Supreme Court in a number of

cases
after
appellants
rearraignment and guilty plea. The
questions propounded to appellant
during the direct and crossexamination likewise fall short of
these requirements. The appellant
was not fully apprised of the
consequences of his guilty plea. In
fact, as argued by appellant, "the
trial court should have informed
him that his plea of guilt would not
affect or reduce the imposable
penalty, which is death as he might
have erroneously believed that
under Article 63, the death
penalty, being a single indivisible
penalty, shall be applied by the
court regardless of any mitigating
circumstances that might have
attended the commission of the
deed." Moreover, the trial court
judge failed to inform appellant of
his right to adduce evidence
despite the guilty plea.
People v. Oden
G.R. no. 155511-22, April 14, 2004
FACTS: Appellant Mario Oden was
charged with twelve (12) counts of
rape before the Regional Trial
Court of Antipolo City. The
complainant is the Anna Oden, the
daughter of appellant. After the
prosecution had rested its case
with the testimony of its lone
witness (the private complainant),
Atty.
Harley
Padolina
(PAO)
manifested that the defense would
not present any evidence. The trial
court rendered a decision finding
appellant guilty beyond reasonable
doubt of twelve counts of rape.
Appellant asserts that his
plea
of
guilty
has
been
improvidently
made
on
the

mistaken belief that he would be


given a lighter penalty with his
plea of guilt.
ISSUE: Is the conviction of the
RTC proper?
HELD: Yes. The manner by which
the plea of guilty is made, whether
improvidently or not, loses much
great significance where the
conviction can be based on
independent evidence proving the
commission by the person accused
of the offense charged. In the
present case, trial on the merits
still ensued despite the plea of
guilt of the appellant and it was
proven therein that the accused
was indeed guilty of the crime as
charged.
MOTION TO QUASH
Condrada v. People
398 SCRA 482, February 28, 2003
FACTS: Petitioner was charged
with rape before the RTC of
Bolognan. When arraigned, he
pleaded not guilty to the charge
against him. On March 31, 1999,
the prosecution moved for the
postponement due to the absence
of the due to the absence of of the
complainant and her witnesses. On
the scheduled reset of hearing,
complainant and her witnesses
were still absent. The third time
that the trial was postponed,
petitioner
moved
for
the
temporary dismissal of the case
invoking his right to speedy trial
on June 2, 1999, the Court
issued a resolution reinstating the
case and ordered the issuance of
warrant
of
arrest.
Petitioner
opposed the reinstatement of the
case alleging that to do so will

violate his right against double


jeopardy.
ISSUE: Is there a violation of the
prohibition
against
double
jeopardy?
HELD: The proscription against
double jeopardy presupposes that
an accused has been previously
charged with an offence and the
case against him is terminated
either
by
his
acquittal
or
conviction, or dismissed in any
other manner without his consent.
When the criminal case against the
accused was dismissed, it was
merely provisional or temporary
without prejudice to the revival
thereof within 30 days from the
date of the dismissal. Thus, the
Court finds that the reinstatement
thereof did not place the petitioner
in double jeopardy. Thus, the
requirement that the dismissal of
the case must be without the
consent of the accused is not
present in this case.
People v. Ramirez,
G.R. Nos. 150079-80, April 17,
2001
FACTS: Appellant Ramirez was
charged with rape committed
against complaint Diana Pagaduan,
14 14 years old minor. Upon his
arraignment, he pleaded not guilty.
At the initial hearing on August 24,
2000, the prosecutor informed the
trial
court
of
an
amicable
settlement reached between the
parties as shown by the Affidavit of
Desistance executed by private
complainant. during the hearing,
private complainant affirmedd the
veracity of the Affidavit and that
she fully understands the contents

thereof. The same is also true with


the mother who likewise affirmed
the veracity of the Affidavit.
Notwithstanding
the
Affidavit
submitted
by
the
Prosecution, RTC proceeded to
hear the case and thereafter
convicted the acpeocused of the
crime as charged. In view of the
denial of motion to dismiss filed by
appellant, the present action was
brought.
ISSUE:
Should
the
criminal
charge
against
appellant
be
dismissed on the grounds of the
Affidavit of Desistance executed by
the complainant?
HELD:
No.
An
affidavit
of
desistance or pardon is not a
ground for the dismissal of an
action, once it has been instituted
in court. By itself, an affidavit of
desistance or pardon is not a
ground for the dismissal of an
action, once it has been instituted
in court.19 In the present case,
private complainant lost the right
or absolute privilege to decide
whether the rape charge should
proceed, because the case had
already
reached
and
must
therefore continue to be heard by
the court a quo.

People v. Salazar
G.R. no. 181900, August 11, 1997
FACTS: Two Informations were
filed before the RTC against
appellant charging him with two
counts of statutory rape of a 12year-old
girl.
During
his
arraignment, appellant pleaded
not
guilty.
Meanwhile,
AAA
purportedly executed an Affidavit

of Desistance wherein she stated


that she was not raped by
appellant and that she no longer
intends to pursue the cases filed
against appellant. during the
hearing, she explained that hew
own mother forced her to execute
the affidavit upon threat of harm.
Nonetheless, RTC rendered a
decision convicting the appellant
as charged
ISSUE: Should the Affidavit of
Desistance be given weight?
HELD: No. recantation or an
affidavit of desistance is viewed
with suspicion and reservation.
Jurisprudence
has
invariably
regarded
such
affidavit
as
exceedingly unreliable, because it
can be easily secured from a poor
and ignorant witness, usually
through
intimidation
or
for
monetary consideration. Moreover,
there is a probability that it would
later on be repudiated, and
criminal prosecution would thus be
interminable.
People v. Sandiganbayan
439 SCRA 390, September 29,
2004
FACTS: Luis Pabalan filed a
complaint in the Office of the
ombudsman against respondents
who are City Administrator of
Quezon City and Chairman of the
Evangelist Methodist Church in
QC. In his complaint, Pabalan
stated
that
respondents
encroached his lot after they
introduced
improvements
on
Pabalans lot. The improvements
were allegedly constructed without
the necessary building permit
Accordingly, the QC Assistant
Building Official ordered the

demolition of the structure. It


appeared however that respondent
city administratoror of QC recalled
the
demolition.
The
Graft
Investigator found probable cause
against
the
respondent
for
violation of Section 3(e) of
Republic (Rep.) Act No. 3019 and
recommended the filing of the
Information against him. The
Ombudsman
approved
the
recommendation and thereafter,
the charge was filed with the
Sandiganbayan.
Respondent
brought the present Motion to
quash information on the ground
of absence of probable cause
ISSUE: Is the motion to quash
proper?
HELD: No. To quash means to
annul, vacate or overthrow. The
absence of probable cause for the
issuance of a warrant of arrest is
not a ground for the quashal of the
information but is a ground for the
dismissal of the case.
Rodis v. Sandiganbayan
G.R. Nos. 71404-09, October 26,
1988
FACTS: Petitioner Rodis, former
president of PHILFINANCE, was
charged before the Sandiganbayan
with five counts of violation of
Anti-Graft and Corrupt Practices
Act. Later, petitioner filed a motion
to qush information as against him
on the ground of lack of
preliminary investigation with an
alternativee prayer that the issue
and/or enforcement of the warrant
of arrests as against him be held in
abeyancewhile
he
seeks
a
reinvestigation by the Tanodbayan
pursuant to his right of preliminary

investigation. This motion to


quash was however opposed by the
Prosecution on the grounds that
lack of preliminary investigation
was not among those grounds in a
motion to quash.
ISSUE: Is the motion to quash
proper?
HELD: No, The absence of a
preliminary
investigation
or
inability to participate in the
preliminary investigation is not a
proper ground for a motion to
quash but for a petition for
reinvestigation.

Lazarte v Sandiganbayan
G.R. No. 180122, October 26, 1988
FACTS: Feliciano Lazarte, an
engineer and chair of the National
Housing Authority (NHA) was
charged of violations of Anti-Graft
and corrupt and Practices Act
before the Sandiganbayan. In the
Information, it was alleged that he
used the public funds amounting to
230,000 to pay a construction
company for a project that is not
part of the plan (ghost project) in
Bacolod City. Petitionerr filed a
motion to quash on the ground
that the facts charged in the
information do not constitute
offense. This motion to quash
however was denied by the
Sandiganbayan.
Hence,
the
present petition.
ISSUE: Should the motion to
quash be granted?
HELD: No. When the ground
relied upon is that the facts
charged do not constitute an

offense, the determinative test is


appreciating a motion to quash
under Sec. 3(a) of Rule 117 is the
sufficiency of the averment in the
information, that is whether the
facts alleged, if hypothetically
admitted would establish the
essential elements of the offense
as
defined
by
law
without
considering matters aliunde. An
information needs only to state the
ultimate facts constituting the
offense , not the finer details of
why and how the illegal acts allege
amounted to undue injury or
damagematters
that
are
appropriate
for
trial.
The
information, in other words, must
allege clearly and accurately the
elements of the crime charged.
People vs. Tabio,
544 SCRA 156, February 6, 2008
FACTS: appellant was charged
with three counts of rape is a
single
Information.
Appellant
pleaded not guilty on arraignment
then trial on the merits ensued.
RTC handed dow the decision
finding appellant guilty of three
counts of rape. This decision was
affirmed by the CA.
ISSUE: Is the conviction valid
despite the duplicity of the
offences
charged
in
one
information?
HELD:Failure of the accused to
interpose an objection on the
ground of duplicity of the offense
charged
in
the
information
constitutes waiver.
Serena v. Sandiganbayan,
542 SCRA 225, January 22, 2008

FACTS: Petitioner was a senior


student of the UP Cebu. She was
then
appointed
by
President
Estrada as a student regent. She
was charged of violation of AntiGraft and Corrupt Practices Act in
view of the allegedly defrauding
the government in the expenses of
the renovation of the Vinzon Hall
of UP. Petitioner moved to quash
Information. She claimed that the
Sandiganbayan does not have any
jurisdiction
over
the
offense
charged in her capacity as student
regent of UP. sandiganbayan
however denied said motion to
quash. Hence, the present petition
for certiorari was brought before
the court on the ground that the
respondent court allegedly acted
with grave abuse of discretion.
ISSUE: Is the present petition for
certiorari proper?
HELD: No. Well established is the
rule that when a motion to quash
in a criminal case is denied, the
remedy is not a petition for
certiorari, but for petitioners to go
trial,
without
prejudice
to
reiterating the special defences
invoked in their motion to quash.
Go
v. Bangko sentral
ng
Pilipinas
G.R. No. 178429, October 23, 2009
FACTS:
An
Information
for
violation of General Banking Act
was filed against petitioner Go
before the RTC. Said accused was
then he President of Orient
Commercial Banking Corporation
borrowed
for
himself
as
representative of other companies
without the written approval of the
majority
of
the
Board
of
Directors.Upon arraignment, Go

pleaded not guilty to the offense


charged and before the trail could
commence, Go claimed that the
Information was defective, as the
facts charged therein allegedly do
not constitute an offense under the
General Banking Act.
ISSUE: Should RTC dismiss the
case on the ground that the
allegations in the Information are
defective?
HELD: Even if information be
defective
because
the
facts
charged do not constitute an
offense, the dismissal of the case
will not necessarily follow. The
prosecution should be given a
chance to correct the defect and
the court can order the dismissal
only upon the prosecutions failure
to do so. It would constitute an
arbitrary
exercise
of
power
correctible by certiorari if the trial
would not provide the prosecution
the opportunity to correct the
defect.The RTC;s failure to provide
the prosecution the opportunity
twice constitutes an arbitrary
exercise of power.

Co v. Lim
604 SCRA 702, October 30, 2009
FACTS:
The
NBI
raided
a
comercial
establishment
in
Masangkay St. , Manila. This raid
was anchored upon an information
relayed by the compalinant Co who
alleged that cell cards stolen from
her were being sold to said
establishment.After the Inquest
Proceedings were conducted, Lim
the seller of said cell cards, was
charged with violation od PD 1612

or the Anti Fencing Law. Later, the


RTC dismissed the case on the
ground that the Office of the City
prosecutor of manila and the DOJ
would not want to prosecute the
case. Complainant Co filed a
motion for reconsideration and the
same was denied. Hence, the
present appeal.
ISSUE: By the present appeal, is
the right of Lim against doubl
jeopardy not violated?
HELD: No it is not violated. THE
RTC categorically stated that the
defense counsel moved for the
dismissal of the case against
respondents. Verily, respondents,
through their counsel, had given
their express consent to the
termination of the case. Therefore,
the
requirement
of
double
jeopardy that the dismissal of the
case without his approval was not
met. Thus, the rule on double
jeopardy is not applicable to this
case
Caes v. IAC
179 SCRA 54, November 6, 1989
FACTS: Petitioner Caes was
charged
in
two
separate
informations
with
illegal
possession of firearms and illegal
possession
of
marijuana.
He
pleaded guilty upon arraignment .
There after the trial was postponed
and rescheduled couple of times.
In view of the repeated absence of
the prosecution witnesses, the
court
issued
a
provisional
dismissal of the case.
A motion to revive the cases
was filed by Maj. Dacanay (he had
been promoted in the meantime)
and Sgt. Lustado who alleged that

they could not attend the hearing


for lack of notice. said motion was
granted by the judge. In view
thereof, petitioner brought the
present action to question the
validity of the revival of the case
arguing that the same will violate
his right against double jeopardy
ISSUE: Is there a double jeopardy?
HELD: Yes. Notwithstanding that
the dismissal of the case was
described as provisional did not
change the nature of the dismissal,
that it was dismissed because of
the violation of the right of the
accused to speedy trial. In effect,
the complaint is not really of
dismissal but of acquittal.

Cruz v. CA
388 SCRA 72 , August 29, 2002
FACTS: The City Prosecutor
charge petitioner of Estafa thru
Fasification of Public Document.
This complaint was anchored from
the execution of petitioner An
Affidavit of Self Adjudication o a
parcel of land stating that she was
the sole heir of the registered
owner whine fact she knew there
were other surviveng heirs. Since
the offended party did not reserve
the right to file a civil action
arising from the criminal offence,
the civil action was deemed
instituted in the criminal case.
Trial on the merits ensued
and accordingly, the RCT rendered
judgment acquitting petitioner on
the ground of reasonable doubt.
Petitioner thereafter filed a motion
for reconsideration as to the civil
aspect of the case but this was to
no avail.

ISSUE: Can petitioner validly


appeal the civil aspect of the case?
HELD:A
reading
of
the
jurisprudence discloses the rule
that the acquittal of the accused
does not affect the right of the
accused to appeal the civil aspect
of the case. Hence, it has been
said that while judgment of
acquittal cannot be appealed by
prosecution, either the offended
party or the accused or both may
appeal the civil aspect of the
judgment despite the acquittal of
the accused,

Salazar v. People
411 SCRA 598 , November 6, 1989
FACTS: Petitioner Anamer Salazar
purchased 300 cavans of rice from
J.Y.
Brothers
Marketing
Corporation. As payment for these
caverns of rice, petitioner gave
private complainant a check drawn
against the Prudential Bank by one
Nena Timario in the amount of
214, 000. This check was accepted
by
J.Y.
Brother
Marketing
Corporation
upon
petitioners
assurance that it was a good
check. Upon presentment, the
check was dishonoured because it
was drawn under a close account.
Petitioner was informed of such
dishonour. thereafter, she replaced
the Prudential Bank Check with
another check drawn against Solid
Bank which however, was returned
with the word DAUD or drawn
against uncollected deposit. An
Information for Estafa was filed
against herein petitioners Salazar
and Timario. The trial court
rendered judgment acquitting the

petitioner of the crime charged but


ordered her to remit to private
complainant the amount of the
check
as
payment
for
her
purchase. Hence, the present
petition.
ISSUE: Does the acquittal of
petitioners
bar
the
private
complainant
herein
from
recovering the payment of the
caverns of rice?
HELD: No. While the prosecution
cannot appeal from a judgment of
acquittal as it would place the
accused in double jeopardy, the
aggrieved party who may be the
offended party or the accused
himself or both may appeal from
the judgment on the civil aspect of
the case.
Cudia v. CA
284 SCRA 173 , January 16, 1998
FACTS: City Prosecutor of Angeles
City filed an Information charging
petitioner of illegal possession of
firearms
and
ammunition.
Petitioner pleaded not guilty
thereto. However, the provincial
prosecutor of Pampanga also filed
an Information against him with
the
same
charge
of
illegal
possession
of
firearms
and
ammunitions. Petitioner filed a
motion to quash on the ground
that his continued prosecution for
the offence of illegal possession of
firearms and ammunition-for which
he had been arraigned and had
been
dismissed
despitee
his
opposition-would violate his right
against double jeopardy. The Court
of Appeals dismissed the motion to
quash. Hence, the present petition.
ISSUE: Is there a double jeopardy?

HELD: In order to successfully


invoke
double
jeopardy,
the
following
requisites
must
be
present:
(a) A first jeopardy must have
attached prior to the second;
(b)The first jeopardy must have
been validly terminated; and
(c) The second jeopardy must be
for the same offense or the
second offense includes or is
necessarily included in the
offense charged in the first
information, or is an attempt
to commit the same or a
frustration thereof.
In determining when the first
jeopardy may be said to have been
attached, it is necessary to prove
the following elements:
(a) The
accused
has
been
convicted or acquitted, or
the case against him was
dismissed
or
terminated
without his express consent;
(b)That the conviction, acquittal
or dismissal was made by a
court
of
competent
jurisdiction;
(c) There is a valid complaint or
information or other formal
charg sufficient in form and
substance
to
sustain
a
conviction;
(d)The accused has pleaded to
the charge;
(e) The subsequent offense is for
an offense which is the same
as the former complaint or
information
or
for
any
attempt of the same or

frustration thereof , or for


any
offense
which
necessarily includes or is
necessarily included in the
offense
charged
in
the
former
complaint
or
information.
People v. Joven de Grano
G.R. 167110, June 5, 2009
FACTS: An Information for murder
was charged against appellant de
Grano
together
with
others.
Duringg arraignment, appellant
pleaded not guilty to the crime
as charged. After the presentation
of the evidence, RCT rendered
decision finding appellant guilty of
murder.
Appellant then filed a motion
for reconsideration praying for
their
acquittal
in
the
aforementioned
RTC
decision.
Acting on said motion, RTC
reversed its decision acquitting
appellant. Petitioner timely filed a
Motion for Reconsideration for
said acquittal which the CA
denied.
Hence
the
present
petition.
ISSUE:
Is
there
a
double
jeopardy?
HELD: None. Any acquittal or
conviction before a court having
no jurisdiction would not violate
the principle of double jeopardy
since it failed to attach in the first
place. in the present case, RT
clearly exceeded its jurisdiction
when it entertained the motion for
reconsideration
of
the
respondents/appellants who were
al large. Thus, appellants were
never placed in double jeopardy
because, from the ver beginning,

the lower court had acted without


jurisdiction.
Zapatos v. People
411 SCRA 148, September 16,
2003
FACTS: Two Informations for
Murder (committed in relation to
office) and murder were filed with
the RTC . Before petitioner could
be arraigned, private prosecutor
moved to refer the case to
Sandiganbayan but was denied.
Petitioner was then arraigned and
pleaded not guilty to both charges
The Private prosecutor filed
with this court a petition for
certiorari questioning the order of
the RTC but the same was
dismissed. Later however, the RCT
issued an Omnibus Order granting
the motion an dimming the two
criminal
cases.
Thereafter,
Prosecution Officer De Llana filed
with the Sandiganbayan the above
two information.
ISSUE:
Is
there
a
double
jeopardy?
HELD:There can be no double
jeopardy
where
the
accused
entered a plea in a court that had
no jurisdiction. When appellant
entered his plea to the RTC, the
same is void as the proper court in
Sandiganbayan, the offence having
been committed in relation to
office. Thus, it ca n be said that
double jeopardy did not attach
when plea was made in the RTC,
the latter having no jurisdiction
over the case.
Grafton v. United States
206 U.S. 333, 1907

FACTS: Petitioner was a private


army of the United States, was
tried before a general court
martial for crime of homicide for
killing
a
Filipino,
Florentino
Castro. The accused pleaded not
guilty to sad charge.
Later,
the
prosecuting
attorney of the province of Iloilo
file a criminal complaint of
homicide. Trial on the merits
ensued. The CFI Iloilo rendered
judgment convicting petitioner of
homicide.
At
this
juncture,
petitioner invoked double jeopardy
arguing that his acquittal by the
court martial forbade him from
being tried in civi courts for the
same offence.
ISSUE:
Is
there
a
double
jeopardy?
HELD: Yes. The judgment of a
court-martial having jurisdiction to
try an officer or soldier for a crime
is entitled to the same finality and
conclusiveness as to the issues
involved as the judgment of a civil
courts
in
cases
within
its
jurisdiction. Grafton, having been
acquitted of the crime of homicide
by a military court-martial of
competent
jurisdiction
cannot
subsequently be tried for the same
offense in civil court exercising
authority in that territory.

Binay v. Sandiganbayan
316 SCRA 65
Topic: Double Jeopardy
FACTS: Two complaints were
raised against petitioners for
violation of Section 3(e) and (g) of
R.A. No. 3019 3019 (giving undue
favor to private parties) which
informations were filed before the
RTC of Batangas City. Another
complaint on the same matter was
eventually
filed
before
the
Sandiganbayan. Petitioner moved
to quash the Criminal Case filed
before the Sandiganbayan on the
grounds that the same complaints
has already been filed with the
RTC. The proceedings of both
cases were suspended by the
Sandiganbayan and the
RTC
pending resolution of the Binay
case as to the jurisdiction of the
Sandiganbayan.
ISSUE: WON the filing of the
information
before
the
Sandiganbayan constitutes double
jeopardy.
HELD: No. The court ruled that
the Sandiganbayan was not ousted
of its jurisdiction even if the
information was first filed in the
RTC since the latter did not have
jurisdiction in the first place as
provided in R.A. 7975. The court
has previously ruled that a filing of
a complaint with one court does
not prevent the plaintiff from filing
the same with the competent
court. This does not amount to

forum shopping since the only


authority of the first court was to
dismiss the case for lack of
jurisdiction. Also, there can be no
double
jeopardy
where
the
accused entered a plea in a court
that
had
no
jurisdiction. The
remedy of petitioners, therefore,
was not to move for the quashal of
the
information
pending
in
the Sandiganbayan on the ground
of double jeopardy. Their remedy
was to move for the quashal of the
information pending in the RTC on
the ground of lack of jurisdiction.

Cudia v. Court Of Apeals


284 SCRA 173
Topic: Double jeopardy
FACTS:
Renato
Cudia
was
arrested for the crime of Illegal
Possession
of
Firearms
and
Ammunition.
Upon
his
arraignment, the court called the
attention of the parties and
contrary
to
the
information,
Renatio Cudia had committed the
offense in Mabalacat and not in
Angeles City. Thus the judge
ordered that the case should be
assigned to a court involving
crimes committed outside Angeles
City consequently it was assigned
to Branch 56 of the Angeles City
RTC. However, the Provincial
Prosecutor of Pampanga filed an
information charging Renato Cudio
with the same crime and it was
likewise assigned to Branch 56 of
the Angeles City RTC which
resulted to two Informations with
the same crime. This prompted the
City Prosecutor to file a Motion to
Dismiss/ Withdraw the Information

which the trial court granted.


Renato filed a Motion to Quash the
criminal
case
filed
by
the
Provincial Prosecutor on the
ground
that
his
continued
prosecution for the offense of
illegal possession of firearms and
ammunition for which he had been
arraigned in the first criminal
case,
and
which
had
been
dismissed despite his opposition
would violate his right not to be
put
twice
in
jeopardy
of
punishment for the same offense.
ISSUE: WON the filing of the
second information violated the
right of the accused against double
jeopardy.
HELD: No. It is necessary that
there be a court of competent
jurisdiction, for jurisdiction to try
the case is essential to place an
accused in jeopardy. Venue in
criminal cases is jurisdictional,
being an essential element of
jurisdiction.
In
all
criminal
prosecutions, the action shall be
instituted and tried in the court of
the
municipality
or
territory
wherein
the
offense
was
committed or any one of the
essential ingredients thereof took
place. It is plainly apparent that
the City Prosecutor of Angeles City
had no authority to file the first
information, the offense having
been
committed
in
the
Municipality of Mabalacat, which
is beyond his jurisdiction. It is thus
the
Provincial
Prosecutor
of
Pampanga, not the City Prosecutor,
who should prepare informations
for offenses committed within
Pampanga but outside of Angeles
City. As the fiscal had no authority

to file the information, the


dismissal of the first information
would not be a bar to petitioner's
subsequent prosecution. As the
first
information
was
fatally
defective for lack of authority of
the officer filing it, the instant
petition must fail for failure to
comply with all the requisites
necessary
to
invoke
double
jeopardy.

Saldana v. Court Of Appeals


G.R. No. 88889 October 11,
1990
Topic: Double jeopardy
FACTS:
Petitioner
Marietta
Saldana was charged with the
crime of estafa. The trial court
issued
a
subpoena duces
tecum and ad
testificandum ordering
the
Manager of the Citytrust Banking
Corporation to appear and testify
on June 17, 1986. However, the
Bank Manager did not come to
court. When the trial was reset.
Another subpoena was served on
the Bank Manager but he again
failed to appear in court. On
August 8, 1986, the prosecution
received a copy of the accused's
"Motion
to
Consider
the
Prosecution to Have Rested Its
Case and to Dismiss the Case for
Insufficiency of Evidence." Later
on, the Court of Appeals granted
the petition of the People that the
latter now has its witness and
other evidence.
ISSUE: WON the Court of Appeals
erred in ordering the reopening of

the case, violating the rule on


double jeopardy.
HELD: No. The order of the Court
of Appeals reinstating the criminal
case for further hearing by the
trial court does not violate the rule
on double jeopardy. One of the
elements of double jeopardy is a
competent court. The trial court in
this case was ousted from its
jurisdiction when it violated the
right of the prosecution to due
process by aborting its right to
complete the presentation of its
evidence. Where the prosecution is
deprived of a fair opportunity to
prosecute and prove its case, its
right to due process is thereby
violated. The cardinal precept is
that where there is a violation of
basic constitutional rights, courts
are ousted of their jurisdiction.
Thus, the violation of the State's
light to due process raises a
serious jurisdictional issue which
cannot
be
glossed
over
or
disregarded at will. Where the
denial of the fundamental right of
due process is apparent, a decision
rendered in disregard of that right
is void for lack of jurisdiction.
Hence, the first jeopardy had not
been terminated. The remand of
the case for further hearing or trial
is merely a continuation of the first
jeopardy. It does not expose the
accused to a second jeopardy does
not expose the accused to a second
jeopardy.

People v. Hon. Sandiganbayan,


Abelardo P. Panlaqui, et al
G.R. No. 173396, September 22,
2010

Topic: Double jeopardy


FACTS: Private respondents were
charged in an Information for
Violation of Section 3(e) of
Republic Act (R.A.) No. 3019.
Private respondents were duly
arraigned on April 10, 1996,
pleading not guilty to the charge
against them. Thereafter, trial on
the merits ensued. On May 19,
2006, the Sandiganbayan rendered
the assailed Decision acquitting
the private respondents. The
People, represented by the Office
of the Ombudsman, through the
Office of the Special Prosecutor,
then filed the present petition
for certiorari.
ISUUE: WON the present petition
violated the right of the accused
against double jeopardy
HELD: Yes. As a rule, an acquittal
rendered by a court of competent
jurisdiction after trial on the
merits is immediately final and
cannot be appealed on the ground
of double jeopardy. There is no
showing that the trial court
hampered
the
prosecution's
presentation of evidence in any
way. Respondent trial court clearly
stated in its decision which pieces
of evidence led it to its conclusion
that the project was actually
undertaken, justifying payment to
the contractor. Clearly, petitioner
failed to show that there was
mistrial resulting in denial of due
process. There being no mistrial in
this case, the acquittal of private
respondents can no longer be
reviewed by the Court as this
would constitute a violation of the

constitutional right against double


jeopardy.
People of the Philippines v.
Dante Tan
G.R. No. 167526, July 26, 2010
Topic: Double jeopardy
FACTS: Two Informations for
violation of Rule 36 (a)-1, in
relation to Sections 32 (a)-1and
56 of the Revised Securities Act,
were filed by petitioner People of
the Philippines against respondent
Dante Tan in the Regional Trial
Court. After
arraignment,
respondent pleaded not guilty to
both
charges
and
the
trial
ensued. Thereafter,
petitioner
made its formal offer of evidence.
RTC admitted some of the exhibits
presented. Aggrieved, petitioner
filed a Motion for Reconsideration,
but it was denied by the RTC.
Respondent filed an Omnibus
Motion for Leave to File Demurrer
to Evidence and to admit the
attached Demurrer to Evidence
which was granted. Upon Petition
for Certiorari, the CA ruled that
the dismissal of a criminal action
by the grant of a Demurrer to
Evidence is one on the merits and
operates as an acquittal, for which
reason, the prosecution cannot
appeal therefrom as it would place
the accused in double jeopardy.

ISSUE:
WON
respondent
court
gravely
erred
in
precluding the people from

prosecuting its cases against


Dante Tan.
HELD: No. Petitioner was given
more than ample opportunity to
present its case as gleaned from
the factual antecedents which led
to the grant of respondents
demurrer. The only instance when
double jeopardy will not attach is
when the trial court acted with
grave
abuse
of
discretion
amounting to lack or excess of
jurisdiction, such as where the
prosecution
was
denied
the
opportunity to present its case or
where the trial was a sham.
However, while certiorari may be
availed of to correct an erroneous
acquittal, the petitioner in such an
extraordinary proceeding must
clearly demonstrate that the trial
court blatantly abused its authority
to a point so grave as to deprive it
of its very power to dispense
justice. While petitioner insists
that the RTC acted with grave
abuse of discretion, the Court finds
that none can be attributed to the
RTC. Consequently, the CA did not
err when it affirmed the assailed
Orders of the RTC. Court would be
violating the rules on double
jeopardy if the twin orders were to
be reviewed after a finding that
the CA did not commit any grave
abuse of discretion in granting the
demurrer to evidence.
People of the Philippines v.
The Sandiganbayan (Fourth

Division) and Alejandro A.


Villapando
G.R. No. 164185, July 23, 2008
Topic: Demurrer to Evidence
FACTS:
On February 4, 2000,
Solomon B. Maagad and Renato M.
Fernandez charged Villapando and
Tiape for violation of Article 244 of
the Revised Penal Code before the
Office of the Deputy Ombudsman
for Luzon. The complaint was
resolved against Villapando and
Tiape and the Information dated
March 19, 2002 charging the two
with violation of Article 244 of the
Revised Penal Code was filed with
the Sandiganbayan. After the
prosecution
rested
its
case,
Villapando moved for leave to file a
demurrer
to
evidence.
The
Sandiganbayan, Fourth Division
denied his motion but gave him
five days within which to inform
the court in writing whether he
will
nonetheless
submit
his
Demurrer
to
Evidence
for
resolution without leave of court.
Villapando
then
filed
a
Manifestation of Intent to File
Demurrer to Evidence, and was
given 15 days from receipt to file
his Demurrer to Evidence.
He
filed his Demurrer to Evidence on
October 28, 2003 which was
granted by the Sandiganbayan.
ISSUE: WON the Sandiganbayan,
Fourth Division, acted with grave
abuse of discretion amounting to
lack or excess of jurisdiction.
HELD: Yes. Although this Court
held
in
the
case
of People
v. Sandiganbayan that once a court
grants the demurrer to evidence,

such order amounts to an acquittal


and any further prosecution of the
accused
would
violate
the
constitutional
proscription
on
double jeopardy, this Court held in
the same case that such ruling on
the
matter
shall
not
be
disturbed in the absence of a grave
abuse of discretion. In this case,
the
Sandiganbayan,
Fourth
Division, in disregarding basic
rules of statutory construction,
acted
withgrave
abuse
of
discretion.
Its interpretation of
the term legal disqualification in
Article 244 of the Revised Penal
Code defies legal cogency. Further,
the
Sandiganbayan,
Fourth
Division
denied
Villapando's
Motion for Leave to File Demurrer
to Evidence yet accommodated
Villapando by giving him five days
within which to inform it in writing
whether he will submit his
demurrer
to
evidence
for
resolution without leave of court.
Notably, a judgment rendered with
grave abuse of discretion or
without due process is void, does
not exist in legal contemplation
and, thus, cannot be the source of
an acquittal. The Sandiganbayan,
Fourth Division having acted with
grave abuse of discretion in
disregarding the basic rules of
statutory construction resulting in
its decision granting Villapando's
Demurrer
to
Evidence
and
acquitting the latter, its decision
null and void.

People vs. Terrado


G.R. No. 148226, July 14, 2008
Topic: Topic: Certiorari

FACTS: Accused Joseph Terrado


was charged with Carnapping
under
Republic
Act
6538.
According to the Information, the
accused carted away a motorized
tricycle after threatening the
driver with a fan knife.
The
accused
was
arraigned
and
pleaded not guilty to the crime
charged. The defense claimed that
the accused merely borrowed the
tricycle from its driver Dalmacio.
However, when accused was about
to return the same, he hit a stone,
lost control of the tricycle and
bumped a tree. The defense did
not deny that the tricycle, when
returned, was damaged and, in
fact, the accused voluntarily paid
the amount of P8,000.00 as partial
remuneration for the repair of the
tricycle. The trial court acquitted
accused Terrado for failure of the
prosecution to establish intent to
take the tricycle and intent to gain
from the same. Thus, the court
held that the prosecution failed to
prove the guilt of the accused
beyond reasonable doubt.
The
prosecution filed a Motion for
Reconsideration which the trial
court denied.
Aggrieved, the
complainants come to this Court
via a Petition for Certiorari seeking
to annul and set aside the decision
ISSUE:
WON
the
public
respondent
in
rendering
the
questioned decision acted with
grave
abuse
of
discretion
amounting to lack of jurisdiction.

HELD:
No. Availment of the
remedy of certiorari to correct an
erroneous
acquittal
may
be
allowed in cases where petitioner
has clearly shown that the public
respondent
acted
without
jurisdiction or with grave abuse of
discretion amounting to lack or
excess
of
jurisdiction.
While
petitioner alleges grave abuse of
discretion amounting to lack or
excess
of
jurisdiction,
the
imputation is premised on the
averment that the trial court
reached its conclusions based on
speculation,
surmises
and
conjectures.
Specifically,
the
allegations
delve
on
the
misapprehension of facts by the
trial court.
As a rule, factual
matters
cannot
be
normally
inquired into by the Supreme
Court in a certiorari proceeding.
Verdicts of acquittal are to be
regarded as absolutely final and
irreviewable. The present recourse
is a petition for certiorari under
Rule 65.

People vs. Asis, G.R.


173089
August 25, 2010
Topic: Topic: Certiorari

No.

FACTS:
Abordo,
private
respondent, was charged with two
(2) counts of attempted murder
before the Regional Trial Court.
The trial court found no treachery
and evident premeditation. Thus,
the RTC held Abordo liable only for
Serious
Physical
Injuries
for

shooting Calvez and Less Serious


Physical Injuries with regard to
Majait. All three complainants
moved
for
a
reconsideration
regarding the civil aspect. On
October 24, 2005, the trial court
dismissed Majaits motion for
reconsideration
while
Calvezs
motion
to
withdraw
was
granted. On said date, the trial
court also dismissed Calvez appeal
for not bearing the conformity of
the Provincial Prosecutor. Petition
for certiorari was then filed but
was denied by the CA. According
to the appellate court, the filing of
the petition for certiorari was the
wrong remedy.
ISSUE: WON the court gravely
abused its discretion in dismissing
the petition
HELD: Yes.
A petition for
certiorari under Rule 65, not
appeal, is the remedy to question a
verdict of acquittal whether at the
trial court or at the appellate level.
In our jurisdiction, we adhere to
the finality-of-acquittal doctrine,
that is, a judgment of acquittal is
final and unappealable. The rule,
however, is not without exception.
In several cases, the Court has
entertained petitions for certiorari
questioning the acquittal of the
accused in, or the dismissals of,
criminal cases. What the OSG is
questioning
are
errors
of
judgment. This, however, cannot
be resolved without violating
Abordos
constitutionally
guaranteed right against double
jeopardy. An appellate court in a

petition for certiorari cannot


review a trial courts evaluation of
the evidence and factual findings.
Errors of judgment cannot be
raised in a Rule 65 petition as a
writ of certiorari can only correct
errors of jurisdiction or those
involving the commission of grave
abuse of discretion. Hence, the CA
clearly erred in dismissing the
petition for certiorari filed before
it by the OSG on the ground that it
was the wrong remedy.
People v. Louel Uy
471 SCRA 668
Topic: Double jeopardy
FACTS: An Information was filed
charging Uy and Panangin with
murder. When arraigned, both
accused entered a plea of not
guilty. When the prosecution
rested its case, Panangin, with
leave of court, filed a demurrer to
evidence on the ground that
when he executed his extrajudicial confession, his rights
under Sec. 12, Bill of Rights of
the Constitution were violated.
Uy, also with leave of court, filed
a
separate
demurrer
to
evidence eon the same ground.
Both were granted resulting in
their acquittal for murder due to
insufficiency of evidence.
ISSUE: WON the prosecution may
appeal the trial courts resolution
granting demurrer to evidence and
acquitting him without violating
the
constitutional
proscription
against double jeopardy

HELD: Yes. Dismissal of a criminal


case by the grant of demurrer to
evidence may not be appealed, for
to do so would be to place the
accused in double jeopardy. By
way of exception, a judgment of
acquittal in a criminal case may be
assailed in a petition for certiorari
under Rule 65 of the Rules of
Court upon clear showing by the
petitioner that the lower court, in
acquitting the accused, committed
not merely reversible errors of
judgment but also grave abuse of
discretion amounting to lack or
excess of jurisdiction or a denial of
due process, thus rendering the
assailed judgment void. This Court
finds that the trial court committed
not only gross reversible error of
judgment but also was actuated
with grave abuse of discretion,
exceeding the parameters of its
jurisdiction,
in
holding
that
Panangins
retracting
of
his
confession
shows
that
the
execution thereof was involuntary
and that in any event it was
inadmissible as it was a fruit of a
poisonous tree. The decision of the
trial court undoubtedly deprived
the prosecution of due process as
it was not given the opportunity to
check the veracity of Panangins
alleged retraction.
People v. Laguio, Jr.
518 SCRA 393
Topic: Double jeopardy
FACTS: The three (3) separate
Informations
filed
against
Lawrence C. Wang namely: (1)
violation of Dangerous Drugs Act);

(2) violation of Illegal Possession of


Firearms; and (3) violation of
COMELEC Gun Ban. Wang filed his
undated
Demurrer
to
Evidence, praying for his acquittal
and the dismissal of the three (3)
cases against him for lack of a
valid arrest and search warrants
and the inadmissibility of the
prosecutions evidence against
him. The respondent judge, issued
the herein assailed Resolution
granting Wangs Demurrer to
Evidence and acquitting him of all
charges for lack of evidence.
Petitioner filed a petition for
review on certiorari to nullify and
set aside the said Resolution.
ISSUE: WON the prosecution may
appeal the trial courts resolution
granting Wangs demurrer to
evidence and acquitting him of all
the charges against him without
violating
the
constitutional
proscription
against
double
jeopardy

the right of the accused against


double jeopardy is not violated.
However, while certiorari may be
availed of to correct an erroneous
acquittal, the petitioner in such an
extraordinary proceeding must
clearly demonstrate that the trial
court blatantly abused its authority
to a point so grave as to deprive it
of its very power to dispense
justice. The trial court in finding
that
the
warrantless
arrest
preceded the warrantless search in
the case at bar, the trial court
granted
private
respondent's
demurrer
to
evidence
and
acquitted him of all the three
charges for lack of evidence,
because
the
unlawful
arrest
resulted in the inadmissibility of
the evidence gathered from an
invalid warrantless search. Hence,
there was no grave abuse of
discretion
in
acquitting
the
accused. Further review of the
judgment of acquittal would put
the accused in double jeopardy.

HELD: No. An order granting an


accuseds demurrer to evidence is
a resolution of the case on the
merits, and it amounts to an
acquittal. Generally, any further
prosecution of the accused after an
acquittal
would
violate
the
constitutional
proscription
on
double jeopardy. To this general
rule, however, the Court has
previously made some exceptions.
When the order of dismissal is
annulled or set aside by an
appellate court in an original
special civil action via certiorari,

People vs. De Grano


G.R. No. 167710, June 5, 2009
Topic: Double jeopardy
FACTS: On November 28, 1991,
an
Information
for
murder
committed
against
Emmanuel
Mendoza was filed with the
Regional
Trial
Court
against
against Joven de Grano, Armando
de Grano, and Estanislao Lacaba,
together with their co-accused
Leonides
Landicho,
Domingo
Landicho, and Leonardo Genil,
who were at-large. The RTC then
found them guilty of charge.

Respondents, thru counsel, then


filed
a
Joint
Motion
for
Reconsideration.
RTC
then
acquitted Joven and Armando, and
downgrading the conviction of
Domingo and Estanislao from
murder to homicide. Estanislao
filed a Notice of Appeal, while the
prosecution
sought
reconsideration of the Order. RTC
issued an Order denying the
motion and giving due course to
Estanislaos notice of appeal.
Subsequently, the CA denied
Peoples motion as it would place
the accused in double jeopardy.
ISSUE: WON court of appeals
committed reversible error and
grave abuse of discretion when it
dismissed
the
petition
for
certiorari on the ground of double
jeopardy.
HELD: Yes. A judgment of
acquittal in a criminal case may be
assailed in a petition for certiorari
under Rule 65 of the Rules of
Court upon a showing by the
petitioner that the lower court, in
acquitting the accused, committed
not only reversible errors of
judgment, but also grave abuse of
discretion amounting to lack or
excess of jurisdiction, or a denial
of due process, thus rendering the
assailed
judgment
void.
Consequently, the accused cannot
be considered at risk of double
jeopardy.
When the Decision was
promulgated,
only
Estanislao

Lacaba was present. The RTC


clearly exceeded its jurisdiction
when it entertained the joint
Motion for Reconsideration with
respect to the respondents who
were at large. It should have
considered the joint motion as a
motion for reconsideration that
was
solely
filed
by
Estanislao. Thus, Joven, Armando,
and Domingo, were not placed in
double jeopardy because, from the
very beginning, the lower tribunal
had acted without jurisdiction.
However,
with
respect
to
Estanislao, the RTC committed no
reversible
error
when
it
entertained
the
Motion
for
Reconsideration and for him alone,
the proscription against double
jeopardy applies.
Summerville General
Merchandising & Co., Inc., Vs.
Hon. Antonio M. Eugenio
G.R. No. 163741, August 7,
2007
Topic: Double jeopardy
FACTS: a complaint for unfair
competition filed by petitioner
against private respondents Elidad
Kho, Violeta Kho, and Roger Kho
for selling facial cream products
which give and depict the general
appearance of the Chin Chun Su
facial cream products and likely
influence the purchasers to believe
that the same are those of the said
Summerville. The DOJ Resolution
was dismissed. The prosecution
filed with the trial court a Motion
to Withdraw Information. Private
respondents, on the other hand,
filed a Motion for Reconsideration,

arguing that the trial court has all


the facts necessary to resolve the
pending incidents and insisted that
the case be dismissed on the
ground of double jeopardy. Later
on, the trial court held that the refiling of the Information would
constitute double jeopardy.
ISSUE: WON revival of the case or
the
reinstatement
of
the
Information
would
constitute
double jeopardy.
HELD: No. The court have held
that the Order granting the
withdrawal of the Information was
committed with grave abuse of
discretion, then the accused was
not acquitted nor was there a valid
and legal dismissal or termination
of the case. There was a violation
of the complainants right to due
process and a grave abuse of
discretion for failure of the trial
court
to
conduct
its
own
determination of a prima facie
case, to independently evaluate
and asses the merits of the case
and simply adopted the resolution
of the Secretary of Justice. Ergo,
the fourth requisite on the
conviction and acquittal of the
accused in the dismissal of the
case, without the approval of the
accused, was not met. Thus,
double jeopardy has not set in.

Tandoc v. Resultan
175 SCRA 37
Topic: Double jeopardy

FACTS:
A
Petition
for certiorari was filed assailing
the
Orders finding
reasonable
ground to believe that petitioners
had
probably
committed
the
crimes of "Trespass to Dwelling",
"Serious Physical Injuries", "Less
Serious Physical Injuries" and
"Grave Threats, and the Order
denying petitioners' motion for a
re-investigation of the complaint.
ISSUE: WON the city court has
the power and authority to conduct
anew a preliminary examination of
charges without violating the right
against double jeopardy.
HELD: Yes. The result of a
preliminary
investigation
can
neither constitute nor give rise to
the defense of double jeopardy in
any
case,
because
such
preliminary investigation is not
and does not in itself constitute a
trial
or
even
any
part
thereof. Preliminary investigation
is merely inquisitorial, and it is
often
the
only
means
of
discovering the persons who may
be reasonably charged with a
crime, to enable the fiscal to
prepare
his
complaint
or
information. It has no purpose
except
that
of
determining
whether
a
crime
has
been
committed and whether there is
probable cause to believe that the
accused is guilty thereof, and it
does not place the person against
whom it is taken in jeopardy. In the
case at bar, the offenses charged
against petitioners for "Trespass to
Dwelling", "Grave Threats" and
"Physical Injuries" were all within

the jurisdiction of the City Court of


San Carlos City. The preliminary
investigation proper conducted by
the Office of the City Fiscal could
have been dispensed with. Neither
did the earlier order of dismissal of
the complaints by the investigating
fiscal bar the filing of said
complaints with the city court on
the ground of double jeopardy.

Torres v. Aguinaldo
G.R. No. 164268, June 28, 2005
Topic: Double jeopardy
FACTS: Spouses Edgardo and
Nelia Aguinaldo filed a complaint
against petitioner Artemio T.
Torres, Jr. for falsification of public
document. Finding probable cause,
the OCP recommended the filing of
an information for falsification of
public document against Torres
which was filed before the MTC.
Torres moved for reconsideration
but was denied. On appeal, the
Secretary of Justice reversed the
findings
of
the
investigating
prosecutor
and
ordered
the
withdrawal of the information.
The motion for reconsideration
filed by Aguinaldo was denied. A
Motion to Withdraw Information
was filed which the MTC granted
on June 11, 2003. Meanwhile,
Aguinaldo filed before the Court of
Appeals a petition for certiorari
which was granted in the assailed
decision dated March 22, 2004.
Torres motion for reconsideration
was denied. Torres contends that
the order granting the withdrawal
of the information rendered moot
the petition for certiorari filed

before the Court of Appeals and


insists that an order dismissing a
case without prejudice is final if no
motion for reconsideration or
appeal therefrom is timely filed.
ISSUE: WON the resolution of the
Secretary of Justice puts the
accused in jeopardy
HELD:
No.
Preliminary
investigation
is
executive
in
character. It does not contemplate
a judicial function. It is essentially
an inquisitorial proceeding, and
often,
the
only
means
of
ascertaining
who
may
be
reasonably charged with a crime.
It is not a trial on the merits and it
does not place the person against
whom it is taken in jeopardy.
Likewise, a motion to withdraw
information differs from a motion
to dismiss. While both put an end
to an action filed in court, their
legal effect varies. In the case at
bar,
a
motion
to
withdraw
information was filed and not a
motion to dismiss.

Aurelio G. Icasiano, Jr., vs. Hon.


Sandiganbayan
G.R. No. 95642 May 28, 1992
Topic: Double jeopardy
FACTS: Respondent judge issued
from two orders of detention
against complainant Magbago for
contempt of court because of her
continued refusal to comply with
a fifth alias writ of execution.
Thereafter,
administrative
complaint was then filed against
the said judge for grave abuse of
authority, manifest partiality and
incompetence. However, this was

dismissed by the Supreme Court


for lack of merit. Thereafter,
Magbago also filed with the Office
of the Ombudsman the same
letter-complaint for violation of the
Anti-Graft and Corrupt Practices
Act.
Special
Prosecutor
recommended dismissal of the
complaint for lack of merit which
was approved by the Special
Prosecutor/Tanodbayan. Later, an
information against petitioner was
filed with the Sandiganbayan.
After said information was filed,
petitioner (as accused) filed a
motion for reinvestigation which
resulted in the issuance of two (2)
separate resolutions from the
respondent Sandiganbayan which
was denied. Petitioner then moved
to quash the information on the
ground that the accused shall be
placed in double jeopardy but this
was also denied.
ISSUE: WON the right against
double jeopardy is violated.
HELD: No. Double jeopardy does
not
apply
in
the
present
controversy because the Supreme
Court case (against the herein
petitioner) was administrative in
character
while
the
Sandiganbayan case also against
said petitioner is criminal in
nature. Moreover, double jeopardy
does not apply in a controversy
when the case in the Supreme
Court
was
administrative
in
character
while
the
Sandiganbayan is criminal in
nature. Also, elements in double

jeopardy do not apply vis-a-vis the


administrative case. In any case,
the dismissal by the Tanodbayan of
the first complaint cannot bar the
present prosecution, since double
jeopardy does not apply in a
preliminary
investigation.
A
preliminary investigation is not a
trial to which double jeopardy
attaches.

Vincoy v. Court Of Appeals


432 SCRA 36
Topic: Double jeopardy
FACTS: Accused George Vincoy
was charged with estafa. It was
alleged that Vincoy, by means of
deceit
and
false
pretenses,
represented to complainants Lizah
Cimafranca and Rolando Flores
that he could mobilize 30 dump
trucks and 2 payloaders for the
complainants use, subject to the
payment of P600,000 mobilization
fund.
However,
Vincoy
misappropriated the money given
and converted such for his own
personal use and benefit. In May
1996, Cimafranca filed a complaint
for estafa with the Office of the
City Prosecutor of Pasay City.
However, it was dismissed on the
ground that Vincoys obligation
was purely civil in nature and for
complainant's failure to attend the
hearings. Thereafter, Cimafranca,
joined by Flores, re-filed the
complaint charging the same
offense against petitioner. The trial
judge found Vincoy guilty beyond
reasonable doubt of the crime of
estafa under Article 315. He then

appealed before the Court


Appeals, but to no avail.

of

ISSUE: WON the dismissal of a


case
during
its
preliminary
investigation give rise to a claim of
double jeopardy.
HELD: No. The dismissal of a
similar complaint for estafa filed
by Lizah Cimafranca before the
City Prosecutor's Office of Pasay
City will not exculpate the
petitioner. The case cannot bar
petitioner's prosecution. It is
settled that the dismissal of a case
during
its
preliminary
investigation does not constitute
double jeopardy. A preliminary
investigation is not part of the trial
and is not the occasion for the full
and exhaustive display of the
parties' evidence but only such as
may engender a well-grounded
belief that an offense has been
committed and accused is probably
guilty thereof. For this reason, it
cannot be considered equivalent to
a
judicial
pronouncement
of
acquittal.

Trinidad vs. Office of The


Ombudsman
G.R. No. 166038, December 4,
2007
Topic: Double jeopardy
FACTS:
The
Office
of
the
Ombudsman
filed
two Informations against
petitioner with the Sandiganbayan
for violation of Section 3(j) and
Section 3(e) of the Anti-Graft and
Corrupt
Practices
Act in
connection
with
the Ninoy Aquino International

Airport International Passenger


Terminal III Project of the DOTC.
During
the pendency of
the
petition, the Sandiganbayan found
no probable cause to proceed with
the trial in, and thus dismissed
Criminal Case No. 28093 by
Resolution ofSeptember 7, 2006,
and
denied
the
prosecutions
motion for reconsideration by
Resolution of February 28, 2007.
ISSUE:
WON
res judicata is
applicable in criminal proceedings.
HELD: No. Res judicata is a
doctrine of civil law and thus has
no
bearing
on
criminal
proceedings.
But
even
if
petitioners argument were to be
expanded
to
contemplate res judicata in prison
grey or the criminal law concept of
double jeopardy, the Court still
finds it inapplicable to bar the
reinvestigation conducted by the
Office of the Ombudsman. For the
dismissal
of
a
case
during
preliminary investigation does not
constitute
double
jeopardy,
preliminary investigation not being
part of the trial. The Ombudsman
is not precluded from ordering
another review of a complaint, for
he or she may revoke, repeal or
abrogate the acts or previous
rulings of a predecessor in office.
Joel
B.
Caes
vs.
Hon.
Intermediate Appellate Court
G.R. Nos. 74989-90 November
6, 1989
Topic: Double jeopardy

FACTS: On November 21, 1981,


petitioner Joel Caes was charged
in two separate Informations with
illegal possession of firearms and
illegal possession of marijuana.
The trial was reset and was
postponed 11 times. On November
14, 1983, the prosecution moved
for the provisional dismissal of the
case because its witnesses had not
appeared. On January 9, 1984, a
motion to revive the cases was
filed by Maj. Dacanay and Sgt.
Lustado who alleged that they
could not attend the hearing
scheduled on November 14, 1983,
for lack of notice.
ISSUE: WON the revival of the
cases would place the petitioner
double jeopardy
HELD: Yes. The trial judge erred
in ordering the revival of the cases
against the petitioner and that the
respondent court also erred in
affirming that order. Caes having
been denied his constitutional
right to a speedy trial, and not
having expressly consented to the
"provisional" dismissal of the cases
against him, he was entitled to
their final dismissal under the
constitutional prohibition against
double jeopardy.The circumstance
that the dismissal of the cases
against
the
petitioner
was
described by the trial judge as
"provisional" did not change the
nature of that dismissal. As it was
based on the "lack of interest" of
the prosecutor and the consequent
delay in the trial of the cases, it

was final and operated as an


acquittal of the accused on the
merits. Thus, trial court is then
precluded to revive the case. In res
judicata in prison grey, the right
against double jeopardy prohibits
the prosecution of a person for a
crime of which he has been
previously acquitted or convicted.
The purpose is to set the effects of
the first prosecution forever at
rest, assuring the accused that he
shall not thereafter be subjected to
the danger and anxiety of a second
charge against him for the same
offense.
Fe Cayao-Lasam vs. Spouses
Claro and Editha Ramolete
G.R. No. 159132, December 18,
2008
Topic: Double jeopardy
FACTS: Editha and her husband
Claro Ramolete (respondents) filed
a Complaint for Gross Negligence
and Malpractice against petitioner
before the PRC. Respondents
alleged that Edithas hysterectomy
was
caused
by
petitioners
unmitigated
negligence
and
professional
incompetence
in
conducting the D&C procedure
and the petitioners failure to
remove the fetus inside Edithas
womb. The Board of Medicine of
the
PRC
rendered
a
Decision, exonerating
petitioner
from the charges filed against her.
On appeal, the PRC reversed the
findings of the Board and revoked
petitioners authority or license to
practice her profession as a
physician. Petitioner now claims that

the law does not allow complainants


to appeal to the PRC from the
decision of the Board and to do so is
double jeopardy.
ISSUE: WON appeal to the PRC
from the decision of the Board
constitutes double jeopardy
HELD: No. The principle of double
jeopardy finds no application in
administrative
cases.
Double
jeopardy attaches only: (1) upon a
valid indictment; (2) before a
competent
court;
(3)
after
arraignment; (4) when a valid plea
has been entered; and (5) when
the defendant was acquitted or
convicted, or the case was
dismissed or otherwise terminated
without the express consent of the
accused. These elements cannot be
present in a proceeding before the
Board
of
Medicine,
as
the
proceedings involved in the latter
are administrative and not criminal
in nature.
Office of The Ombudsman vs.
Medrano
G.R. No.177580, October 17,
2008
Topic: Affidavit of desistance
FACTS: Ma. Ruby A. Dumalaog,
filed before the petitioner Office of
the Ombudsman a sworn lettercomplaint against her superior
herein-respondent
Victorio
N.
Medrano for violation of AntiSexual Harassment Act of 1995
(criminal
case),
and
grave
misconduct (administrative case).
The Ombudsman rendered its

decision with the administrative


case and found Medrano guilty of
grave misconduct. Upon petition,
the CA annulled Ombudsmans
decision in the administrative case
and dismissed the complaint on
the sole ground that Ombudsman
has no jurisdiction over it. The
Ombudsman filed a motion for
reconsideration
of
the
CAs
decision but was denied. Petitioner
opposes
respondents
move,
contending
that
Ma.
Rubys
Affidavit of Desistance and the
dismissal of the criminal case do
not constitute legal bases for
dismissing the present petition and
the administrative complaint.
ISSUE: WON petition has become
moot and academic by virtue of the
affidavit of desistance and the
criminal
case
having
been
dismissed due to her lack of
interest to prosecute the same
HELD: The Court stressed the
distinct and independent character
of the remedies available to an
offended
party
against
any
impropriety
or
wrongdoing
committed by a public officer. The
dismissal in a criminal case does
not result in the dismissal of the
administrative case as they are
different remedies. The settled
rule is that criminal and civil cases
are altogether different from
administrative matters, such that
the first two will not inevitably
govern or affect the third and vice
versa. Likewise, an affidavit of
desistance (or recantation) is, as a

rule, viewed with suspicion and


reservation because it can easily
be secured from a poor and
ignorant witness, usually through
intimidation or for monetary
consideration.
Hence,
such
desistance, by itself, is not usually
a ground for the dismissal of an
action once it has been instituted
in court.
Cudia v. Court Of Apeals
284 SCRA 173
Topic: Double jeopardy
FACTS:
Renato
Cudia
was
arrested for the crime of Illegal
Possession
of
Firearms
and
Ammunition.
Upon
his
arraignment, the court called the
attention of the parties and
contrary
to
the
information,
Renatio Cudia had committed the
offense in Mabalacat and not in
Angeles City. Thus the judge
ordered that the case should be
assigned to a court involving
crimes committed outside Angeles
City consequently it was assigned
to Branch 56 of the Angeles City
RTC. However, the Provincial
Prosecutor of Pampanga filed an
information charging Renato Cudio
with the same crime and it was
likewise assigned to Branch 56 of
the Angeles City RTC which
resulted to two Informations with
the same crime. This prompted the
City Prosecutor to file a Motion to
Dismiss/ Withdraw the Information
which the trial court granted.
Renato filed a Motion to Quash the
criminal
case
filed
by
the
Provincial Prosecutor on the
ground
that
his
continued

prosecution for the offense of


illegal possession of firearms and
ammunition for which he had been
arraigned in the first criminal
case,
and
which
had
been
dismissed despite his opposition
would violate his right not to be
put
twice
in
jeopardy
of
punishment for the same offense.
ISSUE: WON the filing of the
second information violated the
right of the accused against double
jeopardy.
HELD:
No. Information, when
required to be filed by a public
prosecuting officer, cannot be filed
by another. The court does not
acquire jurisdiction rendering the
former
information
defective.
Jeopardy does not attach where a
defendant pleads guilty to a
defective
indictment
that
is
voluntarily
dismissed
by
the
prosecution. It is plainly apparent
that the City Prosecutor of Angeles
City had no authority to file the
first information, the offense
having been committed in the
Municipality of Mabalacat, which
is beyond his jurisdiction. It is thus
the
Provincial
Prosecutor
of
Pampanga, not the City Prosecutor,
who should prepare informations
for offenses committed within
Pampanga but outside of Angeles
City. As the fiscal had no authority
to file the information, the
dismissal of the first information
would not be a bar to petitioner's
subsequent prosecution. As the
first
information
was
fatally
defective for lack of authority of
the officer filing it, the instant
petition must fail for failure to
comply with all the requisites

necessary
jeopardy.

to

invoke

double

Miranda vs. Tuliao


G.R. No. 158763, March 31,
2006
Topic: Double jeopardy
FACTS: Upon discovery of two
burnt cadavers in Isabela, two
informations for murder were filed
against SPO1 Wilfredo Leao,
SPO1 Ferdinand Marzan, SPO1
Ruben B. Agustin, SPO2 Alexander
Micu, SPO2 Rodel Maderal, and
SPO4 Emilio Ramirez in the
Regional Trial Court. Presiding
Judge Tumalian issued warrant of
arrest against the petitioners and
SPO2 Maderal. Petitioners filed an
urgent
motion
to
complete
preliminary
investigation,
to
reinvestigate, and to recall or
quash the warrant of arrest.
Notwithstanding the TRO granted
by Supreme Court, Judge Anghad
issued a Joint Order dismissing the
information against the petition.
The CA rendered the assailed
decision granting the petition and
ordering the reinstatement of the
criminal cases in the RTC as well
as the issuance of warrant of
arrest.
ISSUE: WON there is double
jeopardy in the reinstatement of a
criminal case dismissed before
arraignment
HELD: No. There is no double
jeopardy in the reinstatement of a
criminal case dismissed before

arraignment. In any case, the


reinstatement of a criminal case
dismissed before arraignment does
not constitute double jeopardy.
Double jeopardy cannot be invoked
where the accused has not been
arraigned and it was upon his
express motion that the case was
dismissed.

People v. Magat
G.R. No. 130026, May 31, 2000
Topic: Double jeopardy

Facts: Accused was charged with


the
crime
of
rape.
Upon
arraignment, A pleaded guilty but
bargained for a lesser offense, to
which complainants mother and
the prosecutor agreed. After three
months, the cases were revived at
the instance of the complainant on
the ground that the penalty was
too light. Appellant was rearraigned and he entered a plea of
not guilty. Two months later, he
entered anew a plea of guilty. The
court then imposed the penalty of
death. He now appeals on the
ground that there was double
jeopardy upon the re-arraignment
and trial on the same information.

Issue: WON judgment rendered


by a trial court which is based on a
void plea puts the accused in
jeopardy.

Held: No. Here, accused did not


plead to a lesser offense but
pleaded guilty to the rape charges
and only bargained for a lesser
penalty. It is the essence of a plea
of guilty that that the accused
admits
absolutely
and
unconditionally hid guilt and
responsibility for the offense
imputed to him. The judgment
rendered by a trial court which is
based on a void plea bargaining is
void ab initio and cannot be
considered to have attained finality
for the simple reason that a void
judgment has no legality from its
inception. By
reason
of
this,
jeopardy will not lie. A conviction
under a plea to guilty to a lesser
offense shall be equivalent to a
conviction of the offense charged
for purposes of double jeopardy.
The procedural infirmity in the
arraignment of the accused was
rectified when he was re-arraigned
and entered a new plea. He did not
question the procedural errors in
the first arraignment and having
failed to do so, waived the errors
in procedure.

People v. Balisacan
G.R. No. L-26376, August 31,
1966
Topic: Double jeopardy
Facts: Aurelio
Balisacan
was
charged with homicide in the CFI
of Ilocos Norte. Upon being
arraigned, he entered into a plea
of guilty. In doing so, he was

assisted y counsel. At his counsel


de officio, he was allowed to
present evidence and consequently
testified that he stabbed the
deceased
in
self-defense.
In
addition, he stated that he
surrendered himself voluntarily to
the police authorities. On the basis
of the testimony of the accused, he
was
acquitted.
Thus,
the
prosecution appealed.
Issue: WON the appeal placed the
accused
in
double
jeopardy.
Held: The Supreme Court held
that it is settled that the existence
of plea is an essential requisite to
double jeopardy. The accused had
first entered a plea of guilty but
however testified that he acted in
complete
self-defense.
Said
testimony had the effect of
vacating his plea of guilty and the
court a quo should have required
him to plead a new charge, or at
least direct that a new plea of not
guilty be entered for him. This was
not done. There having been no
standing plea at the time the
court rendered its judgment of
acquittal, there can be no double
jeopardy with respect to the
appeal of the prosecution. In
deciding the case upon the merits
without the requisite trial, the
court a quo not only erred in
procedure
but
deprived
the
prosecution of its day in court and
right to be heard.

People v. Ng Pek
81 Phil. 563
Topic: Plea
FACTS: In the Court of First
Instance of Manila appellant was
accused of, and pleaded guilty to,
attempted bribery. Forthwith he
was sentenced to suffer two
months and one day ofarresto
mayor and to pay a fine of P3, with
subsidiary imprisonment in case of
insolvency, and to pay the costs.c
From that sentence he appealed to
this Court, contending that (1) "the
lower court erred in considering
that the statement given by the
accused during his arraignment to
the court interpreter that he gave
a peso to the complaining witness
is a manifestation or a declaration
of a plea of guilty," and (2) "the
lower court erred in forthwith
sentencing the accused to a prison
term of two months and one day
and to pay a fine of P3 on the day
of the arraignment, without giving
him a chance to defend himself."
ISSUE: WON accused can still
invoke his right to be assisted by a
counsel
HELD: No. A plea of guilty is an
unconditional admission of guilt
with respect to the offense
charged. The record shows that
when the case was called for the
arraignment of the accused on
November 3, 1947, the accused
waived his right to be assisted by
counsel and then and there
entered the plea of guilty. That
plea necessarily foreclosed the
right of the accused to defend

himself and left the court with no


other alternative than to impose
the penalty prescribed by law.
Therefore the second assignment
of error is also devoid of merit.

Tangan Vs. People


155 SCRA 435
Topic: Double jeopardy
FACTS: Tangan was charged with
the crime of murder with the use
of an unlicensed firearm. After a
reinvestigation,
however,
the
information was amended to
homicide with the use of a licensed
firearm,and he was separately
charged with illegal possession of
unlicensed
firearm.On
arraignment, Tangan entered a
plea of not guilty in the homicide
case, but moved to quash the
information for illegal possession
of unlicensed firearm on various
grounds. The motion to quash was
denied, whereupon he filed a
petition for certiorari with this
Court.On November 5, 1987, said
petition was dismissed and the
joint trial of the two cases was
ordered.
ISSUE:
WON
the
petition
constitutes a violation of Tangan's
right against double jeopardy.
HELD: Yes. While the mere filing
of two informations or complaints
charging the same offenses does
not yet afford the accused in those
cases the occasion to complain
that he is being placed in jeopardy
twice for the same offense, for the
simple reason that the primary

basis of the defense of double


jeopardy is that the accused has
already
been
convicted
or
acquitted in the first case or that
the same has been terminated
without his express consent. In
the present case, the Solicitor
General's petition for certiorari
under Rule 65, praying that no
mitigating
circumstance
be
appreciated in favor of accusedappellant and that the penalty
imposed
on
him
be
correspondingly
increased,
constitutes a violation of Tangan's
right against double jeopardy and
should be dismissed.
People V. Court Of Appeals
G.R. No. 159261, February 21,
2007
Topic: Double jeopardy
FACTS: The Regional Trial Court
found Galicia and Ureta guilty for
the crime of homicide with the
aggravating
circumstance
of
dwelling which was of-setted by
the mitigating circumstance of
sufficient
provocation.
Galicia
appealed his conviction for the
death of Abenir. He averred that
the trial court erred in finding him
guilty of homicide with Ureta,
considering that the prosecution
failed to prove that they conspired
to
kill
Abenir,
and
the
prosecutions evidence showed
that it was only Ureta who stabbed
Abenir. The Court of Appeals
acquitted him of the crime
charged. The Office of the Solicitor
General (OSG) now asserts that
the CA erred in aquitting Galicia
as conspiracy was apparent.

ISSUE: WON the petition would


constitute double jeopardy.
HELD: Yes. A verdict of acquittal
is immediately final and a reexamination of the merits of such
acquittal, even in the appellate
courts, will put the accused in
jeopardy for the same offense. The
State
represented
by
the
prosecution had not been deprived
of a fair opportunity to prove its
case. Also, there has been no
grave abuse of discretion by the
CA as they failed to show that the
court abused its authority to a
point so grave as to deprive it of
its very power to dispense justice.
The finality-of-acquittal doctrine
has several avowed purposes.
Primarily, it prevents the State
from using its criminal processes
as an instrument of harassment to
wear out the accused by a
multitude
of
cases
with
accumulated trials. It also serves
the
additional
purpose
of
precluding the State, following an
acquittal,
from
successively
retrying the defendant in the hope
of securing a conviction. And
finally, it prevents the State,
following conviction, from retrying
the defendant again in the hope of
securing a greater penalty.

Galman v. Sandiganbayan
G.R. No. 72670 September 12,
1986
Topic: Double jeopardy
FACTS:
An
investigating
committee
was
created
to
determine the facts on the case

involving the assassination of


Ninoy Aquino. The office of the
Tanod
Bayan
was
originally
preparing a resolution charging
the 26 military accused as
principal to the crime against
Aquino but was recalled upon the
intervention of President Marcos
who insist on the innocence of the
accused.
Marcos
however
recommended the filing of murder
charge and to implement the
acquittal as planned so that double
jeopardy may be invoked later on.
The petitioners filed an action for
miscarriage of justice against the
Sandiganbayan and gross violation
of constitutional rights of the
petitioners for failure to exert
genuine efforts in allowing the
prosecution
to
present
vital
documentary evidence and prayed
for nullifying the bias proceedings
before the Sandiganbayan and
ordering a re-trial before an
impartial tribunal.
ISSUE: WON the right against
double jeopardy has been violated.
HELD: No. The prosecution was
deprived of due process and fair
opportunity to prosecute and prove
their case which grossly violates
the due process clause. When the
lower court who rendered the
judgment of aquittal is not
competent as it is ousted of its
jurisdiction when it violated the
right of the prosecution to due
process, this in effect, the first
jeopardy is never terminated, and
the remand of the criminal case for
further hearing and/or trial before
the lower courts amounts merely

to a continuation of the first


jeopardy, and does not expose the
accused to a second jeopardy. The
denial
on
the
motion
for
reconsideration of the petitioners
by the court was set aside and
rendered the decision of acquittal
of the accused null and void. An
order for a re-trial was granted.

Pacoy v. Judge Cajigal


534 SCRA 338
Topic: Double jeopardy
FACTS:
An
Information
for
Homicide was filed in the RTC
against Petitioner Jose M. Pacoy.
Upon arraignment, petitioner, duly
assisted by counsel de parte,
pleaded not guilty to the charge of
Homicide. However, on the same
day and after the arraignment, the
respondent judge issued another
Order
directing
the
trial
prosecutor to correct and amend
the Information to Murder in view
of the aggravating circumstance of
disregard of rank alleged in the
Information
which
public
respondent registered as having
qualified the crime to Murder.
Acting upon such Order, the
prosecutor entered his amendment
by crossing out the word Homicide
and instead wrote the word
Murder in the caption and in the
opening
paragraph
of
the
Information.
ISSUE: WON
constitutes a

the amendment
dismissal which

would place the accused in double


jeopardy.
HELD: No. There is no dismissal
of the homicide case when it is the
same original information that is
amended by merely crossing out
the word "Homicide" and writing
the word "Murder." If the accused
is neither convicted nor acquitted;
nor was the case against him
dismissed or otherwise terminated
without his express consent, there
can be no double jeopardy. For the
dismissal to be a bar under the
jeopardy clause, it must have the
effect of acquittal. While the
respondent
judge
erroneously
thought that disrespect on account
of rank qualified the crime to
murder, as the same was only a
generic aggravating circumstance,
the court do not find that he
committed any grave abuse of
discretion
in
ordering
the
amendment of the Information
after
petitioner
had
already
pleaded not guilty to the charge of
Homicide.

committed
on
the
territorial
jurisdiction of the said court.
ISSUE: WON the appeal of the
provincial prosecutor would put
the accused Salico under double
jeopardy

People v. Salico
84 Phil. 722
Topic: Double jeopardy

HELD: No. The court ruled on the


negative, for the following reasons:
First, because by the dismissal of
the case by the court below upon
motion of the defendant, the latter
has not been in jeopardy; Second,
because the
appeal by
the
prosecution in the present case
would not place the defendant in
double
jeopardy;
and
Third,
because assuming arguendo that
the defendant had been already in
jeopardy in the court below and
would be placed in double
jeopardy by the appeal, the
defendant
has
waived
his
constitutional right not to be put in
danger of being convicted twice
for the same offense. For double
jeopardy to operate, the dismissal
must be without the express
consent of the accused. If the
accused moved for the dismissal
alleging that the crime was not
committed within the territorial
jurisdiction of the court, and the
same was not dismissed, the
dismissal is with express consent
and cannot be a basis for claim of
double jeopardy.

FACTS: This case is a petition


assailing the decision of the CFI of
Negros Occidental dismissing the
case of Oscar Salico who is being
charged with homicide on the
ground that the prosecutor failed
to establish that the crime was

Caniza V. People
G.R. No. L-53776 March 18,
1988
Topic: Double jeopardy
FACTS: An Information was filed
with the Court of First Instance of

Manila for falsification of public


documents allegedly committed by
petitioner Caiza. Caiza then filed
a Motion to Quash contending that
the allegations in the information
did not constitute an offense.
Petitioner Caiza also contended
that the information contained
averments which, if true, would
constitute a legal excuse or
justification. The Motion to Quash
was then granted and the trial
court dismissed the case against
petitioner Caiza. Subsequently, a
second Information was filed
charging petitioner Caiza with
substantially the same offense as
that charged under the previous
information. Caniza again moved
to quash the second Information
on the grounds that (1) the offense
charged therein
had already
prescribed, (2) quashal of the first
Information had been on the
merits, and (3) the allegations in
the second Information did not
constitute an offense.
ISSUE: WON the dismissal on the
first information puts the accused
in jeopardy
HELD:
No.
As
previously
mentioned, Criminal Case No.
16879 was ordered dismissed by
the trial court with the express
consent of the accused i.e., upon
Motion to Quash filed by petitioner
Caiza. Generally, a dismissal
under such circumstance win not
bar another prosecution for the
same offense; the defendant, in
having the case against him

dismissed, thereby waives his


constitutional right against double
jeopardy for the reason that he
effectively prevents the trial court
from proceeding to trial on the
merits and rendering a judgment
of conviction against him.The
grounds, upon which the trial
court anchored its 2 Order of
dismissal, are clearly directed at
the sufficiency of said information
to sustain the conviction of
petitioner Caniza and, hence,
indicate the absence of the first
requisite
in
double-jeopardy.
Consequently, petitioner Caizas
plea of second jeopardy cannot be
sustained: he effectively waived his
right to assert that plea when he
moved
to
quash
the
first
Information filed against him.

People v. Vergara
G.R. Nos. 101557-58. April 28,
1993
Topic: Double jeopardy
FACTS: Respondent Judge, upon
motion of the Provincial Fiscal,
ordered
without
notice
and
hearing the dismissal of Crim.
Cases Nos. 7396 and 7397 both for
frustrated
murder,
which
thereafter were reinstated upon
initiative of the Secretary of
Justice and docketed anew as
Crim: Cases Nos. 8572 and 8573.
On 13 May 1991, after pleading
"not
guilty"
to
the
new
Informations, the accused moved
to quash on the ground of double

jeopardy, which was opposed by


the Office of the Provincial
Prosecutor. Petitioners
contend
that the filing of the two (2) new
Informations
did
not
place
accused-private respondents in
double
jeopardy
since
the
dismissal of the previous cases was
made with the latter's express
consent, which can be equated
with
their
motion
for
reinvestigation of the cases
ISSUE: WON the dismissal puts
the accused in jeopardy
HELD: Yes. It is undisputed that
valid Informations for frustrated
murder, i.e., Crim. Cases Nos.
7396 and 7397 were filed against
private respondents before the
Regional
Trial,
a
court
of
competent
jurisdiction.
It
is
likewise admitted that private
respondents, after being properly
arraigned, entered a plea of not
guilty. The only question then
remaining is whether the cases
against them were dismissed with
their express consent. This is
hardly what private respondents
gave. What they did was merely to
move for reinvestigation of the
case before the prosecutor. To
equate this with express consent of
the accused to the dismissal of the
case in the lower court is to strain
the meaning of "express consent"
too far. The consent to be deemed
a express one should be positive,
direct, unequivocal, and requiring
no inference or implication to
supply its meaning for such
consent
to
preclude
another
prosecution. Simply, there was no
express consent of the accused
when the prosecutor moved for the

dismissal
of
Informations.

the

original

Andres v. Cacdac
L-45650, March 29, 1982
Topic: Double jeopardy
FACTS: An information was filed
with the Court of First Instance of
Cagayan, Aparri Branch, docketed
therein as Criminal Case No. 3844A, charging the herein petitioners
Cresencio Andres and Proceso
Guimmayen, and Ladislao Tacipit
with the crime of Murder. The trial
was postponed several times.
Thereafter,
the
case
was
provisionally dismissed. More than
seven years after the provisional
dismissal of the case, a new
information
charging
a
new
Cresencio
Andres,
Proceso
Guimmayen, and Ladislao Tacipit
for the murder of Teofilo Ramos
was filed before the Circuit
Criminal Court. The petitioners
claim that the provisional dismissal
predicated on the right of a
defendant to a speedy trial and on
the failure of the Government to
prosecute, amounts to an acquittal
and bars their second prosecution
for the same offense upon the
ground of double jeopardy.
ISSUE: WON the petitioners right
against
double
jeopardy
is
violated.
HELD: No. It must be noted that
the accused and their counsel not
only asked for, but also consented
to, the provisional dismissal of the
case. Their act operates as a
waiver of their defense of double

jeopardy in the second prosecution


for the same offense. When a
criminal case is dismissed upon
the
application
and
express
consent of the accused and his
counsel, the dismissal is not a bar
to another prosecution for the
same offense because his action in
having
the
case
dismissed
constitutes a waiver of his
constitutional prerogative against
double jeopardy as he thereby
prevents
the
court
from
proceeding to the trial on the
merits and rendering judgment of
conviction against him.

Almario v. Court Of Appeals


G.R. No. 127772. March 22,
2001
Topic: Double jeopardy

FACTS: Petitioner is one of the


accused in criminal case for estafa
thru
falsification
of
public
document, and criminal case for
estafa, with respondent RCBC as
the offended party in both cases.
Hearings were cancelled several
times. Later on, when the case was
again called for hearing, private
complainant was not in Court
despite notice. Atty. Alabastro,
counsel for
accused Roberto
Almario, moved that the case
against the latter be dismissed for
failure
to
prosecute
and
considering
that
accused
is
entitled to a speedy trial. Upon
motion, the dismissal was set aside
and the court held that it did not

result in the acquittal of the


accused since the right of the
accused to speedy trial has not
been violated, and its dismissal
having been made upon the motion
of the accused, there is no double
jeopardy.
ISSUE: WON there is a violation
of the right against double
jeopardy.
HELD: No. As observed by
respondent appellate court, delay
in
the
trial
was
due
to
circumstances beyond the control
of the parties and of the trial court.
The first and third postponements
were clearly justified on the
ground of lack of notice to
accused,
co-accused,
and/or
counsel.
Another
was
made
without objection from petitioner's
counsel. There
was
no
unreasonable
delay
of
the
proceedings is apparent from the
chronology of the hearings with
the
reasons
for
their
postponements
or
transfers.
Petitioner could not refute the
appellate court's findings that
petitioner's right to speedy trial
had not been violated. Hence,
there
was
no
violation
of
petitioner's right to speedy trial as
there
were
no
unjustified
postponements
which
had
prolonged
the
trial
for
unreasonable lengths of time.
Where the right of the accused to
speedy trial had not been violated,
there was no reason to support the
initial order of dismissal. It follows

that petitioner cannot invoke the


constitutional right against double
jeopardy when that order was
reconsidered seasonably. For as
petitioners right to speedy trial
was
not
transgressed,
this
exception to the fifth element of
double jeopardy was not met.

People v. Espinosa
G.R. Nos. 153714-20, August
15, 2003
Topic: Double jeopardy
FACTS: Separate cases of estafa
and attempted corruption of public
officers were filed before the SBN
by the Office of the Ombudsman
against
Respondent
Espinosa. Prior to his arraignment,
Espinosa filed a Motion for
Reinvestigation of the cases which
was granted. While the cases were
being reevaluated, Espinosa filed
with the SBN a Motion for Leave
to Travel Abroad. Court then
required private respondent to be
"conditionally arraigned on that
date before it would act on his
Motion to Travel. Subsequently,
the OMB moved to withdraw ex
parte the two cases against private
respondent. The SBN granted the
Motion. Thereafter, the OMB filed
in
the
same
court
seven
Informations for Malversation of
Public Funds against Espinosa and
several others. Espinosa filed a
Motion to Quash the Informations
arguing that double jeopardy had
already attached. Petitioner on the

other hand, countered that the


arraignment for the two previous
cases was "conditional," because it
was made solely for the purpose of
accommodating
private
respondent's request to travel
abroad while the matters were
pending reinvestigation.
ISSUE: WON double jeopardy had
already attached.
HELD: No.
Dismissal secured
without the express consent of an
accused, does not amount to a
waiver of the right against double
jeopardy. A
waiver
of
the
constitutional right against double
jeopardy
must
be
clear,
categorical,
knowing
and
intelligent. Corollary to this rule,
the alleged conditions attached to
an
arraignment
must
be
unmistakable, express, informed
and enlightened. Otherwise, the
plea should be deemed to be
simple and unconditional. The
dismissal of the estafa and the
corruption cases was made upon
petitioner's ex parte Motion for the
withdrawal of the Informations.
Petitioner does not dispute the fact
that private respondent was not
notified of this Motion. Neither
was a hearing held thereon. Also,
the records reveal that the lawyer
for respondent Espinosa was
present when the April 19,1999
Order of the Fourth Division was
issued in open court.

People v. Declaro
G.R. No. L-64362, February 9,
1989
Topic: Double jeopardy
FACTS:
Edgar
Ibabao
was
charged for slight physical injuries
through reckless imprudence in a
complaint that was filed on
September
5,
1980
in
the
Municipal
Circuit
Court
of
Malinao,
Aklan
with
Crispin
Conanan was the offended party.
An information for serious physical
injuries
through
reckless
imprudence was filed against the
same accused in the Regional Trial
Court of Aklan with Eduardo
Salido as the offended party. This
second case arose from the same
incident. The first case was then
dismissed for the reason that both
the offended party and the
prosecuting fiscal, failed to appear
at the scheduled hearing despite
due notice. In the meanwhile,
considering that the said case had
been dismissed, counsel for the
accused filed a motion to dismiss
the second case on the ground that
the dismissal of the prior case is a
bar to the prosecution of the latter.
The trial court dismissed the
second case on the ground of
double jeopardy.
ISSUE: WON double jeopardy has
set-in
HELD: No. Double jeopardy will
apply even if the dismissal is made
with the express consent of the
accused, or upon his own motion,
only if it is predicated on either of

two grounds, i.e., insufficiency of


the evidence or denial of the right
to a speedy trial. In both cases, the
dismissal will have the effect of an
acquittal. Since the dismissal in
this case does not fall under either
of these two instances and it was
made with the express consent of
the accused, it would not thereby
be a bar to another prosecution for
the same offense. In the instant
case, the complaining witness and
the prosecutor failed to appear
only in the first hearing. Even if
the court did not dismiss the case
but merely postponed the hearing
to another date, there would not
have been a denial of the right of
the accused to a speedy trial. The
right of the accused to have a
speedy trial is violated when
unjustified postponements of the
trial are asked for and secured, or
when, without good cause or
justifiable motive, a long period of
time is allowed to elapse without
his case being tried. None of said
situations exists in the present
case.
Condrada v. People
G.R. No. 141646, February 28,
2003
Topic: Double jeopardy
Facts: Petitioner was charged with
rape. When he was arraigned on
February 26, 1999, petitioner
pleaded not guilty to the charge
against him. Later, the prosecution
filed several motions to postpone
the trial. On the last motion, trial
court issued an order temporarily
dismissing
the
case.
The

prosecution filed a Motion for


Reinstatement and/or Revival of
Criminal which was subsequently
granted. Petitioner opposed and
filed a motion for reconsideration
of said resolution insisting that the
reinstatement of the case will
place him in double jeopardy.
Hence, petitioner now filed this
instant petition and claims that
Criminal Case No. 10770 cannot
be revived because the dismissal
of the case is permanent in
character, having been made in
consideration of his right to speedy
trial
Issue:
Whether or not the
reinstatement of criminal case
places the petitioner in double
jeopardy.
Held: No. It is clear from the
records that the dismissal ordered
by the trial court was a temporary
dismissal of the case, and not a
permanent
dismissal
on
the
ground that the right of the
accused to speedy trial had been
violated by the delay in the
prosecution of the said case. The
trial court even expressly stated
that the same was subject to
reinstatement within thirty days
from the date of the temporary
dismissal. Thus, the Court finds
that the reinstatement thereof on
June 25, 1999 did not place
petitioner in double jeopardy.
However, there are two exceptions
and double jeopardy may attach
even if the dismissal of the case
was with the consent of the

accused: first, when there is


insufficiency
of
evidence
to
support the charge against him;
and second, where there has been
an unreasonable delay in the
proceedings, in violation of the
accuseds right to speedy trial. The
grant of a demurrer to evidence on
the ground of insufficiency of
evidence is, by jurisprudence,
deemed
acquittal
and
would
preclude the filing of another
information or an appeal by
prosecution. However, this is not
present in the instant case since
the court merely temporarily
dismissed the case.
Esmena v. Pogoy
G.R. No. L-54110, February 20,
1981
Topic: Double jeopardy
FACTS: Petitioners Esmea and
Alba were charged with grave
coercion for allegedly forcing Fr.
Thomas Tibudan to withdraw a
sum of money worth P5000 from
the bank to be given to them
because the priest lost in a game
of chance. No trial came in after
the arraignment due to the priests
request to move it on another date.
Sometime later Judge Pogoy issued
an
order
setting
the
trial
Aug.16,1979
but
the
fiscal
informed the court that it received
a telegram stating that the
complainant was sick. The accused
invoked their right to speedy trial.
Respondent judge dismissed the
case. After 27 days the fiscal filed
a motion to revive the case and
attached the medical certificate of

the priest proving the fact that the


priest was indeed sick of influenza.
On Oct.24,1979, accused Esmea
and Alba filed a motion to dismiss
the case on the ground of double
jeopardy.
ISSUE: Whether or Not the revival
of the case would place the
accused in double jeopardy
HELD: Yes. The dismissal of a
criminal case upon motion of the
accused because the prosecution
was not prepared for trial since
the complainant and his witnesses
did not appear at the trial is a
dismissal equivalent to an acquittal
that would bar further prosecution
of the defendant for the same
offense. In the present case, it was
evidently shown that the accused
invoked their right to a speedy
trial and asked for the trial of the
case and not its termination which
would mean that respondents had
no expressed consent to the
dismissal of the case which would
make the case filed res judicata
and has been dismissed by the
competent court in order to
protect the respondents as well for
their right to speedy trial which
will be equivalent to acquittal of
the respondents which would be a
bar to further prosecution.

Esguerra vs. De La Costa


66 Phil. 134
Topic: Double jeopardy

FACTS: Chief of police filed a


complaint against the petitioner
charging
him
with
having
committed the crime of abuse of
chastity. The case was dismissed
on the ground that the compliant
was not signed by the offended
party or her parents. Subsequently,
Raymundo Zapanta, father of the
alleged
offended
party,
filed
another complaint in the justice of
the peace court of Cainta against
the same petitioner charging him
with the same offense. The said
complaint was also dismissed. The
offended party again lodged a
complaint in the justice of peace
court of Cainta charging the
petitioner with the commission of
the same offense. The petitioner
now contends that he is in double
jeopardy, that he has been
deprived of his right to a speedy
trial, and that he is being
unjustifiably persecuted by the
alleged offended party.
ISSUE: WON petitioners right
against double jeopardy has been
violated
HELD: Yes. Petitioner was placed
in double jeopardy in connection
with the complaint and information
filed in criminal case No. 10546
(last case) of the Court of First
Instance of Rizal, which precludes
his prosecution anew for the crime
of abuse of chastity allegedly
committed. The petitioner had
already made his plea of "not
guilty" to that complaint as well as
to the information files by the
fiscal in the Court of First Instance
based
upon
said
complaint,
wherefore, upon the dismissal of
criminal case No. 10198 by the

Court of First Instance, upon the


ground that the complaint was
defective because signed by the
father of the offended party and
not by the latter, the petitioner was
placed in jeopardy and he cannot
again be prosecuted for the same
facts
constituting
the
same
offense. Thus, even if the accused
after invoking the right to a speedy
trial, moved for the dismissal of
the case and, therefore consented
to it, the dismissal would still place
them in jeopardy.

People v. Sandiganbayan
G.R. No. 164577, July 5, 2010
Topic: Double jeopardy
FACTS: Private respondents were
charged with having violated
Section 3(e) of Anti-Graft and
Corrupt Practices Act before the
Sandiganbayan.
Before
the
arraignment, the accused filed a
Motion for Leave of Court to File
Motion for Reconsideration/Reinvestigation. Acting thereon, the
Sandiganbayan required the Office
of the Special Prosecutor to
comment and submit the final
action taken by the Office of
Ombudsman. During
the
trial,
instead of presenting evidence, the
respondents filed their respective
motions for leave to file their
demurrer to evidence which was
then
granted.
Subsequently,
Sandiganbayan issued a Resolution
denying
the
demurrers
to
evidence. In their respective
comments on the petition, the
respondents are one in questioning
the propriety of resorting to this

present petition for review on


certiorari under Rule 45 on the
ground that it places them in
double jeopardy.
ISSUE: WON the respondents are
placed in double jeopardy.
HELD: Procedurally, the petitioner
resorted to a wrong remedy.
Section 1 of Rule 122 allows "any
party" to appeal from a judgment
or final order, unless the right of
the
accused
against
double
jeopardy will be violated. It is
axiomatic that an appeal in
criminal cases throws the whole
case wide open for review by an
appellate court. As a consequence,
an appeal by the prosecution from
a judgment of acquittal necessarily
places the accused in double
jeopardy. The prosecution cannot
appeal from a ruling granting the
demurrer to evidence of the
accused as it is equivalent to an
acquittal unless the prosecution
can sufficiently prove that the
courts action is attended with
grave abuse of discretion. In the
dismissal of a criminal case upon
demurrer to evidence, appeal is
not available as such an appeal
will put the accused in double
jeopardy. For being the wrong
remedy taken by petitioner People
of the Philippines in this case, this
petition is outrightly dismissible.
The Court cannot reverse the
assailed dismissal order of the trial
court by appeal without violating
private respondent's right against
double jeopardy.

Ivler v. Modesto-San Pedro

G.R. No. 172716, November 17,


2010
Topic: Double jeopardy
Facts: Jason Ivler was charged
before the MeTC, with two
separate offenses: (1) Reckless
Imprudence Resulting in Slight
Physical Injuries (Criminal Case
No. 82367) for injuries sustained
by Evangeline L. Ponce and (2)
Reckless Imprudence Resulting in
Homicide and Damage to Property
(Criminal Case No. 82366) for the
death of Ponces husband Nestor
C. Ponce and damage to the
spouses Ponces vehicle. Invoking
this conviction, Ivler moved to
quash the Information in Criminal
Case No. 82366 for placing him in
jeopardy of second punishment for
the same offense of reckless
imprudence. The MeTC refused
quashal, finding no identity of
offenses in the two cases. The RTC
dismissed
the
petition
for
certiorari
filed
by
Ivler
in
connection with the decision of the
MeTC. Hence, petitioner filed this
petition
arguing
that
his
constitutional right not to be
placed twice in jeopardy of
punishment for the same offense
bars his prosecution in Criminal
Case No. 82366, having been
previously convicted in Criminal
Case No. 82367 for the same
offense of reckless imprudence
charged in Criminal Case No.
82366.
Issue: WON petitioners rights
against double jeopardy has been
violated
Held:

Yes.

Prior

conviction

or

acquittal of reckless imprudence


bars subsequent prosecution for
the
same
quasi-offense.
The
doctrine that reckless imprudence
under Article 365 is a single quasioffense by itself and not merely a
means to commit other crimes
such that conviction or acquittal of
such
quasi-offense
bars
subsequent prosecution for the
same quasi-offense, regardless of
its
various
resulting
acts,
undergirded this Courts unbroken
chain of jurisprudence on double
jeopardy as applied to Article
365. These cases uniformly barred
the
second
prosecutions
as
constitutionally
impermissible
under the Double Jeopardy Clause.
The law penalizes thus the
negligent or careless act, not the
result thereof. The gravity of the
consequence is only taken into
account to determine the penalty,
it does not qualify the substance of
the offense. And, as the careless
act is single, whether the injurious
result should affect one person or
several
persons,
the
offense
(criminal negligence) remains one
and the same, and cannot be split
into
different
crimes
and
prosecutions.
People v. Diaz
L-6518, March 30, 1954
Topic: Double jeopardy
FACTS:
Francisco
Diaz
was
accused in the Municipal Court of
Pasay City of a violation of Section
52 of the Act 3992 known as the
Revised Motor Vehicles Law. Diaz
pleaded not guilty to the charge
and after the case was called for

hearing
twice
without
the
prosecution
appearing,
the
Municipal Court on motion of the
defense dismissed the case for
failure of the Government to
prosecute. Subsequently, Assistant
City Attorney of Pasay City filed an
information in the Court of First
Instance
charging
the
same
Francisco Diaz with Damage to
Property
thru
Reckless
Imprudence. Defendant's counsel
filed his motion to quash based on
double jeopardy which was then
granted.

reckless driving, the damage to


property was not included. A
defendant should not be harassed
with various prosecutions base on
the same act by splitting the same
into various charges, all emanating
from the same law violation, when
the prosecution could easily and
well embody them in a single
information.

ISSUE: WON
jeopardy

FACTS: There was a bus accident


involving La Mallorca passenger
bus driven by Pascual Silva and
another La Mallorca bus. As a
result of the accident, Felix Lazaro
died. Emilia Estrella and Francisco
Sarmientosustained
serious
physical injuries, while Policarpio
Sarmiento, Eulalio- Sarmiento and
Dolores Cimeno suered physical
injuries. Under date of October 25,
1957, an information charging
Pascual Silva with Slight Physical
Injuries thru Reckless Imprudence
was presented with the Justice of
the Peace Court of Meycauayan,
Bulacan. After a trial on the merits
was conducted, the accused Silva
was acquitted. On the same date
the information was filed with
Justice of Peace Court, other info
charging silva of Homicide with
Serious Physical Injuries thru
Reckless Imprudence. Silva moved
to quash the information on
ground of Double Jeopardy. Lower
Court reconsidered its order and
dismissed the case believing that if
fell squarely within the doctrine of
double jeopardy.

there

is

double

HELD: Yes. Dismissal by the


Municipal Court of a charge of
reckless driving barred the second
information of damage to property
through
reckless
imprudence
based on the same negligent act of
the accused. It is clear that in the
present case the second charge of
damage to Property thru Reckless
Imprudence includes the first
charge of reckless driving; that the
facts alleged in the information on
damage to property thru reckless
driving, if proven, would have been
sufficient to support the first
charge of reckless driving, and
finally, that the offense of reckless
driving is an ingredient of the
offense of damage to property thru
reckless imprudence, all for the
simple reason that the basic
element in both offenses is
reckless driving. Thus it is evident
that we have here a case of double
jeopardy.
And
there
is
no
explanation why when Diaz was
first charged with a violation of the
Motor Vehicles Law because of

People vs. Silva


L-15974, January 30, 1962
Topic: Double jeopardy

ISSUE: Whether or not the


acquittal in the Justice of Peace
court is a bar for prosecution in
CFI
HELD: Yes. Where as a result of
the same vehicular accident, one
man died, two persons were
seriously injured while another
three suffered only slight physical
injuries, the acquittal on a charge
of slight physical injuries through
reckless imprudence, was a bar to
another prosecution for homicide
through
reckless
imprudence.
Supreme
Court
armed
the
decision of CFI. It may be admitted
that the complaint under which the
appellee was acquitted was fatally
defective for want of certain
essential
allegations.
But
conviction or acquittal under such
a complaint is not necessarily void
when no objection appears to have
been raised at the trial and the
fatal defect could have been
supplied by competent proof.

People V. Buan
131 Phil. 498, 500-502
Topic: Double jeopardy
FACTS: A charge was filed against
the accused-appellant, one for
slight physical injuries through
reckless imprudence, in the Justice
of the Peace Court of Guiguinto,
for which he was tried and
acquitted on December 16, 1963.
Prior to this acquittal, however, the
Provincial Fiscal of Bulacan filed in
the Court of First Instance the
information in the case now before
us, for serious physical injuries,

and damage to property through


reckless imprudence. Admittedly,
both charges referred to the same
highway
collision. When
the
accused was arraigned in the
Court of First Instance, his counsel
moved to quash the charges on the
ground that he had already been
acquitted of the same offense by
the Justice of the Peace Court. The
prosecution opposed the motion.
ISSUE: WON the second case
placed the appellant twice in
jeopardy for the same offense, and
is barred by the previous acquittal.
HELD: Yes. The accused may not
be prosecuted again for that same
act. For the essence of the quasi
offense of criminal negligence
under article 365 of the Revised
Penal Code lies in the execution of
an imprudent or negligent act that,
if intentionally done, would be
punishable as a felony. The law
penalizes thus the negligent or
careless act, not the result
thereof. And, as the careless act is
single, whether the injurious result
should affect one person or several
persons, the offense (criminal
negligence) remains one and the
same, and cannot be split into
different crimes and prosecutions.
The charge of slight physical
injuries
through
reckless
imprudence prevents prosecution
for
serious
physical
injuries
through reckless imprudence in
the Court of First Instance of the
province, where both charges are
derived from the consequences of
one and the same vehicular
accident, because
the
second
accusation places the appellant in

second jeopardy
offense.

for

the

same

People vs. Belga


100 Phil. 996
Topic: Double jeopardy
FACTS: As a result of a collision
between
two
public
service
vehicles, one driven by Ciriaco
Belga and the other by Jose Belga,
the Chief of police of Malilipot,
Albay,
filed
a
complaint for
Reckless
Imprudence
with Physical Injury and damage to
property thru reckless imprudence
the former not having been
dismissed. The accused was later
on acquitted. Counsel for Jose
Belga
moved
to
quash
the
complaint for multiple physical
injuries thru reckless imprudence
filed against him by the injured
passengers, contending that the
case was but a duplication of the
one filed by the chief of police
wherein this accused had just been
acquitted. But the motion was
denied and after trial Jose Belga
was convicted. Alleging double
jeopardy, counsel for Jose Belga
asked for the dismissal of both
cases, and the court having
granted the motion, the provincial
fiscal took the present appeal.
ISSUE: WON the acquittal of Jose
Belga in the case filed by the chief
of police constitutes a bar to his
subsequent prosecution.
HELD: Yes. The second charge of
Damage to property through
Reckless Imprudence includes the
first charge of reckless driving;
that the facts alleged in the

information on damage to property


through reckless driving. Thus it is
evident that we have here a case
of double jeopardy. Dismissal of an
information for physical injuries
through needless imprudence as a
result of a collision between two
automobiles was declared, to block
two other prosecutions, one for
damage
to
property
through
reckless imprudence and another
for multiple physical injuries
arising from the same collision.
In The Matter of the
Application for a Writ
of Habeas Corpus, Bernabe
Buscayno, Jose Ma. Sison and
Juliet Sison vs. Military
Commissions Nos. 1, 2, 6 and
25
G.R. No. L-58284, November
19, 1981
Topic: Double jeopardy
FACTS: Buscayno and Sison were
included in the so-called "National
Target List" of active participants
in the conspiracy to seize political
and state power and to take over
the government by force whose
arrest was ordered under 'General
Order No. 2 dated September 22,
1972. Even before Buscayno's
arrest, he and Benigno S. Aquino,
Jr. were charged before Military
Commission No. 2 in an amended
charge with subversion or violation
of the Anti-Subversion Law.
It was alleged that as ranking
leaders of the Communist Party of
the Philippines and its military
arms, the Hukbong Mapagpalaya
ng Bayan and the New People's
Army, constituting an organized

conspiracy
to
overthrow
the
government by force or placing it
under the control of an alien
power.

ISSUE: WON petitioners invoke


their constitutional right not to be
put
twice
in
jeopardy
of
punishment for the same offense

It was alleged that the ninety-two


accused as officers and leaders of
the Communist Party of the
Philippines and its military arm,
the New People's Army, and as
conspirators rose publicly and took
up arms against the government in
Navotas, Rizal and elsewhere in
the Philippines for the purpose of
removing from the allegiance to
said government or its laws the
territory of the Philippines or any
part thereof or of its armed forces
by organizing the Karagatan
Fishing Corporation and operating
the M/V Karagatan a fishing
vessel, to procure firearms and
ammunition for the CPP and NPA
as in fact war materials and
armanents were landed at Digoyo
Point,
Palanan,
Isabela
from
Communist China and were used
against the army.

HELD: No. For an accused to be in


jeopardy, it is necessary (1) that a
valid complaint or information or
other formal charge sufficient in
form and substance to sustain a
conviction is filed against him; (2)
that the charge is filed in a court
of competent jurisdiction and (3)
that after he had pleaded to the
charge, he was convicted or
acquitted or the case against him
was
dismissed
or
otherwise
terminated without his express
consent.

The
second
specification
in
Criminal Case is that Buscayno,
Sison
and
others
committed
rebellion in Manila, Baguio, La
Union, Pangasinan, Bulacan and
elsewhere in the Philippines by
acquiring,
purchasing
and
operating vessels, motor vehicles,
beach houses, lots and other real
and personal properties for use in
distributing
firearms
and
ammunition for the CPP and NPA
to be utilized in resisting the army
and overthrowing the government.

To be in jeopardy, the case against


the accused must be terminated by
means of a final conviction,
acquittal or dismissal without his
express consent, If the case is not
yet terminated, then jeopardy does
not set in. After the accused has
been put in jeopardy, the filing
against him of another charge for
the same offense or for an attempt
or frustrated stage thereof or for
any offense which necessarily
includes or is included in the
offense originally charged places
him in double jeopardy.
In the instant case, the rebellion
charge against the petitioners
embraced the acts committed by
them on or about February 4, 1972
and during the period from
August, 1973 to February, 1974.
The subversion charge against
Buscayno
involved
his
acts
committed in 1965, 1967, 1969,
1970 and 1971. The subversion
charge against the Sison spouses

referred to their acts committed in


1968 and for sometime prior and
subsequent thereto. The common
denominator of the rebellion and
subversion charges is that the
petitioners committed overt acts
as alleged communists or leftists.
The overt acts in the two charges
are different.
Rebellion is an offense that has
existed in the Penal Code for a
long time. It may be committed by
non-communists
without
collaborating with the agents of an
alien power. In contrast, the crime
of subversion came into existence
when the communists sought to
dominate the world in order to
establish a new social economic
and political order.
The constitutionality of the AntiSubversion Law was upheld in
People vs. Ferrer, L-32613-14,
December 27, 1972, 48 SCRA 382
and 56 SCRA 793. Long before the
passage of the Anti-Subversion
Law
membership
in
illegal
associations has been penalized
(Art. 146, Revised Penal Code).
A
statute
which
punishes
membership
in
a
party
or
association that advocates the
overthrow or destruction of the
government by force or violence is
justified on the ground of selfpreservation
The unavoidable conclusion is that
in the present posture of the
pending
cases
against
the
petitioners their plea of double
jeopardy cannot be sustained.

Loney v. People
G.R. No. 152644, February 10,
2006
Topic: Double jeopardy
FACTS: Petitioners John Eric
Loney, Steven Paul Reid and Pedro
B. Hernandez are the Pres. and
CEO,
Senior
Manager,
and
Resident Manager for Mining
Operations,
respectively,
of
Marcopper
Mining
Corp.
Marcopper had been storing
tailings (mine waste) from its
operations in a pit Marinduque. At
the base of the pit ran a drainage
tunnel it appears that Marcopper
had placed a concrete plug at the
tunnels end. On March 24, 1994,
tailings gushed out of or near the
tunnels end. In a few days, Mt.
Tapian pit had discharged millions
of tons of tailings in to the Boac
and Makalupnit rivers. In August
1996, the DOJ separately charged
petitioners with violation of Art. 91
(B), subparagraphs 5 and 6 of P.D.
No. 1067 or the Water code of the
Phil., Sec. 8 of P.D. No. 984 or the
National Pollution Decree of 1976,
Sec. 108 of R.A. No. 7942 or the
Phil. Mining Act of 1995, and Art.
365 of the RPC for Reckless
Imprudence Resulting to Damage
to Property. In the Consolidated
Order of MTC, granting partial
reconsideration to its Joint Order
quashing the information for
violation of PD 1067 and PD 984.
The
MTC
maintained
the
Informations
for
violation
of
RA7942 and Art. 365 of the
RPC.
Petitioners subsequently
filed a petition for certiorari
with the RTC assailing that the

portion of the Consolidated Order


maintaining the Informations for
violation of RA 7942and
the
petition was raffled to Br. 94
while public respondents appeal
assailing
that portion of
the
Consolidated Order quashing the
Info. For violation of P.D. 1067 and
P.D. 984 and this appeal was
consolidated
with
petitioners
petition. MTC Br. 94 granted the
public respondents appeal but
denied
petitioners
petition.
Petitioners then filed for certiorari
with the Court of Appeals alleging
that Br. 94 acted with grave abuse
of
discretion
because
1.the
Informations for violation of PD
1067, PD 984, RA 7942 and the
Art. 365 of the RPC proceeded
from are based on a single act or
incident of polluting the rivers thru
dumping of mine tailings, and the
charge for violation of Art 365 of
the RPC absorbs the other charges
since the element of lack of
necessary or adequate protection,
negligence,
recklessness
and
imprudence is common among
them, 2.the duplicitous nature of
the
Informations contravenes
the ruling in People v. Relova.
The Court ofAppeals affirmed the
Br. 94 ruling.
ISSUE: 1. Whether or not all the
charges filed against petitioners
except one should be quashed for
duplicity of charges
and
only
the
charge
for
Reckless
Imprudence
Resulting
in
Damage to Propertyshould stand
2. whether or not Br. 94s ruling,
as affirmed by the Court of
Appeals,
contravenes
People
v.Relova.

HELD: The petition has no merit.


Duplicity of charges
simply
means a
single complaint or
information charges more than
one offense, as Sec. 13 of Rule 110
of the 1985 Rules of Criminal
Procedure. As early as the start of
the last century, the court ruled
that a single act or incident might
offend against two or more entirely
distinct and unrelated provisions
of
law
thus
justifying
the
prosecution of the accused for
more than one offense and the only
limit
is
the
Constitutional
prohibition that no person shall be
twice put in jeopardy of punishing
for the same offense. In People vs.
Doriquez, the court held that two
or more offenses arising from the
samea ct are not the same. And so,
double jeopardy is not an issue
because not all its elements are
present. On petitioners claim that
the charges for violation of Art.
365 of the RPC absorbs the
charges for violation of PD 1067,
PD 984 and RA 7942, suffice it to
say that a mala in se felony (such
as
Reckless
Imprudence
Resulting
to
Damage
in
Property)
cannot absorb mala
prohibita crimes (such as those
violating PD 1067, PD 984 and RA
7942). What makes the former
felony is criminal inten (dolo) or
negligence (culpa) and what makes
the latter crimes are the special
laws enacting them. Petitioners
reiterate their contention in that
their
prosecution
contravenes
ruling in People vs. Relova.
In
particular, petitioners cite the
courts
statement
in
Relova
that the law seeks to prevent
harassment of the accused by

multiple prosecutions for offenses


which though different from one
another are nonetheless each
constituted by a common set or
overlapping sets
of technical
elements. Thus, Relova is no
authority for petitioners claim
against
multiple
prosecutions
based on a single act not only
because the question of double
jeopardy is not an issue here, but
also because, as the Court of
Appeals held, petitioners are being
prosecuted for an act or incident
punished by four national statutes
and not by an ordinance and a
national
statute.
In
short,
petitioners, if ever fall under the
first sentence of Sec. 21, Art. III
which
prohibits
multiple
prosecution for the same offense,
and not, as in Relova, for offenses
arising from the same incident.
SUERO vs PEOPLE
G.R. No. 156408, January 31,
2005
Topic: Double jeopardy
FACTS: Before us is a Petition for
Certiorari under Rule 65 of the
Rules of Court, seeking to reverse
the December 14, 2001 Order of
the Regional Trial Court (RTC) of
Davao City (Branch 16) in Criminal
Case No. 48167-01, denying the
Motion to Quash Information filed
by petitioner, as well as the
October 3, 2002 Order denying his
Motion for Reconsideration. The
first assailed Order posed for
resolution is the motion to quash
information and/or dismiss the
case filed by the accused along
with the opposition thereto filed by

the Office of the Ombudsman.


Sifting through the arguments and
counter-arguments in support of
and in opposition to the instant
motion, the court rules to deny the
motion to quash the information.
There is no dispute that the
present case and Criminal Case
before the Sandiganbayan arose
out of the same incident or
transaction.
Nonetheless,
as
correctly raised by the Office of
the Ombudsman, the present case
involves
the
prosecution
for
Falsification of Public Documents
as defined and penalized under
Art. 171 of the Revised Penal
Code, while Criminal Case No.
23518 before the Sandiganbayan
pertains to the causing of undue
injury to the government. The
latter case requires the element of
damage while in Falsification of
Public Document, damage is of no
consequence
The dismissal therefore of Criminal
Case before the Sandiganbayan
has no bearing with the present
case since the quantum of
evidence required to sustain both
cases are not similar. In the same
vein, this is a particular case
where one incident results to two
(2) separate and distinct criminal
offenses, such that the dismissal of
one case would not constitute
double
jeopardy
against
the
accused in the other case.
Accordingly, the motion to quash
the information is denied for lack
of merit.

ISSUE: Whether there is double


jeopardy in the case of falsification
of a public document
HELD: NO. Petitioner argues that
the primordial legal issue involved
in the two cases is the same. We do
not agree. It is undisputed that the
two charges stem from the same
transaction. However, it has been
consistently held that the same act
may give rise to two or more
separate and distinct offenses. No
double jeopardy attaches, as long
as there is a variance between the
elements of the offenses charged.
The constitutional right against
double jeopardy protects from a
second prosecution for the same
offense, not for a different one.
Indeed, the crime under Section
3(e) of RA 3019 shares two
common elements with the felony
under Article 171 of the Revised
Penal Code -- that the offender is a
public officer and that the act is
related to the officers public
position. However, the latter
offense is not necessarily inclusive
of the former. The essential
elements of each are not included
among or do not form part of those
enumerated in the former. For
there to be double jeopardy, the
elements of one offense should -like the ribs of an umbrella -ideally encompass those of the
other. The elements of a violation
of Section 3(e) of RA 3019 fall
outside the realm of those of
falsification of a public document
and vice versa. At most, the two
offenses may be considered as two
conjoined umbrellas with one or
two common ribs. Clearly, one
offense does not include the other.

Given the differences between the


elements of the two offenses, there
is
no
merit
to
petitioners
contention that it would be legally
untenable for respondent judge to
make a contradictory appreciation
of the evidence to be presented
and, hence, a ruling contradictory
to that of the Sandiganbayan.
Petitioner is of the erroneous
assumption that the guilt or the
innocence of the accused in both
cases hinges on the exact same set
of evidence; namely, the validity or
the falsity of the documents,
subject of the instant case.
Nierras vs. Dacuycuy
G.R. Nos. 59568-76 January 11,
1990
Topic: Double jeopardy
FACTS: Peter Nierras, a customer
of
Pilipinas
Shell
Petroleum
Corporation,
purchased
oil
products from it.
Simultaneous with the delivery of
the products, he issued 9 checks in
payment
thereof.
Upon
presentation to the Philippine
National Bank at Naval, Leyte,
said checks were dishonored for
the reason that his account was
already
closed.
Thereafter,
Pilipinas
Shell
Petroleum
Corporation repeatedly demanded
of Nierras either to deposit funds
for his checks or pay for the oil
products he had purchased but he
failed and refused to do either.
Nierras was charged for for estafa
under Article 315 (2-d) of the
Revised Penal Code (Criminal

Cases 4379, 4380, 4381, 4382,


4383, 4384, 4385, 4386 and 4387)
and similarly for violation of the
Bouncing Checks Law or Batas
Pambansa 22 (Criminal Cases
3790, 3791, 3792, 3793, 4085,
4122, 4123, 4124, and 4125). In
both sets of criminal cases,
Nierras entered a plea of not guilty
upon arraignment before the lower
court. However, immediately after
his plea of not guilty in these
estafa cases, petitioner moved in
open court to be allowed to
withdraw his plea of not guilty
upon his filing of a motion to
quash, which was denied by Judge
Auxencio C. Dacuycuy in a
resolution dated 17 September
1981. Nierras filed the petition for
certiorari
with
preliminary
injunction.
ISSUE: Whether Nierras may be
held liable for the 9 criminal cases
for violation of BP 22, and
separately also be held liable for
the crime of estafa under Article
315 (2-d) of the Revised Penal
Code for the issuance of the same
bouncing checks.
HELD: Nierras is charged with 2
distinct and separate offenses, first
under
Section
1
of
Batas
Pambansa Bilang 22 and, second,
under Article 315, (2-d) of the
Revised Penal Code. Deceit and
damage are essential elements in
Article 315 (2d) Revised Penal
Code, but are not required in BP
22. Under the latter law, mere
issuance of a check that is
dishonored gives rise to the
presumption of knowledge on the
part of the drawer that he issued
the same without sufficient funds

and hence punishable which is not


so under the Penal Code. Other
differences between the two also
include the following: (1) a drawer
of a dishonored check may be
convicted under BP 22 even if he
had issued the same for a preexisting obligation, while under
Article 315 (2-d) of the
Revised
Penal
Code,
such
circumstance
negates
criminal
liability; (2) specific and different
penalties are imposed in each of
the two offenses; (3) estafa is
essentially
a
crime
against
property, while violation of BP 22
is principally a crime against
public interest as it does injury to
the entire banking system; (4)
violations of Article 315 of the
Revised Penal Code are mala in se,
while those of BP 22 are mala
prohibita. Furthermore, Section 5
of Batas Pambansa Bilang 22
provides that "Prosecution under
this Act shall be without prejudice
to any liability for violation of any
provision of the Revised Penal
Code." While the filing of the two
sets of Information under the
provisions of BP 22 and under the
provisions of the Revised Penal
Code, as amended, on estafa, may
refer to identical acts committed
by Nierras, the prosecution thereof
cannot be limited to one offense,
because a single criminal act may
give rise to a multiplicity of
offenses and where there is
variance or differences between
the elements of an offense in one
law and another law, there will be
no double jeopardy because what
the rule on double jeopardy
prohibits refers to identity of

elements in the 2 offenses.


Otherwise stated, prosecution for
the same act is not prohibited.
What is forbidden is prosecution
for the same offense. Hence, the
mere filing of the 2 sets of
information does not itself give
rise to double jeopardy

People vs Comila
G.R. No. 171448, February 28,
2007
Topic: Double jeopardy
FACTS: That on or about the 10th
day of November, 1998, in the City
of Baguio, Philippines, and within
the jurisdiction of this Honorable
Court, the above-named accused,
conspiring,
confederating
and
mutually aiding one another did
then and there willfully, unlawfully
and feloniously defraud one ZALDY
DUMPILES Y MALIKDAN by way
of false pretenses, which are
executed
prior
to
or
simultaneously
with
the
commission of the fraud, as
follows, to wit: the accused
knowing fully well that he/she/they
is/are
not
AUTHORIZED
job
RECRUITERS
for
persons
intending to secure work abroad
convinced said Zaldy Dumpiles y
Malikdan and pretended that
he/she/they could secure a job for
him/her abroad, for and in
consideration of the sum of
P25,000.00 and representing the
placement and medical fees when
in truth and in fact could not; the
said Zaldy Dumpiles y Malikdan
deceived and convinced by the
false pretenses employed by the
accused parted away the total sum
of P25,000,00 in favor of the
accused, to the damage and
prejudice of the said Zaldy
Dumpiles y Malikdan in the
aforementioned
amount
of
TWENTY FIVE THOUSAND PESOS
(P25,000.00), Philippine currency.

ISSUE: Whether a person can be


charged of both estafa and illegal
recruitment
HELD: YES It is well established
in jurisprudence that a person may
be charged and convicted for both
illegal recruitment and estafa. The
reason therefor is not hard to
discern: illegal recruitment is
malum prohibitum, while estafa is
malum in se. In the first, the
criminal intent of the accused is
not necessary for conviction. In the
second,
such
an
intent
is
imperative. Estafa under Article
315, paragraph 2, of the Revised
Penal Code, is committed by any
person who defrauds another by
using fictitious name, or falsely
pretends
to
possess
power,
influence, qualifications, property,
credit,
agency,
business
or
imaginary transactions, or by
means of similar deceits executed
prior to or simultaneously with the
commission of fraud. Here, it has
been sufficiently proven that both
appellants represented themselves
to the complaining witnesses to
have the capacity to send them to
Italy for employment, even as they
do not have the authority or
license for the purpose. Doubtless,
it is this misrepresentation that
induced the complainants to part
with their hard-earned money for
placement and medical fees. Such
act on the part of the appellants
clearly constitutes estafa under
Article 315, paragraph (2), of the
Revised Penal Code.

People vs Adeser y Rico

G.R. No. 179931, October 26,


2009
Topic: Double jeopardy
FACTS:
Nida
Adeser
was
convicted of the crimes syndicated
illegal recruitment constituting
economic sabotage and estafa.
Josephine R. Palo testified that she
spouses Roberto and Mel Tiongson
introduced her to Adeser, owner
and general manager of Naples
Travel and Tours, to discuss
employment
opportunities
in
Australia. She then gave money for
visa application and placement fee
and was issued a voucher that said
payment was for visa application.
More than three months passed,
however,
but
Palo
was
not
deployed to Australia. Neither did
she get her Australian visa. She
later learned from the NBI that
Naples had closed down and that
they had no license to operate and
deploy workers abroad. Adeser
She admitted that she was the
owner and general manager of
Naples which was a travel agency
that
offered
visa
assistance,
ticketing, documentation, airport
transfer and courier services, but
denied
having
engaged
in
recruitment. She testified that
Roberto, to whom Palo claims to
have given her payment, was
neither her employee nor her
agent but was only her drivers
brother. Appellant appealed her
conviction but the same was
affirmed by the Court of Appeals.
The appellate court did not give
credence to appellants denials and
found
that
the
prosecution
evidence fully supports the finding
that appellant and her co-accused

engaged
in
recruitment
and
placement as defined under the
Labor Code despite having no
authority to do so. It likewise held
that the same evidence proving the
commission of the crime of illegal
recruitment also established that
appellant and her co-accused
acted in unity in defrauding Palo
and in misrepresenting to her that
upon payment of the placement
fee, they could obtain employment
abroad for her. The appellants act
of deception and the resultant
damage suffered by Palo render
appellant guilty of estafa.
ISSUE: WON the court was
correct in finding her guilty of the
crime of estafa
HELD: A person who is convicted
of illegal recruitment may also be
convicted of estafa under Article
315(2) (a) of the Revised Penal
Code provided the elements of
estafa are present. Estafa under
Article 315, paragraph 2(a) of the
Revised Penal Code is committed
by any person who defrauds
another by using a fictitious name,
or falsely pretends to possess
power, influence, qualifications,
property, credit, agency, business
or imaginary transactions, or by
means of similar deceits executed
prior to or simultaneously with the
commission of the fraud. The
offended party must have relied on
the false pretense, fraudulent act
or fraudulent means of the
accused and as a result thereof,
the
offended
party
suffered
damage. Palo parted with her
money upon the prodding and
enticement of appellant and her
co-accused on the false pretense

that they had the capacity to


deploy her for employment in
Australia. Unfortunately, however,
Palo was not able to work abroad
nor get her Australian visa. Worse,
she did not get her money back.
People v. Veridiano
G.R. No. L-62243, October 12,
1984
Topic: BP. 22 and estaffa
FACTS: Private respondent Benito
Go Bio, Jr. was charged with
violation of Batas Pambansa Bilang
22 in Criminal Case No. 5396 in
the then Court of First Instance of
Zambales, presided by respondent
judge.
Before
he
could
be
arraigned respondent Go Bio, Jr.
filed a Motion to Quash the
information on the ground that the
information did not charge an
offense, pointing out that at the
alleged commission of the offense,
which was about the second week
of May 1979, Batas Pambansa
Bilang 22 has not yet taken effect.
The case was then dismissed. Also,
the Court believes that although
the accused can be prosecuted for
swindling (Estafa, Article 315 of
the Revised Penal Code), the Batas
Pambansa Bilang 22 cannot be
given a retroactive effect to apply
to the above entitled case.
ISSUE: WON BP. 22 be given
retroactive effect
HELD: No. The Court upheld the
dismissal by the respondent judge
of the criminal action against the
private respondent. Conviction of
an accused for estafa is not a bar

to a prosecution for violation of


B.P. Blg. 22 for issuing the same
bouncing checks. This is anchored
on the fact that the elements on
both crimes differ from the other.
However, it must be noted that
when private respondent Go Bio,
Jr. committed the act, complained
of in the Information as criminal,
in May 1979, there was then no
law penalizing such act. Following
the special provision of Batas
Pambansa Bilang 22, it became
effective only on June 29, 1979. As
a matter of fact, in May 1979,
there was no law to be violated
and, consequently, respondent Go
Bio, Jr. did not commit any
violation thereof. Thus, BP. 22
which
should
be
applied
prospectively, accused cannot be
convicted thereof.
Merencillo vs People
G.R. Nos. 142369-70, April 13,
2007
Topic: Double jeopardy
FACTS: The RTC of Tagbilaran
City, Branch 47, in found petitioner
guilty of violating Section 3(b) of
RA 3019 and Article 210 of the
Revised Penal Code. Merencillo,
then a public official connected
with the Bureau of Internal
Revenue as its Group Supervising
Examine demand and extort from a
certain
Mrs.
Maria
Angeles
Ramasola Cesar the amount of
P20,000.00, in connection, in
consideration and in exchange for
the release of the certification of
her payment of the capital gains
tax for the land purchased by the

Ramasola [Superstudio] Inc. from


one Catherine Corpuz Enerio, a
transaction wherein the aforesaid
accused has to intervene in his
official
capacity.
Military
authorities
particularly
the
elements of the 702nd Criminal
[Investigation] Command (CIC) set
up the accused for a possible
entrapment resulting to his being
caught in the act of receiving an
envelope supposedly containing
the
amount
of
P20,000.00.
Petitioner pleaded not guilty to
both charges when arraigned.
Thereafter trial ensued and the
cases were tried jointly. Petitioner
appealed the RTC decision to the
Sandiganbayan.
The
Sandiganbayan, however, denied
the appeal and affirmed the RTC
decision. Petitioner contends that
the
Sandiganbayan
feiled
to
recognize that he was placed in
double jeopardy.
ISSUE: WON there was double
jeopardy
HELD: Section 3 of RA 3019
provides that [I]n addition to acts
or omissions of public officers
already penalized by existing
lawOne
may
therefore
be
charged with violation of RA 3019
in addition to a felony under the
Revised Penal Code for the same
delictual act, that is, either
concurrently or subsequent to
being charged with a felony under
the Revised Penal Code. There is
no double jeopardy if a person is
charged
simultaneously
or
successively
for
violation
of
Section 3 of RA 3019 and the
Revised Penal Code. The rule
against double jeopardy prohibits

twice placing a person in jeopardy


of punishment for the same
offense. The test is whether one
offense is identical with the other
or is an attempt to commit it or a
frustration thereof; or whether one
offense necessarily includes or is
necessarily included in the other,
as provided in Section 7 of Rule
117 of the Rules of Court. A
comparison of the elements of the
crime of direct bribery defined and
punished under Article 210 of the
Revised Penal Code and those of
violation of Section 3(b) of RA
3019 shows that there is neither
identity nor necessary inclusion
between the two offenses.
Clearly, the violation of Section
3(b) of RA 3019 is neither identical
nor necessarily inclusive of direct
bribery. While they have common
elements, not all the essential
elements of one offense are
included among or form part of
those enumerated in the other.
Whereas the mere request or
demand of a gift, present, share,
percentage or benefit is enough to
constitute a violation of Section
3(b) of RA 3019, acceptance of a
promise or offer or receipt of a gift
or present is required in direct
bribery. Moreover, the ambit of
Section 3(b) of RA 3019 is specific.
It is limited only to contracts or
transactions involving monetary
consideration where the public
officer has the authority to
intervene under the law. Direct
bribery, on the other hand, has a
wider and more general scope: (a)
performance of an act constituting
a crime; (b) execution of an unjust
act which does not constitute a

crime and (c) agreeing to refrain


or refraining from doing an act
which is his official duty to do.
Although the two charges against
petitioner stemmed from the same
transaction, the same act gave rise
to two separate and distinct
offenses. No double jeopardy
attached since there was a
variance between the elements of
the
offenses
charged.
The
constitutional protection against
double jeopardy proceeds from a
second prosecution for the same
offense, not for a different one.
People vs. Lacson
G.R. No. 149453, May 28, 2002
Topic: Time-bar Rule
FACTS: The respondent and his
co-accused were charged with
multiple murder for the shooting
and killing of eleven male persons
bandied as members of the
Kuratong Baleleng Gang. The case
was provisionally dismissed with
the express consent of respondent
as he himself moved for the said
provisional dismissal when he filed
his
motion
for
judicial
determination of probable cause
and for examination of witnesses.
The petitioners contend that even
on the assumption that the
respondent expressly consented to
a provisional dismissal and all the
heirs of the victims were notified
of the respondents motion before
the hearing thereon and were
served
with
copies
of
the
resolution, the two-year bar in
Section 8 of Rule 117 of the
Revised
Rules
of
Criminal

Procedure should be applied


prospectively and not retroactively
against the State. To apply the
time limit retroactively to the
criminal
cases
against
the
respondent and his co-accused
would violate the right of the
People to due process, and unduly
impair, reduce, and diminish the
States
substantive
right
to
prosecute the accused for multiple
murder. They posit that under
Article 90 of the Revised Penal
Code, the State had twenty years
within which to file the criminal
complaints against the accused.
However, under the new rule, the
State only had two years from
notice of the public prosecutor of
the order of dismissal within which
to revive the said cases. For his
part, the respondent asserts that
the new rule under Section 8 of
Rule 117 of the Revised Rules of
Criminal
Procedure
may
be
applied retroactively since there is
no substantive right of the State
that may be impaired by its
application to the criminal cases in
question
since
[t]he
States
witnesses were ready, willing and
able to provide their testimony but
the prosecution failed to act on
these cases until it became
politically expedient for them to do
so. According to the respondent,
penal laws, either procedural or
substantive, may be retroactively
applied so long as they favor the
accused. He asserts that the twoyear period commenced to run on
March 29, 1999 and lapsed two
years thereafter was more than
reasonable opportunity for the
State to fairly indict him.

ISSUE:WON the State may revive


the cases after the 2-year bar
HELD: If the cases were revived
only after the two-year bar, the
State
must
be
given
the
opportunity to justify its failure to
comply with the said time-bar. It is
emphasized that the new rule fixes
a time-bar to penalize the State for
its
inexcusable
delay
in
prosecuting cases already filed in
court. However, the State is not
precluded
from
presenting
compelling reasons to justify the
revival of cases beyond the twoyear bar. The time-bar under the
new rule does not reduce the
periods under Article 90 of the
Revised Penal Code, a substantive
law. It is but a limitation of the
right of the State to revive a
criminal case against the accused
after the Information had been
filed
but
subsequently
provisionally dismissed with the
express consent of the accused.
Upon the lapse of the timeline
under the new rule, the State is
presumed, albeit disputably, to
have abandoned or waived its right
to revive the case and prosecute
the
accused.
The
dismissal
becomes ipso facto permanent. He
can no longer be charged anew for
the same crime or another crime
necessarily included therein. He is
spared from the anguish and
anxiety as well as the expenses in
any new indictments. The twoyear period fixed in the new rule is
for the benefit of both the State
and the accused. It should not be
emasculated and reduced by an
inordinate retroactive application
of the time-bar therein provided

merely to benefit the accused. For


to do so would cause an injustice
of hardship to the State and
adversely affect the administration
of justice in general and of
criminal laws in particular. The
State may revive a criminal case
beyond the one-year or two-year
periods provided that there is a
justifiable necessity for the delay.
Torres v. Aguinaldo
461 SCRA 599
Topic: Provisional dismissal
FACTS: Spouses Edgardo and
Nelia Aguinaldo filed before the
Office of the City Prosecutor of
Manila
a
complaint
against
petitioner Artemio T. Torres, Jr. for
falsification of public document.
They alleged that titles to their
properties covered by Transfer
Certificates of Title Nos. T-93596,
T-87764,
and
T-87765,
were
transferred
without
their
knowledge and consent in the
name of Torres through a forged
Deed of Sale dated July 21, 1979.
Torres denied the allegations of
forgery
and
claimed
that
Aguinaldo
sold
the
subject
properties to him as evidenced by
the March 10, 1991 Deed of
Absolute Sale.
Finding probable cause, the OCP
recommended the filing of an
information for falsification of
public document against Torres
which was filed before the MTC.
Torres moved for reconsideration
but was denied. On appeal, the
Secretary of Justice reversed the
findings
of
the
investigating
prosecutor
and
ordered
the
withdrawal of the information.

The motion for reconsideration


filed by Aguinaldo was denied. A
Motion to Withdraw Information
was filed which the MTC granted
on June 11, 2003. Meanwhile,
Aguinaldo filed before the Court of
Appeals a petition for certiorari
which was granted in the assailed
decision dated March 22, 2004.
Torres motion for reconsideration
was denied. Torres contends that
the order granting the withdrawal
of the information rendered moot
the petition for certiorari filed
before the Court of Appeals and
insists that an order dismissing a
case without prejudice is final if no
motion for reconsideration or
appeal therefrom is timely filed.
ISSUE:
WON
the
rule
provisional dismissal applies

on

HELD: A motion to withdraw


information differs from a motion
to dismiss. While both put an end
to an action filed in court, their
legal effect varies. In the case at
bar,
a
motion
to
withdraw
information was filed and not a
motion to dismiss. Unlike a motion
to dismiss, a motion to withdraw
information is not time-barred and
does not fall within the ambit of
Section 8, Rule 117 of the Revised
Rules of Criminal Procedure which
provides
that
the
law
on
provisional
dismissal
becomes
operative
once
the
judge
dismisses,
with
the
express
consent of the accused and with
notice to the offended party a
criminal case. There is provisional
dismissal when a motion filed
expressly
for
that
purpose
complies
with
the
following
requisites, viz.: (1) It must be with

the express consent of the


accused; and (2) There must be
notice to the offended party.
Section 8, Rule 117 contemplates
the filing of a motion to dismiss,
and not a motion to withdraw
information. Thus, the law on
provisional dismissal does not
apply in the present case.

6.

7.
8.

9.

10.

11.

12.

13.

14.
Surla, Kristine Angela M.
Cases in the book of Riano
15.
1.
2.

3.

4.

5.

People vs. Webb (G.R. No.


132577; August 17, 1999)
Salvanera v. People (G.R.
No. 143093; May 21,
2007)
People v. Ong (G.R. No.
175940; February 6,
2008)
Cabarles v. Maceda (G.R.
No. 161330; February 20,
2007)
Bernardo v. People (G.R.
No. 166980; April 4,
2007)

16.

17.

18.
19.

People v. Joven de Grano


(G.R. No. 167710; June 5,
2009)
Resayo v. People (G.R. No.
154502; April 27, 2007)
Ceniza-Manantan v.
People (G.R. No. 156248;
August 28, 2007)
People v. Mortera (G.R.
NO. 188104; April 23,
2010)
People v. Tuniaco, et al.,
(G.R. No. 185710; January
19, 2010)
People v. Villanueva (G.R.
No. 181829; September 1,
2010)
Nicolas v. Sandiganbayan
(G.R. Nos. 175930-31;
February 11, 2008)
People v. Crespo (G.R. No.
180500; September 11,
2008)
People v. Tolentino (G.R.
No. 176385; February 26,
2008)
Hun Hyung Park v. Eung
Won Choi (G.R. No.
165496; February 12,
2007)
Cabador v. People (G.R.
No. 186001; October 2,
2009)
People v. Sandiganbayan
(G.R. NO. 164577; July 5,
2010)
People v. Tan (G.R. No.
167526; July 26, 2010)
People v. Sandiganbayan
(G.R. No. 174504; March
21, 2011)

20.

21.

22.

23.

24.

25.

26.

27.

28.

29.

30.

31.

32.

Antiporda, Jr. v.
Garchitorena (G.R. No.
133289; December 23,
1999)
Cruz v. Court of Appeals
(G.R. No. 123340; August
29, 2002)
Cojuangco v.
Sandiganbayan (G.R. No.
134307; December 21,
1998)
Velasco v. Court of
Appeals (G.R. No. 118644;
July 7, 1995)
Lumanog, et al. v. People
(G.R. No. 182555;
September 7, 2010)
BPI v. Leobrera (G.R. No.
137147; January 29,
2002)
Boac v. People (G.R. No.
180597; November 7,
2008)
Chua v. Court of Appeals
(G.R. No. 140842; April
12, 2007)
People v. Serrano (G.R.
No. 135451; September
30, 1999)
People v. Court of Appeals
(G.R. No. 159261;
February 21, 2007)
Abalos v. People (G.R. No.
136994; September 17,
2002)
People v. Alfredo (G.R. No.
188560; December 15,
2010)
Consulta v. People (G.R.
No. 179462; February 12,
2009)

33.
34.

35.

36.

37.

38.
39.

40.

41.
42.

43.

44.

45.
46.

47.

People v. Rellota (G.R. No.


168103; August 3, 2010)
Pangilinan vs. Court of
Appeals (G.R. No. 117363;
December 17, 1999)
People v. Sumingwa (G.R.
No. 183619; October 13,
2009)
Navarrete v. People (G.R.
No. 147913; January 31,
2007)
People v. Corpuz (G.R. No.
168101; February 13,
2006)
People v. Abello (G.R. No.
151952; March 25, 2009)
People v. Taruc (G.R. No.
185202; February 18,
2009)
Estino, et al. v. People
(G.R. Nos. 163957-58;
April 7, 2009)
Briones v. People (G.R.
No. 156009; June 5, 2009)
Neypes v. Court of
Appeals (G.R. No. 141524;
September 14, 2005)
Yu v. Samson-Tatad (G.R.
No. 170979; February 9,
2011)
Estarija v. People (G.R.
No. 173990; October 27,
2009)
Hilario v. People (G.R. No.
161070; April 14, 2008)
Philippine Rabbit v.
People (G.R. No. 147703;
April 14, 2004)
Carino v. De Castro (G.R.
No. 176084; April 30,
2008)

48.

49.

50.

51.

52.

53.

54.

55.

56.
57.
58.

59.

60.

People v. Puig (G.R. Nos.


173654-765; August 28,
2008)
Heirs of Palma Burgos v.
Court of Appeals (G.R. No.
169711; February 8,
2010)
People v. Duca (G.R. No.
171175; October 30,
2009)
Dico v. Court of Appeals
(G.R. No. 141669;
February 28, 2005)
Ferrer v. People (G.R. No.
143487; February 22,
2006)
People v. Montinola (G.R.
Nos. 131856-57; July 9,
2001)
Guy v. People (G.R. Nos.
166794-96; March 20,
2009)
People v. De la Torre (G.R.
No. 176637; October 6,
2008)
People v. Racho (G.R. No.
186529; August 3, 2010)
People v. Lindo (G.R. No.
189818; August 9, 2010)
Asian Terminals v. NLRC
(G.R. No. 158458;
December 19, 2007)
Heirs of Marcelino
Doronio v. Heirs of
Fortunato Doronio (G.R.
No. 169454; December
27, 2007)
Dumo v. Espinas (G.R. No.
141962; January 25,
2006)

61.

62.

63.

64.
65.

66.

67.

68.

69.

70.
71.

72.
73.

74.

75.

Comilang v. Burcena (G.R.


No. 146853; February 13,
2006)
Spouses Romulo v. Layug
(G.R. No. 151217;
September 8, 2006)
People v. Mamaril (G.R.
No. 171980; October 6,
2010)
People v. Musa (G.R. No.
170472; July 3, 2009)
People v. Villasan (G.R.
No. 176527; October 9,
2009)
People v. Relos (G.R. No.
189326; November 24,
2010)
People v. Combate (G.R.
No. 189301; December
15, 2010)
People v. Nachor (G.R. No.
177779; December 14,
2010)
Cacao v. People (G.R. No.
180870; January 22,
2010)
People v. Palma (G.R. No.
189279; March 9, 2010)
People v. Atadero (G.R.
No. 183455; October 20,
2010)
People v. Alipio (G.R. No.
185285; October 5, 2009)
People v. Bernabe (G.R.
No. 185726; October 16,
2009)
People v. Lazaro, Jr. (G.R.
No. 186418; October 16,
2009)
People v. Beduya (G.R.
No. 175315; August 9,
2010)

76.

77.

78.

79.

80.

81.

82.

83.

People v. Dalisay (G.R. No.


188106; November 25,
2009)
People v. Palgan (G.R. No.
186234; December 21,
2009)
People v. Adeser (G.R. No.
179931; October 26,
2009)
People v. Aburque (G.R.
No. 181085; October 23,
2009)
People v. Lascano (G.R.
No. 172605; November
22, 2010)
People v. Rusiana (G.R.
No. 186139; October 5,
2009)
Lejano v. Court of Appeals
(G.R. No. 176389;
December 14, 2010)
People v. Abon (G.R. No.
169245; February 15,
2008)

84.

85.
86.

87.
88.

89.

90.
91.

92.

Batistis v. People (G.R.


No. 181571; December
16, 2009)
Sable v. People (G.R. No.
177961; April 7, 2009)
People v. Abesamis (G.R.
No. 140985; August 28,
2007)
Garces v. People (G.R. No.
173858; July 17, 2007)
Olalia, Jr. v. People (G.R.
No. 177276; August 20,
2008)
Constantino v.
Sandiganbayan (G.R. No.
140656; September 13,
2007)
Ching v. Nicdao (G.R. No.
141181; April 27, 2007)
Lubrica v. People (G.R.
Nos. 156147-54; February
26, 2007)
People v. Taruc (G.R. No.
185202; February 18,
2009)

CRIMINAL PROCEDURE
Case Digests Cassy C. Barras, 2D

CRIMINAL PROCEDURE
Case Digests Cassy C. Barras, 2D

People vs. Webb


G.R. No. 132577
August 17, 1999
Topic: Discovery procedures in
criminal cases
Doctrine: The use of discovery
procedures is directed to the
sound discretion of the trial judge.
Facts: The respondent filed a
motion to take the testimony by
oral
deposition
of
several
witnesses in the United States but
the motion was denied by the trial
court on the ground that the same
is not allowed by the Rules. The
subsequent
motion
for
reconsideration
was
likewise
denied.
Dissatisfied,
the
respondent elevated the case to
the Court of Appeals.
The CA set aside the ruling
of the trial court upon the
constitutional
ground
of
due
process and the right
of an
accused to present his evidence
and for the production of evidence
in his behalf and also on the theory
that the modes of discovery
likewise applies to criminal cases.
Issue: Whether or not the modes of
discovery
apply
to
criminal
proceedings
Ruling: The Supreme Court said
that
the
use
of
discovery
procedures is directed to the
sound discretion of the trial judge.
The deposition taking cannot be

based nor can it be denied on


flimsy reasons. Discretion has to
be exercised in a reasonable
manner and in consonance with
the spirit of the law. There is no
indication in this case that in
denying the motion of respondentaccused, the trial judge acted in a
biased, arbitrary, capricious or
oppressive manner. Grave abuse of
discretion implies such capricious,
and
whimsical
exercise
of
judgment as is equivalent to lack
of jurisdiction, or, in other words
where the power is exercised in an
arbitrary and despotic manner by
reason of passion or personal
hostility, and it must be so patent
and gross as to amount to an
evasion of positive duty or to a
virtual refusal to perform the duty
enjoined
or
to
act
all
in
contemplation of law.
The Supreme Court also
recognized the power of a court
to stop the introduction of further
testimony upon any particular
point when the evidence upon it is
already so full that more witnesses
to the same point cannot be
reasonably
expected
to
be
additionally persuasive.
Salvanera v. People
G.R. No. 143093
May 21, 2007
Topic: Discharged of an accused to
be a state witness
Doctrine: In the discharge of an
accused in order that he may be a

CRIMINAL PROCEDURE
Case Digests Cassy C. Barras, 2D

state witness, the conditions must


be present.
Facts:
petitioner
Rimberto
Salvanera, together with Feliciano
Abutin, Edgardo Lungcay and
Domingo Tampelix, is charged with
the murder of Ruben Parane. The
prosecution
moved
for
the
discharge of accused Feliciano
Abutin and Domingo Tampelix, to
serve as state witnesses. The trial
court denied the prosecutions
motion for the discharge of
accused Abutin and Tampelix. The
Court of Appeals sustained the
prosecution. It discharged accused
Feliciano Abutin and Domingo
Tampelix from the Information to
become state witnesses.
Petitioner
argues
that
prosecution witnesses Parane and
Salazar, who are not accused, do
not have personal knowledge of
the circumstances surrounding the
alleged conspiracy. Thus, they
could not testify to corroborate the
statement of Abutin and Tampelix
that petitioner is the mastermind
or the principal by induction.
Issue: Whether or not Abutin and
Tampelix can be state witnesses in
the case
Ruling: The Supreme Court ruled
in the affirmative.
In the discharge of an
accused in order that he may be a
state
witness,
the
following

conditions
must
be
present,
namely:
(1) Two or more accused are
jointly
charged
with
the
commission of an offense;
(2) The motion for discharge is
filed by the prosecution before it
rests its case;
(3) The prosecution is required to
present evidence and the sworn
statement of each proposed state
witness at a hearing in support of
the discharge;
(4) The accused gives his consent
to be a state witness; and
(5) The trial court is satisfied that:
a)
There
is
absolute
necessity for the testimony of the
accused
whose
discharge
is
requested;
b) There is no other direct
evidence available for the proper
prosecution
of
the
offense
committed, except the testimony of
said accused;
c) The testimony of said
accused can be substantially
corroborated in its material points;
d) Said accused does not
appear to be the most guilty; and
e) Said accused has not at
any time been convicted of any
offense involving moral turpitude.
The Supreme Court said that
the
corroborative
evidence
required by the Rules does not
have to consist of the very same
evidence as will be testified on by
the proposed state witnesses. The
Supreme Court reiterated its

CRIMINAL PROCEDURE
Case Digests Cassy C. Barras, 2D

former ruling that "a conspiracy is


more readily proved by the acts of
a fellow criminal than by any other
method. If it is shown that the
statements of the conspirator are
corroborated by other evidence,
then we have convincing proof of
veracity. Even if the confirmatory
testimony only applies to some
particulars, we can properly infer
that the witness has told the truth
in other respects." It is enough
that the testimony of a coconspirator is corroborated by
some other witness or evidence. In
the case at bar, the Supreme Court
is satisfied from a reading of the
records that the testimonies of
Abutin
and
Tampelix
are
corroborated on important points
by each others testimonies and
the
circumstances
disclosed
through the testimonies of the
other prosecution witnesses, and
"to
such
extent
that
their
trustworthiness
becomes
manifest."
As part of the conspiracy,
Abutin and Tampelix can testify on
the
criminal
plan
of
the
conspirators. Where a crime is
contrived in secret, the discharge
of one of the conspirators is
essential because only they have
knowledge of the crime. The other
prosecution witnesses are not
eyewitnesses to the crime, as, in
fact, there is none. No one except
the
conspirators
knew
and
witnessed
the
murder.
The

testimonies of the accused and


proposed state witnesses Abutin
and Tampelix can directly link
petitioner to the commission of the
crime.
The
decision
to
grant
immunity from prosecution forms a
constituent part of the prosecution
process. It is essentially a tactical
decision to forego prosecution of a
person for government to achieve
a higher objective.
People v. Ong
G.R. No. 175940
February 6, 2008
Topic: Order of trial
Doctrine: In Dangerous Drugs
Cases, it is the duty of the
prosecution to present a complete
picture detailing the buy-bust
operation.
Facts: Two separate Informations
were filed before the trial court. In
the first information, appellant was
accused of illegal sale of shabu. In
the second information, appellant
was
charged
with
illegal
possession
of
shabu.
Upon
arraignment, appellant pleaded
not guilty to both charges.
Col. Zoila Lachica was tipped
off by a female walk-in informant
that a group was engaged in drug
trafficking. Upon verification of the
information, Lachica organized a
team and planned the conduct of a
buy-bust operation.

CRIMINAL PROCEDURE
Case Digests Cassy C. Barras, 2D

The buy bust team went to


the Heritage Hotel where a drug
deal will take place. Upon signal
by the informant, Saballa, part of
the buy-bust team, came up to the
right front door of the hotel.
Saballa showed the contents of the
bag to the driver of the car, who
was later identified as appellant.
He then handed the bag to him.
Instantaneously, a man approached
the car, took the boodle money
from appellant and ran away.
Coballes, also part of the buy-bust
team, ran towards the drivers side
and poked his gun at appellant.
Coballes saw a red bag containing
white crystalline substance inside
the car and took it into custody.
Meanwhile, Lagradilla, part of the
buy-bust team, chased the man
who took the boodle money around
the parking area of Copacabana
Hotel. While on the run, Lagradilla
saw the man throw the money
inside a passing white Toyota car
driven by a certain Chito Cua.
Instead of pursuing the man,
Lagradilla blocked the white
Toyota car and arrested Cua.
Appellant on the other hand,
presented an entirely different
account of the incident.
Finding the testimonies of
the prosecution witnesses credible
as against the bare and selfserving assertions of appellant, the
trial court rendered a decision
finding
appellant
guilty
as
charged. The Court of Appeals

affirmed the decision of the RTC,


relying on the testimonies of the
police officers and dismissed the
imputed inconsistencies in their
statements as being minor.
Issue: Whether or not the evidence
of the prosecution proves all the
material details of the buy-bust
operation.
Ruling: The Supreme Court said
that evidence for the prosecution
failed to prove all the material
details of the buy-bust operation.
The details of the meeting with the
informant, the alleged source of
the information on the sale of
illegal drugs, appear hazy.
The actual exchange of the
bags containing shabu and the
boodle money was not clearly
established. Coballes testified that
he saw Saballa hand the boodle
money to appellant in exchange for
a wrapped object presumed to be
shabu. On the contrary, the
ultraviolet dusting of the boodle
money
was
conducted
but
appellant was found negative for
fluorescent powder.
As between the prosecution
witnesses account that it was
appellant to whom the boodle
money was passed and who was
driving the black Honda Civic car
during
the
alleged
buy-bust
operation and appellants denial
that he owned and drove said car,
the Supreme Court believed the

CRIMINAL PROCEDURE
Case Digests Cassy C. Barras, 2D

appellant. The prosecution failed


to present the purported drivers
license confiscated from appellant.
In fact, they reasoned that it was
missing. On the other hand, the
defense presented a certification
from the Land Transportation
Office and the Philippine Motor
Association stating that appellants
name does not exist in the LTOs
file of licensed drivers and has not
been
issued
a
Philippine
International Driving Permit by the
Automobile Association of the
Philippines.
Furthermore, Lachica, who
was the Chief of the Criminal
Investigation Division of the NCRCIDG, cannot seem to recall the
vital
parts
of
the
buy-bust
operation such as the composition
of the buy-bust team, the strategic
location of the team members, the
presence of the name of the other
accused, Cua, and how much of
the boodle money was recovered.
Thus, in acquitting the
appellant, the Supreme Court said
that in determining the credibility
of prosecution witnesses regarding
the conduct of buy-bust operation,
it is the duty of the prosecution to
present
a
complete
picture
detailing the buy-bust operation
from the initial contact between
the poseur-buyer and the pusher,
the offer to purchase, the promise
or payment of the consideration,
until the consummation of the sale

by the delivery of the illegal


subject of sale.
With the failure of the
prosecution to present a complete
picture of the buy-bust operation,
as highlighted by the disharmony
and incoherence in the testimonies
of its witnesses, acquittal becomes
ineluctable.
Cabarles v. Maceda
G.R. No. 161330
February 20, 2007
Topic:
Reopening
of
the
proceedings
Doctrine: A motion to reopen may
properly be presented only after
either or both parties had formally
offered and closed their evidence,
but before judgment is rendered,
and even after promulgation but
before finality of judgment.
Facts: Cabarles was charged with
murder.
Upon
arraignment,
Cabarles pleaded not guilty.
Through no fault of its own,
the prosecution was unable to
present its evidence on the first
four hearing dates. A day before
the scheduled promulgation of
judgment, Judge Maceda motu
proprio issued the questioned
order reopening the case. In it, he
observed that the prosecution may
not have been given its day in
court resulting in a miscarriage of
justice. He explained that because
there was a mix-up in the dates
specified in the subpoena and the

CRIMINAL PROCEDURE
Case Digests Cassy C. Barras, 2D

hearing dates of when the case


was
actually
heard,
the
prosecution was unable to present
its evidence on the first four of the
five hearing dates.
Judge
Maceda
denied
Cabarless
Motion
for
Reconsideration and set the case
for hearing on May 8, 2003 to hear
the testimonies of Pedrosa and Dr.
Salen. The subpoena issued to
Pedrosa for that hearing was duly
served, but service upon Dr. Salen
failed since the doctor was no
longer assigned to the SPD Crime
Laboratory. Notwithstanding the
service
upon
Pedrosa,
the
prosecution still failed to present a
witness during the May 8, 2003
hearing.
Nonetheless,
Judge
Maceda,
upon motion,
again
decided
to
extend
to
the
prosecution another chance, giving
the People June 19 and July 3,
2003 as additional hearing dates.
Finally, on June 19, 2003,
Pedrosa took the witness stand
and
completed
her
direct
examination.
A
few
days
thereafter, Cabarles filed the
present petition questioning Judge
Macedas order, alleging that it
was issued with grave abuse of
discretion. Since trial in the lower
court continued, on July 3, 2003,
the
Public
Attorneys
Office
conducted its cross-examination of
Pedrosa.
Cabarles was then given a
chance to adduce further evidence

on his behalf. On August 9, 2004,


Judge
Maceda
deferred
the
promulgation of judgment and
ordered the case archived pending
the Supreme Courts resolution of
the case.
Issue: Whether or not respondent
Judge acted with grave abuse of
discretion in issuing motu proprio
the Order reopening the case
Ruling: The Supreme Court ruled
that the respondent judge acted
with grave abuse of discretion.
Section 24, Rule 119 and
existing jurisprudence stress the
following
requirements
for
reopening
a
case:
(1)
the
reopening must be before the
finality
of
a
judgment
of
conviction; (2) the order is issued
by the judge on his own initiative
or upon motion; (3) the order is
issued only after a hearing is
conducted; (4) the order intends to
prevent a miscarriage of justice;
and (5) the presentation of
additional and/or further evidence
should be terminated within thirty
days from the issuance of the
order.
Generally, after the parties
have produced their respective
direct proofs, they are allowed to
offer rebutting evidence only.
However, the court, for good
reasons, in the furtherance of
justice, may allow new evidence
upon their original case, and its

CRIMINAL PROCEDURE
Case Digests Cassy C. Barras, 2D

ruling will not be disturbed in the


appellate court where no abuse of
discretion appears. A motion to
reopen may thus properly be
presented only after either or both
parties had formally offered and
closed their evidence, but before
judgment is rendered, and even
after promulgation but before
finality of judgment and the only
controlling guideline governing a
motion to reopen is the paramount
interest of justice. This remedy of
reopening a case was meant to
prevent a miscarriage of justice.
However,
while
Judge
Maceda is allowed to reopen the
case before judgment is rendered,
Section 24 requires that a hearing
must first be conducted. Judge
Maceda issued the April 1, 2003
Order without notice and hearing
and without giving the prosecution
and accused an opportunity to
manifest their position on the
matter. This failure constitutes
grave abuse of discretion and goes
against the due process clause of
the Constitution which requires
notice and opportunity to be
heard. The issuance of the said
order, without the benefit of a
hearing, is contrary to the express
language of Section 24, Rule 119.
The Supreme Court added
that Participation by the defense
counsel in cross-examining the
witness for the prosecution and in
the proceedings after the case was
reopened by the judge without

prior hearing does not amount to a


waiver of the accuseds objection
to the order reopening the case
to be effective, a waiver must be
certain and unequivocal.
Bernardo v. People
G.R. No. 166980
April 4, 2007
Topic: Trial in absentia
Doctrine: An accused may be tried
in absentia only when the he has
already been arraigned, he has
been duly notified of the trial and
his failure to appear is unjustified.
Facts: Petitioner was charged
before the MeTC with six counts of
violation of Batas Pambansa Blg.
22. Upon arraignment, petitioner,
assisted by a counsel de oficio,
pleaded "not guilty" to the offenses
charged.
At
the
pre-trial
conference petitioner failed to
appear despite notice, prompting
MeTC to issue a warrant of arrest
against him and set the cases for
trial in absentia.
After
the
prosecution
presented
its
first
witness,
petitioner filed a Waiver of
Appearance, a Motion to Lift
Warrant of Arrest, and a Motion to
Quash on the ground that the facts
charged in the Informations do not
constitute an offense. By Order,
the trial court lifted the warrant of
arrest in view of petitioners
appearance but denied the Motion
to Quash for lack of merit.

CRIMINAL PROCEDURE
Case Digests Cassy C. Barras, 2D

At the following trial date,


petitioner failed to appear despite
notice, drawing the trial court to
proceed with his trial in absentia
and issue warrant of arrest against
him. By Decision promulgated in
absentia, the trial court found
petitioner
guilty
beyond
reasonable doubt of violating B.P.
22 in all the cases. Ten months
following the promulgation of the
judgment, petitioner posted a bond
before another branch of the court.
Petitioner having been convicted
and no motion having been filed
for his provisional liberty pending
any appeal from or motion for
reconsideration of the Decision,
the trial court cancelled the bond
and issued an alias warrant of
arrest.
Issue: Whether or not petitioner
may be tried in absentia
Ruling: The Supreme Court held
that the requisites of a valid trial
in absentia are present in the
present case, namely: (1) the
accused
has
already
been
arraigned, (2) he has been duly
notified of the trial, and (3) his
failure to appear is unjustifiable.
The holding of trial in
absentia is authorized under
Section 14 (2), Article III of the
1987 Constitution which provides
that "after arraignment, trial may
proceed
notwithstanding
the
absence of the accused provided

that he has been duly notified and


his
failure
to
appear
is
unjustifiable."
The
Supreme
Court
reiterated its decision in People v.
Salas that escape should have
been considered a waiver of their
right to be present at their trial,
and the inability of the court to
notify them of the subsequent
hearings did not prevent it from
continuing with their trial. The
same fact of their escape made
their failure to appear unjustified
because they have, by escaping,
placed themselves beyond the pale
and
protection
of
the
law.
Thereafter, the trial court had the
duty to rule on the evidence
presented by the prosecution
against all the accused and to
render its judgment accordingly. It
should not wait for the fugitives
re-appearance or re-arrest. They
were deemed to have waived their
right to present evidence on their
own behalf and to confront and
cross-examine the witnesses who
testified against them.
People v. Joven de Grano
G.R. No. 167710
June 5, 2009
Topic:
Instances
when
the
presence of the accused is
required
Doctrine: The presence of the
accused
is
required
in
the
promulgation of sentence, unless it
is for a light offense, in which case,

CRIMINAL PROCEDURE
Case Digests Cassy C. Barras, 2D

the accused may appear


counsel or representative.

by

Facts: An Information for murder


committed
against
Emmanuel
Mendoza was filed with the
Regional Trial Court against Joven
de Grano, Armando de Grano, and
Estanislao Lacaba, together with
their
co-accused
Leonides
Landicho, Domingo Landicho, and
Leonardo Genil, who were at-large.
Duly arraigned, Joven, Armando,
and Estanislao pleaded not guilty
to
the
crime
as
charged.
Thereafter, respondents filed a
motion for bail contending that the
prosecutions evidence was not
strong. RTC granted the motion for
bail. The Court of Appels likewise
denied prosecutions petition for
certiorari. The Supreme Court,
however, granted the petition and
set aside the decision of the CA
together with the Order of the RTC
granting bail to the respondents.
After the presentation of the
parties
respective
sets
of
evidence, the RTC rendered a
Decision finding several accused
guilty of the offense as charged.
Only Estanislao was present at the
promulgation despite due notice to
the
other
respondents.
Respondents, thru counsel, then
filed
a
Joint
Motion
for
Reconsideration praying that the
Decision be reconsidered and set
aside and a new one be entered
acquitting them. In opposition, the

prosecution pointed out that while


the accused jointly moved for the
reconsideration of the decision, all
of them, except Estanislao, were
at-large. Having opted to become
fugitives and be beyond the
judicial ambit, they lost their right
to
file
such
motion
for
reconsideration and to ask for
whatever relief from the court. The
RTC modified its earlier decision
by acquitting Joven and Armando,
and downgrading the conviction of
Domingo and Estanislao from
murder to homicide.
Issue: Whether or not RTC acted
with grave abuse of discretion
when it rendered a decision
acquitting Joven and Armando
Ruling: Section 14(2), Article III of
the Constitution, authorizing trials
in absentia, allows the accused to
be absent at the trial but not at
certain stages of the proceedings,
to wit: (a) at arraignment and plea,
whether of innocence or of guilt;
(b)
during
trial,
whenever
necessary
for
identification
purposes;
and
(c)
at
the
promulgation of sentence, unless it
is for a light offense, in which case,
the accused may appear by
counsel or representative. At such
stages of the proceedings, his
presence is required and cannot be
waived.
In accordance with Section
6, Rule 120 of the Revised Rules of

CRIMINAL PROCEDURE
Case Digests Cassy C. Barras, 2D

Criminal Procedure, the accused


who failed to appear without
justifiable cause shall lose the
remedies available in the Rules
against the judgment. In blatant
disregard of the Rules, the RTC
not only failed to cause the arrest
of the respondents who were at
large, it also took cognizance of
the joint motion. The RTC clearly
exceeded its jurisdiction when it
entertained the joint Motion for
Reconsideration with respect to
the respondents who were at
large. It should have considered
the joint motion as a motion for
reconsideration that was solely
filed by Estanislao. Being at large,
Joven and Domingo have not
regained their standing in court.
Once an accused jumps bail or
flees to a foreign country, or
escapes
from
prison
or
confinement, he loses his standing
in court; and unless he surrenders
or submits to the jurisdiction of the
court, he is deemed to have waived
any right to seek relief from the
court.
Thus, the lower tribunal had
acted without jurisdiction. Verily,
any
ruling
issued
without
jurisdiction
is,
in
legal
contemplation, necessarily null
and void and does not exist. In
criminal cases, it cannot be the
source of an acquittal.
Resayo v. People
G.R. No. 154502

April 27, 2007


Topic: Rules on witness credibility
Doctrine:
It
is
settled
that
contradictions
between
the
contents of an affiants affidavit
and his testimony in the witness
stand do not always militate
against the witness credibility.
This is so because affidavits, which
are usually taken ex parte, are
often incomplete and inaccurate.
Facts: Resayo and Reyes were
charged
in
two
separate
Informations with the crimes of
homicide and frustrated homicide.
The trial court rendered a decision
finding the accused guilty as
charged. On appeal, the Court of
Appeals affirmed the trial courts
ruling.
The prosecution presented
nine witnesses, one of which is
Alfredo Braga. Resayo argues that
there is inconsistency in Bragas
statements, that he failed to state
in his written statement that he
and
Aguinaldo
were
merely
pacifying
their
other
group
members.
Issue:
Whether
or
not
the
inconsistencies
between
the
contents of Bragas affidavit and
his testimony in the witness stand
militate against his credibility.
Ruling: The Supreme Court said
that the contradictions between
the contents of an affiants
affidavit and his testimony in the

CRIMINAL PROCEDURE
Case Digests Cassy C. Barras, 2D

witness stand do not always


militate
against
the
witness
credibility. This is so because
affidavits, which are usually taken
ex parte, are often incomplete and
inaccurate. Likewise settled is the
rule that inconsistencies on minor,
trivial, and insignificant matters do
not affect the credibility of
witnesses. There is hardly a
witness
who
can
perfectly
remember the details of a crime.
Human memory is not as unerring
as a photograph. The Supreme
Court attributes Bragas lapses to
the natural fickleness of a mans
memory.
Ceniza-Manantan v. People
G.R. No. 156248
August 28, 2007
Topic: Rules on witness credibility
Doctrine: It is axiomatic that truth
is established not by the number of
witnesses but by the quality of
their testimonies.
Facts: Private complainant, Alberto
Carilla entered into an agreement
with sisters-in-law Vizconde and
Manantan that they would act as
the latters agent in selling the
pieces of jewelry. Accused sistersin-law failed to remit the purchase
price or return the pieces of
jewelry.
After
several
verbal
demands, the sisters-in-law issued
several checks. Vizconde issued
thirteen postdated checks, while
Manantan issued four postdated

checks.
The
checks
were
dishonored for the reason that the
account from which the checks
were drawn had been closed.
During her direct examination
before the RTC, Manantan denied
that
she
had
any
business
transaction with Carilla.
Petitioner Marissa CenizaManantan was convicted by the
RTC of estafa under paragraph
1(b), Article 315 of the Revised
Penal Code.
Manantan alleged that the
RTC conducted only one hearing
where the prosecution presented
only one witness, which was
Carilla himself, and thereafter
rested its case; that the said lone
hearing was abbreviated at the
expense of the rights and liberty of
Manantan;
that
the
direct
testimony of Carilla, upon which
the RTC based its conviction of
Manantan, consisted only of five
double-spaced pages as shown in
the transcript of stenographic
notes; and that Manantans guilt
cannot be proven on the basis of
the few questions propounded by
the private prosecutor on Carilla
and Manantan.
Issue: Whether or not petitioners
contentions are tenable
Ruling:
The Supreme Court
denied the petition.
It is axiomatic that truth is
established not by the number of

CRIMINAL PROCEDURE
Case Digests Cassy C. Barras, 2D

witnesses but by the quality of


their
testimonies.
In
the
determination of the sufficiency of
evidence, what matters is not the
number of witnesses but their
credibility and the nature and
quality of their testimonies. The
testimony of a lone witness, if
found positive and credible by the
trial court, is sufficient to support
a conviction especially when the
testimony bears the earmarks of
truth and sincerity. While the
number of witnesses may be
considered
a
factor
in
the
appreciation of evidence, proof
beyond reasonable doubt is not
necessarily with the greatest
number.
Witnesses are to be weighed,
not numbered; hence, it is not at
all
uncommon
to
reach
a
conclusion of guilt on the basis of
the testimony of a single witness.
Conviction of the accused may still
be had on the basis of the positive
and credible testimony of a single
witness.
People v. Mortera
G.R. No. 188104
April 23, 2010
Topic: Comments and questions of
the judge during the trial
Doctrine: The trial judge cannot be
faulted for having those remarks,
notwithstanding the sarcastic tone
it
impressed.
Sarcasm
alone
cannot lead to the conclusion that

the judge had taken the side of the


prosecution.
Facts: The Regional Trial Court
found accused Benancio Mortera
guilty beyond reasonable doubt of
the crime of murder for the killing
Robelyn Rojas.
Prosecution witness Ramil
Gregorio
testified
that
one
afternoon, he together with other
men were drinking tuba. They
have just started drinking when
Benancio Mortera, Jr. arrived. He
wanted to hit Alberto Rojas with a
Nescafe glass. Alberto Rojas ran
away. Mortera said, "Sayang." He
listened while the group of Ramil
Gregorio
were
singing
accompanied by a guitar. Jomer
Diaz, brother-in-law of Alberto
Diaz, arrived. Mortera said, "Here
comes another Rojas." Gregorio
and his companions told Jomer
Diaz to run away. Mortera hurled a
stone at Diaz but the latter was not
hit. Mortera left but he said that
he will return. After a few minutes,
Mortera came back. When Jomer
Diaz ran, Robelyn Rojas, brother of
Alberto Rojas went to Jomer.
Mortera met Robelyn at a distance
of about seven meters from the
place where the group were
drinking. Mortera and Robelyn
discussed with each other and
later shook hands. Robelyn turned
his face and Mortera suddenly
stabbed Robelyn Rojas at the back.
After stabbing Robelyn, Mortera

CRIMINAL PROCEDURE
Case Digests Cassy C. Barras, 2D

ran away. Robelyn Rojas tried to


chase Mortera but he was not able
to catch up but he fell down
mortally wounded. He was brought
to the hospital by his brother but
he was pronounced DOA at the
hospital. Jovel Veales who was
drinking together with Ramil
Gregorio and others, corroborated
Ramil Gregorio's testimony.
The RTC rendered judgment
finding the accused guilty of
murder. In rejecting the claim of
self-defense, the trial court stated
that it was not worthy of belief as
it was belied by the credible
testimonies of the prosecution
witnesses. The accused appealed
to the CA raising the issues of
denial of due process of law and
his right to an impartial trial. He
claimed that the trial court judge,
Judge Jesus Carbon, was hostile
towards him and prejudged his
guilt as could be inferred from his
"prosecutor-like" conduct.
In its decision, the CA
affirmed the decision of the RTC
with modification as to the civil
liabilities. The CA ruled that the
trial judge did not transgress the
standard
of
"cold
neutrality"
required of a magistrate and
added that the questions he
propounded were "substantially
clarificatory." Still not satisfied, the
accused now comes before the
Supreme Court.

Issue: Whether or not the accused


was denied of his right to have an
impartial trial
Ruling: As correctly pointed out by
the CA, although the trial judge
might
have
made
improper
remarks and comments, it did not
amount to a denial of his right to
due process or his right to an
impartial trial. Upon perusal of the
transcript as a whole, it cannot be
said that the remarks were
reflective of his partiality. Not only
did the accused mislead the court
by initially invoking a negative
defense only to claim otherwise
during trial, he was also not candid
to his own lawyer, who was kept in
the dark as to his intended
defense. The invocation of Opida
did not persuade the Supreme
Court. In Opida, Supreme Court
did not fail to notice the
"malicious,"
"sadistic"
and
"adversarial"
manner
of
questioning by the trial judge of
the accused therein, including
their defense witness. In Opida,
the accused never admitted the
commission of the crime, and so
the burden of proof remained with
the prosecution.
People v. Tuniaco, et al.
G.R. No. 185710
January 19, 2010
Topic: Corpus delicti in criminal
cases

CRIMINAL PROCEDURE
Case Digests Cassy C. Barras, 2D

Doctrine: Corpus delicti need not


be proved by an autopsy report of
the dead victims body or even by
the testimony of the physician who
examined such body.
Facts: The city prosecutor of
General Santos City charged the
accused Romulo Tuniaco, Jeffrey
Datulayta, and Alex Aleman with
murder before the Regional Trial
Court.
The defense claims that the
prosecution failed to prove corpus
delicti since it did not bother to
present a medical certificate
identifying the remains found at
the dump site and an autopsy
report showing such remains
sustained
gunshot
and
stab
wounds that resulted in death; and
the shells of the guns used in
killing the victim.
Issue:
Whether
or
not
the
prosecution was able to present
evidence of corpus delicti
Ruling: The Supreme Court said
that corpus delicti need not be
proved by an autopsy report of the
dead victims body or even by the
testimony of the physician who
examined such body. While such
report or testimony is useful for
understanding the nature of the
injuries the victim suffered, they
are not indispensable proof of such
injuries or of the fact of death. Nor
is the presentation of the murder

weapons also indispensable since


the physical existence of such
weapons is not an element of the
crime of murder.
Here, the police authorities
found the remains of Cortez at the
place pointed to by accused
Aleman.
That
physical
confirmation, coming after his
testimony of the gruesome murder,
sufficiently establishes the corpus
delicti of the crime. Of course, that
statement must be admissible in
evidence.
Corpus delicti has been
defined as the body, foundation, or
substance of a crime. The evidence
of a dead body with a gunshot
wound on its back would be
evidence that murder has been
committed. Corpus delicti has two
elements: (a) that a certain result
has been established, for example,
that a man has died and (b) that
some
person
is
criminally
responsible for it. The prosecution
is burdened to prove corpus delicti
beyond reasonable doubt either by
direct
evidence
or
by
circumstantial
or
presumptive
evidence.
People v. Villanueva
G.R. No. 181829
September 1, 2010
Topic: Lack of formal offer of
evidence during the trial
Doctrine: A formal offer of
evidence is necessary because
judges are mandated to rest their

CRIMINAL PROCEDURE
Case Digests Cassy C. Barras, 2D

findings
of
facts
and
their
judgment only and strictly upon
the evidence offered by the parties
at the trial.
Facts:
Regional
Trial
Court
convicted appellant guilty beyond
reasonable doubt of three counts
of rape.
Appellant claimed that the
prosecution failed to present
evidence that would overcome the
presumption of his innocence.
Appellant
argued
that
the
prosecution failed to formally offer
in evidence the medical certificate
and to present the doctor who
conducted
the
medical
examination to testify on his
findings. Likewise, AAAs birth
certificate
was
not
formally
offered. Neither did the Municipal
Civil Registrar who allegedly
prepared the same take the
witness stand. Thus appellant
claimed that assuming he was
indeed guilty of the crimes
charged, he should only be held
liable for simple rape and not
qualified
rape
because
the
minority of the victim was not duly
established.
The appellate court found no
reason to reverse the findings of
the trial court on the credibility of
AAA.
Issue: Whether or not

Ruling:
The
Supreme
Court
affirmed
CAs
decision
that
appellant could be convicted of
rape even without the medical
certificate. In rape cases, the
accused may be convicted solely
on the testimony of the victim,
provided the testimony is credible,
natural, convincing, and consistent
with human nature and the normal
course of things. AAAs testimony
was credible and convincing. As
such, appellants conviction could
rest solely on it. The medical
certificate would only serve as
corroborative evidence.
The
Supreme
Court,
however, agree with the appellant
that both the medical certificate
and
AAAs
birth
certificate,
although
marked
as
exhibits
during the pre-trial, should not
have been considered by the trial
court and the CA because they
were not formally offered in
evidence. Section 34, Rule 132 of
the Rules of Court explicitly
provides: The court shall consider
no evidence which has not been
formally offered. The purpose for
which the evidence is offered must
be specified. In this case, the
Supreme Court notes that after the
marking of the exhibits during pretrial, the prosecution did not
formally offer the said medical
certificate or birth certificate in
evidence. In fact, the prosecution
rested its case after presenting the
testimony of AAA without formally

CRIMINAL PROCEDURE
Case Digests Cassy C. Barras, 2D

offering any documentary exhibit


at all.
In view of the foregoing, the
Supreme Court finds that the
prosecution did not present any
satisfactory evidence to prove
AAAs
minority.
Qualifying
circumstances or special qualifying
circumstances must be proved
with equal certainty and clearness
as the crime itself; otherwise,
there can be no conviction of the
crime in its qualified form. As a
qualifying circumstance of the
crime of rape, the concurrence of
the victims minority and her
relationship
to
the
accusedappellant must be both alleged and
proven beyond reasonable doubt.
Thus, appellant is guilty only of
three counts of simple rape.
Nicolas v. Sandiganbayan
G.R. Nos. 175930-31
February 11, 2008
Topic: Demurrer to evidence
Doctrine: A demurrer to evidence
is an objection by one of the
parties in an action to the effect
that the evidence which his
adversary produced is insufficient
in point of law to make out a case
or sustain the issue.
Facts: Nicolas and Arriola, former
Commissioner
and
Deputy
Commissioner, respectively, of the
Economic
Intelligence
and
Investigation
Bureau,
stand
charged before public respondent

for violation of Section 3604 of the


Tariff and Customs Code in the
first case, and Section 3(e) of the
Anti-Graft and Corrupt Practices
Act or Republic Act (R.A.) No.
3019 in the second.
A 40-footer container van
was suspected to be carrying
undeclared goods, was seized by
EIIB
operatives
under
the
command of Arriola. The van was
turned over for safekeeping to the
Armed Forces of the Philippines
Logistics Command compound in
Quezon City. However, the van was
released by military police from
the
LOGCOM
compound
to
representatives of the EIIB and
Trinity Brokerage. While the van
was heading to the docks for
shipment to the alleged consignee,
it surreptitiously exited at the
North harbor with its cargo. It has
since been missing.
Through its testimonial and
documentary
evidence,
the
prosecution attempted to show
that the withdrawal of the van
from the LOGCOM compound was
based on a Notice of Withdrawal
signed by Nicolas, and on the
Authority for the withdrawal of the
van which, though it appeared to
have been issued by Uy, was not
actually signed by him. The
prosecution likewise attempted to
establish that the documents,
including
Official
Receipts
allegedly
presented
to
show

CRIMINAL PROCEDURE
Case Digests Cassy C. Barras, 2D

payment of customs duties and


taxes, were all spurious.
Petitioners separately filed
motions for Leave of Court to File
Demurrer to Evidence with Motion
to Admit Attached Demurrer to
Evidence. They argued that the
prosecution was not only unable to
show
that
they
were
the
perpetrators of the crimes charged
or that they committed any
prohibited act; it was also not able
to prove that undue injury was
caused the government.
Issue: Whether or not petitioners
separate Demurrer to Evidence
should be granted
Ruling:
The
Supreme
Court
granted
petitioners
separate
Demurrer to Evidence on the
ground
that
the
evidence
presented by the prosecution
against petitioners does not prima
facie prove petitioners culpability
beyond reasonable doubt, the
burden of evidence did not shift to
the defense. The Court thus finds
that public respondent gravely
abused its discretion in denying
their Demurrer to Evidence.
A demurrer to evidence is an
objection by one of the parties in
an action to the effect that the
evidence which his adversary
produced is insufficient in point of
law to make out a case or sustain
the issue. The party filing the
demurrer in effect challenges the

sufficiency
evidence.

of

the

prosecutions

People v. Crespo
G.R. No. 180500
September 11, 2008
Topic: Demurrer to evidence with
leave of court
Doctrine: The power to grant leave
to the accused to file a demurrer is
addressed to the sound discretion
of the trial court. The purpose is to
determine whether the accused in
filing his demurrer is merely
stalling the proceedings.
Facts: Appellant Medardo Crespo y
Cruz was charged in eight
amended Informations with the
crime of rape committed against
his own daughter AAA. Upon
arraignment,
the
appellant,
assisted by counsel de parte,
pleaded not guilty to all the
charges against him.
After the prosecution had
rested its case, appellant filed a
Motion for Leave of Court to File
Demurrer to Evidence on the
ground of insufficiency of evidence
of the prosecution; the motion was
granted.
Despite
several
extensions given, within which to
file the aforesaid Demurrer to
Evidence, appellant failed to
submit one. He filed a last and
final motion for extension to
submit the demurrer to evidence,
but the same was denied. The

CRIMINAL PROCEDURE
Case Digests Cassy C. Barras, 2D

Motion for Reconsideration of the


appellant was likewise denied.
Both Regional Trial Court
and the Court of Appeals convicted
appellant guilty beyond reasonable
doubt.
Appellant alleges that the
court a quo committed grave
abuse of discretion in denying his
Urgent Ex-Parte Motion for Last
and Final Extension of Time to File
Demurrer to Evidence.
Issue: Whether or not the trial
judge committed grave abuse of
discretion amounting to lack or
excess in jurisdiction for denying
appellants petition
Ruling: The Supreme Court ruled
that, as aptly found that the
appellate court, the court a quo
had already granted the appellants
Motion for Extension of Time to
File Demurrer to Evidence twice.
In fact, it had already given the
appellant a total of 20 days within
which to file his Demurrer to
Evidence. It was only on the third
Motion for Extension of Time to
File Demurrer to Evidence that the
trial court denied the same.
Considering the several extensions
prayed for by the appellant, the
Supreme Court cannot fault the
trial court for finally denying the
Motion for Extension of Time to
File Demurrer to Evidence filed by
the appellant.

The Supreme Court likewise


stated that the power to grant
leave to the accused to file a
demurrer is addressed to the
sound discretion of the trial court.
The purpose is to determine
whether the accused in filing his
demurrer is merely stalling the
proceedings. Unless there is grave
abuse thereof amounting to lack or
excess of jurisdiction, which is not
present in the instant case, the
trial courts denial of prior leave to
file demurrer to evidence may not
be
disturbed.
Moreover,
the
Supreme
Court
is
in
full
conformity with the appellate
court that concomitant with the
right of the accused to a speedy
trial is the right of the victim to
obtain justice without delay. To
allow and grant every motion for
extension of time would unduly
delay the process of administering
and dispensing justice.

People v. Tolentino
G.R. No. 176385
February 26, 2008
Topic: Demurrer to evidence
without leave of court
Doctrine: When an accused files a
demurrer to evidence without
leave of court and the same is
denied, he waives his right to
present evidence and submits the
case for judgment on the basis of
the evidence of the prosecution.

CRIMINAL PROCEDURE
Case Digests Cassy C. Barras, 2D

Facts: Three separate informations


of Murder and two counts of
Frustrated Murder were filed
before the RTC against appellants,
together with accused Jimmy
Trinidad and Arnel Trinidad.
During
the
arraignment,
appellants entered their respective
pleas of not guilty. Accused Jimmy
and Arnel Trinidad remained at
large.
After
the
prosecution
presented its evidence against the
appellants,
appellants
through
counsel filed a Demurrer to
Evidence, without leave of court.
The RTC denied the demurrer and
submitted the case for decision
pursuant to Section 15, Rule 119
of the 1985 Rules on Criminal
Procedure. Appellants filed a
motion
for
reconsideration,
praying that the order denying
their Demurrer to Evidence be
recalled and that they be allowed
to present evidence. The RTC
denied the said motion. Unfazed,
appellants filed a petition for
certiorari before the Supreme
Court. The Supreme Court denied
the petition in a Resolution which
became final and executory. As a
result, the case was submitted for
decision without any evidence
proffered by the defense. The RTC
rendered
a
decision
finding
appellants guilty of murder and
frustrated murder, respectively.
Issue: Whether or not the court a
quo gravely erred in not allowing

the accused-appellants to present


defense evidence after the denial
of the demurrer to evidence
considering the possibility of the
imposition of the death penalty
Ruling: The Court said that when
the trial court disallowed the
appellants to present evidence on
their behalf, it properly applied
Section 15, Rule 119 of the Rules
of Criminal Procedure, which
provides that when an accused
files a demurrer to evidence
without leave of court and the
same is denied, he waives his right
to present evidence and submits
the case for judgment on the basis
of the evidence of the prosecution..
Not even the gravity of the penalty
for a particular offense can change
this rule.
The filing of a demurrer to
evidence without leave of court is
an unqualified waiver of the right
to present evidence for the
accused. The rationale for this rule
is that when the accused moves for
dismissal on the ground of
insufficiency of evidence of the
prosecution evidence, he does so
in the belief that said evidence is
insufficient
to
convict
and,
therefore, any need for him to
present any evidence is negated.
An accused cannot be allowed to
wager on the outcome of judicial
proceedings
by
espousing
inconsistent viewpoints whenever
dictated by convenience. The

CRIMINAL PROCEDURE
Case Digests Cassy C. Barras, 2D

purpose behind the rule is also to


avoid the dilatory practice of filing
motions
for
dismissal
as
a
demurrer to the evidence and,
after denial thereof, the defense
would then claim the right to
present its evidence.
Hun Hyung Park v. Eung Won
Choi
G.R. No. 165496
February 12, 2007
Topic: Demurrer to evidence with
leave of court
Doctrine: A court may not deny the
demurrer as to the criminal aspect
and at the same time grant the
demurrer as to the civil aspect, for
if the evidence so far presented is
not insufficient to prove the crime
beyond reasonable doubt, then the
same evidence is likewise not
insufficient
to
establish
civil
liability by mere preponderance of
evidence.
Facts: Eung Won Choi was charged
for violation of BP 22 for issuing
PNB Check No. 0077133 which
was dishonored for having been
drawn against insufficient funds.
He pleaded not guilty.
After the prosecution rested
its case, respondent filed a Motion
for Leave of Court to File
Demurrer to Evidence to which he
attached his Demurrer, asserting
that the prosecution failed to prove
that he received the notice of
dishonor, hence, the presumption

of the element of knowledge of


insufficiency of funds did not arise.
The MeTC granted the
demurrer and dismissed the case.
The prosecutions motion for
reconsideration was denied.
Park appealed the civil
aspect of the case to the RTC
contending that the dismissal of
the criminal case should not
include its civil aspect. The RTC
held that while the evidence
presented was insufficient to prove
Chois criminal liability, it did not
altogether extinguish his civil
liability. It accordingly granted
Parks appeal and ordered Choi to
pay him P1,875,000 with legal
interest.
Upon Chois motion for
reconsideration, however, the RTC
set aside its decision and ordered
the remand of the case to the
MeTC for further proceedings, so
that Choi may adduce evidence on
the civil aspect of the case. Parks
motion for reconsideration of the
remand of the case having been
denied, he elevated the case to the
CA which dismissed his petition.
Issue:
Whether
or
not
the
respondent has a right to present
evidence on the civil aspect of the
case in view of his demurrer
Ruling:
The
Supreme
Court
upholds respondents right to
present evidence as reserved by

CRIMINAL PROCEDURE
Case Digests Cassy C. Barras, 2D

his filing of leave of court to file


the demurrer.
In case of a demurrer to
evidence filed with leave of court,
in case of a demurrer to evidence
filed with leave of court, the
accused
may
adduce
countervailing evidence if the
court denies the demurrer. Such
denial bears no distinction as to
the two aspects of the case
because there is a disparity of
evidentiary value between the
quanta of evidence in such aspects
of the case. In other words, a
court may not deny the demurrer
as to the criminal aspect and at the
same time grant the demurrer as
to the civil aspect, for if the
evidence so far presented is not
insufficient to prove the crime
beyond reasonable doubt, then the
same evidence is likewise not
insufficient
to
establish
civil
liability by mere preponderance of
evidence.
On the other hand, if the
evidence so far presented is
insufficient
as
proof
beyond
reasonable doubt, it does not
follow that the same evidence is
insufficient
to
establish
a
preponderance of evidence. For if
the court grants the demurrer,
proceedings on the civil aspect of
the case generally proceed. The
only recognized instance when an
acquittal on demurrer carries with
it the dismissal of the civil aspect
is when there is a finding that the

act or omission from which the


civil liability may arise did not
exist. Absent such determination,
trial as to the civil aspect of the
case must perforce continue.
In the instant case, the
MeTC granted the demurrer and
dismissed the case without any
finding that the act or omission
from which the civil liability may
arise did not exist. Choi did not
assail the RTC order of remand.
He thereby recognized that there
is basis for a remand.
Cabador v. People
G.R. No. 186001
October 2, 2009
Topic: Not every motion to dismiss
is a demurrer to evidence
Doctrine: To determine whether
the pleading filed is a demurer to
evidence or a motion to dismiss,
the Court must consider (1) the
allegations in it made in good
faith; (2) the stage of the
proceeding at which it is filed; and
(3) the primary objective of the
party filing it.
Facts: The public prosecutor
accused
petitioner
Antonio
Cabador before the RTC of
murdering, in conspiracy with
others, Atty. Jun N. Valerio. After
presenting only five witnesses over
five years of intermittent trial, the
RTC declared at an end the
prosecutions
presentation
of
evidence
and
required
the

CRIMINAL PROCEDURE
Case Digests Cassy C. Barras, 2D

prosecution to make a written or


formal offer of its documentary
evidence.
But
the
public
prosecutor
asked
for
three
extensions of time. Still, the
prosecution did not make the
required written offer.
Cabador filed a motion to
dismiss the case invoking his right
to a speedy trial. He claimed that
in the circumstances, the trial
court could not consider any
evidence against him that had not
been formally offered. He also
pointed out that the prosecution
witnesses did not have knowledge
of his alleged part in the crime
charged.
Unknown to Cabador, the
prosecution asked the RTC for
another extension of the period for
its formal offer, which offer it
eventually made on August 1,
2006, the day Cabador filed his
motion to dismiss.
The RTC issued an Order
treating Cabadors motion to
dismiss as a demurrer to evidence.
And, since he filed his motion
without leave of court, the RTC
declared him to have waived his
right to present evidence in his
defense. The trial court deemed
the case submitted for decision
insofar as he was concerned.
Cabador filed a motion for
reconsideration of this Order but
the RTC denied it. The Court of
Appeals affirmed the lower courts
actions.

Issue: Whether or not Cabadors


motion
to
dismiss
can
be
considered as a demurrer to
evidence
Ruling: The Court finds that
Cabador filed a motion to dismiss
on the ground of violation of his
right to speedy trial, not a
demurrer to evidence. He cannot
be declared to have waived his
right to present evidence in his
defense.
To determine whether the
pleading filed is a demurer to
evidence or a motion to dismiss,
the Court must consider (1) the
allegations in it made in good
faith; (2) the stage of the
proceeding at which it is filed; and
(3) the primary objective of the
party filing it.
In this case, it can be seen
from Cabadors motion to dismiss
that petitioner took pains to point
out how trial in the case had
painfully dragged on for years. The
gaps between proceedings were
long,
with
hearings
often
postponed
because
of
the
prosecutors absence. This was
further
compounded,
Cabador
said, by the prosecutions repeated
motions for extension of time to
file its formal offer and its failure
to file it within such time. Cabador
then invoked his right to speedy
trial.

CRIMINAL PROCEDURE
Case Digests Cassy C. Barras, 2D

Cabadors
so-called
"demurrer" did not touch on any
particular testimony of even one
witness. He cited no documentary
exhibit. Indeed, he could not
because, he did not know that the
prosecution finally made its formal
offer of exhibits on the same date
he filed his motion to dismiss.
Besides, a demurrer to evidence
assumes that the prosecution has
already rested its case. Since
Cabador filed his motion to dismiss
before he could object to the
prosecutions formal offer, before
the trial court could act on the
offer, and before the prosecution
could rest its case, it could not be
said that he had intended his
motion to dismiss to serve as a
demurrer to evidence.
People v. Sandiganbayan
G.R. No. 164577
July 5, 2010
Topic: Granting of demurrer is an
acquittal
Doctrine: The prosecution cannot
appeal from a ruling granting the
demurrer to evidence of the
accused as it is equivalent to an
acquittal.
Facts:
Private
respondents
Victorino A. Basco, Romeo S. David
and Rogelio L. Luis were charged
with having violated Section 3(e)
of Anti-Graft and Corrupt Practices
Act before the Sandiganbayan.

During
the
trial,
the
prosecution presented its lone
witness, Atty. Emora C. Pagunuran,
Legal Counsel, Office of the Legal
Affairs, Office of the Ombudsman.
Thereafter, the prosecution filed its
Formal Offer of Evidence. After the
evidence were admitted, the
prosecution rested its case.
Instead of presenting their
evidence, the respondents filed
their respective motions for leave
to file their demurrer to evidence
based
substantially
on
the
following grounds: (i) that Atty.
Pagunuran
had
no
personal
knowledge of the transactions
involved and so her testimony was
hearsay; (ii) that the prosecution
failed to prove that the questioned
contracts were indeed overpriced
as Atty. Pagunuran merely relied
on the Department of Public Works
and Highways table of Typical
Construction Costs, 1999 without
more; and (iii) that the ruling of
the Court of Appeals in an
administrative case, which upheld
the validity of the direct negotiated
contracts, even in the absence of a
public bidding, was already the
law of the case. The motions were
granted and the Sandiganbayan
directed the prosecution to file its
opposition.
Issue: Whether or not the grant of
demurrer to evidence is subject to
appeal

CRIMINAL PROCEDURE
Case Digests Cassy C. Barras, 2D

Ruling:
The
dismissal
order
consequent to a demurrer to
evidence is not subject to appeal,
however, it is still reviewable but
only by certiorari under Rule 65 of
the Rules of Court. In such a case,
the factual findings of the trial
court are conclusive upon the
reviewing court, and the only legal
basis to reverse and set aside the
order of dismissal upon demurrer
to evidence is by a clear showing
that the trial court, in acquitting
the accused, committed grave
abuse of discretion amounting to
lack or excess of jurisdiction or a
denial of due process, thus,
rendering the assailed judgment
void.
The demurrer to evidence in
criminal cases, such as the one at
bench,
is
filed
after
the
prosecution had rested its case. As
such, it calls for an appreciation of
the evidence adduced by the
prosecution and its sufficiency to
warrant
conviction
beyond
reasonable doubt, resulting in a
dismissal of the case on the merits,
tantamount to an acquittal of the
accused. Judicial action on a
motion to dismiss or demurrer to
evidence is best left to the exercise
of
sound
judicial
discretion.
Accordingly,
unless
the
Sandiganbayan
acted
without
jurisdiction or with grave abuse of
discretion, its decision to grant or

deny the demurrer may not be


disturbed.
Not surprisingly, petitioner
has not attributed any commission
of grave abuse of discretion on the
part of Sandiganbayan in issuing
the questioned resolution, on the
mistaken assumption that it can
assail the resolution on purely
legal questions. A judgment of
acquittal cannot be reopened or
appealed because of the doctrine
that nobody can be put twice in
jeopardy for the same offense.
People v. Tan
G.R. No. 167526
July 26, 2010
Topic: Granting of demurrer is an
acquittal
Doctrine: The grant of a demurrer
to evidence operates as an
acquittal and is, thus, final and
unappealable.
Facts:
Two
Informations
for
violation of Rule 36 (a)-1, in
relation to Sections 32 (a)-1 and 56
of the Revised Securities Act, were
filed by petitioner People of the
Philippines against respondent
Dante Tan in the Regional Trial
Court.
After
arraignment,
respondent pleaded not guilty to
both charges and the trial ensued.
Respondent filed an Omnibus
Motion for Leave to File Demurrer
to Evidence and to admit the
attached Demurrer to Evidence.

CRIMINAL PROCEDURE
Case Digests Cassy C. Barras, 2D

RTC granted respondents motion.


Petitioner filed a Petition for
Certiorari before the CA but it was
denied on the ground that the
dismissal of a criminal action by
the grant of a Demurrer to
Evidence is one on the merits and
operates as an acquittal, for which
reason, the prosecution cannot
appeal therefrom as it would place
the accused in double jeopardy.
Issue: Whether or not CA gravely
erred in precluding the people
from prosecuting its cases against
Tan
Ruling: The Supreme Court ruled
in the negative, thus denied the
petition.
The Supreme Court reiterated the
ruling in People v. Sandiganbayan
that the grant of a demurrer to
evidence operates as an acquittal
and
is,
thus,
final
and
unappealable,
to
wit:
The
demurrer to evidence in criminal
cases, such as the one at bar, is
"filed after the prosecution had
rested its case," and when the
same is granted, it calls "for an
appreciation of the evidence
adduced by the prosecution and its
sufficiency to warrant conviction
beyond
reasonable
doubt,
resulting in a dismissal of the case
on the merits, tantamount to an
acquittal of the accused." Such
dismissal of a criminal case by the
grant of demurrer to evidence may

not be appealed, for to do so would


be to place the accused in double
jeopardy. The verdict being one of
acquittal, the case ends there.
People v. Sandiganbayan
G.R. No. 174504
March 21, 2011
Topic: Granting of demurrer is an
acquittal
Doctrine: If the order was issued
with grave abuse of discretion
amounting to lack of jurisdiction, it
may be reviewed by a petition for
certiorari under Rule 65.
Facts: Private respondent was
charged with violation of Section
89 of Presidential Decree No.
14452 before the Sandiganbayan.
The prosecution presented
its lone witness, Manolo Tulibao
Villad, Commission on Audit State
Auditor.
Thereafter,
the
prosecution filed its formal offer of
evidence and rested its case.
Private respondent filed a motion
for leave to file demurrer to
evidence.
The
Sandiganbayan
issued a Resolution granting the
motion.
Issue: Whether or not the grant of
demurrer to evidence is subject of
appeal
Ruling: In criminal cases, the grant
of a demurrer is tantamount to an
acquittal and the dismissal order
may not be appealed because this

CRIMINAL PROCEDURE
Case Digests Cassy C. Barras, 2D

would place the accused in double


jeopardy. Although the dismissal
order is not subject to appeal, it is
still reviewable but only through
certiorari under Rule 65 of the
Rules of Court. For the writ to
issue, the trial court must be
shown to have acted with grave
abuse of discretion amounting to
lack or excess of jurisdiction such
as where the prosecution was
denied the opportunity to present
its case or where the trial was a
sham thus rendering the assailed
judgment void.
In the case at bar, the
Sandiganbayan
granted
the
demurrer to evidence on the
ground that the prosecution failed
to prove that the government
suffered any damage from private
respondents non-liquidation of the
subject cash advance because it
was later shown, as admitted by
the prosecutions witness, that
private respondent liquidated the
same albeit belatedly.
Contrary to the findings of
the Sandiganbayan, actual damage
to the government arising from the
non-liquidation
of
the
cash
advance is not an essential
element of the offense punished
under the second sentence of
Section 89 of P.D. No. 1445 as
implemented by COA Circular No.
90-331. Instead, the mere failure
to timely liquidate the cash
advance is the gravamen of the
offense. Verily, the law seeks to

compel the accountable officer, by


penal
provision,
to
promptly
render an account of the funds
which he has received by reason of
his office.
Nonetheless, even if the
Sandiganbayan proceeded from an
erroneous interpretation of the law
and its implementing rules, the
error committed was an error of
judgment and not of jurisdiction.
Petitioner failed to establish that
the dismissal order was tainted
with grave abuse of discretion
such as the denial of the
prosecutions right to due process
or the conduct of a sham trial. In
fine, the error committed by the
Sandiganbayan is of such a nature
that can no longer be rectified on
appeal by the prosecution because
it would place the accused in
double jeopardy.
CHAPTER
XI:
JUDGMENT,
REMEDIES AFTER JUDGMENT
OF
CONVICTION
AND
PROVISIONAL REMEDIES
Antiporda, Jr. v. Garchitorena
G.R. No. 133289
December 23, 1999
Topic: Requisites of jurisdictional
requirements
Doctrine:
The
jurisdictional
requirements before a judgment
may be validly rendered are
jurisdiction
over
the
subject
matter, the territory and the
person of the accused.

CRIMINAL PROCEDURE
Case Digests Cassy C. Barras, 2D

Facts: Accused Mayor Licerio


Antiporda
and
others
were
charged
for
the
crime
of
kidnapping. The case was filed in
the first division of Sandiganbayan.
Subsequently, the Court ordered
the
prosecution
to
submit
amended information, which was
complied evenly and the new
information contained the place
where the victim was brought.
The accused filed an Urgent
Omnibus Motion praying that a
reinvestigation be conducted and
the issuance of warrants of arrest
be deferred but it was denied by
the Ombudsman.
The accused
thereafter filed a Motion for New
Preliminary investigation and to
hold in abeyance and/or recall
warrant of arrest issued but the
same
was
also
denied.
Subsequently, the accused filed a
Motion
to
Quash
Amended
Information for lack of jurisdiction
over the offense charged, which
was ignored for their continuous
refusal to submit their selves to
the Court and after their voluntary
appearance which invested the
Sandiganbayan jurisdiction over
their persons, their motion for
reconsideration was again denied.
Issue:
Whether
or
not
the
Sandiganbayan had jurisdiction
over the offense charged
Ruling: The Supreme Court held
that
the
Sandiganbayan
has

jurisdiction over the case because


of estoppel and it was thus vested
with the authority to order the
amendment of the Information.
The original Information filed
with the Sandiganbayan did not
mention
that
the
offense
committed by the accused is officerelated. It was only after the same
was filed that the prosecution
belatedly remembered that a
jurisdictional fact was omitted
therein.
However, the Supreme Court
held that the petitioners are
estopped
from
assailing
the
jurisdiction of the Sandiganbayan
for in the supplemental arguments
to motion for reconsideration
and/or reinvestigation filed with
the same court, it was they who
challenged the jurisdiction of the
Regional Trial Court over the case
and clearly stated in their Motion
for Reconsideration that the said
crime is work connected.
It is a well-settled rule that a
party
cannot
invoke
the
jurisdiction of a court to secure
affirmative
relief
against
his
opponent, and after obtaining or
failing to obtain such relief,
repudiate or question that same
jurisdiction.
Cruz v. Court of Appeals
G.R. No. 123340
August 29, 2002
Topic: Requisites of jurisdictional
requirements

CRIMINAL PROCEDURE
Case Digests Cassy C. Barras, 2D

Doctrine:
The
jurisdictional
requirements before a judgment
may be validly rendered are
jurisdiction
over
the
subject
matter, the territory and the
person of the accused.
Facts: The City Prosecutor of
Manila charged petitioner with the
crime of Estafa thru Falsification of
Public
Document
before
the
Manila Regional Trial Court.
Petitioner executed before a
Notary Public in the City of Manila
an Affidavit of Self-Adjudication of
a parcel of land stating that she
was the sole surviving heir of the
registered owner when in fact she
knew there were other surviving
heirs. Since the offended party did
not reserve the right to file a
separate civil action arising from
the criminal offense, the civil
action was deemed instituted in
the criminal case. After trial on the
merits, the trial court acquitted
the petitioner on the ground of
reasonable doubt.
Petitioner filed by registered
mail a motion for reconsideration
assailing the trial courts ruling on
the civil aspect of the criminal
case. The trial court denied
petitioners motion on the ground
that there is nothing to show that
the Office of the City Prosecutor
was actually furnished or served
with a copy of the said Motion for
Reconsideration
within
the
reglementary period of fifteen days

from receipt by the accused of a


copy of the Courts decision. Thus,
the decision becomes final and
executory.
The Court of Appeals denied
due course to the petition and
dismissed the case for being
insufficient in substance. The CA
likewise held that proposition
submitted by petitioner that the
court presided by respondent
Judge had no jurisdiction over the
property because it is located in
Bulacan - outside the territorial
jurisdiction of said court - does not
hold water. Being a civil liability
arising from the offense charged,
the governing law is the Rules of
Criminal Procedure, not the civil
procedure rules which pertain to
civil action arising from the
initiatory pleading that gives rise
to the suit.
Issue: Whether or not the CA erred
in finding that the RTC of Manila
had
jurisdiction
to
render
judgment on the civil aspect
Ruling: The Supreme Court held
that the RTC of Manila had
jurisdiction over the case.
There are three important
requisites which must be present
before a court can acquire criminal
jurisdiction. First, the court must
have jurisdiction over the subject
matter. Second, the court must
have jurisdiction over the territory
where the offense was committed.

CRIMINAL PROCEDURE
Case Digests Cassy C. Barras, 2D

Third, the court must have


jurisdiction over the person of the
accused. In the instant case, the
trial court had jurisdiction over the
subject matter as the law has
conferred on the court the power
to hear and decide cases involving
estafa through falsification of a
public document. The trial court
also had jurisdiction over the
offense charged since the crime
was
committed
within
its
territorial jurisdiction. The trial
court also acquired jurisdiction
over the person of accusedpetitioner because she voluntarily
submitted to the courts authority.
Where
the
court
has
jurisdiction over the subject matter
and over the person of the
accused, and the crime was
committed within its territorial
jurisdiction, the court necessarily
exercises jurisdiction over all
issues that the law requires the
court to resolve. One of the issues
in a criminal case is the civil
liability of the accused arising
from the crime. Article 100 of the
Revised Penal Code provides that
Every person criminally liable for a
felony is also civilly liable. Article
104 of the same Code states that
civil liability includes restitution.
The action for recovery of
civil liability is deemed instituted
in the criminal action unless
reserved by the offended party. In
the instant case, the offended
party did not reserve the civil

action and the civil action was


deemed instituted in the criminal
action. Although the trial court
acquitted petitioner of the crime
charged, the acquittal, grounded
on reasonable doubt, did not
extinguish the civil liability. Thus,
the
Manila
trial
court
had
jurisdiction to decide the civil
aspect of the instant case ordering restitution even if the
parcel of land is located in
Bulacan.
Cojuangco v. Sandiganbayan
G.R. No. 134307
December 21, 1998
Topic: Requisites of jurisdictional
requirements
Doctrine: The giving or posting of
bail by the accused is tantamount
to submission of his person to the
jurisdiction of the court.
Facts: A complaint was filed by the
Office of the Solicitor General
before
the
Presidential
Commission on Good Government
against the former Administrator
of the Philippine Coconut Authority
and the former members of the
PCA Governing Board, petitioner
among them, for violation of the
Anti-Graft and Corrupt Practices
Act.
However, the Supreme Court
ruled that all proceedings in the
preliminary
investigation
conducted by the PCGG were null
and void and the PCGG was

CRIMINAL PROCEDURE
Case Digests Cassy C. Barras, 2D

directed to transmit the complaints


and records of the case to the
Office of the Ombudsman for
appropriate action. After the
Ombudsmans finding that there
no prejudicial question exists, an
information was filed with the
Sandiganbayan. An order for the
arrest of petitioner was issued by
the respondent Sandiganbayan.
Petititioner assailed the existence
of probable cause for his arrest.
Petitioner subsequently posted bail
without
prejudice
to
the
Opposition To Issuance of Warrant
of Arrest with Motion For Leave To
File a Motion For Reconsideration
of the Ombudsmans Resolution
which he filed. Petitioner was
conditionally arraigned pleading
not guilty to the Information solely
to accommodate his request to
travel pending the determination
of probable cause against him at
the reinvestigation stage.
Special Prosecution Officer
Tabanguil found no probable cause
to warrant the filing against
petitioner and recommended the
dismissal of the case which was
approved by the Ombudsman.
Petitioner filed an Urgent Motion
To Dismiss on the ground that
there was nothing on record
before
the
respondent
Sandiganbayan
which
would
warrant the issuance of a warrant
of arrest and the assumption of
jurisdiction over the instant case.

Issue:
Whether
or
not
the
Sandiganbayan
acquired
jurisdiction over the petitioner
Ruling: The Supreme Court ruled
in the affirmative.
The rule is well-settled that
the giving or posting of bail by the
accused
is
tantamount
to
submission of his person to the
jurisdiction of the court. Thus, it
has been held that, when a
defendant in a criminal case is
brought before a competent court
by virtue of a warrant of arrest or
otherwise, in order to avoid the
submission of his body to the
jurisdiction of the court he must
raise the question of the courts
jurisdiction over his person at the
very earliest opportunity. If he
gives bail, demurs to the complaint
or files any dilatory plea or pleads
to the merits, he thereby gives the
court jurisdiction over his person.
By posting bail, herein petitioner
cannot claim exemption from the
effect of being subject to the
jurisdiction of respondent court.
While petitioner has
exerted
efforts to continue disputing the
validity of the issuance of the
warrant of arrest despite his
posting bail, his claim has been
negated when he himself invoked
the jurisdiction of respondent
court through the filing of various
motions
that
sought
other
affirmative reliefs.

CRIMINAL PROCEDURE
Case Digests Cassy C. Barras, 2D

Moreover,
where
the
appearance is by motion for the
purpose of objecting to the
jurisdiction of the court over the
person, it must be for the sole and
separate purpose of objecting to
said jurisdiction. If the appearance
is for any other purpose, the
defendant is deemed to have
submitted
himself
to
the
jurisdiction of the court. Such an
appearance
gives
the
court
jurisdiction over the person.
Verily,
petitioners
participation in the proceedings
before the Sandiganbayan was not
confined to his opposition to the
issuance of a warrant of arrest but
also covered other matters which
called
for
respondent
courts
exercise
of
its
jurisdiction.
Petitioner may not be heard now to
deny said courts jurisdiction over
him.
Velasco v. Court of Appeals
G.R. No. 118644
July 7, 1995
Topic: Requisites of jurisdictional
requirements
Doctrine: The giving or posting of
bail by the accused is tantamount
to submission of his person to the
jurisdiction of the court.
Facts: A warrant of arrest was
issued against accused Lawrence
Larkins for violations of B.P. Blg.
22. A certain Alinea executed and
filed before the National Bureau of

Investigation a complaint-affidavit
accusing Larkins of rape allegedly
committed against her. Acting on
the complaint of Alinea, petitioners
Special Investigators Resurreccion
and Erum, Jr. proceeded to the
office of Larkins and arrested the
latter. Larkins was then detained
at the Detention Cell of the NBI.
Larkins posted bail for the
pending BP 22 case. Judge
Padolina forthwith issued an order
directing the Jail Warden to
release Larkins from confinement
"unless otherwise detained for
some
other
cause."
Special
Investigators Resurreccion and
Erum refused to release Larkins
because he was still detained for
another cause, specifically for the
crime of rape for which he would
be held for inquest. Thereafter, a
complaint against Larkins for rape
was executed by Alinea. Larkins
filed an Urgent Motion for Bail
wherein he alleged, inter alia, that
the evidence of guilt against him
for rape is not strong. Larkins
likewise filed an Urgent Omnibus
Motion for the Dismissal of the
Complaint and for Immediate
Release, principally based on the
alleged illegality of his warrantless
arrest.
The RTC denied Larkins
petitions. Larkins' common-law
wife filed before the Court of
Appeals a petition for habeas
corpus with certiorari. CA granted
the petition. After hearing the

CRIMINAL PROCEDURE
Case Digests Cassy C. Barras, 2D

arguments of the parties, the CA


resolved to order the immediate
release of Larkins.
Issue: Whether or not the court
had acquired jurisdiction over
Larkins
Ruling: The Supreme Court ruled
in the affirmative.
By filing his motion for bail,
Larkins admitted that he was
under the custody of the court and
voluntarily submitted his person to
its jurisdiction. As stated in the
case of De Asiss v. Romero, De Asis
could have, right after his arrest,
objected to the regularity of the
issuance of the warrant of arrest in
question. Instead he not only filed
a petition for bail with the lower
court, thereby
accepting the
court's jurisdiction over his person,
but
he
also
pleaded,
on
arraignment, to the information
filed against him. The filing of a
petition or motion for bail in cases
where no bail is recommended has
the same legal import and effect as
the posting of bail in cases where
bail is recommended. It is settled
that the giving or posting of bail by
the accused is tantamount to
submission of his person to the
jurisdiction of the court.
The Supreme Court also
added that while it may be true
four days after the filing of the
Urgent Motion for Bail, Larkins
filed an Urgent Omnibus Motion

for Dismissal of the Complaint and


for Immediate Release based on
the alleged illegality of his
warrantless arrest, the said motion
was a mere afterthought which
came too late in the day. By then,
the trial court had firmly acquired
jurisdiction over his person.
Lumanog, et al. v. People
G.R. No. 182555
September 7, 2010
Topic: Requisites of judgment
Doctrine: Parties to a litigation
should be informed of how a
judgment was decided, with an
explanation of the factual and legal
reasons that led to the conclusions
of the trial court.
Facts: Appellants were the accused
perpetrators of the ambush-slay of
former Chief of the Metropolitan
Command
Intelligence
and
Security Group of the Philippine
Constabulary, Colonel Rolando N.
Abadilla.
The principal witness for the
prosecution was Freddie Alejo, a
security guard employed assigned
at 211 Katipunan Avenue, Blue
Ridge, Quezon City, where the
ambush-slay
happened.
As
a
purported eyewitness, he testified
on what he saw during the fateful
day, including the faces of the
accused. All the accused raised the
defense of alibi, highlighted the
negative findings of ballistic and

CRIMINAL PROCEDURE
Case Digests Cassy C. Barras, 2D

fingerprint
examinations,
and
further alleged torture in the
hands of police officers and denial
of constitutional rights during
custodial investigation.
The trial court however
convicted the accused-appellants.
The Court of Appeals affirmed with
modification the decision of the
trial court. The CA upheld the
conviction
of
the
accusedappellants based on the credible
eyewitness testimony of Alejo, who
vividly recounted before the trial
court their respective positions
and participation in the fatal
shooting of Abadilla, having been
able to witness closely how they
committed the crime.
Issue: Whether or not the CA
decision
complied
with
the
constitutional standard that no
decision shall be rendered by any
court without expressing therein
clearly and distinctly the facts and
the law on which it is based
Ruling: Perusing the CA decision,
the Supreme Court held that it
cannot be deemed constitutionally
infirm, as it clearly stated the facts
and law on which the ruling was
based, and while it did not
specifically address each and every
assigned
error
raised
by
appellants, it cannot be said that
the appellants were left in the dark
as to how the CA reached its ruling
affirming
the
trial
courts

judgment
of
conviction.
The
principal arguments raised in their
Memorandum submitted before
the
Supreme
Court
actually
referred to the main points of the
CA rulings, such as the alleged
sufficiency
of
prosecution
evidence, their common defense of
alibi,
allegations
of
torture,
probative value of ballistic and
fingerprint
test
results,
circumstances
qualifying
the
offense and modification of penalty
imposed by the trial court. What
appellants essentially assail is the
verbatim copying by the CA of not
only the facts narrated, but also
the arguments and discussion
including the legal authorities, in
disposing of the appeal. On such
wholesale adoption of the Office of
the Solicitor Generals position, as
well as the trial courts insufficient
findings of fact, appellants anchor
their
claim
of
failure
of
intermediate review by the CA.
The
Supreme
Court
emphasized that the parties to a
litigation should be informed of
how it was decided, with an
explanation of the factual and legal
reasons that led to the conclusions
of the trial court. The losing party
is entitled to know why he lost, so
he may appeal to the higher court,
if permitted, should he believe that
the decision should be reversed. A
decision that does not clearly and
distinctly state the facts and the
law on which it is based leaves the

CRIMINAL PROCEDURE
Case Digests Cassy C. Barras, 2D

parties in the dark as to how it was


reached
and
is
precisely
prejudicial to the losing party, who
is unable to pinpoint the possible
errors of the court for review by a
higher tribunal.
BPI v. Leobrera
G.R. No. 137147
January 29, 2002
Topic: Requisites of judgment
Doctrine: Although it is not a good
practice, the Court sees nothing
illegal in the act of the trial court
completely
copying
the
memorandum submitted by a party
provided that the decision clearly
and distinctly state sufficient
findings of fact and the law on
which they are based.
Facts: Carlos B. Leobrera, plaintiffappellee herein is engaged in shell
manufacturer, retail and shell craft
export. He has been a valued client
of Bank of Philippine Islands for
several years. Plaintiff-appellee
obtained a loan of P500,000.00
from BPI covered by a Promissory
Note to be paid within three years
from date of execution, with a
quarterly
amortization
of
P41,666.66. As security, a real
estate
mortgage
on
certain
properties
was
executed
by
plaintiff-appellee in favor of BPI.
When plaintiff-appellee did
not make the payment on the due
date, BPI foreclosed the two
properties
of
plaintiff-appellee

subject
to
the
real
estate
mortgage.
After a prolonged trial, the
Court a quo rendered a decision
resolving the case in favor of the
plaintiff-appellee.
Defendant-appellant filed an
appeal arguing that the lower
court erred: (1) in not making its
own
findings
of
facts
and
conclusions of law which are in
violation
of
the
law
and
constitution; (2) in not inhibiting
itself from the case below inspite
of BPIs motion to inhibit; (3) in
holding that plaintiff did not
violate the loan agreement; (4) in
holding that BPI had no factual
and legal basis to accelerate
maturity of the loan and to
foreclose that mortgage; (5) in
concluding that BPI was negligent
in refusing to accept the $8,350.94
remittance from Darlene Shells;
(6) in holding that plaintiff is
entitled to P1,000,000.00 actual
damages, P4,000,000.00 moral
damages
and
P500,000.00
attorneys fees; (7) in holding that
the foreclosure of mortgage was
void
and
ordering
the
reconveyance of the Quezon City
and Paranaque properties; (8) in
not
awarding
damages
and
attorneys fees in favor of BPI.
The Court
of Appeals
dismissed the case for lack of
merit and modified RTCs decision
in that the moral damages is

CRIMINAL PROCEDURE
Case Digests Cassy C. Barras, 2D

reduced to P1,000,000.00, and the


attorneys fees to P100,000.00.
Issue: Whether or not the Court of
Appeals erred in entirely copying
from the memorandum filed by
respondent Leobrera
Ruling: The Supreme Court said
that though it is not a good
practise, there is nothing illegal in
the act of the trial court
completely
copying
the
memorandum submitted by a party
provided that the decision clearly
and distinctly state sufficient
findings of fact and the law on
which they are based.
Boac v. People
G.R. No. 180597
November 7, 2008
Topic: Contents of a judgment of
conviction
Doctrine:
Well-entrenched
in
jurisprudence is the rule that the
conviction of the accused must
rest, not on the weakness of the
defense, but on the strength of the
prosecution.
Facts: Raul Basilio Boac, Ramon
Betuin Golong, Cesar Fantone
Beltran, Roger Alcantara Basadre,
and Benjamin Castaneda Alfonso
are members of the Philippine
National
Police-Criminal
Investigation and Detection Group.
They were charged with violation
of Sec. 2203 in relation to Sec.

3612 of the Tariff and Customs


Code.
After both parties presented
their side, the Sandiganbayan
convicted petitioners guilty beyond
reasonable doubt. The anti-graft
court ruled that petitioners belong
to the category of officers in Sec.
2203(d); thus, they needed a
written
authority
from
the
Commissioner of Customs or
District Collector in order to
conduct searches, seizures and
arrests. In this case, the court
said, the prosecution established
the lack of said written authority;
even Beltran and Golong admitted
that they did not have any
authorization to search the vans.
Petitioners assert that they
did not conduct any search,
seizure, or arrest; hence, there
was no violation of the Tariff and
Customs Code. Petitioners allege
that they only witnessed the
search; they did not make any
seizures or arrests. After searching
the first van and half of the second
van without any contraband being
found, Customs Police Yamit and
Godoy decided to stop the search
despite the request of petitioners
to continue. Since the Customs
Police were already leaving the
area, Boac instructed his team to
leave the vicinity. Petitioners
further claim that the police's
authority to stop, search, and
effect seizure and arrest, if
necessary, is no longer exclusively

CRIMINAL PROCEDURE
Case Digests Cassy C. Barras, 2D

vested
on
the
Collector
of
Customs. Regular PNP members
are generally empowered by law to
effect arrests in accordance with
Republic Act No. (RA) 6975.
Issue:
Whether
petitioners were
reasonable doubt

or
not
the
guilty beyond

Ruling: The prosecution has the


burden of proving the guilt of the
accused beyond reasonable doubt.
In this case, it is clear that
petitioners neither searched the
container vans nor effected seizure
and arrest. It should be noted that
the container vans were brought to
the consignee's warehouse and not
to the CIDG headquarters. On July
28, 2004, the container vans were
searched but not by petitioners, as
testified to by petitioners Beltran
and Golong.
The
foregoing
testimony
Golong corroborated was not
disputed by the prosecution. It is
thus very clear that the search was
not done by petitioners but by the
Customs Police. Petitioners did not
seize
anything
nor
arrested
anybody. They merely observed the
search which they requested to be
undertaken
to
check
for
contrabands.
Notably,
the
consignee
did
not
file
any
complaint against petitioners.
The
information
charged
petitioners for illegally flagging
down, searching, and seizing the

three container vans on July 27,


2004. Petitioners, however, could
not also be held liable for these
acts. It is a fact that no search and
seizure of the vans was done on
the night of July 27, 2004. The act
of flagging down the vehicles is not
among those proscribed by Sec.
2203 of the Tariff and Customs
Code. Mere flagging down of the
container vans is not punishable
under the said law.
Well-entrenched
in
jurisprudence is the rule that the
conviction of the accused must
rest, not on the weakness of the
defense, but on the strength of the
prosecution. The burden is on the
prosecution to prove guilt beyond
reasonable doubt, not on the
accused to prove his innocence.22
In this case, the prosecution failed
to show that petitioners committed
the acts prohibited by Sec. 2203 of
the Tariff and Customs Code.
There is no such evidence,
testimonial or otherwise, that
identifies
petitioners
as
responsible for the alleged illegal
search. Hence, acquittal is in
order.
Chua v. Court of Appeals
G.R. No. 140842
April 12, 2007
Topic: Contents of a judgment of
conviction
Doctrine: The appropriate remedy
against
the
courts
order
cancelling bond is by filing by the

CRIMINAL PROCEDURE
Case Digests Cassy C. Barras, 2D

Court of Appeals a motion to


review the said order in the same
regular appeal proceedings, as an
incident of his appeal.
Facts: Respondent Wilfred Chiok
represented himself as a licensed
stockbroker and an expert in the
stock market when he met
petitioner Rufina Chua. Petitioner
designated respondent as her
stockbroker after he encouraged
her to invest her money in stocks.
Respondent encouraged her to
purchase shares in bulk, she
entrusted him the amount of
P9,563,900.00 to buy shares of
stocks in bulk. However, weeks
after
petitioner
entrusted
respondent with the money, he
admitted that he spent the money.
Respondent issued two checks as
payment but the checks were
dishonored for insufficient funds.
Petitioner then came to now that
respondent was not a licensed
stockbroker but only a telephone
clerk at Bernard Securities, Inc.
Petitioner caused the filing of
an information for estafa against
the
respondent.
Respondent
pleaded not guilty. Respondent
denied the charge against him.
After the prosecution and the
defense
had
presented
their
respective evidence, RTC set the
promulgation of judgment on
January
26,
1999.
However,
respondent and his counsel failed
to appear on said date despite
notice. The prosecution filed a
motion for cancellation of bail on
the ground that respondent might

flee or commit another crime. The


RTC issued an order for the
cancellation
of
his
bail.
Respondent filed with the CA a
petition
for
certiorari
with
application
for
a
temporary
restraining order and a writ
preliminary injunction assailing
the trial courts order canceling
his bail. The CA granted his
petition.
Issue: Whether or not the filing of
the petition for certiorari with
prayer for a TRO and a writ of
preliminary
injunction
is
the
proper remedy in this case
Ruling: The Supreme Court ruled
in the negative.
Private
respondents
appropriate remedy against the
trial
courts
Omnibus
Order
canceling his bail is by filing with
the Court of Appeals a motion to
review the said order in the same
regular appeal proceedings he
himself initiated. Such motion is an
incident in his appeal. The filing of
a separate petition via a special
civil action or special proceeding
questioning such adverse order
before the appellate court is
proscribed.
Such
independent
special civil action obviously
contravenes the rule against
multiplicity of suits and constitutes
forum shopping. Hence, the Court
of Appeals erred in not dismissing
outright respondents petition for
certiorari. The basic rule is that

CRIMINAL PROCEDURE
Case Digests Cassy C. Barras, 2D

such petition may only be availed


of when "there is no appeal or any
plain,
speedy
and
adequate
remedy in the ordinary course of
law."
People v. Serrano
G.R. No. 135451
September 30, 1999
Topic: Contents of a judgment of
acquittal
Doctrine: A verdict of acquittal is
immediately final.
Facts: Visbal filed with the RTC a
sworn complaint charging accused
Serrano
with
rape.
At
the
arraignment, accused pleaded not
guilty. After due trial, RTC
rendered decision acquitting the
accused on the ground that the
prosecution failed to prove his
guilt beyond reasonable doubt.
Assistant Public Prosecutor
Huamayor filed a notice of appeal
to the Supreme Court from the
decision acquitting the accused for
being contrary to the facts and the
law. Judge Domael, presiding judge
of RTC, issued an order giving due
course to the appeal filed by the
Provincial
Prosecutor.
The
Supreme Court dismissed the
appeal for violation of the rule on
double jeopardy.
Issue: Whether or not a judgment
of acquittal on the merits may be
subject of appeal

Ruling: The Supreme Court ruled


in the negative.
It is elementary that the rule
against double jeopardy proscribes
an appeal from a judgment of
acquittal on the merits. A verdict
of acquittal is immediately final
and a re-examination of the merits
of such acquittal, even in an
appellate court, will put him a
second time in jeopardy for the
same offense.
The
Constitution
itself
provides that no person shall be
twice
put
in
jeopardy
of
punishment for the same offense.
Such a constitutional guarantee
prohibits an appeal from a
judgment of acquittal, and the law
does not provide for exceptions
other than deprivation of due
process
or
grave
abuse
of
discretion
under
exceptional
circumstances.
It
is
true
that
the
Department
of
Justice
in
Memorandum Circular No. 3,
enunciated the proposition that
acquittals may be appealed as long
as a second trial will not be
required and will not place the
accused in second jeopardy, in the
event
the
appeal
succeeds.
However, such rule is not shown to
be applicable to the case at bar. To
use the Memorandum Circular of
the Department of Justice to
overthrow
a
constitutional
guarantee deeply ingrained to
protect the rights of an accused

CRIMINAL PROCEDURE
Case Digests Cassy C. Barras, 2D

would require more than just a


mere statement that the decision
was contrary to the facts and to
the law, as stated by the
prosecution in its notice of appeal.
People v. Court of Appeals
G.R. No. 159261
February 21, 2007
Topic: Contents of a judgment of
acquittal
Doctrine: A verdict of acquittal is
immediately final.
Facts: Private respondent is one of
the accused in two cases for
homicide before the Regional Trial
Court. On arraignment, all accused
entered pleas of not guilty and
thereafter a joint trial ensued. The
RTC
convicted
the
private
respondent guilty of the crime of
Homicide with the aggravating
circumstance of dwelling which
was off-setted by the mitigating
circumstance
of
sufficient
provocation. Private respondent
appealed his conviction for arguing
that the trial court erred in finding
him guilty of homicide considering
that the prosecution failed to prove
that he and Ureta conspired to kill
Ramon; and that the prosecutions
evidence showed it was only Ureta
who stabbed Ramon. The CA
acquitted private respondent of
the crime charged.

Issue:
Whether
or
not
reexamination of the merits of such
acquittal is allowed
Ruling: The Supreme Court ruled
in the negative.
A verdict of acquittal is
immediately
final
and
a
reexamination of the merits of
such acquittal, even in the
appellate courts, will put the
accused in jeopardy for the same
offense. The finality-of-acquittal
doctrine
has
several avowed
purposes. Primarily, it prevents the
State from using its criminal
processes as an instrument of
harassment to wear out the
accused by a multitude of cases
with accumulated trials. It also
serves the additional purpose of
precluding the State, following an
acquittal,
from
successively
retrying the defendant in the hope
of securing a conviction. And
finally, it prevents the State,
following conviction, from retrying
the defendant again in the hope of
securing a greater penalty.
In People v. Velasco, the
Supreme Court stressed that an
acquitted defendant is entitled to
the right of repose as a direct
consequence of the finality of his
acquittal. Hence, it cannot be
disputed that the verdict of the
Court of Appeals acquitting Ramon
Galicia
is
now
final
and
irreviewable.

CRIMINAL PROCEDURE
Case Digests Cassy C. Barras, 2D

Abalos v. People
G.R. No. 136994
September 17, 2002
Topic: Duplicitous complaint or
information
Doctrine: While Rule 110, Section
13, frowns upon multiple offenses
being
charged
in
a
single
information, the failure to raise
this issue during arraignment
amounts to a waiver, and the
objection can no longer be raised
on appeal.
Facts: Two separate informations
for
Falsification
of
Private
Documents was filed against the
accused-appellant Abalos before
the MTC of Dagupan and MTC of
Lingayen.
During
his
arraignment
before the MTC Dagupan, the
accused-appellant entered a plea
of not guilty. After two months, he
filed a Motion to Quash, arguing
that the Municipal Trial Court had
no jurisdiction over the offense
charged. MTC Dagupan ordered
the quashal the case for lack of
jurisdiction. Private complainants
Motion for Reconsideration was
denied. Private complainant filed a
Petition for Certiorari with the
RTC Dagupan. RTC Dagupan
reversed and set aside the Orders
of the MTC Dagupan.
On the other hand, after the
filing of the Information before the
Lingayen court, the accusedappellant filed a Motion to Quash

The court a quo denied the Motion


to Quash. The accused-appellant
went on Certiorari to the RTC
Lingayen
which
denied
the
petition.
Petitioner appealed the both
cases to the Court of Appeals and
moved to consolidate the two
appeals, which the CA granted.
The CA dismissed the consolidated
case for lack of merit.
Issue: Whether or not accusedappellant may raise the issue of
duplicity of offense
Ruling: The Supreme Court ruled
in the negative.
The
Rules
of
Court,
particularly Rule 110, Section 13,
indeed frowns upon multiple
offenses being charged in a single
information. However, petitioner
failed to raise this issue during
arraignment, in Lingayen as well
as in Dagupan. His failure to do so
amounts to a waiver, and his
objection on this point can no
longer be raised on appeal. In his
Motion to Quash filed in Dagupan
City, petitioner alleged lack of
jurisdiction. On the other hand, in
his Motion to Quash filed in
Lingayen,
petitioner
alleged
forum-shopping, double jeopardy,
lack of jurisdiction, and that the
facts do not constitute an offense.
He only raised the issue of
multifariousness
of
offenses
alleged in his petition before the

CRIMINAL PROCEDURE
Case Digests Cassy C. Barras, 2D

Supreme Court. Thus, his objection


is belated, and obviously to no
avail.
People v. Alfredo
G.R. No. 188560
December 15, 2010
Topic: Judgment rendered by judge
who did not hear case
Doctrine: The fact that the trial
judge who rendered judgment was
not the one who had the occasion
to observe the demeanor of the
witnesses during trial, but merely
relied on the records of the case,
does not render the judgment
erroneous, especially where the
evidence on record is sufficient to
support its conclusion.
Facts: The RTC found accusedappellant Alfredo guilty beyond
reasonable doubt of two counts of
rape.
The
CA
affirmed
the
judgment of conviction by the trial
court. Accused-appellant contends
that the judge who penned the
appealed decision is different from
the
judge
who
heard
the
testimonies of the witnesses and
was, thus, in no position to render
a judgment, as he did not observe
firsthand their demeanor during
trial.
Issue:
Whether
or
contention
of
the
appellant is tenable

not
the
accused-

Ruling: The Supreme Court ruled


in the negative.
The fact that the trial judge
who rendered judgment was not
the one who had the occasion to
observe the demeanor of the
witnesses during trial, but merely
relied on the records of the case,
does not render the judgment
erroneous, especially where the
evidence on record is sufficient to
support its conclusion. As the
Supreme Court held in People v.
Competente, The circumstance
that the Judge who rendered the
judgment was not the one who
heard the witnesses, does not
detract from the validity of the
verdict of conviction. Even a
cursory perusal of the Decision
would show that it was based on
the evidence presented during trial
and that it was carefully studied,
with testimonies on direct and
cross examination as well as
questions from the Court carefully
passed upon.
Further, the transcripts of
stenographic notes taken during
the trial were extant and complete.
Hence, there was no impediment
for the judge to decide the case.
Consulta v. People
G.R. No. 179462
February 12, 2009
Topic: Variance Doctrine
Doctrine: When there is a variance
between the offense charged in the
complaint or information, and the

CRIMINAL PROCEDURE
Case Digests Cassy C. Barras, 2D

offense as charged is included in


or necessarily includes the offense
proved, the accused shall be
convicted of the offense proved
which is included in the offense
charged, or of the offense charged
which is included in the offense
proved.
Facts:
The
information
filed
against
Consulta
states
that
Consulta, with intent of gain, and
by means of force, violence and
intimidation, did then and there
willfully, unlawfully and feloniously
take, steal and carry away
complainants gold necklace worth
P3,500.00, belonging to said
complainant.
The trial court, holding that
intent to gain on appellants part
"is presumed from the unlawful
taking" of the necklace, and
brushing aside appellants denial
and claim of harassment, convicted
appellant
of
Robbery.
The
appellate
court
affirmed
appellants conviction.
Issue: Whether or not appellant
has committed the crime of which
he was charged
Ruling: The Supreme Court said
that accused-appellant is not liable
for robbery, but of grave coercion.
The Court finds that the
incidental encounter of the parties,
the taking of Nelias necklace does
not indicate presence of intent to

gain on appellants part. That


intent to gain on appellants part is
difficult to appreciate gains light
given his undenied claim that his
relationship with Nelia is rife with
ill-feelings, manifested by, among
other
things,
the
filing
of
complaints against him by Nelia
and her family which were
subsequently dismissed or ended
in his acquittal. Absent intent to
gain on the part of appellant,
robbery does not lie against him.
He is not necessarily scot-free,
however.
From the pre-existing sour
relations between Nelia and her
family on one hand, and appellant
and family on the other, and under
the circumstances attendant to the
incidental encounter of the parties,
appellants
taking
of
Nelias
necklace could not have been
animated with animus lucrandi.
Appellant is, however, just the
same, criminally liable.
For "when there is variance
between the offense charged in the
complaint or information and that
proved, and the offense as charged
is included in or necessarily
includes the offense proved, the
accused shall be convicted of the
offense proved which is included in
the offense charged, or of the
offense charged which is included
in the offense proved."
People v. Rellota
G.R. No. 168103

CRIMINAL PROCEDURE
Case Digests Cassy C. Barras, 2D

August 3, 2010
Topic: Variance Doctrine
Doctrine: When there is a variance
between the offense charged in the
complaint or information, and the
offense as charged is included in
or necessarily includes the offense
proved, the accused shall be
convicted of the offense proved
which is included in the offense
charged, or of the offense charged
which is included in the offense
proved.
Facts: AAA, the complainant, is 12
years old when the incidents
allegedly happened. Together with
her siblings, AAA lived with her
aunt, DDD and the latters second
husband,
appellant
from
September 1992 to January 1994.
According to AAA, appellant had
been kissing her and touching her
private parts since September
1993. AAA stated that she resisted
the advances of appellant, but was
not
successful.
Appellant,
according to her would usually
place a bolo beside him whenever
she would rape her. She added
that appellant would threaten AAA
by telling her that he would kill
her brother and sister and that he
would stop sending her to school.
On December 20, 1993, appellant
raped AAA twice in the latters
bedroom. He tied her hands with a
rope before forcibly inserting his
penis inside her vagina. The same
incident happened on January 31,

1994, when AAA was inside their


room. Appellant laid her down on
the sofa, kissed her and touched
her private part, while AAA kicked
him and scratched his arms.
Appellant pleaded not guilty
during arraignment. RTC found
appellant guilty beyond reasonable
doubt of three counts of rape and
is penalized to suffer reclusion
perpetua for each count. In not
imposing the penalty of death, the
trial court reasoned out that, the
relationship of AAA and the
appellant had not been sufficiently
established
as
the
marriage
between AAA's aunt and the
appellant was not supported by
any documentary evidence. Upon
appeal, the Court of Appeals
affirmed RTCs decision in so far
as appellant is found guilty of two
counts of consummated rape and
sentenced to reclusion perpetua
for each count.
Appellant insists that he
could not have possibly raped AAA
as based on the testimony of AAA,
appellant
merely
kissed
and
touched her breasts and private
parts, but never did she mention
that he inserted his penis into her
vagina.
Issue:
Whether
or
not
the
appellant may be convicted of acts
of lasciviousness aside from rape
Ruling: The Supreme Court ruled
in the affirmative.

CRIMINAL PROCEDURE
Case Digests Cassy C. Barras, 2D

All the elements of the offense are


present. The actions of appellant
laying AAA on the sofa and kissing
and touching her private parts are,
by definition, lascivious or lewd,
and based on AAA's testimony, the
intimidation from appellant was in
existence and apparent. As case
law has it, intimidation need not
necessarily be irresistible. It is
sufficient that some compulsion
equivalent to intimidation annuls
or subdues the free exercise of the
will of the offended party. This is
especially true in the case of
young, innocent and immature
girls who could not be expected to
act with equanimity of disposition
and with nerves of steel. Young
girls cannot be expected to act like
adults
under
the
same
circumstances or to have the
courage
and
intelligence
to
disregard the threat.
Moreover, under Section 4,
Rule 120 of the Revised Rules of
Criminal Procedure, when there is
a variance between the offense
charged in the complaint or
information, and the offense as
charged
is
included
in
or
necessarily includes the offense
proved, the accused shall be
convicted of the offense proved
which is included in the offense
charged, or of the offense charged
which is included in the offense
proved.
Thus, the Supreme Court
affirmed CAs conviction of two

counts
of
rape
with
the
modification
that
the
same
appellant is also guilty beyond
reasonable doubt of the crime of
acts of lasciviousness as defined in
the Revised Penal Code, in relation
to Section 5, Article III of Republic
Act No. 7610.
Pangilinan vs. Court of Appeals
G.R. No. 117363
December 17, 1999
Topic: Variance Doctrine
Doctrine: The application of the
variance doctrine presupposes that
the court rendering the judgment
has jurisdiction over the case
based on the allegations of the
information.
Facts: Appellant was charged with
estafa. Appellant was arraigned
before the RTC where she entered
a plea of not guilty. After due trial,
RTC convicted the appellant of the
crime
of
estafa.
Appellant
appealed to the CA. The CA
affirmed
the
conviction
and
likewise denied the subsequent
motion
for
reconsideration.
Appellant filed a petition for New
Trial in the CA which was denied.
Appellant elevated the case in the
Supreme Court arguing that trial
court lack jurisdiction over the
crime charged.
Issue: Whether or not the variance
doctrine applies in the case at bar

CRIMINAL PROCEDURE
Case Digests Cassy C. Barras, 2D

Ruling: The Supreme Court ruled


in the negative.
As correctly observed by the
CA in the questioned decision,
appellant was charged under an
information alleging an offense
falling under the blanket provision
of paragraph 1(a) of Article 318 of
the Revised Penal Code, which
treat other Deceits. At the time of
the filing of the information in this
case, the law in force was Batas
Pambansa Blg. 129. Under the
express provision of Section 32 of
B.P. 129, the offense of which the
petitioner was charged with falls
within
the
exclusive
original
jurisdiction of the Municipal Trial
Court.
According to the OSG, since
the offense proved is necessarily
included in the offense charged,
then
the
decision
of
the
respondent court modifying the
court of origins judgment is
perfectly valid and the petitioners
claim that the trial court had no
jurisdiction must necessarily fail.
This argument is erroneous. The
variance
doctrine
applies
exclusively to cases where the
offenses as charged are included
in or necessarily the offense
proved. It presupposes that the
court rendering judgment has
jurisdiction over the case based on
the allegations in the information.
However, in the case at bar, from
the
onset
of
the
criminal
proceedings, the lower court had

no jurisdiction to hear and decide


the case.
Having
arrived
at
the
conclusion that the RTC did not
have jurisdiction to try the case
against the appellant, it is no
longer necessary to consider the
other issues raised as the decision
of the RTC is null and void.
People vs. Sumingwa
G.R. No. 183619
October 13, 2009
Topic: Judgment
Doctrine: Following the "variance
doctrine" embodied in Section 4, in
relation to Section 5, Rule 120 of
the Rules of Criminal Procedure,
appellant can be found guilty of
the lesser crime of Acts of
Lasciviousness committed against
a child.
Facts: In twelve Informations, the
prosecution
charged
appellant
with two counts of Acts of
Lasciviousness, four counts of
Rape, three counts of Unjust
Vexation, one count of Other Light
Threats,
one
count
of
Maltreatment, and one count of
Attempted
Rape
for
acts
committed against his minor
daughter AAA from 1999-2001.
Appellant denied all the allegations
against him and pleaded not guilty.
AAA executed an Affidavit of
Recantation claiming that while
appellant
indeed
committed
lascivious acts against her, she
explained that appellant did not

CRIMINAL PROCEDURE
Case Digests Cassy C. Barras, 2D

actually rape her, as there was no


penetration.
The RTC rendered a decision
convicting appellant of six counts
of acts of lasciviousness, one count
of attempted rape and one count of
unjust vexation. In view of the
withdrawal of AAAs earlier claim
of the fact of penetration, the court
sustained
the
innocence
of
appellant on the rape charges and
concluded
that
the
crime
committed was only Acts of
Lasciviousness.
On appeal, the CA affirmed
the conviction of appellant but
convicted him of Qualified Rape
instead of Acts of Lasciviousness.
The appellate court concluded
that,
notwithstanding
AAAs
retraction
of
her
previous
testimonies,
the
prosecution
sufficiently
established
the
commission of the crime of Rape.
It added that the qualifying
circumstances of minority and
relationship
were
adequately
proven.
Issue: Whether or not may be
convicted of Lasciviousness
Ruling:
The
Supreme
Court
affirmed
that
CA
correctly
convicted appellant of Acts of
Lasciviousness.
In Criminal Case Nos. 1649
and 1654, although appellant was
charged
with
qualified
rape
allegedly committed on the second
week of November 2000 and May
27, 2001, he should be convicted
with
Acts
of
Lasciviousness

committed against a child under


Section 5(b), Article III of R.A.
7610.
AAA testified that in
November 2000, while she and
appellant
were
inside
the
bedroom, he went on top of her
and rubbed his penis against her
vaginal orifice until he ejaculated.
She likewise stated in open court
that on May 27, 2001, while inside
their comfort room, appellant
rubbed his penis against her
vagina while they were in a
standing
position.
In
both
instances,
there
was
no
penetration, or even an attempt to
insert his penis into her vagina.
The aforesaid acts of the appellant
are covered by the definitions of
"sexual abuse" and "lascivious
conduct" under Section 2(g) and
(h) of the Rules and Regulations on
the Reporting and Investigation of
Child Abuse Cases promulgated to
implement the provisions of R.A.
7610. Following the variance
doctrine, appellant can be found
guilty of the lesser crime of Acts of
Lasciviousness committed against
a child.
Appellant is likewise guilty of
two
counts
of
Acts
of
Lasciviousness under Section 5(b),
Article III, R.A. 7610 committed
against AAA on the second week of
August 1999 and on the first week
of September 1999. AAA testified
that in August, appellant, with
lewd design, inserted his hands
inside her shirt then fondled her
breasts; and in September, he
forced her to hold his penis until
he ejaculated.

CRIMINAL PROCEDURE
Case Digests Cassy C. Barras, 2D

Navarrete v. People
G.R. No. 147913
January 31, 2007
Topic: Variance Doctrine
Doctrine: An accused may be
convicted only of the crime with
which he is charged. An exception
to this rule is the rule on variance
in Section 4, Rule 120 of the Rules
of Court.
Facts: Petitioner Navarrete was
charged
with
the
crime
of
statutory
rape
of
BBB.
On
arraignment, petitioner pleaded
not guilty. The facts show that
BBB, who was at that time five
years old, and petitioner were
neighbors. One evening, BBB went
to petitioners house to watch
television, which was something
she often did. Only petitioner and
BBB were there that night. BBB
testified that it was on this
occasion that petitioner sexually
abused her, "placed his penis in
her vagina" twice, poked her
vagina with a "stick with cotton"
and boxed her on the right side of
her eye. Then, petitioner brought
her to the comfort room and
pointed a knife to her throat.
Afterwards, she and petitioner
watched a pornographic movie
together. Findings of the medicolegal officer precluded complete
penetration by an average-sized
Filipino male organ in full erection.
Testifying in his own behalf,

petitioner denied the accusation


against him and claimed that AAA
merely concocted the charge
against him.
RTC absolved petitioner of
statutory rape as there was no
clear and positive proof of the
entry of petitioners penis into the
labia of the victims vagina.
However, it convicted petitioner
for acts of lasciviousness under
Article 336 of the Revised Penal
Code (RPC) in relation to Section
5(b), Article III of RA 7610. On
appeal, the CA affirmed the
decision of the RTC.
Issue: Whether or not petitioner
may be convicted of acts of
lasciviousness
Ruling: The Supreme Court ruled
in the affirmative.
The Constitution mandates
that the accused, in all criminal
prosecutions, shall enjoy the right
to be informed of the nature and
cause of the accusation against
him.
From
this
fundamental
precept proceeds the rule that the
accused may be convicted only of
the crime with which he is
charged.
An exception to this rule is
the rule on variance in Section 4,
Rule 120 of the Rules of Court:
When there is variance between
the offense charged in the
complaint or information, and that
proved, and the offense as charged

CRIMINAL PROCEDURE
Case Digests Cassy C. Barras, 2D

is included in or necessarily
includes the offense proved, the
accused shall be convicted of the
offense proved which is included in
the offense charged, or of the
offense charged which is included
in that which is proved.
People v. Corpuz
G.R. No. 168101
February 13, 2006
Topic: Variance in the mode of the
commission of the offense
Doctrine: A variance in the mode
of commission of the offense is
binding upon the accused if he
fails to object to evidence showing
that the crime was committed in a
different manner than what was
alleged.
Facts: An Information was filed
before the RTC charging the
herein appellant with the crime of
qualified rape for raping his 13year-old daughter. The accusedappellant pleaded not guilty to the
crime charged.
The trial judge issued an
order stating that in the process of
preparing a decision, he noticed
that there is variance between the
offense charged and that proved.
Thus, in order to avoid the
miscarriage of justice, he directed
the trial prosecutor to amend the
information to conform with the
evidence, specifically, that the rape
was committed while the woman is
unconscious instead of by the use

of force and intimidation. Accusedappellant


was
convicted
of
qualified rape and sentenced to
suffer the death penalty.
The accused-appellant next
that the trial courts order to
amend the information as to the
mode of commission of the rape
from "by force and intimidation" to
"while the offended party was
asleep or unconscious" is not
sanctioned by the Rules on
Criminal Procedure.
Issue: Whether or not accusedappellant may be convicted of rape
committed under one mode when
the information alleged another
more
Ruling: The Supreme Court ruled
in the affirmative.
The appellant never raised
any
objection
when
the
prosecution showed another mode
of commission of the crime
charged as alleged in the original
information. Nor did he interpose
any objections after the judge
issued his order directing the trial
prosecutor
to
amend
the
information to conform with the
evidence and before he rendered
judgment.
Be that as it may, the
amendment of the information did
not affect the crime committed by
the appellant, that is, qualified
rape. In cases of incestuous rape,
force or intimidation need not even

CRIMINAL PROCEDURE
Case Digests Cassy C. Barras, 2D

be proven. The overpowering


moral influence of the father over
the daughter takes the place of
violence and offer of resistance
required in rape cases committed
by an accused unrelated to the
victim.
Consequently,
his
conviction is in order.
People v. Abello
G.R. No. 151952
March 25, 2009
Topic: Variance in the mode of a
commission of an offense
Doctrine: A variance in the mode
of commission of the offense is
binding upon the accused if he
fails to object to evidence showing
that the crime was committed in a
different manner than what was
alleged.
Facts: On June 30, 1998 at around
4:00 oclock in the early morning,
while sleeping in their house with
her sister-in-law and nephew, AAA
was suddenly awakened when
his stepfather Abello mashed her
breast. The
same situation
happened on July 2, 1998. In
these two occasions, AAA was able
to recognize Abello because of the
light coming from outside which
illuminated the house. Then on
July 8, 1998, at around 2:00 a.m.,
Abello this time placed his soft
penis inside the mouth of AAA. The
latter got awaken when Abello
accidentally kneeled on her right
hand.
AAA
exclaimed
Aray

forcing the accused to hurriedly


enter
his
room.
He
was
nevertheless seen by AAA. The
victim on the same date reported
the incident to her sister-in-law
and mother.
The RTC found Abello guilty
under the three Informations filed
against the appellant. The CA
affirmed Abellos conviction on
appeal but modified the penalties
imposed.
Appellant contends that it
was impossible for him to have
committed
these
crimes
considering that: (a) he is AAAs
stepfather who has a healthy
sexual
relationship
with
her
mother; (b) AAA was not alone
during these alleged incidents; and
(c) AAA admitted that she was
asleep
when
these
incidents
happened making it likely that she
could have just dreamed of them.
Issue: Whether or not the variance
between the allegations in the
Information for rape and that
proven at the trial on the mode of
committing the offense affects the
conviction of the appellant
Ruling: The Court held that
statements of AAA satisfy the
presence of the elements of rape.
AAAs
testimony
covers
the
commission of the sexual assault
through the insertion of Abellos
male organ into her mouth; AAA
also consistently identified Abello

CRIMINAL PROCEDURE
Case Digests Cassy C. Barras, 2D

as the perpetrator of the sexual


assault. She also testify that she
was roused from sleep with
Abellos male organ inserted in her
mouth. In this respect, the
Supreme Court observes that both
the RTC and the CA failed to notice
the
variance
between
the
allegations in the Information for
rape and that proven at the trial on
the mode of committing the
offense. The Information alleges
"force and intimidation" as the
mode of commission, while AAA
testified during the trial that she
was asleep at the time it happened
and only awoke to find Abellos
male organ inside her mouth.
This variance is not fatal to
Abellos conviction for rape by
sexual assault. In People v. Corpuz,
the Supreme Court ruled that a
variance
in
the
mode
of
commission of the offense is
binding upon the accused if he
fails to object to evidence showing
that the crime was committed in a
different manner than what was
alleged. In the present case, Abello
did not object to the presentation
of evidence showing that the crime
charged was committed in a
different manner than what was
stated in the Information. Thus,
the variance is not a bar to
Abellos conviction of the crime
charged in the Information.
People v. Taruc
G.R. No. 185202

February 18, 2009


Topic: If the accuse fails to appear
in the promulgation of judgment
Doctrine: If the accused fails to
appear before the trial court,
promulgation of judgment shall be
made in accordance with Rule 120,
Section 6, paragraphs 4 and 5 of
the Revised Rules of Criminal
Procedure.
Facts: Accused-appellant
Taruc
was charged in before the RTC
with the crime of murder. Upon
arraignment,
accused-appellant
pleaded not guilty. RTC found
accused guilty beyond reasonable
doubt as principal by direct
participation of the crime of
murder and sentenced to suffer
death penalty.
Accused-appellant elevated
the case to the CA for review.
Thereafter, accused-appellant filed
a Motion for Extension of Time to
File Appellants Brief. Considering
that the Notice to File Brief
addressed to accused-appellant
was returned to the appellate
court with postal notation moved
out, the CA directed accusedappellants counsel to furnish it
with the present and complete
address of his client within five
days from notice. PAO lawyer
informed
CA
that
accusedappellant escaped from prison. The
CA, notwithstanding the escape of
the accused-appellant, granted the
motion for extention of time. The

CRIMINAL PROCEDURE
Case Digests Cassy C. Barras, 2D

CA affirmed accused-appellants
conviction but modified the penalty
to reclusion perpetua. PAO lawyer
appealed before the Supreme
Court on questions of law and fact.
Issue: Whether or not the accusedappellant lost his right to appeal
his conviction after he escaped
from jail
Ruling: The Supreme Court ruled
in the affirmative.
An accused is required to be
present before the trial court at
the promulgation of the judgment
in a criminal case. If the accused
fails to appear before the trial
court, promulgation of judgment
shall be made in accordance with
Rule 120, Section 6, paragraphs 4
and 5 of the Revised Rules of
Criminal Procedure, to wit:
In case the accused fails to
appear at the scheduled date of
promulgation of judgment despite
notice, the promulgation shall be
made by recording the judgment in
the criminal docket and serving
him a copy thereof at his last
known address or thru his counsel.
If the judgment is for
conviction and the failure of the
accused to appear was without
justifiable cause, he shall lose the
remedies available in these Rules
against the judgment and the court
shall order his arrest. Within
fifteen
(15)
days
from
promulgation
of
judgment,

however,
the
accused
may
surrender and file a motion for
leave of court to avail of these
remedies. He shall state the
reasons for his absence at the
scheduled promulgation and if he
proves that his absence was for a
justifiable cause, he shall be
allowed to avail of said remedies
within fifteen (15) days from
notice.
Estino, et al. v. People
G.R. Nos. 163957-58
April 7, 2009
Topic: New trial
Doctrine: Rule 121 of the Rules of
Court allows the conduct of a new
trial
before
a
judgment
of
conviction becomes final when new
and material evidence has been
discovered which the accused
could
not
with
reasonable
diligence have discovered and
produced at the trial and which if
introduced and admitted would
probably change the judgment.
Facts: Estino was elected ViceGovernor of Sulu along with Gov.
Abdusakur Tan. On June 23, 1998,
the Supreme Court issued a status
quo order suspending the effects
of the proclamation of Gov. Tan.
Thus, Estino acted as Governor of
Sulu from July 27, 1998 up to May
23, 1999 when the Supreme Court
lifted the suspension order Ernesto
G. Pescadera, on the other hand,
was Provincial Treasurer of Sulu

CRIMINAL PROCEDURE
Case Digests Cassy C. Barras, 2D

during Estinos stint as Acting


Governor.
Three
informations
were
filed against Estino and Pescadera.
Sandiganbayan ruling on the three
informations:
(1)
Estino
and
Pescadera guilty of violation of
Section 3(e), Republic Act No. (RA)
3019 or the Anti-Graft and Corrupt
Practices Act for failure to pay the
Representation and Transportation
Allowance (RATA) of the provincial
government employees of Sulu; (2)
Pescadera guilty of the crime of
malversation of public funds under
Article 217 of the Revised Penal
Code for failure to remit the
Government Service Insurance
System (GSIS) contributions of the
provincial government employees
amounting to PhP 4,820,365.30,
Estino acquitted; and (3) both
acquitted of the charge for
violation of RA 3019, Sec. 3(e).
Petitioners filed a Motion for
Reconsideration
and
a
Supplemental
Motion
for
Reconsideration and New Trial
which
were
denied
by
Sandiganbayan.
In
their
Supplemental
Motion
for
Reconsideration and Motion for
New Trial, petitioners presented to
the Sandiganbayan a Certification
dated May 11, 2002 issued by the
Provincial Auditor Abdurasad J.
Undain,
stating
that
the
Representation and Transportation
Allowance for the period January
to May 1999 was paid to the

officials entitled to it and that the


GSIS premiums pertaining to prior
years were also settled by the
Provincial Government of Sulu.
Issue: Whether or not the motion
for new trial should be granted
Ruling: The Supreme Court ruled
in the affirmative and remanded
the to the Sandiganbayan for new
trial in relation to the conviction of
Estino and Pesccadera for violation
of Sec. 3(e) of R.A. 3019.
Rule 121 of the Rules of
Court allows the conduct of a new
trial
before
a
judgment
of
conviction becomes final when new
and material evidence has been
discovered which the accused
could
not
with
reasonable
diligence have discovered and
produced at the trial and which if
introduced and admitted would
probably change the judgment.
Although the documents offered by
petitioners are strictly not newly
discovered, it appears to the
Supreme Court that petitioners
were mistaken in their belief that
its production during trial was
unnecessary. In their Supplemental
Motion and/or Motion for New
Trial, they stressed that they no
longer presented the evidence of
payment
of
RATA
because
Balabaran testified that the subject
of the charge was the nonpayment
of benefits under the 1999 budget,
without mention of the RATA nor

CRIMINAL PROCEDURE
Case Digests Cassy C. Barras, 2D

the 1998 reenacted budget. It


seems that they were misled
during trial. They were precluded
from presenting pieces of evidence
that may prove actual payment of
the
RATA
under
the
1998
reenacted budget because the
prosecutions
evidence
was
confined to alleged nonpayment of
RATA under the 1999 budget.
In this instance, the Supreme
Court is inclined to give a more
lenient interpretation of Rule 121,
Sec. 2 on new trial in view of the
special circumstances sufficient to
cast doubt as to the truth of the
charges against petitioners. The
situation of the petitioners is
peculiar,
since
they
were
precluded
from
presenting
exculpatory evidence during trial
upon the honest belief that they
were being tried for nonpayment
of RATA under the 1999 budget.
This belief was based on no less
than
the
testimony
of
the
prosecutions lone witness, COA
Auditor Mona Balabaran. Even
Associate Justice Palattao of the
Sandiganbayan had to clarify from
Balabaran which budget she was
referring to. Balaraban, however,
made it very clear that the unpaid
benefits were those provided
under the 1999 budget.

Briones v. People
G.R. No. 156009
June 5, 2009

Topic:
Requisites
for
newly
discovered evidence
Doctrine:
Evidence,
to
be
considered newly discovered, must
be one that could not, by the
exercise of due diligence, have
been discovered before the trial in
the court below.
Facts: A criminal information was
filed against Briones for crime of
robbery. Briones allegedly took the
service firearm of S/G Gual while
the latter approached the group
where the former is involved in a
mauling.
S/G
Gual
positively
identified Briones. RTC found
Briones guilty of the crime of
simple theft after giving weight to
prosecutions positive testimony as
against the defenses of denial and
alibi. On his appeal, he raised the
issue of self-defense. The Court of
Appeals found Briones guilty of
robbery, and not of theft.
Briones thereafter filed an
Omnibus
Motion
for
Reconsideration, Motion for New
Trial and Motion to Dismiss, and
Supplemental Omnibus Motion for
Reconsideration, Motion for New
Trial and Motion to Dismiss with
the CA where he confessed his
physical
presence
and
participation
on
the
alleged
robbery of the firearm, but claimed
that he was merely protecting his
brother, Vicente, when he took the
firearm.

CRIMINAL PROCEDURE
Case Digests Cassy C. Barras, 2D

Issue: Whether or not a new trial


may be granted on the ground of
newly discovered evidence.
Ruling: The Supreme Court ruled
in the negative.
For new trial to be granted
on the ground of newly discovered
evidence, the concurrence of the
following conditions must obtain:
(a) the evidence must have been
discovered after trial; (b) the
evidence could not have been
discovered at the trial even with
the
exercise
of
reasonable
diligence; (c) the evidence is
material, not merely cumulative,
corroborative, or impeaching; and
(d) the evidence must affect the
merits of the case and produce a
different result if admitted. In this
case,
although
the
firearm
surfaced after the trial, the other
conditions were not established.
Evidence, to be considered
newly discovered, must be one
that could not, by the exercise of
due
diligence,
have
been
discovered before the trial in the
court below. The determinative
test is the presence of due or
reasonable diligence to locate the
thing to be used as evidence in the
trial.
Under the circumstances,
Briones failed to show that he had
exerted reasonable diligence to
locate the firearm; his allegation in
his Omnibus Motion that he told
his brothers and sisters to search

for the firearm, which yielded


negative results, is purely selfserving. He also now admits
having taken the firearm and
having immediately disposed of it
at a nearby house, adjacent to the
place of the incident. Hence, even
before the case went to court, he
already knew the location of the
subject firearm, but did not do
anything; he did not even declare
this knowledge at the trial below.
In any case, the Supreme
Court fails to see how the recovery
of the firearm can be considered
material evidence that will affect
the outcome of the case; the
recovery of the subject firearm
does not negate the commission of
the crime charged. It furthers that
in petitions for new trial in a
criminal proceeding where a
certain
evidence
was
not
presented, the defendant, in order
to secure a new trial, must satisfy
the court that he has a good
defense, and that the acquittal
would in all probability follow the
introduction
of
the
omitted
evidence.
Neypes v. Court of Appeals
G.R. No. 141524
September 14, 2005
Topic: Fresh Period Rule
Doctrine: A party litigant may
either file his notice of appeal
within fifteen days from receipt of
the Regional Trial Courts decision
or file it within fifteen days from

CRIMINAL PROCEDURE
Case Digests Cassy C. Barras, 2D

receipt of the order (the "final


order") denying his motion for new
trial or motion for reconsideration.
Facts: Petitioners filed an action
for annulment of judgment and
titles of land and/or reconveyance
and/or reversion with preliminary
injunction before the RTC against
the private respondents. Later, in
an order, the trial court dismissed
petitioners complaint on the
ground that the action had already
prescribed. Petitioners allegedly
received a copy of the order of
dismissal on March 3, 1998 and,
on the 15th day thereafter or on
March 18, 1998, filed a motion for
reconsideration.
On July 1, 1998, the trial
court
issued
another
order
dismissing
the
motion
for
reconsideration which petitioners
received on July 22, 1998. Five
days later, on July 27, 1998,
petitioners filed a notice of appeal
and paid the appeal fees on August
3, 1998. On August 4, 1998, the
court a quo denied the notice of
appeal, holding that it was filed
eight days late. This was received
by petitioners on July 31, 1998.
Petitioners filed a motion for
reconsideration but this too was
denied
in
an
order
dated
September 3, 1998.
Via a petition for certiorari
and mandamus under Rule 65,
petitioners assailed the dismissal
of the notice of appeal before the

CA. In the appellate court,


petitioners claimed that they had
seasonably filed their notice of
appeal. They argued that the 15day reglementary period to appeal
started to run only on July 22,
1998 since this was the day they
received the final order of the trial
court denying their motion for
reconsideration. When they filed
their notice of appeal on July 27,
1998, only five days had elapsed
and they were well within the
reglementary period for appeal.
On September 16, 1999, the CA
dismissed the petition. It ruled that
the 15-day period to appeal should
have been reckoned from March 3,
1998 or the day they received the
February
12,
1998
order
dismissing
their
complaint.
According to the appellate court,
the order was the final order
appealable under the Rules.
Issue: Whether or not petitioners
file their notice of appeal on time
Ruling: The Supreme Court ruled
in the affirmative invoking Section
3 of Rule 41, it emphasized that
the period for appeal is not only
within fifteen days from the notice
of judgment but also within fifteen
days from notice of the final order
appealed from. If a motion for
reconsideration or a motion for
new trial is denied, such denial is
to be deemed as the final order.
From receipt of such notice of

CRIMINAL PROCEDURE
Case Digests Cassy C. Barras, 2D

denial, the movant has another


fresh period within which to
appeal. The Supreme Court added
that the new 15-day period may be
availed of only if either motion is
filed, otherwise, the decision
becomes final and executor after
the lapse of the original appeal
period from notice of judgment.
The Supreme Court held that
to standardize the appeal periods
provided in the Rules and to afford
litigants fair opportunity to appeal
their cases, the Court deems it
practical to allow a fresh period of
15 days within which to file the
notice of appeal in the Regional
Trial Court, counted from receipt
of the order dismissing a motion
for a new trial or motion for
reconsideration.
Henceforth, this fresh period
rule shall also apply to Rule 40
governing
appeals
from
the
Municipal Trial Courts to the
Regional Trial Courts; Rule 42 on
petitions for review from the
Regional Trial Courts to the Court
of Appeals; Rule 43 on appeals
from quasi-judicial agencies to the
Court of Appeals and Rule 45
governing appeals by certiorari to
the Supreme Court. The new rule
aims to regiment or make the
appeal period uniform, to be
counted from receipt of the order
denying the motion for new trial,
motion
for
reconsideration
(whether full or partial) or any
final order or resolution.

Yu v. Samson-Tatad
G.R. No. 170979
February 9, 2011
Topic: Neypes Doctrine
Doctrine: While Neypes involved
the period to appeal in civil cases,
a fresh period to appeal should
equally apply to the period for
appeal in criminal cases under
Section 6 of Rule 122 of the
Revised
Rules
of
Criminal
Procedure.
Facts: In a May 26, 2005 decision,
the Regional Trial Court convicted
the petitioner of estafa. On
November 16, 2005, the petitioner
filed a notice of appeal with the
RTC, alleging that pursuant to our
ruling in Neypes v. Court of
Appeals, she had a fresh period
of 15 days from November 3, 2005,
the receipt of the denial of her
motion for new trial, or up to
November 18, 2005, within which
to file a notice of appeal. On
December 8, 2005, the prosecution
filed a motion to dismiss the
appeal for being filed ten days late,
arguing that Neypes is not
applicable to appeals in criminal
cases.
Issue: Whether or not the Neypes
rule applies to appeal in criminal
cases
Ruling: The Supreme Court ruled
in the affirmative.

CRIMINAL PROCEDURE
Case Digests Cassy C. Barras, 2D

The Supreme Court said that while


Neypes involved the period to
appeal in civil cases, the Courts
pronouncement of a fresh period
to appeal should equally apply to
the period for appeal in criminal
cases under Section 6 of Rule 122
of the Revised Rules of Criminal
Procedure,
for
the
following
reasons:
First, BP 129, as amended,
the substantive law on which the
Rules of Court is based, makes no
distinction between the periods to
appeal in a civil case and in a
criminal case.
Second, the provisions of
Section 3 of Rule 41 of the 1997
Rules of Civil Procedure and
Section 6 of Rule 122 of the
Revised
Rules
of
Criminal
Procedure,
though
differently
worded, mean exactly the same.
There is no substantial difference
between the two provisions insofar
as legal results are concerned the
appeal period stops running upon
the filing of a motion for new trial
or reconsideration and starts to
run again upon receipt of the order
denying said motion for new trial
or reconsideration. It was this
situation that Neypes addressed in
civil cases. No reason exists why
this situation in criminal cases
cannot be similarly addressed.
Third, while the Court did
not consider in Neypes the
ordinary appeal period in criminal
cases under Section 6, Rule 122 of

the Revised Rules of Criminal


Procedure since it involved a
purely civil case, it did include
Rule 42 of the 1997 Rules of Civil
Procedure on petitions for review
from the RTCs to the Court of
Appeals (CA), and Rule 45 of the
1997 Rules of Civil Procedure
governing appeals by certiorari to
the Supreme Court, both of which
also apply to appeals in criminal
cases, as provided by Section 3 of
Rule 122 of the Revised Rules of
Criminal Procedure.
Clearly, if the modes of
appeal to the CA (in cases where
the RTC exercised its appellate
jurisdiction) and to the Supreme
Court in civil and criminal cases
are the same, no cogent reason
exists why the periods to appeal
from the RTC (in the exercise of its
original jurisdiction) to the CA in
civil and criminal cases under
Section 3 of Rule 41 of the 1997
Rules of Civil Procedure and
Section 6 of Rule 122 of the
Revised
Rules
of
Criminal
Procedure should be treated
differently.
Estarija v. People
G.R. No. 173990
October 27, 2009
Topic: Appeal not a natural right
Doctrine: The right to appeal is not
a natural right or a part of due
process, but merely a statutory
privilege and may be exercised
only in the manner and in

CRIMINAL PROCEDURE
Case Digests Cassy C. Barras, 2D

accordance with the provisions of


the law.
Facts: Edgardo Estarija is a public
officer, being then the Harbor
Master of the Philippine Ports
Authority at Sasa, Davao City An
Information was filed before the
RTC against him for violating
Section 3, paragraph b of Republic
Act No. 3019. T,he Information
alleged that Estarija, while in the
performance of his official function
willfully, unlawfully and feloniously
request and consequently receive
the amount of P5,000.00 from
Davao
Pilot
Association
in
consideration
of
accuseds
issuance of berthing permits.
Estarija pleaded not guilty to the
charge. Thereafter, trial on the
merits ensued. The RTC rendered
a decision convicting Estarija of
the crime charged and imposing
upon him a straight penalty of
seven years. Estarija filed a motion
for reconsideration, which was
denied by the RTC.
On appeal, the Court of
Appeals affirmed the conviction of
Estarija. However, the CA modified
the penalty to an indeterminate
sentence ranging from 6 years and
1 day to 9 years, with the
accessory penalty of perpetual
disqualification from public office.
Issue: Whether or not Estarija
correctly filed his appeal with the
Court of Appeals

Ruling: The Supreme Court ruled


in the negative.
Pursuant to Batas Pambansa
Blg. 129, the Sandiganbayan shall
exercise
exclusive
appellate
jurisdiction over final judgments,
resolutions or orders of regional
trial courts whether in the exercise
of their own original jurisdiction or
of their appellate jurisdiction as
herein provided.
Time and again, it has been
held that the right to appeal is not
a natural right or a part of due
process, but merely a statutory
privilege and may be exercised
only in the manner and in
accordance with the provisions of
the law. The party who seeks to
avail himself of the same must
comply with the requirements of
the rules, failing in which the right
to appeal is lost.
Having failed to comply with
the requirements set forth in the
rules, Estarijas appeal should
have been dismissed by the Court
of Appeals. In the instant case,
instead of appealing his conviction
to the Sandiganbayan, Estarija
erroneously filed an appeal with
the Court of Appeals, in utter
disregard of paragraph 3, Section
4(c) of Republic Act No. 8249. The
Court of Appeals did not notice
this conspicuous misstep, since it
entertained the appeal. This fatal
flaw committed by Estarija did not
toll the running of the period for

CRIMINAL PROCEDURE
Case Digests Cassy C. Barras, 2D

him to perfect his appeal to the


Sandiganbayan.
Because
of
Estarijas failure to perfect his
appeal to the Sandiganbayan
within the period granted therefor,
the Decision of the RTC convicting
him of violating Section 3(a) of
Republic Act No. 3019 has thus
become
final
and
executory.
Inasmuch as the decision of the
RTC has long been final and
executory, it can no longer be
altered or modified. Nothing is
more settled in law than that when
a judgment becomes final and
executory, it becomes immutable
and unalterable.
Hilario v. People
G.R. No. 161070
April 14, 2008
Topic: Appeal not a natural right
Doctrine: While the right to appeal
is statutory, once it is granted by
law, however, its suppression
would be a violation of due
process, a right guaranteed by the
Constitution.
Facts: Petitioner, together with one
Gilbert Alijid was charged with two
counts of Murder in the RTC. RTC
found petitioner and his coaccused
Alijid
guilty
beyond
reasonable doubt of the crime of
homicide.
Petitioner,
unassisted
by
counsel, filed with the RTC a
Petition for Relief together with an
affidavit of merit. In his petition,

petitioner contended that at the


time of the promulgation of the
judgment, he was already confined
at Quezon City Jail and was
directed to be committed to the
National
Penitentiary
in
Muntinlupa; that he had no way of
personally filing the notice of
appeal thus he instructed his
lawyer to file it on his behalf; that
he had no choice but to repose his
full trust and confidence to his
lawyer; that he had instructed his
lawyer to file the necessary motion
for reconsideration or notice of
appeal; that on May 2, 2002, he
was already incarcerated at the
New Bilibid Prisons, Muntinlupa
City
and
learned
from
the
grapevine
of
his
impending
transfer to the Iwahig Penal
Colony, Palawan; that believing
that the notice of appeal filed by
his counsel prevented the Decision
dated December 5, 2001 from
becoming final to warrant his
transfer,
he
instructed
his
representative to get a copy of the
notice of appeal from the RTC; that
no notice of appeal was filed by his
lawyer in defiance of his clear
instructions; and that the RTC
Decision showed that it was
received by his counsel on
February 1, 2002 and yet the
counsel did not inform him of any
action taken thereon.
The RTC denied the petition
for relief as it found petitioner's
claim that his counsel did not heed

CRIMINAL PROCEDURE
Case Digests Cassy C. Barras, 2D

his instruction to file an appeal to


be
unsubstantiated
and
selfserving; and that if there was
indeed such omission committed
by the counsel, such negligence is
binding on the client.
Petitioner insists that the
failure of his counsel to timely file
a notice of appeal of his judgment
of conviction despite his explicit
instruction to do so constitutes
excusable negligence and so his
petition for relief should have been
granted.
Issue: Whether or not the RTC
committed
grave
abuse
of
discretion
in
dismissing
petitioner's petition for relief from
judgment.
Ruling: The Supreme Court ruled
in the affirmative.
To determine the veracity of
petitioner's
claim,
it
was
incumbent upon the RTC to have
required the PAO lawyer to
comment on the petition for relief.
However, it appears from the
records that the RTC only required
the City Prosecutor to file a
comment on the petition. The RTC
Order dismissing the petition for
relief did not touch on the question
whether the PAO lawyer was
indeed negligent in not filing the
appeal as it merely stated that
even if said omission, i.e., not filing
the appeal despite his clients
instruction to do so, should be

considered as negligence, it is a
well-settled rule that negligence of
counsel is binding on the client.
While as a general rule, negligence
of counsel may not be condoned
and should bind the client, the
exception is when the negligence
of counsel is so gross, reckless and
inexcusable that the client is
deprived of his day in court.
Furthermore, the PAO lawyer filed
his Withdrawal of Appearance on
September 30, 2002, almost three
months before the RTC rendered
its assailed Order dated December
13, 2002, dismissing the petition
for relief. The RTC had ample time
to require the PAO lawyer to
comment on the petition for relief
from judgment, before issuing the
questioned Order. Thus, there was
no basis for the RTC to conclude
that the claim of petitioner that he
instructed the PAO lawyer to file
an appeal as self-serving and
unsubstantiated.
The
RTC's
dismissal of the petition for relief
was done with grave abuse of
discretion amounting to an undue
denial of the petitioner's right to
appeal.
In all criminal prosecutions,
the accused shall have the right to
appeal in the manner prescribed
by law. The importance and real
purpose of the remedy of appeal
has been emphasized in Castro v.
Court of Appeals where the
Supreme Court ruled that an
appeal is an essential part of our

CRIMINAL PROCEDURE
Case Digests Cassy C. Barras, 2D

judicial system and trial courts are


advised to proceed with caution so
as not to deprive a party of the
right to appeal and instructed that
every party-litigant should be
afforded the amplest opportunity
for the proper and just disposition
of his cause, freed from the
constraints of technicalities. While
this right is statutory, once it is
granted by law, however, its
suppression would be a violation of
due process, a right guaranteed by
the
Constitution.
Thus,
the
importance of finding out whether
petitioner's loss of the right to
appeal was due to the PAO
lawyer's negligence and not at all
attributed to petitioner.
Philippine Rabbit v. People
G.R. No. 147703
April 14, 2004
Topic: Who may appeal
Doctrine: The accused may appeal
from a judgment of conviction.
However, when he appeals his
conviction,
he
waives
the
protection on the prohibition
against double jeopardy and runs
the risk of being sentenced to a
penalty higher than that imposed
by the trial court.
Facts: Napoleon Roman was found
guilty and convicted of the crime
of reckless imprudence resulting
to
triple
homicide,
multiple
physical injuries and damage to
property and was sentenced to
suffer imprisonment and to pay

damages. The court further ruled


that in the event of the insolvency
of accused, petitioner shall be
liable for the civil liabilities of the
accused. Evidently, the judgment
against accused had become final
and executory.
Admittedly,
accused
had
jumped bail and remained at-large.
The CA ruled that the institution of
a criminal case implied the
institution also of the civil action
arising from the offense. Thus,
once determined in the criminal
case
against
the
accusedemployee,
the
employers
subsidiary civil liability as set forth
in Article 103 of the Revised Penal
Code becomes conclusive and
enforceable.
Simultaneously,
petitioner
filed its notice of appeal from the
judgment of the trial court. The
trial court gave due course to the
notice of appeal. Subsequently,
petitioner filed its brief.
Issue: Whether or not accusedappellee waived his right against
double jeopardy when he appealed
before the trial court
Ruling: The Supreme Court ruled
in the affirmative.
An appeal from the sentence
of the trial court implies a waiver
of the constitutional safeguard
against
double
jeopardy
and
throws the whole case open to a
review by the appellate court. The
latter is then called upon to render

CRIMINAL PROCEDURE
Case Digests Cassy C. Barras, 2D

judgment as law and justice


dictate, whether favorable or
unfavorable to the appellant. This
is the risk involved when the
accused decides to appeal a
sentence of conviction. Indeed,
appellate courts have the power to
reverse, affirm or modify the
judgment of the lower court and to
increase or reduce the penalty it
imposed.
If the present appeal is given
course, the whole case against the
accused-employee becomes open
to review. It thus follows that a
penalty higher than that which has
already been imposed by the trial
court may be meted out to him.
Petitioners appeal would thus
violate his right against double
jeopardy, since the judgment
against him could become subject
to
modification
without
his
consent.
Carino v. De Castro
G.R. No. 176084
April 30, 2008
Topic: Who may appeal
Doctrine: It is only the Solicitor
General who can bring or defend
such actions on behalf of the
Republic of the Philippines or the
People of the Philippines.
Facts: Petitioner Carmencita G.
Cario filed a complaint-affidavit for
violation of Batas Pambansa Blg.
22 against respondent Merlin de
Castro before the Office of the City

Prosecutor
of
Manila.
After
conducting
preliminary
investigation,
Assistant
City
Prosecutor issued a Resolution
finding prima facie evidence and
recommending
respondents
indictment.
Accordingly,
respondent was charged with five
(5) counts of violation of BP 22
before the Metropolitan Trial
Court of Manila.
During
arraignment,
respondent
manifested
her
intention to file a Motion for
Preliminary
Determination
of
Existence of Probable Cause which
was granted. MeTC. found that the
checks were issued by respondent
without valuable consideration;
that petitioner was not authorized
to collect rental payments from
respondent; and that consequently,
respondent can legally refuse
payment on the ground that said
checks
were
issued
without
valuable and legal consideration.
Thus, finding no probable cause
against de Castro, MeTC dismissed
the instant cases.
Petitioner appealed to the
RTC. RTC affirmed the Decision of
the MeTC and dismissed the
appeal for lack of merit. The
motion for reconsideration was
likewise
denied.
Thereafter,
petitioner, through counsel and
with the conformity of Asst. City
Prosecutor, Sawadjaan Issan, filed
a petition for review before the
Court of Appeals. However, the CA

CRIMINAL PROCEDURE
Case Digests Cassy C. Barras, 2D

dismissed the petition because it


was filed only by the private
prosecutor and not by the Office of
the Solicitor General as mandated
by
law.
The
motion
for
reconsideration
was
likewise
denied.
Issue:
Whether
or
not
the
petitioner has a legal standing to
file petition for review
Ruling: The Supreme Court ruled
in the negative.
In criminal proceedings on
appeal in the Court of Appeals or
in the Supreme Court, the
authority to represent the People
is vested solely in the Solicitor
General.
Under
Presidential
Decree No. 478, among the
specific powers and functions of
the OSG was to represent the
government in the Supreme Court
and the Court of Appeals in all
criminal
proceedings.
This
provision has been carried over to
the Revised Administrative Code
particularly in Book IV, Title III,
Chapter 12 thereof. Without doubt,
the OSG is the appellate counsel of
the People of the Philippines in all
criminal cases.
Although the petition for
review before the Court of Appeals
was filed with the conformity of
the Assistant City Prosecutor, such
conformity is insufficient, as the
rules and jurisprudence mandate

that the same should be filed by


the Solicitor General.
While a private prosecutor
may be allowed to intervene in
criminal proceedings on appeal in
the Court of Appeals or the
Supreme Court, his participation is
subordinate to the interest of the
People, hence, he cannot be
permitted to adopt a position
contrary to that of the Solicitor
General. To do so would be
tantamount to giving the private
prosecutor the direction and
control of the criminal proceeding,
contrary to the provisions of law.
In the instant case, the
Solicitor General opined that
petitioner had no legal standing to
file the petition for review and that
the Court of Appeals correctly
dismissed the petition. As such,
the Assistant City Prosecutor or
the private prosecutor cannot take
a contrary view.
People v. Puig
G.R. Nos. 173654-765
August 28, 2008
Topic: Who may appeal
Doctrine: It is only the Solicitor
General who can bring or defend
such actions on behalf of the
Republic of the Philippines or the
People of the Philippines.
Facts:
Provincial
Prosecutors
Office filed before the RTC cases of
112
Qualified
Theft
against
respondents Teresita Puig and

CRIMINAL PROCEDURE
Case Digests Cassy C. Barras, 2D

Romeo Porras who were the


cashier
and
bookkeeper,
respectively,
of
private
complainant
Rural
Bank
of
Pototan, Inc.
After
perusing
the
Informations in these cases, the
trial court did not find the
existence of probable cause that
would
have
necessitated
the
issuance of a warrant of arrest.
Thus, the RTC dismissed the cases
filed.
The
motion
for
reconsideration
was
denied.
Petitioner went directly to the
Supreme Court via Petition for
Review on Certiorari under Rule
45. Respondents further claim that
the Department of Justice, through
the Secretary of Justice, is the
principal party to file a Petition for
Review on Certiorari, considering
that the incident was indorsed by
the DOJ.
Issue: Whether or not the DOJ is
the principal party who may file
the instant petition
Ruling: The Supreme Court said
that the Mobilia Products, Inc. v.
Hajime
Umezawa
ruling
is
instructive. The Supreme Court
thus enunciated: In a criminal
case in which the offended party is
the State, the interest of the
private
complainant
or
the
offended party is limited to the
civil liability arising therefrom.
Hence, if a criminal case is

dismissed by the trial court or if


there
is
an
acquittal,
a
reconsideration of the order of
dismissal or acquittal may be
undertaken,
whenever
legally
feasible, insofar as the criminal
aspect thereof is concerned and
may be made only by the public
prosecutor; or in the case of an
appeal, by the State only, through
the OSG.
Heirs of Palma Burgos v. Court
of Appeals
G.R. No. 169711
February 8, 2010
Topic: Who may appeal
Doctrine:
The
mandate
or
authority to represent the state
lies only in the OSG.
Facts: A number of assailants
attacked the household of Sarah
Marie Palma Burgos while all were
asleep, killing Sarah and her uncle
Erasmo Palma while Victor Palma
and Benigno Oquendo survived the
attack.
Four
months
after
the
incident,
the
police
arrested
Cresencio
Aman
and
Romeo
Martin who executed confessions,
allegedly admitting their part in
the attack. They pointed to two
others who helped them, namely,
Artemio Pong Bergonia and Danilo
Say, and to respondent Co who
allegedly masterminded the whole
thing. The RTC tried the case
against Aman and Martin while the

CRIMINAL PROCEDURE
Case Digests Cassy C. Barras, 2D

three others remained at large.


After trial, the RTC acquitted them
both. After 10 years, respondent
Co surrendered to the NBI. The
prosecution charged him with two
counts of murder for the deaths of
Sarah and Erasmo and two counts
of frustrated murder committed
against Oquendo and Victor. Upon
arraignment, Co pleaded not guilty
to the charges.
Respondent
Co
filed
a
petition for admission to bail. The
RTC granted bail on the ground
that the evidence of guilt of
respondent Co was not strong.
Petitioner heirs of Sarah moved for
reconsideration but the RTC, now
presided over by another judge,
denied the same. This prompted
the victims heirs to file a special
civil action of certiorari with
prayer for a temporary restraining
order or preliminary injunction
before the Court of Appeals. The
CA
dismissed
the
petition,
however, for having been filed
without involving the Office of the
Solicitor General. Petitioner heirs
of
Sarah
moved
for
reconsideration but the CA denied
it for lack of merit
Issue: Whether or not the CA
correctly dismissed the special
civil action of certiorari
Ruling: The Supreme Court denied
the petition and affirmed CAs
decision.

As a general rule, the


mandate or authority to represent
the state lies only in the OSG.
Thus, actions essentially involving
the interest of the state, if not
initiated by the Solicitor General,
are,
as
a
rule,
summarily
dismissed.
In the present case, the
question of granting bail to the
accused is but an aspect of the
criminal action, preventing him
from eluding punishment in the
event of conviction. The grant of
bail or its denial has no impact on
the civil liability of the accused
that depends on conviction by final
judgment. Here, respondent Co
has already been arraigned. Trial
and judgment, with award for civil
liability when warranted, could
proceed even in his absence.
In Narciso v. Sta. RomanaCruz, the Supreme Court allowed
the offended party to challenge
before it the trial courts order
granting bail. But in that case, the
trial court gravely abused its
discretion amounting to lack of
jurisdiction
in
granting
bail
without conducting any hearing at
all. Thus, to disallow the appeal on
the basis of lack of intervention of
the OSG would leave the private
complainant without any recourse
to rectify the public injustice. It is
not the case here. The trial court
took time to hear the parade of
witnesses that the prosecution
presented before reaching the

CRIMINAL PROCEDURE
Case Digests Cassy C. Barras, 2D

conclusion that the evidence of


guilt of respondent Co was not
strong.
People v. Duca
G.R. No. 171175
October 30, 2009
Topic: Who may appeal
Doctrine: It is only the Solicitor
General who can bring or defend
such actions on behalf of the
Republic of the Philippines or the
People of the Philippines.
Facts: Pedro Calanayan filed an
action for ejectment and damages
against Cecilia F. Duca and several
of her relatives. The case was
decided in favor of Calanayan.
Decision
became
final
and
executory. Ultimately properties
belonging to Cecilia hit the auction
block to satisfy judgment and a
certificate of sale was issued in
favor of Jocelyn Barque, the
highest bidder in the auction sale.
Cecilia Duca filed an action for the
Declaration of Nullity of Execution
and Damages with prayer for Writ
of
Injunction and Temporary
Restraining order. When the said
case was heard, Cecilia Duca
testified to the effect that the
house erected on the lot subject of
the ejectment case is owned by her
son Aldrin Duca.
Cecilia (Mother) and Arturo
(Son) Duca feloniously prepared a
Declaration of Real Property over a
bungalow type residential house

by making it appear that the


signature appearing on the sworn
statement of owner is that of her
other son, Aldrin who was abroad.
Accused Arturo F. Duca who
affixed his own signature thereon
and by doing so caused damage to
private
complainant
Pedro
Calanayan.
Because
of
the
misrepresentation,
Cecilia
and
Arturo were able to mislead the
RTC such that they were able to
get
a
TRO
against
Sheriff
Hortaleza and the policemen
ordering them to stop from
evicting the plaintiffs from the
property in question. Both accused
denied that they falsified the
signature
of
Aldrin.
Cecilia
testified
that
she
had
no
participation in the execution as
she was in Manila at that time.
Arturo insisted there was no
falsification. The MTC found him
guilty. RTC affirmed. Duca filed
petition for review before the CA.
The CA ruled that Arturo was
empowered by Aldrin to issue that
tax declaration, hence no crime
had been committed. Aggrieved,
the Solicitor Geneneral declared
that the CA should have given the
Republic a chance to be heard
before ruling such.
Issue: Whether or not the CA erred
by resolving the appeal without
giving the Solicitor General a
chance to be heard.

CRIMINAL PROCEDURE
Case Digests Cassy C. Barras, 2D

Ruling: The Supreme Court ruled


that CA erred by resolving the
appeal without giving Solicitor
General a chance to be heard. The
authority to represent the State in
appeals of criminal cases before
the CA and the Supreme Court is
solely vested in the Office of the
Solicitor General.
Respondent Duca appealed
to the CA from the decision of the
RTC via a petition for review under
Rule 42 of the 1997 Rules of Court.
The respondent was mandated
under Section 1, Rule 42 of the
Rules of Court to serve copies of
his petition for review upon the
adverse party, in this case, the
People of the Philippines through
the OSG. Respondent Duca failed
to serve a copy of his petition on
the OSG and instead served a copy
upon the Assistant City Prosecutor
of Dagupan City. The service of a
copy of the petition on the People
of the Philippines, through the
Prosecutor would be inefficacious
for the reason that the Solicitor
General is the sole representative
of the People of the Philippines in
appeals before the CA and the
Supreme Court. Certiorari was
thereby granted and the Supreme
Court remanded the case to the
CA.
Dico v. Court of Appeals
G.R. No. 141669
February 28, 2005

Topic: Subject matter for review on


appeal
Doctrine:
Where
an
accused
appeals the decision against him,
he throws open the whole case for
review and it then becomes the
duty of the Supreme Court to
correct any error as may be found
in the appealed judgment, whether
it was made the subject of
assignment of errors or not.
Facts: Petitioner Dico was charged
with three counts of violation of
Batas
Pambansa
Bilang
22.
Accused pleaded not guilty to each
of the charges. MTC convicted
petitioner of the crimes charged
and a motion for reconsideration
was denied. Petitioner appealed to
the RTC by filing a notice of
appeal. RTC affirmed en toto the
decision of the MTC. Motion for
reconsideration was denied. Upon
petition for review, CA acquitted
petitioner in one of the cases.
Petitioner
filed
petition
for
certiorari praying that he be
acquitted of the other two cases.
Petitioner argues that not all
essential elements of the offense
under BP Blg 22 are present in the
other two cases.
Issue: Whether or not the Supreme
Court may correct any error on the
appealed judgment
Ruling: The Supreme Court ruled
in the affirmative. It is axiomatic

CRIMINAL PROCEDURE
Case Digests Cassy C. Barras, 2D

that where an accused appeals the


decision against him, he throws
open the whole case for review
and it then becomes the duty of
the Supreme Court to correct any
error as may be found in the
appealed judgment, whether it was
made the subject of assignment of
errors or not.
In
the
first
case,
the
Supreme
Court
checked
the
records of the parties and the
court and found out that there is
an issue as to the identity of the
check. The Supreme Court said
though the issue of identity is not
raised as an error, it should be
considered
in
favor
of
the
petitioner. Thus, There being a
discrepancy in the identity of the
checks
described
in
the
information and that presented in
court, petitioners constitutional
right to be informed of the nature
of the offense charged will be
violated if his conviction is upheld.
With regard to the second
case, the Supreme Court likewise
reverse
and
set
aside
the
conviction of the petitioner there
being no evidence presented by
the prosecution to show that
petitioner had knowledge of the
insufficiency of his funds at the
time he issued the check. Thus, the
second element of the offense was
not satisfied.
Ferrer v. People
G.R. No. 143487

February 22, 2006


Topic: Subject matter for review on
appeal
Doctrine:
Where
an
accused
appeals the decision against him,
he throws open the whole case for
review and it then becomes the
duty of the Supreme Court to
correct any error as may be found
in the appealed judgment, whether
it was made the subject of
assignment of errors or not.
Facts: Brothers Tommy and Ramon
Ferrer were charged before the
RTC of the crime of Frustrated
Homicide for attacking, with intent
to kill, Ricardo Ferrer with a
bladed weapon inflicting upon him
multiple stab wounds.
On
even
date,
another
Information was filed with the
same trial court charging Ramon,
Tommy,
Che-Che
Ferrer
and
Robert Tan with the same crime of
Frustrated Homicide. According to
the information, the accused
armed with bladed weapon, glass
and wooden cane, with intent to
kill, attack, assault, hit and stab
Roque Ferrer, inflicting upon him
multiple injuries.
All of the accused pleaded
not guilty, except for Tan who
remained at large. Both cases
were subsequently consolidated.
After trial, the RTC convicted
Ramon and Tommy guilty of
attempted homicide in one case,
and
frustrated
homicide
in

CRIMINAL PROCEDURE
Case Digests Cassy C. Barras, 2D

another. The CA denied their


motion for reconsideration. Thus,
they elevated the case to the
Supreme Court raising only the
credibility of the witness against
them.
Issue: Whether or not the Supreme
Court may correct any error on the
appealed judgment
Ruling: The Supreme Court ruled
in the affirmative. It is a wellsettled rule that an appeal in a
criminal case throws the whole
case wide open for review and that
it becomes the duty of the Court to
correct such errors as may be
found in the judgment appealed
from, whether they are assigned as
errors or not.
The Supreme Court affirmed
the decision of the CA in finding
the finding petitioner and Ramon
guilty of Attempted Homicide in
Criminal Case No. L-4302 and
Frustrated Homicide in Criminal
Case No. L-4303. However, based
on the evidence adduced, the
Supreme Court found that the
lower courts erred in awarding
some of the damages in favor of
the
victims,
insofar
as
the
petitioner is concerned. Thus, the
Supreme Court modified and
deleted the following awards: the
award of P11,979.60 for unrealized
earnings
and
P5,000.00
as
expenses of litigation in favor of
Roque Ferrer; and the award of

P10,000.00
for
unrealized
earnings,
P2,000.00
for
hospitalization
and
medical
expenses, P5,000.00 as expenses
in
attending
hearings,
and
P8,000.00 for moral damages, in
favor of Ricardo Ferrer.
People v. Montinola
G.R. Nos. 131856-57
July 9, 2001
Topic: Subject matter for review on
appeal
Doctrine:
Where
an
accused
appeals the decision against him,
he throws open the whole case for
review and it then becomes the
duty of the Supreme Court to
correct any error as may be found
in the appealed judgment, whether
it was made the subject of
assignment of errors or not.
Facts: Accused-appellant William
Muyco Montinola was charged
before the RTC with robbery with
homicide in Criminal Case No.
47168 and illegal possession of
firearm in Criminal Case No.
47269. Upon his arraignment,
William entered a plea of not guilty
to
both
charges.
After
the
prosecution had presented three
witnesses,
William
moved
to
withdraw his previous plea of not
guilty; and pleaded guilty to both
charges. RTC found William guilty.
It sentenced him to reclusion
perpetua for the robbery with
homicide and to the penalty of

CRIMINAL PROCEDURE
Case Digests Cassy C. Barras, 2D

death for illegal possession of


firearm.
William appealed before the
CA, arguing that the use of an
unlicensed firearm in the crime of
murder or homicide should be
appreciated as an aggravating
circumstance and not as a
separate offense pursuant to R.A.
No. 8294.
Issue: Whether or not the notice of
appeal of William with the CA is
correct
Ruling: The Supreme Court ruled
in the negative.
It must be noted that it is the
Supreme Court, and not the Court
of Appeals, that has appellate
jurisdiction over all criminal cases
in which the penalty imposed is
reclusion perpetua or higher. As to
judgments in which death penalty
is imposed, such as the judgment
in Criminal Case No. 47169, no
notice of appeal is necessary, as
the same is subject to automatic
review pursuant to Article 47 of
the Revised Penal Code, as
amended by R.A. No. 7659. But as
to judgments imposing reclusion
perpetua, such as that in Criminal
Case No. 47168, the appeal to the
Supreme Court shall be by filing a
notice of appeal with the trial
court.
Williams notice of appeal
from the judgment in Criminal
Cases
Nos.
47168-69,
albeit

erroneous since it was directed to


the
Court
of
Appeals,
may
nevertheless be given due course.
For even without that or even if he
did not appeal from said judgment,
the
Supreme
Court
would
nevertheless review the same
conformably with the ruling in
People vs. Alitagtag, as affirmed in
People vs. Contreras. The Supreme
Court ruled therein that where
cases have been consolidated and
jointly tried, and only one decision
is
rendered
sentencing
the
accused to death in one and to
reclusion perpetua in the others,
he would be deemed to have
appealed from the judgment in the
latter cases.
In resolving the case, the
Supreme Court acquitted William
of the crime of illegal possession
and affirmed the conviction of
robbery with homicide
Guy v. People
G.R. Nos. 166794-96
March 20, 2009
Topic: Subject matter for review on
appeal
Doctrine:
Where
an
accused
appeals the decision against him,
he throws open the whole case for
review and it then becomes the
duty of the Supreme Court to
correct any error as may be found
in the appealed judgment, whether
it was made the subject of
assignment of errors or not.

CRIMINAL PROCEDURE
Case Digests Cassy C. Barras, 2D

Facts:
Petitioners
Ripalda,.
Esperas, Villamor, and Martinez
are officers and employees of the
City Engineers Office of the City of
Tacloban while petitioners Guy and
Grefiel are the Barangay Chairman
and
Barangay
Treasurer,
respectively of Barangay 36,
Sabang District, Tacloban City.
Said petitioners, together with
Amago, a private individual, owner
and
proprietor
of
Amago
Construction were charged in
three separate Informations with
violation of the Anti-Graft and
Corrupt
Practices
Act,
in
connection with the construction
of three infrastructure projects in
Barangay 36, namely: an elevated
path walk, a basketball court and a
day care center.
After an audit investigation
was conducted COA in response to
a letter-complaint, the audit team
found
that
the
Sangguniang
Barangay of Barangay 36, acting
as the Pre-Qualification, Bids and
Awards Committee accepted bid
proposals
from
Amago
Construction without issuing the
proper plans and specifications for
the basketball court and day care
projects and that the work
programs for the day care center
and the elevated path walk were
prepared
long
after
the
construction had been completed.
Likewise, Guy and Grefiel reported
the construction of the projects to
the City Engineers Office only

after they had already been


completed. Petitioner employees
approved the accomplishment of
the projects despite the absence of
material documents, according to
the audit teams report. Finally, the
audit team found material defects
in the projects and discovered that
the contract cost for the basketball
court and elevated path walk was
overpriced.
Sandiganbayan
found
the
petitioners guilty of the offense
charged. Sandiganbayan denied all
their
separate
motions
for
reconsideration.
Petitioners
separately raised the case to the
Supreme Court arguing that the
Sandiganbayan had not acquired
jurisdiction
over
them
or
alternatively, assuming that the
Sandiganbayan
had
actually
acquired
jurisdiction,
the
prosecution failed to prove the
guilt of the accused beyond
reasonable doubt, as well as the
existence of conspiracy.
Issue: Whether or not the Supreme
Court may correct any error on the
appealed judgment
Ruling: The Supreme Court ruled
in the affirmative. In criminal
cases, an appeal throws the whole
case wide open for review and the
reviewing tribunal can correct
errors or even reverse the trial
courts decision on grounds other
than those that the parties raise as

CRIMINAL PROCEDURE
Case Digests Cassy C. Barras, 2D

errors. However, after examining


the records of the case, the
Supreme Court found no cogent
reason to disturb the factual
findings of the Sandiganbayan and
the evidence on record amply
supports
the
findings
and
conclusions of the respondent
court. The elements of the offense
charged have been successfully
proven by the prosecution. Thus,
the Supreme Court dismissed the
case for the lack of merit.
People v. Dela Torre
G.R. No. 176637
October 6, 2008
Topic: Subject matter for review on
appeal
Doctrine: An appeal in a criminal
case opens the entire case for
review. The Court can correct
errors unassigned in the appeal.
Facts: Dela Torre called AAA, an
11 years old girl, and pulled her
towards a parked jeep where
Richie Bisaya and Leo Amoroso
were waiting. Dela Torre brought
AAA inside the jeep and asked her
if she loved him, AAA said no. After
kissing and touching AAA, Dela
Torre passed AAA to Bisaya who
took his turn in kissing and
touching AAA. Bisaya then passed
AAA to Amoroso who poked a knife
on AAA's neck, removed her
clothes, inserted his penis in her
vagina, and kissed her. AAA felt
pain and her vagina bled.

Meanwhile,
AAA's
uncle
went out of the house to look for
AAA. He saw Dela Torre looking
out from the jeep and a man on top
of AAA whom, because of lack of
illumination, he did not recognize.
The men ran away when they saw
AAA's uncle. AAA's uncle tried to
run after the man who was on top
of AAA but was not able to catch
him. He dressed AAA, who was
crying inside the jeep, then
brought her to the house.
AAA's uncle told her what
happened. They immediately went
to Eva Abejero, President of the
Manggahan
Homeowners
Association to report the incident.
Thereafter,
barangay
tanods
looked for Dela Torre, Bisaya, and
Amoroso but were only able to find
Dela Torre inside a hut. The
barangay tanods brought Dela
Torre to Abejero's house, then
brought him to the police station
where AAA positively identified
him as one of the offenders.
Medical report found a deep
healing laceration and shallow
healing lacerations of the genital.
Dela Torre, Bisaya, and
Amoroso with rape were charged
with rape in one information and
acts of lasciviousness in another
information. Since Bisaya was
allegedly
already
dead
and
Amoroso was still at large, trial
proceeded against Dela Torre only.
Dela Torre pleaded not guilty to
both charges. RTC dismissed the

CRIMINAL PROCEDURE
Case Digests Cassy C. Barras, 2D

charge for acts of lasciviousness


and found Dela Torre guilty beyond
reasonable doubt of rape. The CA
affirmed the RTC's Decision.
Issue: Whether or not the Supreme
Court may correct any error on the
appealed judgment
Ruling: The Supreme Court ruled
in the affirmative. An appeal in a
criminal case opens the entire case
for review. The Supreme Court can
correct errors unassigned in the
appeal.
The Supreme Court agrees
with the lower courts that there
was conspiracy exists when the
acts of the accused demonstrate a
common
design
towards
the
accomplishment of the same
unlawful purpose. Thus, the act of
any one was the act of all and each
of them is equally guilty of all the
crimes committed. The Supreme
Court notes that AAA's uncle
positively identified Dela Torre as
one of the offenders even at the
time he executed the sworn
affidavit.
The Supreme Court affirmed the
decision of CA and modified Dela
Torre's civil liability. He is still
ordered to pay AAA P75,000 as
civil indemnity and P75,000 as
moral
damages.
Instead
of
P75,000, however, he is ordered to
pay
AAA
only
P25,000
as
exemplary damages.

People v. Racho
G.R. No. 186529
August 3, 2010
Topic: Subject matter for review on
appeal
Doctrine: An appeal in a criminal
case opens the entire case for
review. The Court can correct
errors unassigned in the appeal.
Facts: A confidential agent of the
police transacted through cellular
phone with appellant for the
purchase of shabu. The agent
reported the transaction to the
police authorities who immediately
formed a team to apprehend the
appellant. The team members
posted themselves
along the
national highway in Baler, Aurora,
and at around 3:00 p.m. of the
same day, a Genesis bus arrived in
Baler. When appellant alighted
from the bus, the confidential
agent pointed to him as the person
he transacted with, and when the
latter was about to board a
tricycle, the team approached him
and invited him to the police
station as he was suspected of
carrying shabu. When he pulled
out his hands from his pants
pocket, a white envelope slipped
therefrom which, when opened,
yielded a small sachet containing
the suspected drug. The team then
brought appellant to the police
station for investigation and the
confiscated specimen was marked
in the presence of appellant. The

CRIMINAL PROCEDURE
Case Digests Cassy C. Barras, 2D

field
test
and
laboratory
examinations on the contents of
the confiscated sachet yielded
positive
results
for
methamphetamine hydrochloride.
Appellant was charged in two
separate informations, one for
violation of Section 5 of R.A. 9165,
for transporting or delivering; and
the second, of Section 11 of the
same
law
for
possessing,
dangerous
drugs.During
the
arraignment, appellant pleaded
"Not Guilty" to both charges.
The RTC rendered a Joint
Judgment convicting appellant of
Violation of Section 5, Article II,
R.A. 9165 but acquitted him of the
charge of Violation of Section 11,
Article II, R.A. 9165. On appeal,
the CA affirmed the RTC decision.
The appellant brought the case to
Supreme Court assailing for the
first time the legality of his arrest
and the validity of the subsequent
warrantless search.
Issue: Whether or not the Supreme
Court may review matters not
raised on appeal
Ruling: It is well-settled that an
appeal in a criminal case opens the
whole case for review. The
Supreme Court is clothed with
ample authority to review matters,
even those not raised on appeal, if
the Supreme Court finds them
necessary in arriving at a just
disposition of the case. Every

circumstance in favor of the


accused shall be considered. This
is
in
keeping
with
the
constitutional mandate that every
accused
shall
be
presumed
innocent unless his guilt is proven
beyond reasonable doubt.
Thus,
even
though
the
inadmissibility of evidence was not
raised by the appellant, the
Supreme Court ruled that due to
an unlawful warrantless seizure,
the
confiscated
item
is
inadmissible in evidence consonant
with Article III, Section 3(2) of the
1987 Constitution, "any evidence
obtained in violation of this or the
preceding
section
shall
be
inadmissible for any purpose in
any proceeding." Without the
confiscated
shabu,
appellants
conviction cannot be sustained
based on the remaining evidence.
Thus, an acquittal is warranted,
despite the waiver of appellant of
his right to question the illegality
of his arrest by entering a plea and
his active participation in the trial
of the case.
People v. Lindo
G.R. No. 189818
August 9, 2010
Topic: Subject matter for review on
appeal
Doctrine: An appeal throws the
whole case open for review, and it
becomes the duty of the appellate
court to correct such errors as may
be found in the judgment appealed

CRIMINAL PROCEDURE
Case Digests Cassy C. Barras, 2D

from, whether they are made the


subject of the assignment of errors
or not.
Facts:
AAA,
the
private
complainant was 11 years old at
the time, and accused-appellant
Lindo was her neighbor. AAA
attended a pabasa at a neighbors
place, during which she fell asleep
under a platform that served as a
stage. While AAA was sleeping,
Lindo took her away to a place
near a creek and AAA woke up, as
Lindo removed her short pants and
underwear, and also undressed
himself. He tried inserting his
penis into her vagina, whereupon
his penis made contact with her
sex organ but there was no
complete penetration. He then
made her bend over, and inserted
his penis into her anus. Lindo then
sensed the arrival of a friend of
AAA, so he discontinued his act,
and told AAA to put on her clothes
and go home. AAA related the
incident to her parents, who
reported it to the barangay
authorities. Lindo was arrested the
same day. AAA was examined by a
Medico-Legal Officer and was
found to have extragenital physical
injuries as well as abrasions on her
anal orifice. Lindo raised the
defenses of denial and alibi.
RTC found the testimony of
AAA to be more credible and
convicted Lindo of statutory rape.
Lindo appealed to the CA, assailing

the credibility of AAA. Lindo failed


to persuade the CA, which
affirmed
his
conviction,
but
modified the award of damages to
AAA. The CA found the award of
civil indemnity proper, in line with
prevailing
jurisprudence.
Exemplary damages were also
found to be proper, for the purpose
of being a deterrent to crime.
Issue: Whether or not the Supreme
Court may review matters not
raised on appeal
Ruling: It is well-settled that an
appeal in a criminal case opens the
whole case for review.
The Supreme Court found
out the both the RTC and the CA
erred in finding only one count of
rape in the present case. From the
information filed, it is clear that
accused-appellant was charged
with two offenses, rape under Art.
266-A, par. 1 (d) of the Revised
Penal Code, and rape as an act of
sexual assault under Art. 266-A,
par. 2. Accused-appellant was
charged
with
having
carnal
knowledge of AAA, who was under
twelve years of age at the time,
under par. 1(d) of Art. 266-A, and
he
was
also
charged
with
committing an act of sexual assault
by inserting his penis into another
persons mouth or anal orifice, or
any instrument or object, into the
genital or anal orifice of another
person
under
the
second

CRIMINAL PROCEDURE
Case Digests Cassy C. Barras, 2D

paragraph of Art. 266-A. Two


instances of rape were indeed
proved at the trial, as it was
established that there was contact
between accused-appellants penis
and AAAs labia; then AAAs
testimony
established
that
accused-appellant was able to
partially insert his penis into her
anal
orifice.
The
medical
examination also supports the
finding of rape under Art. 266-A
par. 1(d) and Art. 266-A par. 2,
considering
the
extragenital
injuries and abrasions in the anal
region reported.
Since the accused-appellant
fails to object to it before trial, the
court may convict the appellant of
as many as are charged and
proved, and impose on him the
penalty for each offense, setting
out separately the findings of fact
and law in each offense. As
accused-appellant failed to file a
motion to quash the Information
he can be convicted of two counts
of rape.
Asian Terminals v. NLRC
G.R. No. 158458
December 19, 2007
Topic: Subject matter for review on
appeal
Doctrine: The Court of Appeals is
imbued with sufficient authority
and discretion to review matters,
not otherwise assigned as errors
on appeal.

Facts: Respondent Labrague was a


stevedore antigo employed with
Asian Terminals, Inc. but failed to
report for work allegedly because
he was arrested and placed in
detention for reasons not related
to his work. After respondent had
been absent for more than one
year, Asian Terminals, Inc., sent
him (a letter requiring him to
explain within 72 hours why he
should not suffer disciplinary
penalty for his prolonged absence.
The following month, petitioner
sent respondent another notice of
similar tenor.
Still, without
response, petitioner terminated
respondent for having incurred
absence without official leave.
Though addressed to respondent,
the foregoing memorandum does
not indicate whether it was sent to
the latter at his last known
address.
Following his acquittal and
release from detention, respondent
reported for work but was advised
by petitioners to file a new
application so that he may be
rehired. Thus, respondent filed
with the NLRC a complaint for
illegal dismissal, separation pay,
non-payment of labor standard
benefits, damages and attorney's
fees. The Labor Arbiter ordered
the petitioner to pay the total sum
of P152,700.00 as separation pay,
13th month and service incentive
leave pay of complainant while
other
issues
or
claims
are

CRIMINAL PROCEDURE
Case Digests Cassy C. Barras, 2D

dismissed for want of substantial


evidence. The NLRC modified the
separation pay in the sum of
P124,800.00 and the awards
representing 13th month pay and
service incentive leave pay are
deleted. Petitioners motion for
reconsideration was denied by the
NLRC. Upon appeal, petitioners
petition for certiorari, the CA
affirmed NLRCs decision and
ordered the petitioners to pay
Labrague his backwages from the
time of his illegal dismissal in July
1996 up to the date of finality of
this decision, computed also in
accordance with Section 2 of the
same CBA.
Issue: Whether or not the CA erred
in ordering the petitioner to pay
respondent of his backwages
Ruling: The Supreme Court ruled
in the affirmative.
Since
the
respondents
dismissal is illegal, he is entitled to
backwages as a matter of right
provided by law. In line with the
ruling in St. Michael's Institute v.
Santos, The fact that the NLRC did
not award backwages to the
respondents
or
that
the
respondents themselves did not
appeal the NLRC decision does not
bar the Court of Appeals from
awarding backwages. While as a
general rule, a party who has not
appealed is
not entitled to
affirmative relief other than the

ones granted in the decision of the


court below, the Court of Appeals
is imbued with sufficient authority
and discretion to review matters,
not otherwise assigned as errors
on appeal, if it finds that their
consideration is necessary in
arriving at a complete and just
resolution of the case or to serve
the interests of justice or to avoid
dispensing piecemeal justice.
Moreover, substantive rights
like the award of backwages
resulting from illegal dismissal
must not be prejudiced by a rigid
and technical application of the
rules. The order of the Court of
Appeals to award backwages being
a mere legal consequence of the
finding that respondents were
illegally dismissed by petitioners,
there was no error in awarding the
same.
Heirs of Marcelino Doronio v.
Heirs of Fortunato Doronio
G.R. No. 169454
December 27, 2007
Topic: Subject matter for review on
appeal
Doctrine: The Court is clothed with
ample authority to review matters,
even if they are not assigned as
errors on appeal, if it finds that
their consideration is necessary at
arriving at a just decision of the
case.
Facts: Spouses Simeon Doronio
and Cornelia Gante, were the

CRIMINAL PROCEDURE
Case Digests Cassy C. Barras, 2D

registered owners of the subject


property. Marcelino Doronio and
Fortunato Doronio, now both
deceased, were among children of
the spouses and that the parties in
this
case
are
their
heirs.
Petitioners are the heirs of
Marcelino
Doronio,
while
respondents are the heirs of
Fortunato Doronio.
On April 24, 1919, a private
deed of donation propter nuptias
was executed by spouses in favor
of Marcelino Doronio and the
latters wife, Veronica Pico. There
is a discrepancy in the property
described in the deed of donation.
Based on OCT No. 352, the
adjacent owners are Zacarias
Najorda and Alejandro Najorda,
whereas based on the deed of
donation, the owner of the
adjacent property is Fortunato
Doronio. Furthermore, said deed of
donation
remained
a
private
document
as
it
was
never
notarized.
Heirs of Marcelino Doronio
and Veronica Pico filed before the
a petition for the Registration of a
Private Deed of Donation. No
respondents were named in the
said petition although notices of
hearing were posted on the
bulletin
boards
of
Barangay
Cabalitaan,
Municipalities
of
Asingan
and
Lingayen.
No
objections to the petition were
interposed, thus, the RTC ordered
a general default which led to the

registration of the deed of


donation, cancellation of OCT No.
352 and issuance of a new
Transfer Certificate of Title No.
44481 in the names of Marcelino
Doronio and Veronica Pico.
Heirs of Fortunato Doronio
filed a petition before the RTC
praying for the declaration of
nullity of the registration of the
private deed of donation and that
TCT No. 44481 be cancelled. The
petition was dismissed. Heirs of
Fortunato Doronio filed an action
for reconveyance and damages
with
prayer
for
preliminary
injunction against petitioner heirs
of Marcelino Doronio before the
RTC. After due proceedings, the
RTC ruled in favor of petitioner
heirs of Marcelino Doronio. Upon
appeal, The CA ruled that the
donation of the entire property in
favor of petitioners predecessors
is invalid on the ground that it
impairs
the
legitime
of
respondents
predecessor,
Fortunato Doronio.
Issue: Whether or not the CA erred
in including the issue of the
validity of the deed of donation
which was not raised by the
respondent
Ruling: The Supreme Court ruled
in the negative.
Although respondents did
not directly raise the issue of
validity of the deed of donation at

CRIMINAL PROCEDURE
Case Digests Cassy C. Barras, 2D

the commencement of the case


before the trial court, it was
stipulated by the parties during
the pre-trial conference. In any
event, the Supreme Court has
authority to inquire into any
question necessary in arriving at a
just decision of a case before it.
Though not specifically questioned
by the parties, additional issues
may also be included, if deemed
important for substantial justice to
be rendered.
Furthermore, the Supreme
Court has held that although a
factual issue is not squarely raised,
still in the interest of substantial
justice, the Supreme Court is not
prevented from considering a
pivotal
factual
matter.
The
Supreme Court is clothed with
ample authority to review palpable
errors not assigned as such if it
finds that their consideration is
necessary in arriving at a just
decision.
A rudimentary doctrine on
appealed
cases
is
that
the
Supreme Court is clothed with
ample authority to review matters,
even if they are not assigned as
errors on appeal, if it finds that
their consideration is necessary at
arriving at a just decision of the
case. Also, an unassigned error
closely related to an error properly
assigned or upon which the
determination of the question
raised by the error properly
assigned is dependent, will be

considered by the appellate court


notwithstanding the failure to
assign it as an error.
Dumo v. Espinas
G.R. No. 141962
January 25, 2006
Topic: Subject matter for review on
appeal
Doctrine: An appellate court is
clothed with ample authority to
review rulings even if they are not
assigned as errors.
Facts: Spouses Dumo are the
owners-possessors of a parcel of
sandy beach resort in La Union.
Severa J. Espinas filed a "Quieting
of Title and/or Ownership and
Possession against spouses Sandy
and Presnida Saldana, subject
matter of the case being the same
resort. Although a decision has
been
rendered
against
the
defendants in the case against
spouses Saldana, the same was not
enforced.
Disgruntled with the refusal
of the sheriff to put them in
possession over the questioned
real property, and in open defiance
with the official action taken by
the sheriff, all defendants acting
for the interest of Espinas took it
upon themselves, employing force,
intimidation, and threat, to enter
the property. Despite protests
made by Spouses Dumo, who were
there then present and visibly
outnumbered by defendants and

CRIMINAL PROCEDURE
Case Digests Cassy C. Barras, 2D

their agents who were armed with


sticks, bolos, hammers, and other
deadly
weapons,
successfully
drove out plaintiffs, and took over
the premises. It was alleged that
the defendants were boasting
aloud that they were under
instructions by the "judge" to do
just that to forcibly enter and
take over the premises. While
inside
the
premises,
they
demolished and totally tore down
all the improvements.
Spouses Dumo prayed for
the payment of actual, moral and
exemplary damages. The MTC
rendered judgment holding that
petitioners were able to prove
their right of possession over the
subject
property.
Respondents
appealed the case to the RTC. The
RTC reversed and set aside the
Decision of the MTC. It also ruled
that as regards damages, the only
damage that can be recovered is
the fair rental value or the
reasonable compensation for the
use and occupation of the leased
property. Spouses Dumo filed a
petition for review with the CA.
The CA held that the MTC
correctly found that the petitioners
were in possession of the subject
land and agreed with the ruling of
the RTC that in forcible entry and
unlawful detainer cases, the only
damage that can be recovered is
the fair rental value or the
reasonable compensation for the

use and occupation of the property


concerned.
Petitioners point out that in
their petition for review filed with
the CA, they did not raise as an
issue the award of damages made
by the MTC. Similarly, respondents
did not specifically assign as error
the award of damages by the MTC
when they appealed the case to
the RTC; neither did they file an
appeal with the CA questioning the
award of damages by the MTC. On
this basis, petitioners conclude
that the award for damages not
having been appealed, the same
had become final and executory.
Hence, the RTC had no authority
to reverse the judgment of the
MTC respecting the award of
damages. In the same way,
petitioners contend that the CA did
not have jurisdiction to rule on the
matter of damages because this
issue was not raised in the appeal
filed before it.
Issue:
Whether
or
not
CA
committed a reversible error in
deleting the awards made by the
MTC considering that the said
awards are already res judicata
Ruling: The Supreme Court ruled
in the negative.
An appellate court is clothed
with ample authority to review
rulings even if they are not
assigned as errors. This is
especially so if the court finds that

CRIMINAL PROCEDURE
Case Digests Cassy C. Barras, 2D

their consideration is necessary in


arriving at a just decision of the
case before it. The Supreme Court
has consistently held that an
unassigned error closely related to
an error properly assigned, or
upon which a determination of the
question raised by the error
properly assigned is dependent,
will be considered by the appellate
court notwithstanding the failure
to assign it as an error. Petitioners
admit in the present petition that
herein respondents, in their appeal
with the RTC, raised the question
of whether or not the prevailing
party may be awarded damages.
Since
this
issue
had
been
seasonably raised, it became open
to further evaluation. It was only
logical and natural for the RTC to
deal with the question of whether
petitioners are indeed entitled to
the damages awarded by the MTC.
Moreover, even if the issue
on damages was not raised by
herein respondents in their appeal
with the RTC, it is not erroneous
on the part of the RTC to delete
the award of damages in the MTC
decision considering that the RTC
judgment reversed the decision of
the MTC.
Lastly, the Supreme Court
agrees with the CA and the RTC
that there is no basis for the MTC
to award actual, moral and
exemplary damages in view of the
settled rule that in ejectment
cases, the only damage that can be

recovered is the fair rental value


or the reasonable compensation
for the use and occupation of the
property.
Comilang v. Burcena
G.R. No. 146853
February 13, 2006
Topic: Subject matter for review on
appeal
Doctrine: An appellate court is
clothed with ample authority to
review rulings even if they are not
assigned as errors.
Facts: Respondents, together with
their mother, Dominga, filed a
complaint
for
annulment
of
document with damages against
petitioner. The complaint alleges
that: (1) respondents are the
owners of a parcel of land located
in Ilocos Sur and the house built
thereon; (2) respondents acquired
the subject property through their
earnings while working abroad; (3)
the subject property was declared
for taxation purposes in Domingas
name as administrator thereof; (4)
petitioner caused the execution of
a Deed of Donation over said
property by taking advantage of
Domingas blindness, old age and
physical infirmity; the said Deed of
Donation is null and void.
The RTC rendered a Decision
in favor of the respondents. The
CA found no cogent reason to
disturb the factual findings of the
RTC and held that the case

CRIMINAL PROCEDURE
Case Digests Cassy C. Barras, 2D

involves an implied trust known as


purchase price resulting trust
under Article 1448 of the Civil
Code. Petitioner asserts that the
existence of an implied trust
between respondents and Dominga
in relation to the subject property
was never treated by the RTC nor
was it brought in issue on appeal
before the CA.
Issue: Whether or not the CA has
the judicial prerogative to rule on
matters not assigned as errors in
an appeal
Ruling: The Supreme Court ruled
in the affirmative.
Once
a
court
acquires
jurisdiction over a case, it has wide
discretion to look upon matters
which, although not raised as an
issue, would give life and meaning
to the law. Indeed, the Rules of
Court
recognize
the
broad
discretionary
power
of
an
appellate court to consider errors
not assigned.
Thus, an appellate court is clothed
with ample authority to review
rulings even if they are not
assigned as errors in the appeal in
these instances: (a) grounds not
assigned as errors but affecting
jurisdiction
over
the
subject
matter; (b) matters not assigned as
errors on appeal but are evidently
plain or clerical errors within
contemplation of law; (c) matters
not assigned as errors on appeal

but consideration of which is


necessary in arriving at a just
decision and complete resolution
of the case or to serve the
interests of justice or to avoid
dispensing piecemeal justice; (d)
matters not specifically assigned
as errors on appeal but raised in
the trial court and are matters of
record having some bearing on the
issue submitted which the parties
failed to raise or which the lower
court ignored; (e) matters not
assigned as errors on appeal but
closely
related
to
an
error
assigned; and (f) matters not
assigned as errors on appeal but
upon which the determination of a
question properly assigned, is
dependent.
In this case, since the
petitioner directly brought in issue
on appeal in his Appellants Brief
the declaration of the RTC that
Dominga could not have validly
disposed of the subject property
because respondents are the real
owners of the subject property
since it was bought with money
sent by them, it was well-within
the CAs authority to review and
evaluate the propriety of such
ruling. In holding that an implied
trust exists between respondents
and Dominga in relation to the
subject property and therefore
Dominga had no right to donate
the same to petitioner, the CA
merely clarified the RTCs findings.

CRIMINAL PROCEDURE
Case Digests Cassy C. Barras, 2D

Spouses Romulo v. Spouses


Layug
G.R. No. 151217
September 8, 2006
Topic: Subject matter for review on
appeal
Doctrine: An appellate court is
clothed with ample authority to
review rulings even if they are not
assigned as errors.
Facts: Petitioners averred in their
complaint that they obtained from
respondents a loan in the amount
of P50,000.00 which subsequently
ballooned to P580,292.00. To
secure the payment of the loan,
respondents
allegedly
duped
petitioners into signing a Contract
of Lease and a Deed of Absolute
Sale covering petitioners house
and lot. The Deed of Absolute Sale
purportedly
facilitated
the
cancellation of petitioners title on
the house and lot and the issuance
of TCT No. 20489 in the name of
respondents.
Thus,
petitioners
prayed for the nullification of the
Deed of Absolute Sale, the
contract of lease and TCT No.
20489, and the award of moral and
exemplary damages. Respondents
argued that spouses voluntarily
executed the deed for the purpose
of
extinguishing
petitioners
indebtedness to respondents.
Prior
to
the
filing
of
petitioners petition, respondent
Moises
filed
an
action
for
ejectment against petitioners. The

MeTC dismissed the complaint for


lack of cause of action. The RTC,
likewise dismissed Moises appeal
based on its finding that the
parties did not intend to enter into
a lease agreement. The Court of
Appeals denied Moises petition for
review on the ground of late filing.
Upon elevation to the Supreme
Court, Moises petition for review
on certiorari was denied with
finality.
With regard to the petition
for nullification, the trial court
rendered judgment in favor of
petitioners. Likewise, respondents
are hereby ordered to pay jointly
and severally petitioners the
following: (1) P100,000.00 as and
by way of moral damages; (2)
P80,000.00 as exemplary damages;
(3) P50,000.00 as by way of
attorneys fees; and (4) the costs of
suit. Respondents elevated the
matter to the CA. The CA reversed
and set aside the RTC Decision.
Issue: Whether or not the Supreme
Court may review matters not
raised on appeal
Ruling: The Supreme Court is
clothed with ample authority to
review matters, even if they are
not assigned as errors on appeal, if
it finds that their consideration is
necessary in arriving at a just
decision of the case. Thus,
although petitioners did not raise
in issue the appellate courts

CRIMINAL PROCEDURE
Case Digests Cassy C. Barras, 2D

reversal of the award of damages


in their favor, the Supreme Court
has the discretion to pass upon
this matter and determine whether
or
not
there
is
sufficient
justification for the award of
damages.
The trial court described
respondents acts as malevolent,
necessitating the award for moral
and exemplary damages. An award
of moral damages would require
certain conditions to be met, to
wit: (1) first, there must be an
injury, whether physical, mental or
psychological, clearly sustained by
the claimant; (2) second, there
must be a culpable act or omission
factually established; (3) third, the
wrongful act or omission of the
defendant is the proximate cause
of the injury sustained by the
claimant; and (4) fourth, the award
of damages is predicated on any of
the cases stated in Article 2219.
However, petitioners are not
completely without fault. Had they
exercised ordinary diligence in
their affairs, petitioners could have
avoided executing documents in
blank. Respondents wrongful act,
although the proximate cause of
the injury suffered by petitioners,
was mitigated by petitioners own
contributory negligence. Hence,
the award of moral and exemplary
damages must be reduced to onehalf of the amounts awarded by
the trial court.

People v. Mamaril
G.R. No. 171980
October 6, 2010
Topic: Change of theory on appeal
Doctrine: Points of law, theories,
issues
and
arguments
not
adequately
brought
to
the
attention of the trial
court
ordinarily will not be considered
by a reviewing court.
Facts: SPO4 Alexis Gotidoc, along
with
the
members
of
Intel
Operatives of Tarlac City Police
Station
and
Philippine
Drug
Enforcement Agency implemented
a search warrant against the
appellant in her residence.
During
SPO4
Gotidocs
search, he found on the top cover
of the refrigerator one plastic
sachet containing white crystalline
substance. Thereafter he prepared
a Certificate of Good Search and
Confiscation Receipt which the
appellant refused to sign. The
plastic sachet was brought to the
crime laboratory for qualitative
examination which yielded positive
results as shabu. Appellant claims
that the police officers framed her
up and planted the shabu inside
her house because of her refusal to
give them money.
The trial court found the
accused-appellant
guilty
of
violation of Section 11, Article II,
of R.A. 9165. On appeal, the Court
of Appeals ruled that the evidence
for the prosecution fully proved

CRIMINAL PROCEDURE
Case Digests Cassy C. Barras, 2D

beyond reasonable doubt the


elements necessary to successfully
prosecute a case for illegal
possession of a regulated drug. In
this appeal with the Supreme
Court, accused-appellant, through
her new counsel from the Public
Attorneys Office, presents new
arguments, that (1) the search
warrant was not based on probable
cause,
hence,
the
evidence
allegedly obtained through it may
not be admitted to support the
accused-appellants conviction; and
(2) the presumption of regularity
in the performance of official
functions by public officers cannot
prevail over the presumption of
innocence.
Issue: Whether or not the new
arguments
of
the
accusedappellant should be allowed
Ruling: The Supreme Court ruled
in the affirmative.
The rule is that a party
cannot change his theory on
appeal nor raise in the appellate
court any question of law or of fact
that was not raised in the court
below or which was not within the
issue raised by the parties in their
pleadings. In a long line of cases,
the Supreme Court held that
points of law, theories, issues and
arguments not adequately brought
to the attention of the trial court
ordinarily will not be considered
by a reviewing court as they

cannot be raised for the first time


on appeal because this would be
offensive to the basic rules of fair
play, justice and due process.
The
Supreme
Court,
however, said that technicalities
must serve, not burden the cause
of justice. It is a prudent course of
action to excuse a technical lapse
and afford the parties a review of
the case on appeal to attain the
ends of justice. Thus, the Supreme
Court allows the new arguments
for the final disposition of the case.
In resolving the case, the
Supreme Court denied the appeal
on the ground that accusedappellant
failed
to
present
substantial rebuttal evidence to
defeat
the
presumption
of
regularity of duty of the issuing
judge.
People v. Musa
G.R. No. 170472
July 3, 2009
Topic: Trial courts factual findings
Doctrine: Trial courts factual
findings
are
accorded
great
respect and even conclusive effect
if duly supported by evidence.
Facts: The prosecution charged the
appellants and Barredo (deceased)
before the RTC with the special
complex crime of robbery with
homicide. On arraignment, the
appellants pleaded not guilty to
the charge.

CRIMINAL PROCEDURE
Case Digests Cassy C. Barras, 2D

Dr. Freyra, Medico-Legal


Officer, declared on the witness
stand that she conducted a
postmortem examination on the
body of Harold Herrera at the
request of the Marikina City
police. According to Dr. Freyra, the
victim died from a gunshot wound
on the right side of his neck.
Nancy, one of the witnesses
presented by the prosecution,
narrated
that
she
and
her
boyfriend, Harold, boarded a
jeepney bound for Montalban.
Michael, Jojo, Robert, August,
Cesar, and Barredo also boarded
on the jeepney. When the jeepney
reached the flyover in Barangka,
Barredo fired a gun and Jojo
declared
a
hold-up.
Barredo
pointed a gun at Harold as he tried
to get the latters wristwatch.
Nancy heard a gunshot, but did
not know that Harold had been hit.
The other appellants, all carrying
bladed weapons, also took part in
divesting the passengers of their
personal belongings. After the
appellants had left, Harold leaned
on Nancy who noticed blood
oozing from Harolds neck. Nancy
directed the driver to bring them
to the nearest hospital. On crossexamination, Nancy testified that
she and Harold identified the six
persons who held them up from
among the many pictures shown to
them in the hospital. At the police
station, she recognized her copassengers who were then in a

detention cell, and she identified


them as the persons who had
robbed them. Thereafter, she
executed an affidavit before PO3
Manuel Ragay (PO3 Ragay). She
likewise testified that she had
given a description of the robbers
to a police inspector.
The RTC convicted the
appellants of the crime of robbery
with homicide. The CA ruled that
the positive, clear, and categorical
testimonies of witnesses Nancy
and Ryan deserve full merit in both
probative weight and credibility
over the mere alibi of the
appellants. The CA upheld the
RTCs finding of conspiracy, as it
had been proven that there was
unity of purpose and design in the
commission of the crime.
Issue: Whether or not appellants
guilt beyond reasonable doubt was
proved
Ruling: The Supreme Court upheld
the decision of the CA.
An
established
rule
in
appellate review is that the trial
courts factual findings including its
assessment of the credibility of the
witnesses, the probative weight of
their
testimonies,
and
the
conclusions drawn from the factual
findings
are
accorded
great
respect and even conclusive effect
if duly supported by evidence.
These
factual
findings
and
conclusions assume greater weight

CRIMINAL PROCEDURE
Case Digests Cassy C. Barras, 2D

if they are affirmed by the CA. This


jurisprudential
rule
notwithstanding,
we
fully
scrutinized the records of this
case; the penalty of reclusion
perpetua that the CA imposed on
the appellants demands no less
than
this
kind
of
careful
consideration.
A distinctive feature of this
case is the presence of a witness,
Nancy, who was inside the jeepney
during the robbery, and who
positively
identified
all
the
appellants
in
her
testimony.
Nancys testimony was clear,
detailed, and straightforward; she
never wavered in pointing to the
appellants as the persons who
robbed her and her co-passengers.
She remained consistent and
steadfast
under
the
defense
counsels
cross-examination.
Nancys
testimony
finds
full
support and corroboration from
the
testimony
of
another
passenger, Ryan, on the events
that transpired before, during, and
after the robbery.
In
considering
the
testimonies of Nancy and Ryan,
the Supreme Court finds it
significant that the defense failed
to refute their testimonies through
evidence of motive impelling them
to falsely testify against the
appellants. The absence of such
evidence immeasurably enhances
the worth and credit of their
testimonies.

People v. Villasan
G.R. No. 176527
October 9, 2009
Topic: Trial courts factual findings
Doctrine: Trial courts factual
findings
are
accorded
great
respect and even conclusive effect
if duly supported by evidence.
Facts: The prosecution charged the
appellant before the RTC with the
crime of murder. The appellant
pleaded not guilty to the charge
upon arraignment.
Gaudioso narrated that he
heard a gunshot and saw the
appellant shoot Bayron twice in
the head. On cross examination,
Gaudioso
recalled
that
he
immediately turned his head
towards the passengers side when
he heard the first shot; two more
shots followed. On re-direct, he
maintained that the appellant shot
Bayron.
P/Sr. Insp. Salinas testified
that appellant tested negative for
the
presence
of
gunpowder
nitrates. On cross examination,
P/Sr. Insp. Salinas explained that
the absence of gunpowder nitrates
was not conclusive proof that
person did not fire a gun.
According to him, a person could
remove traces gunpowder nitrates
by washing his hands.
The RTC convicted the
appellant of the crime of murder
imposing upon him the penalty of

CRIMINAL PROCEDURE
Case Digests Cassy C. Barras, 2D

reclusion
perpetua.
The
CA
affirmed the RTC Decision in toto.
Issue: Whether or not appellants
guilt beyond reasonable doubt was
proved
Ruling: The Supreme Court upheld
the decision of the CA.
An
established
rule
in
appellate review is that the trial
courts factual findings, including
its assessment of the credibility of
the witnesses and the probative
weight of their testimonies, as well
as the conclusions drawn from the
factual findings, are accorded
respect, if not conclusive effect.
These
factual
findings
and
conclusions assume greater weight
if they are affirmed by the CA.
Despite the RTC and the CAs
unanimity in the findings of fact,
the Supreme Court nevertheless
carefully scrutinized the records of
this case, as the penalty of
reclusion perpetua demands no
less than this kind of scrutiny.
Gaudioso, in his July 25,
2000
testimony,
positively
identified the appellant as the
person who shot Bayron inside the
latters own jeepney on June 1,
2000; he never wavered in
pointing to the appellant as the
assailant.
Time
and
again,
the
Supreme Court have ruled that the
credibility of witnesses is a matter
best left to the determination of

the trial court as this tribunal had


the actual opportunity to observe
the witnesses firsthand and to note
their demeanor, conduct, and
attitude.
The
trial
courts
assessment of the credibility of
witnesses is binding on the
Supreme Court, except when that
tribunal overlooked facts and
circumstances of weight and
influence that can alter the result.
The Supreme Court carefully
scrutinized the records of the case
and found no reason to disbelieve
Gaudiosos
straightforward
narration
of
the
events
surrounding Bayrons death. Nor
did there is anything on record
indicating any improper motive
that could have led Gaudioso to
falsely
testify
against
the
appellant. In fact, the appellant
never imputed any ill motive on
Gaudioso. To reiterate, Gaudioso
and the appellant were in the same
jeep during the shooting incident;
there was light inside the jeep.
More importantly, Gaudioso saw
the actual shooting because he
was very near the appellant when
the
latter
shot
Bayron.
To
Gaudioso, what he witnessed must
have been a shocking and startling
event he would not forget in a
long, long time. Under these
circumstances, there is no doubt
on the positive identification of the
appellant as the assailant.
People v. Relos

CRIMINAL PROCEDURE
Case Digests Cassy C. Barras, 2D

G.R. No. 189326


November 24, 2010
Topic: Trial courts factual findings
Doctrine: Trial courts factual
findings
are
accorded
great
respect and even conclusive effect
if duly supported by evidence.
Facts: Appellant Francisco Relos,
together with his brother, Oliver;
sons, Francisco Jr. and Regie;
nephews, Georgie, Larry, and
Olijames; and sons-in-law, Allan
and Steve, was charged for killing
his cousin, Ramon Relos.
The victim and his son,
Ramon Jr., walked along the
highway towards the house of the
victims brother. Appellant was
then leaning on the fence in front
of his house, while his son,
Francisco, Jr., and Oliver were
across the road. The victim was
walking about five meters ahead of
his son and was almost in front of
appellants house when Oliver
approached him while drawing his
knife. Appellant approached the
victim from behind and suddenly
hacked him with a bolo on the
right
shoulder.
Francisco,
Jr.
followed it with hack to the victims
left shoulder. Oliver then placed
his arm over the victims shoulders
and stabbed the victim several
times on the front portion of his
body. The victim fell on the ground
after he was stabbed by Oliver.
Rogelio Jr., the victims nephew,
witnessed
the
incident
and

shouted to Feliciano, Jr. that his


brother had been killed. Regie and
Steve pushed the victims body
towards a canal. Thereafter, Oliver
cut off the victims head, showed it
to passersby, and then dropped it
on the road. He then went to his
house and brought out a gun and a
hand grenade. He tried to shoot
Gloria and Rogelio, Jr., but the gun
would not fire. Instead, he threw
the hand grenade at them, but it
hit a tree near him and exploded.
Thereafter, the police arrived and
arrested
appellant,
Oliver,
Francisco, Jr., Larry, and Georgie.
The others ran away.
During arraignment, Oliver
pleaded guilty, while the others,
including
herein
appellant,
pleaded not guilty. The RTC found
Oliver, and Francisco, Jr. guilty of
murder. On appeal, the CA
affirmed the RTC decision. The CA
gave credence to the testimonies
of Ramon, Jr. and Rogelio, Jr.,
noting that their narration of
events
was
corroborated
by
physical evidence, that is, the
location and the nature of the
wounds sustained by the victim as
indicated in the autopsy report.
The CA, likewise affirmed the trial
courts findings of conspiracy and
the qualifying circumstance of
treachery.
Issue: Whether or not appellants
guilt beyond reasonable doubt was
proved

CRIMINAL PROCEDURE
Case Digests Cassy C. Barras, 2D

Ruling:
The
Supreme
Court
affirmed the decision of the CA.
Findings of trial courts,
which are factual in nature and
which
involve
credibility
of
witnesses, are accorded respect
when no glaring errors; gross
misapprehension of facts; or
speculative,
arbitrary,
and
unsupported conclusions can be
gathered from such findings. None
of these circumstances is present
in this case. The Court therefore
sustains the findings of fact of the
trial court, as affirmed by the CA,
particularly on the weight given to
the testimony of the victims son,
Ramon, Jr.
The testimonies of Ramon, Jr.
and the other witnesses firmly
established appellants identity
and his participation in the killing
of Ramon, Sr. Thus, Olivers
testimony that he did not see
appellant at the scene of the crime
when the victim was killed was
obviously a blatant lie, not worthy
of any credence.
People v. Combate
G.R. No. 189301
December 15, 2010
Topic: Trial courts factual findings
Doctrine: Trial courts factual
findings
are
accorded
great
respect and even conclusive effect
if duly supported by evidence.

Facts: Combate was charged for


killing Prayco and Guiro, Jr. The
prosecutions version of the facts
was that on March 16, 1995,
Tomaro parked his passenger
jeepney
at
the
garage
of
Leopoldos mother, at the house of
Leopoldo. Upon entering the gate
of Leopoldos house, he met
Leopoldo and Edmundo who was
on their way out and also invited
him to join then in drinking liquor
which he decline for he was
already tired. As he was about to
go up the stairs he heard a
gunshot. He rushed back to the
road and saw Combate pointing a
gun at the fallen Leopoldo.
Edmund was about to intervene,
but Combate also shot him at a
very close range. Thereafter, he
fired
another
shot
against
Leopoldo. Tomaro rushed to help
them and pleaded for his life, but,
Combate pointed his gun at
Tomaro and pulled the trigger but
the gun did not fire. Tomaro then
jumped on Combate and was able
to get the gun, which thereafter he
tried to shoot Combate but it did
not fire. Combate then fled to the
direction
of
Bacolod
City.
Combantes defense, on the other
hand, was confined to a denial.
Combate was convicted of
the crime of Murder and Homicide
by the RTC. The CA affirmed the
judgment of the lower court
Combate appealed and averred
that there are inconsistencies in

CRIMINAL PROCEDURE
Case Digests Cassy C. Barras, 2D

the testimony of the witnesses and


that he is not guilty beyond
reasonable doubt.
Issue: Whether or not the trial
court erred in convicting him of
the crimes of homicide and murder
Ruling: The Supreme Court upheld
the ruling of the trial court.
Time-tested is the doctrine
that the trial courts assessment of
the credibility of a witness is
entitled
to
great
weight,
sometimes even with finality. The
Supreme Court will not interfere
with that assessment, absent any
indication that the lower court has
overlooked some material facts or
gravely abused its discretion.
Complementing the above
doctrine is the equally established
rule that minor and insignificant
inconsistencies in the testimony
tend to bolster, rather than
weaken,
the
credibility
of
witnesses, for they show that the
testimony is not contrived or
rehearsed. As the Supreme Court
put it in People v. Cristobal,
Trivial inconsistencies do not rock
the pedestal upon which the
credibility of the witness rests, but
enhances
credibility
as
they
manifest spontaneity and lack of
scheming.
A careful review of the
records shows that the RTC, as
well as the CA, committed no
reversible error when it gave

credence to the testimonies of the


prosecution witnesses, as opposed
to accused-appellants bare denials.
In this case, the Supreme Court
agrees with the trial court that the
alleged
inconsistencies
merely
refer to minor details which do not
affect the witnesses credibility.
People v. Nachor
G.R. No. 177779
December 14, 2010
Topic: Credibility of witness
Doctrine: The trial court is in a
better position than the appellate
court to evaluate testimonial
evidence properly.
Facts: Appellant was accused of
raping AAA, who was 14 years old
that time.
AAA was left alone with her
father, the appellant. While she
was cooking, appellant suddenly
poked a bolo at her neck, pulled
her wrist and dragged her towards
the room. AAA attempted to
escape but the appellants strength
was too much for her. While inside
the room, the appellant, with a
bolo still in his hand, forced AAA to
lie down on the bed. Thereafter, he
took off his shorts and underwear
and started kissing her neck and
breasts. He proceeded to mount
AAA, inserted his penis inside her
vagina.
After
the
appellant
satisfied his lust, he again poked
his bolo at AAA and threatened to
kill her, her mother and siblings if

CRIMINAL PROCEDURE
Case Digests Cassy C. Barras, 2D

she would report the incident to


anyone. AAA kept the incident to
herself out of fear. AAA was again
raped by the appellant when her
mother and siblings were not
around.
A few months later, the
abdomen of AAA started to bulge.
The mother of AAA therefore
sought the assistance of one of her
wedding sponsors to whom AAA
finally revealed the sexual abuse
she experienced in the hands of
her father. After this revelation
was relayed to her mother, AAA
was immediately taken to the
Regional Office of the DSWD
where she declared in an interview
that her father sired the child she
was carrying. She was then taken
to the NBI for a medico-legal
examination. The results confirmed
that AAA was pregnant. AAA gave
birth to a baby boy she named
BBB. The appellant asserted that it
was
his
son,
Randy,
who
impregnated AAA. He confronted
Randy on the pregnancy of AAA
but the latter refused to reply and
cried instead. The appellant also
claimed that his wife assisted AAA
in filing the cases to get rid of him
so that she could continue having
an affair with the man often seen
in her company.
RTC found appellant guilty of
the crime of rape in both charges
sentencing him of death penalty.
Upon appeal, the CA found him
guilty of two counts of qualified

rape and sentencing him in each


case to suffer the supreme penalty
of death.
Issue: Whether or not the trial
court erred in giving full credence
weight to the testimony of the
private complainant
Ruling: The Supreme Court agrees
with the observations of the trial
court, as sustained by the CA, that
the testimony of AAA on both
occasions of her rape is worthy of
credence. Her statements under
oath are sufficient evidence to
convict the appellant for having
carnal knowledge of her by means
of force and intimidation. AAA
positively identified the appellant
as her abuser. She never wavered
in her testimony and maintained
even on cross-examination that the
appellant was her rapist.
On the issue of credibility of
witnesses,
the
trial
courts
assessment deserves great weight,
and is even conclusive and
binding, if not tainted with
arbitrariness or oversight of some
fact or circumstance of weight or
influence. Having the advantage of
directly observing the deportment
and manner of testifying of the
witness, the trial court is in a
better position than the appellate
court to evaluate testimonial
evidence properly.
In this case, the Supreme
Court sees no reason to deviate

CRIMINAL PROCEDURE
Case Digests Cassy C. Barras, 2D

from the findings of the trial court


as affirmed by the CA. The
evaluation of the testimony of AAA
has been appreciated properly and
the evidence is overwhelming to
convict the appellant of the crime
charged.
The Supreme Court affirmed
with modifications that appellant is
sentenced to suffer the penalty of
reclusion
perpetua
without
eligibility for parole for each count
of rape.
Cacao v. People
G.R. No. 180870
January 22, 2010
Topic: Factual findings of the trial
court
Doctrine: If there is any indication
that the trial court overlooked
certain facts or circumstances
which would substantially affect
the disposition of the case, the
Court will not hesitate to review
the same.
Facts: Two separate informations
were filed against Joseph Canlas
and Cacao indicting them for
violation of Section 11, Article II of
RA 9165 before the RTC. When
arraigned, Cacao pleaded not
guilty.
PO3 Pang-ag received a
telephone call from an informant
about a drug session being held
inside Room 5 of the Starlight
Hotel. Acting on the information,
PO3 Pang-ag, together with PO2

Jonel Mangapit, went immediately


to the Starlight Hotel. PO3 Pangag and PO2 Mangapit approached
the lady clerk manning the
information counter of Starlight
Hotel and inquired about the
alleged drug session at Room 5 of
the hotel. The lady told them that
the roomboy of the hotel was
about to deliver a softdrink to
Room 5 and they could follow him.
Thus, PO3 Pang-ag and PO2
Mangapit followed the roomboy to
Room 5. PO3 Pang-ag and PO2
Mangapit saw petitioner sniffing
shabu assisted by Joseph Canlas.
PO3 Pang-ag and PO2 Mangapit
arrested petitioner and Joseph and
confiscated from them the drug
paraphernalia,
glass
tooter,
scissors,
lighters
and
plastic
sachets. PO2 Mangapit frisked
petitioner and recovered from him
one plastic sachet containing
shabu. PO3 Pang-ag and PO2
Mangapit brought them to the
Laoag City Police Station and
turned them over to the police
officer
on
duty
while
the
confiscated items were turned over
to SPO3 Loreto Ancheta. The PNP
laboratory
conducted
an
examination on the specimen
recovered from appellant and his
companion which tested positive
for shabu. Petitioner argued that
he is innocent and he was only
framed-up.
The trial court rendered its
judgment finding Cacao guilty of

CRIMINAL PROCEDURE
Case Digests Cassy C. Barras, 2D

the offense charged. Upon appeal,


the appellate court rendered
judgment
affirming
Cacaos
conviction. As to the alleged
contradictory
statements,
the
appellate court ruled that they refer
only to minor details which are not
sufficient to overthrow the probative
value accorded them by the trial
court.
Issue: Whether or not the Supreme
Court may deviate from the factual
findings of the trial court
Ruling: As a general rule, factual
findings and conclusions of the trial
court and the CA are entitled to great
weight and respect and will not be
disturbed on appeal. However, if
there is any indication that the trial
court overlooked certain facts or
circumstances
which
would
substantially affect the disposition of
the case, The Supreme Court will not
hesitate to review the same. In this
case, the Supreme Court finds it
imperative to review the factual
findings of the trial court because of
certain
inconsistencies
in
the
testimonies of the prosecution
witnesses on material points.
Jurisprudence holds that in
prosecution of cases involving illegal
possession of prohibited drugs, the
prosecution must establish with
moral certainty the elemental act of
possession of a prohibited substance
coupled with the fact that such
possession is not authorized by law.

Essential, however, in a drug-related


case is that the identity of the
dangerous drug be established
beyond reasonable doubt. Since the
dangerous drug constitutes the
corpus delicti of the offense and the
fact of its existence is vital to a
judgment of conviction, it behooves
upon the prosecution to establish and
prove with certainty that the
dangerous drug presented in court
as evidence against the accused is
the same item recovered from his
possession.
Upon scrutiny in detail the
testimonies of the prosecution
witnesses and found not only glaring
inconsistencies on material points
but more importantly a failure to
identify indubitably the prohibited
drug allegedly confiscated from
Cacao. Petitioners defenses of denial
and
frame-up
are
concededly
inherently weak and commonly used
in drug-related cases. However, it
must be stressed that conviction of
the accused must rest not on the
weakness of the defense but on the
strength of the evidence of the
prosecution.
Thus, the Supreme Court set
aside and reversed the CA decision
and acquitted Cacao on the ground
of reasonable doubt.
People v. Palma
G.R. No. 189279
March 9, 2010
Topic: Credibility of witness

CRIMINAL PROCEDURE
Case Digests Cassy C. Barras, 2D

Doctrine: The trial court is in


the best position to observe
that
elusive
and
incommunicable evidence of
the witnesses deportment
on the stand while testifying,
which opportunity is denied
the appellate court.
Facts: Appellant forcibly brought
AAA to a dark place under the C-5
bridge, covered by big stones that
blocked the view of passersby.
Then, appellant hit her on the
stomach and told her to undress.
When she refused, he slashed her
clothes
and
underwear
and
threatened her with the knife.
When AAA was already naked,
appellant lowered his own short
pants and briefs, and forcibly
inserted his penis into her vagina.
After satisfying his lust, he
withdrew his penis and fixed
himself.
Appellant
thereafter
grabbed AAAs bag and took her
cellular phone and transportation
money amounting to P40.00. AAA
was able to locate only her blouse
that she used to cover herself. A
male passerby helped her by
giving her a pair of short pants,
and escorted her to Bagong Ilog
Barangay Hall, where the incident
was entered in the police blotter.
While
conducting
their
routine patrol, members of the
barangay security force chanced
upon appellant, whom they found
sleeping, using several ladies

wallets as pillows, under the C-5


bridge, near the place where AAA
was raped. The barangay security
force brought appellant to the
Barangay Hall for verification.
That same day, AAA positively
identified
appellant
as
her
assailant. Appellant immediately
bowed his head and asked AAA for
forgiveness.
Appellant was charged in an
Information for Robbery with
Rape. When arraigned, appellant
pleaded not guilty. Appellant
denied liability and insisted that he
only saw AAA in the precinct. The
RTC found appellant guilty beyond
reasonable doubt of Robbery with
Rape. On appeal, the appellate
court affirmed the RTC decision in
its entirety. Petitioner questions
the credibility of AAA because of
allegedly inconsistent statements
in her testimony.
Issue: Whether or not the Supreme
Court may deviate from the
findings of facts of the trial court
on the credibility of witnesses and
their testimonies
Ruling: It is well-settled that
findings of fact of the trial court on
the credibility of witnesses and
their testimonies are generally
accorded great respect by the
appellate court. The assessment of
the credibility of witnesses is a
matter best left to the trial court,
because it is in the best position to

CRIMINAL PROCEDURE
Case Digests Cassy C. Barras, 2D

observe
that
elusive
and
incommunicable evidence of the
witnesses deportment on the stand
while testifying, which opportunity
is denied the appellate court.
People v. Atadero
G.R. No. 183455
October 20, 2010
Topic: Credibility of witness
Doctrine: The trial court is in the
best position to observe that
elusive
and
incommunicable
evidence
of
the
witnesses
deportment on the stand while
testifying, which opportunity is
denied the appellate court.
Facts: Accused-appellant Romy
Atadero visited the home of private
complainant XXX and her husband
YYY to get acquainted with XXXs
husband and to look for land to
work on. Accused appellant stayed
in their house from September
1998 to February 1999 until he
moved to the house of Carmelita
Lago.
One
morning
accusedappellant arrived in their house
and immediately looked for XXXs
husband. Upon learning that her
husband was not around, he lay
down on the floor of the room.
XXX, for her part, went inside the
room and rocked the cradle of her
child. Meanwhile, appellant closed
the main door. When XXX opened
the door that appellant closed, he
got up, held her shoulders, and

told her that he was going to kiss


her. She resisted by pushing him
away, but he pointed a gun at her
head. As he held his gun with his
left hand, he used his right hand to
unzip his pants, held her left
shoulder and pushed her to the
floor. Accused-appellant removed
XXXs shorts and panty, went on
top of her and forcefully forced his
penis into her vagina, even as she
resisted by moving her body left
and right. He threatened her not
to tell anyone. XXX also narrated
two other instances of sexual
assault. As appellant was acquitted
on these two charges, the facts
relating thereto become irrelevant
in this appeal.
The accused interposed the
defense of alibi and advanced the
theory that the XXX and YYY
imputed such a crime on him
because he was the president of a
rival farmers association. His alibi
was corroborated by Valiente who
testified that they were together
cooking, eating, and working at
the time the rape allegedly took
place.
The RTC found accusedappellant guilty of the crime rape.
On appeal, the CA affirmed in toto
the RTC conviction.
Issue: Whether or not the trial
court erred in giving full weight
and credence to the incredible
testimony
of
the
private
complainant.

CRIMINAL PROCEDURE
Case Digests Cassy C. Barras, 2D

Ruling:
The Supreme Court
agrees with both the RTC and CA
in finding the testimony of the
private
complainant
to
be
straightforward,
candid,
categorical,
spontaneous,
consistent,
and
never
contradictory despite the rigorous
and gruelling cross-examination,
thereby bearing the earmarks of
truthfulness. The RTC also noted
that the private complainant was
crying
during
her
direct
examination.
Such,
further
bolsters the credibility of her
testimony as the crying of a victim
during her testimony is evidence of
the credibility of the rape charge
with the verity born out of human
nature and experience.
A review of Article 266-A of
the Revised Penal Code will reveal
that the law does not impose upon
a rape victim the burden of
proving resistance. All that is
necessary is that force and
intimidation were employed by the
accused
against
her,
which
enabled him to commit the crime.
The oft-repeated principle is
that the assessment of the
credibility of witnesses and their
testimonies is best undertaken by
a trial court because of its unique
opportunity
to
observe
the
witnesses firsthand and to note
their demeanor, conduct and
attitude under examination. Its
findings on such matters are

binding
and
conclusive
on
appellate courts unless some facts
or circumstances of weight and
substance have been overlooked,
misapprehended, misinterpreted,
or the court gravely abused its
discretion.
None
of
these
exceptions are present in this case.
Although the conduct of the
victim immediately following the
alleged sexual assault is of utmost
importance as tending to establish
the truth or falsity of the charge of
rape, it is not accurate to say that
there is a typical reaction or norm
of behavior among rape victims, as
not every victim can be expected
to act conformaby with the usual
expectation of mankind and there
is no standard behavioral response
when one is confronted with a
strange or startling experience,
each situation being different and
dependent
on
the
various
circumstances prevailing in each
case.
People v. Alipio
G.R. No. 185285
October 5, 2009
Topic: Credibility of witnesses
Doctrine:
The
trial
courts
evaluation of the credibility of
witnesses and their testimonies is
deserving of the highest respect
because of its unique opportunity
to observe the witnesses firsthand
and note their demeanor, conduct,
and
attitude
under
grilling
examination.

CRIMINAL PROCEDURE
Case Digests Cassy C. Barras, 2D

Facts: An Information filed with


the RTC charged Paul with one
count of rape. Upon arraignment,
Paul entered a plea of not guilty.
AAA is a 41-year old mentally
retarded woman whom Marilou
Alipio often hired to watch over
her children. Marilou is Pauls
sister. Sometime in June 2000,
Marilou sent AAA to her fathers
house to borrow money. Saul,
Marilous father, told AAA that he
would give the necessary amount
to Marilou directly. While about to
head for home, AAA heard Paul
calling her from his house.
Suddenly, Paul held her hand,
pushed her inside and, while
covering AAAs mouth, brought her
to his bedroom. He then removed
her shorts and panty and likewise,
undressed himself. Eventually, he
entered her, first using his finger,
then his penis. Paul threatened her
with death should she disclose to
anybody what had just happened
between them. Several months
later, BBB, AAAs mother, noticed
that the latter had missed her
monthly period. AAA told her
mother what Paul had done to her.
Thereupon, AAAs mother went to
see Marilou and her father to
apprise
them
about
AAAs
pregnancy. The Alipios promised
financial help, albeit Paul would
later disown responsibility for
AAAs condition. AAA eventually
gave birth to a baby girl.

Psychiatric evaluation revealed


that AAA, although 42 years old at
that time, had the mental capacity
and disposition of a nine or 10
year-old child. To the doctor, AAA
was very well qualified to be a
witness provided the questions are
asked in a simple manner.
To
establish
Pauls
innocence, the defense presented
four witnesses and that he himself
was a psychiatric case that he
was suffering from schizoaffective
disorder,
a
temporary
and
reversible psychiatric condition
affecting basically an individuals
thinking, perception, and emotion.
The RTC convicted Paul of
rape penalized under paragraph
1(a) and (d), Article 266-A of the
Revised Penal Code. The CA
denied Pauls appeal and affirmed
the RTCs judgment.
Issue: Whether or not the trial
court erred in giving full weight
and credence to the incredible
testimony
of
the
private
complainant.
Ruling: The Supreme Court ruled
in the negative.
AAAs
testimony
is
not
without
discrepancies
and
inconsistencies, given of course
her mental state. Rape is a
harrowing experience, the exact
details of which are usually not
remembered. Inconsistencies, even
if they do exist, tend to bolster,

CRIMINAL PROCEDURE
Case Digests Cassy C. Barras, 2D

rather than weaken, the credibility


of the witness, for they show that
the testimony was not contrived or
rehearsed. Minor inconsistencies
in
testimonies
should
be
disregarded. Likewise, a medical
examination of the victim is not
indispensable to the successful
prosecution for rape inasmuch as
her testimony alone, if credible, is
sufficient
to
convict
the
perpetrator of the crime. And as
aptly observed by the OSG, AAA
was already pregnant when BBB
found out about the rape and that
the former had already given birth
when she testified, making a
hymeneal examination a worthless
exercise.
AAAs mental condition does
not prevent her from being a
competent and credible witness.
As has been held, a mental
retardate is not disqualified from
being a witness; the retardates
mental condition does not, on that
ground alone, vitiate his or her
credibility.
If
the
mental
retardates testimony is coherent,
it is admissible in court. Evidently,
the trial court had ascertained the
veracity and credibility of AAAs
testimony sufficient to support a
finding of conviction
The unyielding rule has been
that the trial courts evaluation of
the credibility of witnesses and
their testimonies is deserving of
the highest respect because of its
unique opportunity to observe the

witnesses firsthand and note their


demeanor, conduct, and attitude
under grilling examination. Such
assessment binds the Court except
when the assessment was reached
arbitrarily or when the trial court
overlooked,
misunderstood,
or
misapplied
some
facts
or
circumstances of weight and
substance
which
could
have
affected the results of the case.
None of these exceptions exists in
this case.
People v. Bernabe
G.R. No. 185726
October 16, 2009
Topic: Credibility of witnesses
Doctrine:
The
trial
courts
evaluation of the credibility of
witnesses and their testimonies is
deserving of the highest respect
because of its unique opportunity
to observe the witnesses firsthand
and note their demeanor, conduct,
and
attitude
under
grilling
examination.
Facts:
Accused-appellant
with
murder. When arraigned, accusedappellant pleaded not guilty and
trial on the merits ensued.
Accused-appellant
invited
Jomar, Alvin, and three other girls
to his house for a drinking spree.
Accused-appellant
allowed
his
guests to stay on and sleep in his
bedroom. At about 2:00 a.m. of
May
26,
2005,
Jomar
was
awakened by the voice of accused-

CRIMINAL PROCEDURE
Case Digests Cassy C. Barras, 2D

appellant telling Alvin to join him


in buying some cigarettes. Outside
the house, they met the victim
Jann Michael Olivo. Accusedappellant poked a gun at the
victim and ordered the latter to go
with them to accused-appellants
house
where
he
started
questioning the victim why the
latter was roaming around the
house. Jomar, who was in the
bedroom, heard accused-appellant
to forcing the victim to tell him
who is paying him to look over
him. Unable to go back to sleep,
Jomar
peeped
outside
the
bedroom.
He
saw
accusedappellant holding a piece of wood
while the victim was sitting near
the front door of the house. He
also saw Alvin, who was seemingly
frightened, seated near another
room.
Accused-appellant
commanded the victim to bend
over and the former then put lotion
on the victims butt. The victim
shouted in pain as accusedappellant inserted the toothbrush
into the victims anus. Accusedappellant continued to interrogate
the victim and hit the latter two
times with a metal pipe. He then
ordered the victim to lie down and
tied the latters hands with a
plastic straw. Accused-appellant
strangled the latter with the wire.
While
doing
this,
accusedappellant called out to Jomar and
Alvin and ordered the two to hold
the struggling victims feet. When

the victim stopped breathing,


accused-appellant got hold of two
sacks
from
his
bodega
or
stockroom, put the lifeless body
inside the sacks, placed it at a
corner of the house, and covered it
with yero or GI sheets. Accusedappellant commanded Alvin and
Jomar to load the victims body on
the sidecar and dispose of the
same. The two dumped the corpse
in a water lily-filled vacant.
Thereafter,
accused-appellant
threatened Alvin and Jomar that he
will kill them if they report the
incident to the police. Jomar and
Alvin then went their separate
ways and into hiding.
Relatives of Jomar and Alvin
arranged the surrender of the two
minors to the authorities. They
voluntarily offered themselves to
help in the immediate arrest of
accused-appellant. Alvin and Jomar
executed
their
respective
Sinumpaang Salaysay.
Accused-appellant
was
arrested on follow-up operation at
his hideout. The defense presented
different set of facts. Accusedappellant denied the charges
hurled against him. He claimed
that he had no capacity to strangle
the victim because he could not
use his left hand effectively after
undergoing an operation on his
two (2) fingers.
The
RTC
rendered
its
judgment
convicting
accusedappellant of the crime charged.

CRIMINAL PROCEDURE
Case Digests Cassy C. Barras, 2D

The CA affirmed with modification


in that, the accused-appellant is
sentenced to reclusion perpetua
without eligibility for parole.
Issue: Whether or not accusedappellants guilt is proved beyond
reasonable doubt
Ruling:
The
Supreme
Court
affirmed
accused-appellants
conviction.
The Supreme Court is aware
that there were discrepancies in
the
testimonies
of
the
two
eyewitnesses, particularly as to
their
participation
(or
nonparticipation) in the murder of the
victim.
These
discrepancies,
however, are not sufficient to
negate the guilt of accusedappellant. The evident attempt of
Alvin and Jomar to downplay their
participation in the commission of
the crime did not completely
render weightless the evidentiary
value of their testimonies.
Alvin
and
Jomar
were
consistent in pointing to accusedappellant as the one who hit the
victim with a metal pipe in the
head causing the latter to lose
consciousness, and who strangled
the victim to death using a G.I.
wire (alambre).
The trial court accorded
greater weight to the testimonies
of the prosecution witnesses and
dismissed
accused-appellants
defenses of denial and alibi,

holding the same as self-serving


evidence that cannot be given
evidentiary weight greater than
that of credible witnesses who
testify on affirmative matters. As
often stressed by the Supreme
Court, the issue of credibility of
witnesses is a function properly
lodged with the trial court, whose
findings are entitled to great
weight and accorded the highest
respect by the reviewing courts.
People v. Lazaro, Jr.
G.R. No. 186418
October 16, 2009
Topic: Credibility of witnesses
Doctrine:
The
trial
courts
evaluation of the credibility of
witnesses and their testimonies is
deserving of the highest respect
because of its unique opportunity
to observe the witnesses firsthand
and note their demeanor, conduct,
and
attitude
under
grilling
examination.
Facts: Two separate informations
were filed before the RTC against
appellant for illegal sale and
possession of shabu. The next day,
another information was filed with
the RTC against appellant for
illegal use of shabu. Subsequently,
these cases were consolidated.
When
arraigned,
appellant,
pleaded not guilty to each of the
charges.
Appellant was arrested by
the buy-bust operation. SPO1

CRIMINAL PROCEDURE
Case Digests Cassy C. Barras, 2D

Indunan marked with DG-06-15-04


the plastic sachet containing white
substance sold to him by appellant,
as well as the plastic sachet with
white substance found inside the
green box. The defense proffered
the testimonies of appellant and
his father, namely Alfredo Lazaro,
Sr.
to
refute
the
foregoing
accusations. Appellant denied any
liability and claimed he was
framed.
After trial, the RTC rendered
a Decision convicting appellant in
all of the criminal cases. The
appellate
court
affirmed
the
conviction of appellant. However,
it acquitted the appellant of illegal
use of shabu.
Issue: Whether or not the trial
court gravely erred in giving
credence to the testimony of the
prosecution witnesses
Ruling: The Supreme Court ruled
in the negative.
The
testimonies
of
the
prosecution witnesses regarding
appellants
illegal
sale
and
possession of shabu are consistent
with the documentary and object
evidence
submitted
by
the
prosecution.
The RTC and the
Court of Appeals found the
testimonies of the prosecution
witnesses to be credible.
Both
courts also found no ill motive on
their part to testify against
appellant.

The rule is that the findings


of the trial court on the credibility
of witnesses are entitled to great
respect because trial courts have
the advantage of observing the
demeanor of the witnesses as they
testify. This is more true if such
findings were affirmed by the
appellate court. When the trial
courts findings have been affirmed
by the appellate court, said
findings are generally binding
upon the Supreme Court.
The defenses of denial and
frame-up have been invariably
viewed by the Supreme Court with
disfavor for it can easily be
concocted and is a common and
standard
defense
ploy
in
prosecutions
for
violation
of
Dangerous Drugs Act. In order to
prosper, the defenses of denial and
frame-up must be proved with
strong and convincing evidence.
The appellant failed to present
sufficient evidence in support of
his claims. Aside from his selfserving assertions, no plausible
proof was presented to bolster his
allegations.
People v. Beduya
G.R. No. 175315
August 9, 2010
Topic: Credibility of witness
Doctrine: Appellate courts, as a
rule, will not disturb the findings
of the trial court on the credibility
of witnesses.

CRIMINAL PROCEDURE
Case Digests Cassy C. Barras, 2D

Facts:
Both
appellants
were
arrested for committing the crime
of murder. They entered separate
pleas of not guilty during their
arraignment.
At around 12:30 a.m. of May
7, 2002, the victim and his son,
Dominador Acope, Jr. were roused
from their sleep by a voice coming
from the road in front of their
house. The victim saw Bughao who
readily identified himself and said
that Elizer pointed a knife at him.
As the Beduya brothers entered
the yard of the victims house,
Bughao hid himself. While in
hiding, he saw the Beduya
brothers approach the victim after
they were advised to go home
since it was already late. The
Beduya brothers did not heed the
advice and instead Ric slapped the
victim while Elizer stabbed him.
The victim retaliated by striking
them with a piece of wood he got
hold of. Elizer and Ric ran away
but one of them stumbled on the
pile of firewood and the clothesline
in the yard before they succeeded
in departing from the premises.
The incident was reported to their
Barangay Captain, who responded
by going to the residence of the
victim. The victim told him that he
was boxed by Ric and stabbed by
Elizer. On the next day, the victim
died. Beduya brothers both denied
the allegations against them.
The trial court rendered
judgment
in
favor
of
the

prosecution. On the other hand,


the trial court found Eduardo, at
86 years of age as unreliable the
witnesses
presented
by
the
defense. The CA affirmed RTCs
decision.
Issue: Whether or not the trial
court gravely erred in giving
credence to the testimony of the
prosecution witnesses
Ruling: The Supreme Court ruled
in the negative.
It has been "consistently
held that appellate courts, as a
rule, will not disturb the findings
of the trial court on the credibility
of witnesses. The Supreme Court
has sustained trial courts in this
respect, considering their vantage
point in their evaluation of
testimonial evidence, absent any
showing of serious error or
irregularity that otherwise would
alter the result of the case." Here,
the Supreme Court finds no
serious irregularity.
Besides, the inconsistencies
ascribed
to
the
prosecution
witnesses involve minor details,
too trivial to adversely affect their
credibility. Said inconsistencies do
not depart from the fact that these
witnesses saw the fatal stabbing of
the victim by Elizer. To the extent
that inconsistencies were in fact
shown, they appear to us "to relate
to details of peripheral significance
which do not negate or dissolve

CRIMINAL PROCEDURE
Case Digests Cassy C. Barras, 2D

the positive identification [by said


eyewitnesses of Elizer] as the
perpetrator of the crime."
Further,
the
failure
of
Bughao to immediately report the
incident to the police authorities
and to extend help to the victim
cannot destroy his credibility as a
witness. There is no standard of
behavior when a person becomes a
witness to a shocking or gruesome
event. "The workings of a human
mind
placed
under
severe
emotional stress are unpredictable
and people react differently. The
determining factor to consider is
that Bughao testified in candid and
straightforward
manner
and
implicated Elizer and Ric as the
perpetrators of the crime.
Aside from the eyewitness
testimonies of the prosecution
witnesses, the dying declaration of
the victim also established the
guilt of the appellants beyond
reasonable doubt. He was well
aware of his imminent death and
his declaration that Elizer was
responsible for his stab wound was
made in the belief that he would
not
survive
his
injury.
The
declarations by the victim certainly
relate to circumstances pertaining
to his impending death and he
would have been competent to
testify had he survived in view of
the general presumption that a
witness is competent to testify.

The victim also executed a Sworn


Statement32 on May 7, 2002,
while in serious condition in the
hospital,
declaring
that
the
appellants assaulted him and it
was Elizer who delivered his fatal
stab wound. His dying declaration
and
sworn
statement,
taken
together with the findings and
conclusions of the trial court,
establish the guilt of the appellants
beyond reasonable doubt.
People v. Dalisay
G.R. No. 188106
November 25, 2009
Topic: Credibility of witness
Doctrine: Appellate courts, as a
rule, will not disturb the findings
of the trial court on the credibility
of witnesses.
Facts: The victim at the time of the
incident, was a 16-year-old lass.
With her siblings, they stayed with
her
mothers
live-in
partner,
appellant Dalisay Their mother
worked as a baby-sitter and helper
and only came home at the end of
every month.
One
evening,
appellant
entered the room where the girl
was alone to change his clothes.
He touched her breasts and thighs.
Then, he forced the girl down on
the floor, took off her shorts and
underwear, and placed himself on
top of her. He warned her that he
would kill her entire family. Prior
to this assault, appellant had

CRIMINAL PROCEDURE
Case Digests Cassy C. Barras, 2D

already been repeatedly molesting


the girl since she was 13 years old
by inserting his finger into her
genitalia. Her sister told their aunt
of the sexual abuse of the
appellant as she witnessed an
incident. Medical reports revealed
that the victim experienced sexual
abuse.
An Information for rape in
relation to Republic Act (R.A.) No.
7610 was filed against the
appellant. After trial on the merits,
the RTC found appellant guilty of
qualified rape. On appeal, the CA
convicted the accused not of
qualified rape but of simple rape
Issue: Whether or not the findings
of the appellate court prove the
guilt of the appellant
Ruling: The Supreme Court ruled
in the affirmative.
In a determination of guilt
for the crime of rape, primordial is
the credibility of the complainants
testimony, because, in rape cases,
the accused may be convicted
solely on the testimony of the
victim, provided it is credible,
natural, convincing and consistent
with human nature and the normal
course of things. In this case, the
testimony of the victim was found
by the trial court to have been
made in a simple, straightforward
and spontaneous manner.
This eloquent testimony of
the victim, coupled with the

medical findings attesting to her


non-virgin state, should be enough
to confirm the truth of her
charges.
Further,
deeply
entrenched in our jurisprudence is
the rule that the findings of the
trial court on the credibility of
witnesses are entitled to the
highest respect and are not to be
disturbed on appeal in the absence
of any clear showing that the trial
court overlooked, misunderstood
or
misapplied
facts
or
circumstances of weight and
substance which would have
affected the result of the case.
The
Court
discredits
appellants defense of denial for it
is a negative and self-serving
evidence,
which
pales
in
comparison to the victims clear
and convincing narration and
positive
identification
of
her
assailant.
The Court, therefore, finds
appellant guilty beyond reasonable
doubt of the crime of simple rape.
While it has been proven that
appellant was the common-law
spouse of the parent of the victim
and the child was a minor at the
time of the incident, the Court
cannot
convict
appellant
of
qualified rape because the special
qualifying
circumstances
of
minority and relationship were not
sufficiently
alleged
in
the
information.
People v. Palgan

CRIMINAL PROCEDURE
Case Digests Cassy C. Barras, 2D

G.R. No. 186234


December 21, 2009
Topic: Credibility of witness
Doctrine: Appellate courts, as a
rule, will not disturb the findings
of the trial court on the credibility
of witnesses.
Facts: AAA, who was at the time 14
years old, is the stepdaughter of
the appellant. Appellant got angry
of AAA because he was spreading
rumors that he was not her father.
When AAA denied appellants
allegations, he removed her dress
and then undressed himself, held
his penis and forcibly inserted it
into her vagina. AAA was again
raped by the appellant when
everyone was sleeping. AAA kept
quiet about the incident until her
mother, Angelina, caused AAA to
reveal that the latter was raped by
appellant.
The
internal
examination of AAA revealed that
laxity of the vaginal wall could be
due to repeated manipulation or
entry of a foreign body.
Two informations were filed
against the appellant. Appellant
denied the allegations against him.
After trial on the merits, the RTC
convicted appellant of two counts
of rape. On review, the appellate
court affirmed trial courts ruling.
Issue: Whether or not the findings
of the appellate court prove the
guilt of the appellant

Ruling: The Supreme Court ruled


in the affirmative.
The determination of guilt of
appellant depends primarily on the
credibility of a victim. Her
testimony alone, if credible, would
render
appellants
conviction
inevitable. In the case at bar, the
trial court gave full weight and
credence to AAAs testimony that
appellant raped her on two
occasions. AAA testified in a clear,
spontaneous and candid manner;
she positively identified appellant
as the person who raped her; and
she stated that she was unable to
resist appellant because he was
angry and strong.
The rule is settled that the
trial courts findings on the
credibility of witnesses and of their
testimonies are entitled to the
highest respect and will not be
disturbed on appeal, in the
absence of any clear showing that
the
court
overlooked,
misunderstood or misapplied some
facts or circumstances of weight
and substance which would have
affected the result of the case. This
is because the trial court, having
seen and heard the witnesses
themselves and observed their
behavior and manner of testifying,
is in a better position to decide the
question of credibility.
People v. Adeser
G.R. No. 179931
October 26, 2009

CRIMINAL PROCEDURE
Case Digests Cassy C. Barras, 2D

Topic: Credibility of witness


Doctrine: Appellate courts, as a
rule, will not disturb the findings
of the trial court on the credibility
of witnesses.
Facts:
Private
complainant
Josephine R. Palo and her sister
Teresa Caraig testified that the
spouses
Roberto
and
Mel
Tiongson, agents of Naples Travel
and Tours, introduced Palo to
appellant, owner and general
manager of Naples, to discuss
employment
opportunities
in
Australia.
Appellant
and
the
spouses Tiongson informed Palo
that for a placement fee of
P80,000, she can work as an apple
picker in Australia with a monthly
salary of $1,400. Thus, they gave
Roberto Tiongson and Lourdes
Chang P15,000 as first installment
for the placement fee. Palo and
Caraig returned to the Naples
office and paid P58,500. More than
three months passed, Palo was not
deployed to Australia. Neither did
she get her Australian visa.
Thereafter,
she
learned
that
Naples had closed down and had
no license to operate and deploy
workers abroad.
Appellant on the other hand
denied the charges against her.
She admitted that she was the
owner and general manager of
Naples which was a travel agency
that
offered
visa
assistance,
ticketing, documentation, airport

transfer and courier services, but


denied
having
engaged
in
recruitment.
The trial court convicted
appellant guilty of the crime of
Syndicated Illegal Recruitment
constituting Economic Sabotage
and estafa. The appellate court did
not give credence to appellants
denials and found that the
prosecution
evidence
fully
supports the finding that appellant
and her co-accused engaged in
recruitment and placement as
defined under the Labor Code
despite having no authority to do
so.
Issue: Whether or not appellants
guilt is proved beyond reasonable
doubt
Ruling: The Supreme Court ruled
in the negative.
As against the positive and
categorical testimonies of Palo and
Caraig, appellants denials cannot
prevail. Moreover, there is no
reason to overturn the trial and
appellate courts findings on the
credibility of the prosecution
witnesses as there is no showing
that any of them had ill motives
against appellant or her coaccused and especially since it
appears they were motivated
solely by the desire to bring
appellant and her co-accused to
justice for the crimes they have
committed.

CRIMINAL PROCEDURE
Case Digests Cassy C. Barras, 2D

Neither can the Supreme


Court
sustain
appellants
contention that her participation in
the recruitment is negated by the
fact that her signature does not
even appear on the vouchers
issued to Palo. Even if Palo did not
present
receipts
signed
by
appellant, this would not rule out
the fact that appellant did receive
the money. The Supreme Court has
consistently ruled that absence of
receipts as to the amounts
delivered to a recruiter does not
mean that the recruiter did not
accept or receive such payments.
Neither in the Statute of Frauds
nor in the rules of evidence is the
presentation of receipts required
in order to prove the existence of a
recruitment agreement and the
procurement of fees in illegal
recruitment cases. Such proof may
come from the credible testimonies
of witnesses as in the case at bar.
People v. Aburque
G.R. No. 181085
October 23, 2009
Topic: Credibility of witness
Doctrine: Appellate courts, as a
rule, will not disturb the findings
of the trial court on the credibility
of witnesses.
Facts: Appellant was charged of
murder.
At
the
arraignment,
appellant admitted killing the
victim, but he interposed the

justifying circumstance of selfdefense.


There was a party where the
appellant and the victim and other
guests
were
having
drinks.
Appellant noticed that the victim
took out a knife or a bolo without a
handle and a scabbard. He then
started
swinging
it
around.
Appellant advised the victim to
keep the weapon away to which
the latter acceded. Then the victim
took out two arnis sticks and
again, in front of his guests,
started
swinging
it
around.
Appellant had to advise the victim
again to sit down, stop playing
with the arnis and keep them
away. Thereafter, while he was
eating,
appellant
felt
that
something hit his forehead which
made him dizzy and caused him to
fall. As it was painful, he reached
out for his bolo, held it close to his
chest and slung it over his
shoulder.
According
to
the
appellant, while he was slowly
opening the door, the victim kicked
him and pinned him to the door
with his feet. He then heard the
victim say, Why dont I just finish
you of or, what if I finish you of?
Fearing that he would be hacked,
appellant took out his bolo and
stabbed Ablay in the stomach. The
testimony of the accused was
further
corroborated
by
the
testimonies
given
by
Canuto
Sarne, who testified that appellant

CRIMINAL PROCEDURE
Case Digests Cassy C. Barras, 2D

sent him to call the policemen for


his surrender
For the defense, Banaybanay
testified that after eating and
drinking tuba, appellant pulled out
his pinuti (bolo) and stabbed the
victim hitting him in the stomach
and it pierced through his back.
The victims wife also testified as
to the suddenness of the stabbing
of her husband. Medical reports
say that the victim died due loss of
blood, brought about by a fivecentimeter incise wound in his
epigastric area.
After trial, the trial court
found the appellant guilty of
murder as charged. The CA
affirmed in toto the RTC decision.
Issue: Whether or not the lower
court erred in rejecting appellants
plea of self-defense while giving
full faith and credence to the
prosecutions evidence
Ruling: The Supreme Court ruled
in the negative.
Taking
into
serious
consideration the circumstances in
this case as proved by testimonies
of witnesses for the prosecution as
well as the arguments of the
parties,
the
Supreme
Court
sustains appellants conviction.
Against
the
positive
declarations of the prosecution
witnesses
who
testified
that
appellant stabbed Miguel Ablay
abruptly without any provocation

on the part of the latter, appellants


self-serving and uncorroborated
assertion
deserves
scant
consideration. It is a well-settled
rule that a plea of self-defense
cannot be justifiably entertained
where
it
is
not
only
uncorroborated by any separate
competent evidence but is also
extremely doubtful in itself. Absent
any showing that the prosecution
witnesses were moved by improper
motive to testify against the
appellant, their testimonies are
entitled to full faith and credit.
It is a well-settled rule that
where the credibility of witnesses
is in issue, the appellate courts will
generally not disturb the findings
of the trial court, which is in a
better position to determine the
issue, having the advantage of
hearing
and
witnessing
the
deportment of
the
witnesses
during trial. While this rule admits
of exceptions, the Supreme Court
finds no reason to apply any to the
instant appeal.
People v. Lascano
G.R. No. 172605
November 22, 2010
Topic: Credibility of witness
Doctrine: Appellate courts, as a
rule, will not disturb the findings
of the trial court on the credibility
of witnesses.
Facts: Appellant was arrested
during the buy-bust operation

CRIMINAL PROCEDURE
Case Digests Cassy C. Barras, 2D

organized by the Office of the Drug


Enforcement Unit of the Malabon
City Police after receiving a call
from a confidential informant. P01
was able to recover from her the
marked money and a yellow plastic
bag containing one plastic sachet
of marijuana and a brick of
marijuana. The two plastic sachets
subject of the illegal sale were
marked by PO1 Joel, while the
other plastic sachet and the brick
of marijuana were marked by PO1
Allan before they were given to
Police
Investigator
Vicente
Mandac. A request for laboratory
examination of the seized items
was made to the PNP Crime
Laboratory.
Appellant
denied
the
accusation
against
her.
She
testified that she was at home with
her husband when three persons
entered their house and brought
her to the police station. Defense
witness Celestino testified that he
was having coffee in the alley
when he saw men open appellant's
door by means of a screw driver,
after which four persons entered
the house with one left at the door.
He saw appellant being dragged
outside of her house to the main
road and was forced to board an
owner
type-jeep.
Sabenal
corroborated Celestino's testimony
and added that she followed
appellant to the police station
After
trial,
RTC
found
appellant
guilty
of
illegal

possession
of
prohibited
drug/marijuana and drug pushing.
Appellant filed a Motion for New
Trial before the Supreme Court
alleging
newly-discovered
evidence which consisted of the
Sinumpaang Salaysay of a certain
Nonie Villaester, who claimed to be
a police informer of the Narcotics
Unit of the Malabon Police Station.
The Supreme Court denied the
motion and referred it the case to
the CA.
The CA upheld the RTCs
findings according credence to the
testimonies of the police officers
who
conducted
the
buy-bust
operation.
The
CA
rejected
appellant's claim that no drug
pusher in her right mind would
bring a large amount of marijuana
when the transaction was only for
two sachets worth P200.00, saying
that drugs dealers are known to
sell their goods even to strangers
and even ply their wares wherever
prospective customers may be
found. The CA also brushed aside
appellant's defense of frame up as
she failed to present convincing
evidence
to
overcome
the
presumption that the arresting
officers regularly performed their
official duties.
Issue:
Whether
or
not
the
prosecution was able to prove
beyond reasonable doubt the
crimes charged against appellant

CRIMINAL PROCEDURE
Case Digests Cassy C. Barras, 2D

Ruling: The Supreme Court ruled


in the affirmative.
Well settled is the rule that
findings of trial courts, which are
factual in nature and which involve
the credibility of witnesses, are to
be respected when no glaring
errors, gross misapprehension of
facts and speculative, arbitrary
and unsupported conclusions can
be gleaned from such findings.
Such findings carry even more
weight if they are affirmed by the
Court of Appeals, as in the instant
case.
The testimony of poseurbuyer PO1 Joel was corroborated
by PO1 Allan, who acted as the
former's
back-up.
PO1
Allan
testified that he saw PO1 Joel and
the confidential informant enter an
alley going to appellants house and
saw them talking with appellant;
that, after a while, he saw
appellant receive something from
PO1 Joel and appellant in turn
gave something to the latter. After
such exchange, PO1 Joel then gave
the pre-arranged signal by holding
the back of his head, thus, he (PO1
Allan) immediately approached
them and was able to recover from
appellant the buy-bust money,
which was also presented in court.
The testimonies of these
prosecution witnesses had clearly
established
that
a
sale
of
marijuana took place between
appellant and poseur-buyer PO1
Joel. The delivery of the illicit drug

to the poseur-buyer and the


receipt by the seller of the marked
money successfully consummated
the buy-bust transaction.
People v. Rusiana
G.R. No. 186139
October 5, 2009
Topic: Credibility of witness
Doctrine: Absent any showing that
certain facts of relevance and
substance bearing on the elements
of the crime have been overlooked,
misapprehended, or misapplied by
a trial court, its findings must be
respected.
Facts: P02 Mendoza testified that
someone arrived in their office and
informed PO2 Paule of a certain
Unads illegal drug activities. PO2
Paule reported the information to
Police
Inspector
Raquion.
Thereafter, they formed a buy-bust
team. The team proceeded to the
Manukan in Las Pias where the
transaction will happen. After the
exchange of marked money and
the suspected shabu, Unad tried to
flee but was caught by P02 Paule.
According
to
accusedappellant, he was home with his
children
when
the
buy-bust
operation took place. Camposano,
accused-appellants mother-in-law,
testified that she was likewise
home on the night of the alleged
buy-bust
operation. Badoy,
Camposanos
15-year
old
grandchild, and Ramirez, accused-

CRIMINAL PROCEDURE
Case Digests Cassy C. Barras, 2D

appellants stepson, corroborated


Camposanos testimony.
The RTC found appellantaccused guilty of Violation of Sec.
5, Art. II. of R.A. 9165. The CA
agreed with the RTC that the
elements in the crime of illegal
sale of drugs were adequately
proved. It gave no merit to
accused-appellants argument that
the chain of custody over the
evidence was broken. It likewise
found the defense of frame-up
lacking in merit, as accusedappellant was not able to show
convincing evidence that the police
officers involved in the buy-bust
did not perform their duties in a
regular and proper manner, or that
they were harboring ill motives
against him.
Issue: Whether or not the court a
quo gravely erred in giving
credence to the evidence of the
prosecution
Ruling: The Supreme Court ruled
in the negative.
Jurisprudence dictates that
conviction can be had in a
prosecution for illegal sale of
regulated or prohibited drugs if
the
following
elements
are
present: (1) the identity of the
buyer and the seller, the object,
and the consideration; and (2) the
delivery of the thing sold and the
payment for it. What is material is
the proof that the transaction or
sale actually took place, coupled

with the presentation in court of


the corpus delicti of the crime. The
Supreme Court holds that these
elements have been satisfied by
the prosecutions evidence.
Trial courts are the Supreme
Courts eyes they have the
distinct advantage of observing the
demeanor
and
conduct
of
witnesses during trial. Absent any
showing that certain facts of
relevance and substance bearing
on the elements of the crime have
been overlooked, misapprehended,
or misapplied by a trial court, we
must defer to its findings. As found
by the trial court and affirmed by
the CA, the police officers who
testified gave a straightforward
narration
of
the
buy-bust
operation. The Supreme Court
sees
no
circumstance
contradicting this finding.
Lejano v. Court of Appeals
G.R. No. 176389
December 14, 2010
Topic: Credibility of witness
Doctrine:
Facts: Estrellita Vizconde and her
daughters Carmela, nineteen years
old, and Jennifer, seven, were
brutally slain at their home. Four
years later in 1995, the National
Bureau of Investigation or NBI
announced that it had solved the
crime. It presented star-witness
Jessica M. Alfaro who claimed that
she witnessed the crime. She

CRIMINAL PROCEDURE
Case Digests Cassy C. Barras, 2D

pointed to accused Webb, Lejano,


Ventura,. Gatchalian, Fernandez,
Estrada, Rodriguez, and Filart as
the culprits.
Webbs alibi appeared the
strongest since he claimed that he
was then across the ocean in the
United States of America. He
presented the testimonies of
witnesses as well as documentary
and object evidence to prove this.
In addition, the defense presented
witnesses to show Alfaro's bad
reputation for truth and the
incredible nature of her testimony.
But impressed by Alfaros
detailed narration of the crime and
the events surrounding it, the trial
court found a credible witness in
her. It noted her categorical,
straightforward, spontaneous, and
frank testimony, undamaged by
grueling cross-examinations. The
trial court remained unfazed by
significant discrepancies between
Alfaros April 28 and May 22, 1995
affidavits,
accepting
her
explanation that she at first
wanted to protect her former
boyfriend, accused Estrada, and a
relative, accused Gatchalian; that
no lawyer assisted her; that she
did not trust the investigators who
helped her prepare her first
affidavit; and that she felt unsure if
she would get the support and
security she needed once she
disclosed all about the Vizconde
killings.

Issue:
Whether
or
not
prosecutions main witness is
credible to convict an accused
Ruling: The Supreme Court ruled
in the negative.
Records disclose that the
trial court was impressed by the
witness
supposed
detailed
narration of the crime and the
events surrounding it, having
allegedly
testified
in
a
categorical,
straightforward,
spontaneous, and frank testimony.
The
High
Court
however,
expressed surprise why the trial
court
remained
unfazed
by
significant discrepancies between
two affidavits of witness. In sum,
the Court found the testimony of
the prosecutions main witness
devoid of the credibility necessary
to convict an accused not because
of having observed the conduct of
the witness while being examined
in open court but because of
discrepancies in the testimony.
Thus, the Supreme Court
acquits the accused of the crimes
of which they were charged for
failure of the prosecution to prove
their guilt beyond reasonable
doubt.
They
are
ordered
immediately
released
from
detention unless they are confined
for another lawful cause.
People v. Abon
G.R. No. 169245
February 15, 2008

CRIMINAL PROCEDURE
Case Digests Cassy C. Barras, 2D

Topic: How to appeal


Doctrine: Under Sec. 2 of RA 9346,
the imposition of the death penalty
is prohibited, and in lieu thereof, it
imposes the penalty of reclusion
perpetua.
Facts: While sleeping, accusedappellant moved her daughter,
AAA, who was then 13 years old,
away from her brother BBB.
Thereafter, he removed AAAs
pajama, then his shorts and brief,
and went on top of AAA. Accusedappellant succeeded in inserting
his penis inside AAAs vagina, and
then made a push and pull
movement of his penis inside her
vagina for about 20 to 30 minutes.
Thereafter, he left.
An Information for qualified
rape was filed against the accusedappellant.
When
arraigned,
accused-appellant
pleaded
not
guilty to the crime charged.
During the trial, he interposed
denial and alibi as his defenses.
The RTC found accusedappellant guilty beyond reasonable
doubt of the crime charged and to
suffer death penalty as provided
for by Section 11 of Republic Act
No. 7659. Due to the penalty
imposed, the case was forwarded
to
the
Supreme
Court
for
automatic review. However, in
accordance with the ruling in
People v. Mateo, the Supreme
Court transferred the case to the
CA for intermediate review. The

CA affirmed the trial courts


judgment of conviction, but it
modified the award of damages.
Issue: Whether or not death
penalty should be imposed as
penalty
Ruling:
The
Supreme
Court
reduced the penalty to reclusion
perpetua without eligibility for
parole pursuant to RA 9346.
Section 3 of Rule 122 of the
2000 Rules on Criminal Procedure
provides that where the penalty
imposed by the RTC is reclusion
perpetua or life imprisonment, an
appeal is made directly to the
Supreme Court by filing a notice of
appeal with the court which
rendered the judgment or final
order appealed from and by
serving a copy thereof upon the
adverse party. On the other hand,
a case where the penalty imposed
is death will be automatically
reviewed by the Court without a
need for filing a notice of appeal.
However,
Mateo
ruling
modified these rules by providing
an intermediate review of the
cases by the CA where the penalty
imposed is reclusion perpetua, life
imprisonment, or death.
Also affecting the rules on
appeal is the enactment of
Republic Act No. 9346 or An Act
Prohibiting the Imposition of the
Death Penalty in the Philippines,
which took effect on June 29, 2006.

CRIMINAL PROCEDURE
Case Digests Cassy C. Barras, 2D

Under Sec. 2 of RA 9346, the


imposition of the death penalty is
prohibited, and in lieu thereof, it
imposes the penalty of reclusion
perpetua, when the law violated
makes use of the nomenclature of
the penalties of the Revised Penal
Code (RPC); or life imprisonment,
when the law violated does not
make use of the nomenclature of
the
penalties
of
the
RPC.
Consequently, in the provisions of
the Rules of Court on appeals,
death penalty cases are no longer
operational.
Batistis v. People
G.R. No. 181571
December 16, 2009
Topic: Appeal
Doctrine: The review on appeal of
a decision in a criminal case,
wherein the CA imposes a penalty
other
than
death,
reclusion
perpetua, or life imprisonment, is
by petition for review on certiorari.
Facts: The Fundador trademark
was
duly
registered
in
the
Principal
Register
of
the
Philippines Patent Office. Allied
Domecq
Philippines,
Inc.,
a
Philippine corporation exclusively
authorized to distribute Fundador
brandy products initiated this case
against Batistis.
Upon its request, agents of
the NBI conducted a test-buy in
the premises of Batistis, and
thereby confirmed that he was

actively
engaged
in
the
manufacture, sale and distribution
of counterfeit Fundador brandy
products. Secured with search
warrant,
NBI
agents
seized
Batistis premises. The Office of
the
City
Prosecutor
formally
charged Batistis in the RTC with
two separate offenses, namely,
infringement of trademark and
unfair competition. After trial, the
RTC convicted Batistis guilty
beyond reasonable doubt of the
crimes charged. Batistis appealed
to the CA, which affirmed his
conviction for infringement of
trademark, but acquitted him of
unfair competition. The CA denied
Batistis
motion
for
reconsideration.
Thus,
Batistis
filed a petition for review before
the Supreme Court.
Issue: Whether or not petition for
review is the correct remedy of
Batistis
Ruling: The Supreme Court ruled
in the negative. Pursuant to
Section 3, Rule 122, and Section 9,
Rule 45, of the Rules of Court, the
review on appeal of a decision in a
criminal case, wherein the CA
imposes a penalty other than
death, reclusion perpetua, or life
imprisonment, is by petition for
review on certiorari. A petition for
review on certiorari raises only
questions of law.

CRIMINAL PROCEDURE
Case Digests Cassy C. Barras, 2D

Accordingly, the Supreme


Court rejected the appeal for the
following reasons: (1) the petition
for review replicates Batistis
appellant's brief filed in the CA, a
true indication that the errors he
submits for our review and
reversal
are
those
he
had
attributed to the RTC; (2) Batistis
assigned errors stated in the
petition for review on certiorari
require a re-appreciation and reexamination of the trial evidence.
As
such,
they
raise
issues
evidentiary and factual in nature;
and (3) the factual findings of the
RTC,
its
calibration
of
the
testimonies of the witnesses, and
its assessment of their probative
weight are given high respect, if
not
conclusive
effect,
unless
cogent facts and circumstances of
substance, which if considered,
would alter the outcome of the
case, were ignored, misconstrued
or misinterpreted.
Sable v. People
G.R. No. 177961
April 7, 2009
Topic: Appeal
Doctrine: Except as provided in
the last paragraph of Section 13,
Rule 124, all other appeals to the
Supreme Court shall be by petition
for review on certiorari under Rule
45.
Facts: Petitioner and co-accused
Ildefonsa are the grand-daughters

of Eleuteria Abangan, who is one


of the registered owners of a
property.
Ildefonsa,
allegedly
falsified
an
Extrajudicial
Declaration of Heirs with Waiver of
Rights and Partition Agreement, as
the signatures contained therein
were not the signatures of the true
owners of the land. Petitioner and
Ildefonsa also allegedly caused it
to appear that a certain Remedios
Abangan, who was already dead,
signed the document.
By virtue of the Extrajudicial
Declaration of Heirs, the property
was subdivided into two lots and
OCT No. RO-2740 was cancelled.
The RTC convicted petitioner
of the crime of Falsification of
Public Documents under Article
172(1) in relation to Article 171 of
the Revised Penal Code, but
acquitted Ildefonsa. Petitioners
motion for reconsideration was
denied.
Due to petitioners failure to
interpose a timely appeal, an entry
of judgment was issued. Petitioner
filed Motions to Recall Warrant of
Arrest and to Vacate Entry of
Judgment with Reconsideration
and Explanation alleging that
petitioners counsel did not receive
the Order. Pending resolution of
the Motions to Recall Warrant of
Arrest and to Vacate Entry of
Judgment with Reconsideration,
petitioner filed a Notice of Appeal.
Respondent Judge denied the
Motions to Recall Warrant of

CRIMINAL PROCEDURE
Case Digests Cassy C. Barras, 2D

Arrest and to Vacate Entry of


Judgment. Petitioners Notice of
Appeal was also denied for having
been filed out of time. Petitioners
motion for reconsideration, motion
to recall warrant of arrest and
motion to allow accused to avail of
the benefits of the Probation Law,
all are likewise denied.
Petitioner filed a Petition for
Certiorari under Rule 65 before
the Court of Appeals raising the
sole issue of whether or not the
respondent court acted with grave
abuse of discretion in denying the
application for probation. The CA
denied the petition for lack of
merit, stating that the alleged
failure of petitioners counsel to
timely appeal the judgment of
conviction following the denial of
the reconsideration thereof could
not
amount
to
excusable
negligence. The CA refused to
reconsider its earlier Decision.
Hence, a Petition for Certiorari
under Rule 65 of the Rules of
Court was
filed
before
the
Supreme Court.
Issue: Whether or not petition for
certiorari is the correct remedy
Ruling: The Supreme Court ruled
in the negative.
Under Rule 122, Section 3(e)
of the Rules of Court, except as
provided in the last paragraph of
Section 13, Rule 124, all other
appeals to the Supreme Court shall

be by petition for review on


certiorari under Rule 45.
In the case at bar, petitioner
elevated the petition via a Petition
for Certiorari under Rule 65.
Under the Rules, subject to the
exceptions, appeal to the Supreme
Court must be via a petition for
Review under Rule 45. Since, this
appeal
is
not
within
the
exceptions, the proper mode of
appeal should be a Petition for
Review under Rule 45, not under
Rule 65. One of the requisites of
certiorari is that there be no
available appeal or any plain,
speedy and adequate remedy.
Where an appeal is available,
certiorari will not prosper even if
the ground therefor is grave abuse
of discretion. Accordingly, when a
party adopts an improper remedy,
as in this case, his petition may be
dismissed outright.
People v. Abesamis
G.R. No. 140985
August 28, 2007
Topic: Appeal not mooted by
accuseds release on parole
Doctrine: The appeal was not
mooted by accused-appellants
release on parole. Parole is not one
of
the
modes
of
totally
extinguishing
criminal
liability
under Article 89 of the Revised
Penal Code.
Facts: An information for murder
was filed against the brothers

CRIMINAL PROCEDURE
Case Digests Cassy C. Barras, 2D

accused-appellant and Rodel in the


RTC. Accused-appellant admitted
stabbing Ramon with a butchers
knife but claimed that he did so
only to defend himself.
The trial court ruled that,
while it was established that
accused-appellant killed Ramon,
the prosecution failed to prove the
existence
of
either
evident
premeditation or treachery. Thus,
the trial court found accusedappellant guilty of homicide. On
appeal, the CA found accusedappellant
guilty
of
murder
qualified by treachery and is
meted the penalty of reclusion
perpetua. However, considering
the penalty imposed on the
Appellant, the CA certifies the case
to
the
Supreme
Court
for
appropriate review. Julio Arciaga,
assistant director for prisons and
security
of
the
Bureau
of
Corrections, informed the Court
that
accused-appellant
was
granted parole by the Board of
Pardons and Parole and released
from the custody of the Bureau of
Corrections.
Issue: Whether or not the grant of
parole rendered this case moot
Ruling: The Supreme Court ruled
in the negative.
The appeal was not mooted
by accused-appellants release on
parole. His release only meant
that, according to the Board, he

had already served the minimum


penalty imposed on him and that
he was fitted by his training for
release, that there was reasonable
probability that he would live and
remain at liberty without violating
the law and that such release
would not be incompatible with the
welfare of society. Should he
violate the conditions of his parole,
accused-appellant may be ordered
rearrested, to serve the remaining
unexpired portion of the maximum
sentence.
Parole
refers
to
the
conditional release of an offender
from a correctional institution
after he serves the minimum term
of his prison sentence. The grant
thereof does not extinguish the
criminal liability of the offender.
Parole is not one of the modes of
totally
extinguishing
criminal
liability under Article 89 of the
Revised Penal Code. Inclusio unius
est exclusio alterius.
Moreover,
the grant of
parole would be improvident if the
CA decision finding accusedappellant guilty of murder and
sentencing him to suffer the
penalty of reclusion perpetua were
to be affirmed by the Supreme
Court. In such a case, the
determination of the Board that
accused-appellant
would
have
already served the minimum
penalty imposed on him would turn
out to be erroneous. Worse, in
basing
its
determination
of

CRIMINAL PROCEDURE
Case Digests Cassy C. Barras, 2D

accused-appellants eligibility for


parole on the penalty imposed in
the RTC decision, the Board
effectively ignored the decision of
the CA.
Furthermore,
the
Board
violated its own rules disqualifying
from parole those convicted of an
offense punished with reclusion
perpetua. Thus, the Board should
be warned in no uncertain terms
for acting ultra vires, carelessly
disregarding the CA decision and
improvidently granting parole to
accused-appellant.
Garces v. People
G.R. No. 173858
July 17, 2007
Topic: Effects on appeal by any of
several accused
Doctrine: An appeal taken by one
or more of several accused shall
not affect those who did not
appeal, except insofar as the
judgment of the appellate court is
favorable and applicable to the
latter.
Facts: The trial court found
Pacursa
guilty
of
Forcible
Abduction
with
Rape
while
petitioner Garces was found guilty
as an accessory to the crime.
Antonio Pira, Jr. and Aurelio Pira
were acquitted for insufficiency of
evidence.
Both
Pacursa
and
petitioner appealed the decision
with the Court of Appeals.
However, Pacursa subsequently

withdrew his appeal. The CA


affirmed the decision of the trial
court.
Petitioner filed a motion for
reconsideration but same was
denied. Hence, the instant petition
for review on certiorari.
Issue: Whether or not the ruling of
the Supreme Court will also apply
to Pacursa
Ruling: The Supreme Court ruled
in the affirmative.
The Supreme Court ruled
that the crime committed is simple
rape and not forcible abduction
with rape.
Although
Pacursa
has
withdrawn his appeal, the Courts
ruling that the crime committed is
simple rape and not forcible
abduction with rape, shall apply to
him. Section 11 (a), Rule 122 of
the Rules of Court specifically
provides that an appeal taken by
one or more of several accused
shall not affect those who did not
appeal, except insofar as the
judgment of the appellate court is
favorable and applicable to the
latter.
Olalia, Jr. v. People
G.R. No. 177276
August 20, 2008
Topic: Effects on appeal by any of
several accused

CRIMINAL PROCEDURE
Case Digests Cassy C. Barras, 2D

Doctrine: An appeal taken by one


or more of several accused shall
not affect those who did not
appeal, except insofar as the
judgment of the appellate court is
favorable and applicable to the
latter.
Facts: The RTC found accused,
Jeffrey
Calpao
Poquiz,
Pedro
Pidlaoan Poquiz and Graciano
Santos Olalia, Jr., guilty beyond
reasonable doubt of the crime of
Frustrated Murder penalized by
Article 248 in relation to Articles 6
and 50 of the Revised Penal Code,
as amended by R.A. No. 7659.
Graciano, Pedro and Jeffrey
filed a notice of appeal. Despite
their notice of appeal, Pedro and
Graciano, nonetheless, filed a
Motion for Reconsideration. Since
all the accused already perfected
their appeal, and since the RTC
lost jurisdiction over the case by
reason of the appeal, it did not
resolve
the
motion
for
reconsideration. The CA affirmed
the RTC decision. Petitioner alone
filed a Motion for Reconsideration
which was denied by the CA in a
Resolution. Hence, the instant
petition
filed
by
petitioner
Graciano Santos Olalia, Jr.
Issue: Whether or not the ruling of
the Supreme Court will also apply
to Pedro and Jeffrey

Ruling: The Supreme Court ruled


in the affirmative.
Since the Supreme Court
downgraded the crime committed
by petitioner from frustrated
murder to attempted murder, and
considering that the same set of
facts were used to convict Pedro
Poquiz and Jeffrey Poquiz, the
Supreme Court holds, that the
favorable verdict on petitioners
appeal
should
likewise
be
extended to Pedro Poquiz and
Jeffrey Poquiz, since under Section
11(a), Rule 122 of the present
Rules on criminal procedure, an
appeal taken by one or more of
several accused shall not affect
those who did not appeal, except
insofar as the judgment of the
appellate court is favorable and
applicable to the latter.
Constantino v. Sandiganbayan
G.R. No. 140656
September 13, 2007
Topic: Effects on appeal by any of
several accused
Doctrine: An appeal taken by one
or more of several accused shall
not affect those who did not
appeal, except insofar as the
judgment of the appellate court is
favorable and applicable to the
latter.
Facts: Finding that the prosecution
had proven beyond reasonable
doubt the guilt of Constantino, in
his
capacity
as
mayor
of

CRIMINAL PROCEDURE
Case Digests Cassy C. Barras, 2D

Malungon, Sarangani Province and


Lindong of the violation of Section
3 (e) of R.A. No. 3019, the
Sandiganbayan
rendered
the
assailed decision.
Constantino and Lindong
filed separate appeals to the
Supreme Court which have taken
disparate routes. On 25 April 2006,
during the pendency of his present
appeal, Constantino passed away.
Lindong himself likewise filed a
petition for review on certiorari to
seek
a
reversal
of
the
Sandiganbayan
decision.
The
Supreme Court denied Lindongs
petition for failure to state the
material date of receipt of the
assailed
decision
of
the
Sandiganbayan. His subsequent
attempts
for
reconsideration
proved futile. The Court issued the
Entry of Judgment in the case.
Thereafter,
the
Sandiganbayan issued three orders
relative to the execution of
judgment against Lindong, all of
which are assailed by the latter, in
his petition for certiorari for
having been issued with grave
abuse of discretion.
With
the
demise
of
Constantino during the pendency
of his appeal, the same should
normally be regarded as moot and
academic following the norm that
the death of the accused marks the
extinction of his criminal liability.
However, the present two petitions
are so intertwined that the

absolution
of
Constantino
is
ultimately determinative of the
absolution of Lindong. Indeed, the
exoneration of Constantino will
necessarily signify the injustice of
carrying out the penalty imposed
on Lindong. Thus, the Supreme
Court in this instance has to
ascertain
the
merits
of
Constantinos appeal to prevent a
developing miscarriage of justice
against Lindong.
Issue: Whether or not the acquittal
of Constantino will also result to
the acquittal of Lindong
Ruling: The Supreme Court ruled
in the affirmative.
It is therefore apparent that
in light of the finding of the
Supreme Court that Constantino
should be acquitted of the crime, it
cannot sustain the execution of
judgment against Lindong. The
reversal of the decision of the
Sandiganbayan makes it legally
absurd to execute any such
judgment against him pursuant to
Rule 122, Section 11(a) of the
Revised
Rules
of
Criminal
Procedure an appeal taken by
one or more of several accused
shall not affect those who did not
appeal, except insofar as the
judgment of the appellate court is
favorable and applicable to the
latter.
Ching v. Nicdao

CRIMINAL PROCEDURE
Case Digests Cassy C. Barras, 2D

G.R. No. 141181


April 27, 2007
Topic: Appeal from a civil aspect
Doctrine: The period to appeal
granted to petitioner is the same
as that granted to the accused.
Facts: Nicdao was charged eleven
counts of violation of Batas
Pambansa Bilang 22. MTC found
her of guilty of said offenses. RTC
affirmed MTCs decision.
Nicdao filed an appeal before
the Court of Appeals. CA reversed
the
decision
and
acquitted
accused. Ching is now appealing
the civil aspect of the case to the
Supreme Court. Ching vigorously
argues
that
notwithstanding
respondent Nicdaos acquittal by
the CA, the Supreme Court has the
jurisdiction
and
authority
to
resolve and rule on her civil
liability. He anchors his contention
on Rule 111, Sec 1B The criminal
action for violation of BP Blg. 22
shall be deemed to necessarily
include the corresponding civil
action, and no reservation to file
such civil action separately shall
be
allowed
or
recognized.
Moreover, under the above-quoted
provision, the criminal action for
violation of BP 22 necessarily
includes the corresponding civil
action, which is the recovery of the
amount of the dishonored check
representing the civil obligation of
the drawer to the payee.

Issue: Whether or not Ching may


appeal the civil aspect of the case
within the reglementary period
Ruling: The Supreme Court ruled
in the affirmative.
Following
the
long
recognized rule that the appeal
period accorded to the accused
should also be available to the
offended party who seeks redress
of the civil aspect of the decision,
the period to appeal granted to
petitioner Ching is the same as
that granted to the accused. With
petitioner Chings timely filing of
the instant petition for review of
the civil aspect of the CAs
decision, the Supreme Court thus
has the jurisdiction and authority
to determine the civil liability of
respondent
Nicdao
notwithstanding her acquittal.
In resolving the case, the
Supreme
Court
finds
that
respondent
Nicdaos
acquittal
likewise carried with it the
extinction of the action to enforce
her civil liability. There is simply
no basis to hold respondent Nicdao
civilly liable to petitioner Ching.
Sable v. People
G.R. No. 177961
April 7, 2009
Topic:
Period
to
apply
for
probation
Doctrine:
An
application
for
probation must be made within the

CRIMINAL PROCEDURE
Case Digests Cassy C. Barras, 2D

period for perfecting an appeal


which.
Lubrica v. People
G.R. Nos. 156147-54
February 26, 2007
Topic:
Period
to
apply
for
probation
Doctrine: The benefit of stay of
execution afforded to a co-accused
who timely files an appeal cannot
be extended to those who failed to
file the same.
Lubrica v. People
G.R. Nos. 156147-54
February 26, 2007
Topic:
Period
to
apply
for
probation
Doctrine: The benefit of the stay of
execution afforded to a co-accused
who timely files an appeal cannot
be extended to those who failed to
file the same. Thus, the period to
appeal continued to run against
petitioner
notwithstanding
the
petitions for review filed by his coaccused
Facts:
The
Sandiganbayan
rendered
judgment
finding
petitioner Cipriano L. Lubrica,
together with Rolando Balderama
and Rolando Nagal, guilty beyond
reasonable doubt of committing
seven counts of direct bribery as
defined and penalized under
Article 210 of the Revised Penal
Code. In addition, accused were
also found guilty of violating

Section 3(e) of Republic Act 3019,


otherwise known as the Anti-Graft
and Corrupt Practices Act, for
causing
undue
injury
to
complainant with evident bad faith
when they impounded one of the
latters taxi cabs on false charges.
The accused filed separate
motions asking the court to
reconsider
its
judgment.
Unconvinced, the Sandiganbayan
denied all the motions in a
resolution. Thereafter, all the
accused,
except
petitioner,
individually
elevated
their
convictions to the Supreme Court.
Consequently, the decision and
resolution of the Sandiganbayan
became final and executory with
respect to petitioner upon the
lapse of the appeal period. Five
months after the denial of his
motion
for
reconsideration,
petitioner filed a notice of appeal
and a motion to admit the notice of
appeal and suspend execution of
judgment with the Sandiganbayan.
The motion was denied. Petition
for review on certiorari under Rule
45 of the Revised Rules of Court.
Petitioner insists that, despite his
failure to file an appeal, his service
of sentence should be suspended
in view of the appeals interposed
by his co-accused.
Issue: Whether or not the benefit
of stay of execution afforded to
petitioners co-accused can be
extended to him

CRIMINAL PROCEDURE
Case Digests Cassy C. Barras, 2D

Ruling: The Supreme Court ruled


in the negative.
It is clear that the appeals
taken by co-accused Balderama
and Nagal cannot help petitioner
insofar as suspension of execution
is concerned. The benefit of stay of
execution afforded to a co-accused
who timely files an appeal cannot
be extended to those who failed to
file the same. Thus, the period to
appeal continued to run against
petitioner
notwithstanding
the
petitions for review filed by his coaccused.
Petitioner cannot invoke the
exception contained in the second
clause of Section 11(a) because it
speaks of a judgment rendered by
the appellate court which is
favorable to accused-appellant.12
It is too strained to construe the
exception as including procedural
consequences of a pending appeal
although it may be beneficial to
the accused.
Petitioners
petition
is
nothing but a vain attempt to avoid
the consequences of a conviction.
By petitioners fatal omission, the
judgment became final and the
sentence must be carried out
accordingly.
People v. Taruc
G.R. No. 185202
February 18, 2009
Topic: Dismissal of appeal by the
Court of Appeals

Doctrine: Although Rule 124,


Section 8 particularly applies to
the Court of Appeals, it has been
extended to the Supreme Court by
Rule 125, Section 1 of the Revised
Rules of Criminal Procedure.
Facts: Accused-appellant
Taruc
was charged in before the RTC
with the crime of murder. Upon
arraignment,
accused-appellant
pleaded not guilty. RTC found
accused guilty beyond reasonable
doubt as principal by direct
participation of the crime of
murder and sentenced to suffer
death penalty.
Accused-appellant elevated
the case to the CA for review.
Thereafter, accused-appellant filed
a Motion for Extension of Time to
File Appellants Brief. Considering
that the Notice to File Brief
addressed to accused-appellant
was returned to the appellate
court with postal notation moved
out, the CA directed accusedappellants counsel to furnish it
with the present and complete
address of his client within five
days from notice. PAO lawyer
informed
CA
that
accusedappellant escaped from prison. The
CA, notwithstanding the escape of
the accused-appellant, granted the
motion for extention of time. The
CA affirmed accused-appellants
conviction but modified the penalty
to reclusion perpetua. PAO lawyer

CRIMINAL PROCEDURE
Case Digests Cassy C. Barras, 2D

appealed before the Supreme


Court on questions of law and fact.
Issue: Whether or not Rule 124,
Section
8
which
particularly
applies to the Court of Appeals can
be extended to the Supreme Court
Ruling: The Supreme Court ruled
in the affirmative.
An accused is required to be
present before the trial court at
the promulgation of the judgment
in a criminal case. If the accused
fails to appear before the trial
court, promulgation of judgment
shall be made in accordance with
Rule 120, Section 6, paragraphs 4
and 5 of the Revised Rules of
Criminal Procedure. Consistently,
Rule 124, Section 8, paragraph 2
of the same Rules allows the Court
of Appeals, upon motion of the
appellee or motu proprio, to
dismiss the appeal of the accusedappellant
who
eludes
the
jurisdiction of the courts over his
person.

Although Rule 124, Section 8


particularly applies to the Court of
Appeals, it has been extended to
the Supreme Court by
Rule 125, Section 1 of the Revised
Rules of Criminal Procedure,
which reads:
SECTION 1. Uniform procedure. Unless otherwise provided by the
Constitution or by law, the
procedure in the Supreme Court in
original and in appealed cases
shall be the same as in the Court
of Appeals.
Thus, the Supreme Court
said that accused-appellant cannot
so audaciously hope that his
appeal before the Supreme Court
would succeed. He only hopes in
vain.

S-ar putea să vă placă și